You are on page 1of 734

www.TaiLieuLuyenThi.

com
ANG THANH NAM
(Trung tam Nghien cu va phat trien san pham giao duc Newstudy.vn)

SOAN THEO CAU TRUC MI AP DUNG K THI THPT QUOC GIA

(PHIEN BAN MI NHAT)

Danh cho hoc sinh 10, 11, 12 nang cao kien thc.
Boi dng hoc sinh gioi luyen thi Quoc Gia.

NHA XUAT BAN AI HOC QUOC GIA HA NOI

www.TaiLieuLuyenThi.com

MUC LUC
Chng 1: Bat ang thc va cac ky thuat c ban
Chu e 1. Ky thuat bien oi tng ng .............................................. 04
Chu e 2. Ky thuat minh phan chng .................................................... 45
Chu e 3. Ky thuat quy nap toan hoc ..................................................... 56
Chu e 4. Ky thuat mien gia tr ............................................................. 60
Chu e 5. Ky thuat s dung nguyen l Diricle ....................................... 68
Chu e 6. Ky thuat tam thc bac hai ..................................................... 73
Chu e 7. Ky thuat anh gia bat ang thc tch phan ......................... 93
Chng 2: Bat ang thc va phng phap tiep can
Chu e 1. Cac ky thuat s sung bat ang thc AM-GM c ban .......... 102
Chu e 2. Ky thuat ghep cap trong chng minh ang thc AM-GM ............. 198
Chu e 3. Ky thuat s dung bat ang thc AM-GM dang cong mau so ........211
Chu e 4. Ky thuat s dung bat ang thc Cauchy-Schwarz .............. 218
Chu e 5. Ky thuat s dung bat ang thc Cauchy-Schwarz dang
phan thc ................................................................................ 243
Chu e 6. Ky thuat tham so hoa ........................................................... 278
Chu e 7. Bat ang thc Holder va ng dung ...................................... 291
Chu e 8. Ky thuat s dung bat ang thc Chebyshev ....................... 304
Chu e 9. Bat ang thc Bernoulli va ng dung .................................. 314
Chng 3: Phng trnh ham so trong giai toan bat ang thc va
cc tr
Chu e 1. Ky thuat s dung tnh n ieu vi bai toan cc tr va bat
ang thc mot bien so ......................................................... 325
Chu e 2. Ky thuat s dung tnh n ieu cho bai toan cc tr va bat
ang thc hai bien so .......................................................... 351
Chu e 3. Ky thuat s dung tnh n ieu cho bai toan cc tr va bat
ang thc ba bien so............................................................ 379
Chu e 4. Ky thuat s dung tnh thuan nhat....................................... 427
Chu e 5. Ky thuat s dung bat ang thc tiep tuyen ........................ 484
Chu e 6. Ky thuat khao sat ham nhieu bien...................................... 502
Chu e 7. Ky thuat s dung tnh chat cua nh thc bac nhat va tam
thc bac hai .......................................................................... 534
Chu e 8. Bat ang thc phu ang chu y va ap dung giai e thi tuyen sinh .. 540
Chu e 9. Bai toan chon loc bat ang thc va cc tr ba bien ............ 617
Chng 4: So phng phap chng minh bat ang thc khac
Chu e 1. Ky thuat lng giac hoa ....................................................... 654
Chu e 2. Ky thuat s dung bat ang thc Schur ................................ 684
Chu e 3. Ky thuat don bien ................................................................. 694

www.TaiLieuLuyenThi.com
Cty TNHH MTV DVVH Khang Vit

Chng 1:
BT NG THC V CC K THUT C BN
KIN THC C BN V BT NG THC
I. nh ngha bt ng thc
Gi s A v B l hai biu thc bng ch hoc bng s.
+ A B (hoc B A) , A B (hoc B A) c gi l cc bt ng thc.
+ A B A B 0; A B 0 A B.
+ Mt bt ng thc c th ng hoc sai v ta quy c khi ni v mt bt ng
thc m khng ni g thm th ta hiu l bt ng thc ng.
II. Tnh cht c bn ca bt ng thc
a ; a a .
a b

ac.
b c

a, b, m ; a b a m b m .
a b

a+cb+d .
c d
a b+c acb.
ma mb khi m >0
a, b, ; a b
.
ma mb khi m < 0

a b
m m khi m >0
+
.
a, b, ; a b
a b khi m < 0
m
m

Nu a > b > 0

1 1
< .
a b

a c
a, b, c, d + ;
ab cd .
b d

a n b n
ab0
, n .
n a n b
a > 1 a x > a y
x > y > 0;
.
0 < a < 1 a x < a y

a > b a 2 n +1 > b 2 n +1; 2 n +1 a > 2 n +1 b , n .


3

www.TaiLieuLuyenThi.com
Khm ph t duy K thut gii bt T Bi ton Max Min ng Thnh Nam

1. Bt ng thc v gi tr tuyt i
a a a , a .
a < < a < ( khi > 0 ) .
a >
a >
( khi > 0 ) .
a <

a b a + b a + b , ( a, b ) .
2. Bt ng thc lin quan n hm s m v logarit
a > 1
0 < a < 1

ax > a y ;
ax < a y .
>
>
x
y
x
y

a > 1
0 < a < 1

log a x > log a y;
log a x < log a y .
x > y > 0
x > y > 0
3. Bt ng thc AM GM
Cho n s thc khng m a1 , a2 ,..., an ta c

a1 + a2 + ... + an n
a1a2 ...an .
n
ng thc xy ra khi v ch khi a1= a2= ...= an .

4. Bt ng thc Cauchy Schwarz


Cho 2 dy s thc ( a1 , a2 ,..., an ) ; ( b1 , b2 ,..., bn ) ta c

( a1b1 + a2b2 + ... + anbn )2 ( a12 + a22 + ... + an2 )( b12 + b22 + ... + bn2 ) .
ng thc xy ra khi v ch khi=
ai kb
=
i , i 1, n, k .
CH 1: K THUT BIN I TNG NG
Xut pht t bt ng thc c bn v ht sc t nhin x 2 0; A B 0 vi mi
s thc x ta c cc bt ng thc ht sc p mt. Ni dung ch ny cp n
k nng bin i bt ng thc v dng lun ng. Cc bi ton cp n l cc
bi ton trong ch ny cc bn ch s c s dng n trong cc ch khc
cc chng sau nh mt bi ton ph.
A. NI DUNG PHNG PHP
I. Cc bt ng thc c bn
Bnh phng ca mt s thc
Vi mi s thc x ta lun c x 2 0. ng thc xy ra khi v ch khi x = 0 .
T ta c cc bt ng thc vi 2 bin v 3 bin thng s dng nh sau:

( a b )2 0 hay

a 2 + b 2 2ab. ng thc xy ra khi v ch khi a = b .

www.TaiLieuLuyenThi.com
Cty TNHH MTV DVVH Khang Vit

( a b )2 + ( b c )2 + ( c a )2 0 hay

a 2 + b 2 + c 2 ab + bc + ca

hoc ( a + b + c ) 3 ( ab + bc + ca ) hoc 3 a 2 + b 2 + c 2 ( a + b + c ) .
2

ng thc xy ra khi v ch khi a= b= c .


Bt ng thc v tr tuyt i
Vi 2 s thc x,y ta lun c x + y x + y . ng thc xy ra khi v ch khi
xy 0 .

Vi 2 s thc x,y ta lun c x y x y . ng thc xy ra khi v ch khi


x( x y) 0 .

Bt ng thc v di cnh ca mt tam gic


a + b > c; b + c > a; c + a > b .
a > b c ;b > c a ;c > a b .
a 2 + b 2 + c 2 < 2 ( ab + bc + ca ) .
II. Mt s hng ng thc cn lu

( a + b + c ) ( a 2 + b2 + c 2 ab bc ca ) .

a3 + b3 + c3 3abc =

( a + b + c )3 = a3 + b3 + c3 + 3 ( a + b )( b + c )( c + a ) .
( a + b )( b + c )( c + a ) = ( a + b + c )( ab + bc + ca ) abc .
( a b )( b c )( c a )
a b bc ca
+
+
=

.
c
a
b
abc
a b bc ca ab bc ca

+
+
+
.
.
=
0.
a+b b+c a+c a+b b+c c+a
1) K thut bin dng nh ngha
chng minh bt ng thc: A B . Ta chng minh bt ng thc
A B 0 ng.

(x
V d 1. Cho x > y v xy = 1. Chng minh rng

+ y2

( x y )2

8.

Li gii
Ta c x 2 + y 2 =( x y ) + 2 xy =( x y ) + 2
2

(x

+ y2

(v xy = 1)

=( x y ) + 4. ( x y ) + 4 .
4

Do BT cn chng minh tng ng vi

( x y )4 + 4 ( x y )2 + 4 8.( x y )2
5

www.TaiLieuLuyenThi.com
Khm ph t duy K thut gii bt T Bi ton Max Min ng Thnh Nam

( x y )4 4 ( x y )2 + 4 0

( x y ) 2 2 0 .

Bt ng thc cui ng nn ta c iu phi chng minh.


V d 2.
a) Cho x,y l hai s thc tho mn iu kin xy 1 .

Chng minh rng


1
1+ x

1
1+ y

2
.
1 + xy

b) Cho a,b,c l cc s thc khng nh hn 1 chng minh


1
1
1
3
.
+
+

3
3
3
1 + a 1 + b 1 + c 1 + abc

c) Cho x, y, z [ 0;1] . Tm gi tr ln nht ca biu thc

1
1
1
P=
+
+
(1 + xyz )
3
3
3
.
1+ x 1+ y 1+ z
Li gii
a) Bt ng thc cho tng ng vi:
1
1 1
1

2 1 + xy
2 1 + xy
0
1+ x
1+ y

xy x 2

xy y 2

(1 + x ).(1 + xy ) (1 + y ).(1 + xy )
2

x( y x)

1 + x . (1 + xy )
2

y( x y)

1 + y 2 . (1 + xy )

( y x )2 ( xy 1)
0
(1 + x2 ).(1 + y 2 ).(1 + xy )

BT cui ny ng do xy 1. Vy ta c pcm. ng thc xy ra khi v ch khi


x = y hoc xy = 1 .
b) S dng bt ng thc:
1
1+ x

Ta c
6

1
1+ a

1
1+ b

1
1+ y

2
1 + a 3b 3

2
, ( xy 1) .
1 + xy

www.TaiLieuLuyenThi.com
Cty TNHH MTV DVVH Khang Vit

1
1+ c

1
2

1 + abc 1 + abc 4

1
1
2
+

3 3
1 + abc 4
1+ a b

2.

1+

2
a3b3 . abc 4

.
4
=
1 + abc

Cng theo v ba bt ng thc trn ta c pcm.


Ch . Bt ng thc ny c p dng kh ph bin trong mt s bi ton cc tr.
Mt s dng tng t bt ng thc trn nh sau
1
1
2

, ( 1 < xy 1) .
2
2
1 + xy
1+ x
1+ y
1

1+ x

1 + x2

1+ y

1 + y2

2
, ( xy 1) .
1 + xy

2
, ( 1 < xy 1) .
1 + xy

c) S dng kt qu bi ton trn ta c :


1
1+ z

1
1+ x

1
1+ y

2
1 + x3 y 3

1
2

1 + xyz 1 + xyz 4

2
1 + x3 y 3

2
1 + xyz 4

1+

x4 y 4 z 4

4
=
1 + xyz

Cng theo v cc bt ng thc trn ta suy ra


1
1+ x

1
1+ y

1
1+ z

1
3
1
1
P=
+
+
(1 + xyz )
3
3
3
3
1 + xyz
1+ x 1+ y 1+ z

ng thc xy ra khi v ch khi x= y= z


Vy gi tr ln nht ca P = 3 .
V d 3. Chng minh rng vi mi s thc x,y ta c
2
3 1 + ( x + y )
.
1 + y2
4
Li gii

(1 + x )(
2

( 2 xy 1) + ( x y ) 1 + x + y 2 .
4
2
Ch : 1 + x 2 1 + y 2 =1 + ( x + y ) +
(
)
3
3

)(

www.TaiLieuLuyenThi.com
Khm ph t duy K thut gii bt T Bi ton Max Min ng Thnh Nam

1
.
2
V d 4. Chng minh rng vi mi s thc a,b khng m tho mn a, b < 1;

Bt ng thc c chng minh. ng thc xy ra khi v ch khi x = y =

(1 2a )(1 2b ) 4 1 a b 2 .
3
a + b , ta c

2
(1 a )(1 b )
2a b
Li gii
Bt ng thc cho tng ng vi:

( a b )2 ( 2a + 2b 3) 0 .
(1 a )(1 b )( 2 a b )2
Bt ng thc lun ng v ta c pcm.
Bi tp tng t
1
Chng minh rng vi mi s thc a,b khng m tho mn a, b < 1; a + b ta c
2

(1 + 2a )(1 + 2b ) 4 1 + a + b 2 .

(1 a )(1 b )
2a b
2) K thut phn tch hng ng thc
Phn tch thnh tng cc bnh phng

( xi yi )

0.

i =1

V d 1. Cho a,b,c l cc s thc bt k chng minh


a) a 2 + b 2 + c 2 ab + bc + ca .
b) ( ab + bc + ca ) 3abc ( a + b + c ) .
2

c) ( a + b + c )
2

)(

1
( b c )2 + ( c a )2 + ( a b )2 3 ( ab + bc + ca ) .
4

)(

d) a 2 + 2 b 2 + 2 c 2 + 2 3 ( a + b + c ) .
2

Li gii
a) Bt ng thc cho tng ng vi:
2a 2 + 2b 2 + 2c 2 2ab 2bc 2ca 0

) (

) (

a 2 2ab + b 2 + b 2 2bc + c 2 + c 2 2ca + a 2 0 .


( a b) + (b c ) + (c a ) 0
2

Bt ng thc cui ng. ng thc xy ra khi v ch khi a= b= c .


8

www.TaiLieuLuyenThi.com
Cty TNHH MTV DVVH Khang Vit

b) Thc hin tng t cu a) a v bt ng thc lun ng

( ab bc )2 + ( bc ca )2 + ( ca ab )2 0 .
c) Ta c:
1
1
1
( a + b + c )2 ( b c=
)2
( 2a b c )2 + ab + bc + ca ab + bc + ca .
3
4
12
Tng t ta c:
1
1
( a + b + c )2 ( c a )2 ab + bc + ca
3
4
.
1
1
2
2
a
+
b
+
c

ab
+
bc
+
ca
(
)
(
)
3
4
Cng li theo v ba bt ng thc trn ta c pcm.
ng thc xy ra khi v ch khi a= b= c .
d) Ch ng thc:

(a

)(

)(

)
( a + 2 )( b

+ 2 b2 + 2 c2 + 2 3( a + b + c )

1 2
3
2
2
2
c + 2 ( a b ) + 2 ( ab 1) + ( ac + bc 2 ) .

2
2
2

)(

+ 2 c2 + 2 3( a + b + c )

Ta c iu phi chng minh. ng thc xy ra khi v ch khi a= b= c= 1 .


V d 2. Cho x, y, z l cc s thc tha mn iu kin x 2 + y 2 + z 2 =
1.
Chng minh rng
1
a) xy + yz + zx 1 ;
2
8
2
b) ( xy + yz + 2 xz )
3.
2
( x + y + z ) xy yz + 2
Li gii
a) Bt ng thc v tri tng ng vi:
2 ( xy + yz + zx ) + 1 0 2 ( xy + yz + zx ) + x 2 + y 2 + z 2 0 ( x + y + z ) 0 .
2

Bt ng thc c chng minh.


0
x + y + z =
1
1
ng thc xy ra khi v ch khi 2 2 2 , chng hn ti x =

,y=
.
2
2
1
x + y + z =
Bt ng thc v phi:
1 ( xy + yz + zx ) = x 2 + y 2 + z 2 ( xy + yz + zx )

1
( x y )2 + ( y z )2 + ( z x )2 0
2
xy + yz + zx 1

www.TaiLieuLuyenThi.com
Khm ph t duy K thut gii bt T Bi ton Max Min ng Thnh Nam

b) Ch iu kin ta rt gn v tri v a v chng minh


8
3
( xy + yz + 2 xz )2
xy + yz + 2 zx + 3
.
3
xy + yz + 2 zx + 1)
(

0
xy + yz + 2 zx + 3
Vy ta ch cn chng minh
xy + yz + 2 zx 1 = x 2 y 2 z 2
2

1
3

( x + z ) + y + y ( x + z ) 0 x + z + y + y2 0
2
4

Bt ng thc cui ng v ta c pcm.


2

y = 0

1
1
1

ng thc xy ra khi v ch khi x + z + y =0 x = , y =0, z =


.
2
2
2

x 2 + y 2 + z 2 =
1

V d 3. Cho x,y,z l cc s thc khng m. Chng minh:


a) x3 + y 3 + z 3 3 xyz .
b)

x3 + y 3 + z 3
3
xyz + ( x y )( y z )( z x ) .
3
4
3

y+z

c) x3 + y 3 + z 3 3 xyz 2
x .
2

Li gii
a) Bt ng thc cho tng ng vi:

( x + y + z ) ( x 2 + y 2 + z 2 xy yz zx ) 0

.
1
( x + y + z ) ( x y )2 + ( y z )2 + ( z x )2 0
2
Bt ng thc c chng minh. ng thc xy ra khi v ch khi x= y= z .

b) Bt ng thc cho tng ng vi:


1
3
( x + y + z ) ( x y )2 + ( y z )2 + ( z x )2 ( x y )( y z )( z x )
6
4
.
2
2
2

( ( x + y ) + ( y + z ) + ( z + x ) ) ( x y ) + ( y z ) + ( z x ) 9 ( x y )( y z )( z x )

Ch x + y x y ; y + z y z ; z + x z x v s dng bt ng thc
chng minh c cu a) ta c
10

www.TaiLieuLuyenThi.com
Cty TNHH MTV DVVH Khang Vit

( x + y) + ( y + z ) + ( z + x)

x y + y z + z x 33 ( x y )( y z )( z x )

( x y )2 + ( y z )2 + ( z x )2 3 3 ( x y )2 ( y z )2 ( z x )2

Nhn theo v hai bt ng thc trn ta c iu phi chng minh. ng thc xy


ra khi v ch khi x= y= z .
c) Theo cu a) ta c x3 + y 3 + z 3 3 xyz 0, do nu
lun ng.
+ Ngc li xt y + z 2 x > 0 ( y x ) + ( z x ) > 0 .

y+z
x 0 bt ng thc
2

t y =2a + x, z =2b + x bt ng thc tr thnh

12 x a 2 ab + b 2 + 6 ( a + b )( a b ) 0.
2

y+z
x>0.
2
ng thc xy ra khi v ch khi a= b= c hoc=
b c=
,a 0 .

Bt ng thc ng v a +=
b

Bi tp tng t
Cho a,b l hai s thc khc 0 tho mn iu kin ab

1 1
+ +3.
a b

1
1
Chng minh rng ab 3 + 3 .
b
a
V d 4. Cho x,y,z l cc s thc dng chng minh
3( x 2 + xy + y 2 )( y 2 + yz + z 2 )( z 2 + zx + x 2 ) ( x + y + z )2 ( xy + yz + zx) 2 .
Li gii
3
1
3
Ch x 2 + xy + y 2 =
( x + y )2 + ( x y )2 x 2 + xy + y 2 ( x + y )2 .
4
4
4
3
3
2
2
y 2 + yz + z 2 ( y + z ) ; z 2 + zx + x 2 ( z + x )
4
4
27
2
2
2
Do ( x 2 + xy + y 2 )( y 2 + yz + z 2 )( z 2 + zx + x 2 ) ( x + y ) ( y + z ) ( z + x ) .
64
Ta ch cn chng minh
64
[( x + y )( y + z )( z + x)]2 [( x + y + z )( xy + yz + zx)]2
81
8
( x + y )( y + z )( z + x ) ( x + y + z )( xy + yz + zx)
9

x( y z )2 + y ( z x)2 + z ( x y )2 0
Bt ng thc cui ng. Ta c iu phi chng minh.
11

www.TaiLieuLuyenThi.com
Khm ph t duy K thut gii bt T Bi ton Max Min ng Thnh Nam

V d 5. Cho a,b,c l cc s thc dng tha mn iu kin 0 < a b c 1 .

Tm gi tr ln nht ca biu thc P=

a + b c (a + b) c .

Li gii
Ta c
P=

a + b c ( a + b ) c=

ac + bc a b ac + bc a b

1
1
1 1

a + b a b =
a b +
2
2
2 2

ng thc xy ra khi v ch khi c= 1, a= b=

1
.
2

3) K thut thm bt hng s


Vic cng hoc tr hai v ca bt ng thc cho mt s no lm lc b i
phn phc tp ca bt ng thc.
V d 1. Cho x,y,z l cc s thc khng m tho mn iu kin x y z. Chng
minh rng
xy + yz + zx x + z
a) 2
.

y + yz + z 2 y + z
xy + yz + zx

b)

x + xy + y
2

( x + z )( y + z )
( x + z ) + ( x + z )( y + z ) + ( y + z )2
2

Li gii
a) Bt ng thc cho tng ng vi:
xy + yz + zx y 2 + yz + z 2

x+z
y+z

xy + yz + zx
y 2 + yz + z 2
y
y
x+z
y+z

zx
z2

z x ( y + z ) z ( x + z ) 0 z xy z 2 0
x+z y+z

Bt ng thc cui ng. ng thc xy ra khi v ch khi x= y= z hoc


=
z 0,=
x y.
b) Bt ng thc cho tng ng vi:

( x + z )2 + ( x + z )( y + z ) + ( y + z )2 1 ( x + z )( y + z ) 1
x 2 + xy + y 2

12

3z ( x + y + z )
x 2 + xy + y 2

xy + yz + zx

z2
z 3 ( x + y + z )( xy + yz + zx ) z x 2 + xy + y 2 0

xy + yz + zx

www.TaiLieuLuyenThi.com
Cty TNHH MTV DVVH Khang Vit

Bt ng thc cui ng v

3 ( x + y + z )( xy + yz + zx ) 3 ( x + y ) z ( x + =
y ) 3z ( x + y ) z x 2 + xy + y 2 .
2

Bt ng thc c chng minh. ng thc xy ra khi v ch khi z = 0 .


4) K thut bin i vi bt ng thc cha cn
+ Php bnh phng hai v c u tin.
+ Cn chng minh

A1 + A2 + ... + An b1 + b2 + ... + bn .

Ta c chng minh

A1 =

b12 + c12 b12 = b1 .

Ri cng li theo v cc bt ng thc trn ta c pcm.


V d 1. Chng minh vi mi s thc x,y cng du v s thc k, ta c
k2 + x + k2 + y k + k2 + x + y .

Li gii
Bt ng thc cho tng ng vi
k 2 + x + k 2 + y + 2 (k 2 + x)(k 2 + y ) 2k 2 + x + y + 2k k 2 + x + y
k + k ( x + y ) + xy k k + x + y xy 0
4

Bt ng thc cui ng ta c pcm. ng thc xy ra khi v ch khi hoc x


bng 0 hoc y bng 0.
Bi 2. Chng minh rng vi x,y l hai s thc khng m tha mn x + y 1, ta lun
c

x2 + x + 4 + y 2 + y + 4 2 +

( x + y )2 + x + y + 4 .

Li gii
Bt ng thc cho tng ng vi
x2 + y 2 + x + y + 8 + 2

(x

)(

+ x + 4 y2 + y + 4

( x + y )2 + x + y + 4 + ( x + y )2 + x + y + 4 .

4+4

(x

)(

+ x + 4 y 2 + y + 4 xy + 2

)(

x 2 + x + 4 y 2 + y + 4 x 2 y 2 + 4 xy

( x + y )2 + x + y + 4 .

( x + y )2 + x + y + 4 + 4 ( x + y )2 + x + y + 4 .

2
xy 4 ( x + y ) + x + y + 4 + x + y 7 0 (lun ng do x + y 1 ).

Tng qut. Tng t ta c cc bt ng thc cng dng sau


+ Vi mi s thc khng m x,y ta lun c

x2 + x + k 2 + y 2 + y + k 2 k +

( x + y )2 + x + y + k 2

.
13

www.TaiLieuLuyenThi.com
Khm ph t duy K thut gii bt T Bi ton Max Min ng Thnh Nam

ng thc xy ra khi v ch khi x = 0 hoc y = 0 .


+ Vi mi s thc khng m ta lun c
x2 x + 1 + y 2 y + 1 1 +

( x + y )2 x y + 1 .

ng thc xy ra khi v ch khi x = 0 hoc y = 0 .


+ 1 + a + 1 + b 1 + 1 + a + b , ( ab 0; a, b 1; a + b 1) .
5) K thut nh gi phn thc
1 1
< .
A B
V d 1. Chng minh rng vi mi a,b,c dng ta c
a
b
c
1<
+
+
<2.
a+b b+c a+c
Li gii
1
1
a
a
Ta c : a + b < a + b + c
>

>
(1)
a+b a+b+c
a+b a+b+c
b
b
c
c
Tng t ta c :
>
(2) ,
>
(3)
b+c a+b+c
a+c a+b+c
Cng v theo v cc bt ng thc (1), (2), (3), ta c :
a
b
c
+
+
> 1 (*)
a+b b+c a+c
a
a+c
Ta c : a < a + b
<
(4)
a+b a+b+c
c
c+b
b
a+b
Tng t :
<
(6)
<
(5) ,
c+a a+b+c
b+c a+b+c
Cng v theo v cc bt ng thc (4), (5), (6), ta c :
a
b
c
+
+
< 2 (**)
a+b b+c a+c
a
b
c
T (*) v (**) , ta c : 1 <
+
+
< 2 (pcm)
a+b b+c a+c
V d 2. Cho a,b,c l cc s thc khng m tho mn iu kin
a
b
c
+
+
=
3.
1 + bc 1 + ca 1 + ab
a
b
c
3
Chng minh rng
+
+
.
1 + a + bc 1 + b + ca 1 + c + ab 4
Li gii
a
b
c
t=
x
=
;y
=
;z
x + y +=
z 3.
1 + bc
1 + ca
1 + ab

S dng nh gi c bn: A > B > 0

14

www.TaiLieuLuyenThi.com
Cty TNHH MTV DVVH Khang Vit

Ta cn chng minh
Ch

x
y
z
3
+
+
.
1+ x 1+ y 1+ z 4

x
x
.

1+ x 1+ x + y + z
y
y

1+ y 1+ x + y + z
z
z

1+ z 1+ x + y + z

Cng theo v ba bt ng thc trn ta c iu phi chng minh.


ng thc xy ra khi v ch khi c mt s bng 3 v hai s bng 0.
V d 3. Cho a,b,c l cc s thc khng m c tng bng 1. Chng minh
1 + a2
1+ b

1 + b2

1 + c2

7
.
2

1+ c
1+ a
Li gii
Ta thy du bng t ti khi mt s bng 1 v hai s bng 0.
2

Vy gi s a = max {a, b, c}. Khi ta mnh dn nh gi 1 + b 2 1;1 + c 2 1 .


Ta c 1 + b 2 1
1 + c2 1

1 + a2
1 + b2
1 + b2
1+ c

Suy ra P 2 + a 2 + b 2 +

1 + a2 ; .
1 + b2
1 + c2
1+ a

2 + a2 + (b + c ) +
2

2 + a 2 + b2 + c2 +
1

1 + a2

=2 + a 2 + (1 a ) +

1+ a

Ta ch cn chng minh 2 + a 2 + (1 a ) +
2

1
1+ a

1 + a2

7
.
2

( a 1) 4a + 3a 1 0
3

Bt ng thc cui lun ng nn ta c pcm. ng thc xy ra khi v ch khi


a= 1, b= c= 0 hoc cc hon v.
Ch . Bng cch tng t ta chng minh c

1 + ak
1+ b

1 + bk
1+ c

1 + ck
1+ a

7
.
2

Vi k l s nguyn dng.

15

www.TaiLieuLuyenThi.com
Khm ph t duy K thut gii bt T Bi ton Max Min ng Thnh Nam

V d 4. Cho x,y,z l cc s thc tho mn iu kin x, y 1; x + y + z =3 .


Tm gi tr ln nht
ca biu thc P
=

x2
x 2 + y 2 + 4 ( xy + 1)

y2 1
z2 4z + 5

Li gii
Trc ht nh gia hai mu s hai phn thc bng cch thay z = 3 x y .
Ta chng minh x 2 + y 2 + 4 ( xy + 1) z 2 4 z + 5

x 2 + y 2 + 4 xy ( 3 x y ) + 4 ( 3 x y ) 1 0
2

2 ( x + y ) + 2 xy + 2 0 ( x + 1)( y + 1) 0

Bt ng thc ng.
Vy ta c P

x2 + y 2 1

2
x + y ) 2 xy 1
(
.
=

z2 4z + 5
z2 4z + 5
Ch . xy 1 x y = z 4; x + y = 3 z .

Khi

2
2
3 z ) 2 ( z 4 ) 1 z 2 8 z + 16
2 z 3)
(
(
P
=
=

+55.
z2 4z + 5
z2 4z + 5
3

3
x+ y=

5
3
2

z = 2

x=
1, y =, z =

2
2.
Du bng t ti x + y + z =3 xy =
5
3
2
x =, y =
xy= z 4

1, z =

2
2

z = 2

z2 4z + 5

6) K thut nh gi bt ng thc c cha du gi tr tuyt i


S dng hai bt ng thc quen thuc: x + y x + y ; x y x y .
Ch . T duy u tin l kh du gi tr tuyt i mun vy ta xt trng hp.
V d 1. Chng minh vi mi s thc a,b,c ta c
a + b + c + a+b+c a+b + b+c + c+a .

Li gii
Trong ba s a,b,c c t nht hai s cng du khng mt tnh tng qut gi s l a
v b khi a + b = a + b .
Vy ta ch cn chng minh c + a + b + c b + c + c + a
c 2 + ( a + b + c ) + 2 c ( c + a + b ) ( a + c ) + ( b + c ) + 2 ( a + c )( b + c )
2

ab + c ( c + a + b ) ( a + c )( b + c )= c ( c + a + b ) + ab

Bt ng thc cui lun ng (pcm).


16

www.TaiLieuLuyenThi.com
Cty TNHH MTV DVVH Khang Vit

V d 2. Cho x,y,z l cc s thc i mt khng ng thi bng 0. Chng minh

(x
1
(x

y2

+ y2

)( y
)( y

z2

+ z2

)( z
)( z

x2

+ x2

) 1.
)

Li gii

Ta c x 2 y 2 x 2 + y 2 x 2 y 2
T suy ra

(x
(x

y2

+ y2

(x
1
(x

)( y
)( y

z2

+ z2

y2

+ y2

)( y
)( y

) (x
2

)( z
)( z

x2

z2

+ z2

4x2 y 2 0 .

) 1
+x )
)( z x ) 1
)( z + x )

+ y2

Bt ng thc c chng minh.


V d 3. Cho a,b,c l cc s thc khng m chng minh
3 3 abc + a b + b c + c a a + b + c.

Li gii
Khng mt tnh tng qut gi s a b c. Khi bt ng thc tng ng vi:
3 3 abc + (a b) + (b c) + (c a) a + b + c
a b 3c + 3 3 abc 0 (a b) + 3 3 c

ab c 2 0
3

Bt ng thc cui lun ng v ta c pcm.

Bi tp tng t
Cho a,b,c l cc s thc dng tho mn iu kin
3 3 abc + a b + b c + c a =
1.

1
Chng minh rng a bc + b ca + c ab .
3
V d 4. Cho x,y,z l cc s thc chng minh
x y + y z + z x 2 x 2 + y 2 + z 2 xy yz zx .

Li gii
Khng mt tnh tng qut gi s x y z. Bt ng thc tr thnh
( x y) + ( y z) + ( x z) 2

1
1
1
( x y )2 + ( y z )2 + ( z x )2
2
2
2

2
2
2
2 ( x z ) 2 ( x y ) + ( y z ) + ( z x )

17

www.TaiLieuLuyenThi.com
Khm ph t duy K thut gii bt T Bi ton Max Min ng Thnh Nam
2
2
2
2
4 ( x z ) 2 ( x y ) + ( y z ) + ( z x )

( x z) ( x y) + ( y z)
2

[ ( x y ) + ( y z ) ] ( x y ) + ( y z ) 2 ( x y )( y z ) 0
2

Bt ng thc cui lun ng ta c pcm.


V d 5. Cho n s thc x1 , x2 ,..., xn (vi n 3 ). Chng minh

x1 + x2 + ... + x n x1 x2 + x2 x3 + ... + xn 1 xn + xn x1
+
.
n
2n
Li gii
Ch . Vi hai s thc x,y bt k ta lun c
max { x1 , x2 ,..., xn }

min { x, y} x, y max { x, y} v max { x, y} =

S dng max { x, y} =

x+ y+ x y
2

x+ y+ x y
2

ta c:

x1 + x2 + ... + x n x1 x2 + x2 x3 + ... + xn 1 xn + xn x1
+
n
2n
x + x + xn x1
x1 + x2 + x1 x2 x2 + x3 + x2 x3
+
+ ... + n 1
2n
2n
2n
max { x1 , x2 } + max { x2 , x3 } + ... + max { xn 1 , xn } + max { xn , x1}

max { x1 , x2 ,..., xn }
n
Bi ton c chng minh. Du bng t ti chng hn x1= x2= ...= x n .

7) K thut t n ph
Vi bt ng thc i xng hai bin ta c th t u =
a + b; v =
ab .
Vi phn thc ta c t cc mu s l cc bin mi.
V d 1. Chng minh rng vi mi a,b dng, ta c

a 2b 2 a 2 + b 2 2 ( a + b )( ab 1) .

Li gii
t a + =
b 2u , ab
= v , v > 0. Khi bt ng thc tng ng vi:
2

a 2b 2 (a 2 + b 2 2) (a + b)(ab 1)
a 2b 2 ( a + b ) 2ab 2 ( a + b )( ab 1)

v 4 (4u 2 2v 2 2) 2u (v 2 1)
2v 4u 2 (v 2 1)u v 4 (v 2 + 1) 0
18

www.TaiLieuLuyenThi.com
Cty TNHH MTV DVVH Khang Vit

iu ny chng t u
Mt khc u =
v

v 2 1 + (v 2 1) 2 + 8v8 (v 2 + 1)
4v 4

a+b
ab = v do ta ch cn chng minh:
2

v 2 1 + (v 2 1) 2 + 8v8 (v 2 + 1)
4

(v 1) 2 (v + 1)(v 2 + v + 1) 0

4v
Bt ng thc cui ng. ng thc xy ra khi v ch khi a= b= 1 .
V d 2. Cho a,b,c l cc s thc dng chng minh rng
x2 z 2 y 2 x2 z 2 y 2
+
+
0.
y+z
z+x
x+ y

Li gii
t a =
x + y, b =y + z , c =+
z x khi v tri ca bt ng thc l

( a b) c + (b c ) a + (c a ) b =
b

ab bc ca
+ + a bc
c
a
b

1 ab
bc
1 bc
ca
1 ca
ab
=

+
+
0
2 c
a
2 a
b
2 b
c
Bt ng thc c chng minh. ng thc xy ra khi v ch khi x= y= z.

V d 3. Cho a,b,c l cc s thc dng tha mn iu kin a + b + c =


3.
4
4
4
a
b
c
.
Chng minh rng
+
+

+
+
3
3
3
( a + b) (b + c ) (c + a ) b + c c + a a + b
Li gii
x
a + b =
a= 3 y

t b + c =
3 nn x, y, z > 0 v b= 3 z .
y do a, b, c > 0 v a + b + c =
c + a =
c= 3 x
z

Khi bt ng thc tr thnh:

4
x

4
y

4
z

3 y 3 x 3 z
.
+
+
y
x
z

4 3 x 4 3 y 4 3 z
3

+
0
+
x y 3
y z 3
z
x
2
2
2
x + 1)( x 2 )
y + 1)( y 2 )
z + 1)( z 2 )
(
(
(

+
+
0.

x3

y3

z3

Bt ng thc cui lun ng, t ta c pcm.

19

www.TaiLieuLuyenThi.com
Khm ph t duy K thut gii bt T Bi ton Max Min ng Thnh Nam

8) K thut s dng php th


T bi ton c iu kin t hai bin tr ln ta rt mt bin theo cc bin cn li
ri thay vo bt ng thc cn chng minh.
+ Dng ny ton nu c cn kt hp nh gi mt s l max hoc mt s l min.
V d 1. Cho a,b,c l cc s thc khng m tho mn iu kin ab + bc + ca =
1.
5abc
Chng minh rng a + b + c +
2.
3
Li gii
Khng mt tnh tng qut gi s c b a .
1 ab
1 ab
1 ab
ta phi chng minh a + b +
Thay c =
+ 5ab.
2
3( a + b )
a+b
a+b
ab ( 2 5ab ) + 3 ( a + b 1) 0
2

1
Bt ng thc hin nhin ng v ab .
3

9) K thut nh gi theo cp
p dng vi dng tch bt ng thc dng tch.

V d 1. Cho a,b,c l cc s thc thuc khong ( 0;1) . Chng minh rng

( a a )(b b )( c c ) ( a bc )( b ca )( c ab ) .
2

Li gii
a bc > 0
Khng mt tnh tng qut gi s a b c. Khi do a, b, c ( 0;1)
.
b ca > 0
Nu c ab < 0 bt ng thc lun ng.

Nu c ab 0 khi ta chng minh


Tht vy

bc (1 a )

bc (1 a )

( b ac )( c ab ) .

( b ac )( c ab ) bc (1 a )2 ( b ac )( c ab ) .

bc a 2 2a + 1 bc ab 2 ac 2 + a 2bc a ( b c ) 0 (lun ng).


2

ac (1 b ) ( a bc )( c ab )

.
Tng t ta c:
ab (1 c ) ( a bc )( b ca )
Nhn theo v 3 bt ng thc trn ta c iu phi chng minh.
ng thc xy ra khi v ch khi a= b= c .
V d 2. Cho a,b,c l cc s thc dng tha mn iu kin a + b + c =
1. Chng

minh rng 8a 2b 2 c 2 ( a bc )( b ca )( c ab ) .
20

www.TaiLieuLuyenThi.com
Cty TNHH MTV DVVH Khang Vit

Li gii
a bc > 0
.
Khng mt tnh tng qut gi s a b c khi do a, b, c ( 0;1)
b ca > 0
Nu c ab < 0 bt ng thc lun ng.

Nu c ab 0 khi ta chng minh 2ab


Tht vy 2ab

( a bc )( b ca ) .

( a bc )( b ca ) 4a 2b2 ( a bc )( b ca ) .

4a 2b 2 ab a 2 c b 2 c + abc 2 4a 2b 2 + c a 2 + b 2 ab c 2 + 1 0

4a 2b 2 + c ( a b ) + ab 2c c 2 1 0 4a 2b 2 + c ( a b ) ab ( c 1) 0
2

2
2
2
ab 4ab ( a + b ) + c ( a b ) 0 ( c ab )( a b ) 0 (lun ng).

( b ca )( c ab ) ;2ca ( c ab )( a bc ) .

Tng t ta c 2bc

Nhn theo v 3 bt ng thc trn ta c ngay iu phi chng minh. ng thc


1
xy ra khi v ch khi a= b= c=
.
3
10). K thut s dng tnh thun nht
a bt ng thc v dng ng bc s d x l hn(xem thm chng 3).
V d 1. Cho a,b,c l cc s thc tho mn iu kin a 2 + b 2 + c 2 =
3.
Chng minh rng a3 ( b + c ) + b3 ( c + a ) + c3 ( a + b ) 6 .
Li gii
Bt ng thc cho tng ng vi:
a 3 ( b + c ) + b3 ( c + a ) + c 3 ( a + b )

) (

2 2 2 2
a +b +c
3

2 a 4 + b 4 + c 4 + 4 a 2b 2 + b 2 c 2 + c 2 a 2 3ab a 2 + b 2 + 3bc b 2 + c 2 + 3ca c 2 + a 2

Bt ng thc trn l tng ca ba bt ng thc c dng:

a 4 + b 4 + 4a 2b 2 3ab a 2 + b 2 = ( a b ) + ab ( a b )
4

= ( a b ) a 2 ab + b 2 0
2

Bt ng thc c chng minh. ng thc xy ra khi v ch khi a= b= c= 1.


11) Bin i hm lng gic
V d 1. Chng minh vi mi s thc x ta c cos(sin x) > sin(cos x) .
Li gii

Bt ng thc cho tng ng vi: sin sin x sin(cos x) > 0


2

21

www.TaiLieuLuyenThi.com
Khm ph t duy K thut gii bt T Bi ton Max Min ng Thnh Nam

2cos 2

sin x + cos x

.sin 2

sin x cos x

2
Bt ng thc cui lun ng do

V vy 0 < 2

0< 2

>0.

2 sin x 2; sin x + cos


=
x
4

sin x cos
=
x

2
2
2

< 2

sin x cos x
2
sin x cos x

2
2
Bt ng thc c chng minh.

+ 2
2
+ 2
2

<

<

2 sin x + 2 .
4

B. BI TON CHN LC
Bi 1. Cho x, y, z l cc s thc dng tha mn iu kin x y z .
1 1 1
1 1
Chng minh rng y + + ( x + z ) + ( x + z ) .
x z y
x z

Li gii
BT tng ng vi:

( x + z)
xz

y( x + z)
xz

x+z
y

y ( x + z ) y 2 + zx y 2 y ( z + x ) + zx 0 ( y x )( y z ) 0

Bt ng thc cui ng v 0 < x y z .


Ta c iu phi chng minh. ng thc xy ra khi v ch khi x= y= z .
Bi 2. Cho x,y,z l cc s thc dng tho mn iu kin x y z. Chng minh
x( x 2 + y 2 ) y ( z 2 + x 2 ) z ( y 2 + z 2 )
+
+
x2 + y 2 + z 2 .
x+ y
z+x
y+z

Li gii
Bt ng thc cho tng ng vi:
1
xy ( y x) 1
1
1

+ yz ( y z )
0
x+ y x+z x+ y
y+z x+z

xy ( x y )( y z ) yz ( x y )( y z )
+
0
( x + y )( x + z )
( x + z )( y + z )

Ta c iu phi chng minh.


22

www.TaiLieuLuyenThi.com
Cty TNHH MTV DVVH Khang Vit

Bi 3. Cho x, y, z l cc s thc thuc on [ 0;1] . Tm gi tr ln nht ca biu

) (

thc P= 2 x3 + y 3 + z 3 x 2 y + y 2 z + z 2 x .
Li gii
Ta c x, y, z [ 0;1] x x x; y 3 y 2 y; z 3 z 2 z.
3

T suy ra 2 x3 + y 3 + z 3 x 2 + x + y 2 + y + z 2 + z.

P x + y + z + x2 + y 2 + z 2 x2 y + y 2 z + z 2 x .

Ta chng minh x + y + z + x 2 + y 2 + z 2 x 2 y + y 2 z + z 2 x 3 .
x 2 (1 y ) + y 2 (1 z ) + z 2 (1 x ) + x + y + z 3 0

x 2 1 (1 y ) + y 2 1 (1 z ) + z 2 1 (1 x ) 0 .

Bt ng thc cui ng do

( x 1) (1 y ) 0; ( y 1) (1 z ) 0; ( z 1) (1 x ) 0 .
2

Vy gi tr ln nht ca P bng 3 xy ra khi x= y= z= 1 .


Bi 4. Chng minh rng vi mi s thc khng m a,b,c ta c

a 4 + b 4 + c 4 + abc ( a + b + c ) 2 a 2b 2 + b 2 c 2 + c 2 a 2 .

Li gii
Khng mt tnh tng qut gi s a b c. Khi c 2 ( c a )( c b ) 0 v
a 2 ( a b )( a c ) + b 2 ( b c )( b a ) = ( a b ) a 2 ( a c ) b 2 ( b c )

( a b ) a 2 ( b c ) b2 ( b c )

=( a b )( b c ) a 2 b 2 0

Bt ng thc c chng minh. ng thc xy ra khi v ch khi a= b= c .


Nhn xt. y l mt trng hp ring ca bt ng thc Schur. Vi a,b,c l cc
s thc khng m v k > 0 ta lun c
a k ( a b )( a c ) + b k ( b c )( b a ) + c k ( c a )( c b ) 0 .

Bi 5. Cho a, b, c 0 thoa man iu kin a + b + c =


1.
Tim gia tri ln nht cua biu thc P =

2
b c)
(
a+

2
c a)
(
b+

2
a b)
(
c+

Li gii
Chuyn mi biu thc trong cn v cng bc hai ta co :
23

www.TaiLieuLuyenThi.com
Khm ph t duy K thut gii bt T Bi ton Max Min ng Thnh Nam
2
b c)
(
a+
=

a (a + b + c)

2
b c)
(
+

= a + a (b + c )
2

Suy ra

2
b c)
(
a+

Tng t ta c :

a+

b+

2
b + c ) 4bc
(
+
=

b+c
b+c .

a +
bc a +

2
2

b+c
2

( c a )2
4

2
a b)
(
c+

b+

c+a
;
2

a+b
4
2
Cng theo v 3 bt ng thc trn ta c :

( b c )2

c+

( c a )2

( a b )2

2 ( a + b + c )= 2.
4
4
4
Vy gi tr ln nht ca P bng 2 t ti a= b= 0, c= 1 hoc cac hoan vi.
P=

a+

+ b+

+ c+

Nhn xt. Ta c th tng qut thnh bi ton nh sau :


Cho a,b,c,k l cc s thc khng m tha mn a + b + c =
k . Chng minh rng
2
b c)
(
ka +

2
c a)
(
kb +

2
a b)
(
kc +

+
+
2k .
4
4
4
Bi 6. Cho a,b,c l cc s thc khng m tha mn iu kin a + b + c =
1.

Chng minh rng

a + ( b c ) + b + ( c a ) + c + ( a b ) 3.
2

Li gii
Khng mt tnh tng qut gi s a b c khi :
S dng bt ng thc Mincopsi ta c :
a + (b c ) + b + (c a ) + c + ( a b)
2

a+ b+ c

+ ( a b ) + ( b c ) + ( c a )

a+ b+ c

+ 4(a c) .
2

Bt ng thc c chng minh nu ta chng minh c bt ng thc sau ng.

)
2
4(a c) (

24

a+ b+ c

+ 4 ( a c ) 3( a + b + c )
2

a b

) +(
2

b c

) +(
2

c a

www.TaiLieuLuyenThi.com
Cty TNHH MTV DVVH Khang Vit

Ta c :

) (

a b +

Suy ra

(
(
(

) + ( b c ) + 2 ( a b )(
2
2
a b) +( b c)
2
2
2
c a) ( a b) +( b c)
2
2
2
c ) + ( c a ) 2( c a ) .

b c =

a b

) +(
2

a b

b c

Mt khc :
4(a c) 2
2

c a = 2

a c

(
= 2(
2(
=2 (

= 2

) 2 (
2

a+ c

) 2 ( a + c ) 1 + 4 ac
2
c ) 2a + c + c 1 + 4 ac
2
c ) a + b + c + c 1 + 4 ac
2
c ) c + 4 ac 0

a c
a
a
a

Bi ton c chng minh. ng thc xy ra khi v ch khi a= b= c=

1
hoc
3

a= 1, b= c= 0 v cc hon v.

Bi 7. Cho a,b,c l cc s thc dng tha mn iu kin a + b + c =


3.

Chng minh rng 2 a 2b 2 + b 2 c 2 + c 2 a 2 + 3 3 a 2 + b 2 + c 2 .


Li gii
Khng mt tnh tng qut =
gi s a min {a, b, c} a 1 .

Ta c 2 a 2b 2 + b 2 c 2 + c 2 a 2 + 3 3 a 2 + b 2 + c 2 0

(
( 2a

)(
)
3) ( b + c ) + 2b c + 2 ( 3 2a ) bc + 3 3a 0
( 2a 3) ( 3 a ) + 2b c + 2 (3 2a ) bc + 3 3a 0 .

2a 2 3 b 2 + c 2 + 2b 2 c 2 + 3 3a 2 0

=
P

2 2

2 2

b + c 3 a
2
Ta c bc
=

. V a 1 3 2a > 0 do
2
2

25

www.TaiLieuLuyenThi.com
Khm ph t duy K thut gii bt T Bi ton Max Min ng Thnh Nam

P 2a 3 ( 3 a )
=

3a
2 3a
2
+ 2.
+ 2 3 2a
+ 3 3a
2
2
.

3
( a 1)2 3a 2 14a 1 0
8

V 3a 2 14a
=
1 3a ( a 1) 11a 1 < 0, a ( 0;1] .
Bi ton c chng minh. ng thc xy ra khi v ch khi a= b= c= 1 .
3
Bi 8. Cho cc s thc a, b, c [ 0;1] thoa man a + b + c = .
2

Tim gia tri ln nht v gi tr nho nht cua biu thc=


P cos a 2 + b 2 + c 2 .
Li gii
Do a, b, c [ 0;1] nn 0 a 2 + b 2 + c 2 a + b + c =

3
< .
2 2

Vy P ln nht( nho nht) khi a 2 + b 2 + c 2 nho nht (ln nht)


- Tim gia tri nho nht cua a 2 + b 2 + c 2 .
1
3
3
2
Ta co a 2 + b 2 + c 2 ( a + b + c ) = . Suy ra GTLN cua P bng cos ; xay
3
4
4
1
ra khi a= b= c=
2
- Tm gia tri ln nht cua a 2 + b 2 + c 2 .
3
1
Gia s a b c a + b + c = 3c c .
2
2
Vy a + b + c = ( a + b ) 2ab + c ( a + b )
2

5
3

+ c =c + c
4
2

Do ( c 1)( 2c 1) 0
1
5

; xay ra khi ( a, b, c ) = 0,0, hoc cac hoanvi.


2
4

Bi 9. (TSH Khi D 2008) Cho x, y la cac s thc khng m.

Suy ra GTNN cua P bng cos

Tim gia tri ln nht, gia tri nho nht cua biu thc P =

( x y )(1 xy ) .
( x + 1)2 ( y + 1)2

Li gii
Ta co :
=
P
26

xy )
( x y )(1=
( x + 1)2 ( y + 1)2

+ yx )
( x + xy ) ( y =
2

( x + 1) ( y + 1)
2

x ( y + 1) y ( x + 1)
=
( x + 1)2 ( y + 1)2
2

( x + 1)

( y + 1)2

www.TaiLieuLuyenThi.com
Cty TNHH MTV DVVH Khang Vit

Vi x, y 0 ta c : 0

( x + 1)

1
y
1
;0
.
2
4
( y + 1) 4

Suy ra gi tr ln nht ca P bng

1
t ti =
x 1,=
y 0 v gi tr nh nht ca
4

1
t ti=
x 0,=
y 1.
4
Cch 2 : Ta c nh gi thng qua tr tuyt i nh sau :

P =

( x y )(1 xy )
(1 + x )2 (1 + y )2

( x + y )(1 + xy ) ( x + y + 1 + xy )2

(1 + x )2 (1 + y 2 ) 4 (1 + x )2 (1 + y )2

1
.
4

1
1
Do P . Ta c kt qu tng t.
4
4
Bi 10. Cho a, b, c 0 la cac s i mt khac nhau. Chng minh rng

( ab + bc + ca )

4.
2
c

a
(
)
1

(b c )
(a b)
Li gii
Gia s c = min {a, b, c} , khi o do a, b, c 0 ta suy ra: ab + bc + ca ab;
2

(b c )

(a c)

1
b2

;
1
a2

1
1
1
4.
Vy ta chi cn chng minh : ab
+
+
( a b )2 b 2 a 2

ab

( a b )2

(a b) 2 0
a b
ab
+ 40
+
b a
ab
( a b )2
2

a b )
(
ab

0
( a b )2
ab

Bi ton c chng minh. Xem thm chng 3.

Bi 11. Cho cc s thc tho mn iu kin a, b, c > 0 va


Tim gia tri nho nht cua biu thc
=
P

1 1 2
+ =.
a c b

a+b
c+b
.
+
2 a b 2c b
27

www.TaiLieuLuyenThi.com
Khm ph t duy K thut gii bt T Bi ton Max Min ng Thnh Nam

Li gii
2ac
thay vao biu thc ca P ta c :
a+c
2ac
2ac
a+
c+
a + 3c c + 3a
3 a c
a
c
a
+
+c =
P=
+
+
=+
1
+ 4.
2ac
2ac
2a
2c
2 c a
2a
2c
a+c
a+c
(ng theo AM-GM).
Vy gi tr nh nht ca P bng 4 t ti a= b= c .

Ta co b =

Bi 12. Cho cc s thc a, b, c [1;3] thoa man iu kin a + b + c =


6.
Tim gia tri ln nht cua biu thc P = a 2 + b 2 + c 2 .
Li gii
t a =x + 1; b =y + 1; c =z + 1; x, y, z [ 0;2].
Khi o P = a 2 + b 2 + c 2 = ( x + 1) + ( y + 1) + ( z + 1)
2

= x2 + y 2 + z 2 + 2 ( x + y + z ) + 3
= ( x + y + z ) 2 ( xy + yz + zx ) + 2 ( x + y + z ) + 3
2

=
2 ( xy + yz + zx ) + 18

T x, y, z [ 0;2] ( 2 x )( 2 y )( 2 z ) 0
8 4 ( x + y + z ) + 2 ( xy + yz + zx ) xyz 0
2 ( xy + yz + zx ) = 4 xyz 4 do xyz 0

T o suy ra P 2 ( xy + yz + zx ) + 18 14
ng thc xay ra khi va chi khi ( a, b, c ) = (1, 2,3) hoc cac hoan vi
Ch . t a =x + 1; b =y + 1; c =z + 1 chung ta tn dung tich xyz 0
Bi 13. Cho cc s thc a, b, c [ 0;1] .
1
1
1
+
+
3abc .
2a 2b 2c
Li gii
1
2
Ta c : ( a 1) 0 a ( 2 a ) 1
a.
2a
1
1
Tng t ta c :
b;
c.
2b
2c
Cng theo v cac bt ng thc trn ta c:

Chng minh rng

28

www.TaiLieuLuyenThi.com
Cty TNHH MTV DVVH Khang Vit

1
1
1
+
+
a + b + c 3 3 abc 3abc do abc 1 .
2a 2b 2c
ng thc xay ra khi va chi khi a= b= c= 1 .

Bi 14. Cho cc s thc a, b, c [ 0;1] va a + b + c 0 .

1
1
1
5
.
+
+

ab + 1 bc + 1 ca + 1 a + b + c
Li gii
Do bt ng thc i xng vi ba bin nn khng mt tinh tng quat ta gia s
1 a b c 0 .

Chng minh rng

Khi o

c
a
b
a + b + c 1 + b + c + (1 b )(1 c ) + bc
+
+

=
2
ab + 1 bc + 1 ca + 1
bc + 1
bc + 1

Mt khac
=

a+b b+c c+a a+b


b+c
c+a

+
+ =
1 +
1 +
1 + 3
ab + 1 bc + 1 ca + 1 ab + 1 bc + 1 ca + 1

(1 a )(1 b ) (1 b )(1 c ) (1 c )(1 a ) + 3 3

ab + 1
bc + 1
ca + 1
Cng theo v cac bt ng thc trn ta suy ra iu phi chng minh.
ng thc xay ra khi va chi khi a= b= 1, c= 0 hoc cac hoan vi.

Bi tp tng t
Cho a,b,c l cc s thc thuc on [0;1] v a + b + c > 0.
5
ab
bc
ca
.
Tm gi tr nh nht ca biu thc P=
+
+
+
a + b + c 1 + ab 1 + bc 1 + ca
Bi 15. Cho cc s thc khng m a,b,c. Chng minh rng
a 2 + b 2 + c 2 4 ( a + b + c )( a b )( b c )( c a ) .

Li gii
Khng mt tinh tng quat ta gia s b nm gia a va c , ta xet hai trng hp
- Nu a b c VT 0 VP , ta co pcm.
- Nu c b a , khi o v phai
VP= 4 ( a + b + c )( a b )( b c )( c a ) = 4 ( a + b + c )( b a )( c b )( c a )

( a + b + c )( b a ) + ( c b )( c a )

Ta chi cn chng minh ( a + b + c )( b a ) + ( c b )( c a ) a 2 + b 2 + c 2 .


Tht vy bt ng thc nay tng ng vi : a ( 2a + 2c b ) 0 , ung v ta
c pcm.
29

www.TaiLieuLuyenThi.com
Khm ph t duy K thut gii bt T Bi ton Max Min ng Thnh Nam

C. BI TP RN LUYN
Bi 1. Chng minh rng vi mi s thc khng m a v b ta c

( a + b ) ( a 2 + b2 ) 8ab ( a + b ) 12ab

ab .

Bi 2. Cho a,b,c l di ba cnh mt tam gic tho mn a b c. Chng minh

( a + b + c )2 9bc .
x 2 cos a 2 x + cos a
Bi 3. Cho a ( 0; ) , x ; y =2
. Chng minh rng 1 y 1 .
x 2 x cos a + 1
Bi 4. Cho x,y,z l cc s thc dng tho mn iu kin x y z .

Chng minh rng

x2 y y 2 z z 2 x
+
+
x2 + y 2 + z 2 .
z
x
y

Bi 5. Cho a, b, c, d, e l cc s thc. Chng minh cc bt ng thc


a 2 + b2 + c 2 + d 2 + e2 a ( b + c + d + e ) ;
a 2 + b 2 + c 2 + d 2 + 1 a ( b + c + d + 1)

Bi 6. Cho a,b,c l cc s thc dng. Chng minh


a+b+c
.
3
a + ab + b
b + bc + c
c + ca + a
Bi 7. Cho x,y l hai s thc khng m khng ng thi bng 0. Chng minh
a3

b3

c3

3 x 2 + 2 xy + 3 y 2
2 x2 + 2 y 2 .
2( x + y)

Bi 8. Cho x,y l 2 s thc dng. Chng minh 1

Bi 9. Chng minh vi mi s thc a v b ta c

( x + y ) ( x3 + y 3 )

(x

a+b
1+ a + b

+y

2 2

9
.
8

a+b
1+ a + b

Du bng xy ra khi no?


Bi 10. Cho a,b l cc s thc v a khc 0. Chng minh a 2 + b 2 +
Bi 11. Chng minh rng vi mi s thc dng a,b,c ta c
a 2 b2 c2
+
+
a +b+c.
b
c
a
Bi 12. Chng minh rng vi mi s thc a,b,c ta c

a 4 + b 4 + c 4 abc ( a + b + c ) .
30

1
a

b
3.
a

www.TaiLieuLuyenThi.com
Cty TNHH MTV DVVH Khang Vit

Bi 13. Cho a,b,c l di ba cnh mt tam gic. Chng minh


a b c b c a
+ + <1.
b c a a b c

Bi 14. Cho a,b,c l di ba cnh mt tam gic tho mn iu kin a < b < c.

Chng minh rng a3 b 2 c 2 + b3 c 2 a 2 + c3 a 2 b 2 < 0 .


Bi 15. Cho a,b,c l di ba cnh mt tam gic. Chng minh
a ( b c ) + b ( c a ) + c ( a + b ) > a 3 + b3 + c 3 .
2

Bi 16. Cho a,b,c l di ba cnh mt tam gic. Chng minh


2a 2b 2 + 2b 2 c 2 + 2c 2 a 2 a 4 b 4 c 4 > 0 .

Bi 17. Cho x,y,z l cc s thc dng c tch bng 1 v z = min { x, y, z}.


2

Chng minh rng

x+2 y+2 z+2


.

1.
2x + 1 2 y + 1 2z + 1

Bi 18. Cho a,b,c l cc s thc khng m tho mn iu kin ab + bc + ca =


1.
1
1
1
5
Chng minh rng
+
+
.
a+b b+c c+a 2
Bi 19. Chng minh vi mi a,b,c l cc s thc ta c
a ( a + b) + b (b + c ) + c (c + a ) 0 .
3

Bi 20. Cho a,b,c l di 3 cnh mt tam gic. Chng minh


a
b
c
a)
+
+
<2;
b+c c+a a+b
b) a 2 + b 2 + c 2 < 2 ( ab + bc + ca ) ;
c)

a
3

b +c
3

b
3

a +c
3

c
3

b +a
3

< 23 4 .

Bi 21. Cho a,b l cc s thc dng. Chng minh

a 3 + b3 a 2 + b 2
.

2
a+b

Bi 22. Chng minh rng vi mi s thc x, y tho mn iu kin x + y 1, xy 2.


Ta c x3 + y 3 7 .
Bi 23. Cho a, b, c, d l cc s thc.

)(

Chng minh rng a 2 + b 2 c 2 + d 2 ( ac + bd ) .


2

Bi 24. Cho a,b l 2 s thc khng m. Chng minh rng ta lun c


3 2
3
1
1
2
a + b + b + a + 2a + 2b + .
4
4
2
2

31

www.TaiLieuLuyenThi.com
Khm ph t duy K thut gii bt T Bi ton Max Min ng Thnh Nam

Bi 25. Cho x, y, z [ 0;2] thoa man x + y + z =


3.
Tim gia tri ln nht cua biu thc P = x 2 + y 2 + z 2 .
Bi 26. Cho a,b,c l cc s thc dng. Chng minh
3

a)

b)

1b+c
b+c
1+
1+
.
2 a
a
a3
a3 + ( b + c )

b3

b3 + ( c + a )

c)

a
2a
.

b+c a+b+c

d)

a
b
c
+
+
2.
b+c
c+a
a+b

e)

a
a + 8bc
2

4
a3

4
+ b3

c3 + ( a + b )

1.

4
+ c3

1.
a 2 + 8bc
b 2 + 8ca
c 2 + 8ab
Bi 27. Cho a,b,c l cc s thc i mt phn bit. Chng minh

f)

4
a3

c3

a2

b2

c2

( b c )2 ( c a )2 ( a b )2

2.

Bi 28. Chng minh rng vi ba s thc a, b, c ta lun co

( a + 1)(b + 1)( c + 1) ( ab + bc + ca 1) .
2

Bi 29. Cho x,y,z l cc s thc dng tha mn x = max { x, y, z} .


Chng minh

x +1 y +1 z +1 x y z
+
+
+ + .
y +1 z +1 x +1 y z x

Bi 30. Cho x,y l hai s thc dng tho mn iu kin x + 2 y =


1.
Chng minh

1 1
25
.
+
x y 1 + 48 xy 2

Bi 31. Cho a,b l hai s thc khc 0 tho mn iu kin ab


3

1
1
Chng minh rng ab 3 + 3 .
b
a
32

1 1
+ +3.
a b

www.TaiLieuLuyenThi.com
Cty TNHH MTV DVVH Khang Vit

Bi 32. Cho a,b,c,d l cc s thc thuc on [ 0;2] . Chng minh


a + b + c + d 1 + ab + 1 + bc + 1 + ca + 1 + ad .

Bi 33. Cho a, b, c l cc s thc thuc on [ 0;2] v a + b + c =


3.
Tm gi tr ln nht v nh nht ca biu thc P =

a 2 + b2 + c2
.
ab + bc + ca

Bi 34. Cho a, b, c, d l cc s thc tha mn iu kin a 2 + b 2 + c 2 + d 2 =


1.
Tm gi tr ln nht ca biu thc
=
P a 3 ( b + c + d ) + b3 ( c + d + a ) + c 3 ( d + a + b ) + d 3 ( a + b + c ) .

Bi 35. Cho a, b, c l cc s thc thuc on [1;3] tha mn iu kin a 2 + b 2 + c 2 =


14 .
c
b
Chng minh 1 2 + 8 .
a
a
Bi 36. Cho a,b,c l cc s thc khng m. Chng minh rng

a 2 + b 2 + c 2 + ab 2 + bc 2 + ca 2 + 9 5 ( a + b + c ) .

Bi 37. Cho a, b, c l cc s thc dng. Chng minh rng

9 a3 + b3 + c3 + abc 8 ( a + b + c ) a 2 + b 2 + c 2 .

Bi 38. Cho a, b > 0 thoa man iu kin a 2 + b 2 =


1.
2

a
1 1
b
Chng minh rng + 2 2 +

.
a b
a
b

5
Bi 39. Cho a, b, c > 0 thoa man iu kin a 2 + b 2 + c 2 =.
3
1 1 1
1
.
Chng minh rng + <
a b c abc
Bi 40. Cho a, b l hai s thc tha mn a , b < 1, ta lun c
1
1 a

1
1 b

Bi 41. Vi mi s thc a, b 1 ta lun c

2
.
1 ab

1
1
ab
3
+
+
.
a + 1 b + 1 ab + 1 2

Bi 42. Cho a, b, c [ 0;1] . Chng minh rng a (1 b ) + b (1 c ) + c (1 a ) 1.


Bi 43. Cho a, b, c > 0 tha mn a b c .
1 1 1
Chng minh rng ( a b + c ) + 1 .
a b c
33

www.TaiLieuLuyenThi.com
Khm ph t duy K thut gii bt T Bi ton Max Min ng Thnh Nam

Bi 44. Cho cc s thc a, b, c tha mn iu kin a + b + c =


1.
Tm gi tr nh nht ca biu thc
P=

a 2 + ab + b 2 + b 2 + bc + c 2 + c 2 + ca + a 2 .

(a
Bi 45. Cho a,b,c,d thuc on [1;2] . Chng minh rng

)(

+ b2 c2 + d 2

( ac + bd )

) 25 .
12

Bi 46. Cho x,y,z l cc s thc. Chng minh


3 ( x y )2 3 ( y z )2 3 ( z x )2
x 2 + y 2 + z 2 xy yz zx max
,
,
.
4
4
4

Bi 47. Cho x,y,z l cc s thc khng m. Chng minh


3
2
2
2
x3 + y 3 + z 3 3 xyz ( x + y + z ) .max ( x y ) , ( y z ) + ( z x ) .
4
Bi 48. Cho a,b,c l cc s thc khng m. Chng minh
2
2
2
a+b+c 3
abc max
a b ; b c ; c a
3

{(

) (

)}

) (

D. HNG DN GII P S
Bi 1. Bt ng thc tng ng vi:
Ch ( a + b )

a+ b

6ab =

a b

( a b )2 + 2

( a + b )

a+ b

6ab 0 .

ab a + b ab 0.

Bt ng thc c chng minh. ng thc xy ra khi v ch khi a = b .


Bi 2. Ta c a b ( a + b + c ) ( 2b + c ) .
2

Vy ta chng minh ( 2b + c ) 9bc 4b 2 5bc + c 2 0 ( b c )( 4b c ) 0 .


2

Bt ng thc cui lun ng do b c 0; 4b c 3b + a c = ( a + b c ) + 2b > 0 .


Bt ng thc c chng minh. ng thc xy ra khi v ch khi a= b= c
Bi tp tng t

Cho a,b,c l di ba cnh mt tam gic ta c 4a 2b 2 > a 2 + b 2 c 2


Bi
3. Ta c 1 y 2
=

(x

( x 1)2 sin 2 a
2

Bi 4. Bt ng thc tng ng vi
34

2 x cos a + 1

0 1 y 1 .

x2 ( y z )
z

y2 ( z x)
x

z2 ( x y )
y

0.

www.TaiLieuLuyenThi.com
Cty TNHH MTV DVVH Khang Vit

x2 ( y z ) x2 ( y z )

y z
z
y

Theo gi thit ta c
.
y2 ( z x) y2 ( z x)

x y z
x
y

Suy ra
x2 ( y z )
z

y2 ( z x)
x

z2 ( x y )

x2 ( y z ) + y 2 ( z x ) + z 2 ( x y )
y

( x y )( x z )( y z ) 0
y

Ta c iu phi chng minh. ng thc xy ra khi v ch khi x= y= z .


Bi 5. Bt ng thc tng ng vi
4a 2 + 4b 2 + 4c 2 + 4d 2 + 4e2 4a ( b + c + d + e )

) (

) (

) (

a 2 4ab + 4b 2 + a 2 4ac + 4c 2 + a 2 4ad + 4d 2 + a 2 4ae + 4e2 0


( a 2b ) + ( a 2c ) + ( a 2d ) + ( a 2e ) 0.
2

Bt ng thc lun ng. Dng thc xy ra khi v ch khi =


a 2=
b 2=
c 2=
d 2e .
Bt ng th hai l trng hp ring khi e = 1
Bi 6. Ta chng minh

x3
x + xy + y
2

2x y
vi mi s thc dng x v y.
3

Tht vy bt ng thc tng ng vi: ( x + y )( x y ) 0 (lun ng).


2

p dng bt ng thc trn ta c


a3
a + ab + b
2

b3
b + bc + c
2

2a b
;
3

2b c
;
3

2c a
.
3
c + ca + a
Cng theo v ba bt ng thc trn ta c iu phi chng minh. ng thc xy
ra khi v ch khi a= b= c .
Bi tp tng t
Cho n l s nguyn dng ln hn hoc bng 2 v cc s thc x1 , x2 ,..., xn c
tch bng 1. Chng minh
c3

35

www.TaiLieuLuyenThi.com
Khm ph t duy K thut gii bt T Bi ton Max Min ng Thnh Nam
n

6
1i < j n xi

xi9 + x9j
+

xi3 x3j

x 6j

n ( n 1)
3

Bi 7. Bt ng thc cho tng ng vi :


2

3 x 2 + 2 xy + 3 y 2
2
2

2 x + 2 y
2
x
+
y
(
)

3 x 2 + 2 xy + 3 y 2

8 x2 + y 2

)( x + y)

0 ( x y) 0
4

Bt ng thc c chng minh. ng thc xy ra khi v ch khi x = y .


Bi 8. Bt ng thc v tri tng ng vi:

xy ( x y )

0.

x2 + y 2

4 xy + y 2

Du bng xy ra khi v ch khi x = y .

(x
Bt ng thc v phi tng ng vi:
ng thc xy ra khi v ch khi =
x

8 x +y
2

2 2

0.

(2 3) y .

Bi 9. Bin i tng ng bt ng thc cho v dng lun ng


a + b a + b . ng thc xy ra khi v ch khi ab 0 .
Ch . Thc cht bt ng thc xut pht t tnh ng bin trn khong
x
.
( 1; + ) ca hm s y =
x +1
Bi tp tng t
Chng minh vi mi s thc a v b ta c

2014 + a + b
a + b + 2015

2014 + a + b
a + b + 2015

1
3

2
Bi 10. Bt ng thc tng ng vi b +
+a + 2 30
2a
4a

2
2a 2 3
1

b +
0
+
2a
4a 2

Bt ng thc c chng minh. ng thc xy ra khi v ch khi

1
3
b=
,a =
4 .
2a
4
Bi 11. Bt ng thc chnh l phn rt gn ca bt ng thc sau
36

www.TaiLieuLuyenThi.com
Cty TNHH MTV DVVH Khang Vit

( a b )2 + ( b c )2 + ( c a )2 0 .

b
c
a
ng thc xy ra khi v ch khi a= b= c .
Bi 12. Bt ng thc cho chnh l phn rt gn ca bt ng thc sau

(a

b2

) + (b
2

c2

) + (c
2

a2

) + (a
2

bc

) + (b
2

ca

) + (c
2

ab

0.

ng thc xy ra khi v ch khi a= b= c .


Bi 13. Ch ng thc
a b c b c a a 2 b2 b2 c2 c2 a 2
+ + =

+
+
b c a a b c
ab
bc
ca
=
=

Ta c

) (

) (

c a 2 b2 + a b2 c2 + c a 2 b2
abc
( a c )( a b )( c b )

( a c )( a b )( c b )
abc

).

abc

<

abc
=
1.
abc

Bt ng thc c chng minh.


Bi 14. Ch nu coi v tri l mt a thc bc ba ca a th ta c 2 hai nghim
=
a b=
, a c. V vy ta phn tch c v tri di dng

R rng vi

( a b )( a c )( b c )( ab + bc + ca ) .
a < b < c ( a b )( a c )( b c )( ab + bc + ca ) < 0 .

Bt ng thc c chng minh.


Bi 15. Bt ng thc cho tng ng vi
2
2
2
a ( b c ) a 2 + b ( c a ) b 2 + c ( a + b ) c 2 > 0

( a + b c )( b + c a )( c + a b ) > 0

Bt ng thc cui lun ng v ta c pcm.


Bi 16. Gi P l biu thc v tri ta c ng thc sau
P=

( a + b + c )( a + b c )( b + c a )( c + a b ) > 0 .

Bt ng thc c chng minh.


Bi 17. Quy ng rt gn bt ng thc tng ng vi :
1 + 2 x + 2 y 2 xz 2 + 2 z 2 + z
1 z + 2 (1 z )

( x + y) 0
37

www.TaiLieuLuyenThi.com
Khm ph t duy K thut gii bt T Bi ton Max Min ng Thnh Nam

Bt ng thc cui ng do vy ta c pcm. ng thc xy ra khi v ch khi


x= y= z= 1 .
Bi 18. S dng bt ng thc a + b + c +

5abc
2 v quy ng rt gn bt ng
3

thc cn chng minh


2 ( a + b )( a + c ) + 2 ( b + c )( b + a ) + 2 ( c + a )( c + b ) 5 ( a + b )( b + c )( c + a )
2 ( a + b + c ) + 5abc + 2 5 ( a + b + c )
2

S dng 5abc 6 3 ( a + b + c ) ta c
2 ( a + b + c ) + 5abc + 2 5 ( a + b + c ) 2 ( a + b + c ) + 8 8 ( a + b + c )
2

= 2 ( a + b + c 2) 0
2

Bt ng thc c chng minh. ng thc xy ra khi v ch khi a= b= 1, c= 0


hoc cc hon v.
2a = x + y z

Bi 19. t x = a + b, y = b + c, z = c + a 2b = y + z x .
2c = z + x y

Bt ng thc tr thnh
x3 ( x + y z ) + y 3 ( y + z x ) + z 3 ( z + x y ) 0
x 4 + y 4 + z 4 + x3 y + y 3 z + z 3 x xy 3 yz 3 zx3 0

Bi 20. a) Ta c
2a
2a
a
=
<
b+c b+c+b+c a+b+c
2b
2b
b
.
=
<
c+a c+a+c+a a+b+c
2c
2c
c
=
<
a+b a+b+a+b a+b+c
Cng theo v 3 bt ng thc trn ta c iu phi chng minh.
b) Ta c
0 < a < b + c a2 < a (b + c )
0 < b < c + a b2 < b ( c + a ) .
0 < c < a + b c2 < c ( a + b )

Cng theo v ba bt ng thc trn ta c iu phi chng minh.


c) Ta c

38

www.TaiLieuLuyenThi.com
Cty TNHH MTV DVVH Khang Vit

a3 + b3 =( a + b ) 3ab ( a + b ) ( a + b )
3

3 4.

3
1
( a + b )2 .( a + b ) = ( a + b )3
4
4

c
a+b

a 3 + b3
Tng t ri cng theo v 3 bt ng thc trn ta c iu phi chng minh.
Bi 21. Bt ng thc cho tng ng vi:
3

a 3 + b3 a 2 + b 2
4

( a b ) a 2 + ab + b 2 0 .
a + b
2

Bi 22. Bt ng thc cho tng ng vi

x3 + y 3 + 1 3 xy + 3 ( xy + 2 ) 0

( x + y + 1) x 2 + y 2 + 1 x y xy + 3 ( xy + 2 ) 0

1
( x + y + 1) ( x y )2 + ( x 1)2 + ( y 1)2 + 3 ( xy + 2 ) 0
2
Bt ng thc cui lun ng theo gi thit ta c pcm.

)(

Bi 23. Ta c a 2 + b 2 c 2 + d 2 = ( ac + bd ) + ( ad bc ) ( ac + bd ) .
2

ng thc xy ra khi v ch khi ad = bc .


2

1
1
1
1

Bi 24. Ta c : a 0; b 0 a 2 + a; b 2 + b .
2
2
4
4

Suy ra :
3 2
3
1
1
1
2
a + b + b + a + a + b + a + b + = a + b +
4
4
2
2
2

1 1
1 1
1
1

= a + + b + 4 a + b + = 2a + 2b +
4
4
4
4
2
2

1
Bt ng thc c chng minh. ng thc xy ra khi v ch khi a= b=
.
2
Bi 25. Theo gi thit ta c :

x ( x 2) + y ( y 2) + z ( z 2) 0 x2 + y 2 + z 2 2 ( x + y + z ) =
6.

Bi 26. a) t
=
x
1 + x3 1 +

b+c
> 0. Ta cn chng minh
a

1 2
x x2 + 2
2

4 x3 + 1

x 4 4 x3 + 4 x 2 0 x 2 ( x 2 ) 0
2

39

www.TaiLieuLuyenThi.com
Khm ph t duy K thut gii bt T Bi ton Max Min ng Thnh Nam

Bt ng thc cui ng suy ra pcm.


ng thc xy ra khi v ch khi b + c =
2a .
b)p dng cu a) ta c
a3
a3 + ( b + c )

b3
b3 + ( c + a )

c3
c3 + ( a + b )

a2

a 2 + b2 + c2
b2

a 2 + b2 + c2
c2

a 2 + b2 + c2

Cng theo v ba bt ng thc trn ta c iu phi chng minh. ng thc xy


ra khi v ch khi a= b= c .
a) Bt ng thc tng ng vi:
a ( a + b + c ) 4a 2 ( b + c )
2

.
2
2
a ( a + b + c ) 4 a ( b + c ) 0 a ( a b c ) 0

Bt ng thc lun ng. ng thc xy ra khi v ch khi a = 0 hoc a= b + c .


b) p dng cu c) ta c
a
2a
b
2b
c
2c
.

b+c a+b+c c+a a+b+c a+b a+b+c


Cng theo v ba bt ng thc trn ta c pcm. Vi a,b,c th ng thc khng
xy ra.
c) Bt ng thc tng ng vi:

(a

4/3

+ b 4/3 + c 4/3

)(

a8/3 = b 4/3 + c 4/3 2a 4/3 + b 4/3 + c 4/3

2(bc) 2/3 .4a 2/3 (bc)1/3

a 4/3 + b 4/3 + c 4/3

a8/3 + 8bc.a 2/3


= a 2/3 a 2 + 8bc

a 4/3

4/3
4/3
4/3
a 2 + 8bc a + b + c
Bt ng thc cui ng theo AM GM.
d) p dng chng minh cu e) xy dng ba bt ng thc cng dng ri cng li
ta c iu phi chng minh.
Bi 27. Ch hng ng thc
bc
ca
ab
+
+
=
1.
( a b )( a c ) ( b c )( b a ) ( c a )( c b )

40

www.TaiLieuLuyenThi.com
Cty TNHH MTV DVVH Khang Vit

Khi

a2

b2

c2

( b c )2 ( c a )2

b
c
a
+
=
+
+

+22.
2
(a b) b c c a a b

Bi 28. Ch ng thc :

( a + 1)(b + 1)( c + 1) =
2

( a + b + c ab bc ca )2 + ( ab + bc + ca 1)2 .

Bi 29. Bt ng thc tng ng vi:


x + y +1
y + z +1
2
+ ( y z)
0 (lun ng) do
( x z )( x y )
xy ( x + 1)( y + 1)
yz ( y + 1)( z + 1)
x z 0, x y 0 .

Bt ng thc c chng minh ng thc t ti x= y= z .


t +a t +b t +c
. Lc ny
+
+
t +b t +c t +a
v tri bt ng thc thay s 1 bi mt s dng bt k bt ng thc vn ng.
Bi 30. Thay x = 1 2 y vo bt ng thc cn chng minh a v chng minh.

Nhn xt. Nu tinh ta c th kho st hm f (t ) =

2
1
1
25
+
12 y 2 7 y + 1 0 (lun ng).
2
1 2 y y 1 + 48 (1 2 y ) y

Bi ton c chng minh. ng thc xy ra ti=


x

1
1
1
hoc x= y=
.
=
,y
3
2
4

1
1
3
Bi 31. t x =
ab , y =
3 ,z =
3 xyz =
1.
a
b

Theo gi thit ta c x3 + y 3 + z 3 3 =
3 xyz

( x + y + z ) x 2 + y 2 + z 2 xy yz zx 0 .

Do x 2 + y 2 + z 2 xy yz zx=

1
1
1
( x y )2 + ( y z )2 + ( z x )2 0 .
2
2
2
3

1
1
Do x + y + z 0 x y z ab 3 + 3 .
b
a

Bt ng thc c chng minh.

Bi 32. Xut pht t a b 2, a, b [ 0;2] ta c:

( a b )2 4 a 2 + b2 4 + 2ab .
( a + b ) 4 + 2ab + 2ab= 4 (1 + ab ) a + b 2 1 + ab .
2

Tng t ta c: b + c 2 1 + bc ; c + d 2 1 + cd ; d + a 2 1 + da .
Cng theo v 4 bt ng thc trn ta c ngay iu phi chng minh.
41

www.TaiLieuLuyenThi.com
Khm ph t duy K thut gii bt T Bi ton Max Min ng Thnh Nam

+ bc + ca )
( a + b + c )2 2 ( ab=

9
2.
ab + bc + ca
ab + bc + ca
1
2
S dng bt ng thc c bn ta c: ab + bc + ca ( a + b + c ) =
3 suy ra
3
9
P 2=
1.
3
Vy gi tr nh nht ca P bng 1 t ti a= b= c= 1 .
tm gi tr ln nht ca P ta tm gi tr nh nht ca ab + bc + ca .

Bi 33.
Ta c: P
=

V a, b, c [ 0;2] nn

( a 2 )( b 2 )( c 2 ) 0 abc 2 ( ab + bc + ca ) + 4 ( a + b + c ) 8 0 .
abc + 4 ( a + b + c ) 8 4 + abc
9
5
.
ab + bc + ca
=
2 P =
2
2

5
t ti=
a 2,=
b 1,=
c 0 v cc hon v.
2
Bi 34. Vit li biu thc P di dng:

Vy gi tr ln nht ca P bng

) (

) (

P= ab a 2 + b 2 + bc b 2 + c 2 + cd c 2 + d 2 + da d 2 + a 2 + ac a 2 + c 2 + bd b 2 + d 2 .
:

( x y )4 = x 4 + y 4 + 6 x 2 y 2 4 xy ( x 2 + y 2 ) xy ( x 2 + y 2 ) =

x4 + y 4 + 6 x2 y 2 ( x y )

p dng vo bi ton suy ra

3 a 2 + b2 + c2 + d 2
P

3 a +b +c +d
2

( a b )4 + ( b c )4 + ( c d )4 + ( d a )4 + ( a c )4 + ( b d )4
4

2 2

3
=
4
4
3
1
.
Vy gi tr ln nht ca P bng t ti a= b= c= d=
4
2
Tng qut. Cho n s thc khng m x1 , x2 ,..., xn tha mn mn x1 + x2 + ... + xn =
1.

xi2 x 2j .
j =1
1i j n

Bi 35. t v tri bt ng thc l P. Khi

Tm gi tr ln nht ca biu thc P =

42

www.TaiLieuLuyenThi.com
Cty TNHH MTV DVVH Khang Vit

=
P

ac 2ab bc
=
+2
a2

( a b + c )2 b

(a

+ b 2 + c 2 + 2ac 2ab 2bc 2ab 14


2a 2

+2

b 7
+2 2 +2
a a
a a
2a
b
7
Mt khc a, b, c [1;3] nn 3, 2 7 P 3 7 + 2 = 8 .
a
a
Bt ng thc c chng minh. ng thc xy ra khi v ch khi a = 1, b = 3,
c = 2.
Bi 36. Nhn xt. D on du bng ca bt ng thc t ti a= b= c= 1 nn ta
t n ph a = x + 1, b = y + 1, c = z + 1, ( x, y, z 1) a v chng minh bt ng thc:

( x + 1)2 + ( y + 1)2 + ( z + 1)2 + ( x + 1)( y + 1)2 + ( y + 1)( z + 1)2 + ( z + 1)( x + 1)2 + 9


5 ( x + y + z + 3) .
Khai trin v rt gn ta c:

2 x 2 + y 2 + z 2 + 2 ( xy + yz + zx ) + xy 2 + yz 2 + zx 2 0 .
( x + y + z ) + x 2 ( z + 1) + y 2 ( x + 1) + z 2 ( y + 1) 0 (lun ng).
2

Bi ton c chng minh.


ng thc xy ra khi v ch khi x = y = z = 0 a = b = c = 1 .
Bi 37. Bt ng thc cho tng ng vi:

5 a3 + b3 + c3 + 9abc 4 ab ( a + b ) + bc ( b + c ) + ca ( c + a ) .

Theo bt ng thc AM-GM ta c: a3 + b3 + c3 3abc .

Suy ra 5 a3 + b3 + c3 + 9abc 4 a3 + b3 + c3 + 3abc .


Ta i chng minh: a3 + b3 + c3 + 3abc ab ( a + b ) + bc ( c + a ) + ca ( a + b ) .
a ( a b )( a c ) + b ( b c )( b a ) + c ( c a )( c b ) 0 .

Khng mt tnh tng qut ta gi s a b c khi : c ( c a )( c b ) 0 v


a ( a b )( a c ) + b ( b c )( b a ) = ( a b ) a ( a c ) b ( b c )
2
= ( a b ) a 2 b2 + c ( b a ) = ( a b ) ( a + b c ) 0

Suy ra a ( a b )( a c ) + b ( b c )( b a ) + c ( c a )( c b ) 0 .
Bt ng thc c chng minh. ng thc xy ra khi v ch khi a= b= c .
Bi 38. Bt ng thc cn chng minh tng ng vi
43

www.TaiLieuLuyenThi.com
Khm ph t duy K thut gii bt T Bi ton Max Min ng Thnh Nam

1 1
a b
+ 2 2 + + 2 a + b + 2ab 2 2ab + 1
a b
b a
( a + b )2 1
2
+1
a + b + (a + b) 1 2 2
2

1 2 t 2 + t + 2 2 0 (*) ; vi t = a + b 1; 2

(Vi ( a + b ) = a 2 + b 2 + 2ab > a 2 + b 2 =1 a + b > 1


2

Va ( a + b ) 2 a 2 + b 2 = 2 a + b 2
2

Suy ra t 1; 2 ). Bt ng thc (*) lun ung vi t 1; 2 .


Bi 39. Do a, b, c > 0 nn bt ng thc tng ng vi : bc + ca ab < 1
Ta c : ( a + b c ) 0 bc + ca ab
2

Bi ton c chng minh.


1
1
Bi 40. Ta c:
+

2
1 a
1 b2

1 2
5
a + b 2 + c 2 = < 1 (lun ung).
2
6

(1 a )(1 b )
2

Ta chng minh

(1 a )(1 b ) 1 ab 1 a
2

b 2 + a 2b 2 1 2ab + a 2b 2 ( a b ) 0

(lun ng).

Bi 41. Quy ng rt gn a v bt ng thc: ( ab 1)( a 1)( b 1) 0 (lun


ng do a, b 1 ).
Bi 42. Bt ng thc cn chng minh tng ng vi :
a + b + c ( ab + bc + ca ) 1

Xut pht t gi thit ta c :

(1 a )(1 b )(1 c ) 0 1 ( a + b + c ) + ( ab + bc + ca ) abc 0


( a + b + c ) ( ab + bc + ca ) 1 abc 1
ng thc xay ra khi va chi khi a= b= 0, c= 1 hoc cac hoan vi.
Bi 43. Bt ng thc cn chng minh tng ng vi
1 1 1
1
1 1 1
1
+
+ +
a b c a b+c
a c b a b+c
a+c
a+c
1
1

ac
b(a b + c)
ac b ( a b + c )
44

www.TaiLieuLuyenThi.com
Cty TNHH MTV DVVH Khang Vit

b ( a b + c ) ac a ( b c ) b ( b c ) 0 ( a b )( b c ) 0

Bt ng thc cui lun ng do a b c . Ta c pcm.


3
1
3
2
2
2
Bi 44. Ta c a 2 + ab + b 2 = ( a + b ) + ( a b ) ( a + b ) .
4
4
4
a 2 + ab + b 2

3
3
a+b
(a + b) .
2
2

Tng t ta c : b 2 + bc + c 2

3
3
b+c
(b + c ) ;
2
2

c 2 + ac + a 2

3
3
c+a
(c + a) .
2
2

Cng theo v ba bt ng thc trn ta c : P 3 ( a + b + c ) =3 .


ng thc xy ra khi v ch khi a= b= c=
Bi 46. Gi s

3( x y )
4

1
.
3

3 ( x y )2 3 ( y z )2 3 ( z x )2
,
,
= max
.
4
4
4

Ta chng minh x + y + z xy yz zx
2

3( x y )

4 x 2 + 4 y 2 + 4 z 2 4 xy 4 yz 4 zx 3x 2 + 3 y 2 6 xy .

x 2 + y 2 + 4 z 2 + 2 xy 4 yz 4 zx 0 ( x + y 2 z ) 0 (lun ng).
2

Bi ton c chng minh.

Bi 47. Ta c: x3 + y 3 + z 3 3 xyz = ( x + y + z ) x 2 + y 2 + z 2 xy yz zx .
Bi ton a v chng minh
3 ( x y )2 3 ( y z )2 3 ( z x )2
x 2 + y 2 + z 2 xy yz zx max
,
,
.
4
4
4

y chnh l kt qu bi ton trn v ta c iu phi chng minh.

CH 2: K THUT CHNG MINH PHN CHNG


A. NI DUNG PHNG PHP
Gi s cn chng minh bt ng thc no ng, ta gi s bt ng thc
l sai v kt hp vi iu kin gi thit ch ra iu v l. iu v l c th l tri
vi gi thit hoc tri vi mt iu ng. T suy ra bt ng thc cn chng
minh l ng.
45

www.TaiLieuLuyenThi.com
Khm ph t duy K thut gii bt T Bi ton Max Min ng Thnh Nam

B. BI TP MU
Bi 1. Chng minh rng t nht mt trong cc bt ng thc sau l ng
a 2 + b 2 2bc; b 2 + c 2 2ca; c 2 + a 2 2ab .

Li gii
a 2 + b 2 < 2bc

Gi s tt c cc bt ng thc trn u sai khi b 2 + c 2 < 2ca .


2
2
c + a < 2ab

Cng theo v 3 bt ng thc trn ta c 2 a 2 + b 2 + c 2 < 2 ( ab + bc + ca )

) (

) (

a 2 2ab + b 2 + b 2 2bc + c 2 + c 2 2ca + a 2 < 0


( a b ) + ( b c ) + ( c a ) < 0 (mu thun).
2

V vy iu phn chng l sai nn khng nh bi ng (pcm).

Bi 2. Cho a,b,c l cc s thc thuc khong ( 0;1) . Chng minh t nht mt trong
1
1
1
cc bt ng thc sau l ng a (1 b ) ; b (1 c ) ; c (1 a ) .
4
4
4
Li gii
1

a (1 b ) > 4

Gi s tt c cc bt ng thc trn u sai khi b (1 c ) > .


4

c (1 a ) > 4

Nhn theo v ba bt ng thc trn ta c a (1 a ) b (1 b ) c (1 c ) >


2

1
a +1 a
Ch . 0 < a (1 a )
=.
2
4

1
b +1 b
0 < b (1 b )
=
2
4

1
c +1 c
0 < c (1 c )
=
2
4

Nhn theo v ba bt ng thc trn ta c


1
a (1 a ) b (1 b ) c (1 c ) , dn ti mu thun vi (1).
64
46

1
(1).
64

www.TaiLieuLuyenThi.com
Cty TNHH MTV DVVH Khang Vit

Vy iu phn chng l sai do t nht mt trong cc bt ng thc cho l


ng (pcm).
Bi tp tng t
Cho a,b,c l cc s thc thuc khong ( 0;1) . Chng minh t nht mt trong cc

bt ng thc sau l sai


1
1
1
a (1 b ) > ; b (1 c ) > ; c (1 a ) > .
4
4
4

Bi 3. (HOMC 2007) Cho p = abcd l mt s nguyn t c bn ch s.


Chng minh rng phng trnh ax3 + bx 2 + cx + d =
0 khng c nghim hu t.
Li gii
Gi s phng trnh c nghim hu t th nghim ny phi m gi s nghim
p
l x0 =
, p , q * , ( p , q ) =
1. Khi
q
a

p3
q

+b

p2
q

a q
p
.
+ d = 0 ap3 + bp 2 a cpq 2 + dq3 = 0
q
d p

Do p,q l cc s t nhin c mt ch s v v p,q l nghim ca phng trnh

( qx + p ) ( ex 2 + fx + g ) , e, f , g * .

nn f ( x) = ax3 + bx 2 + cx + d =
Ta c abcd = f (10)=

(10q + p )(100e + 10 f

+ g )= qp.efg tri vi gi thit

abcd l mt s nguyn t.
Vy iu phn chng l sai ta c iu phi chng minh.
Bi tp tng t

(HOMC 2007) Cho p = abc l mt s nguyn t c ba ch s.


Chng minh rng phng trnh ax 2 + bx + c =
0 khng c nghim hu t.
Bi 4. Gi s f1 ( x) =x 2 + a1 x + b1; f 2 ( x) =x 2 + a2 x + b2 l hai tam thc bc hai vi
h s nguyn c nghim chung l a. Chng minh rng nu a khng l s nguyn th
tam thc bc hai sau lun c nghim thc f ( x) = x 2 + ( a1 + a2 ) x + b1 + b2 .
Li gii
p
Ta chng minh a khng th l s hu t tht vy gi s a = , p, q , ( p, q ) =1 .
q

Do a l nghim ca
f1 ( x) a 2 + a1a + b1 =0

p2
q

+ a1

p
+ b1 =0 p 2 + a1 pq + b1q 2 =0 .
q

47

www.TaiLieuLuyenThi.com
Khm ph t duy K thut gii bt T Bi ton Max Min ng Thnh Nam

Suy ra p 2 q p q ( p, q ) = q =1 a tri vi gi thit a khng l s nguyn.


V vy a khng l s hu t.
Do a l nghim chung ca
2
0
a + a1a + b1 =
f1 ( x), f 2 ( x)
( a1 a2 ) a =b2 b1 (1) .
2
0
a + a2 a + b2 =
Do a khng l s hu t nn (1) a=
1 a2 ; b=
1 b2 .

Khi f1 ( x) =x 2 + a1 x + b1; f ( x) =x 2 + 2a1 x + 2b1 .


Do f1 ( x) c nghim nn a12 4b1 0 khi f(x) c

1 2
1
a1 4b1 + a12 0 .
2
2
iu chng f(x) c nghim ta c pcm.
=' a12 2b1=

Bi 5. Cho a,b,c l cc s thc dng tho mn iu kin


Chng minh rng

1 1 1
+ + =
3.
a b c

a+b + b+c + c+a 3 2 .


Li gii

a = x 2 + z 2 y 2

a+b
b+c
c+a
,y=
,z =
t x =
b = x 2 + y 2 z 2 .
2
2
2

2
2
2
c = z + y x
Ta cn chng minh x + y + z 3 vi iu kin
1
x +y z
2

1
y +z x
2

1
z + x2 y 2
2

=
3.

Tht vy gi s x + y + z < 0. Khi s dng bt ng thc AM GM cho ba


s dng ta c
1
1
1
3
.
+ 2 2 2+ 2 2 2
2
2
2
x +y z
y +z x
z +x y
3 x2 + y 2 z 2 y 2 + z 2 x2 z 2 + x2 y 2

)(

)(

Mt khc

)(

)(

x+ y+z
z 2 + x2 y 2 x2 y 2 z 2
< 1.
3

1
1
1
Suy ra 2
+ 2
+ 2
> 3 mu thun vi gi thit.
2
2
2
2
x +y z
y +z x
z + x2 y 2
x2 + y 2 z 2

y 2 + z 2 x2

Vy iu phn chng l sai v ta c iu phi chng minh.


48

www.TaiLieuLuyenThi.com
Cty TNHH MTV DVVH Khang Vit

Ta cng xt bi ton quen thuc sau trch t thi IMO 2001


Bi 6.( IMO 2001) Cho a,b,c l cc s thc dng. Chng minh rng:
a
b
c
+
+
1.
a 2 + 8bc
b 2 + 8ac
c 2 + 8ba
Li gii
a
b
c
t x =
=
;y =
;z
,( x; y; z (0;1))
a 2 + 8bc
b 2 + 8ac
c 2 + 8ba
a2
x2
b2
y2
c2
z2
.
=
rng:
=
;
=
;
8bc 1 x 2 8ac 1 y 2 8ba 1 z 2

Suy ra

1
x2
y2
z2
=(
)(
)(
).
512 1 x 2 1 y 2 1 z 2

Ta cn chng minh x + y + z 1 vi x, y, z (0;1) v


(1 x 2 )(1 y 2 )(1 z 2 ) =
512( xyz )2 (1).

Gi s ngc li x + y + z < 1 .
Theo bt ng thc AM GM ta c:
(1 x 2 )(1 y 2 )(1 z 2 ) > [( x + y + z )2 x 2 ][( x + y + z )2 y 2 ][( x + y + z )2 z 2 ]
.
= ( x + x + y + z )( y + z )( x + y + z + y )( z + x)( z + z + x + y )( x + y )
1

512( xyz ) 2
4( x 2 yz ) 4 .2( yz ) 2 .4( y 2 zx) 4 .2( xz ) 2 .4( z 2 xy ) 4 .2( xy ) 2 =

iu ny mu thun vi (1).
Vy iu phn chng l sai v ta c iu phi chng minh.
Bi tp tng t
Cho a,b,c l cc s thc dng c tch bng 1. Chng minh
1
1
1
+
+
1.
5a + 1
5b + 1
5c + 1
Bi 7. Cho a,b,c l cc s thc dng tha iu kin a + b + c + abc =
4.
Chng minh rng: a + b + c ab + bc + ca .
Li gii
Gi s ngc li c a + b + c < ab + bc + ac .
S dng bt ng thc Schur bc 3 ta c(Xem chng 4).
9abc
4(ab + bc + ac) (a + b + c) 2
a+b+c
> (a + b + c)[4 (a + b + =
c)] abc(a + b + c)
a +b+c <3.
Khi abc < 1 4= abc + a + b + c < 4 . Mu thun.
49

www.TaiLieuLuyenThi.com
Khm ph t duy K thut gii bt T Bi ton Max Min ng Thnh Nam

Suy ra bt ng thc u ng.


T bt ng thc ny ta chng minh c mt bt ng thc kh sau
Cho a,b,c l cc s thc dng v k s thc tho mn iu kin
3

a
b
c
1

4k 2 + 2k 1 0 ta lun c k +
k +
k +
k + .
b + c
c + a
a+b
2

C. BI TP RN LUYN
Bi 1. Chng minh rng nu a + b =
2cd th t nht mt trong cc bt ng thc
sau l ng c 2 a; d 2 b .
Bi 2. Chng minh rng t nht mt trong ba bt ng thc sau l ng
a 2 + b2

( b + c )2 ; b 2 + c 2 ( c + a )2 ; c 2 + a 2 ( a + b )2 .
2

Bi 3. Cho a,b,c l cc s thc thuc khong ( 0;2 ) . Chng minh rng t nht mt
trong cc bt ng thc sau l sai a ( 2 b ) > 1; b ( 2 c ) > 1; c ( 2 a ) > 1 .
Bi 4. Cho a,b,c l cc s thc tho mn a + b + c > 0; ab + bc + ca > 0; abc > 0 .
Chng minh rng a > 0, b > 0, c > 0 .
Bi 5. Chng minh rng nu a1a2 2 ( b1 + b2 ) th t nht mt trong hai phng
trnh sau c nghim x 2 + a1 x + b1= 0; x 2 + a2 x + b2= 0 .
Bi 6. Cho abc 0. Chng minh t nht mt trong ba phng trnh sau c nghim
ax 2 + 2bx +=
c 0; bx 2 + 2cx +=
a 0; cx 2 + 2ax +=
b 0.

Bi 7. Cho f ( x) = x 2 + ax + b . Chng minh rng vi mi gi tr ca a v b th t


nht mt trong ba s
1
f (0) , f (1) , f (1) ln hn hoc bng .
2
Bi 8. Cho a,b,c l cc s thc tho mn iu kin
1. a + b + c =2; a 2 + b 2 + c 2 =2 .
4
2. Chng minh rng c ba s a,b,c u thuc on ;0 .
3

Bi 9. Cho a,b,c l cc s thc tho mn iu kin a 2 + b 2 + ab + bc + ca < 0 .


Chng minh rng a 2 + b 2 < c 2 .
Bi 10. Cho a,b,c l cc s thc i mt khc nhau chng minh rng t nht mt
trong ba bt ng thc sau l sai
9ab ( a + b + c ) ;9bc ( a + b + c ) ;9ca ( a + b + c ) .
2

50

www.TaiLieuLuyenThi.com
Cty TNHH MTV DVVH Khang Vit

Bi 11. Cho a,b,c,d l cc s thc dng chng minh t nht mt trong cc bt


ng thc sau l sai a + b < c + d ; ( a + b )( c + d ) < ab + cd ; ( a + b ) cd < ( c + d ) ab .
Bi 12. Cho a,b,c l cc s thc dng c tch bng 1. Chng minh rng:
1
1
1
+
+
1
1 + 8a
1 + 8b
1 + 8c
Bi 13. Cho a,b,c,d l cc s thc chng minh. Chng minh rng:
1
min{a b 2 ; b c 2 ; c d 2 ; d a 2 }
4
Bi 14. (USAMO 2000 ) Cho a,b,c l cc s thc dng.
Chng minh rng:
a+b+c 3
abc max ( a b ) 2 ;( b c ) 2 ;( c a ) 2
3
Bi 15. Cho a,b,c l cc s thc dng v k s thc tho mn iu kin

a
b
c
1

4k + 2k 1 0 ta lun c k +
k +
k +
k + .
b + c
c + a
a+b
2

D. HNG DN GII P S
Bi 1. Gi s c hai bt ng thc u sai khi
c 2 < a;
d2 < b
Cng theo v hai bt ng thc trn ta c
c 2 + d 2 < a + b= 2cd ( c d ) < 0 v l.
2

Vy iu phn chng l sai ta c iu phi chng minh.


Bi tp tng t
Cho a,b,c,d l cc s thc tho mn iu kin ac 2 ( b + d ) . Chng minh rng
t nht mt trong cc bt ng thc sau l sai a 2 < 4b; c 2 < 4d .
Bi 2. Gi s tt c cc bt ng thc trn u sai khi
a +b
2

b +c
2

2
b + c)
(
<
;

2
c + a)
(
<
;

c2 + a2 <

( a + b )2 .

2
Cng theo v ba bt ng thc trn ta c
51

www.TaiLieuLuyenThi.com
Khm ph t duy K thut gii bt T Bi ton Max Min ng Thnh Nam

2 a +b +c
2

2
2
2
a + b) + (b + c ) + (c + a )
(
<

2 a +b +c
2

) < 2 ( ab + bc + ca )

(v l).

( a b) + (b c ) + (c a ) < 0
2

iu phn chng l sai do ta c iu phi chng minh.


Bi 3. Gi s tt c cc bt ng thc trn l ng khi
a ( 2 b ) > 1; b ( 2 c ) > 1; c ( 2 a ) > 1 .

Nhn theo v ba bt ng thc trn ta c a ( 2 a ) b ( 2 b ) c ( 2 c ) > 1 (1).


Ch .
0 < a ( 2 a ) = ( a 1) + 1 1
2

0 < b ( 2 b ) = ( b 1) + 1 1 .
2

0 < c ( 2 c ) = ( c 1) + 1 1
2

Nhn theo v ba bt ng thc trn ta c a ( 2 a ) b ( 2 b ) c ( 2 c ) 1 mu


thun vi (1).
Vy iu phn chng l sai(pcm).
Bi 4. Gi s tn ti mt s nh hn hoc bng 0 gi s l a khi do abc > 0 nn
a < 0; bc < 0 .
Khi ab + bc + ca= a ( b + c ) + bc > 0 a ( b + c ) > bc > 0 b + c < 0 .
Suy ra a + b + c < 0 mu thun vi gi thit. Vy iu phn chng l sai(pcm).
a 2 4b1 < 0
Bi 5. Gi s c hai phng trnh u v nghim khi 1
.
2
a2 4b2 < 0

Cng theo v hai bt ng thc trn ta c 4 ( b1 + b2 ) > a12 + a22 .


Mt khc a12 + a22 2a1a2 4 ( b1 + b2 ) .
Suy ra 4 ( b1 + b2 ) > 4 ( b1 + b2 ) v l.
Vy iu phn chng l sai ta c iu phi chng minh.
Bi 6. Gi s c ba phng trnh u v nghim khi
'1 = b 2 ac < 0

2
'2 = c ab < 0 .

2
'3 = a bc < 0

Cng theo v ba bt ng thc trn ta c a 2 + b 2 + c 2 ab bc ca < 0


52

www.TaiLieuLuyenThi.com
Cty TNHH MTV DVVH Khang Vit

1
1
1
( a b )2 + ( b c )2 + ( c a )2 < 0 (v l).
2
2
2
Vy iu phn chng l sai ta c pcm.

Bi 7. Gi s khng c s no trong ba s f (0) , f (1) , f (1) ln hn hoc bng


1
1
1

< b < (1)


f (0) = b < 2

2
2

1
1
1

3
khi f (1) = a + b + 1 <
< a + b < (2) .
2
2
2

2
1
1

3
f (1) = a + b + 1 < 2
2 < a + b < 2 (3)

Cng theo v ca (2) v (3) ta c


1
3 < 2b < 1 b < mu thun vi (1).
2
Vy iu phn chng l sai ta c pcm.
4
Bi 8. Gi s tn ti mt s khng thuc on ;0 ta c th gi s l a khi
3

b + c =2 a
b + c =2 a
b + c =2 a
.

2
2
2
2 a2
( a + 1)2
( b + c ) 2bc =
b + c =2 a
bc =
4
2
2
2
Ch . ( b + c ) 4bc ( 2 a ) 4 ( a + 1) 3a 2 + 4a 0 a 0 mu thun.
3
Vy iu phn chng l sai ta c iu phi chng minh.

Bi 9. Gi s ngc li a 2 + b 2 c 2 khi

2 a 2 + b 2 + ab + bc + ca a 2 + b 2 + c 2 + 2 ( ab + bc + ca ) = ( a + b + c ) 0
2

(mu thun vi gi thit).


Vy iu phn chng l sai ta c pcm.
Bi 10. Gi s tt c cc bt ng thc trn l ng cng theo v ba bt ng thc
ta c 9 ( ab + bc + ca ) 3 ( a + b + c )

( a + b + c ) 3 ( ab + bc + ca ) 0
2

1
1
1
( a b )2 + ( b c )2 + ( c a )2 0 a = b = c
2
2
2
iu ny mu thun vi gi thit ba s a,b,c i mt phn bit.
Vy iu phn chng l sai v bi ton c chng minh.
Bi 11. Gi s tt c cc bt ng thc l ng khi

53

www.TaiLieuLuyenThi.com
Khm ph t duy K thut gii bt T Bi ton Max Min ng Thnh Nam

a + b < c + d (1)

( a + b )( c + d ) < ab + cd (2) .
( a + b ) cd < ( c + d ) ab (3)
Nhn theo v ca (1) v (2) ta c

( a + b )2 ( c + d ) < ( c + d )( ab + cd ) ( a + b )2 < ab + cd
.
2
cd > a 2 + ab + b 2 = ( a b ) + 3ab 3ab cd > 3ab (4)
Nhn theo v ca (2) v (3) ta c

( a + b )2 cd ( c + d ) < ( c + d ) ab ( ab + cd )
2
( a + b ) cd < ( ab + cd ) ab ab ( ab + cd ) > 4abcd .
ab + cd > 4cd cd > 3ab (5)

T (4) v (5) ta c iu mu thun.


Vy phn chng l sai v ta c pcm.
1
1
Bi 12.
t x =
=
;y =
;z
1 + 8a
1 + 8b
D thy 0 < x, y, z <=
1 v a

1
1 + 8c

1 x2
1 y2
1 z2
.
=
;b =
;
c
8x
8 y2
8z 2

Do abc=1 nn ta c 83 x 2 y 2 z 2 =
(1 x 2 )(1 y 2 )(1 z 2 ) .
Theo bi ta cn chng minh x + y + z 1 .
Gi s ngc li x + y + z < 1 .
Ta c:
1 x 2 > ( x + y + z )2 x 2 =( z + y )[( x + y ) + ( x + z )] 2( y + z ) ( x + y )( x + z > 0 .

Thit lp tng t vi hai biu thc cn li ta c


1 y 2 > 2( x + z ) ( y + z )( y + x) > 0
1 z 2 > 2( y + x) ( x + z )( z + y ) > 0

Nhn li theo v ba bt ng thc trn ta c:


83 x 2 y 2 z 2 =(1 x 2 )(1 y 2 )(1 z 2 ) > 8( x + z )2 ( y + x)2 ( z + y )2
.
8 xyz > ( x + y )( z + x)( y + z )

iu ny mu thun do theo bt ng thc AM GM ta c


( x + y )( z + x)( y + z ) 8 xyz .
Do ta c iu phi chng minh.
Bi 13. Gi s min{a b 2 ; b c 2 ; c d 2 ; d a 2 } >
54

1
.
4

www.TaiLieuLuyenThi.com
Cty TNHH MTV DVVH Khang Vit

Suy ra a + b + c + d (a 2 + b 2 + c 2 + d 2 ) > 1 .
Mt khc:
(a + b + c + d )2
(a + b + c + d ) + 1 1
4
.
a+b+c+d
2
= (
1) 1 > 1
2

a 2 + b 2 + c 2 + d 2 (a + b + c + d ) >

Suy ra a + b + c + d (a 2 + b 2 + c 2 + d 2 ) < 1 nn iu gi s l sai .


Suy ra iu cn chng minh.
Bi 14. Gi s ngc li
a+b+c 3
abc > max ( a b ) 2 ;( b c ) 2 ;( c a ) 2
3

Suy ra a + b + c 3 3 abc > 2(a + b + c ab bc ca ) .


2 ab + 2 bc + 2 ca > a + b + c + 3 3 abc

i bin li x3 + y 3 + z 3 + 3 xyz < 2 ( xy )3 + 2 ( yz )3 + 2 ( zx)3 .


Nhng theo bt ng thc Schur ta c:
x3 + y 3 + z 3 + 3 xyz xy ( x + y ) + yz ( y + z ) + zx( z + x)
2 ( xy )3 + 2 ( yz )3 + 2 ( zx)3

Suy ra mu thun, vy gi s sai, suy ra iu cn chng minh.


2a
2b
2c
Bi 15. t x=
, y=
,=
z
xy + yz + zx + xyz= 4 .
b+c
c+a
a+b
Ta c xyz 1; x + y + z xy + yz + zx .
Bt ng thc tr thnh: ( 2k + x )( 2k + y )( 2k + z ) ( 2k + 1)

4k 2 ( x + y + z ) + 2k ( xy + yz + zx ) + xyz 12k 2 + 6k + 1 .

Ta c 4k 2 ( x + y + z ) + 2k ( xy + yz + zx ) + xyz
4k 2 ( xy + yz + zx ) + 2k ( xy + yz + zx ) + xyz
=

( 4k

= 16k 2

) ( xy + yz + zx ) + xyz= ( 4k
+ 8k xyz ( 4k + 2k 1)

+ 2k

+ 2k ( 4 xyz ) + xyz

16k 2 + 8k 4k 2 2k + 1= 12k 2 + 6k + 1
Bt ng thc c chng minh.
ng thc xy ra khi v ch khi a= b= c
55

www.TaiLieuLuyenThi.com
Khm ph t duy K thut gii bt T Bi ton Max Min ng Thnh Nam

CH 3: K THUT QUY NP TON HC


A. NI DUNG PHNG PHP
Gi s ta cn chng minh bt ng thc A(n) 0 vi A(n) l mt biu thc c
cha s nguyn dng n vi n n0 .
Ta thc hin chng minh bng quy np nh sau
Chng minh bt ng thc ng vi gi tr u tin ca n l n0 .
Gi s bt ng thc ng vi n= k > n0 tc A(k ) 0 .
Sau chng minh A(k + 1) 0 da vo A(k ) 0 .
Kt lun bt ng thc ng.
B. BI TP MU
V d 1. Cho n l s nguyn dng n 5. Chng minh rng 2n > n 2 .
Li gii
+ Vi n = 5;25 = 32 > 52 = 25 . Vy bt ng thc ng vi n = 5 .
+ Gi s bt ng thc ng vi n= k > 5 tc 2k > k 2 .
Khi
2k +1 = 2.2k > 2k 2 =

( k + 1)2 + k 2 2k 1 = ( k + 1)2 + k ( k 5) + 3k 1 > ( k + 1)2 .

Vy bt ng thc ng vi n= k + 1 .
Theo nguyn l quy np ta c iu phi chng minh.
Bi tp tng t
Cho n l s nguyn dng ln hn 1. Chng minh n n > ( n + 1)

n 1

Bi 2. Cho a,b l cc s thc khng m v n l s nguyn dng. Chng minh rng


n

a n + bn a + b

.
2
2
Li gii
+ Vi n = 1 bt ng thc tr thnh ng thc.
k

a k + bk a + b
+ Gi s bt ng thc ng vi n= k > 1 tc

.
2
2

+ Ta cn chng minh

a k +1 + b k +1 a + b

2
2

k +1

Ch
k +1

a + b a k + bk
a+b
a + b a + b
.
.
=

2
2
2
2 2
56

www.TaiLieuLuyenThi.com
Cty TNHH MTV DVVH Khang Vit

a k +1 + b k +1 a + b a k + b k
.

a k +1 + b k +1 ab k + ba k
2
2
2

Vy ta chng minh

a a k bk + bk ( b a ) 0 ( a b ) a k bk 0

Bt ng thc cui lun ng.


Vy theo nguyn l quy np ta c iu phi chng. ng thc xy ra khi v ch
khi a = b .
Tng qut vi n s thc khng m v m l mt s nguyn dng ta c
m

a1m + a2m + ... + anm a1 + a2 + ... + an

.
n
n

Bi 3. Cho s nguyn dng M > 3. Gi s x1 , x2 ,..., x2014 l cc s nguyn dng

sao cho x1.x2 .....x2014 = M . Tm gi tr ln nht ca biu thc:


3
.
S = x13 + x23 + ... + x2014

Li gii
Ta chng minh vi n l s nguyn dng x1 , x2 ,..., xn tho mn iu kin
x1 x2 ...xn = M th x13 + x23 + ... + xn3 ( x1 x2 ...xn ) + n 1 .
3

V bi ton chnh l trng hp ring khi n bng 2014.


+ Vi n bng 1 bt ng thc chnh l hng ng thc.

)(

+ Vi n bng 2 ta c x13 1 x23 1 0 x13 + x23 1 + x13 x23 .


Vy bt ng thc ng vi n bng 1,2.
+ Gi s bt ng thc ng vi n= k > 2 tc
x13 + x23 + ... + xk3 ( x1 x2 ...xk ) + k 1 (1).
3

Ta cn chng minh x13 + x23 + ... + xk3+1 ( x1 x2 ...xk +1 ) + k .


3

Do (1) nn ta ch cn chng minh xk3+1 + ( x1 x2 ...xk ) 1 + ( x1 x2 ...xk xk +1 )


3

) (( x x ...x ) 1) 0

xk3+1 1

1 2

Bt ng thc cui ng.


Theo nguyn l quy np ta c iu phi chng minh.
Vy gi tr ln nht ca S bng M 3 + 2013 t ti mt s bng M v 2013 s
bng 1.
Bi 4. Cho n l s nguyn dng xk 1, k =
1, n. Chng minh rng
1
1
1
n
.
+
+ ... +

n
1 + x1 1 + x2
1 + xn 1 + x1 x2 ...xn
57

www.TaiLieuLuyenThi.com
Khm ph t duy K thut gii bt T Bi ton Max Min ng Thnh Nam

Li gii
+ Vi n = 1 bt ng thc tr thnh ng thc.
+ Gi s bt ng thc ng vi n= k > 1, tc
k
1
1
1
(1).
+
+ ... +

k
1 + x1 1 + x2
1 + xk 1 + x1 x2 ...xk

+ Ta cn chng minh
1
1
1
1
k +1
.
+
+ ... +
+

1 + x1 1 + x2
1 + xk 1 + xk +1 1 + k +1 x1 x2 ...xk +1
Do (1) nn ta ch cn chng minh
k
k +1
1
(2).
+

1 + k x1 x2 ...xk 1 + xk +1 1 + k +1 x1 x2 ...xk +1
Ch

k 1
1
+

1 + xk +1 1 + k +1 x1 x2 ...xk +1
k
1+

2k

k 1

( x1 x2 ...xk ) k +1 xkk++11

k
1+

k 1 2 k
x1 x2 ...xk k +1 xkk++11

k
1 + k x1 x2 ...xk

2k
1 + k +1 x1 x2 ...xk +1

Cng theo v hai bt ng thc trn ta c (2). Tc bt ng thc ng vi


n= k + 1 .
Vy theo nguyn l quy np ta c iu phi chng minh. ng thc xy ra khi
v ch khi x1= x2= ...= xn .
Bi 5. (VMO 2011) Chng minh rng vi mi s thc dng x v s nguyn
dng n ta c

) x +1

x n x n +1 + 1

xn + 1
ng thc xy ra khi no?

2 n +1

Li gii

3
x x2 + 1
x +1
+ Vi n = 1 bt ng thc tr thnh

x +1
2

( x + 1) 8 x x 2 + 1 0 ( x 1) 0 .
4

Vy bt ng thc ng vi n = 1 .
+ Gi s bt ng thc ng vi n = k tc
Ta cn chng minh
58

) x +1

x k x k +1 + 1
xk + 1

2 k +1

(1).

www.TaiLieuLuyenThi.com
Cty TNHH MTV DVVH Khang Vit

) x +1

x k +1 x k + 2 + 1

2 k +3

x k +1 + 1
2
Do (1) nn ta ch cn chng minh

k +1
k +1 k + 2
2 k
x
+1
x +1 x x +1 x

k
k
+
1
x +1
x +1
2

( x + 1) x k x k +1 + 1 4 x k + 1 x k +1 x k + 2 + 1 0 .
2

( x 1)

(x

k +1

1 0

Bt ng thc lun ng. Vy theo nguyn l quy np ta c iu phi chng minh.


ng thc xy ra khi v ch khi x = 1 .
kt thc ti trnh by li gii chng minh bt ng thc AM GM bng
quy np.
Bi 6. (BT AM GM ) Cho a 1 , a 2 , , a n l cc s thc khng m.
Chng minh rng ta lun c a1 + a2 + ... + an n n a1a2 ...an
Li gii
C s quy np vi n = 1, 2 c kim tra d dng.
Gi s bt ng thc c chng minh cho n s.
Xt n+1 s khng m a 1 , a 2 , , a n+1 . t a 1 a 2 a n+1 = An+1. Nu tt c cc s
bng nhau th bt ng thc ng.
Trong trng hp ngc li, phi tn ti hai s a i , a j sao cho a i < A < a j .
Khng mt tnh tng qut, c th gi s a n < A < a n+1 .
Khi ta c (a n A)(a n+1 A) < 0, suy ra a n + a n+1 > a n a n+1 /A + A.
T ta c a 1 + a 2 + + a n + a n+1 > a 1 + + a n-1 + a n a n+1 /A + A (1)
By gi p dng bt ng thc Cauchy cho n s a 1 + + a n-1 + a n a n+1 /A ta
c a1 + a2 + ... + an 1 + an n n a1a2 ...an 1

an an +1
nA .
=
A

Kt hp vi (1) ta c iu phi chng minh.


C. BI TP RN LUYN
Bi 1. Cho a,b,c l di ba cnh mt tam gic vung c c l cnh huyn. Chng
minh a 2 n + b 2 n c 2 n vi mi s nguyn dng n.
Bi 2. Cho n s thc khng m x1 , x2 ,..., xn tho mn iu kin
2 ( x1 + x2 + ... + xn ) 1 .
59

www.TaiLieuLuyenThi.com
Khm ph t duy K thut gii bt T Bi ton Max Min ng Thnh Nam

1
.
2
Bi 3. Chng minh rng vi n s nguyn dng phn bit ta c

Chng minh rng (1 x1 )(1 x2 ) ... (1 xn )

a13 + a23 + ... + an3 ( a1 + a2 + ... + an ) .


2

Bi 4. Cho x 1 , x 2 , , x n l cc s thc thuc [0, 1]. Chng minh rng


x 1 (1-x 2 ) + x 2 (1-x 3 ) + + x n (1-x 1 ) [n/2]
Bi 5. Cho n 2 v x 1 , x 2 , , x n l n s nguyn phn bit.
Chng minh rng (x 1 -x 2 )2 + (x 2 -x 3 )2 + + (x n x 1 )2 4n 6
Bi 6. Chng minh rng vi x 1 x 2 x n 0 ta c bt ng thc
n

xi

xi2

=i 1 =i 1

Bi 7. Chng minh rng nu a 1 , a 2 , , a n l cc s nguyn dng phn bit th ta


2
n 3
7
5
bt ng thc
(ai + ai ) 2 ai
=i 1 =
i 1
n

Bi 8. (Bt ng thc Mc-Lauflin) Vi mi s thc a 1 , a 2 , , a 2n v b 1 , b 2 , , b 2n


ta c bt ng

2n
n
thc
ak2 bk2 (a2 k b2 k 1
=
=
k 1=
k 1
k 1
2n

2n

a2 k 1b2 k ) ak bk .
=
k 1

D. HNG DN GII P S
Bi 1. Vi n = 1 bt ng thc tr thnh ng thc.
Gi s bt ng thc ng vi n= k > 1 tc a 2 k + b 2 k c 2 k (1).
2 k +1
2 k +1
2 k +1
Ta cn chng minh a ( ) + b ( ) c ( ) .

2 k +1
2 k +1
Ch do c (1) nn ta ch cn chng minh c 2 a 2 k + b 2 k a ( ) + b ( )

)(

2 k +1
2 k +1
a 2 + b 2 a 2 k + b 2 k a ( ) + b ( ) a 2b 2 k + b 2 a 2 k 0 .

Bt ng thc. Vy theo nguyn l quy np ta c iu phi chng minh.


CH 4: K THUT MIN GI TR
A. GII THIU
Phng php ny kh c bn v thng c s dng trong cc bi ton tm
cc tr vi cc bt ng thc c dng: a f ( x) b vi x D .
60

www.TaiLieuLuyenThi.com
Cty TNHH MTV DVVH Khang Vit

Nguyn tc chung l a v tm iu kin phng trnh m = f ( x) c nghim


trn D.
Trong trng hp c nhiu bin ta cn a v phng trnh vi mt bin hoc
a v h phng trnh nu c.
B. NI DUNG PHNG PHP
1) Tm min gi tr bng cch xt iu kin c nghim ca phng trnh
bc hai
Mt phng trnh bc hai c dng: Ax 2 + Bx + C =
0 vi A 0 .
iu kin phng trnh c nghim l:
= B 2 4 AC 0 .
V vy cn tm gi tr ln nht v gi tr nh nht ca mt biu thc
m = f ( x, y , z ) .
Ta bin i tng ng m a v mt phng trnh bc ca x hoc y hoc
ca z.
Khi s dng iu kin c nghim ta tm c Max v Min ca m.
3
x y + z =
.
V d 1. Cho x,y,z l cc s thc tho mn iu kin 2
2
2
5
x + y + z =

Tm gi tr ln nht v nh nht ca biu thc P =

x+ y2
.
z+2

Li gii
Trong biu thc ca P c z khc so vi x v y do vy ta tm cch rt x + y theo
z v a v phng trnh bc hai i vi z. Vic tm Max v Min ca P ta chn
bng iu kin c nghim ca phng trnh bc hai i vi z.
Ta c ( z + 2 ) P = x + y 2 ( z + 2 ) P + 2 =
2

( x + y )2 .

Ch
x y =3 z
x y = 3 z

3z 2 + 6 z + 1 .
2
1
2 1
2
2 ( x + y) =
2
2
x
y
x
y
5
z
+
+

=
)
(
)
x + y =5 z
2 (
2
V vy

( z + 2 ) P + 2 =3 z 2 + 6 z + 1
2

( z + 2 ) P 2 + 4 ( z + 2 ) P + 3z 2 6 z + 3 =
0
2

P 2 + 3 z 2 + 4 P 2 + 4 P 6 z + 4 P 2 + 8P + 3 =
0 (1)

Ta c (1) l phng trnh bc hai i vi z iu kin c nghim l

61

www.TaiLieuLuyenThi.com
Khm ph t duy K thut gii bt T Bi ton Max Min ng Thnh Nam

=
'

( 2P

+ 2P 3

) (P
2

)(

+ 3 4 P 2 + 8P + 3 0

36
P0
23
+ Vi=
x 2,=
y 0,=
z 1 ta c P bng 0. Vy gi tr ln nht ca P bng 0.
23P 2 + 36 P 0

20
66
7
+ Vi x =
,y=
, z =th P bng -36/23. Vy gi tr nh nht ca P bng
31
31
31
36
.
23
Bi tp tng t
3
x y + z =
Cho x,y,z l cc s thc tho mn iu kin 2
.
2
2
5
x + y + z =

Tm gi tr ln nht v nh nht ca biu thc P =

x + y 1
.
z+2

3 2 3
3+ 2 3
.
P
3
3
2) K thut s dng iu kin c nghim ca phng trnh lng gic
Phng trnh dng: A sin x + B cos x =
C

p s:

c nghim khi v ch khi A2 + B 2 C 2 .


V d 1. Chng minh rng vi mi s thc x ta c

2 cos x + 2sin x + 3

2.
11 2cos x sin x + 4

Li gii
t y=

cos x + 2sin x + 3
y ( 2cos x sin x + 4 )= cos x + 2sin x + 3 .
2cos x sin x + 4

( 2 y 1) cos x ( y + 2 ) sin x =3 4 y (1) .

( 2 y 1)2 + ( y + 2 )2 ( 3 4 y )2

iu kin (1) c nghim l

11 y 2 24 y + 4 0

2
y2
11

Bi ton c chng minh.


3) K thut iu kin c nghim ca phng trnh - h phng trnh
Thay v tm trc tip Max v Min ca biu thc ta a v tm iu kin c
nghim ca h phng trnh.
V d 1. Cho x,y l hai s thc tho mn iu kin

(x
62

y 2 + 1 + 4 x2 y 2 x2 y 2 =
0.

www.TaiLieuLuyenThi.com
Cty TNHH MTV DVVH Khang Vit

Chng minh rng

3 5
3+ 5
.
x2 + y 2
2
2
Li gii

t m
= x 2 + y 2 . Ta cn tm Max v Min ca m.
Thay v i chng minh trc tip ta tm iu kin h phng trnh sau c
m
= x2 + y 2

nghim
2
2
2
2 2
2
2
0
x y + 1 + 4 x y x y =

m
= x2 + y 2

4
4
2
2
2 2
0
x + y + x 3 y + 2 x y + 1 =
m
= x2 + y 2

2
2 2
3 x2 + y 2 + 4 x2 + 1 =
0
x + y
2
m 2 3m + 1
x
=

= x2 + y 2
m

2
2
2
m + 1 + 4x 0 2 m + m + 1
m 3=
y =
4
iu kin h phng trnh c nghim l

m 2 3m + 1
0

3 5
3+ 5

4
.

m
2
2
2
m + m +1
0

4
3 5
3+ 5
.
x2 + y 2
2
2
Bi tp tng t

V vy

Cho x,y l cc s thc tho mn iu kin x, y 1;4 x 2 y 2 x 2 y 2= 18 xy 16 .


Tm gi tr ln nht v gi tr nh nht ca biu thc
P=

p s: Pmin =16

2 x+ y
x2 + x + y 2 + y + 5 + x + y

17 4 ; Pmax = 64

37 6 .

V d 2. Cho x,y l cc s thc thay i tho mn iu kin


x + y =
2

2x + 1 + 2 y + 1 .

Tm gi tr ln nht ca biu thc P


=

( x y )2 .
63

www.TaiLieuLuyenThi.com
Khm ph t duy K thut gii bt T Bi ton Max Min ng Thnh Nam

Li gii

u2 1
x
=

2x + 1

2 .
, ( u, v 0 )
2
2y +1
v 1

y = 2

u
=
t
=
v

iu kin bi ton tr thnh

u 2 + v2 2
2 = u + v u 2 + v2 6 = 2 (u + v )
2

( u + v ) 2 ( u + v ) 2uv 6 =
0.
2

2
u 2 v2
u 2 1 v2 1
Khi P =

=
2
2
4

( u + v )2 4uv ( u + v )2

=
4
4

( u v )2 ( u + v )2

( u + v )2 2 ( u + v ) 2uv 6 =
0 (1)

.
Vy ta c h phng trnh
( u + v )2 4uv ( u + v )2

P=
(2)

4
1
2
2
T (1) ta c 2uv = ( u + v ) 2 ( u + v ) 6 ( u + v )
2
( u + v ) 4 ( u + v ) 12 0 2 u + v 6 .
2

Ch u + =
v

2( x + y) + 2 + 2

( 2 x + 1)( 2 y + 1)

V vy ta c t = u + v 6;6 .
Khi

{(u + v )
P=

2( x + y) + 2 6 .

2
2
2 ( u + v ) 2 ( u + v ) 6 ( u + v )

4
( u + v )2 + 4 ( u + v ) + 12 ( u + v )2 t 2 t 2 + 4t + 12
=

4
4
2

Xt hm s f (t ) =

t 2 t 2 + 4t + 12
4

) lin tc trn on

t 6;6

f '(t )= t t 2 + 3t + 6 ; f '(t )= 0
t=

64

6;6 ta c

3 + 33
.
2

www.TaiLieuLuyenThi.com
Cty TNHH MTV DVVH Khang Vit

3 + 33 3 69 + 11 33
Ta c f ( 6) =
9 + 6 6; f
; f (6) =
0.
=
2
8

3 + 33 3 69 + 11 33
.
V vy P=
f=
f
=

max
max
8
2
Du bng xy ra khi v ch khi

3 + 33
7 + 33
=
=
u + v
x + y

2
2

3 + 33
x + y 2 3 + 33
2=
=
x +1 + 2y +1

2
2

14 + 2 33 414 + 66 33
14 + 2 33 + 414 + 66 33
x =
,y
8
8

+ 2 33 + 414 + 66 33
14 + 2 33 414 + 66 33
x 14
=
,y

8
8

3 69 + 11 33

Vy gi tr ln nht ca P bng

).

8
V d 3. (TSH Khi A 2006) Cho x,y l 2 s thc v x 0, y 0 tha mn iu

kin xy ( x + y ) = x 2 + y 2 xy .
Tm gi tr ln nht ca biu thc =
A

1
x

1
y3

Li gii
u
x + y =
Nhn thy x, y i xng nn t:
.
x. y = v

Gi thit bi ton tr thnh: u.v = u 2 3v v =


Ta c

1
x3

1
y3

x3 + y 3

V u 2 4v u 2

( xy )3

u (u 2 3v)
v3

u2
(do u 3 )
u+3

u +3
= 2 =

v
u
u2

u 1
4u 2
4
u 1
.

1
0
u+3
u+3
u+3
u 3

Ch tm Max ca A ta ch cn xt vi
Max ca A bng 16 vy :

u+3
> 0 nn ta ch cn chng minh
u

u+3
4 vi u 1 hoc u 3 .
u
65

www.TaiLieuLuyenThi.com
Khm ph t duy K thut gii bt T Bi ton Max Min ng Thnh Nam

3
u+3
f(u) = 2 < 0
u
u
Trn mi khong ( ; 3 ) v [1; +) do f(u) f(1); u 1 .

Xt hm s f(u) =

Cn 0 < f(-3) < f(u) <1, u > -3.


Vy gi tr ln nht ca A bng 16.
C. BI TP RN LUYN
1 2sin x + cos x + 1
Bi 1. Chng minh rng vi mi s thc x ta c
2.
2 sin x 2cos x + 3
Bi 2. Chng minh rng vi mi s thc x v y ta c
2

3y2 + 2 + 2 3y2 + 1
cos3 x + y sin 3 x + 1
.


cos3 x + 2
9

Bi 3. Chng minh rng vi mi s thc x,y tho mn iu kin

x2 + y 2
x2 + y 2
5
2
2
x y=
+

3.
4
2
2

Chng minh rng

5 5
5+ 5
.
x2 + y 2
2
2

Bi 4. Cho x,y l cc s thc tho mn iu kin x 2 + xy + y 2 3 .


Chng minh rng 4 3 3 x 2 xy 3 y 2 4 3 3 .
3
x + y + z =
Bi 5. Cho x,y,z l cc s thc tho mn iu kin 2
.
2
2
5
x + y + z =

Tm gi tr ln nht, gi tr nh nht ca biu thc P =

x y+3
.
z+2

Bi 6. Cho x,y l hai s thc thay i tho mn iu kin


Tm gi tr ln nht v gi tr nh nht ca biu thc P =

2x + 3 + y + 3 =
4.
x+2 + y+9 .

D. HNG DN GII P S
2sin x + cos x + 1
Bi 1. t y =
( 2 y ) sin x + ( 2 y + 1) cos x = 3 y 1 .
sin x 2cos x + 3
iu kin phng trnh c nghim l: ( 2 y ) + ( 2 y + 1) ( 3 y 1)
2

1
4 y2 6 y 4 0 y 2 .
2
Bi ton c chng minh.
66

www.TaiLieuLuyenThi.com
Cty TNHH MTV DVVH Khang Vit

cos3 x + y sin 3 x + 1
Bi 2. t m =
( m 1) cos3 x y sin 3 x =
1 2m .
cos3 x + 2

iu kin phng trnh c nghim l ( m 1) + y 2 (1 2m )


2

3m 2 2m y 2 0

1 3y2 + 1
1 + 3y2 + 1
.
m
3
3
2

1 + 3y2 + 1
1 + 3y2 + 1
3y2 + 2 + 2 3y2 + 1
=
Suy ra m
.
m2

3
3
9

Bi ton c chng minh.

2
m 2 5m + 5
x
=

4
.
Bi 3. Tm c:
2
2 m m+5
y =
4
iu kin h phng trnh c nghim l:
m 2 5m + 5
0

5 5
5+ 5

4
.
m 2 5m + 5 0
m
2
2
2
m m + 5
0

4
Bi ton c chng minh.
Bi 4. Nu y = 0 bt ng thc lun ng.

Xt vi =
y 0 t m

x 2 xy 3 y 2 t 2 t 3
.
=
x 2 + xy + y 2 t 2 + t + 1

Suy ra m t 2 + t + 1 = t 2 t 3 ( m 1) t 2 + ( m + 1) t + m + 3 = 0 .
iu kin phng trnh c nghim l =
3m 2 6m 13 0

( m + 1)2 4 ( m 1)( m + 3) 0

3 4 3
3 + 4 3
.
m
3
3

Ch do 0 < x 2 + xy + y 2 3 4 3 3 x 2 xy 3 y 2 4 3 3 .
Bi ton c chng minh.
5
x + y =
Bi 5. Ch z 2 v ngc li ta c 2
v nghim.
2
1
x + y =

Khi ( z + 2 ) P = x y + 3 ( z + 2 ) P 3 = x y .
T h iu kin ta c
67

www.TaiLieuLuyenThi.com
Khm ph t duy K thut gii bt T Bi ton Max Min ng Thnh Nam

x + y =3 z
x + y = 3 z

2
1
3z 2 + 6 z + 1 .
2
2 1
2
2 ( x y) =
2
2
x
y
x
y
5
z
+
+

=
(
)
(
)
x + y =5 z
2
2

Khi ( z + 2 ) P 3 =3 z 2 + 6 z + 1
2

( z + 2 ) P 2 6 ( z + 2 ) P + 3z 2 6 z + 8 =
0
2

P 2 + 3 z 2 + 4 P 2 6 P 6 z + 4 P 2 12 P + 8 =
0

iu kin phng trnh c nghim l

( 4P

3P 3

) (P
2

23P 2 54 P + 15 0

V vy Pmin
=

)(

+ 3 4 P 2 12 P + 8 0
27 8 6
27 + 8 6
P
23
23

27 8 6
27 + 8 6
.
=
; Pmax
23
23

2
3 10
Bi 6. Thc hin tng t bi tp mu: Pmax =
+ 22; Pmin = .
2
2

CH 5: K THUT S DNG NGUYN L DIRICLE

BI TP MU
Bi 1. Cho x,y,z l cc s thc dng tho mn iu kin x + y + z + 1 =
4 xyz .
Chng minh rng xy + yz + zx x + y + z .
Li gii
Theo nguyn l Dirichlet tn ti 2 trong 3 s ( x 1),( y 1),( z 1) cng du,
khng mt tnh tng qut gi s ( x 1)( y 1) 0 xy x + y 1 .
xy + yz + zx x + y 1 + yz + zx .

Vy ta cn chng minh x + y 1 + yz + zx x + y + z z ( x + y 1) 1 .
Theo gi thit ta c z =

x + y +1
.
4 xy 1

Vy ta chng minh
x + y +1
2
2
. ( x + y 1) 1 ( x + y ) 1 4 xy 1 ( x y ) 0 .
4 xy 1
Bt ng thc c chng minh. ng thc xy ra khi v ch khi x= y= z= 1 .
68

www.TaiLieuLuyenThi.com
Cty TNHH MTV DVVH Khang Vit

Bi tp tng t
Cho a,b,c l cc s thc dng tho mn iu kin a + b + c + abc =
4.
Chng minh rng a + b + c ab + bc + ca .
Bi 2. Cho a,b,c l cc s thc khng m tha mn iu kin a + b + c =
3.

)(

)(

Chng minh rng 1 a + a 2 1 b + b 2 1 c + c 2 1 .


Li gii
Trong ba s 1 a,1 b v 1 c lun tn ti hai s cng du khng mt tnh tng
qut gi s hai s l 1 b v 1 c suy ra (1 b )(1 c ) 0 .

)(

) (b

Khi 1 b + b 2 1 c + c 2 =

)(

b c2 c + b2 + c2 b c + 1

= bc ( b 1)( c 1) + b 2 + c 2 b c + 1
b2 + c2 b c + 1

1
( b + c )2 ( b + c ) + 1
2

1
a 2 4a + 5
( 3 a )2 ( 3 a ) +=1
2
2
V vy bt ng thc c chng minh nu ta chng minh c
=

(1 a + a ). a

4a + 5
2
1 ( a 1) a 2 3a + 3 0 (lun ng).
2
ng thc xy ra khi v ch khi a= b= c= 1 .
Nhn xt. Ngoi ra bi ton c th gii bng k thut dn bin xem chng 4.
Bi 3. Cho x,y,z l cc s thc khng m tho mn iu kin
2

x 2 + y 2 + z 2 + 2 xyz =
1.

Tm gi tr ln nht ca biu thc P = xy + yz + zx 2 xyz .


Li gii
Trong 3 s (2 x 1),(2 y 1),(2 z 1) lun tn ti hai s cng du, khng mt tnh
z
tng qut gi s ( 2 x 1)( 2 y 1) 0 2 ( x + y ) 4 xy 1 z ( x + y ) 2 xyz .
2
1 z
.
Ch 1 z 2= 2 xyz + x 2 + y 2 2 xy + 2 xyz= 2 xy ( z + 1) xy
2
1 z z 1
V vy P
+ =.
2
2 2
1
1
1
th P bng . Vy gi tr ln nht ca P bng .
Vi x= y= z=
2
2
2

69

www.TaiLieuLuyenThi.com
Khm ph t duy K thut gii bt T Bi ton Max Min ng Thnh Nam

Bi tp tng t
Cho a,b,c l cc s thc dng tho mn iu kin a 2 + b 2 + c 2 + abc =
4.
Chng minh rng ab + bc + ca abc 2 .
Bi 4. Cho x,y,z l cc s thc dng c tch bng 1.
1
1
1
2
Chng minh rng
+
+
+
1.
2
2
2
( x + 1) ( y + 1) ( z + 1) ( x + 1)( y + 1)( z + 1)
Li gii
Trong ba s ( x 1),( y 1),( z 1) lun c hai s cng du, khng mt tnh tng
qut gi s ( x 1)( y 1) 0 xy + 1 x + y 2 ( xy + 1) ( x + 1)( y + 1) .
S dng bt ng thc

( x + 1)

( y + 1)

1
.
1 + xy

Chng minh xem chng 3.


1
1
1
z
1
z
Suy ra VT
+
+
= +
+
=
1.
2
2
1 + xy ( z + 1)
(1 + xy )(1 + z ) 1 + z ( z + 1) (1 + z )2
Bt ng thc c chng minh. ng thc xy ra khi v ch khi x= y= z= 1 .
Bi 5. Cho a,b,c l cc s thc khng m. Chng minh
1
abc + 2 +
( a 1)2 + ( b 1)2 + ( c 1)2 a + b + c .

2
Li gii
Lun tn ti hai trong ba s (a 1),(b 1),(c 1) cng du, khng mt tnh tng
qut gi s ( a 1)( b 1) 0 ab a + b 1 .
Khi ta ch cn chng minh
1
c ( a + b 1) + 2 +
( a 1)2 + ( b 1)2 + ( c 1)2 a + b + c

2
( a 1) + ( b 1) + ( c 1) 2 ( a + b 2 )( c 1)
2

S dng bt ng thc C S v bt ng thc AM GM ta c

( a 1) + ( b 1) + ( c 1)
2

( a + b 2 )2 +
2

( c 1) 2
2

( a + b 2 )2 .

2 ( a + b 2 )( c 1) 2 ( a + b 2 )( c 1)

Bi 6. Cho a,b,c l cc s thc khng m chng minh


a 2 + b 2 + c 2 + 2abc + 1 2 ( ab + bc + ca ) .
70

( c 1)2

www.TaiLieuLuyenThi.com
Cty TNHH MTV DVVH Khang Vit

Li gii
Bi ton c trnh by trong ch K thut s dng tam thc bc hai.
Di y l li gii tip cn theo nguyn l Dircihlet.

Lun tn ti 2 trong 3 s ( a 1) , ( b 1) , ( c 1) cng du, khng mt tnh tng

qut ta gi s ( a 1)( b 1) 0 2c ( a 1)( b 1) 0 2abc 2 ( ac + bc c ) .


Vy ta ch cn chng minh
a 2 + b 2 + c 2 + 1 + 2 ( ac + bc c ) 2 ( ab + bc + ca )
( a b ) + ( c 1) 0
2

Bt ng thc c chng minh. ng thc xy ra khi v ch khi a= b= c= 1 .


Bi ton tng t
Cho a,b,c l cc s thc khng m chng minh
a 2 + b 2 + c 2 + a 2b 2 c 2 + 2 2 ( ab + bc + ca ) .

Bi 7. Cho a,b,c l cc s thc khng m tho mn iu kin a 2 + b 2 + c 2 + abc =


4.
Chng minh rng abc ab + bc + ca abc + 2 .
Li gii
Bt ng thc v tri n gin bi trong 3 s c t nht mt s khng vt qu 1
gi s l c khi ab + bc + ca abc= ab (1 c ) + c ( a + b ) 0 .
ng thc xy ra chng hn ti a= 2, b= c= 0 .
Ta chng minh bt ng thc v phi:
Trong ba s (a 1),(b 1),(c 1) lun c hai s cng du, khng mt tnh tng
qut gi s ( a 1)( b 1) 0 ab a + b 1 abc c ( a + b 1) .
Vy ta chng minh c ( a + b 1) + 2 ab + bc + ca ab 2 c .
Ch 4 = (a 2 + b 2 ) + c 2 + abc 2ab + c 2 + abc ab 2 c .
Bt ng thc c chng minh.
Bi 8. Cho a,b,c l cc s thc dng c tch bng 1.
1
1
1
1
Chng minh rng
+
+
+
1.
2
2
2
(1 + a ) (1 + b ) (1 + c ) a + b + c + 1
Li gii
Theo nguyn l Dirichlet th 2 trong 3 s ( a 1) , ( b 1) , ( c 1) cng du, khng
mt tnh tng qut ta gi s ( a 1)( b 1) 0 1 + ab
=
Ch .

(1 + a )

(1 + b )

c +1
a +b.
c

1
c
.
=
1 + ab c + 1
71

www.TaiLieuLuyenThi.com
Khm ph t duy K thut gii bt T Bi ton Max Min ng Thnh Nam

1
1
c
1.
+
+
=
2
c +1
c + 1 ( c + 1)
+ c +1
c
Bt ng thc c chng minh. ng thc xy ra khi v ch khi a= b= c= 1 .

Gi P l biu thc v tri ta c P

A. BI TP RN LUYN
Bi 1. Cho a,b,c l cc s thc tho mn iu kin
1
1
1
1
.
+ 2
+ 2
=
2
a + 8 4b + 8 9c + 8 3
Tm gi tr ln nht v gi tr nh nht ca biu thc P =a + 2b + 3c .
B. HNG DN GII P S
Bi 1. n gin t=
x a=
, y 2b=
, z 3c ta c

1
x +8
2

1
y +8
2

1
.
=
z +8 3
2

Ta cn Max v Min ca P = x + y + z .
Theo gi thit kt hp s dng bt ng thc AM GM ta c
1
1
1
1
1
= 2
+ 2
2
6
6
x +8
y +8
z +8
=

y2 + 2

6 y2 + 8

z2 + 2

) 6( z

+8

1
3

y2 + 2 z2 + 2
.
y2 + 8 z2 + 8

Tng t ta c
1
y2 + 8

1 x2 + 2 z 2 + 2
1
1

. 2
; 2
2
3 x +8 z +8 z +8 3

y 2 + 2 x2 + 2
.
y 2 + 8 x2 + 8

Nhn theo v ba bt ng thc trn ta c:

(x

)( )
Mt khc ( x + 2 )( y + 2 )( z
2

)(

+ 2 y 2 + 2 z 2 + 2 27 (1).
2

+ 2 3 ( x + y + z ) (2).
2

Tht vy bt ng thc cho tng ng vi:

x 2 y 2 z 2 + 2 x 2 y 2 + y 2 z 2 + z 2 x 2 + 8 6 ( xy + yz + zx ) .

Trong ba s ( x 2 1),( y 2 1),( z 2 1) lun c hai s cng du, khng mt tnh


tng qut gi s

( x 1)( y 1) 0 x
2

72

y x2 + y 2 1 x2 y 2 z 2 z 2 x2 + y 2 1 .

2 2

www.TaiLieuLuyenThi.com
Cty TNHH MTV DVVH Khang Vit

Do ta ch cn chng minh

z 2 x 2 + y 2 1 + x 2 + y 2 + z 2 + 2 x 2 y 2 + y 2 z 2 + z 2 x 2 + 8 6 ( xy + yz + zx )
( x y ) + 3 ( yz 1) + 2 ( xy 1) + 3 ( xz 1) 0
2

Bt ng thc cui ng suy ra iu phi chng minh.


Kt hp (1) v (2) ta c

( x + y + z )2 9 3 x + y + z 3 .
1
1
Vy gi tr nh nht ca P bng -3 t ti a =
1, b =
,c =
.
2
3
1
1
Gi tr ln nht ca P bng 3 t ti =
.
a 1,=
b
=
,c
2
3

CH 6: K THUT S DNG TAM THC BC HAI


Cc tnh cht v nghim v du ca tam thc bc hai c mt ng dng ht sc
su rng trong gii ton. Di y ti trnh by mt ng dng ca tam thc bc hi
trong chng minh bt ng thc. Bt ng thc tip cn bng phng php ny c
th ni l rt t nhin.

A. NI DUNG PHNG PHP


Xt tam thc bc hai f ( x) = ax 2 + bx + c .
nh l 1. Nu a > 0 th f(x) t gi tr nh nht ti x0 =
f ( x0 ) =

b 2 4ac
.
4a

nh l 2. Nu a < 0 th f(x) t gi tr ln nht ti x0 =


f ( x0 ) =

b
v gi tr nh nht
2a

b
v gi tr ln nht
2a

b 2 4ac
.
4a

nh l 3. Nu = b 2 4ac 0 th f(x) lun c nghim.


a > 0
nh l 4. Nu
th f ( x) 0 vi mi x.
2
= b 4ac 0
a < 0
nh l 5. Nu
th f ( x) 0 vi mi x.
2
= b 4ac 0

73

www.TaiLieuLuyenThi.com
Khm ph t duy K thut gii bt T Bi ton Max Min ng Thnh Nam

B. BI TP MU
Bi 1. Chng minh rng vi x,y,z l cc s thc c tng bng 1 ta c

( 3x + 4 y + 5 z )2 44 ( xy + yz + zx ) .
Li gii
Thay z =1 x y bt ng thc tr thnh:

( 3x + 4 y + 5 5 x 5 y )2 44 xy + 44 ( x + y )(1 x y )
48 x 2 + 16 x ( 3 y 4 ) + 45 y 2 54 y + 25 0
V tri l tam thc bc hai ca x vi h s ca x 2 dng v c

'x =
64 ( 3 y 4 ) 48 45 y 2 54 y + 25 =
176 ( 3 y 1) 0 .
2

Vy bt ng thc c chng minh. ng thc xy ra khi v ch khi


1
1
1
.
=
x =
,y =
,z
2
3
6
Cch 2: Bt ng thc cho tng ng vi:
9 x 2 + 16 y 2 + 25 z 2 20 xy + 4 yz + 14 zx .

S dng bt ng thc AM GM ta c
5
5
4 x 2 + 9 y 2 2. 4 x 2 .9 y 2 20 xy
3
3
7
7
4 x 2 + 36 z 2 2.
4 x 2 .36 z 2 14 xz
12
12

y 2 + 4 z 2 2 y 2 .4 z 2 4 yz

Cng theo v ba bt ng thc trn ta c pcm. Nh vy khng cn gi thit bi


ton ba s c tng bng 1(Xem thm ch k thut tham s ho Chng 2).

Bi 2. Chng minh rng vi mi s thc a v b ta lun c

)(

) (

3 1 a + a 2 1 b + b 2 2 1 ab + a 2b 2 .

Li gii
Vit li bt ng thc di dng:

(a

3a + 3 b 2 3a 2 5a + 3 b + 3a 2 3a + 1 0 .

V tri l tam thc bc hai ca b c a 2 3a + 3 > 0, a v

b =
3a 2 5a + 3

)(

) (

Do v tri lun khng m. Bi ton c chng minh.

74

4 a 2 3a + 3 3a 2 3a + 1 =
a 2 3a + 1 0, a

www.TaiLieuLuyenThi.com
Cty TNHH MTV DVVH Khang Vit

a 2 3a + 1 =
0

3 5
2
ng thc xy ra khi v ch khi
.
3a 5a + 3 a = b =
b
=
2

2 a 2 3a + 3

Bi tp tng t
Chng minh rng vi mi s thc a,b,c,d ta c
3(a 2 ab + b 2 )(c 2 cd + d 2 ) 2(c 2 a 2 abcd + b 2 d 2 ) .

Ta cng xt mt s bi ton cng dng sau y

Bi 2.1. Cho a,b,c l cc s thc khng m. Chng minh rng

)(

)(

3 1 a + a 2 1 b + b 2 1 c + c 2 1 + abc + a 2b 2 c 2 .

Li gii

)(
)
Suy ra 2 (1 a + a )(1 b + b ) 1 + a b

Ta c 2 1 a + a 2 1 b + b 2 =1 + a 2b 2 + ( a b ) + (1 a ) (1 b ) .
2

2 2

)(

Do ta ch cn chng minh bt ng thc sau

) (

3 1 + a 2b 2 1 c + c 2 2 1 + abc + a 2b 2 c 2 .

3 + a 2b 2 c 2 3 + 2ab + 3a 2b 2 c + 1 + 3a 2b 2 0 .

Coi v tri l tam thc bc hai ca c c h s ca c 2 dng v

= 3 + 2ab + 3a 2b 2

)(

4 3 + a 2b 2 1 + 3a 2b 2 = 3 (1 ab ) 0 .
4

Do v tri lun khng m.


Bi ton c chng minh ng thc xy ra khi v ch khi a= b= c= 1 .

Bi 2.2. Cho x v y l hai s thc khng cng dng ta lun c

)(

4 2
x x + 1 y 2 y + 1 x 2 y 2 xy + 1 .
3
Li gii
Theo gi thit trong hai s lun c mt s khng dng khng mt tnh tng
qut ta gi s y 0 khi vit li bt ng thc di dng:

( y 2 )2 x 2 ( 4 y 2 7 y + 4 ) x + (1 2 y )2 0 .
Coi v tri l tam thc bc hai ca x ta c
=

(4y

7y + 4

4 ( y 2 ) (1 2 =
y)
y 24 y 2 51 y + 24 0, y 0 .
2

75

www.TaiLieuLuyenThi.com
Khm ph t duy K thut gii bt T Bi ton Max Min ng Thnh Nam

Bi ton c chng minh ng thc xy ra khi v ch khi=


x

1
=
, y 0 hoc
2

1
.
2
M rng bi ton cho 3 bin ta c kt qu
Cho x,y,z l cc s thc khng cng dng ta lun c
16 2
x x + 1 y 2 y + 1 z 2 z + 1 1 xyz + x 2 y 2 z 2 .
9
=
x 0,=
y

)(

)(

Bi 3. Cho a,b,c l cc s thc khng m tha mn iu kin a 2 + b 2 + c 2 =


2.
Chng minh rng: 1 + 2abc ab + bc + ac .
Li gii
2
.
3
Khi t S =+
a b, P =
ab theo gi thit bi ton ta c

Gi s c max {a, b, c} c
=

S 2 2 P + c 2 =2 P =

Ta cn chng minh 1 + 2c.

S 2 + c2 2
.
2

S 2 + c2 2 S 2 + c2 2

+ c.S
2
2

( 2c 1) S 2 2cS + 2c3 c 2 4c + 4 0

V tri l tam thc bc hai vi h s dng do vy ta ch cn chng minh

(
4 ( c 1) ( c

'S = c 2 ( 2c 1) 2c3 c 2 4c + 4 0
2

+ c 1 0

2
.
3
Bt ng thc c chng minh. ng thc xy ra khi v ch khi a= b= 1, c= 0
hoc cc hon v.

Bt ng thc cui lun ng do c

Bi tp tng t
Cho a, b, c l cc s thc dng tho mn iu kin
(a + 1)(b + 1)(c + 1) =1 + 4abc .
Chng minh rng: a + b + c 1 + abc .
Bi 4. Cho a,b,c l cc s thc khng m thay i tha mn iu kin
a+b+c =
1.
Tm gi tr ln nht ca biu thc P = a ( b c ) + b ( c a ) + c ( a b ) .
4

76

www.TaiLieuLuyenThi.com
Cty TNHH MTV DVVH Khang Vit

Li gii
Khng mt tnh tng qut gi s a b c, khi
P = a ( b c ) + b ( c a ) + c ( a b ) a ( b + c ) + ba 4 + ca 4
4

= a ( b + c ) + a 4 ( b + c ) = a ( b + c ) ( b + c ) + a 3

3
= a ( b + c ) ( a + b + c ) 3a ( b + c )( a + b + c )

=a ( b + c ) (1 3a ( b + c ) ) =3 a ( b + c ) + a ( b + c )
2

1
1
1

=
3 a ( b + c ) +
6 12 12

Vy gi tr ln nht ca P bng

1
3+ 6
3 6
t
ti a =
=
,b =
, c 0 hoc
12
6
6

cc hon v.
Ch . Ta c bt ng thc mnh hn nh sau
a (b + c ) + b (c + a ) + c ( a + b)
4

1
( a + b + c )4
12

Bi 5. Cho cc s thc thay i x,y,z tha mn iu kin


x2 + y 2 + z 2 +

16
xy =
3.
25

Tm gi tr nh nht ca biu thc


3 2
5
( x + y 2 ) + z 2 + xy 10( xy + yz + zx) .
5
6
Li gii
Nhn xt. Trc ht ta cha quan tm n iu kin ca x,y,z m n tnh
cht i xng ca x v y trong iu kin cng nh biu thc ca P nn ta s dng
P=

( x + y )2
.
2
S dng bt ng thc AM-GM ta c:
3
5
P ( x + y ) 2 + z 2 + xy 10( xy + yz + zx)
10
6
3
5
2
( x + y )2 . z 2 + xy 10 ( xy + yz + zx )
10
6

nh gi: x 2 + y 2

= ( x + y ) z + xy 10( xy + yz + zx) xy + yz + zx 10( xy + yz + zx )

Mt nh gi d nhn ra l a v tam thc bc 2:


77

www.TaiLieuLuyenThi.com
Khm ph t duy K thut gii bt T Bi ton Max Min ng Thnh Nam
2

5 5
5
xy + yz + zx 10 ( xy + yz + zx =
) xy + yz + zx .
2
2
2

ng thc xy ra khi v ch khi


5

xy + yz + zx =

2
5
5
2

3
,y=
,z =
x =
x
y
=
17
34
34

16
5
5
2
x2 + y 2 + z 2 +
xy =
3=
x =
,y =
,z 3

25
17

34
34
z ( x + y) 0

5
5
2
5
Vy gi tr nh nht ca P bng t ti
hoc
,y=

,z =
3
17
2
34
34
=
x

5
=
,y
34

5
2
.
=
,z 3
17
34

Bi 6. Cho x,y,z l cc s thc tha mn iu kin x 2 + y 2 + z 2 =


3.
Tm gi tr ln nht v gi tr nh nht ca biu thc P = xy + yz + 2 yz .
Li gii
Ta tm gi tr nh nht ca P
Xut pht t bt ng thc c bn:

( x + y + z )2 0 xy + yz + zx

x2 + y 2 + z 2
3
= .
2
2

3
3 x2 + z 2
3 x2 + y 2 + z 2

= 3 .
Suy ra P = ( xy + yz + zx ) + zx + zx
2
2
2
2
2
ng thc xy ra khi v ch khi

3
3
x + y + z =
0
0, z =
,y=
x =
2
2
2
2
2
.
=

3
x + y + z

3
3

2
2
2
2
2
, y = 0, z =
x =
x +z
x +y +z
3
xz =

=
=

2
2

2
2
2
3
3
hoc
Vy gi tr nh nht ca P bng 3 t ti x =
, y=
0, z =
2
2
3
3
.
, y = 0, z =
2
2
Cch 2: Vit li biu thc P di dng
x=

P + 3 = ( x + z ) + y ( x + z ) + y 2 ( x + z ) + y ( x + z ) + y 2 P 3 = 0 (1) .
2

78

www.TaiLieuLuyenThi.com
Cty TNHH MTV DVVH Khang Vit

Coi (1) l phng trnh bc 2 vi n l ( x + z ) ta c iu kin c nghim ca

phng trnh l ( x + z=
y 2 4 y 2 P 3 0 P 3 +
)

3y2
3 .
4

y = 0

3
3
0, z =
,y=

x =
y
2
2

ng thc xy ra khi v ch khi x + z = =0


.

2
3
3

, y = 0, z =
x =
x 2 + y 2 + z 2 =
3
2
2
Ta c kt qu tng t cch trn.
Tm gi tr ln nht ca P

Ta tm s thc k > 0 nh nht sao cho P = xy + yz + 2 zx k x 2 + y 2 + z 2 ng


vi mi x,y,z v x 2 + y 2 + z 2 . Khi gi tr ln nht ca P bng 3k.
Vit li bt ng thc trn di dng: ky 2 y ( x + z ) + kx 2 + kz 2 2 xz 0 .
y l mt tam thc bc hai vi h s ca y 2 dng nn ta ch cn tm k sao
cho y 0 .
Tc

( x + z )2 4k ( kx 2 + kz 2 2 xz ) 0 (1 4k 2 )( x 2 + z 2 ) + 2 xz (1 + 4k ) 0, x, z .
y l mt bt ng thc i xng vi x v z nn ta chn x= z= 1 .

1 3
k
2 .
Suy ra 1 4k 2 + 1 + 4k 0 2k 2 2k 1 0

1+ 3
k
2

Vy=
k

3 1+ 3
1+ 3
.
P
2
2

3+ 3
3 3
.
, y=
4
2
Nhn xt. Ngoi li gii bng tam thc bc hai nh trn ta c th s dng k thut
tham s ha hoc s dng C S.

ng thc xy ra khi v ch khi x= z=

1
Bi 7. Cho a,b,c l cc s thc thuc on ;3 . Chng minh
3
a
b
c
7
+
+
.
a+b b+c c+a 5

79

www.TaiLieuLuyenThi.com
Khm ph t duy K thut gii bt T Bi ton Max Min ng Thnh Nam

Li gii
Khng mt tnh tng qut gi s a = max {a, b, c} .
Bt ng thc cho tng ng vi

( 3a 2b ) c 2 ( 2a 2 ab 3b2 ) c + 3a 2b 2ab2 0 .

V 3a 2b > 0 ta ch cn chng minh

( 2a

ab 3b 2

4 ( 3a 2b ) 3a 2b 2ab 2 0

( a b )( a 9b ) 4a 2 + b 2 0

1
Bt ng thc cui ng do a b 0; a 9b 3 9. =
0.
3

Bt ng thc c chng minh. ng thc xy ra khi=


a 3,=
b

1
=
, c 1.
3

Nhn xt. Bng cch tng t ta chng minh c


a
b
c
8
+
+
.
a+b b+c c+a 5
Ngoi ra c th a v xt tnh n iu ca hm s (Xem chng 3).
Bi tp tng t

(TSH Khi A 2011) Cho cc s x, y, z [1;4] tha mn iu kin


x y, x z .

Tm gi tr nh nht ca biu thc P=

x
y
z
.
+
+
2x + 3y y + z z + x

HD: Chng minh Pmin =

34
.
33

Bi 8. Cho a,b,c l cc s thc khng m.


Chng minh rng a 2 + b 2 + c 2 + 2abc + 1 2 ( ab + bc + ca ) .
Li gii
Vit li bt ng thc di dng: a 2 + 2 ( bc b c ) a + ( b c ) + 1 0 .
2

Coi v tri bt ng thc trn l tam thc bc hai ca a ta c:


'a = ( bc b c ) ( b c ) 1 = bc ( b 2 )( c 2 ) 1 .
2

TH1: Nu bc b c 0 ta c ngay iu phi chng minh.


TH2: Nu bc b c 0 ( b 1)( c 1) 1 ta xt hai kh nng sau:
Kh nng 1. Nu ( b 2 )( c 2 ) 0 'a 1 ta c ngay iu phi chng minh.
80

www.TaiLieuLuyenThi.com
Cty TNHH MTV DVVH Khang Vit

b, c 2
Kh nng 2. Nu ( b 2 )( c 2 ) 0
v ( b 1)( c 1) 1 nn b, c 2
b, c 2
khi s dng bt ng thc AM-GM ta c:
2

b+ 2b c+ 2c
'a= b ( 2 b ) .c ( 2 c ) 1
.
1= 0, ta c iu phi
2
2

chng minh.
Ch . Xem thm li gii bng Nguyn l Dirichlet trong ch tng ng.
Bi tp tng t

Cho a,b,c l cc s thc dng tho mn iu kin a 2 + b 2 + c 2 2abc 1 0 .


Chng minh rng a b + b c + c a a 2 + b 2 + c 2 2abc 1 .
Bi 9. Cho x,y,z l cc s thc tho mn iu kin x + y + z = xy + yz + zx .
Chng minh rng

x
x +1
2

y
y +1
2

1
.
2
z +1

Li gii
Theo gi thit ta c: z ( x + y 1) = x + y xy .
1
x + y =
+ Nu x + y =1
, v nghim.
xy = 1
x + y xy
+ Vy x + y 1 v z =
.
x + y 1

Vy ta cn chng minh

y + 1 x + y xy

+1
x + y 1
2 ( x + y xy )( x + y 1)
2x
2y
2
+ 2
+
1
x + 1 y + 1 ( x + y xy )2 + ( x + y 1)2

( x + 1)2 ( y + 1)2
x2 + 1

y2 + 1

x +1

x + y xy
x + y 1

1
2

( xy 1)2
( x + y xy )2 + ( x + y 1)2

Ch theo bt ng thc CS ta c
2
+ 1)
( x + 1)2 + ( y=
( x + 1)2 (1 y )2 + ( y + 1)2 (1 x )2
x2 + 1
y 2 + 1 ( x 2 + 1) (1 y )2 ( y 2 + 1) (1 x )2
2
( x + 1)(1 y ) + ( y + 1)(1 x )
4 ( xy 1)

=
2
2
2
2
x 2 + 1 (1 y ) + y 2 + 1 (1 x )
x 2 + 1 (1 y ) + y 2 + 1 (1 x )
2

81

www.TaiLieuLuyenThi.com
Khm ph t duy K thut gii bt T Bi ton Max Min ng Thnh Nam

Vy ta ch cn chng minh

4 ( x + y xy ) + 4 ( x + y 1) x 2 + 1 (1 y ) + y 2 + 1 (1 x )
2

y 2 3 y + 3 x2 3 y 2 8 y + 3 x + 3 y 2 3 y + 1 =
0

V tri bt ng thc l tam thc bc hai ca x vi h s ca x2 dng v c

x =
3y2 8 y + 3

)(

4 y2 3y + 3 3y2 3y + 1 =
3 y 2 1 0 .

iu chng t bt ng thc ng. ng thc xy ra khi v ch khi


x=
y=
1, z =
1 hoc cc hon v.

Bi 10. Cho a,b,c l cc s thc tha mn iu kin a + b + c =


2 v
a3 + b3 + c3 3abc =
2.

Tm gi tr ln nht ca biu
thc P max {a, b, c} min {a, b, c} .
=
Li gii
Khng mt tnh tng qut gi s a = max {a, b, c} , c = min {a, b, c} P = a c .
Ta c: 2 = a3 + b3 + c3 3abc =
2=

( a + b + c ) ( a 2 + b2 + c 2 ab bc ca ) .

1
( a + b + c ) ( a b )2 + ( b c )2 + ( c a )2
2

2
( a b) + (b c ) + (c a ) =
2

Suy ra: P 2 + ( a b ) + ( b c ) =
2.
2

P 2 + ( a c ) + ( c b ) + ( b c ) =2
2

P 2 + ( a c ) + 2 ( b c ) 2 ( a c )( b c ) =
2.
2

P2 P (b c ) + (b c ) 1 =
0
2

Coi ng thc trn l phng trnh bc 2 vi n l (b c). phng trnh ny


4
2
c nghim ta phi c b c = P 2 4 P 2 1 0 P 2 P
.
3
3

2+ 3
a c = 3
a =
3

2
1

2
2

0 b =
(b c ) + =
ng thc xy ra khi v ch khi ( b c )
.
3
3
3

a + b + c =

0
2 3

c =
3

a b c

82

www.TaiLieuLuyenThi.com
Cty TNHH MTV DVVH Khang Vit

Vy gi tr ln nht ca P bng

2
2+ 3
2
2 3
t ti=
hoc
a
=
,b =
,c
3
3
3
3

cc hon v.
Cch 2: Khng mt tnh tng qut gi s a = max {a, b, c} , c = min {a, b, c} P = a c .
Ta c
a 2 + b 2 + c 2 ab bc ca =1 ( a + b + c ) 3 ( ab + bc + ca ) =1
2

ab + bc + ca =
1

Suy ra: P 2 =( a c ) =( a + c ) 4ac =( 2 b ) 4 1 b ( a + c )


2

2
4 4
2

=( 2 b ) 4 + 4b ( 2 b ) =3b 2 + 4b =3 b +
3
3 3

=
a
b 3=

ng thc xy ra khi v ch khi a =


c
b
=
3

a + b + c =
2

c =

2+ 3
3
2
3
2 3
3

Cch 3: Khng mt tnh tng qut gi s c = min {a, b, c} .


t a =
x + c, b =y + c, ( x, y 0 ) .
Ta c: a 2 + b 2 + c 2 ab bc ca =
1 suy ra
( x + c ) + ( y + c ) + c 2 c ( x + c ) c ( y + c ) ( x + c )( y + c ) =
1.
2

y 3y2

=
1 x
x +
2
4

x 2 xy + y 2 =

1
2
2

x y + 3 x =
1 y

2
4

Suy ra P max { x, y}
=

2
3
.
2
3

2
.
3

C. BI TP RN LUYN
Bi 1. Cho a,b,c l di 3 cnh mt tam gic v x,y,z l cc s thc thay
i tha mn ax + by + cz =
0 . Chng minh xy + yz + zx 0 .
1
x + y + z =
Bi 2. Cho x,y,z l cc s thc tho mn iu kin 2
.
2
2
4
x + 2 y + 3 z =
83

www.TaiLieuLuyenThi.com
Khm ph t duy K thut gii bt T Bi ton Max Min ng Thnh Nam

Tm gi tr ln nht ca x.
Bi 3. Cho x,y l hai s thc tho mn iu kin x 2 + xy + y 2 6 ( x + y ) + 11 =
0.
Tm gi tr ln nht, gi tr nh nht ca biu thc P= y + 2 x .
Bi 4. Cho x,y,z l cc s thc khng m c tng bng 1. Tm gi tr ln nht ca
biu thc P =9 xy + 10 yz + 11zx .
Bi 5. Cho hai s thc dng x,y tho mn iu kin x 2 y = 1 . Chng minh rng
x x2 + y 2 + x2 2 .

Bi 6. Cho n s thc a1 , a2 ,..., an thuc on [0;1]. Chng minh

(1 + a1 + a2 + ... + an )2 4 ( a12 + a22 + ... + an2 ) .

Bi 7. Tm tt c cc s nguyn dng n sao cho vi mi s thc x1 , x2 ,.., xn ta c


x12 + x22 + ... + xn2 ( x1 + x2 + ... + xn 1 ) xn .

Bi 8. Chng minh vi mi s thc x,y,z v a,b,c l di ba cnh mt tam gic ta


c a ( x y )( x z ) + b ( y z )( y z ) + c ( z x )( z y ) 0 .
Bi 9. (Vasile-Cirtoaje) Chng minh vi mi s thc a,b,c ta c

(a

+ b2 + c2

3 a3b + b3c + c3 a .

Bi 10. Cho a,b,c l cc s thc dng c tng bng 3. Chng minh


9
a + ab + 2abc .
2
Bi 11. Cho a,b,c l cc s thc khng m. Chng minh rng
a 2 + b 2 + c 2 + 2abc + 3 (1 + a )(1 + b )(1 + c ) .

Bi 12. Cho a,b,c l cc s thc khng m. Chng minh

2 a 2 + b 2 + c 2 + abc + 8 5 ( a + b + c ) .

Bi 13. Cho a,b,c l cc s thc khng m. Chng minh

(a

)(

)(

+ 2 b 2 + 2 c 2 + 2 9 ( ab + bc + ca ) .

Bi 14. Cho tam gic c ba gc A,B,C chng minh vi mi s thc x ta c


1+

1 2
x cos A + x ( cos B + cos C ) .
2

Bi 15. Chng minh rng vi mi s thc x v y ta c

x 2 1 + sin 2 y + 2 x ( sin y + cos y ) + 1 + cos 2 y > 0 .


84

www.TaiLieuLuyenThi.com
Cty TNHH MTV DVVH Khang Vit

Bi 16. Chng minh rng vi mi s thc x v y ta lun c

( x + y )2 xy + 1

3 ( x + y) .

Bi 17. Chng minh rng vi mi s thc x,y,z v ba gc A,B,C ca mt


tam gic ta c x 2 + y 2 + z 2 2 xy cos C + 2 yz cos A + 2 zx cos B .
Bi 18. Cho x,y,z l cc s thc khng m tho mn iu kin
xyz + x + y + z =
4.
Chng minh rng x + y + z xy + yz + zx .
0 < a b c
. Chng minh
x, y , z > 0

Bi 19. Cho

x y z (a + c)
ac ( xa + yb + zc ) + +
4ac
a b c

( x + y + z )2 .
a 2 + b 2 =
1
.
3
c + d =

Bi 20. Cho a,b,c,d l cc s thc tha mn iu kin


Chng minh rng ac + bd + cd

9+6 2
.
4

Bi 21. (TSH Khi A 2011) Cho cc s x, y, z [1;4] tha mn iu kin


x y, x z . Tm gi tr nh nht ca biu thc P=

x
y
z
.
+
+
2x + 3y y + z z + x

Bi 22. Cho a,b,c,d l cc s thc tha mn b > c > d . Chng minh

( a + b + c + d )2 > 8 ( ac + bd ) .
Bi 23. Cho a,b,c,d,p,q l cc s thc tha mn iu kin
p 2 + q 2 a 2 b2 c2 d 2 > 0 .

)(

Chng minh p 2 a 2 b 2 q 2 c 2 d 2 ( pq ac bd ) .
2

Bi 24. Cho a,b,c l di ba cnh mt tam gic v p,q,r l ba s thc thay


i tha mn iu kin p + q + r =
0 . Chng minh a 2 qr + b 2 rp + c 2 pq 0 .

)(

Bi 25. Chng minh rng vi mi s thc a,b ta c a 2 + 2 b 2 + 2 3 ( ab + a + b ) .


D. HNG DN GII P S
ax + by
. Vit li bt ng thc cn chng minh di
Bi 1. T gi thit ta c: z =
c
ax + by
dng: xy
( x + y ) 0 ax 2 y ( a + b c ) x + by 2 0 .
c
85

www.TaiLieuLuyenThi.com
Khm ph t duy K thut gii bt T Bi ton Max Min ng Thnh Nam

Coi v tri bt ng thc trn l tam thc bc hai ca x ta c:


2
2
2
=
y 2 ( a + b c ) 4aby=
y 2 ( a + b c ) 4ab
x

2
= y 2 ( a b ) + c 2 2c ( a + b ) y 2 c 2 + c 2 2c ( a + b=
) 2cy 2 ( c a b ) 0

Ta c iu phi chng minh.


Bi 2. Thay z =1 x y vo phng trnh th hai ta c

x 2 + 2 y 2 + 3 (1 x y ) 4 =
0
2

0
5 y 2 + 6 ( x 1) y + 4 x 2 6 x 1 =

' y 9 ( x 1) 5 4 x 2 6 x 1 0
=
2

11x 2 12 x 14 0

Khi
=
x

6 190
6 + 190
x
11
11

6 + 190
15 3 190
10 2 190
.
=
y
=
,z
11
55
55

6 + 190
.
11
Bi 3. Rt y= P 2 x thay vo iu kin bi ton ta c:

Vy gi tr ln nht ca x bng

x 2 + x ( P 2 x ) + ( P 2 x ) 6 ( x + P 2 x ) + 11 =
0
2

3 x 2 ( 3P 6 ) x + P 2 6 P + 5 =
0
=
x

( 3P 6 )2 12 ( P 2 6 P + 5) 0

P 2 12 P + 8 0 6 2 7 P 6 + 2 7

Vy Pmin =
6 2 7; Pmin =
6+2 7 .
Bi 4. Thay z =1 x y vo biu thc ca P v rt gn ta c
11x 2 + (12 y 11) x + 10 y 2 10 y + P =
0
=
x

(12 y 11)2 44 (10 y 2 10 y + P ) 0

.
2

74 2 22
121
74
11 195 195
y

y
= y +
11
37
196
11
37 148 148

195
25 11 27
Vy Pmax = ( x; y; z ) =
; ; .
148
74 37 74

Bi 5. Gi P l biu thc v tri ta c y =

86

1
x2

suy ra

www.TaiLieuLuyenThi.com
Cty TNHH MTV DVVH Khang Vit

P = x2 + x4 +

1
x

P x2 =

P 2 2 Px + x 4 = x 4 +

1
x

x4 +

1
x2

2 Px 4 P 2 x 2 + 1= 0

=
P 8P 0 P P3 8 0 P 2
2
4

1
=
, y 2 th P bng 2. Vy Min ca P bng 2.
2
Bi 6. Xt tam thc bc hai

Khi
=
x

f ( x) = x 2 (1 + a1 + a2 + ... + an ) x + a12 + a22 + ... + an2 .

Ta c
f (0) = a12 + a22 + ... + an2 0; f (1) = a1 ( a1 1) + a2 ( a2 1) + ... + an ( an 1) 0 .

Suy ra f (0). f (1) 0 x0 [ 0;1] sao cho f ( x0 ) = 0 tc f(x) c nghim.

V vy 0 (1 + a1 + a2 + ... + an ) 4 a12 + a22 + ... + an2 .


2

x1= x2= ...= xn 1


.
Bi 7. Bt ng thc ng vi mi s thc nn ng vi
xn = 2

Khi bt ng thc tr thnh ( n 1) + 22 2 ( n 1) n 5 1 n 5 .


Ta chng minh vi n=1,2,3,4,5 l cc gi tr cn tm.
Xt tam thc bc hai f ( xn ) = xn2 ( x1 + x2 + ... + xn 1 ) xn + x12 + x22 + ... + xn21 .
Ta c xn =

( x1 + x2 + ... + xn1 )2 4 ( x12 + x22 + ... + xn21 ) .

Do n 5 v theo bt ng thc CS ta c

4 x12 + x22 + ... + xn21 ( n 1) x12 + x22 + ... + xn21


( x1 + x2 + ... + xn 1 )

xn 0 f ( xn ) 0

iu chng t bt ng thc lun ng vi n = 1,5 . Vy n=1,2,3,4,5 l tt c


cc gi tr cn tm.
Bi 8. V tri vit li thnh tam thc bc hai ca x ta c

ax 2 ( ay + az + by bz + cz cy ) x + ayz + by 2 byz + cz 2 cyz .

( ay + az + by bz + cz cy )2 4a ( ayz + by 2 byz + cz 2 cyz ) .

Ta c =

(a

+ b 2 + c 2 2ab 2bc 2ca ( y z )

87

www.TaiLieuLuyenThi.com
Khm ph t duy K thut gii bt T Bi ton Max Min ng Thnh Nam

Ch a 2 + b 2 + c 2 < 2 ( ab + bc + ca ) 0 .
Bi 9. t b =
a + x; c =
a + y. Bt ng thc c vit li di dng:

(x

xy + y 2 a 2 x3 5 x 2 y + 4 xy 2 + y 3 a + x 4 3x3 y + 2 x 2 y 2 + y 4 0 .

V tri l tam thc bc hai ca a c h s ca a2 khng m.


Ch x 2 xy + y 2 = 0 x = y = 0 bt ng thc lun ng.
Vi x 2 xy + y 2 > 0 khi

a = x3 5 x 2 y + 4 xy 2 + y 3

=
3 x3 x 2 y 2 xy 2 + y 3

4 x 2 xy + y 2
2

)( x

3x3 y + 2 x 2 y 2 + y 4

iu chng t bt ng thc lun ng(pcm).


Bi 10. Thay b = 3 a c v a v chng minh

( 2c + 1) a 2 + ( 2c 2 5c 4 ) a +

f (a) =

Ch =
a

( 2c

5c 4

18 ( 2c + 1=
)

9
0.
2

( 2c 1)2 c ( c 4 ) 2 0, c [0;3] .

iu chng t bt ng thc lun ng.


3
1
ng thc xy ra khi v ch khi =
.
a
,=
b 1,=
c
2
2
Bi 11. Bt ng thc tng ng vi:
a 2 + b 2 + c 2 + 2abc + 3 1 + a + b + c + ab + bc + ca + abc .

a 2 + a ( bc b c 1) + b 2 + c 2 bc b c + 2 0 .
a 2 + a ( bc b c 1) + ( b c ) + bc b c + 2 0 .
2

Coi v tri bt ng thc trn l tam thc bc hai ca a ta c:


a=

( bc b c 1)2 4 ( b c )2 + bc b c + 2

= b 2 c 2 2b 2 c 2bc 2 3b 2 3c 2 + 4bc + 6b + 6c 7 .

) (

= b 2 c 2 2c 3 2 c 2 2c 3 b 3c 2 + 6c 7

Li coi y l tam thc bc hai ca b ta c:

(
) ( c 2c 3)( 3c + 6c 7 )
.
( c 2c 3)( c 2c 3 + 3c 6c + 7 )
4 ( c 2c 3) ( c 1) = 4 ( c 1) ( c + 1)( c 3)

'b = c 2 2c 3
=
=
88

www.TaiLieuLuyenThi.com
Cty TNHH MTV DVVH Khang Vit

TH1: Nu bc b c 1 0 ta c ngay iu phi chng minh.


TH2: Nu bc b c 1 0 ( b 1)( c 1) 2 ta xt hai kh nng
Kh nng 1. Nu c 3 c 2 2c 3 0, 'b 0 a 0 ta c ngay iu phi
chng minh.
2
Kh nng 2. Nu c 3 b 1
1 b 2 lc ny ta li coi a l tam
c 1
thc bc hai ca c v thc hin tng t trn ta c ngay 'c 0 a 0 ta c
ngay iu phi chng minh.
Bi 12. Vit li bt ng thc di dng:
2a 2 + a ( bc 5 ) + 2b 2 + 2c 2 5b 5c + 8 0 .

Coi v tri ca bt ng thc l tam thc bc hai ca a ta c:

( bc 5)2 8 ( 2b2 + 2c 2 5b 5c + 8)

a=

= b 2 c 2 16b 2 16c 2 10bc + 40b + 40c 39 .

= b 2 c 2 16 10b ( c 4 ) 16c 2 + 40c 39

Ta c: 2b 2 + 2c 2 5b 5c + 8 > 0, b, c (xt tam thc bc hai vi b hoc c).


TH1: Nu bc 5 ta c ngay iu phi chng minh.
TH2: Nu bc 5 li coi a l tam thc bc hai ca b ta c:

)(

'=
25 ( c 4 ) c 2 16 16c 2 + 40c 39
b
2

= 8 2c 4 5c3 24c 2 + 55c 28 = 8 ( c 1) ( c 4 )( 2c + 7 )


2

Kh nng 1. Nu c 4 c 2 16 0, 'b 0 a 0 ta c ngay iu phi


chng minh.
5 5
Kh nng 2. Nu c 4 b < 4 lc ny coi a l tam thc bc hai ca c
c 4
v thc hin tng t trn ta c 'c 0 a 0 ta c iu phi chng minh.
Vy bt ng thc c chng minh. ng thc xy ra khi a= b= c= 1 .

)(

)(

Bi 13. Ta cn chng minh: a 2 + 2 b 2 + 2 c 2 + 2 9 ( ab + bc + ca ) 0 .

) (

) (
)
2 ( c a + 1) + a b c

4 a 2 + b 2 + c 2 + 2 a 2b 2 + 1 + 2 b 2 c 2 + 1 +
2 2

2 2 2

+ 1 9 ( ab + bc + ca ) 0 .

S dng bt ng thc AM-GM ta c:

) (

) (

2 a 2b 2 + 1 + 2 b 2 c 2 + 1 + 2 c 2 a 2 + 1 + a 2b 2 c 2 + 2 4ab + 4bc + 4ca + 2abc + 1 .


89

www.TaiLieuLuyenThi.com
Khm ph t duy K thut gii bt T Bi ton Max Min ng Thnh Nam

Ta cn i chng minh.

(
3( a

4 a 2 + b 2 + c 2 + 4ab + 4bc + 4ca + 2abc + 1 9 ( ab + bc + ca ) 0 .

+ b 2 + c 2 ab bc ca + a 2 + b 2 + c 2 + 2abc + 1 2 ( ab + bc + ca ) 0

(lun ng) do
2

a 2 + b 2 + c 2 ab bc ca 0 v a 2 + b 2 + c 2 + 2abc + 1 2 ( ab + bc + ca ) 0 .

ng thc xy ra khi v ch khi a= b= c= 1 .

Bi tp tng t
Cho a,b,c l cc s thc khng m. Chng minh

abc + 3 a 2 + b 2 + c 2 + 11 7 ( a + b + c ) .

Bi 14. Vit li bt ng thc di dng


x 2 2 x ( cos B + cos C ) + 2 (1 cos A ) 0 .

Ta c =
' x ( cos B + cos C ) 2 (1 cos =
A ) 4sin 2
2

Do bt ng thc c chng minh.


Bi 15. Ch

)(

A 2 B C
1 0 .
cos
2
2

'x =
( sin y + cos y ) 1 + sin 2 y 1 + cos2 y = (1 sin y cos y ) < 0 .
2

Bi 16. Ch x =

3 y 1 0 dpcm .

Bi 17. Ch 'x =
( y sin C z cos B ) 0 .
2

x
sin A

ng thc xy ra khi v ch khi =

y
z
.
=
sin B sin C

Bi 18. Khng mt tnh tng qut gi


s z min { x, y, z} z 1. Khi
=
x=

4 yz
.
y + z + yz

Ta cn chng minh

4 yz
4 yz
+ y + z ( y + z ).
+ yz
y + z + yz
y + z + yz

1 + z z 2 y2 + z2 + z 4 y + ( z 2) 0
y=

90

(z

+ z4

4 1 + z z2

) ( z 2) =
2

z ( z 1) ( 5 z 8 ) 0
2

www.TaiLieuLuyenThi.com
Cty TNHH MTV DVVH Khang Vit

Bi ton c chng minh. ng thc xy ra khi v ch khi x= y= z= 1


hoc x= y= 2, z= 0 hoc cc hon v.(xem thm k thut phn chng).
Bi 19. t f ( x) = x 2 (a + c) x + ac = 0 c 2 nghim a,c
M: a b c f (b) 0 b 2 (a + c)b + ac 0
ac
y
a + c yb + ac ( a + c ) y
b
b
x
y
z

xa + ac + ( yb + ac ) + ( zc + ac ) ( a + c ) x + ( a + c ) y + (a + c) z
a
b
c

x y z
xa + yb + zc + ac + + ( a + c )( x + y + z )
a b c
Theo bt ng thc AM GM ta c
b+

( xa + yb + zc ) ac

x y z
+ + ( a + c )( x + y + z )
a b c

2
2
x y z
4 ( xa + yb + zc ) ac + + ( a + c ) ( x + y + z )
a b c

x y z (a + c)
( xa + yb + zc ) ac + +
( x + y + z )2 (pcm)
4ac
a b c
Bi 20. Thay d = 3 c vo biu thc v tri c tam thc bc hai ca c
2

f (c) =ac + b(3 c) + c(3 c) =c 2 c(b a 3) + 3b


3 + a b 12b + ( a b + 3)
f
=
2
4

Ta ch cn chng minh
12b + ( a b + 3)

9+6 2
6 ( a + b ) 2ab 6 2 1
4
4
2
6 ( a + b ) ( a + b ) 1 6 2 1
.

)(

a+b 2 a+b+6 2 0

Bt ng thc cui ng do a + b 2 a 2 + b 2 =2 .

Bt ng thc c chng minh. ng thc xy ra khi v ch khi


a= b=

1
3
.
, c= d=
2
2

91

www.TaiLieuLuyenThi.com
Khm ph t duy K thut gii bt T Bi ton Max Min ng Thnh Nam

Bi 21. Ta chng minh


P=

x
y
z
34
+
+

2 x + 3 y y + z z + x 33

( 31x 3 y ) z 2 + 96 y 2 35 x 2 5 xy z + 31x 2 y 3 xy 2 0

V tri l tam thc bc hai ca z vi h s ca z2 dng nn ta ch cn


chng minh

(96 y

) 4 (31x 3 y ) (31x y 3xy ) 0


( x y )( x 4 y ) (1125 x + 2631xy + 2304 y ) 0
2

35 x 2 5 xy

Bt ng thc cui ng do x y 0; x 4 y 0 .
Nhn xt. Vi cch lm ny ta ch cn gi thit bi ton l x y .
Bi 22. Vit li bt ng thc di dng:
a 2 + 2 ( b + d 3c ) a + ( b + c + d ) 8bd > 0 .
2

V tri l tam thc bc hai ca a t gi tr nh nht ti 3c b d gi tr nh


nht ny bng

( 3c b d )2 2 ( 3c b d )2 + ( b + c + d )2 8bd
2
2
= ( b + c + d ) ( 3c b d ) 8bd
=
8 c 2 ( b + d ) c + bd =
8 ( c b )( c d ) > 0
8 ( b + d c ) c 8bd =

Pmin =

Bt ng thc c chng minh.


Bi 23.

V p 2 + q 2 a 2 b 2 c 2 d 2 =

(p

) (

a 2 b 2 + q 2 c 2 d 2 > 0 nn

tn ti t nht mt trong hai s p 2 a 2 b 2 , q 2 c 2 d 2 dng. Khng mt


tnh tng qut ta gi s p 2 a 2 b 2 > 0 .
Xt tam thc bc hai
f ( x) =

(p

a 2 b 2 x 2 2 ( pq ac bd ) x + q 2 c 2 d 2 .

Ta c: p 2 . f (q ) =
( aq pc ) ( bq pd ) 0 nn f ( x) lun c nghim.
2

)(

Suy ra ' x 0 ( pq ac bd ) p 2 a 2 b 2 q 2 c 2 d 2 .
2

Ta c iu phi chng minh.


Tng qut vi n s thc tho mn iu kin a12 a22 ... an2 > 0 ta c

(a

2
1

92

)(

a22 ... an2 b12 b22 ... bn2 ( a1b1 a2b2 ... an bn ) .
2

www.TaiLieuLuyenThi.com
Cty TNHH MTV DVVH Khang Vit

Bi 24. Thay r = p q vo bt ng thc cn a v chng minh:

a 2 q ( p q ) + b 2 ( p q ) p + c 2 pq 0 a 2 q 2 + p c 2 b 2 a 2 q p 2b 2 0 .

Coi v tri bt ng thc trn l tam thc bc hai ca q ta c:

=
p 2 c2 b2 a 2
q

= p2 c2 ( a + b )

)(

2
4 p 2 a 2b=
p 2 c 2 b 2 a 2 2ab c 2 b 2 a 2 + 2ab

)(c

(a b)

)0

Do c > a b , c < a + b . Vy ta c iu phi chng minh.


Bi 25. Bt ng thc cho tng ng vi:

a 2b 2 + 2 a 2 + b 2 + 4 3ab 3 ( a + b ) 0

a 2 b 2 + 2 3a ( b + 1) + 2b 2 3b + 4 0

Ta ch cn chng minh rng

)(

a= 9 ( b + 1) 4 b 2 + 2 2b 2 3b + 4
2

=
( b 1) 8b 2 + 4b + 23 0
2

Bt ng thc cui ng.


Bt ng thc c chng minh. ng thc xy ra khi v ch khi a= b= 1 .
CH 7: K THUT NH GI BT NG THC TCH PHN
Di y ch trnh by mt s bi ton bt ng thc tch phn m phng php
gii ch thng qua cc bt ng thc i s ri ly tng tch phn s dn n kt
qu ca bi ton.
Bt ng thc tch phn l bi ton kh hay tm hiu thm bn c tm c
cc Chuyn khc v bt ng thc tch phn trn Din n hc tp trc tuyn
Mathlinks.vn
A. NI DUNG PHNG PHP

Bi ton: Cho f , g , h : [ a, b ] l cc hm kh tch trn [ a, b ] ng thi tha


mn iu kin g ( x) f ( x) h( x) vi mi x [ a, b ] ta c
b

g ( x)dx f ( x)dx h( x)dx .


Cc bt ng thc ph hay c s dng
sin x < x v x

x3
x3 x5
< sin x < x + , x > 0 .
3!
3! 5!
93

www.TaiLieuLuyenThi.com
Khm ph t duy K thut gii bt T Bi ton Max Min ng Thnh Nam

cos x > 1

x2
x2
x2 x4
v 1
< cos x < 1
+ , x > 0 .
2
2
2 4!

x
< ln (1 + x ) < x .
x +1
Yu cu. Cc bt ng thc i s nh C si, Cauchy Schwarz, cn nm vng.
Nh vy dng bt ng thc c bn ca tch phn chnh l quy bi ton v nh
gi bt ng thc trong du tch phn.

B. BI TP MU
b

Bi 1. [IMC 2011] Chng minh rng

(x

+ 1 e x dx e a e b

thc a < b .
Li gii
Vi mi s thc x ta lun c x + 1 2 x suy ra
2

(
a

x 2 + 1 e x dx 2 xe x dx =
e a e b .
a

Bi ton c chng minh hon ton.


1

Bi 2. Chng minh rng

x sin x

1 + x sin x dx < 1 ln 2 .
0

Li gii
1

Vit li bt ng thc cn chng minh di dng:

dx

1 + x sin x > ln 2 .
0

dx
dx

>
=
>ln2.
2
1 + x sin x 0 1 + x
4
0

Vi mi x > 0 sin x < x

Bi ton c chng minh hon ton.


sin x
x , x ( 0;1]
.
Bi 3. Cho hm s f ( x) c xc nh bi f ( x) =
sin
x
lim
,x = 0
x0+ x
1

17
1703
Chng minh rng
.
< f ( x)dx <
18 0
1800

94

vi mi s

www.TaiLieuLuyenThi.com
Cty TNHH MTV DVVH Khang Vit

Li gii
R rng vi gi thit hm s f ( x) cho th f ( x) lin tc trn [ 0;1] .
Tip n s dng bt ng thc ph x

x3
x3 x5
< sin x < x +
, x > 0 , ta
6
6 120

x 2 sin x
x2 x4
.
<
<1
+
6
6 120
x
Chuyn qua tch phn hai v ca bt ng thc trn ta c

c 1

1
1
1
x2
x2 x4
sin x
17
sin x
1703

<
<

<
.
1
dx
dx
1
dx
dx <
6 x
6 120

18
x
1800

0
0
0
0
Bi ton c chng minh hon ton.
1

Bi 4. Chng minh rng

4
1
1
vi mi s nguyn dng
< tan n xdx <
2 ( n + 1) 0
2 ( n 1)

n2.

Li gii

t an = tan n xdx vi mi 0 x
0

0 tan x 1 tan n x tan n 1 x

an an 1 tc dy {an } gim.

Suy ra an + 2 + an < 2an < an + an 2 .

Mt khc ta li c an + 2=
+ an

n
2
x dx
tan x 1 + tan =
0

V an=
2 + an

tan
0

T suy ra

n2

1
1
x 4
tan n=
n +1
n +1
0

1
1
2
.
x 1 + tan=
x dx
x 4
tan n=
n 1
n 1
0
2

1
1
.
< an <
2 ( n + 1)
2 ( n 1)

Vy

4
1
1
< tan n xdx <
, n 2 .
2 ( n + 1) 0
2 ( n 1)

T chng minh trn suy ra lim n tan n xdx =


n

1
.
2
95

www.TaiLieuLuyenThi.com
Khm ph t duy K thut gii bt T Bi ton Max Min ng Thnh Nam
4

Bi 5. [VMC 1994] Xt tch phn I n =

4 xdx, n * .

a) Tnh I n .
b) Chng minh rng I n < 22 n +3 ( 2ne )

1
2

Li gii
Ta c

I=
n

2
2

= 22 n + 4 cos 2 n +1t sin 2 tdt= 22 n + 4 cos 2 n +1tdt cos 2 n +3tdt


4 xdx
0

0
0


vi x 4cos 2 t , t 0, .
=
2

=
Xt tch
phn J m

cos

tdt , m > 1 .

Tch phn tng phn ta c:


=
Jm

m
tdt
cos =

m 1
t)
cos td ( sin=

( m 1) sin

tcos m 2tdt

2
2

=
( m 1) cosm2tdt cosmtdt
0

m 1
c J1
J m = J m 2 . Ta=
m

cos tdt
=

1 suy ra

2n 2 2
2n + 2 2n
2
2 n + 4 2n
=
I n 22 n + 4 ( J 2 n +1 J 2 n=
.
... .1
.
... .1
+3 ) 2

2n + 3 2n + 1 3
2n + 1 2n 1 3

22 n + 4

2n ( 2n 2 ) ...2 2n + 2
2n ( 2n 2 ) ...2
2n+4
.
=
1
2
( 2n + 1)( 2n 1) ...3 2n + 3
( 2n + 3)( 2n + 1)( 2n 1) ...3

Chng minh I n < 22 n +3 ( 2ne )

96

1
2

www.TaiLieuLuyenThi.com
Cty TNHH MTV DVVH Khang Vit

t x = 4t. =
Khi I n 2

2 n +3

1 tdt ta chng minh t n 1 t <

1
hay
2ne

1
.
2ne
S dng bt ng thc AM GM ta c:
t 2 n (1 t ) <

2 n +1

t
t
t

+ + ... + + 1 t

t
2n
2n

n
2=
=
t 2 n (1 t ) ( 2n ) (1 t ) ( 2n ) 2n 2n

+
n
n
2
2
1

Nh vy ta chng minh

( 2n )2 n .
( 2n + 1)2n+1

2n

( 2n )2n < 1 e < 2n + 1 2n+1 1 < 2n + 1 ln 1 + 1 .


(
)

2n
2n
( 2n + 1)2n+1 2ne
D dng chng minh c ln (1 + x ) >

x
, x > 0 .
x +1

T ta c iu phi chng minh.


2

Bi 6. Chng minh rng

sin nx
x

dx >

2 1
1
1
+
+ ... + .

1+ n 2 + n
2n

Li gii
2

2 n

sin nx
I n =
t t = nx. Khi
dx
=
x

sin t
=
t dt
n

2 n 1 ( k +1)


k =n

sin t
t

dt .

( k +1)

2 n 1
1
2
( k + 1) sin t dt =
( k + 1) .
k n=
k n
=
k

>

2 n 1

Bi ton c chng minh hon ton.

Bi 7. Chng minh rng

6n 2 + 2n + 1
6n

< n 1 + xdx <


0

2n + 1
vi mi n , n 2 .
2n

Li gii
Ta s chng minh
1+

x n ( n 1) 2 n
x

x 1 + x 1 + vi mi x 0, n *
2
n
n
2n

(1.1)

Khi ly tch phn hai v ca (1.1) trn [ 0,1] ta c iu phi chng minh.

97

www.TaiLieuLuyenThi.com
Khm ph t duy K thut gii bt T Bi ton Max Min ng Thnh Nam

x
x2 1
1 2n + 1

1
=
Tht vy ta c 1 + dx =
v
x + =+
n
2n 0
2n
2n

0
1

3
2

x 2 ( n 1) x 1 6n + 3n ( n 1) 6n 2 + 2n + 1
.

=
=
x+

2n
6n 2 0
6n 2
6n 2

Vic chng minh bt ng thc (1.1) ch vic kho st hm s.


Bnh lun: V sao li gii ngh n vic chng minh bt ng thc (1.1)?.
y l mt kinh nghim khi gii ton, l da vo khai trin Taylor ca hm
1

x n 1 2
1 + n 2n2 x dx =
0

s f ( x=
)
n

1 + x ti x = 0 ta c

1 + x =1 +

x n ( n 1) 2 n ( n 1)( n 2 ) 3

x +
x + ...
n
2n 2
6n3

Cng t tng ny vi cos


x
=

1 ix
e + e ix
2

v =
sin x

1 ix
e e ix
2i

v khai

trin Taylor cho hm e x v ta c cc bt ng thc sau y.


x2
x2 x4
x3
x3 x5
v 1
vi mi x > 0 .
< cos x < 1
+
< sin x < x +
2
2 24
6
6 120
Lc ny ch vic ly tch phn hai v trn [ a, b ] , ( 0 < a < b ) ta c cc bt ng
x

thc ht sc th v.
2

Bi 8. Chng minh rng

sin x 2 dx > 0 .

Li gii
Trc tin ta chng minh: sin x x

x3
, x 0
6

Tht vy xt hm s f ( x=
) sin x x +

x3
ta c
6

x2
f ''( x) = sin x + x f '''( x) =1 cos x 0 .
2
Suy ra f ''( x) f ''(0) =
0 f '( x) f '(0) =
0 f ( x) f (0) =
0.
f '( x) =cos x 1 +

Tc l sin x x

x3
, x 0 .
6

Suy ra
2

98

sin x dx
2

2 x6
x3 x7 2 2 2 4 3 2 2 2 2
=

=
x dx =
1 > 0 .
6
3
21
3
7

3 42 0

www.TaiLieuLuyenThi.com
Cty TNHH MTV DVVH Khang Vit

Bi 9. Chng minh rng vi mi n nguyn dng ta lun c


1

xn + 1

xn + xn1 + ... + x + 1 dx n + 1 .
0

Li gii

)(

Vi mi k [ 0, n ] v x [ 0,1] ta c 1 x n 1 x n k 0 x k + x n k x n + 1 .
Cho k = 0,1, 2,..., n v cng theo v ca n + 1 bt ng trn theo v, suy ra

2 x n + x n 1 + ... + x + 1 ( n + 1) x n + 1
1

Suy ra

x +1
n

xn + 1
x +x
n

n 1

+ ... + x + 1

2
.
n +1

.
xn + xn1 + ... + x + 1 dx n + 1 dx =
n +1
0

Bi ton c chng minh hon ton.


Bi 10. Tm gi tr nh nht ca biu
thc I n
=

3
8

sin n + 2 x cos n + 2 x
+

dx .
n
sin n x
cos x

Li gii
3
Vi mi x , cos x > 0,sin x > 0 khi ta c
4 8

2( n +1)
2 n +1
n
x + cos ( ) x
sin n + 2 x cos n + 2 x 2 sin
2 n +1
2 n +1
=
+
2n sin ( ) x + cos ( ) x .
n
n
n
cos x
sin x
sin 2 x

Mt khc hm f (n) = a n +1 l hm li nn ta c

) (

2 n +1
2 n +1
f sin 2 x + f cos 2 x = sin ( ) x + cos ( ) x

cos 2 x + sin 2 x
cos 2 x + sin 2 x
2f
= 2

2
2

n +1

1
2n

Suy ra

2 n +1
2 n +1
1 In
2n sin ( ) x + cos ( ) x =

3
8

3
8
sin n + 2 x cos n + 2 x

.
+
=

dx
dx

n
n

8
sin x

cos x

Bi 11. Vi mi s thc ai , i = 1, 2,..., n . Chng minh rng

ai a j

1i n ,1 j n i +

j +1

0.
99

www.TaiLieuLuyenThi.com
Khm ph t duy K thut gii bt T Bi ton Max Min ng Thnh Nam

Li gii
2

c f ( x) =
Xt a thc f ( x) = ai x ta =
ai xi
=
i =1
i 1

1i n,1 j n

ai a j xi + j .

n y ly tch phn hai v trn [ 0,1] ta c iu phi chng minh.


C. BI TP RN LUYN
Bi 1. Chng minh rng
0

a) 2 3 <

b)

3x 2
6 x + 5dx < 34 .
x
2
3

dx
4 x x
2

2
8

d)

sin 2 x

dx >

e)

1
2

1
dx
f)

, n * .
2 0 1 x 2n 6
Bi 2. Chng minh rng vi mi x > 0 ta lun c
c)

e x 1 < e 2t + e t dt <
0

1 2 sin x
ln ln 2
.
dx

2
2cos1 1 x
3 x

x +1

( e 1) e
x

sin x

5 2 sin x
3
.
<
dx <
6 x
2
6
e

Bi 4. Chng minh rng

( ln x )2013 dx >
x

1
.
2014.2015.2016

( cos x )sin x
( cos x )sin x + ( sin x )cos x dx < 1 .
0
2

Bi 5. Chng minh rng

Bi 6. Cho 0 < a b chng minh rng


b

a)

( x + 2 )5
a
b

b)

100

x2 + x + 1

dx

x4 + x2 + 1

(x

+ 3x + 2

1 2b + 1
.
ln
54 2a + 1

dx
5

( 2b 1)( 2a + 1) .
1
ln
2916 ( 2a 1)( 2b + 1)

dx

1
.
2

Bi 3. Chng minh rng

3
.
2

12e

www.TaiLieuLuyenThi.com
Cty TNHH MTV DVVH Khang Vit
1

n
e2
e 1
Bi 7. Chng minh rng 1 +
e x dx 1 +
vi n * .
n
n
0

Bi 8. Chng minh rng

1 + x 2 ln 2 ( x + 1)dx

17
.
16

Bi 9. Vi mi s nguyn dng n . Chng minh rng

tan

xdx >

.
2n + 4

Bi 10. Chng minh rng

<

1
1 k sin x
2

dx <

2 1 k2

vi mi k ( 0,1) .

n 2 2
sin nx
<
dx
vi n 1 .
Bi 11. Chng minh rng x

sin x
8
0
4

Bi 12. Vi mi s nguyn dng n 2 .


1

n
n+2
e+n
< e x dx <
Chng minh rng
.
n +1 0
n +1
1

Bi 13. Vi k 0 chng minh rng

(x x

2 k

dx

2
4

k + 1 + 2k + 1

1
,n 2 .
2ln 2n
Bi 14. Cho cc s thc dng a1 , a2 ,..., aN .
In >

Chng minh rng

a a

m +mn n+ 1 ak2 .

m
n 0
= 0=

k 1
=

101

www.TaiLieuLuyenThi.com
Khm ph t duy K thut gii bt T Bi ton Max Min ng Thnh Nam

Chng 2:
BT NG THC V PHNG PHP TIP CN
CH 1:

CC K THUT S DNG BT NG THC AM GM C BN


A. NI DUNG PHNG PHP
I. Bt ng thc C si (AM-GM) cho 2 s khng m
a+b
ab .
2
ng thc xy ra khi v ch khi a = b.
Cc dng tng ng ca bt ng thc trn
2
(a + b) .
a+b
2
2
ab
v a + b
2
2
V d 1. Chng minh rng vi mi s thc dng a v b ta c
1 1
4
.
+
a b a+b
Li gii. S dng bt ng thc AM-GM cho 2 s ta c
2

( a + b )

1 1
2
+ 2 ab .
=
4.
a
b
ab

1 1
4
.
+
a b a+b
ng thc xy ra khi v ch khi a = b .
II. Bt ng thc C si (AM-GM) cho 3 s khng m
a+b+c 3
abc .
3
ng thc xy ra khi v ch khi a= b= c .
Cc dng tng ng ca bt ng thc trn

Do

a+b+c
abc
.
3

V d 2. Chng minh rng vi mi s thc dng a,b,c ta c


1 1 1
9
.
+ +
a b c a+b+c
Li gii. S dng bt ng thc AM-GM cho 3 s dng ta c
102

www.TaiLieuLuyenThi.com
Cty TNHH MTV DVVH Khang Vit

( a + b + c )

1 1 1 3
3
+ + 3 abc . 3
=
9.
abc
a b c

1 1 1
9
.
+ +
a b c a+b+c
ng thc xy ra khi v ch khi a= b= c .
III. Bt ng thc C si (AM-GM) cho n s khng m
a1 + a2 + ... + an n
a1a2 ...an .
n
ng thc xy ra khi v ch khi a1= a2= ...= an .

Do

1. Cc bt ng thc hay s dng


Trc tin nhc li mt s bt ng thc hay c s dng
1
2
2
a 2 + b 2 ( a + b ) ; ( a + b ) 4ab
2
1
2
a 2 + b 2 + c 2 ab + bc + ca; a 2 + b 2 + c 2 ( a + b + c )
3

( a + b + c )2 3 ( ab + bc + ca ) ; ( ab + bc + ca )2 3abc ( a + b + c )
8
9

( a + b )( b + c )( c + a ) ( a + b + c )( ab + bc + ca )
a 2b + b 2 c + c 2 a + abc

(1)

4
( a + b + c )3 (2)
27

Chng minh (1) v (2) xem bn di


2. K thut nh gi trung bnh cng qua trung bnh nhn
Gi thit bi ton thng cho tch ca cc s
S dng bt ng thc AM GM cho n s dng
a1 + a2 + ... + an n
a1a2 ...an .
n
V d 1. Chng minh vi mi a,b,c l cc s thc ta c

(a

)(

)(

+ b 2 b 2 + c 2 c 2 + a 2 8a 2 b 2 c 2 .

Li gii
Ta c
a 2 + b 2 2 a 2b 2 =
2 ab
b2 + c 2 2 b2c 2 =
2 bc
c2 + a2 2 c2a2 =
2 ca

Nhn theo v 3 bt ng thc trn ta c iu phi chng minh.


103

www.TaiLieuLuyenThi.com
Khm ph t duy K thut gii bt T Bi ton Max Min ng Thnh Nam

V d 2. (TSH Khi D 2005) Cho x,y,z l cc s thc dng tho mn iu kin


xyz = 1. Chng minh rng

1 + x3 + y 3
1 + y3 + z3
1 + z 3 + x3
+
+
3 3.
xy
yz
zx

Li gii
S dng bt ng thc AM GM cho 3 s dng ta c
3 3 x3 y 3 .1
1 + x3 + y 3
3

=
xy
xy
xy
3 3 y 3 z 3 .1
1 + y3 + z3
3

=
yz
yz
yz
1 + z 3 + x3
3 z 3 x3 .1
3

=
zx
zx
zx
3

Cng theo v 3 bt ng thc trn ta c


1
1 + x3 + y 3
1 + y3 + z3
1 + z 3 + x3
1
1
+
+
3
+
+
.

xy
yz
zx
yz
zx
xy

Mt khc AM GM cho 3 s dng ta c


1
1
1
1
1
1
.
.
3.
+
+
33
=
xy
yz
zx
xy yz zx

T suy ra iu phi chng minh. ng thc xy ra khi v ch khi x= y= z= 1 .


V d 3. Cho a,b,c l cc s thc dng c tng bng 3.
Chng minh rng

a ( b + c ) + b ( c + a ) + c ( a + b ) 3 2abc .

Li gii
S dng bt ng thc AM GM ta c
a ( b + c ) + b ( c + a ) + c ( a + b ) 36 abc ( a + b )( b + c )( c + a ) .
3

Mt khc

a+b+c
abc
1 .
=
3

( a + b )( b + c )( c + a ) 8abc

Suy ra
a ( b + c ) + b ( c + a ) + c ( a + b ) 3 8a 2b 2 c 2 3 8a 3b3c3 =
3 2abc .
6

Bt ng thc c chng minh. ng thc xy ra khi v ch khi a= b= c= 1 .


104

www.TaiLieuLuyenThi.com
Cty TNHH MTV DVVH Khang Vit

V d 4. Cho a,b,c l cc s thc dng tho mn iu a > b; a + b + c =


4.
Tm gi tr nh nht ca biu thc P = 4a + 3b +

c3
.
(a b)b

Li gii
Ch iu kin a + b + c= 4, a > b, c3 ta s tch P thnh tng ca cc s dng
s dng AM GM sao cho mt i (a-b) v xut hin tng (a+b+c) v phi.
S dng bt ng thc AM GM cho ba s dng ta c
P = 4a + 3b +

c3
=
(a b)b

33 ( a b ) .b.

(a b) + b +

c3
+ 3( a + b )
(a b)b

c3
+ 3 ( a + b )= 3 ( a + b + c )= 12
(a b)b

c3
a b = b =
ng thc xy ra khi v ch khi
( a b ) b a = 2; b = c =1

4
a + b + c =

Vy gi tr nh nht ca P bng 12.


Bi tp tng t
Cho a > b > c > 0 chng minh a +

108

( a b )3 ( b c )2 c

7 .

V d 5. (IMO 2012) Cho a2 , a3 ,..., an l cc s thc dng tho mn iu kin


a2 a3 ...an = 1 . Chng minh rng (1 + a2 ) (1 + a3 ) ... (1 + an ) > n n .
3

Li gii
S dng bt ng thc AM GM ta c
k

( ak + 1)k

1
1
1
1
k
= ak +
+
+ ... +
, k =2, n .
k .ak .

k
k
k
1
1
1

( k 1)k 1


( k 1) times

ng thc xy ra khi v ch khi


=
ak
Khi a2 a=
3 ...an

1
=
, k 2, n .
k 1

1
= 1, v l. Vy du bng khng xy ra khi
(n 1)!

Nhn theo v (n 1) bt ng thc trn ta c:

(1 + a2 )2 (1 + a3 )3 ...(1 + an )n >

22 33
nn
.
...
.a a ...an =
nn .
1
2
n 1 2 3
1 2 (n 1)
105

www.TaiLieuLuyenThi.com
Khm ph t duy K thut gii bt T Bi ton Max Min ng Thnh Nam

Bt ng thc c chng minh.


V d 6. Cho x,y,z l cc s thc chng minh
1 ( x + y )( y + z )( z + x )(1 xy )(1 yz )(1 zx ) 1

.
2
2
2
8
8
1 + x2 1 + y 2 1 + z 2

)(

)(

Li gii
S dng ng thc v bt ng thc AM GM cho hai s ta c

(1 + x )(1 + y ) =( x + y ) + ( xy 1) 2 ( x + y )( xy 1)
(1 + y )(1 + z ) =( y + z ) + ( yz 1) 2 ( y + z )( zyz 1)
(1 + z )(1 + x ) =( z + x ) + ( zx 1) 2 ( z + x )( zx 1)
2

Nhn theo v ba bt ng thc trn ta c

8 ( x + y )( y + z )( z + x )(1 xy )(1 yz )(1 zx ) 1 + x 2

) (1 + y ) (1 + z )
2

2 2

1 ( x + y )( y + z )( z + x )(1 xy )(1 yz )(1 zx ) 1


8
8
2 2
2 2
2 2
1+ x
1+ y
1+ z

)(

)(

Bt ng thc c chng minh.


Bi tp tng t

1 ( x + y )(1 xy ) 1
Cho x,y l hai s thc. Chng minh
.
2 1 + x2 1 + y 2
2

)(

V d 7. Cho x,y,z l cc s thc dng tha mn iu kin x max{ y, z}.


Tm gi tr nh nht ca biu thc P =

x
y
z
+ 2 1 + + 33 1 + .
y
z
x

Li gii
S dng bt ng thc AM GM ta c
P

x
y
z
y
z x
+2 2
+ 33 2
=
+ 2 2 4 + 33 2 6
y
z
x y
z
x

2x
2x
y
z
+ 4 4 + 6 6 + 1
3
2 y
2 y
z
x
S dng bt ng thc AM GM ta c

232

x4 y 6 z
11 .


=
y z x
Ch gi thit bi ton ta c
x
y
z
+ 4 4 + 6 6 1111
y
z
x

106

) 6 xz

2 2

www.TaiLieuLuyenThi.com
Cty TNHH MTV DVVH Khang Vit

2x
2
x
z
; 3
1; 1 1
1
2 y
2
y
x

232

) 6 xz 3(

232 .

2
2
3
3
+ 1
3 2 2 =1 + 2 2 + 3 2 .
2
2
Du bng xy ra khi v ch khi x= y= z .

T suy ra P 11.

Vy gi tr nh nht ca P bng 1 + 2 2 + 3 3 2 .
Bi tp tng t
Cho x,y,z l cc s thc dng tha mn iu kin x max{ y, z} .
x
y
z
Tm gi tr nh nht ca biu thc P = + 2 1 + + 4 4 1 + .
y
z
x

3. nh gi trung bnh nhn qua trung bnh cng


S dng bt ng thc AM GM theo chiu

a1a2 ...an

a1 + a2 + ... + an
.
n

Ta c 2 hng x l
+ nh gi trc tip.
+ Nhn thm hng s mc ch lc b bin hoc hng s khng thch hp.
Nhn dng bt ng thc c dng:

A1 + m A2 + ... + m An B .

Khi p dng bt ng thc AM GM theo chiu trung bnh nhn qua trung
bnh cng cho mi cn thc v tri.
V d 1. Chng minh rng vi mi a, b 1 ta c a b 1 + b a 1 ab .
Li gii
Nhn xt. V phi khng cha hng s do vy s dng AM GM trit tiu cc
s 1 trong 2 cn thc do nhn thm vo mi cn vi 1.
S dng bt ng thc AM GM ta c
b 1 + 1 ab
a b =
1 a ( b 1) .1 a.
=
2
2
a 1 + 1 ab
b a =
=
1 b ( a 1) .1 b.
2
2
Cng theo v 2 bt ng thc trn ta c iu phi chng minh.
ng thc xy ra khi v ch khi a= b= 2 .
Bi tp tng t

1) Chng minh rng x 12 y + y 12 x 2 12 vi x, y l cc s thc cc


cn thc c ngha.
2) Cho x,y l hai s thc tho mn iu kin x 1 y 2 + y 1 x 2 =
1.
107

www.TaiLieuLuyenThi.com
Khm ph t duy K thut gii bt T Bi ton Max Min ng Thnh Nam

Chng minh rng : x 2 + y 2 =


1.
0 x 3
V d 2. Cho
. Tm gi tr ln nht ca biu thc
0 y 4

P=
( 3 x )( 4 y )( 2 x + 3 y ) .

Li gii
Nhn xt. Ta cn trit tiu i x v y v vy nhn thm vo (3 x) v (4 y ) ln
lt thnh 2 ( 3 x ) ,3 ( 4 y ) khi s dng AM GM cho 3 s dng.
S dng bt ng thc AM GM cho 3 s khng m ta c
1
P = 2 ( 3 x ) . 3 ( 4 y ) . ( 2 x + 3 y )
6
1 2 (3 x ) + 3( 4 y ) + ( 2 x + 3 y )

36
=
6
3

ng thc xy ra khi v ch khi=


x 0,=
y 2 . Vy gi tr ln nht ca P bng 36.
Bi tp tng t
5
Cho 0 x ,0 y 3 . Tm gi tr ln nht ca biu thc
2
P =( 5 2 x )( 3 y )( 2 x + 3 y 5 ) .

V d 3. Cho a,b,c l cc s thc khng m tha mn iu kin a + b + c =


1.
Tm gi tr ln nht ca biu thc P = ( a + 2b + 3c )( 6a + 3b + 2c ) .
Li gii
Vi cc h s lch nhau trc tin gim bin bng php th c =1 a b. Khi
a v P =( 3 2a b )( 2 + 4a + b ) .
Ta thy nu p dng trc tip AM GM cho hai s trit tiu c b tuy nhin
cn d 2a khng nh gi c v vy ngh n trit tiu a bng cch nhn thm
2 vo (3 2a b) .
Ta c: P =

( a + 2b + 3 (1 a b ) ) ( 6a + 3b + 2 (1 a b ) )

=( 3 2a b )( 2 + 4a + b ) =2 ( 3 2a b ) 1 + 2a +
2

b
b

3 2a b + 1 + 2a + 2
4 2
2.
8
2
=
=
2
2

Vy gi tr ln nht ca P bng 8 t ti a= c=
108

1
, b= 0 .
2

www.TaiLieuLuyenThi.com
Cty TNHH MTV DVVH Khang Vit

V d 4. Cho a,b,c l cc s thc khng m c tng bng 3.


Tm gi tr ln nht ca biu thc P =
(5a + b)(b 2 + 4ac) .
Li gii
b
b
b
Ta c: 5a + b 5(a + ); b 2 + 4ac =b 2 + 2(ab + bc) + 4ac =4(a + )(c + ) .
2
2
2
S dng bt ng thc AM GM ta c:
b
2a + 2b + 2c 3
P 10(a + ) 2 (2c + b) 10(
) =
80 .
2
3
Du bng xy ra khi =
b 0;=
a 2=
c 2.
V d 5. Chng minh rng vi mi s thc dng a v b ta c

) (

2a ( a + b ) + b 2 a 2 + b 2 3 a 2 + b 2 .
3

Li gii
Trc ht chuyn bt ng thc v dng mt bin bng cch chia hai v bt
ng thc cho b 2 v a v chng minh bt ng thc:

) (

2 x ( x + 1) + 2 x 2 + 1 3 x 2 + 1 .
3

Ch du bng t ti x = 1 v bc v phi l bc hai do vy ta cn tch


2 x ( x + 1) = 2 x ( x + 1) . ( x + 1) s dng bt ng thc AM GM thun tin
3

cho vic so snh vi v phi.


S dng bt ng thc AM GM ta c
2 x ( x + 1)=
3

2 x ( x + 1) . ( x + 1)
2

2 x ( x + 1) + ( x + 1)

2 + x +1
2
Cng theo v hai bt ng thc trn ta c
2

2 x2 + 1

2 x ( x + 1) + 2 x 2 + 1
3

( x + 1)2 + 2 x ( x + 1) + x 2 + 3 .
2

Vy ta ch cn chng minh
2
x + 1) + 2 x ( x + 1) + x 2 + 3
(

( )
6 ( x + 1) 4 x
3 x +1
2

+ 4 x + 4 2 ( x 1) 0
2

Bt ng thc cui ng v ta c pcm.


ng thc xy ra khi v ch khi a = b .
109

www.TaiLieuLuyenThi.com
Khm ph t duy K thut gii bt T Bi ton Max Min ng Thnh Nam

V d 6. Cho a,b,c l cc s thc dng chng minh

81abc a 2 + b 2 + c 2 ( a + b + c ) .
5

Li gii
S dng bt ng thc 3abc ( a + b + c ) ( ab + bc + ca ) .
2

Ta ch cn chng minh 27 a 2 + b 2 + c 2 ( ab + bc + ca ) ( a + b + c ) .
2

Tht vy s dng bt ng thc AM GM cho ba s dng ta c


3

a 2 + b 2 + c 2 + 2 ( ab + bc + ca )
1
a + b + c ( ab + bc + ca )
=

( a + b + c )6 .

27
Bt ng thc c chng minh. ng thc xy ra khi v ch khi a= b= c .
Bi tp tng t
Cho a,b,c l cc s thc dng c tch bng 1. Chng minh

a + b + c 5 a 2 + b2 + c2
.

3
3
V d 7. Cho a,b,c l cc s thc dng. Chng minh rng
a+b 3
a+b a+b+c
.
+ abc 33 a.
.
2
2
3
Li gii
Bt ng thc cho tng ng vi
a + 3 ab.

6a 2
3b
6bc
+3
+3
3.
( a + b )( a + b + c ) a + b + c ( a + b )( a + b + c )

S dng bt ng thc AM GM cho ba s dng ta c


3

6a 2
=
( a + b )( a + b + c )

3b
3
=
a+b+c
3

3 1.1.

3 1.

2a
3a
1
2a
3a
.
1 +
+

a + b a + b + c 3 a + b a + b + c

3b
1
3b
1 + 1 +

a + b + c 3
a+b+c

6bc
=
( a + b )( a + b + c )

3 1.

2b
3c
1
2b
3c
.
1 +
+

a + b a + b + c 3 a + b a + b + c

Cng theo v ba bt ng thc trn ta c pcm. ng thc xy ra khi v ch khi


a= b= c .
Bi tp tng t
1) Cho a,b,c l cc s thc dng. Chng minh

110

www.TaiLieuLuyenThi.com
Cty TNHH MTV DVVH Khang Vit

a + 3 ab + 3 abc 3 a + b a + b + c
.
a.
.
3
2
3
2) Cho a,b,c,d l cc s thc dng. Chng minh

3a
bc
2b3 d
25
.
+ 33
+ 44

3
a+b+c
( a + b )( a + b + c + d )
81( a + b ) ( a + b + c + d ) 6

V d 7. Chng minh rng vi mi x,y,z khng m ta c


y3 + z3
z 3 + x3
x3 + y 3 ( x + y ) + ( y + z ) + ( z + x )
.
+ zx 3
+ xy 3

2
2
2
8
Li gii
S dng bt ng thc AM GM ta c
3

yz 3

y3 + z3
=
yz 3
2

y+z
. yz. yz. yz. y 2 yz + z 2
2

3
y + z 3 yz + y 2 yz + z 2
y+z
.
=

2
4
2

3
3
z 3 + x3 z + x
3 x + y x+ y .

;
xy

2
2
2
2
Cng theo v ba bt ng thc trn ta c pcm. ng thc xy ra khi v ch khi
x= y= z .

Tng t ta c zx 3

V d 8. Cho a,b,c l cc s thc khng m chng minh rng

( a + b )2 ( b + c )2 ( c + a )2 4 ( a 2 + bc )( b2 + ca )( c 2 + ab ) .
Li gii

Khng mt tnh tng qut gi s a b c. Khi a 2 + bc a 2 + ac ( a + c ) .


2

)(

Vy ta chng minh ( a + b ) ( b + c ) 4 b 2 + ca c 2 + ab .
2

Tht vy s dng bt ng thc AM GM ta c

)(

) (
= (b

4 b 2 + ca c 2 + ab b 2 + ca + c 2 + ab

+ ab + bc + ca + c 2 bc

( ( b + a )( b + c ) + c ( c b ) )

(b + a ) (b + c )
2

Bt ng thc c chng minh. ng thc xy ra khi v ch khi =


a b=
,c 0
hoc cc hon v.
111

www.TaiLieuLuyenThi.com
Khm ph t duy K thut gii bt T Bi ton Max Min ng Thnh Nam

Bi tp tng t
Cho a,b,c l cc s thc khng m tho mn iu kin
2
( a + b )( b + c )( c + a ) =

)(

)(

Tm gi tr ln nht ca biu thc P =


a 2 + bc b 2 + ca c 2 + ab .
V d 9. Cho a, b, c l cc s thuc on [0, 1].
Tm gi tr ln nht ca biu thc A = ( a b )( b c )( c a )( a + b + c ) .
Li gii

Gi s a = max {a, b, c} .
Nu a b c A 0 .
Nu a c b .

2(c b) (1 + 3)(a c) ( 1 + 3)(a + b + c)


.
4
S dng bt ng thc AM-GM cho ba s dng ta c :

Khi A

2(c b) (1 + 3)(a c) (1 + 3)(a + b + c)

2 ( c b ) + 1 + 3 ( a c ) + 1 + 3 ( a + b + c )
.

3b + 2 3a + 3b 2 3 8 3
=

=

3
9

3
1
2 3
ti (a, b, c) = (1,0, ) .
9
3
Ch . Ta c th m rng on [0;1] v gii bi ton bng tnh n iu ca hm
s(xem chng 3).
4. K thut ghp cp
Xem chi tit trong ch sau
V d 1. Cho a,b,c l cc s thc dng c tng bng 1. Chng minh
a + bc b + ca c + ab
+
+
2.
b+c
c+a
a+b
Li gii
Ch iu kin tng bng 1 ta c
a + bc b + ca c + ab
+
+
=
b+c
c+a
a+b
a ( a + b + c ) + bc b ( a + b + c ) + ca c ( c + a + b ) + ab
=
+
+
b+c
c+a
a+b
( a + b )( a + c ) + ( b + c )( b + a ) + ( c + a )( c + b )
=
b+c
c+a
a+b

Suy ra Pmax =

112

www.TaiLieuLuyenThi.com
Cty TNHH MTV DVVH Khang Vit

S dng bt ng thc AM GM cho 2 s dng ta c


( a + b )( a + c ) + ( b + c )( b + a ) 2 a + b
(
)
b+c
c+a
( b + c )( b + a ) + ( c + a )( c + b ) 2 b + c
(
)
c+a
a+b
( a + b )( a + c ) + ( c + a )( c + b ) 2 c + a
(
)
b+c
a+b
Cng theo v 3 bt ng thc trn ta c iu phi chng minh. ng thc xy ra
1
khi v ch khi a= b= c=
.
3
V d 2. Cho a,b,c l cc s thc dng. Chng minh
1 + a2
1 + b2
1 + c2
+
+
3.
b+c
c+a
a+b
Li gii
S dng bt ng thc AM GM ta c

)(

)(

1 + a 2 1 + b2 1 + c2
1 + a2
1 + b2
1 + c2
6
.
+
+
3
b+c
c+a
a+b
( a + b )( b + c )( c + a )

Ta ch cn chng minh

(1 + a )(1 + b )(1 + c ) 1
2

( a + b )( b + c )( c + a )
(1 + a 2 )(1 + b 2 )(1 + c 2 ) ( a + b )( b + c )( c + a )

)(

Ti y ta nh gi ghp cp 1 + a 2 1 + b 2 ( a + b ) .
2

Ch s dng bt ng thc CS ta c:

(1 + a )(1 + b ) = (1 + a )(b + 1) ( a + b ) ;
(1 + b )(1 + c ) = (1 + b )( c + 1) ( b + c ) ;
(1 + c )(1 + a ) = (1 + c )( a + 1) ( a + c ) .
2

Nhn theo v ba bt ng thc trn ta c pcm.


ng thc xy ra khi v ch khi a= b= c= 1 .

)(

Ch . Ta c ng thc sau: 1 + x 2 1 + y 2 =( x + y ) + ( xy 1) .
2

T cng c ngay bt ng thc trn tng t nh dng CS.


Bng cch tng t ta chng minh c vi mi k dng ta c
113

www.TaiLieuLuyenThi.com
Khm ph t duy K thut gii bt T Bi ton Max Min ng Thnh Nam

k + a2
k + b2
k + c2
+
+
34 k .
b+c
c+a
a+b
5. K thut nh gi mu s
Cc bt ng thc hay c s dng nh gi mu s

x3 + y 3 xy ( x + y ) ; x5 + y 5 x 2 y 2 ( x + y ) .

Ch dng ton ny thng cho cc iu kin rng buc ta cn tinh t phn tch
mu s thnh nhn t da vo iu kin.
Ch nu mu s c cha cn hoc tng cc bin ta c th t mi mu s l
mt n khi a v s dng bt ng thc AM GM n gin hn.
V d 1. Cho a,b,c l cc s thc dng.
1
1
1
a+b+c
.
Chng minh rng 2
+ 2
+ 2

2abc
a + bc b + ac c + ab
Li gii
p dng bt ng thc AM GM ta c
a 2 + bc 2a bc

2
a + +bc
2

1
1 1
1
+ .
a bc 2 ab ac

1 1
1
+
b + ac b ac 2 bc ab
2
1
1 1
1

+
2
c + ab c ab 2 ac bc
Cng theo v ba bt ng thc trn ta c
2
2
2
a+b+c
.
+ 2
+ 2

2
2abc
a + bc b + ac c + ab
Bt ng thc c chng minh. ng thc xy ra khi v ch khi a= b= c .
V d 2. Cho a,b,c l cc s thc dng. Chng minh
1
1
1
1
.
+ 3 3
+ 3

3
3
3
a + b + abc b + c + abc c + a + abc abc
Li gii
Ta c
2

Tng t ta c:

a3 + b3 = ( a + b ) a 2 ab + b 2 = ab ( a + b ) + ( a + b )( a b ) ab ( a + b )

114

1
a + b + abc
3

1
1
c
=
=
ab ( a + b ) + abc ab ( a + b + c ) abc ( a + b + c )

www.TaiLieuLuyenThi.com
Cty TNHH MTV DVVH Khang Vit

Tng t ta c

1
b + c + abc
3

1
c + a + abc
3

a
abc ( a + b + c )

b
abc ( a + b + c )

Cng theo v 3 bt ng thc trn ta c iu phi chng minh. ng thc xy ra


khi v ch khi a= b= c .
Bi tp tng t
1) Cho x,y,z l cc s thc dng c tch bng 1. Chng minh
1
1
1
+
+
1.
x + y +1 y + z +1 z + x +1
3
3
HD: t
=
x a=
, y b=
, z c3 a v bt ng thc trn.

2) Cho x,y,z l cc s thc dng c tch bng 1. Chng minh


xy
yz
zx
+ 5
+ 5
1.
5
5
5
x + y + xy y + z + yz z + x5 + zx
V d 3. Cho a,b,c l cc s thc dng tho mn iu kin a 2 + b 2 + c 2 =
1.
Chng minh rng

a
b +c

3 3
.
2

c +a
a +b
Li gii
Ch iu kin ta c th vit bt ng thc di dng
2

a
b +c
2

b
c +a
2

c
a +b

iu ny lm ta suy ngh nh gi
Tht vy ta c

a
b +c
2

a
a
a2
=
=
=
b2 + c2 1 a 2 a 1 a 2

3 3 2
a + b2 + c2 .
2
3 3 2
a .
2

2a 2

2a 1 a
2

2a 2
2a 2 + 1 a 2 + 1 a 2

)(1 a )

3 3a 2
=
2

3 3b 2
c
3 3c 2
;

2
2
c2 + a2
a 2 + b2
Cng theo v 3 bt ng thc trn ta c iu phi chng minh.
1
.
ng thc xy ra khi v ch khi a= b= c=
3

Tng t ta c

115

www.TaiLieuLuyenThi.com
Khm ph t duy K thut gii bt T Bi ton Max Min ng Thnh Nam

Bi tp tng t
Cho a,b,c l cc s thc dng. Chng minh
a
b +c
2

b
c +a
2

c
a +b
2

3 3

2 a 2 + b2 + c2
V d 4. Cho a,b,c l cc s thc dng c tng bng 1.
ab
bc
ca
1
Chng minh rng
+
+
.
c + ab
a + bc
b + ca 2

Li gii
Theo gi thit ta c
ab
ab
=
=
c + ab
c ( a + b + c ) + ab

1 ab
ab

+
;
( c + a )( c + b ) 2 c + a c + b
ab

bc
=
a + bc

1 bc
bc

+
;
( a + b )( a + c ) 2 c + a a + b

ca
=
b + ca

1 ca
ca

+
.
( b + c )( b + a ) 2 a + b b + c

bc

ca

Cng theo v 3 bt ng thc trn ta c


ab
bc
ca
a+b+c 1
.
+
+

=
2
2
c + ab
a + bc
b + ca
Bt ng thc c chng minh. ng thc xy ra khi v ch khi a= b= c=
Bi tp tng t
Cho a,b,c l cc s thc dng c tng bng 1. Chng minh
1
1
1
1
.
+
+

a + bc
b + ca
c + ab 6abc
V d 5. Cho x,y,z l cc s thc dng tho mn iu kin xyz = 1 .
Chng minh rng

1
x + 2y + 3
2

1
y + 2z + 3
2

1
z + 2x + 3
2

Li gii
S dng bt ng thc AM GM ta c
x 2 + y 2 2 xy;
y 2 + 1 2 y.

Cng li theo v ta c: x 2 + 2 y 2 + 3 2 ( x + xy + 1) .
Suy ra
116

1
1
.
.
x + 2 y + 3 2 x + xy + 1
2

1
.
2

1
.
3

www.TaiLieuLuyenThi.com
Cty TNHH MTV DVVH Khang Vit

1
1
.
y + 2 z + 3 2 y + yz + 1
1
1
1
.
2
2
z + 2 x + 3 2 z + zx + 1
Cng theo v 3 bt ng thc trn v gi P l biu thc v tri ca bt ng thc

Tng t ta c

1
1
1
1
ta c P
+
+
.
2 x + xy + 1 y + yz + 1 z + zx + 1
1
1
1
Vy ta cn chng minh
+
+
1.
x + xy + 1 y + yz + 1 z + zx + 1

Tuy nhin y l mt ng thc lun ng, tht vy


1
1
1
+
+
x + xy + 1 y + yz + 1 z + zx + 1
1
1
1
=
+
+
x + xy + 1 y + y. 1 + 1 1 + 1 .x + 1
xy
xy xy
x
xy
1
=
+
+
=1
x + xy + 1 xy + 1 + x 1 + x + xy
Bt ng thc c chng minh. ng thc xy ra khi v ch khi x= y= z= 1 .
Nhn xt. Bn c lu ng thc quan trng vi x,y,z c tch bng 1 ta c
1
1
1
+
+
=
1.
x + xy + 1 y + yz + 1 z + zx + 1
Bi tp tng t
1) Cho x,y,z l cc s thc dng tho mn iu kin xyz = 8 .
Chng minh rng

1
1
1
1
+
+
.
2x + y + 6 2 y + z + 6 2z + x + 6 4

2) Cho x,y,z l cc s thc dng c tch bng 1. Chng minh


1
1
1
1
+ 5
+ 5
.
5
2
2
2
x x + 3 xy + 6 y y + 3 yz + 6 z z + 3 zx + 6 3
V d 6. Cho a,b,c l cc s thc dng tho mn iu kin

1 1 1
+ + =
1.
a b c

a2
b2
c2
a+b+c
.
+
+

a + bc b + ca c + ab
4
Li gii
Ch iu kin c vit li di dng: abc = ab + bc + ca .

Chng minh rng

117

www.TaiLieuLuyenThi.com
Khm ph t duy K thut gii bt T Bi ton Max Min ng Thnh Nam

a2
a3
a3
Khi = =
=
a + bc a 2 + abc a 2 + ab + bc + ca

a3
.
( a + b )( a + c )

S dng bt ng thc AM GM ta c
a3
a + b a + c 3a
a3
4a b c
.
+
+

a
+
b
a
+
c
8
8
4
a
+
b
a
+
c
8
(
)(
)
(
)(
)
b2
4b a c c 2
4c a b
.

b + ca
8
c + ab
8
Cng theo v ba bt ng thc trn ta c pcm. ng thc xy ra khi v ch khi
a= b= c= 3 .
V d 7. Cho a,b,c l cc s thc dng.
Tm gi tr nh nht ca biu thc
a + 3c
4b
8c
.
P=
+

a + 2b + c a + b + 2c a + b + 3c
Li gii
x =a + 2b + c a = x + 5 y 3 z

t y = a + b + 2c b = x 2 y + z .
z = a + b + 3c
c = z y

Tng t ta c

x + 2 y 4( x 2 y + z ) 8( z y )
Khi P = +

y
z
x
2 y 4x 4z 8 y
=
+
+
+
17
x
y
y
z
y 2x
z 2y
= 2 + + 4 +
17
x y
y z
S dng bt ng thc AM GM ta c
P4

y 2x
z 2y
.
+8 .
17= 12 2 17 .
x y
y z

(
(
(

)
)

a 5 2 7 x
=
y 2x
=

x y
y = 2 x

ng thc xy ra khi v ch khi

b = 3 2 2 x .
2 y 2x
z = 2y =
z =
y z
=
c 2 2 x

Vy gi tr nh nht ca P bng 12 2 17 .
Bi tp tng t
Cho x,y l cc s thc dng.
Tm gi tr nh nht ca biu thc
118

www.TaiLieuLuyenThi.com
Cty TNHH MTV DVVH Khang Vit

x+3
4y
8
.
+

x + 2y +1 x + y + 2 x + y + 3

P=

6. K thut nghch o
Mt s trng p dng trc tip AM GM lm bt ng thc i chiu v vy
ta kho lo chia phn thc cho t thc p dng AM GM. K thut ny hay c
p dng vi bt ng thc Bernoulli(xem ch sau).
V d 1. Cho a,b,c l cc s thc dng chng minh
a3
a3 + ( b + c )

b3

b3 + ( c + a )

c3

c3 + ( a + b )

1.

Li gii
S dng bt ng thc AM GM ta c

( x + 1) ( x 2 x + 1)

x2 x + 1 + x + 1 x2 + 2
.
=
2
2
ng thc xy ra khi v ch khi x = 0 hoc x = 2 .
1 + x3=

Ta c

a3
=
3
a3 + ( b + c )

1
b+c
1+

2
2

b+c

+2
a

2a 2
2a 2
a2
=

=
2
a 2 + b2 + c2
2a 2 + 2 b 2 + c 2
2a 2 + ( b + c )

Tng t ta c

b3
b3 + ( c + a )

c3
c3 + ( a + c )

b2
a 2 + b2 + c2
c2
a 2 + b2 + c2

Cng theo v 3 bt ng thc trn ta c iu phi chng minh.


ng thc xy ra khi v ch khi a= b= c.
Bi tp tng t
Cho a,b,c l cc s thc dng tho mn iu kin abc = 8.
Chng minh

a2

b2

c2

(1 + a )(1 + b ) (1 + b )(1 + c ) (1 + c )(1 + a )


3

4
.
3

V d 2. Cho a,b,c l cc s thc dng chng minh rng


a
b
c
+
+
>2.
b+c
c+a
a+b
119

www.TaiLieuLuyenThi.com
Khm ph t duy K thut gii bt T Bi ton Max Min ng Thnh Nam

Li gii
S dng bt ng thc AM GM ta c
2a
2a
a
=

;
b + c 2 a (b + c ) a + b + c
2b
2b
b
=

;
c + a 2 b (c + a) a + b + c
2c
2c
c
=

.
a + b 2 c (a + b) a + b + c

Cng theo v 3 bt ng thc trn ta c


a
b
c
a
b
c

+
+
2
+
+
2.
=
b+c
c+a
a+b
a+b+c a+b+c a+b+c
ng thc xy ra khi v ch khi a =+
b c, b =+
c a, c =+
a b
a + b + c= 2 ( a + b + c ) v l. Vy ng thc khng xy ra ta c iu phi
chng minh.
Bi tp tng t
1) Cho a,b,c l cc s thc dng. Chng minh
2

2a
2b
2c

+3
+3
3.
b+c
c+a
a+b

2) Cho a,b,c l cc s thc khng m tho mn iu kin a 2 + b 2 + c 2 =


2.
Chng minh rng

1 + bc

(b + c )

1 + ca

(c + a)

c
2.
a+b

3) Cho x,y,z l cc s thc khng m v tng i mt khc 0. Chng minh rng


x
y
z
+
+
2.
2
2
2
2
2
y +z
z +x
x + y2
4) Cho a,b,c l cc s thc khng m tho mn iu kin ab + bc + ca > 0. Chng minh
a
b
c
+
+2
2.
b+c
2c + a
a+b+c
5) Cho x,y l cc s thc khng m. Chng minh rng

x
+
y +1

y
+
x +1

1
2.
x+ y

V d 3. Cho a,b,c l cc s thc dng c tng bng 3. Chng minh


a 2 + b2 + c
a + b + c2
120

b2 + c2 + a
b + c + a2

c2 + a2 + b
c + a + b2

3.

www.TaiLieuLuyenThi.com
Cty TNHH MTV DVVH Khang Vit

Li gii
S dng bt ng thc AM GM cho hai s dng ta c
a 2 + b2 + c
=
a + b + c2
b2 + c2 + a

a 2 + b2 + c

( a + b + c )( a + b
2 (b + c + a )
2

+c

2 a 2 + b2 + c

a + b + c + a 2 + b2 + c2

c2 + a2 + b

2 c2 + a2 + b

b+c+a
a+b+c+a +b +c
c+a+b
a + b + c + a + b + c2
Cng theo v ba bt ng thc trn v a v chng minh
2

4 a 2 + b2 + c2 + 2 ( a + b + c )

3 a 2 + b2 + c2 3 .

a+b+c+a +b +c
Bt ng thc cui ng theo AM GM.
Bi ton c chng minh. ng thc xy ra khi v ch khi a= b= c= 1 .
Bi tp tng t
Cho a,b,c,d l cc s thc dng. Chng minh
a
b
c
d
+
+
+
2.
2
2
2
2
2
2
2
2
2
2
b +c +d
c +d +a
d +a +b
a + b2 + c2
2

V d 4. Cho a,b,c l cc s thc khng m tho mn iu kin


Tm gi tr nh nht ca biu thc
a (b + c )

P=

Ch

a 2 + bc

b (c + a)

b 2 + ca
Li gii

c (a + b)
c 2 + ab

) (
)
( a + bc ) ( b + c=) b ( a + c ) + c ( a + b )
Do vy nu t x =a ( b + c ) ; y =b ( c + a ) ; z =c ( a

a ( b + c ) = a b 2 + c 2 + 2bc a b 2 + c 2
2

a (b + c )
a + bc
2

b (c + a)
x

;
y + z b 2 + ca

c (a + b)
y

;
z + x c 2 + ab

+ b 2 ta c

z
.
x+ y

Nh vy sau khi cng theo v ba bt ng thc trn a v bi ton trnh by.


x
+
y+z

y
+
z+x

z
2.
x+ y

Ta c gi tr nh nht ca P bng 2 t ti=


a b=
, c 0 hoc cc hon v.
Cch 2: Nu c mt s bng 0, khng mt tnh tng qut gi s l c, khi
121

www.TaiLieuLuyenThi.com
Khm ph t duy K thut gii bt T Bi ton Max Min ng Thnh Nam

a
b
+ =
b
a

P=

a b

+ 2 2.
ab
Du bng xy ra khi v ch khi=
a b=
,c 0 .
Nu c ba s u dng khi s dng bt ng thc AM GM ta c
a (b + c )
a 2 + bc

(a

Tng t ta c

a (b + c )
2

+ bc a ( b + c )

b (c + a)
b + ca
2

a (b + c )
a 2 + bc + a ( b + c )
2

2b ( c + a )

( b + a )( b + c )

c (a + b)
c + ab
2

2a ( b + c )

( a + b )( a + c )
2c ( a + b )

( c + a )( c + b )

Cng theo v ba bt ng thc trn ta c:


a (b + c )
a + bc
2

b (c + a)
b + ca
2

c (a + b)
c + ab
2

2a ( b + c )

2b ( c + a )

2c ( a + b )

( a + b )( a + c ) ( b + a )( b + c ) ( c + a )( c + b )
2
2
2
2a ( b + c ) + 2b ( c + a ) + 2c ( a + b )
=
( a + b )( b + c )( c + a )
2 ( a + b )( a + c )( b + c ) + 8abc
=
( a + b )( b + c )( c + a )
8abc
=
>2
2+
+
a
b
(
)( b + c )( c + a )

So snh hai trng hp ta c gi tr nh nht ca P bng 2 t ti


=
a b=
, c 0 hoc cc hon v.
7. K thut s dng bt ng thc AM GM dng lu tha
m

a1m + a2m + ... + anm a1 + a2 + ... + an

.
n
n

Trong ak , k = 1, n l cc s khng m v m,n l cc s nguyn dng.


ng thc xy ra khi v ch khi a1= a2= ...= an .
Ch . Xem chng minh trong ch Bt ng thc Bernoulli.
Cc bt ng thc dng ny hay c s dng
1
1
3
4
a 3 + b3 ( a + b ) ; a 4 + b 4 ( a + b ) .
4
8
V d 1. Cho a,b,c l cc s thc dng. Chng minh
a3

( b + 3c )

122

b3

( c + 3a )

c3

( a + 3b )

3
.
64

www.TaiLieuLuyenThi.com
Cty TNHH MTV DVVH Khang Vit

Li gii
S dng bt ng thc AM GM dng lu tha ta c
3

b
c
a
b + 3c + c + 3a + a + 3b
a
b
c
+
+
3
.
3
( b + 3c )3 ( c + 3a )3 ( a + 3b )3

Ch s dng bt ng thc CS ta c
3

(a + b + c)
a
b
c
+
+

b + 3c c + 3a a + 3b a ( b + 3c ) + b ( c + 3a ) + c ( a + 3b )
2

Suy ra

a3

( b + 3c )

(a + b + c)

4 ( ab + bc + ca )

b3

( c + 3a )

c3

( a + 3b )

3 ( ab + bc + ca )

4 ( ab + bc + ca )

.
=

3
4

3
.
64

ng thc xy ra khi v ch khi a= b= c .


Bi tp tng t
Cho a,b,c l cc s thc dng. Chng minh
3

a b c 1

+
+
.
a + 2b b + 2c c + 2a 9
V d 2. Cho a,b,c l cc s thc dng. Chng minh
1
1
4
a 4 + b4 + c4 ( a + b + c ) .
8
64
Li gii
S dng bt ng thc AM GM dng lu tha ta c
4

c c

a+b+ 2 + 2
1 4 1 4
1
4
4 1 4
4
4
a + b + c = a + b + c + c 4
( a + b + c )4 .
=
8
16
16
4
64

c
.
2
Ngoi ra bt ng thc c dng ng cp ta c th chun ho a v xt hm
s(xem chng 3).
V d 3. Cho a,b,c l di ba cnh ca mt tam gic. Chng minh

Bt ng thc c chng minh. ng thc xy ra khi v ch khi a= b=

a +bc + 3 b+ca + 3 c+ a b 3 a + 3 b + 3 c .

Li gii
S dng bt ng thc AM GM dng lu tha ta c
123

www.TaiLieuLuyenThi.com
Khm ph t duy K thut gii bt T Bi ton Max Min ng Thnh Nam
3

a + b c + 3 b + c a 3 4(a + b c + b + c a) =
23 b

b + c a + 3 c + a b 3 4 (b + c a + c + a b) =
23 c

c + a b + 3 a + b c 3 4(c + a b + a + b c) =
23 a

Cng theo v ba bt ng thc trn ta c iu phi chng minh.


ng thc xy ra khi v ch khi a= b= c.
Bi tp tng t
Cho a,b,c l di ba cnh mt tam gic. Chng minh
a +bc + b+ca + c+ a b a + b + c .
V d 4. Cho x,y,z l cc s thc dng. Chng minh
4

x+ y 4 x+ y 4 x+ y
x
y
z
+
+
2 4
+4
+4
.
z
z
z
z+x
x + y
y+z
Li gii

S dng bt ng thc: 8( A4 + B 4 ) ( A + B ) 4 ta c:
4

y+z

4
8x
.
=
4
4
4
y+4z
(4 y + z )
8

4
x
x 1
1

.
4 y +4 z
4 4 y 4 z
Xy dng cc BT cn li ri cng v theo v, ta c:
4

Mt khc:

VT

2 4x+4z
.
.
4 y
2

Li c:

x+4z
8( z + x)
.
4
4 y
y

Suy ra: VT 4

z+x 4 x+ y 4 y+z
+
+
=
VP .
y
z
x

Bt ng thc c chng minh. ng thc xy ra khi v ch khi x= y= z .


8. K thut bin i tng ng kt hp AM GM
i khi nh gi bt ng thc trc tip bng AM GM khng hiu qu khi
cn kt hp bin i gia iu kin bi ton v bt ng thc cn chng minh. a
v cc bt ng thc n gin hn p dng AM GM.

124

www.TaiLieuLuyenThi.com
Cty TNHH MTV DVVH Khang Vit

V d 1. Cho a,b,c l cc s thc dng tho mn iu kin


1
1
1
+
+
=
2.
a +1 b +1 c +1
1
1
1
Chng minh rng
+
+
1.
4a + 1 4b + 1 4c + 1
Li gii
Bt ng thc cho tng ng vi

( 4a + 1)( 4b + 1) ( 4a + 1)( 4b + 1)( 4c + 1) 2 ( a + b + c ) + 1 32abc .

Ch iu kin bi ton tr thnh

( a + 1)( b + 1)= 2 ( a + 1)( b + 1)( c + 1) ab + bc + ca + 2abc=

1.

S dng bt ng thc AM GM ta c
a + b + c 3 ( ab + bc + ca=
)

3 (1 2abc ) .

Vy ta ch cn chng minh 2 3 (1 2abc ) + 1 32abc


1

abc 32

1
2 3 (1 2abc ) 32abc 1 abc 1
abc .
8

32

2
12 (1 2abc ) ( 32abc 1)
Bt ng thc cui ng bi s dng AM GM cho 3 s dng ta c
1= 2abc + ab + bc + ca 2abc + 3 a 2b 2 c 2 ; t=
3

abc
1
1
2
2t 3 + 3t 2 1 0 ( 2t 1)( t + 1) 0 t abc
2
8

Bt ng thc c chng minh. ng thc xy ra khi v ch khi a= b= c=

1
.
2

V d 2. Cho a,b,c l cc s thc dng tho mn iu kin


1
1
1
+
+
=
2.
a +1 b +1 c +1
1 1 1
Chng minh rng + + 4 ( a + b + c ) .
a b c
Li gii
Cch 1: Thc hin tng t bi ton trn.
x
y
z
Cch 2: Ch tn ti cc s dng x,y,z sao=
cho a =
.
,b =
,c
y+z
z+x
x+ y
Bi ton a v chng minh
125

www.TaiLieuLuyenThi.com
Khm ph t duy K thut gii bt T Bi ton Max Min ng Thnh Nam

x
y+z z+x x+ y
y
z
+
+
4
+
+
.
x
y
z
y+z z+x x+ y
Bt ng thc trn l tng ca ba bt ng thc sau
1 1
1 1
4z
4x
1 1 4y
.
y +
;z +
; x +
x z z+x y x x+ y z y y+z

Bt ng thc c chng minh. ng thc xy ra khi v ch khi a= b= c=

1
.
2

V d 3. Cho a,b,c l cc s thc dng c tch bng 1. Chng minh


1
1
1
2
+
+

+1.
a +1 b + c c +1 a + b + c +1
Li gii
Bt ng thc cho tng ng vi:
2 ( a + b + c ) + ab + bc + ca + 3

3+ a +b + c
a + b + c + ab + bc + ca + 2
a + b + c +1
( a + b + c + 1) 2 ( a + b + c ) + ab + bc + ca + 3

( a + b + c + 3)( a + b + c + ab + bc + ca + 2 )

a 2 + b2 + c2 3
Bt ng thc cui lun ng bi theo AM GM ta c

a 2 + b 2 + c 2 3 a 2b 2 c 2 =
3.
3

Bt ng thc c chng minh. ng thc xy ra khi v ch khi a= b= c= 1 .


9. K thut sp th t
p dng vi cc bt ng thc hon v ta c th gi s a = min {a, b, c} hoc
a = max {a, b, c} hoc a l s nm gia b v c.

Di y l mt bt ng thc c s dng kh nhiu ti xin trnh by


4
3
a 2b + b 2 c + c 2 a + abc ( a + b + c ) .
27
Chng minh.
Khng mt tnh tng qut gi s b l s nm gia a v c ta c:

( b a )( b c ) 0 b2 + ac ab + bc b2c + c 2 a abc + bc 2

a 2b + b 2 c + c 2 a + abc a 2b + bc 2 + 2abc = b a 2 + c 2 + 2ac = b ( a + c )

126

www.TaiLieuLuyenThi.com
Cty TNHH MTV DVVH Khang Vit
3

a+c a+c

b+ 2 + 2
a+c a+c
4
2
Khi VT b ( a + c=
.
4
=
( a + b + c )3 .
) 4.b.

2
2
3

27

Bt ng thc c chng minh. ng thc xy ra khi v ch kh a= b= c .

V d 1. Cho a,b,c l cc s thc dng c tng bng 3. Chng minh

( ab

+ bc 2 + ca 2 ( ab + bc + ca ) 9 .

Li gii
Khng mt tnh tng qut gi s b l s nm gia a v c
Ta c: a 2b + b 2 c + c 2 a = b(c + a ) 2 abc c(a b)(b c)
b(c + a ) 2 abc= b(c 2 + ac + a 2 )

p dng bt ng thc AM GM ta c:

( ab

+ bc 2 + ca 2 ( ab + bc + ca ) b(c 2 + ac + a 2 )(ab + bc + ca )
(c 2 + ac + a 2 ) + (ab + bc + ca )
b

b(c + a )2 (a + b + c)2 9b(a + c)2


=
4
4

.
3

a+c a+c

b + 2 + 2
a+c a+c
9b.
.
9
=
9
=
2
2
3

Ta c iu phi chng minh.


V d 2. Cho a,b,c l cc s thc khng m tha mn iu kin a + b + c =
3.
Tm gi tr ln nht ca biu thc =
P a b3 + 1 + b c 3 + 1 + c a 3 + 1 .
Li gii
S dng bt ng thc AM-GM ta c:
a b3 + 1 = a

( b + 1) ( b2 b + 1) a.

b + 1 + b2 b + 1
ab 2
.
= a+
2
2

bc 2
ca 2
.
; c a3 + 1 c +
2
2
Cng theo v 3 bt ng thc trn ta c:

Tng t: b c3 + 1 b +

Pa+b+c+

ab 2 bc 2 ca 2
ab 2 + bc 2 + ca 2
.
+
+
= 3+
2
2
2
2
127

www.TaiLieuLuyenThi.com
Khm ph t duy K thut gii bt T Bi ton Max Min ng Thnh Nam

Khng mt tnh tng qut gi s b l s nm gia a v c ta c:

( b a )( b c ) 0 b2 + ac ab + bc ab2 + a 2c a 2b + abc
2
ab 2 + bc 2 + ca 2 a 2b + abc + bc 2 = b ( a 2 + ac + c 2 ) b ( a + c )
S dng bt ng thc AM-GM ta c:
3

a+c a+c

b+ 2 + 2
a+c a+c
2
b ( a + c ) 4.b.
.
=
4
=
4.
2
2
3

ng thc xy ra khi v ch khi =


a 0,=
b 1,=
c 2 hoc cc hon v.
b(a + c)
ab 2 + bc 2 + ca 2
Suy ra P 3 +
3+
5.
2
2
Vy gi tr ln nht ca P bng 5 t ti =
a 0,=
b 1,=
c 2 hoc cc hon v.
2

V d 3. Cho a,b,c l cc s thc khng m tha mn iu kin a + b + c =


5.
Tm gi tr ln nht ca biu thc P = a 4b + b 4 c + c 4 a .
Li gii
Khng mt tnh tng qut gi s a = max {a, b, c} , khi

a 4 c a 3c 2
ac c 2
+
= a3 ab + bc +
+

2
2
2
2

a a a a+c
c
256. . . .
. b +
4 4 4 4
2

P = a 4b + b 4 c + c 4 a a 4b + a3bc +
c

= a3 ( a + c ) b + =
2

3c
c
a a+c

5
3. 4 + 4 + b + 2
a+b+ 4
a+b+c
256
= 256
256
= 256
5
5
5

ng thc xy ra khi v ch khi =


a 4,=
b 1,=
c 0 hoc cc hon v.
Vy gi tr ln nht ca P bng 256 t ti =
a 4,=
b 1,=
c 0 hoc cc hon v.

Nhn xt. Tng qut vi a, b, c khng m tha mn a + b + c =


k ta lun c
a nb + bn c + cn a

n n k n +1

( n + 1)n+1

Chng minh.
Khng mt tnh tng qut gi s a = max {a, b, c} khi

128

www.TaiLieuLuyenThi.com
Cty TNHH MTV DVVH Khang Vit

P = a n b + b n c + c n a a n b + a n 1bc +

a n c a n 1c 2
+
2
2

ac c 2
c

n 1
= a n 1 ab + bc +
+ =
a (a + c) b +

2
2
2

S dng bt ng thc AM-GM ta c

c
a a a a+c
c
1
. b +
a n =
( a + c ) b + nn . . ... .
2
2
n
n
n n

n 1

a a+c
c

n 1) . +
+b+
(

2
n
n
nn

1
n
+

a+b+c
nn

n +1

n +1

n +1

n +1

1 1
a + b + 2 + n c


nn
=
n +1

n n k n +1
=
( n + 1)n+1

kn
k
=
,b =
, c 0 hoc cc hon v.
n +1
n +1
V d 4. Cho a,b,c l cc s thc khng m tha mn iu kin a + b + c =
3.
Tm gi tr ln nht ca biu thc

ng thc xy ra khi v ch=


khi a

P=

(a

)(

)(

ab + b 2 b 2 bc + c 2 c 2 ca + a 2 .

Li gii
Khng mt tnh tng qut gi s a b c khi
b 2 bc + c 2 = b 2 + c ( c b ) b 2
c 2 ca + a 2 = c ( c a ) + a 2 a 2

Suy ra:

P a 2b 2 a 2=
ab + b 2

4 3ab 3ab 2
.
.
a ab + b 2
9 2 2

3ab 3ab

+
+ a 2 ab + b 2
4 2
4
4
6
2

=
c ) 12
( a + b )6
( a + b +=

9
3
9.27
9.27

ng thc xy ra khi v ch khi =


a 2,=
b 1,=
c 0.
Vy gi tr ln nht ca P bng 12 t ti =
a 2,=
b 1,=
c 0 hoc cc hon v.

129

www.TaiLieuLuyenThi.com
Khm ph t duy K thut gii bt T Bi ton Max Min ng Thnh Nam

10. K thut c si ngc du


Vic p dng bt ng thc AM GM cho mu s i khi lm ngc chiu bt
ng thc do vy trc khi p dng ta bin i phn thc c th p dng trc
tip bt ng thc AM GM.
V d 1. Cho a,b,c l cc s thc dng c tng bng 3. Chng minh
a
b
c
3
+
+
.
2
2
2
2
1+ b 1+ c
1+ a
Li gii
Ta c
a
1 + b2

a 1 + b 2 ab 2
1 + b2

ab 2
1 + b2

ab 2
ab
=a
2b
2

bc
c
ca 2
ca
;
=
c

c
2
2
2
2
2 1+ a
2
1+ c
1+ c
1+ a
Cng theo v 3 bt ng thc trn ta c
a
b
c
ab + bc + ca
ab + bc + ca 3
+
+
a+b+c
= 3
.
2
2
2
2
2
2
1+ b 1+ c
1+ a
b

=b

bc 2

=a

1
3.
( a + b + c )2 =
3
Bt ng thc c chng minh. ng thc xy ra khi v ch khi a= b= c= 1 .
V d 2. Cho x, y, z l cc s thc dng. Chng minh

V ab + bc + ca

yz
x + 2 yz

xy
zx
.
+
y + 2 zx z + 2 xy

Li gii
S dng bt ng thc AM GM ta c
1

1
x
x
= 1
1

x + 2 yz 2 x + 2 yz 2 x + y + z
1

zx
1
y
y
= 1
1

y + 2 zx 2
y + 2 zx 2 x + y + z
1
xy

1
z
z
= 1
1

z + 2 xy 2 z + 2 xy 2 x + y + z
Cng theo v 3 bt ng thc trn ta c iu phi chng minh.
ng thc xy ra khi v ch khi x= y= z.
yz

V d 3. Cho a,b,c l cc s thc dng tho mn iu kin a + b + c =


3.
Chng minh rng
130

www.TaiLieuLuyenThi.com
Cty TNHH MTV DVVH Khang Vit

a2
a + 2b 2

b2

c2

b + 2c 2 c + 2 a 2
Li gii
S dng bt ng thc AM GM ta c

a2
a + 2b 2

a a + 2b 2 2ab 2
a + 2b 2
b2

=a

2ab 2
a + 2b 2

1.

2ab 2
3

3 ab 4

=a

2
2
( ab ) 3
3

2
2
c2
2
2
bc

ca
3;
3
(
)
(
)
3
3
b + 2c 2
c + 2a 2
Cng theo v 3 bt ng thc trn ta ch cn chng minh

(ab) 3 + (bc) 3 + (ca ) 3 3 .

Bt ng thc trn ng theo AM GM v


2

a + ab + b 3(ab) 3
2
b + bc + c 3(bc) 3

c + ca + a

2
3(ca ) 3

1
3
Bt ng thc c chng minh. ng thc xy ra khi v ch khi a= b= c= 1 .
V d 4. Cho a,b,c l cc s thc khng m c tng bng 3. Tm gi tr nh nht
ca biu thc
a
b
c
.
P= 3
+ 3
+ 3
b + 16 c + 16 a + 16
Li gii
Ta c:
ab3
ab3
bc3
ca3
bc3
ca3
.
+b 3
+c 3
= 3 3
+ 3
+ 3
16 P = a 3
b + 16 c + 16 a + 16
b + 16
c + 16
a + 16

9
( a + ab + b ) + ( b + bc + c ) + ( c + ca + a ) 2 ( a + b + c ) + ( a + b + c )2 =

tm gi tr ln nht ca P ta tm gi tr nh nht ca

ab3

bc3

ca3

b3 + 16 c3 + 16 a3 + 16

S dng bt ng thc AM-GM ta c:


ab3
ab3
ab3 ab 2
.
=

=
b3 + 16 b3 + 8 + 8 12b 12

Tng t:

bc3
c3 + 16

bc 2 ca3
ca 2
.
; 3

12 a + 16 12
131

www.TaiLieuLuyenThi.com
Khm ph t duy K thut gii bt T Bi ton Max Min ng Thnh Nam

Cng li theo v 3 bt ng thc trn ta c:


ab3
b3 + 16

bc3
c3 + 16

ca3
a3 + 16

ab 2 + bc 2 + ca 2
.
12

Bi ton a v tm gi tr ln nht ca ab 2 + bc 2 + ca 2 y l mt bi ton


quen thuc.
Khng mt tnh tng qut gi s b l s nm gia a v c ta c:

( b a )( b c ) 0 b2 + ac ab + bc ab2 + ca 2 a 2b + abc

ab 2 + bc 2 + ca 2 a 2b + abc + bc 2 = b a 2 + ac + c 2

a+c a+c

b+ 2 + 2
a+c a+c
2
b ( a + c ) 4.b.
.
=
4
=
4
2
2
3

ng thc xy ra khi v ch khi =


a 0,=
b 1,=
c 2 v cc hon v.
Suy ra 16 P 1 P

1
.
16

1
t ti =
a 0,=
b 1,=
c 2 hoc cc hon v.
16
V d 5. Cho a,b,c l cc s thc dng c tng bng 3.

Vy gi tr nh nht ca P bng

Chng minh rng

1
2+a b
2

4 2

a 2b

2 + b c 2 + c2a
Li gii
2

1.

1
1
Ta c:
=1
1 a 3 b 3 1 a 2 + ab + ab .
2
2
3
9
2+a b
2+a b
2

1 2
2
1
b + bc + ab ;
1 c 2 + ca + bc .
2
9
9
2+b c
2+c a
Cng theo v ba bt ng thc trn ta c:

Tng t ta c:

1
1
(a + b + c) 2
1
2
3
2
+
+

9
2 + a 2b 2 + b 2 c 2 + c 2 a
1
1
1

+
+
1
2
2
2 + a b 2 + b c 2 + c2a

Bt ng thc c chng minh. ng thc xy ra khi v ch khi a= b= c= 1 .

132

www.TaiLieuLuyenThi.com
Cty TNHH MTV DVVH Khang Vit

11. S dng bt ng thc ( a + b )( b + c )( c + a )

8
( a + b + c )( ab + bc + ca ) .
9

Chng minh.
y l mt bt ng thc c s dng kh thng xuyn vi bt ng thc i
xng 3 bin da trn ng thc
( a + b )( b + c )( c + a ) = ( a + b + c )( ab + bc + ca ) abc .
Ta c ( a + b + c )( ab + bc + ca ) abc =

( a + b )( b + c )( c + a ) .
a + b b + c c + a ( a + b )( b + c )( c + a )

ab . bc . ca
Mt khc: abc =

=
8
2 2 2
Suy ra:
( a + b )( b + c )( c + a ) = ( a + b + c )( ab + bc + ca ) abc
( a + b + c )( ab + bc + ca )

( a + b )( b + c )( c + a ) .
8

8
9
Bt ng thc c chng minh. ng thc xy ra khi v ch khi a= b= c .

Do :

( a + b )( b + c )( c + a ) ( a + b + c )( ab + bc + ca ) .

V d 1. Cho a,b,c l cc s thc dng tho mn iu kin ( a + b )( b + c )( c + a ) =


1.
3
.
4
Li gii
S dng bt ng thc quen thuc:
8
( a + b )( b + c )( c + a ) ( a + b + c )( ab + bc + ca ) .
9
Theo gi thit bi ton kt hp vi bt ng thc AM GM ta c:
8
( a + b )( b + c )( c + a ) ( a + b + c )( ab + bc + ca )
9
8

3 ( ab + bc + ca ) . ( ab + bc + ca ) .
9
3
ab + bc + ca
4

Chng minh rng ab + bc + ca

1
.
2
1 1 1
V d 2. Cho a,b,c l cc s thc dng tho mn iu kin a + b + c + + .
a b c
1
1
1
3
Chng minh rng
+
+
.
( 2a + b + c )2 ( 2b + c + a )2 ( 2c + a + b )2 16

Bt ng thc c chng minh. ng thc xy ra khi v ch khi a= b= c=

133

www.TaiLieuLuyenThi.com
Khm ph t duy K thut gii bt T Bi ton Max Min ng Thnh Nam

Li gii
S dng bt ng thc AM GM ta c

( 2a + b + c )2 =( a + b ) + ( a + c )

( 2a + b + c )

4 ( a + b )( a + c )

1
1
.
4 ( a + b )( a + c )

Tng t cho 2 phn thc cn li a v chng minh


1
1
1
3
+
+

( a + b )( a + c ) ( b + c )( b + a ) ( c + a )( c + b ) 4

2(a + b + c)

( a + b )( b + c )( c + a )

3
4

8
(a + b + c)
3
8
Ch ta c ( a + b )( b + c )( c + a ) ( a + b + c )( ab + bc + ca )
9
Vy ta ch cn chng minh ab + bc + ca 3 .
( a + b )( b + c )( c + a )

Theo gi thit ta c abc ( a + b + c ) ab + bc + ca .


S dng bt ng thc AM GM ta c

( ab + bc + ca )2 3abc ( a + b + c ) 3 ( ab + bc + ca ) .

ab + bc + ca 3
Bt ng thc c chng minh. ng thc xy ra khi v ch khi a= b= c= 1 .
V d 3. Cho a,b,c l cc s thc dng tho mn iu kin ab + bc + ca 1 .
1
1
1
Chng minh rng a + b + c + 3 8abc 2
+ 2
+ 2
.
a +1 b +1 c +1
Li gii
Ch iu kin bi ton ab + bc + ca 1 ta c nh gi

2(a + b + c)

.
a 2 + 1 a 2 + ab + bc + ca =
( a + b )( a + c ) =
( a + b )( b + c )( c + a )
S dng bt ng thc ( a + b )( b + c )( c + a )
Ta c

a2 + 1 8

9
Vy ta chng minh

134

2(a + b + c)

8
( a + b + c )( ab + bc + ca ) .
9

9
.
=
+
+
4
ab
bc
ca
(
)
( a + b + c )( ab + bc + ca )

www.TaiLieuLuyenThi.com
Cty TNHH MTV DVVH Khang Vit

a + b + c + 3 8abc.

9
4 ( ab + bc + ca )

( a + b + c )( ab + bc + ca ) + 3 ( ab + bc + ca ) 18abc
3

ab + bc + ca
1
Ch abc
.
=
3

3 3
Do bt ng thc cui l tng ca 2 bt ng thc.
( a + b + c )( ab + bc + ca ) 9abc
2 2
3 ( ab + bc + ca ) 3 3. a 2b=
c
3

3 3abc 3 3abc

= 9abc .
3
abc
1
3
3 3

Bt ng thc c chng minh. ng thc xy ra khi v ch khi a= b= c=

1
.
3

12. K thut chun ho


Bt ng thc c dng thun nht bc k c dng
P ( x1 , x2 ,..., xn ) 0 trong P (tx1 , tx2 ,..., txn ) = t k P ( x1 , x2 ,..., xn ) .

Khi ta c th thc hin


+ Chun ho cho x m + y m + z m =
const (chuyn bi ton khng c iu kin v c
iu kin).
+ Hoc nhn(chia) vi i lng cha bin a bt ng thc v dng thun
nht gip chng minh n gin hn.
V d 1. Cho a,b,c l cc s thc dng. Chng minh
3

( a + b )2 ( b + c )2 ( c + a )2
abc

4
(a + b + c) .
3

Li gii
Bt ng thc c dng thun nht bc 1 ta chun ho a + b + c =
3.
Ta cn chng minh

( a + b )2 ( b + c )2 ( c + a )2
abc

4 .

( a + b )( b + c )( c + a ) 8 abc

Ch

( a + b )( b + c )( c + a ) = ( 3 a )( 3 b )( 3 c ) = 3 ( ab + bc + ca ) abc
ab + bc + ca 3abc ( a + b + c ) =
3 abc
Vy ta chng minh 9 abc abc 8 abc abc 1 .
135

www.TaiLieuLuyenThi.com
Khm ph t duy K thut gii bt T Bi ton Max Min ng Thnh Nam
3

a+b+c
Bt ng thc cui ng bi v abc
1.
=
3

Bt ng thc c chng minh. ng thc xy ra khi v ch khi a= b= c .


Bi tp tng t
Cho a,b,c l cc s thc khng m. Chng minh

( a + b )( b + c )( c + a )

ab + bc + ca
.
8
3
V d 2. Cho a,b,c l cc s thc dng c tng bng 1. Chng minh
3

a 2 + b 2 + c 2 3 a 2b + b 2 c + c 2 a .

Li gii
Ta chuyn bt ng thc v dng thun nht bc ba bng cch thay
a 2 + b2 + c2 =

Ta cn chng minh

(a

(a

+ b2 + c2 ( a + b + c ) .

+ b 2 + c 2 ( a + b + c ) 3 a 2b + b 2 c + c 2 a

) (

) (

) (

a3 + ab 2 + b3 + bc 2 + c3 + ca 2 2 a 2b + b 2 c + c 2 a

Bt ng thc ny l tng ca 3 bt ng thc


a3 + ab 2 2a 2b; b3 + bc 2 2b 2 c; c3 + ca 2 2c 2 a .

Bt ng thc c chng minh. ng thc xy ra khi v ch khi a= b= c=


V d 3. Cho a,b,c l cc s thc dng c tng bng 1. Chng minh
a 2 + b 2 + c 2 + 2 3abc 1 .
Li gii
Ch ta a bt ng thc v dng thun nht bc 2 bng cch thay
2=
3abc 2 3abc ( a + b + c )
1=

( a + b + c )2

Bt ng thc tng ng vi
a 2 + b 2 + c 2 + 2 3abc ( a + b + c ) ( a + b + c )
ab + bc + ca 3abc ( a + b + c )
( ab + bc + ca ) 3abc ( a + b + c )
2

136

1
.
3

www.TaiLieuLuyenThi.com
Cty TNHH MTV DVVH Khang Vit

Bt ng thc cui lun ng. Bt ng thc c chng minh. ng thc xy


1
ra khi v ch khi a= b= c=
.
3
V d 4. Cho a,b,c l cc s thc dng. Chng minh

3 a 2 + b2 + c2
a b c
.
+ +
3
b c a
abc
Li gii
Chun ho abc = 1 ta cn chng minh
a b c
+ + 3 a 2 + b2 + c2
b c a

a b c
+ + 3 a 2 + b2 + c2
b c a

a b2
b c2
c
2
2
+
+
+
+ 2 + 2 3 a 2 + b2 + c2
2
2
c c
a a
b
b
Bt ng thc cui l tng ca 3 bt ng thc

a2

a2
b2
b2
c2
c2

a a
a4
+ 33 2 2 =
3a 2
c c
b c

b b
b4
+ 33 2 2 =
3b 2
a a
a c

c c
c4
3
+

3
=
3c 2
2
2
2
b b
a
a b
Bt ng thc c chng minh. ng thc xy ra khi v ch khi a= b= c .
V d 5. Cho a,b,c l cc s thc dng chng minh
+

6(a + b + c)
a+b
b+c
c+a
.
+
+

3
c
a
b
abc

Li gii
Bt ng thc c dng thun nht bc 0 chun ho abc = 1 .
a+b
b+c
c+a
+
+
6(a + b + c) .
c
a
b
S dng bt ng thc Mincopski ta c

Ta cn chng minh

a+b
b+c
c+a
+
+
=
c
a
b

137

www.TaiLieuLuyenThi.com
Khm ph t duy K thut gii bt T Bi ton Max Min ng Thnh Nam
2

a c
b a
c b

+
+
+
+
+

b b
c c
a a
2

a
b
c c
a
b

+
+
+
+
+

c
a b
c
a
b

S dng bt ng thc trnh by trong v d 4 ta c


3( a + b + c )
a
b
c
+
+

=
3
b
c
a
b. c. a

3( a + b + c )

3( c + a + b )
c
a
b
+
+

=
3
b
c
a
b. c. a

3( a + b + c )

Suy ra iu phi chng minh. ng thc xy ra khi v ch khi a= b= c .


13. K thut t n ph
i khi t n ph a v bin mi gip nh gi AM GM n gin hn.
V d 1. Cho a, b, c l cc s thc dng.
b(a c) c(3b + a ) 2c(a b)
.
Tm gi tr nh nht ca biu thc P =
+
+
c(a + b) a (b + c)
b( a + c )
Li gii
t x =ab > 0 ; y =bc > 0 ; z =ca > 0, khi
x y 3y + z 2z 2 y
.
P=
+
+
z+ y x+z
x+ y

t m = z + y > 0 ; n = x + z > 0 ; p = x + y > 0 .


Suy ra 2 x =
n + p m ; 2y =
m + p n ; 2z =
p + n+ m.
n 2m p 2n
Khi P = +
+ + 4.
m n n p
S dng bt ng thc AM-GM ta c

n 2m
p 2n
n 2m p 2n
P= +
.
+2
.
4= 4 2 4 .
+ + 42
m n
n p
m n n p

Vi ( a, b, c ) = 1,1 + 2 2, 5 + 4 2 P =4 + 4 2 .
V vy Pmin =4 + 4 2 .
V d 2. Cho x,y,z l cc s thc ln hn 1 v tho mn iu kin
xy + yz + zx + xyz =
20 .
138

www.TaiLieuLuyenThi.com
Cty TNHH MTV DVVH Khang Vit

Chng minh rng

3
( x 1)( y 1)( z 1) .
x+ y + z 3

Li gii
t a =x 1, b =y 1, c =z 1; a, b, c > 0 ta cn chng minh abc ( a + b + c ) 3 .
Ch gi thit bi ton tr thnh:
20
( a + 1)( b + 1) + ( b + 1)( c + 1) + ( c + 1)( a + 1) + ( a + 1)( b + 1)( c + 1) =
ab + bc + ca + 2 ( a + b + c ) + 3 + abc + a + b + c + ab + bc + ca + abc =
20
3 ( a + b + c ) + 2 ( ab + bc + ca ) + 2abc =
17

S dng bt ng thc AM GM ta c 2 ( ab + bc + ca ) 2 3abc ( a + b + c )


2 a +b+c
4 abc ( a + b + c )
+ abc

3
3
3
3

7(a + b + c) 7 4
7
=
( a + b + c )3 .( a + b + c ) 4 27abc.( a + b + c )
3
3
3
Cng theo v ba bt ng thc trn suy ra
4 abc ( a + b + c ) 7 4
+
27 abc. ( a + b + c ) 17
3
3
3
V tri l hm ng bin vi abc(a + b + c) t suy ra abc ( a + b + c ) 3 .
2 3abc ( a + b + c ) +

Bt ng thc c chng minh. ng thc xy ra khi v ch khi x= y= z= 2 .


14. K thut la chn im ri
nh gi bt ng thc iu kin tin quyt l m bo du bng. Do vy vi
bt ng thc i xng cc bin thng bng nhau y l c s cho vic ta p dng
bt ng thc AM GM cho cc s dng hp l.
Trong trng hp kh on bin du bng xy ra ti y ta c th s dng
tham s ho hoc gii phng trnh o hm ring.
Chi tit hn v v phng php ny xem ch tham s ho.
V d 1. Cho a,b,c l cc s thc dng c tng bng 1. Tm gi tr ln nht ca
biu thc P = 3 a + b + 3 b + c + 3 c + a .
Li gii
Phn tch tm li gii:
2

a + b =
3

a+b+c 1
2
Biu thc i xng 3 bin du bng t ti a = b = c =
= b + c = .
3
3
3
2

c + a =
3

139

www.TaiLieuLuyenThi.com
Khm ph t duy K thut gii bt T Bi ton Max Min ng Thnh Nam

Do la chn cc s

2
ghp cp vi tng tng ( a + b ) , ( b + c ) , ( c + a ) .
3

Li gii chi tit:


S dng bt ng thc AM GM cho 3 s dng ta c
2 2
a+b+ +
9
2
2
9
3
3 3;
a=
+ b 3 .3 ( a + b ) . . 3 .
4
3 3
4
3
2 2
b+c+ +
9
2
2
9
3
3 3;
b=
+ c 3 .3 ( b + c ) . . 3 .
4
3 3
4
3
2 2
c+a+ +
9
2
2
9
3
3 3.
c=
+ a 3 .3 ( c + a ) . . 3 .
4
3 3
4
3
Cng theo v 3 bt ng thc trn ta c

a + b + 3 b + c + 3 c + a 3 18 .

Vy gi tr ln nht ca P bng 3 18 du bng t ti a= b= c=

1
.
3

Bi tp tng t
Cho a,b,c l cc s thc dng c tng bng 1. Tm gi tr ln nht ca biu thc
P=

2a + 3b + 3 2b + 3c + 3 2c + 3a .

V d 2. Cho a,b,c l cc s thc dng tho mn iu kin a 2 + b 2 + c 2 =


1.
1
.
Tm gi tr nh nht ca biu thc P = a + b + c +
abc
Li gii
Phn tch tm li gii:
a 2 + b2 + c2
1
1
Du bng t ti a =
3 3.
b=
c=
=
=
3
abc
3

Vy tch ghp cp nh sau: a + b + c +

1
1
8

.
= a +b + c +
+
abc
9abc 9abc

Li gii chi tit:


S dng bt ng thc AM GM ta c
1
8
1
8

P = a +b + c +
4 4 a.b.c.
+
+
9abc 9abc
9abc

a 2 + b2 + c2
9

Vy gi tr nh nht ca P bng 4 3 t ti a= b= c=
140

1
3

= 4 3.

www.TaiLieuLuyenThi.com
Cty TNHH MTV DVVH Khang Vit

V d 3. Cho a,b,c l cc s thc dng tho mn iu kin a + b + c


Tm gi tr nh nht ca biu thc P = a 2 + b 2 + c 2 +

3
.
2

1 1 1
+ + .
a b c

Li gii
Phn tch tm li gii:
2
a=
1
Du bng t ti a= b= c=

2 1
=
a

2
2
b=
c=

1
4

.
1 1
= = 2
b c
1 1 1
Vy cn tch ghp nh sau: a 2 + b 2 + c 2 + + +
a b c
1
1
1
1
1
1 3 1 1 1

= a2 +
+ + b2 + + + c2 + + + + + .
8a 8a
8b 8b
8c 8c 4 a b c

Li gii chi tit:


S dng bt ng thc AM GM cho 3 s dng ta c

1
1
1
1
1 1 3 1 1 1

+ + b2 + + + c2 + + + + +
P= a 2 +
8a 8a
8b 8b
8c 8c 4 a b c

1 1
1 1
1 1
9
. + 33 b2 . . + 33 c 2 . . + 3
8a 8a
8b 8b
8c 8c 4 abc
9
9
27
+

4 4. a + b + c 4
3
27
1
Vy gi tr nh nht ca P bng
t ti a= b= c=
.
4
2
Cch 2: Ta c
1
9
2
P (a + b + c) +
3
a+b+c
33 a 2 .

( 2 ( a + b + c ) 3) ( 2 ( a + b + c )2 + 3 ( a + b + c ) 36 )
12 ( a + b + c )

.
27 27
+

4
4

V d 4. Cho a,b,c l cc s thc dng tho mn iu kin a 2 + b 2 + c 2 =


12 .
Chng minh rng a b 2 + c 2 + b c 2 + a 2 + c a 2 + b 2 24 .
Li gii
3

2
Du bng t ti a= b= c= 2 . Khi 4=
a 2a=
b 2 + c 2 ta s dng bt ng
thc AM GM nh sau:

141

www.TaiLieuLuyenThi.com
Khm ph t duy K thut gii bt T Bi ton Max Min ng Thnh Nam

4a.2a 2 . b 2 + c 2

4b.2b . c 2 + a 2

4c.2c . a 2 + b 2

b c +a
=
3

c a +b
=
3

a b +c
=
3

) 4a + 2a
) 4b + 2b
) 4c + 2c

+ b2 + c2
6

+ c2 + a2
.
6

+ a 2 + b2
6

Cng theo v 3 bt ng thc trn ch a + b + c 3 a 2 + b 2 + c 2 =


6 ta c
iu phi chng minh.
ng thc xy ra khi v ch khi a= b= c= 2 .
9
a + b + c =
.
V d 5. Cho a,b,c l cc s thc dng tho mn iu kin
a 5, a + b 8
Tm gi tr ln nht ca biu thc P = abc .
Li gii
T iu kin ta d on du bng t ti =
a 5,=
b 3,=
c 1. V vy vit li P v s
dng AM GM cho ba s dng t trung bnh nhn sang trung bnh cng.
Ta c a 5 b + c 4; a + b 8 c 1 .
S dng bt ng thc AM GM cho ba s dng ta c:

P
=

1
1 3a + 5b + 15c
. ( 3a.5b.15c )

15
15
3

1 3 ( a + b + c ) + 2 ( b + c ) + 10c
1 3.9 + 2.4 + 10.1
=


15
15
3
3

15
3

Vi =
a 5,=
b 3,=
c 1 th P bng 15. Vy gi tr ln nht ca P bng 15.
V d 6. Cho a,b,c l cc s thc dng c tng bng 1. Chng minh
1
1
+
30 .
2
2
2
abc
a +b +c
Li gii
S dng bt ng thc AM - GM ta c
1
1
1
7
1
7

.
=
+
+
33
+
VT 2
+
2
2
9abc 9abc 9abc
a +b +c
a 2 + b 2 + c 2 .9abc.9abc 9abc

a 2 + b 2 + c 2 + 3 ( ab + bc + ca )
Ta c a + b + c .3ab.3bc.3ca

142

www.TaiLieuLuyenThi.com
Cty TNHH MTV DVVH Khang Vit
4

(a + b + c)

1
+ ab + bc + ca 1 + 3
1

.
=

4
81
4

1
a+b+c
Ch abc
.
=
3
27

T suy ra pcm.

Cch 2: Ta c: 3(ab + bc + ca ) (a + b + c)2


Mt khc: (ab + bc + ca )(a + b + c) 9abc
Do :

1
9

abc ab + bc + ca

1
1
7
+
+
ab + bc + ca ab + bc + ca ab + bc + ca
a +b +c
9
21

+
=
30
2
(a + b + c)
(a + b + c)2

1
3

ab + bc + ca (a + b + c) 2

Ta c pcm.
B. BI TON CHN LC
Bi 1. Cho a,b,c l cc s thc dng tha mn iu kin b a > c > 0 .
Chng minh rng

ab +

c (a b)
ab

1
3.
c (a c)

Li gii
Nhn xt. Ta chng minh bt ng thc quen thuc sau y:
ab c ( a c ) + b ( b c ) .

Tht vy bt ng thc tng ng vi:

c (a c)

c (b c )

ab
S dng bt ng thc AM-GM cho 2 s dng ta c

c (a c)

ab

1.

c (b c )

1c ac c bc
+
+ +
1.
=
2b
ab
ab
a
a
b
Ta c iu phi chng minh.
Gi ta p dng vo bi
+

143

www.TaiLieuLuyenThi.com
Khm ph t duy K thut gii bt T Bi ton Max Min ng Thnh Nam

ab +

c (a b)
ab

c ( a c ) c (b c )
1
1
=
+
ab +
c (a c)
c
a
( c)
ab
c ( a c ) c (b c )

1
c (a c)

c ( a c ) + c (b c ) + c ( a c ) c (b c ) +

1
c (a c)

c ( a c ) + c (b c ) +
=

= 2 c (c a) +

c ( a c ) + c (b c )

1
=
c (a c)

c (a c) + c (a c) +

33 c ( a c ). c ( a c ).

1
c (a c)

1
=
3
c (a c)

a= b= 2
ng thc xy ra khi v ch khi
.
c = 1
Bi 2. Cho a,b,c l cc s thc dng tha mn iu kin a + b + c =
3.
a+b
b+c
c+a
Chng minh rng
+
+
3.
c + ab
a + bc
b + ac
Li gii
S dng bt ng thc AM-GM ta c
a+b
b+c
c+a
a+b b+c c+a
.
+
+
36
.
.
c + ab
a + bc
b + ac
c + ab a + bc b + ca
Ta ch cn chng minh ( a + b )( b + c )( c + a ) ( a + bc )( b + ca )( c + ab ) .

S dng bt ng thc AM-GM ta c


2
( a + b ) ( c + 1) .
a + bc + b + ca
( a + bc )( b + ca )
=
2
4

Tng t ta c:
2

( b + ca )( c + ab )

( b + c )2 ( a + 1)2 ;

( c + ab )( a + bc )

4
Nhn theo v 3 bt ng thc trn ta c

( a + bc )( b + ca )( c + ab ) ( a + b )( b + c )( c + a )
2

( a + c )2 ( b + 1)2 .
4

2
2
2
a + 1) ( b + 1) ( c + 1)
(
.
.

kt thc bi ton ta ch cn chng minh

64
3

a +1+ b +1+ c +1
V ( a + 1)( b + 1)( c + 1)
8.
=
3

ng thc xy ra khi v ch khi a= b= c= 1 .


144

64

( a + 1) ( b + 1) ( c + 1)
2

1 (lun ng).

www.TaiLieuLuyenThi.com
Cty TNHH MTV DVVH Khang Vit

Bi tp tng t
Cho a,b,c l cc s thc dng c tng bng 1. Chng minh
a+b
b+c
c+a
3
.
+
+
3
c + ab
a + bc
b + ac
2
Bi 3. Cho a,b,c l cc s thc khng m tho mn iu kin ab + bc + ca > 0 .

1 + a2
1 + b2
1 + c2
+
+
3.
b+c
c+a
a+b
Li gii
S dng bt ng thc AM GM cho 3 s dng ta c

Chng minh rng

)(

)(

1 + a 2 1 + b2 1 + c2
1 + a2
1 + b2
1 + c2
6
.
+
+
3
b+c
c+a
a+b
( a + b )( b + c )( c + a )

(1 + a )(1 + b )(1 + c ) 1
2

Vy ta chng minh

)(

)(

( a + b )( b + c )( c + a )

1 + a 2 1 + b 2 1 + c 2 ( a + b )( b + c )( c + a ) .

( )( )
(1 + b )(1 + c ) ( b + c ) ;
(1 + c )(1 + a ) ( a + c ) .

S dng bt ng thc C S ta c 1 + a 2 1 + b 2 ( a + b ) ;
2

Nhn theo v 3 bt ng thc trn ta c iu phi chng minh.


Bt ng thc c chng minh. ng thc xy ra khi v ch khi a= b= c .
Bi 4. Cho a,b,c l cc s thc khng m tho mn iu kin ab + bc + ca > 0 .
Chng minh rng

a
b
c
+
+
2.
b+c
c+a
a+b
Li gii

S dng bt ng thc AM GM ta c ( a + b + c ) 4a ( b + c )
2

a
4a 2
a
2a
.

2
b + c (a + b + c)
b+c a+b+c

Tng t ta c

b
2b

c+a a+b+c

2c
c

a+b a+b+c
Cng theo v 3 bt ng thc trn ta c
145

www.TaiLieuLuyenThi.com
Khm ph t duy K thut gii bt T Bi ton Max Min ng Thnh Nam

a
b
c
a
b
c

+
+
2
+
+
2.
=
+
+
+
+
+
+
b+c
c+a
a+b
a
b
c
a
b
c
a
b
c

Bt ng thc c chng minh. ng thc xy ra khi v ch khi c 1 s bng 0


v 2 s cn li bng nhau.
Nhn xt. Ngoi li gii trn ta c th chng minh bt ng ny bng bin i
tng ng, bt ng thc Holder hoc Bernuoli.
Bi 5. Cho a,b,c l cc s thc dng. Chng minh
2

2a
2b
2c

+3
+3
3.
b+c
c+a
a+b
Li gii
S dng bt ng thc AM GM cho 3 s dng ta c
3

2a

=
b+c
3

2b

=
c+a
3

2c

=
a+b
3

a
a.

b+c b+c
.
2
2

a
3a
=
b+c b+c a+b+c
+
a+
2
2
3

b
b
3b

=
c+a c+a b+ c+a + c+a a+b+c
.
b.
2
2
2
2
3

c
3c
=
a+b a+b c+ a+b + a+b a+b+c
.
c.
2
2
2
2
3
Cng theo v 3 bt ng thc trn ta c iu phi chng minh. ng thc xy ra
khi v ch khi a= b= c .
Bi 6. Chng minh rng vi mi a,b,c dng ta c
3

b b
c c
a
a
+
+
+

1.
b + c c + a c + a a + b a + b b + c
Li gii
S dng bt ng thc AM GM cho 2 s dng ta c
a
b
a
b
+ =
+1+
+1 2
b+c c+a b+c
c+a
1
1
= (a + b + c)
+
2
b+c c+a

146

www.TaiLieuLuyenThi.com
Cty TNHH MTV DVVH Khang Vit

2(a + b + c)
=

2
( b + c )( c + a )

( a + b) + ( a + c ) + (b + c ) 2
( b + c )( c + a )

( a + c )( b + c )
a+b
2=
( a + c )( b + c )
( a + c )( b + c )

a+b+2

Tng t ta c

b+c

b
c
+

c+a a+b

( c + a )( a + b )

c+a

c
a
+

a+b b+c

( a + b )( b + c )

Nhn theo v 3 bt ng thc trn ta c iu phi chng minh.


ng thc xy ra khi v ch khi a= b= c .
Bi 7. Cho a,b,c,x,y,z l cc s thc dng tho mn iu kin

(a

( a + b + c )( x + y + z )=
Chng minh rng abcxyz <

+ b2 + c2

)( x

+ y2 + z2 = 4 .

1
.
36

Li gii
S dng bt ng thc AM GM ta c
4 ( ab + bc + ca )( xy + yz + zx )

2
2
= ( a + b + c ) a 2 + b 2 + c 2 ( x + y + z ) x 2 + y 2 + z 2

2
2
= 20 ( a + b + c ) x 2 + y 2 + z 2 + a 2 + b 2 + c 2 ( x + y + z )

) (

20 2 ( a + b + c )( c + y + z )

(a

+ b2 + c2

)( x

+ y2 + z2 =
4

Do ( ab + bc + ca )( xy + yz + zx ) 1 .
Mt khc ( ab + bc + ca ) 3abc ( a + b + c ) .
2

( xy + yz + zx )2 3xyz ( x + y + z )
Nhn theo v 2 bt ng thc trn ta c
1
.
36
ng thc xy ra khi v ch khi a= b= c; x= y= z khi tri vi gi thit
9abcxyz ( a + b + c )( x + y + z ) 1 abcxyz

( a + b + c )( x + y + z )=
Vy abcxyz <

(a

+ b2 + c2

)( x

+ y2 + z2 = 4 .

1
.
36
147

www.TaiLieuLuyenThi.com
Khm ph t duy K thut gii bt T Bi ton Max Min ng Thnh Nam

1 + a2

Bi 8. Cho a, b, c > 1. Chng minh

1 + b + c2
Li gii

1 + b2
1 + c + a2

1 + c2
1 + a + b2

2.

S dng bt ng thc AM GM ta c 1 + b + c 2 > 0,1 + b + c 2 1 +


Suy ra

1 + a2
1+ b + c

Tng t ta c

2 1 + a2

1 + b2 + 2 1 + c2

1 + b2
1+ c + a

1 + c2
1+ a + b

2 1 + b2

1 + c + 2 1 + a2
2

1 + b2
+ c2 .
2

2 1 + c2

1 + a 2 + 2 1 + b2

)
)

Cng theo v 3 bt ng thc trn v t x =a 2 + 1, y =b 2 + 1, z =c 2 + 1 ta c


1 + a2

1 + b2

1 + c2

1+ b + c
1+ c + a
1+ a + b
S dng bt ng thc C S ta c
2

2x
2y
2z
.
+
+
y + 2z z + 2x x + 2 y

(x + y + z)
x
y
z
+
+

y + 2z z + 2x x + 2 y x ( y + 2z ) + y ( z + 2x) + z ( x + 2 y )
2

( x + y + z )2 3 ( xy + yz + zx ) = 1
3 ( xy + yz + zx ) 3 ( xy + yz + zx )

Bt ng thc c chng minh. ng thc xy ra khi v ch khi a= b= c= 1 .


Bi 9. Cho a,b,c l cc s thc khng m tha mn iu kin ab + 2bc + 3ca =
6.
Tm gi tr nh nht ca biu thc P =

( a + b )( b + c )( c + a ) + 4a + b + c .

Li gii
S dng bt ng thc AM-GM ta c
P2

( a + b )( b + c )( c + a ) .( 4a + b + c ) .

Mt khc s dng bt ng thc C-S ta c

( a + b )( b + c )( c + a ) .( 4a + b + c=)

a ( b + c )2 + b ( c a )2 + c ( a + b )2 . ( 4a + b + c )

2a ( b + c ) + b ( c a ) + c ( a + b )
= ( ab + 2bc + 3ca ) = 36
2

148

www.TaiLieuLuyenThi.com
Cty TNHH MTV DVVH Khang Vit

ng thc xy ra khi v ch khi =


a 1,=
b 0,=
c 2.
Vy gi tr nh nht ca P bng 12 t ti =
a 1,=
b 0,=
c 2.
Bi 10. Cho cc s thc a,b,c thuc on [ 0;1] .
Tm gi tr ln nht v nh nht ca biu=
thc P a5b5c5 ( 3 ( ab + bc + ca ) 8abc ) .
Li gii
S dng bt ng thc AM-GM ta c:
2 2 2
3 ( ab + bc + ca ) 3.3 a=
b c
9 a 2b 2 c 2 9abc 8abc .
3

Suy ra P 0 . Vy gi tr nh nht ca P bng 0 xy ra khi c t nht mt s


bng 0.
S dng bt ng thc AM-GM ta c:
P

abc.abc.abc.abc.abc 3 ( ab + bc + ca ) 8abc

6
6
ab + bc + ca abc ) .
3 ( ab + bc + ca ) 8abc + 5abc
(

=
6
26

Mt khc:

ab + bc + ca abc = a ( b + c bc ) + bc = a b + c (1 b ) + bc
.
b + c bc + bc = b + c 2
Do P 1 . Vy gi tr ln nht ca P bng 1 xy ra khi a= b= c= 1 .
Bi 11. Cho a,b,c l cc s thc dng c tch bng 1. Chng minh
7
( a + b )( b + c )( c + a ) ( a + b + c ) + 1 .
3
Li gii
8
S dng b . ( a + b )( b + c )( c + a ) ( a + b + c )( ab + bc + ca ) , a, b,c 0
9
ta c:
8
8
( a + b )( b + c )( c + a ) ( a + b + c )( ab + bc + ca ) ( a + b + c ) .33 a 2b2c 2
9
9
8
7
1
= ( a + b + c=
) (a + b + c) + (a + b + c)
3
3
3
7
7
( a + b + c ) + 3 abc=
(a + b + c) +1
3
3
Bt ng thc c chng minh. ng thc xy ra khi v ch khi a= b= c= 1 .
Nhn xt. Ngoi ra ta c bt ng thc mnh hn nh sau. Vi gi thit cng iu
kin bi ton ta c ( a + b )( b + c )( c + a ) 4 ( a + b + c 1) .

149

www.TaiLieuLuyenThi.com
Khm ph t duy K thut gii bt T Bi ton Max Min ng Thnh Nam

Chng minh.
Ta c:
( a + b )( b + c )( c + a ) =
Mt khc:

( a + b + c )( ab + bc + ca ) abc = ( a + b + c )( ab + bc + ca ) 1

( ab + bc + ca )2 3abc ( a + b + c )= 3 ( a + b + c ) ab + bc + ca

3( a + b + c ) .

Vy bi ton c chng minh nu bt ng thc sau ng:

( a + b + c ) 3 ( a + b + c ) 1 4 ( a + b + c 1) .
Tht vy, t t =

a + b + c , t 3 . Xt hm s

f (t ) =

3t 3 4t 2 + 3 vi

t 3.

Ta c f=
'(t ) 3 3t 2 8t > 0, t 3 nn f (t ) ng bin trn 3; + hay
f (t ) f ( 3) =
0.

Do ( a + b + c ) 3 ( a + b + c ) 1 4 ( a + b + c 1) .
Bt ng thc c chng minh. Ngoi ra bi ton trn c th thc hin bng
phng php dn bin(xem chng 4).
Bi 15. Cho x,y,z l cc s thc dng tha mn iu kin
8.
( x + y )( y + z )( z + x ) =
Tm gi tr nh nht ca biu thc P =

2
1
1
1
.
+
+
+
xyz x + 2 y y + 2 z z + 2 x

Li gii
S dng bt ng thc AM-GM ta c:
1
1
1
9
3
+
+

= .
x + 2 y y + 2z z + 2x x + 2 y + y + 2z + z + 2x x + y + z
Mt khc:
8
9

( x + y )( y + z )( z + x ) ( x + y + z )( xy + yz + zx ) x + y + z

9
.
xy + yz + zx

Suy ra
P

2
xy + yz + zx
+

3
xyz

2
+ 3 x2 y 2 z 2 =
xyz

1
+
xyz

1
+ 3 x2 y 2 z 2 3 .
xyz

Vy gi tr nh nht ca P bng 3 t ti x= y= z= 1 .
Bi 16. Cho a,b,c l cc s thc khng m tha mn iu kin a + b + c =
3.
Chng minh rng

a + b + c ab + bc + ca .
Li gii

Ta c: 2 ( ab + bc + ca ) =( a + b + c ) a 2 + b 2 + c 2 =9 a 2 + b 2 + c 2 .
2

150

www.TaiLieuLuyenThi.com
Cty TNHH MTV DVVH Khang Vit

Vy ta chng minh bt ng thc: a 2 + b 2 + c 2 + 2

a + b + c 9.

Theo bt ng thc AM-GM ta c:


a 2 + a + a 3a, b 2 + b + b 3b, c 2 + c + c 3c .
Cng theo v ba bt ng thc trn ta c:

a 2 + b2 + c2 + 2 a + 2 b + 2 c 3( a + b + c ) =
9.

Bt ng thc c chng minh. ng thc xy ra khi v ch khi a= b= c .


Bi 17. Cho x,y,z,t l cc s thc khng m tha mn iu kin x + y + z + t =
4.
Tm gi tr ln nht v gi tr nh nht ca biu thc
P= x 3 + yz + y 3 + zt + z 3 + tx + t 3 + xy .

Li gii
Do x,y,z,t khng m nn
P x 3+ y 3+ z 3+t 3 =

(x + y + z + t)

3 = 4 3.

Vy gi tr nh nht ca P bng 4 3 t ti x = 4, y = z = t = 0 hoc cc hon v.


tm gi tr ln nht ca P ta s dng bt ng thc AM-GM cho hai s dng:
1
2 x 3 x + xyz + 2 y 3 y + yzt + 2 z 3z + ztx + 2 t 3t + txy
P
=
2
1
( 4 x + 3 x + xyz + 4 y + 3 y + yzt + 4 z + 3z + ztx + 4t + 3t + txy )
4
7
1
1
= ( x + y + z + t ) + ( xyz + yzt + ztx + txy ) =7 + ( xyz + yzt + ztx + txy )
4
4
4
Mt khc:

x+ y
z+t
xyz + yzt + ztx + txy= xy ( z + t ) + zt ( x + y )
(z + t) +
( x + y)
2
2
2

1
x+ y+ z+t
4
=( x + y )( z + t )( x + y + z + t ) =
( x + y )( z + t )
=
4
2

Vy gi tr ln nht ca P bng 8 t ti x = y = z = t = 1 .
Bi 18. Cho cc s thc a,b,c thay i tha mn iu kin a 2 + b 2 + c 2 =
1.
Tm gi tr ln nht, gi tr nh nht ca biu thc
P = a3 + b3 + c3 3abc .
Li gii

Ta c: P =

( a + b + c ) ( a 2 + b2 + c 2 ab bc ca ) = ( a + b + c )(1 ab bc ca ) .
( a + b + c )2 (1 ab bc ca )2
=(1 + 2ab + 2bc + 2ca )(1 ab bc ca )(1 ab bc ca )

Suy ra: P 2 =

151

www.TaiLieuLuyenThi.com
Khm ph t duy K thut gii bt T Bi ton Max Min ng Thnh Nam

Do ab + bc + ca a 2 + b 2 + c 2 =
1 nn cc s trong tch ca P 2 khng m ta s
dng bt ng thc AM-GM ta c:

1 + 2ab + 2bc + 2ca + 2 (1 ab bc ca )


P2
1.
=
3

Vy gi tr nh nht ca P bng 1 t ti a =
1, b =
c=
0 hoc cc hon v.
Gi tr ln nht ca P bng 1 t ti a= 1, b= c= 0 hoc cc hon v.
Bi 19. Cho a,b,c l cc s thc khng m tha mn iu kin a + b + c =
1.
3

)(

)(

Tm gi tr ln nht ca biu thc P =


a 3 + b3 b3 + c 3 c 3 + a 3 .
Li gii
Khng mt tnh tng qut ta gi s c = min {a, b, c} .
Khi :

)(

)(

P a3 + b3 b3 + b 2 c a3 + a 2 c = ( a + b ) a 2 ab + b 2 ( a + c )( b + c ) a 2b 2

= ( a + b )( b + c )( c + a ) a 2 ab + b 2 a 2b 2

Mt khc:
( a + b )( b + c )( c + a ) =
Suy ra:

( a + b + c )( ab + bc + ca ) abc ( a + b + c )( ab + bc + ca )

P ( a + b + c )( ab + bc + ca ) a 2 ab + b 2 a 2b 2
ab + bc + ca + a 2 ab + b 2 + ab + ab
( a + b + c ) .

( a + b + c ) ( a + b )2 + c ( a + b ) ( a + b )4 ( a + b + c )5 ( a + b + c )9
256

256

1
=
256
256

1
1
t ti a= b=
, c= 0 hoc cc hon v.
256
2
1
Bi 20. Cho a,b,c,d,e,f l cc s thc khng m c tng bng 1 v ace + bdf
.
108
1
Chng minh rng abc + bcd + cde + def + efa + fba
.
36
Li gii
S dng bt ng thc AM-GM cho ba s dng ta c:
abc + bcd + cde + def + efa + fba + ace + bdf =( a + d )( b + e )( c + f )

Vy gi tr ln nht ca P bng

1
a+d +b+e+c+ f

=
3
27

152

www.TaiLieuLuyenThi.com
Cty TNHH MTV DVVH Khang Vit

Suy ra abc + bcd + cde + def + efa + fba

1
1
1
1
( ace + bdf )

=.
27
27 108 36

Bt ng thc c chng.
1
.
6
Bi 21. Cho cc s thc khng m a,b,c tha mn iu kin ab + bc + ca > 0 v
a + 2b + 3c =
4.
1
1
.
Tm gi tr nh nht ca=
biu thc P
+
ab + bc + ca
ab + bc + c 2

ng thc xy ra khi v ch khi a= b= c= d= e= f =

Li gii
Nhn xt. P nh nht t a + 2b + 3c =
4 ta s suy ra a ln nht v c nh nht.
1
1
2
D on vi c =0 P =
+
=
, a + 2b =4 . P lc ny t gi tr
ab
ab
ab
nh nht ti=
a 2,=
b 1.
Ta c:
1
1
2
+

P=
ab + bc + ca
ab + bc + c 2 4 ( ab + bc + ca ) ab + bc + c 2

2 2

=
2ab + 2bc + ca + c 2

2 2

( a + c )( 2b + c )

Vy gi tr nh nht ca P bng

4 2
4 2

=
a + 2b + 2c a + 2b + 3c

2 xy ra khi

c = 0
a = 2
2b + c = a + c

b =
1.

2
ab + bc + ca = ab + bc + c
c = 0

a + 2b + 3c =
4

Bi 22. Cho a,b,c l cc s thc tha mn 0 < a b c v


a 2 1 b2 1 c2 1
+
+
=
0.
a
b
c

Tm gi tr nh nht ca biu thc P =


a + b 2014 + c 2015 .
Li gii
T 0 < a b c suy ra 0 =
a 1 v 0 3.

a 2 1 b2 1 c2 1
a2 1
+
+
3.
a
b
c
a

c2 1
c 1.
c
153

www.TaiLieuLuyenThi.com
Khm ph t duy K thut gii bt T Bi ton Max Min ng Thnh Nam

Ta c:
b2 1 c2 1 1 a 2
1 1
1

+ =
0 b + c 0 ( b + c ) 1 0 bc 1 .
b
c
a
b c
bc

Mt khc:
c2 1 1 a 2 1 b2
1

=
+
= ( a + b ) 1 2 ab 1
c
a
b
ab
ab
1
1

= 2
ab
ab
ab
ab

1
1
1
ab
1

Suy ra c
ab c
abc 2 1 .
1 +
0 c

c
c
ab
ab
ab

Khi s dng bt ng thc AM-GM ta c:

P=
a + b 2014 + c 2015 3 ab 2014 c 2015 =
33 abc 2 ( bc )
3

2013

3.

Vy gi tr nh nht ca P bng 3 t ti a= b= c= 1 .
Bi 23. Cho a,b,c,d l cc s thc dng.
Chng minh rng

b(a + c)
c (a + b)

c (b + d )
d (b + c )

d (c + a)
a (c + d )

a (d + b)

b(d + a)

4.

Li gii
t P l v tri ca bt ng thc.
Vit li P di dng:
b

d
c
a
+
+
P = (a + c)
+ (b + d )

c ( a + b ) a ( c + d )
d ( b + c ) b ( d + a )

a+c
b+d
= ( abc + bcd + cda + dab )
+

ac ( a + b )( c + d ) bd ( b + c )( a + d )

1 1
1 1
+
+

1 1 1 1
a c
b d
.

= + + +
+
a b c d 1 + 1 1 + 1 1 + 1 1 + 1

a b
c d b c a d

1 1
1 1
4 +
4 +

1 1 1 1
a c
b d
=
+
4
+ + + .
2
2

a
b
c
d

1 1 1 1
1 1 1 1

+ + +
+ + +
a b c d
a b c d

Bt ng thc c chng minh. ng thc xy ra khi v ch khi a= b= c= d .

154

www.TaiLieuLuyenThi.com
Cty TNHH MTV DVVH Khang Vit

Bi 24. Cho a,b,c l cc s thc khng m. Chng minh


a 3 + b3 + c 3
3
abc + ( a b )( b c )( c a ) .
3
4
Li gii
Bt ng thc tng ng vi:
a3 + b3 + c3 3abc 3
( a b )( b c )( c a ) .
3
4
( a + b + c ) ( a b )2 + ( b c )2 + ( c a )2 3

( a b )( b c )( c a ) .
6
4
S dng bt ng thc AM-GM cho v tri ta c:
( a + b + c ) ( a b )2 + ( b c )2 + ( c a )2 ( a + b + c ) 3 ( a b )2 ( b c )2 ( c a )2

6
2
Vy ta chng minh

( a + b + c ) 3 ( a b )2 ( b c )2 ( c a )2
2

3
( a b )( b c )( c a ) .
4

2 ( a + b + c ) 33 ( a b )( b c )( c a ) .

Khng mt tnh tng qut gi s a b c ta cn chng minh


2 ( a + b + c ) 3 3 ( a b )( b c )( a c ) .

S dng bt ng thc AM-GM cho v phi ta c:


3 3 ( a b )( b c )( a c ) a b + b c + a c = 2a 2c 2 ( a + b + c ) .

Bt ng thc c chng minh. ng thc xy ra khi v ch khi a= b= c .


Mt s kt qu tng t:
Bi 1. Cho a,b,c l cc s thc khng m. Chng minh
a 3 + b3 + c 3
1
abc + 3 ( a b )( b c )( c a ) .
3
2

Bi 2. Cho a,b,c l cc s thc khng m tha mn 2 min {a, b, c} max {a, b, c} .


a 3 + b3 + c 3
3
abc + 3 ( a b )( b c )( c a ) .
3
2
Bi 25. Cho a,b,c l cc s thc khng m tha mn iu kin a + b + c =
1.
Tm gi tr nh nht ca biu thc
1
1
1
.
P=
+
+
2
2
2
(b c )
(c a)
(a b)
a2 +
b2 +
c2 +
4
4
4

Chng minh

155

www.TaiLieuLuyenThi.com
Khm ph t duy K thut gii bt T Bi ton Max Min ng Thnh Nam

Li gii
Khng mt tnh tng qut gi s c = max {a, b, c} .
Ta c:
c2 +

a2 +

( b c )2

( a b )2
4

(c a) b + c .
c
a + , b2 +
2
4
2
2

c
c

c 2 + 2a 2 + 2b 2 2 a + + 2 b + .
2
2

c
c
t x =
a+ ,y=
b + , ( x, y > 0 ) v x + y =
1.
2
2
1 1
1
5
.
Ta chng minh c: + +

2
2
x y
x
+
y
2 x +y

Do P 5 .
Vy gi tr nh nht ca P bng 5 t ti a= b= 0, c= 1 hoc cc hon v.
C. BI TP RN LUYN
BI TP TRUNG BNH
Bi 1. Cho x,y l hai s thc dng. Tm gi tr nh nht ca biu thc

P=

x 3 + y 3 + 7 xy ( x + y )
xy x 2 + y 2

Bi 2. Cho x,y l hai s thc dng.

Chng minh rng x 2 + y +

3 2
3
1
1
y + x + 2 x + 2 y + .
4
4
2
2

Bi 3. Cho a,b l cc s thc dng. Chng minh

(1 + a ) + (1 + b )
8

128ab ( a + b ) .
2

Bi 4. Chng minh rng vi mi s thc x,y,z ta c

x2 + y 2 + z 2 2 x ( y + z ) .
Bi 5. Cho a,b,c l cc s thc tha mn iu kin ( a + c )( a + b + c ) 0 .
Chng minh rng ( b c ) 4a ( a + b + c ) .
2

Bi 6. Cho a,b,c l cc s thc khng m tha mn iu kin


ab + bc + ca + abc =
4.

Chng minh rng 3 a 2 + b 2 + c 2 + abc 10 .


156

www.TaiLieuLuyenThi.com
Cty TNHH MTV DVVH Khang Vit

Bi 7. Cho a,b l cc s thc tho mn iu kin 2 < a < 2, 3 < b < 3; ab =1 .


Chng minh rng

4
9
12
+
.
2
2
4a 9b
5

Bi 8. Cho x,y l hai s thc dng c tng bng 2. Chng minh

1
1
2
2
+
+ 2
2+
.
2
x + xy + y
3
x
y
a
b
c
+
+
=
2.
a +1 b +1 c +1

Bi 9. Cho a,b,c l cc s thc dng tha mn iu kin


Tm gi tr nh nht ca biu thc P = ab + bc + ca .
Bi 10. Cho a,b,c l cc s thc dng tha mn iu kin

1
1
1
+
+
=
1.
a +1 b +1 c +1
Chng minh rng abc 8 .
Bi 11. Chng minh rng

x ( 25 y 2 ) + y 25 x 25 .

Bi 12. Chng minh rng

x (108 y 2 ) + y 108 x 108 .

Bi 13. Cho x,y l hai s thc tho mn iu kin

x 2014 y 2 + y 2014 x 2 =
2014 .
Chng minh rng : x 2 + y 2 =
2014 .
Bi 14. Cho hai s thc x > y > 0. Chng minh x +

( x y )( y + 1)

3.

Bi 15. Cho a,b,c l cc s thc dng tho mn iu kin a + b + c 3 .

1
2014
2015
.
+

2
2
a + b + c ab + bc + ca
3
Bi 16. Cho a,b,c l cc s thc dng tho mn iu kin a 3 + b3 + c 3 3 .
1
8
4
Chng minh rng 3
+
.
3
3
a + b + c ( a + b )( b + c )( c + a ) 3
Chng minh rng

Bi 17. Cho a,b l hai s thc dng tho mn iu kin


Chng minh rng ab 2

1
2
+
=
2.
a +1 b +1

1
.
8

157

www.TaiLieuLuyenThi.com
Khm ph t duy K thut gii bt T Bi ton Max Min ng Thnh Nam

1
1
1
1.
+
+ ... +
=
1 + a1 1 + a2
1 + an

Bi 18. Cho n s thc dng tho mn iu kin


Chng minh rng a1a2 ...an ( n 1) .
n

Bi 19. Cho a,b,c l cc s thc dng. Chng minh

a+b+c
a 2 + b 2 + c 2 1 ab bc ca

+ + .
3
3
3 c
a b
Bi 20. Cho a,b,c l cc s thc dng tha mn iu kin a 2 + b 2 + c 2 =
1.
Chng minh a + b + c +

1
4 3.
abc

Bi 21. Cho a,b,c l di ba cnh mt tam gic c chu vi bng 1.


Tm gi tr nh nht ca biu thc

(a + b c)
P=

3c

(b + c a )
+

3a

(c + a b)
+

3b
Bi 22. Cho a,b,c l cc s thc khng m tha mn a + b + c =
3.
2
2
2
Chng minh rng a b + b c + c a + abc 4 .
Bi 23. Cho a,b,c l cc s thc khng m tha mn a + b + c =
3.
2
2
2
Chng minh rng a b + b c + c a + 2abc + 4 3 ( ab + bc + ca ) .
Bi 24. Cho a,b,c l cc s thc khng m. Chng minh rng

( a b + b c + c a ) ( ab + bc + ca ) 271 ( a + b + c )
2

Bi 25. Cho a,b,c l cc s thc dng. Tm gi tr ln nht ca biu thc

P=

abc a + b + c + a 2 + b 2 + c 2

(a

+b +c
2

) ( ab + bc + ca )

).

Bi 26. Cho a,b,c l cc s thc tha mn iu kin abc=1 .


Chng minh rng:

b+c c+a a+b


+
+
a + b + c +3.
a
b
c

Bi 27. Cho a,b,c l cc s thc dng tho mn iu kin a 2 + b 2 + c 2


Tm gi tr nh nht ca biu thc

P = ( a + b )( b + c )( c + a ) +

1 1 1
+ + .
a 2 b2 c2

Bi 28. Cho a,b,c l cc s nguyn dng c tng bng 100.


Tm gi tr ln nht ca tch ba s .
158

3
.
4

www.TaiLieuLuyenThi.com
Cty TNHH MTV DVVH Khang Vit

Bi 29. Cho a,b,c l cc s nguyn dng c tng bng 2014.


Tm gi tr ln nht ca biu thc P = abc .
Bi 30. Cho x,y,z l cc s thc dng. Chng minh rng

1
1
1
27
.
+ 2
+ 2

x + xy y + yz z + zx 2 ( x + y + z )2
2

Bi 31. Cho a,b,c l cc s thc dng tho mn iu kin ab + bc + ca =


3.
Chng minh rng a 6 + b 6 + 1 + b 6 + c 6 + 1 + c 6 + a 6 + 1 3 3 .
Bi 32. Cho a,b,c l cc s thc dng chng minh

a 2 + 2b 2 + b 2 + 2c 2 + c 2 + 2a 2 3 ( a + b + c ) .
Bi 33. Cho n s thc dng c tng bng 1. Chng minh

1
1
1
n
1 1 ... 1 ( n 1) .
a1 a2 an
Bi 34. Cho a,b l cc s thc dng tho mn iu kin a + b 1 .
1
1
.
Tm gi tr nh nht ca biu
thc P
=
+
2
2
1 + a + b 2ab
Ch . C th a v hm s(xem chng sau).
Bi tp tng t
Cho a,b l hai s thc dng tho mn iu kin a + b 2 .

1
.
1 + a + b ab
Bi 35. Cho a,b,c l cc s thc dng tho mn iu kin a + 2b + 3c 20 .
3 9 4
Tm gi tr nh nht ca biu thc P = a + b + c + +
+ .
a 2b c
Tm gi tr nh nht ca biu
thc P
=

Bi 36. Cho x,y,z l cc s thc dng. Chng minh

2 xy
2 yz
3 xz
5
+
+
.
( x + z )( z + y ) ( x + y )( x + z ) ( y + z )( y + x) 3
Bi 37. Cho a,b,c l cc s thc tho mn a, b, c 1 .
Chng minh rng

a ( b 2 + 3)
3c 2 + 1

b ( c 2 + 3)
3a 2 + 1

c ( a 2 + 3)
3b 2 + 1

3.

Bi 38. Cho x, y, z l cc s thc dng c tch bng 1.


Tm gi tr nh nht ca biu thc

159

www.TaiLieuLuyenThi.com
Khm ph t duy K thut gii bt T Bi ton Max Min ng Thnh Nam

x4

y4
z4
.
P=
+
+
(1 + y )(1 + z ) (1 + z )(1 + x ) (1 + x )(1 + y )
Bi 38. S dng bt ng thc AM GM cho bn s dng ta c:

x4
1+ y 1+ z 1

(1 + y )(1 + z ) + 8 + 8 + 4 x

y4
1+ z 1+ x 1

+
+
+ y
8
8
4
(1 + z )(1 + x )

z4
1+ x 1+ y 1

+
+
+ z
(1 + x )(1 + y )
8
8
4

Cng li theo v ba bt ng thc trn ta c:


Bi 39. Cho a,b,c l cc s thc dng tho mn iu kin a + b + c

1 1 1
+ + .
a b c

3
2
.
+
a + b + c abc

Chng minh rng a + b + c

Bi 40. Cho x,y l cc s thc dng. Chng minh


6

x4 y 4

( x + y ) ( x + 1) ( y + 1)
2

3
.
8

Bi 41. Cho x,y,z l cc s thc dng tho mn iu kin z ( z x y ) = x + y + 1 .


Tm gi tr ln nht ca biu thc M =

x4 y 4
.
( x + yz )( y + zx)( z + xy )3

Bi 42. Cho x,y,z l cc s thc tho mn iu kin x 2 + y 2 + z 2 =


3.
Chng minh rng x + y + z xyz 4 .
Bi 43. Cho a,b,c l cc s thc dng c tng bng 3. Chng minh

a +1
b +1
c +1
+
+
3.
ab + 1
bc + 1
ca + 1
Bi 44. Cho a,b,c l cc s thc dng c tng bng 3. Chng minh

a ( a + c 2b ) b ( b + a 2c ) c ( c + b 2a )
+
+
0.
ab + 1
bc + 1
ca + 1

Bi 45. Cho x,y,z l cc s thc ln hn 1. Chng minh

x4

( y 1)
160

y4

( z 1)

z4

( x 1)

48 .

www.TaiLieuLuyenThi.com
Cty TNHH MTV DVVH Khang Vit

3
4

Bi 46. Cho a,b,c l cc s thc dng tho mn iu kin a + b + c = abc .


Chng minh rng

a
b
c
3
+
+
.
a+2 b+2 c+2 2

Bi 47. Cho a,b l hai s thc dng tho mn iu kin a + b 4 .


Tm gi tr nh nht ca biu thc
=
P

3a 2 + 4 2 + b3
+ 2 .
4a
b

Bi 48. Cho a,b,c l cc s thc dng tm gi tr ln nht ca biu thc

Bi 49. Cho cc s thc dng a, b, c tha mn iu kin

.
Chng minh rng

Bi 50. Cho x,y l hai s thc dng. Chng minh rng

.
Bi 51. Cho

. Tm gi tr ln nht ca biu thc


.

Bi 52. Cho a,b,c l cc s thc dng c tng bng

3
. Chng minh
4
.

Bi 53. (TSH Khi B 2007) Cho x,y,z l cc s thc dng. Tm gi tr nh nht


ca biu thc

Bi 54. Cho a,b,c l cc s thc khng m tho mn iu kin

.
.

Chng minh rng


Bi 55. Cho a,b,c l cc s thc khng m c tng bng 3. Chng minh
.

161

www.TaiLieuLuyenThi.com
Khm ph t duy K thut gii bt T Bi ton Max Min ng Thnh Nam

Bi 56. Cho a,b,c l cc s thc dng c tng bng 3. Chng minh

.
Bi 57. Cho a,b,c l cc s thc dng tho mn iu kin

Chng minh rng

BI TP RN LUYN NNG CAO

1.
Bi 58. Cho x,y,z l cc s thc dng tha mn x > y v xy + ( x + y ) z + z 2 =
Tm gi tr nh nht ca biu thc P =

1
4( x y)

(x + z) ( y + z)
Bi 59. Cho a,b,c l cc s thc dng tha mn a > b v ( a + c )( b + c ) =
1.
Tm gi tr nh nht ca biu thc P =

(a b)

(b + c )

12

(a + c)

Bi 60. Cho a,b,c,x,y,z l cc s thc dng tho mn iu kin


.

Chng minh rng

Bi 61. Cho x,y,z l cc s thc dng. Chng minh rng


.
Bi 62. Cho a,b,c l cc s thc khng m tho mn iu kin

Chng minh rng

Bi 63. Cho a,b,c l cc s thc khng m tho mn iu kin

.
.

Chng minh rng


Bi 64. Cho s nguyn dng n v cc s thc dng

Chng minh rng

.
Bi 65. Cho a,b,c l cc s thc dng. Chng minh
.
162

www.TaiLieuLuyenThi.com
Cty TNHH MTV DVVH Khang Vit

Bi 66. Cho a,b,c l cc s thc khng m tha mn iu kin ab + bc + ca > 0 .


Tm gi tr nh nht ca biu thc

1
1

.
P = ( a + 2b + 2014c )
+
a ( b + c ) + 2c 2

b
a
+
4
c
(
)

Bi 67. Cho a,b,c l cc s thc i mt phn bit tha mn ab + bc + ca > 0 v


a+b+c =
1.
Tm gi tr nh nht ca biu thc

P=

2
2
2
5
.
+
+
+
a b bc ca
ab + bc + ca

Bi 68. Cho x,y,z l cc s thc khng m tho mn iu kin


.
Chng minh rng

Bi 69. Cho a,b,c l cc s thc dng tho mn iu kin

.
Chng minh rng

Bi 70. Cho a,b,c l di 3 cnh mt tam gic tho mn iu kin

.
Chng minh rng

Bi 71. Cho a,b,c l cc s thc khng m. Chng minh


.
Bi 72. Cho x,y,z l cc s thc dng tho mn iu kin

.
.

Chng minh rng


Bi 73. Cho a,b,c l di 3 cnh mt tam gic c chu vi bng 3.
Chng minh rng

Bi 74. Cho x,y,z l cc s thc khng m tho mn iu kin


.
163

www.TaiLieuLuyenThi.com
Khm ph t duy K thut gii bt T Bi ton Max Min ng Thnh Nam

Tm gi tr nh nht ca biu thc

.
Bi 75. Cho a,b,c l cc s thc khng m tho mn iu kin

Chng minh rng

Bi 76. Cho a,b,c l cc s thc khng m tho mn iu kin

.
.

Chng minh rng


Bi 79. Cho a,b,c l cc s thc dng c tng bng 3.

Chng minh rng

Bi 80. Cho a,b,c l cc s thc dng tho mn iu kin


.
Chng minh rng

Bi 81. Cho a,b,c l cc s thc dng tho mn iu kin


.

Tm gi tr nh nht ca biu thc

Bi 82. Cho a,b,c l cc s thc khng m c tng bng 2 chng minh


.
Bi 83. Cho x,y,z l cc s thc khng m tho mn iu kin

Tm gi tr nh nht ca biu thc


.
Bi 84.(Vit Nam TST 2010) Cho a,b,c l cc s thc dng tho mn iu kin
.
Chng minh rng
.

164

www.TaiLieuLuyenThi.com
Cty TNHH MTV DVVH Khang Vit

Bi 85. Cho a,b,c l cc s thc dng c tng bng 1. Chng minh


.
Bi 86. Cho a,b,c l cc s thc dng c tng bng 1. Chng minh
.
Bi 87. Cho a,b,c l cc s thc dng c tng bng 3. Chng minh
.
Bi 88. Cho a,b,c l cc s thc dng c tng bng 1. Chng minh

.
Bi 89. Cho a,b,c l cc s thc khng m tho mn iu kin
Chng minh rng

Bi 90. Cho a,b,c l cc s thc khng m c tng bng 3. Chng minh


.
Bi 91. Cho a,b,c l cc s thc dng. Chng minh rng

.
Bi 92. Cho a,b,c l di ba cnh mt tam gic c chu vi bng 3. Chng minh
.
Bi 93. Cho a,b,c l cc s thc dng. Chng minh
.
Bi 94. Cho a,b,c l cc s thc khng m tho mn iu kin
Chng minh rng

.
Bi 95. Cho a,b,c l cc s thc tho mn iu kin
.
165

www.TaiLieuLuyenThi.com
Khm ph t duy K thut gii bt T Bi ton Max Min ng Thnh Nam

Chng minh rng

Bi 96. Cho a,b,c l cc s thc dng tho mn iu kin

Chng minh rng

Bi 97. Cho x,y,z l cc s thc dng c tch bng 1. Chng minh


.
Bi 98. Cho a,b,c l cc s thc dng. Chng minh

.
Bi 99. Cho x,y,z l cc s thc dng c tch bng 1. Chng minh
.
Bi 100. Cho a,b,c l cc s thc dng c tng bng 3. Chng minh

D. HNG DN GII P S
Bi 1. S dng bt ng thc AM-GM ta c:

x 3 + y 3 + 7 xy ( x + y ) =
( x + y ) + 4 xy ( x + y ) 4 xy ( x + y ) .
3

xy=
x +y
2

xy
. 2 xy ( x 2 + y 2 )
2

xy 2 xy + x 2 + y 2
=
.
2
2

xy ( x + y )
2 2

Do P 8 2 . ng thc xy ra khi v ch khi x = y .


Nhn xt. Ngoi ra ta c th t
=
x t. y, ( t > 0 ) a v kho st hm mt bin vi t.
Bi 2. Ta c:
2

1
1 2
3
1
3
1

2
2
x = x x + = x + y + x + y + x + y + x + y +
2
4
4
2
4
2

166

www.TaiLieuLuyenThi.com
Cty TNHH MTV DVVH Khang Vit

Tng t y 2 + x +

3
1
x+ y+ .
4
2

Suy ra:
2

3 2
3
1
1
1
2
x + y + y + x + x + y + = x + + y +
4
4
2
4
4

1
1
1
1

4 x + y + = 2 x + 2 y +
4
4
2
2

1
.
2

Bt ng thc c chng minh. ng thc xy ra khi v ch khi x= y=


Bi 3. S dng bt ng thc AM GM cho hai s dng ta c

(1 + a ) + (1 + b )
8

2 (1 + a ) (1 + b )
4

4
= 2 ( ab + 1) + ( a + b ) 2 2

( ab + 1)( a + b )

= 32 ( ab + 1) ( a + b ) 128ab ( a + b )
2

Bt ng thc c chng minh. ng thc xy ra khi v ch khi


Bi 4. S dng bt ng thc AM GM ta c

.
Bi 5. Ta c: ( b c ) =
2

( a + b + c a 2c )

4 ( a + b + c )( a 2c ) .

Vy bt ng thc c chng minh nu ta chng minh c:

( a + b + c )( a 2c ) a ( a + b + c ) ( a + c )( a + b + c ) 0 (lun ng theo

gi thit).
Bi 6. Ta c:

1
a+b+c
2
( a + b + c ) +
a +b+c 3.
3
3

4 = ab + bc + ca + abc
Khi

3 ( a 2 + b 2 + c 2 ) + abc= 3 ( a 2 + b 2 + c 2 ) + 4 ( ab + bc + ca )

1
2
(a + b + c)
3
2
2
2
=
( a + b + c ) + 4 .32 + 4= 10
3
3
Bt ng thc c chng minh ng thc xy ra khi v ch khi a= b= c= 1 .
(a + b + c) + 4
2

167

www.TaiLieuLuyenThi.com
Khm ph t duy K thut gii bt T Bi ton Max Min ng Thnh Nam

Bi 7. S dng bt ng thc AM GM ta c

Bt ng thc c chng minh. ng thc xy ra khi v ch khi

hoc

Bi 8. S dng bt ng thc AM GM ta c

Cng theo v hai bt ng thc trn ta c pcm.


Bi 9. Theo gi thit ta c

a
b
c
1
1
=
1
+1
= +

a +1
b +1
c +1 b +1 c +1
Tng t ta c:

b +1

( a + 1)( c + 1)

Nhn theo v hai bt ng thc trn ta c:


168

( b + 1)( c + 1)

www.TaiLieuLuyenThi.com
Cty TNHH MTV DVVH Khang Vit

ab

( a + 1)( b + 1) ( c + 1)
Tng t ta c: bc

( a + 1)( b + 1)

( b + 1)( c + 1)
a +1

; ca

ab

( a + 1)( b + 1)

c +1

( c + 1)( a + 1)
b +1

Cng li theo v 3 bt ng thc trn v s dng AM-GM ta c:


P = ab + bc + ca 12 .
ng thc xy ra khi v ch khi a= b= c= 2 .

a
1
1
1
.
=
1
= +
a +1
a +1 b +1 c +1
a
2
S dng bt ng thc AM-GM ta c:
.

a +1
( b + 1)( c + 1)

Bi 10. Theo gi thit ta c

Tng t ta c:

b +1

( a + 1)( c + 1)

c +1

( a + 1)( b + 1)

Nhn theo v 3 bt ng thc trn ta c ngay iu phi chng minh.


ng thc xy ra khi v ch khi a= b= c= 2 .
Nhn xt. Vi cch lm tng t trn ta x l c bi ton tng qut cho n
s dng.
Bi ton. Vi x1 , x2 ,..., xn > 0 tha mn

1
1
1
1 ta lun c
+
+ ... +
=
1 + x1 1 + x2
1 + xn

x1 x2 ...xn ( n 1) .
n

Bi 11. S dng bt ng thc AM GM cho hai s ta c

Cng li theo v hai bt ng thc trn ta c pcm. ng thc xy ra khi v ch


khi

Bi 14. S dng bt ng thc AM GM ta c

169

www.TaiLieuLuyenThi.com
Khm ph t duy K thut gii bt T Bi ton Max Min ng Thnh Nam

Bt ng thc c chng minh.


ng thc xy ra khi v ch khi

Bi 15. Du bng t ti

S dng bt ng thc AM GM ta c

1
2014
1
2
2012

+
= 2
+
+
2
2
2
2
a + b + c ab + bc + ca a + b + c ab + bc + ca ab + bc + ca
3
2012

+
2
3 a 2 + b2 + c2
(
) ( ab + bc + ca ) 13 ( a + b + c )2
9
2012
2015

2
3
( a + b + c ) 1 ( a + b + c )2
3
2

Bt ng thc c chng minh. ng thc xy ra khi v ch khi


Bi 17. Theo gi thit ta c

Nhn theo v hai bt ng thc trn ta c pcm. ng thc xy ra khi v ch khi

.
Bi 18. Ta c

Nhn theo v n bt ng thc trn ta c iu phi chng minh.


.
ng thc xy ra khi v ch khi

Bi 19. Bt ng thc v tri l hin nhin theo C S.


Bt ng thc v phi tng ng vi

170

www.TaiLieuLuyenThi.com
Cty TNHH MTV DVVH Khang Vit

Bt ng thc c chng minh. ng thc xy ra khi v ch khi


Bi 20. S dng bt ng thc AM-GM cho ba s dng ta c:

1 = a 2 + b 2 + c 2 3 3 a 2b 2 c 2 abc

1
3 3

1
3 3
abc

S dng bt ng thc AM-GM cho bn s dng ta c:

1
1
4
4 4 abc.
= .
9abc
9abc
3

a+b+c+

1
1
8
4 8

= a +b+c +

+ .3 3 = 4 3 .
+
abc
9abc 9abc
3 9
1
Bt ng thc c chng minh. ng thc xy ra khi v ch khi a= b= c=
.
3

Suy ra a + b + c +

Bi 21. S dng bt ng thc AM-GM cho ba s dng ta c:

(a + b c)
3c

(a + b c) a + b + 4 c 1 .
c 1
+ + a+bc
3 3
3c
c
3

(b + c a )

(c + a b) a + c + 4 b 1 .
4
b + c + a 1;
Tng t:
3a
3
3b
3
Cng theo v 3 bt ng thc trn kt hp vi iu kin a + b + c =
3 ta c ngay
P 1 .ng thc xy ra khi v ch khi a= b= c= 1 .
Vy gi tr nh nht ca P bng 1 t ti a= b= c= 1 .
3

Bi 22. Khng mt tnh tng qut gi s b l s nm gia a v c.


Ta c:

( b a )( b c ) 0 b2 + ac ab + bc b2c + c 2 a abc + bc 2
2
a 2b + b 2 c + c 2 a + abc a 2b + bc 2 + 2abc = b ( a + c )
a+c a+c

b+ 2 + 2
a+c a+c
4
3
= 4.b.
4
=
.
( a + b + c=
) 4

2
2
3

27

171

www.TaiLieuLuyenThi.com
Khm ph t duy K thut gii bt T Bi ton Max Min ng Thnh Nam

Bt ng thc c chng minh. ng thc xy ra khi v ch khi a= b= c= 1 .


Bi 23. Ta c

3 ( ab + bc + ca ) = ( a + b + c )( ab + bc + ca ) = ( a 2b + b 2 c + c 2 a ) + ( ab 2 + bc 2 + ca 2 ) + 3abc .

Do vy ta ch cn chng minh ab 2 + bc 2 + ca 2 + abc 4 .


y l mt bi ton quen thuc c chng minh bi ton trn.
Bi ton c chng minh ng thc xy ra khi v ch khi a= b= c= 1 .
Bi 24. Khng mt tnh tng qut gi s b l s nm gia a v c.
Ta c:

( b a )( b c ) 0 b2 + ac ab + bc b2c + c 2 a abc + bc 2

a 2b + b 2 c + c 2 a a 2b + bc 2 + abc = b ( a 2 + ac + c 2 )

S dng bt ng thc AM-GM ta c:

a 2 + ac + c 2 + ab + bc + ca
b ( a + ac + c ) ( ab + bc + ca ) b

2
2
2
b ( a + c ) + b ( a + c )
b (a + c) (a + b + c)
a+c a+c
2

=
= b.
.
.( a + b + c )
4
4
2
2

a+c a+c

+
b+

1
2
5
2
2 =
(a + b + c)
(a + b + c)

27

Bt ng thc c chng minh. ng thc xy ra khi v ch khi a= b= c .


Bi 25. S dng bt ng thc C-S ta c:

3 ( a 2 + b2 + c2 ) ( a + b + c ) a + b + c 3 ( a 2 + b2 + c2 ) .
2

Suy ra

abc 3 ( a 2 + b 2 + c 2 ) + a 2 + b 2 + c 2
3 + 1 abc

=
.
P
( a 2 + b2 + c 2 ) ( ab + bc + ca )
( ab + bc + ca ) a 2 + b 2 + c 2

Mt khc: ( ab + bc + ca ) a 2 + b 2 + c 2 3 3 a 2b 2 c 2 . 3 3 a 2b 2 c 2 =
3 3abc .
Suy ra P

3 +1 3 + 3
= . ng thc xy ra khi v ch khi a= b= c .
9
3 3

Vy gi tr ln nht ca P bng

172

3+ 3
t ti a= b= c .
9

www.TaiLieuLuyenThi.com
Cty TNHH MTV DVVH Khang Vit

Bi 26. S dng AM GM cho 2 s dng ta c

Xy dng tng t cho 2 bt ng thc cn li ri cng li theo v 3 bt ng


thc ta c

.
Bi 27. Theo gi thit v bt ng thc AM GM ta c:

.
Suy ra

S dng bt ng thc AM - GM ta c
.
S dng bt ng thc Cauchy - Schwarz ta c

Suy ra

ng thc xy ra khi v ch khi

. Vy gi tr nh nht ca P bng 13.

Bi 28. Nu khng c iu kin rng buc a,b,c nguyn dng ta c ngay


.
ng thc xy ra ti

V vy ta d on du bng t ti

Khng mt tnh tng qut gi s

ta c
173

www.TaiLieuLuyenThi.com
Khm ph t duy K thut gii bt T Bi ton Max Min ng Thnh Nam

ng thc xy ra khi v ch khi

hoc cc hon v.

Bi 29. Gi s

S dng bt ng thc AM GM ta c
3

b
c
a
3
+
+

672 ( a + b + c ) a
a b c
672
671
671
.
.

=
1
672 671 671
3
671.672.3

P= abc 6712.672
Bi 30. S dng bt ng thc AM-GM cho ba s dng ta c:

1
1
1
+ 2
+ 2

x + xy y + yz z + zx
2

3
3

(x

+ xy )( y 2 + yz )( z 2 + zx )

3
= =
3 xyz x + y
( )( y + z )( z + x )

3
3

( xz + yz )( xy + zx )( yz + xy )

3
27

2 ( xy + yz + zx ) 2 ( x + y + z )2
3

Bt ng thc c chng minh. ng thc xy ra khi v ch khi x= y= z .


Bi 31. S dng bt ng thc AM GM cho 3 s dng ta c
.
Tng t cho hai cn thc cn li ri cng li theo v ta c pcm.
ng thc xy ra khi v ch khi
.
Bi 32. S dng bt ng thc AM GM dng lu tha ta c

Tng t ta c
174

www.TaiLieuLuyenThi.com
Cty TNHH MTV DVVH Khang Vit

.
Cng li theo v ba bt ng thc trn ta c iu phi chng minh. ng thc
xy ra khi v ch khi
.
Bi tp tng t
Cho a,b,c l cc s thc dng. Chng minh

.
Bi 33. S dng bt ng thc AM GM cho (n-1) s dng ta c

.
Nhn theo v n bt ng thc trn ta c pcm. ng thc xy ra khi v ch khi

Bi 34. Du bng t ti

Ta vit li P v s dng bt ng thc AM GM cho hai s

8
t ti
.
3
Bi 35. Vit li P v s dng bt ng thc AM GM ta c

Vy gi tr nh nht ca P bng

.
Du bng t ti
.
Bi 36. Bt ng thc cho tng ng vi:

175

www.TaiLieuLuyenThi.com
Khm ph t duy K thut gii bt T Bi ton Max Min ng Thnh Nam

n y ch cn p dng AM-GM ta c:

Cng li theo v ba bt ng thc trn ta c pcm.


Bi 37. S dng bt ng thc AM GM cho ba s dng ta c

.
Ch

bi v
.

Bt ng thc c chng minh. ng thc xy ra khi v ch khi

.
Bi tp tng t
Cho x, y, z l cc s thc dng c tch bng 1.
Tm gi tr nh nht ca biu thc
.
Bi 39. Theo gi thit ta c

176

www.TaiLieuLuyenThi.com
Cty TNHH MTV DVVH Khang Vit

Do

Cng li theo v hai bt ng thc trn ta c iu phi chng minh.


.
ng thc xy ra khi v ch khi
Bi 40. S dng bt ng thc AM GM cho bn s dng ta c
.
Tng t ta c

Do
.
Bt ng thc c chng minh. ng thc xy ra khi v ch khi

Bi 42. Bt ng thc cho l tng ca hai bt ng thc

Bt ng thc c chng minh.


ng thc xy ra chng hn ti

ta s dng bt ng thc C S (xem


Ch . Nu cn tm Max ca
ch bt ng thc C S ).
Bi 43. S dng bt ng thc AM GM cho ba s dng ta c

.
Ta ch cn chng minh

Bt ng thc cui ng v

Bt ng thc c chng minh. ng thc xy ra khi v ch khi

.
177

www.TaiLieuLuyenThi.com
Khm ph t duy K thut gii bt T Bi ton Max Min ng Thnh Nam

Bi 45. D on du bng t ti

Vy ta s dng bt ng thc AM GM nh sau

Cng theo v ba bt ng thc trn ta c pcm.


Bi 46. Bt ng thc cho tng ng vi:

Bt ng thc ng v

Bt ng thc c chng minh. ng thc xy ra khi v ch khi


Bi 47. p dng bt ng thc AM GM ta c

V vy

khi

Bi 48. t

Ch

178

www.TaiLieuLuyenThi.com
Cty TNHH MTV DVVH Khang Vit

t ti

Vy

Ch . Ta c th chng minh bng bt ng thc C S.


Bi 49. Theo gi thit ta c:
Ch

Tng t cho hai biu thc cn li suy ra

3 1 ( b + c )( c + a ) ( c + a )( a + b ) ( a + b )( b + c )
+
+
+

2
2
2 2 ( a + b )2
(b + c )
( c + a )
3 3 ( b + c )( c + a ) ( c + a )( a + b ) ( a + b )( b + c )
.
.
3
+ 3
=
2
2
2
2 2
(a + b)
(b + c )
(c + a)

Bt ng thc c chng minh.


Bi 50. S dng bt ng thc AM GM ta c

x
=
x + 3y

x
x+ y 1 x
x+ y

+
.
;
x + y x + 3y 2 x + y x + 3y

y
=
x + 3y

1 2y
11
2y
.
+
;
2 x + 3y 2 2 x + 3y

Cng theo v hai bt ng thc trn ta c

Tng t ta c

Cng theo v hai bt ng thc trn c cpm.


ng thc xy ra khi v ch khi
.
Bi 51. Ta c P =

x2
+
x
2 ( x 2)

y 3
z4
+
y
z
+

3 ( y 3)

4 ( z 4)

2z
x 2
y 3
2+ x 2 3+ y 3 4+ z 4
1
1
1

+
+
=
+
+
4z
2 2x
2 3y
2 2 2 3 4
179

www.TaiLieuLuyenThi.com
Khm ph t duy K thut gii bt T Bi ton Max Min ng Thnh Nam

ng thc xy ra khi v ch khi

Bi 52. S dng bt ng thc AM GM ta c

Cng theo v 3 bt ng thc trn ta c iu phi chng minh. ng thc xy ra


khi v ch khi

Bi 53. Ta c

D on du bng t ti khi ba bin bng nhau, khi


.
Tc du bng t ti

Vy ta s dng bt ng thc AM GM nh sau

ng thc xy ra khi v ch khi

Bi 54. S dng bt ng thc AM GM ta c


.

Tng t ta c

b2
3b + ( c + a )
2

c2
3c + ( a + b )
2

180

b
2(a + b + c)

c
2(a + b + c)

www.TaiLieuLuyenThi.com
Cty TNHH MTV DVVH Khang Vit

Cng li theo v 3 bt ng thc trn ta c iu phi chng minh.


ng thc xy ra khi v ch khi=
a b=
, c 0 hoc cc hon v.
Bi 55. S dng bt ng thc AM GM ta c
2

2
b+c+

2 = a ( 7 2a ) .
ab + 2abc
= 2ab c + 2a

2
2
8

a ( 7 2a )

9
2
( 4 a )( 2a 3) 0 .
8
2
Bt ng thc c chng minh. ng thc xy ra khi v ch khi
3
1
.
=
a
,=
b 1,=
c
2
2

Ta ch cn chng minh a +

Bi 56. Ch ( a + b + c ) 3 ( ab + bc + ca ) ab + bc + ca 3 .
2

ab

Ta c

c +3
2

1 ab
ab

+
.
+
+b
4
c
a
c
( c + a )( c + b )

ab

ab

=
c + ab + bc + ca
2

Tng t cho 2 cn thc cn li cng theo v 3 bt ng thc ta c iu phi


chng minh.
ng thc xy ra khi v ch khi a= b= c= 1 .
Bi 57. S dng bt ng thc AM GM ta c
2
a + b)
(
1
1 a2
ab
b2
=
.

4 c 2 + a 2 c 2 + b 2
c2 + 1 4 c2 + a 2 + c2 + b2

) (

1 b2
c 2 ca
1 c2
a2

+
;

a 2 + 1 4 a 2 + b 2 a 2 + c 2 b 2 + 1 4 b 2 + c 2 b 2 + a 2
Cng theo v 3 bt ng thc trn ta c iu phi chng minh. ng thc xy ra
1
.
khi v ch khi a= b= c=
3
Bi 58. Ta c:
bc

( x + z)

( y + z)

Suy
ra P
=

x2 + y 2 + 2 z ( x + y ) + 2 z 2

( xy + xz + yz + z )

2 2

1
4( x y)

+ ( x y) + 2 2

ng thc xy ra khi v ch khi

4( x y)

1
4( x y)

= x 2 + y 2 + 2 (1 xy ) = ( x y ) + 2

. ( x y=
) +2 3.
2

( x y )2 x y =

1
do x > y .
2
181

www.TaiLieuLuyenThi.com
Khm ph t duy K thut gii bt T Bi ton Max Min ng Thnh Nam

Khi z l nghim dng ca phng trnh: z 2 + z ( x + y ) + xy 1 =


0.
3
2
2
x y
x
+
y

4
xy

y
+
x

y
+
4
(
)
(
)
(
)
2
.
z
=
=
2
2
2
Vy gi tr nh nht ca P bng 3 t ti
3
x y
1
3

2
x =
y
=
,z
, x + y <
.
2
2
2

Nhn xt. y l mt bi ton kh n gin da vo bin i ch iu kin


1 ngoi ra ta c th t a = x + z , b = z + y ab = 1 v khi
( z + x )( z + y ) =
x y +

=
P

4(a b)

1
a

1
b2

y l dng ton quen thuc c cp n trong ch tnh ng cp hoc


s dng hm s ta c 2 cch x l nh sau:
1
ab
ab ab
a b
Cch=
1: P
+ 2=
+ 2
+ + .
2
b b a
a
a
b
4( a b)
4 2 +
a
b
1
a b
t t =+ , ( t > 2 ) a v kho st hm s
=
f (t )
+ t hoc nh gi
4 (t 2)
b a

da vo bt ng thc AM-GM nh sau:


=
P

1
1
1
=
+t
+ (t 2) + 2 2
. ( t 2=
) +2 3.
4 (t 2)
4 (t 2)
4 (t 2)

Cch 2: Th b =

1
a v kho st hm s =
f (a)
a

1
2

1
2

+ a 2 ta tm

a
1

4 a
a

c Min ca f(a) bng 3 v c kt qu tng t hai cch cp trn.


Bi 59. Nhn xt. Hnh thc bi ton ny tng t bi ton trc tuy nhin hai
nhn t sau khng c dng i xng ta cn kho lo khai thc gi thit tch
1.
( b + c )( a + c ) =

Ta c:

12

( b + c )2 ( a + c )2

2
2
6 ( a + c ) + 2 ( b + c )
= 6 a + c 2 + 2 b + c 2 .
=
(
)
(
)
2

( a + c )( b + c )

Theo gi thit ta c:

( a b )2 =
( a + c ) ( b + c )
182

=+
( a c ) + (b + c ) 2 .
2

www.TaiLieuLuyenThi.com
Cty TNHH MTV DVVH Khang Vit

t x = a + c, y = b + c xy = 1
v
=
P

1
x2 + y 2 2

12
1
12
6 x2
+=
+ 2 +=
2
2
1
y
x
x2 + 2 2 x
x

n y ta c th kho st hm s=
f (t )
a>bt = a+c>

x2

( t 1)2

( x 1)
2

6 x 4 + 12
x2

6t 2 + 12
vi
=
t x 2 , ( t > 1) do
t

( a + b )( a + c ) = 1 .

Hoc bin i tng ng nh sau:


t

6t 2 + 12
+=
=
P
+

20
20
t
( t 1)2

( t 2 )2 ( 6t 2 8t + 3)
t ( t 1)

+ 20 20 (vi t > 1 ).

Vy gi tr nh nht ca P bng 20 t ti t = 2 a + c = 2, b + c =

1
.
2

Bi 60. V tri bt ng thc c vit li di dng


bc ca ab zx xy yz
+ +
+ +
+
y
z
x
a
b
c
bc
ca
ab
zx
xy
yz

= + c + + a + + b + + z + + x + + y 3
y
z
x
a
b
c




c a b z x y
a z b x c y
= + + + + + 3= + + + + + 3
y z x a b c
z a x b y c

Bi 61. Bt ng thc cho tng ng vi:


z( x + y)
1
y (x y + z ) 0
.
xz
xz
x+ y

0 (
+ 1) 3
3
y+z
y+z
y+z
ng theo AM GM.
1
3
Bi 62. Ta c a3 + b3 ( a + b ) .
4
Do s dng bt ng thc AM GM ta c
3
1
c3
9
3 c
3 a 3 + b3 + c 3 3 ( a + b ) + + 3 c 2 ( a + b ) .
4

2 2 2 2

Vy ta ch cn chng minh

c 2 ( a + b ) 2c ( ab + bc + ca ) 3 2 c 2 ( a + b ) 2abc .
2 2
2 2

183

www.TaiLieuLuyenThi.com
Khm ph t duy K thut gii bt T Bi ton Max Min ng Thnh Nam

Bt ng thc cui lun ng do

9
3

2 2

2 > 1; c ab .

Bt ng thc c chng minh.


Bi 63. S dng bt ng thc AM GM ta c
3

a+b+b+c+c+a
8
=
( a + b + c )3 .
( a + b )( b + c )( c + a )

3
27

Do vy ta ch cn chng minh
9abc

3+

12 ( ab + bc + ca )

(a + b + c)
( a + b + c )2
3
( a + b + c ) + 3abc 4 ( a + b + c )( ab + bc + ca )
a3 + b3 + c3 + 3abc ab ( a + b ) + bc ( b + c ) + ca ( c + a )
3

Bt ng thc cui ng theo BT Schur bc 3.


Bt ng thc c chng minh. ng thc xy ra khi v ch khi a= b= c .
Bi 64. Ta t
a1 =

x1
1 + x1

a2 =

x2
(1 + x1 )(1 + x1 + x2 )

...
an =

xn
(1 + x1 + x2 + + xn 1 )(1 + x1 + x2 + + xn )

an +1 =

1
1 + x1 + x2 + + xn

Khi ta c a1 + a2 + + an + an +1 =
1.
Ta cn chng minh: a1a2 an +1

1
(n + 1) n +1

Bt ng thc cui ng theo AM GM nn c pcm.


Bi 65. S dng bt ng thc AM GM kt hp v tri l hm ng bin vi tch
ab ta c

184

www.TaiLieuLuyenThi.com
Cty TNHH MTV DVVH Khang Vit

abc
c.ab
=
2
( a + b )( b + c )( c + a ) ( a + b ) c + c ( a + b ) + ab

Vy ta chng minh

a+b
c

2
2
a+b
a
b
c
c
a
b
+
+
+
+
(
)
(
)

2

c (a + b)

( a + b + 2c )2
c (a + b)
( a + b )( a + b + 2c ) .

2
( 3a + 3b + 2c )2
( a + b + 2c )
2
3
c ( 3a + 3b + 2c ) ( a + b + 2c )

S dng bt ng thc AM GM ta c
1
2
.8c. ( 3a + 3b + 2c )( 3a + 3b + 2c )
c ( 3a + 3b + 2=
c)
8
3

1 8c + 3a + 3b + 2c + 3a + 3b + 2c
3

= ( a + b + 2c )
8
3

Bt ng thc c chng minh. ng thc xy ra khi v ch khi a= b= c .


Bi 66. Ta c:
1
a ( b + c ) + 2c 2

1
b ( a + 4c )

Suy ra P ( a + 2b + 2014c ) .

( ab + ac + 2c ) ( ab + 4bc )

2 2

( a + 2c )( 2b + c )

2 2
2ab + ac + 4bc + 2c 2

4 2
4 2

a + 2b + 3c a + 2b + 2014c

4 2
=
4 2.
a + 2b + 2014c

Vy gi tr nh nht ca P bng 4 2 xy ra khi =


c 0,=
a 2b > 0 .
Bi 67. Khng mt tnh tng qut ta gi s a > b > c ta c:
2
2
2
5
.
P=
+
+
+
a b bc a c
ab + bc + ca
S dng bt ng thc:

1 1
4
+
, x, y > 0 ta c:
x y x+ y

185

www.TaiLieuLuyenThi.com
Khm ph t duy K thut gii bt T Bi ton Max Min ng Thnh Nam

P 2.

4
2
5
1
2

+
+
= 5
+

ab+bc ac
ab + bc + ca
ab + bc + ca
ac
5.2 2

20
=
4 a c 2 ab + bc + ca
(
) (
) 4 ( a c )2 ( 4ab + 4bc + 4ca )
20

20 2
20 2
=
=
( a c )2 + 4ab + 4bc + 4ca
( a + c )2 + 4b ( a + c ) ( a + c )( a + c + 4b )
2
20 2

(1 b )(1 + 3b )

20 6

( 3 3b )(1 + 3b )

40 6
= 10 6
3 3b + 1 + 3b

2+ 6
a =
a b = b c
6

1
1

2
ng thc xy ra khi v ch khi
.
=
=
b

c
3
ab + bc + ca

3 3b =+

1 3b
2 6
c =
6

1 1
1
1 1
Vy gi tr nh nht ca P bng 10 6 t ti a =
hoc
+
,b =
,c =

3
3
3
6
6
cc hon v.
Nhn xt. Ta c th x l bi ton trn bng phng php hm s xem chng 3.
Bi 68. Khng mt tnh tng qut gi s z = min { x, y, z} ta c

x 2 + y 2 + z 2= 2 ( xy + yz + zx )
( x + y ) + z 2 2 z ( x + y ) = 4 xy ( x + y z ) = 4 xy
2

S dng bt ng thc AM GM ta c
x+ yz x+ yz
2
+
+ 2z
z( x + y z)
x+ y+z
2
2
=
3
=
3
3
2

2 xyz .

4z
5z =
x + y z =
x + y =
z, y z
x 4=
.
ng thc xy ra khi v ch khi

=
, y 4z
2 xy xy = 4 z 2
x + y z =
x z=

Cch 2: Theo gi thit ta c


Ta cn chng minh

186

xy + yz + zx

( x + y + z)

xyz

( x + y + z)

1
.
54

1
.
4

www.TaiLieuLuyenThi.com
Cty TNHH MTV DVVH Khang Vit

1
a + b + c =
x
y
z

t a
=
=
,b
=
,c

1.
x+ y+z
x+ y+z
x+ y+z
ab + bc + ca =
4

1
(xem ch k thut s dng tnh ng cp).
54
Bi 69. Theo gi thit kt hp s dng bt ng thc AM GM ta c

Ta cn chng minh=
P abc

2(a + b + c)
8 = ( a + b )( b + c )( c + a )

a+b+c3
Bt ng thc tng ng vi:

( a + b + c )27 326 ( a3 + b3 + c3 )
(a + b + c)

27

3
326 ( a + b + c ) 3 ( a + b )( b + c )( c + a )

(a + b + c)

27

3
326 ( a + b + c ) 24

t x =

( a + b + c )3 x 27 ta cn chng minh

Xt hm s f ( x) =
=
f '( x)

x9
vi x 27 ta c
x 24

8 x8 ( x 27 )

( x 24 )2

x9
326 .
x 24

0, x 27 .

Do f(x) l hm ng bin trn [ 27;+ ) . V vy f ( x) f (27) =


326 .
Bt ng thc c chng minh. ng thc xy ra khi v ch khi a= b= c= 1 .
Bi 70. t x = b + c a, y = c + a b, z = a + b c xyz = 1 v
=
a

y+z
z+x
x+ y
.
=
,b =
,c
2
2
2

Bt ng thc tr thnh
x + y + z 5 x 2 + y 2 + z 2 + xy + yz + zx

3
6
5
x + y + z ( x + y + z ) ( xy + yz + zx )


3
6

xy + yz + zx 3 3 x 2 y 2 z 2 =
3
.
Ch iu kin xyz = 1
3
x + y + z 3 3 xyz =
187

www.TaiLieuLuyenThi.com
Khm ph t duy K thut gii bt T Bi ton Max Min ng Thnh Nam

Suy ra ( x + y + z ) xy yz zx ( x + y + z ) 3 .
2

5
x + y + z ( x + y + z) 3
.
Vy ta chng minh

3
6

x+ y+z
t t
=
1 ta cn chng minh
3
2

t5

3t 2 1
1
1

( t 1) t 4 + t t 2 + t 2 0 .
2 2
2
2

Bt ng thc c chng minh. ng thc xy ra khi v ch khi a= b= c= 1 .


Bi 71. Gi s a = min {a, b, c} .
t b =
a + u, c =
a + v, ( u , v 0 ) bt ng thc tr thnh
(3

3
3

)(u 2 uv + v 2 )a + u 3

3
3

4
4
Bt ng thc cui lun ng do

u 3 + v3 = u 3 +

uv 2 + v3 0 .

v3 v3
u 3v 6
3
+ 33
= 3 uv 2 .
2 2
4
4

Bi 72. t
=
a 1=
/ x; b 1=
/ y; c 1 / z bt ng thc tr thnh:
a2 + 3
b2 + 3
c2 + 3 3
+
+
.
ac + 3b ba + 3c cb + 3a 2
S dng bt ng thc C S ta c

4 a2 + 3
4 a2 + 3
a2 + 3
a+3
=
=

ac + 3b 2 ac + b ( a + b + c ) 2 ( b + a )( b + c ) 2 ( b + a )( b + c )
b2 + 3
b+3
c2 + 3
c+3

;
ba + 3c 2 ( c + a )( c + b ) cb + 3a 2 ( a + b )( a + c )

Vy ta ch cn chng minh
a+3
b+3
c+3
3
+
+

2 ( b + a )( b + c ) 2 ( c + a )( c + b ) 2 ( a + b )( a + c ) 2
( a + 3)( a + c ) + ( b + 3)( a + b ) + ( c + 3)( b + c ) 3 ( a + b )( b + c )( c + a )
a 2 + b 2 + c 2 + ab + bc + ca + 18 3 ( a + b )( b + c )( c + a )

Theo bt ng thc AM GM ta c

188

www.TaiLieuLuyenThi.com
Cty TNHH MTV DVVH Khang Vit

a 2 + b 2 + c 2 + ab + bc + ca + 18 = ( a + b + c ) ( ab + bc + ca ) + 18
2

(a + b + c)
2

1
( a + b + c )2 + 18 =24
3
3

a+b+b+c+c+a
3 ( a + b )( b + c )( c + a ) 3
24
=
3

Do bt ng thc cui ng v ta c pcm. ng thc xy ra khi v ch khi


x= y= z= 1 .

Bi 73. t x =

a + b c, y=

b + c a, z=

c+ a b .

x2 + y 2 + z 2 = a + b + c = 3

2
x 2 y 2 + y 2 z 2 + z 2 x=
( a + b c )( b + c a )

2
= b 2 ( a c ) = 4 ( ab + bc + ca ) 9

Ta cn chng minh

1 1 1
36
+ +
2
2
x y z 9 + x y + y 2 z 2 + z 2 x2

x 2 y 2 + y 2 z 2 + z 2 x 2 + 9 ( xy + yz + zx ) 36 xyz .

S dng bt ng thc AM GM ta c
x 2 y 2 + y 2 z 2 + z 2 x 2 + 9 12 3 xyz
xy + yz + zx 3 3 x 2 y 2 z 2

Nhn theo v 2 bt ng thc trn ta c ngay iu phi chng minh.


ng thc xy ra khi v ch khi a= b= c= 1 .
Bi 74. t a = x + y, b = y + z , c = z + x a, b, c 0; ab + bc + ca > 0 .
Khi P =

a
b
c
.
+
+
b+c
c+a
a+b

a
2a
b
2b
c
2c
.

b+c a+b+c c+a a+b+c a+b a+b+c


Cng theo v ba bt ng thc trn ta c P 2 .
Du bng t ti x > 0; y ==
z 0 hoc cc hon v.

Tm gi tr nh nht:

Bi 75. Trc ht ta chng minh


(2a 2 + bc)(2b 2 + ca )(2c 2 + ab)

Tht vy ta c

( a + 2b )2 ( b + 2c )2 ( c + 2a )2
8

3 2
(2a + bc)(2b 2 + ca )(2c 2 + ab) 9abc =

2
189

www.TaiLieuLuyenThi.com
Khm ph t duy K thut gii bt T Bi ton Max Min ng Thnh Nam

(
( a + 2b ) ( 2c + ab ) ( b + 2c ) ( 2a + bc ) ( c + 2a ) ( 2b

)
+ ca )

= ( a + 2b ) 2c 2 + ab + ( b + 2c ) 2a 2 + bc + ( c + 2a ) 2b 2 + ca
33

Mt khc (2a 2 + bc)(2b 2 + ca )(2c 2 + ab) 9abc (2a 2 + bc)(2b 2 + ca )(2c 2 + ab) .
Suy ra
3

( a + 2b ) ( 2c 2 + ab ) ( b + 2c ) ( 2a 2 + bc ) ( c + 2a ) ( 2b2 + ca )
(2a 2 + bc)(2b 2 + ca )(2c 2 + ab)
2

(2a 2 + bc)(2b 2 + ca )(2c 2 + ab)

( a + 2b )2 ( b + 2c )2 ( c + 2a )2
8

S dng bt ng thc AM GM ta c
1
( a + 2b )( b + 2c )( c + 2a ) = .( a + 2b )( 4b + 8c )( c + 2a )
4
1 3 ( a + 2b + 3c )

16
=
4
3

T suy ra iu phi chng minh. ng thc xy ra khi v ch khi


=
a 2,=
b 1,=
c 0.
Bi 76. Khng mt tnh tng qut =
gi s c min {a, b, c} c 1 .
S dng bt ng thc AM GM ta c
c + ab c +

( a + bc )( b + ca )

3 c 2 c 2 + 2c + 3
a 2 + b2
;
=c+
=
2
2
3

( a + b + bc + ca )2 ( a + b )2 ( c + 1)2
=

( c + 1)( a

+b

)=
(3 c )( c + 1)
2

2
2 ( 2 a )( 2 b ) =8 4 ( a + b ) + 2ab

=4 a 2 b 2 + ( a + b 2 ) 4 a 2 b 2 =c 2 + 1
2

Do vy ta ch cn chng minh

)(

) (

8 ( 2 c ) 3 c 2 3 + 2c c 2 7 c 2 ( c 1) 0 .
2

Bt ng thc cui ng v ta c pcm. ng thc xy ra khi v ch khi


a= b= c= 1 .
Bi 79. S dng bt ng thc C S v AM GM ta c
190

www.TaiLieuLuyenThi.com
Cty TNHH MTV DVVH Khang Vit

(a

9
+ b + c (1 + b + c ) ( a + b + c ) =

a2

a +b+c
2

a b + c + 1 a 3 ( b + c + 1) a ( b + c + 1 + 3)
=

3
3 3
6 3
b2

Tng t ta c

b ( a + c + 1 + 3)

c2

c ( b + a + 1 + 3)

6 3
6 3
b +c+a
c +a+b
Gi P l biu thc v tri v cng theo v 3 bt ng thc trn v ch
1
2
ab + bc + ca ( a + b + c ) =
3.
3
2

4 ( a + b + c ) + 2 ( ab + bc + ca )

4.3 + 2.3
3.
=
6 3
6 3
Bt ng thc c chng minh. ng thc xy ra khi v ch khi a= b= c= 1 .

Ta c P

Bi 80. Theo gi thit ta c ( ab + bc + ca ) 3abc ( a + b + c )= 3 ( ab + bc + ca )


2

ab + bc + ca 3; ab ac bc bc 1

( a 1)2 + ( c b )( bc 1) 0

Do

bc

1 1
1
+ 2b 0
a c
b
1
1

a + c +a +b + c 2b 0
b
b

1
1
a + c 1 0 b
b
a + c 1
Bt ng thc c chng minh. ng thc xy ra khi v ch khi a= b= c= 1 .
a+c+

Bi 82. Khng mt tnh tng qut gi


s c min {a, b, c} c 2 bc 0; c 2 ac 0 .
=
Suy ra

(a

(b

1
2

bc + c

)( c

ca + a

1
2

ab + b

)(b

bc + c

) (c

1
.
ab
1

ca + a

)( a

1
1
+

a 2 ab + b 2 b 2 bc + c 2
c 2 ca + a 2
S dng bt ng thc AM GM ta c

ab + b

1
1 1

a 2 ab + b 2 a b

191

www.TaiLieuLuyenThi.com
Khm ph t duy K thut gii bt T Bi ton Max Min ng Thnh Nam

1
a ab + b
2

(a

ab + bc + ca
2

ab + b

) ( ab + bc + ca )

2 ab + bc + ca
a + c ( a + b ) + b2
2

2 ab + bc + ca
2 ab + bc + ca
ab
=

( a + b )( a + b + c ) 2ab 2 ( a + b ab ) a + b ab

Gi P l biu thc v tri ta c


P

1
1
ab 1 1 1
ab
=
+
+
+ +
a + b ab a b ab ab
a
b
+
ab
ab

33

1 1
ab
.
.
=
ab ab a + b ab

3
3

ab ( a + b ab )

a+b

ng thc xy ra khi v ch khi a= b= 1, c= 0 v cc hon v.


Bi 83. Khng mt tnh tng qut gi s x y z khi ta c
(i)
(ii)

xy + yz + zx
y + yz + z
2

xy + yz + zx
x + xy + y
2

x + z xy + yz + zx y + z
.
;

y + z x 2 + zx + z 2 x + z

( x + z )( y + z )
( x + z ) + ( x + z )( y + z ) + ( y + z )2
2

Cc bt ng thc (i) v (ii) xem chng minh trong chng 1.


p dng vo bi ton ta c
P

x+z
+
y+z

t t =

y+z
+
x+z

x+z
+
y+z

( x + z )( y + z )
( x + z ) + ( x + z )( y + z ) + ( y + z )2
2

y+z
1
.
t 2; P f (t ) = t +
x+z
t2 1
1

Xt hm s f (t ) = t +

t2 1

(t

trn na khong [ 2;+ ) ta c

(t 1) t > t t
(t 1)
(t 1)
3

f '(t ) = 1

> 0, t 2 .

V vy f(t) ng bin trn na khong [ 2;+ ) suy ra f (t ) f (2) =


2+
Vi x= y= 1, z= 0 th P bng 2 +
192

1
.
3

1
1
. Vy gi tr nh nht ca P bng 2 +
.
3
3

www.TaiLieuLuyenThi.com
Cty TNHH MTV DVVH Khang Vit

Bi 84. S dng bt ng thc AM GM ta c

(a + b +

2(a + c)

(a + b +

a+c
a+c
27
= a + b +
+
( a + b )( a + c )
2
2
2

2
1
.
3
27 ( a + b )( a + c )

2(a + c)

Xy dng tng t cho hai biu thc cn li v a v chng minh


1
1
1
+
+
12
a
+
b
a
+
c
b
+
c
b
+
a
c
+
a
(
)(
) (
)(
) (
)( c + b )

2(a + b + c)

( a + b )( b + c )( c + a )

12 6 ( a + b )( b + c )( c + a ) a + b + c

Ch s dng bt ng thc quen thuc

9 ( a + b )( b + c )( c + a ) 8 ( a + b + c )( ab + bc + ca ) .

Bi ton a v chng minh ab + bc + ca

3
16 ( ab + bc + ca ) 3 .
16

Bt ng thc cui ng theo gi thit v


16
3
2
ab + bc + ca 16abc ( a + b + c ) ( ab + bc + ca ) ab + bc + ca .
3
16
1
Bi ton c chng minh. ng thc xy ra khi v ch khi a= b= c=
.
4
a bc
2bc
2bc
2bc
.
Bi 85. Ta c
=
1
=
1
=
1
a + bc
a + bc
a ( a + b + c ) + bc
( a + b )( a + c )
Bi ton a v chng minh
3
bc
ca
ab
+
+

( a + b )( a + c ) ( b + c )( b + a ) ( c + a )( c + b ) 4
ab + bc + ca 9abc ( a + b + c )( ab + bc + ca ) 9abc

Bt ng thc cui ng theo AM GM.


Bi ton c chng minh ng thc xy ra khi v ch khi a= b= c=

1
.
3

Bi 86. Ch iu kin bi ton ta c


1
1
=
=
2
2
ab + 2c + 2c ab + 2c + 2c ( a + b + c )
=

( 2c + a )( 2c + b )

ab
ab

=
( 2bc + ab )( 2ac + ab ) 2bc + 2ca + 2ab 2

ab

( ab + bc + ca )2
193

www.TaiLieuLuyenThi.com
Khm ph t duy K thut gii bt T Bi ton Max Min ng Thnh Nam

Tng t ta c

1
bc + 2a + 2a
2

bc

( ab + bc + ca )

1
ca + 2b + 2b
2

ca

( ab + bc + ca )2

Cng li theo v ba bt ng thc trn ta c pcm. ng thc xy ra khi v ch


1
khi a= b= c=
3
Bi 87. S dng bt ng thc AM GM ta c
1
1
1
1
1
1 a+b+c
+ 2+ 2
+ +
=
2
ab bc ca
abc
a
b
c
9
27
=

abc ( a + b + c ) ( ab + bc + ca )2
Vy ta chng minh
27
2
a 2 + b 2 + c 2 a 2 + b 2 + c 2 ( ab + bc + ca ) 27 .
2
( ab + bc + ca )

Bt ng thc cui ng theo AM GM v


3

a 2 + b 2 + c 2 + 2 ( ab + bc + ca )
a + b + c ( ab + bc + ca )
27 .
=
3

Bt ng thc c chng minh. ng thc xy ra khi v ch khi a= b= c= 1 .


1 c
1 a
1 b 9
Bi 88. Vit li bt ng thc di dng
+
+

ab + c bc + a ca + b 2
a+b
b+c
a+c
9

+
+
.
( c + a )( c + b ) ( a + b )( a + c ) ( b + a )( b + c ) 2

S dng bt ng thc AM GM cho v tri a v chng minh


8
( a + b )( b + c )( c + a ) .
27
Bt ng thc cui ng theo AM GM.
Bi 89. Thay 1 bi iu kin vo bt ng thc cho v a v chng minh
4ab
4bc
4ca
+
+
+a+b+c4.
a+b b+c c+a
S dng bt ng thc C S ta c

( ab + bc + ca )
ab
bc
ca
1
.
+
+

a + b b + c c + a ab ( a + b ) + bc ( b + c ) + ca ( c + a ) a + b + c
2

Ta ch cn chng minh

194

4
+ a +b+c 4.
a+b+c

www.TaiLieuLuyenThi.com
Cty TNHH MTV DVVH Khang Vit

Bt ng thc cui ng theo AM GM. ng thc xy ra khi v ch khi


a= b= 1, c= 0 hoc cc hon v.
9 a 2 b2 c2
Bi 90. S dng ab + bc + ca =
a v chng minh bt ng thc
2

a 2 + b2 + c2 + 2

a + b + c 9.

Bt ng thc trn l tng ca cc bt ng thc


a 2 + a + a 3a
b 2 + b + b 3c
c 2 + c + c 3c
Bt ng thc c chng minh. ng thc xy ra khi v ch khi a= b= c= 1 .
Bi 91. Bt ng thc cho tng ng vi
ab + bc + ca ab + bc + ca ab + bc + ca 9
+
+

(a + b)b
(b + c ) c
(c + a) a 2
a
b
c
a + b b + c c + a 15
+
+
+
+
+

a+b b+c c+a


a
b
c
2
Bt ng thc cui l tng ca hai bt ng thc

1a+b b+c c+a


a
b
c
+
+
+
+
+
3
a +b b+c c+ a 4 a
b
c
3 a +b b+c c+ a 9
+
+

4 a
b
c 2

Bi 92. t x =

b + c a; y =

c + a b; z =

a + b c x2 + y 2 + z 2 = 3 .

Bi ton a v chng minh


1 1 1
36
+ +
2
2
x y z 9 + x y + y 2 z 2 + z 2 x2

x 2 y 2 + y 2 z 2 + z 2 x 2 + 9 ( xy + yz + zx ) 36 xyz

Bt ng thc trn l tch ca hai bt ng thc


xy + yz + zx 3 3 x 2 y 2 z 2 ; x 2 y 2 + y 2 z 2 + z 2 x 2 + 9 12 6 xyz .

Bt ng thc c chng minh. ng thc xy ra khi v ch khi a= b= c= 1 .


Bi 93. Ch ng thc:

(a
(b
(c

2
2

)
(
+ ca ) ( c + a )= a ( b
+ ab ) ( a + b =
) b(a

) (
+ c ) + c(a
+ c ) + a (b

)
+ b );
+c )

+ bc ( b + c )= b c 2 + a 2 + c a 2 + b 2 ;
2
2

195

www.TaiLieuLuyenThi.com
Khm ph t duy K thut gii bt T Bi ton Max Min ng Thnh Nam

Bi ton a v chng minh vi ba s dng x,y,z ta c


x+ y
+
z

y+z
+
x

z+x
3 2.
y

Bt ng thc ny ng theo AM GM

( x + y )( y + z )( z + x ) 36 2 xy .2 yz .2 zx =
x+ y
y+z
z+x
3 2.
+
+
36
z
x
y
xyz
xyz
Bi 94. Tng t bi ton trn bt ng thc a v chng minh vi ba s khng
x
y
z
m x,y,z ta c
+
+
2.
y+z
z+x
x+ y
Bt ng thc cui ng(xem chng minh trong bi tp mu).
ng thc xy ra khi v ch khi=
a b=
, c 0 hoc cc hon v.
Bi 95. Ta c c 2 + 2ab = a 2 + b 2 + 2c 2 2bc 2ca = ( a c ) + ( b c ) .
2

Do

a b
c 2 + 2ab

( a b )2
( c a )2 + ( c b )2
x
+
y+z

Bi ton a v chng minh

.
y
+
z+x

z
2.
x+ y

Vi x =
( a b ) , y =
( b c ) , z =
( c a ) xem bi tp mu.
2

Bi 96. Bt ng thc tng ng vi 3abc + ab + bc + ca 3 + a + b + c .


a b c (a + b + c)
+ +
b c a ab + bc + ca
2

Bt ng thc lun ng do a + b + c
ab + bc + ca a + b + c
a+b+c

a b c 1 2a b 1 2b c 1 2c a a + b + c
.
+ +=
+ + + + + 3
b c a 3 b c 3 c a 3 a b
abc

abc 1
Bt ng thc c chng minh. ng thc xy ra khi v ch khi a= b= c= 1 .
Bi 97. S dng bt ng thc AM GM ta c
x3 + 1
=
x4 + y + z

x x 2 + yz
=
x 4 + xyz ( y + z )

2 x x 2 + yz

x x 2 + yz

(x

xy + yz + zx

( y + z ) + 1) ( x3 + yz ( y + z ) )

xy + yz + zx

2 x xy + yz + zx
=
x+ y+z
x ( y + z ) + 1 + x + yz ( y + z )
2

Tng t cho 2 cn thc cn li ri cng li theo v ta c iu phi chng minh.


Bt ng thc c chng minh. ng thc xy ra khi v ch khi x= y= z= 1 .
196

www.TaiLieuLuyenThi.com
Cty TNHH MTV DVVH Khang Vit

Bi 98. S dng bt ng thc AM GM ta c


2

1
3
3a
2a
.

=
b+c b+c
a+b+c
b+c b+c
b+c
+
+
1
3
.
.1
2a
2a
2a 2a
2

3b
3c
2b
2c
.
;3



a+b+c a+b
a+b+c
c+a
Cng theo v ba bt ng thc trn ta c pcm. ng thc xy ra khi v ch khi
a= b= c .
Ch . Ngoi ra ta c th chng minh bng bt ng thc Bernoulli(xem ch sau).

Tng t ta c

Bi 99. Ch x5 + y 5 x 2 y 2 ( x + y )

xy
x + y + xy
5

Tng t ta c

xy
2 2

( x + y ) + xy

yz

x y

1
z
.
=
xy ( x + y ) + 1 x + y + z

x
zx
y
.
; 5

5
x + y + z z + x + zx x + y + z

y + z + yz
Cng theo v ba bt ng thc trn ta c pcm. ng thc xy ra khi v ch khi
x= y= z= 1 .
Bi 100. S dng bt ng thc AM GM ta c
5

1
ab
ab
ab
=1
1
=1
1 + ab
1 + ab
2
2 ab
1
bc
bc
bc
=1
1
=1
1 + bc
1 + bc
2
2 bc
1
ca
ca
ca
=1
1
=1
1 + ca
1 + ca
2
2 ca
Cng theo v ba bt ng thc trn ta c
1
1
1
ab + bc + ca 15
+
+
3
=

1 + ab 1 + bc 1 + ca
2
4

15
Vy ta ch cn chng minh

a+ b+ c
4

a+ b+ c

9
a+ b+ c

3 < t 3 a v chng minh

t t=

a + b + c,

15 t 2

4 4
Bt ng
a= b= c=

9
t 3 15t + 18 0 ( t 3) t 2 + 3t 6 0 .
2t
thc cui ng v ta c pcm. ng thc xy ra khi v ch khi
1.

197

www.TaiLieuLuyenThi.com
Khm ph t duy K thut gii bt T Bi ton Max Min ng Thnh Nam

K THUT GHP CP TRONG


CHNG MINH BT NG THC AM GM

CH 2:

A. NI DUNG PHNG PHP


1. Ghp cp trc tip 2 hoc 3 hng t mt v ca bt ng thc
Ta cn chng minh bt ng thc
A+ B +C X +Y + Z .
Nu X AB , Y BC , Z CA .
Ta thc hin ghp cp s dng bt ng thc AM- GM cho 2 s dng nh sau:
A + B 2 AB 2 X
B + C 2 BC 2Y
C + A 2 CA 2Z
Cng theo v 3 bt ng thc trn ta c ngay iu phi chng minh.
2. Ghp cp kt hp 2 gia hoc 3 nhn t c 2 v ca bt ng thc
Ta cn chng minh bt ng thc A + B + C X + Y + Z .
Ta c th thc hin nh gi nh sau
A + X 2 AX 2Y
B + Y 2 BY 2 Z
C + Z 2 CZ 2 X
Cng li theo v 3 bt ng thc trn ta c ngay iu phi chng minh.
3. Bin i iu kin hoc bt ng thc trc khi ghp cp
p dng vi cc bt ng thc c iu kin cha tng ng vi bt ng thc
cn chng minh. Thng thng ta s dng php bnh phng 2 v, quy ng 2
v bt ng thc(xem bi tp mu).

B. BI TP MU
Bi 1. Cho x,y,z l cc s thc dng tho mn iu kin 2 xy + xz =
1.
Chng minh rng

3 yz 4 zx 5 xy
+
+
4.
x
y
z

Li gii
Gi P l biu thc v tri ca bt ng thc.
Ta vit li P di dng
yz zx
yz xy xy zx
P = + + 2 + + 3 + .
y
z z
y
x
x
S dng bt ng thc AM GM ta c
198

www.TaiLieuLuyenThi.com
Cty TNHH MTV DVVH Khang Vit

yz zx
yz zx
+ 2
. =
2 z;
x
y
x y
yz xy
yz xy
+
2
. =
2 y;
x
z
x z
xy zx
xy zx
+ 2
. =
2 x.
z
y
z y

Suy ra P 2 z + 4 y + 6 x = 2 ( x + z ) + 4 ( x + y ) 4 zx + 8 xy = 4 .
x= y= z
1
ng thc xy ra khi v ch khi
x=y=z= .
3
1
2 xy + zx =
Nhn xt. c phn tch trn ta s dng ng nht thc nh sau:
yz zx
3 yz 4 zx 5 xy
yz xy xy zx
+
+
= a + + b + + c +
x
y
z
y
z z
y
x
x
+b 3 =
a=
a 1
yz zx

yz xy xy zx
b + c = 5 b = 2 P = + + 2 + + 3 +
y
z z
y
x
x
c=
c 3
+a 4 =

Bi 2. Cho x,y,z l cc s thc dng tho mn iu kin x 2 + y 2 + z 2 =


3.
Chng minh rng

xy yz zx
+
+ 3.
z
x
y

Li gii
Phn tch tm li gii:
xy yz
+
2 y nhng iu kin cho di dng
z
x
bc 2 do vy ta khng th ghp cp s dng trc tip c v cn to ra cc

Nu s dng ghp cp chng hn

nhn t c cha x 2 , y 2 , z 2 nh vy la chn php bnh phng 2 v ca bt


ng thc.
Li gii chi tit:
Bnh phng 2 v bt ng thc cn chng minh c vit di dng
x2 y 2
z2

x2 y 2
z2

y2 z2
x2
+

z 2 x2

y2 z2
x2

y2
+

+ 2 x2 + y 2 + z 2 9

z 2 x2
y2

S dng bt ng thc AM GM cho 2 s dng ta c

199

www.TaiLieuLuyenThi.com
Khm ph t duy K thut gii bt T Bi ton Max Min ng Thnh Nam

x2 y 2
z2

y2 z2
x2

Tng t ta c

y2 z2
x2

x2 y 2 y 2 z 2
. 2 =
2 y2 .
z2
x

z 2 x2
y2

2z2 ;

x2 y 2
z2

z 2 x2
y2

2 x2 .

Cng theo v 3 bt ng thc trn ta c ngay iu phi chng minh. ng thc


xy ra khi v ch khi x= y= z= 1 .
Bi 3. Chng minh rng vi mi a,b,c dng ta lun c
1
1
1
9
.
+
+

b ( a + b ) c ( c + b ) a ( a + c ) 2 ( ab + bc + ca )
Li gii
Bt ng thc cn chng minh tng ng vi
ab + bc + ca ab + bc + ca ab + bc + ca 9
+
+

2
b (a + b)
c (c + b)
a (a + c)
9
c b a
a
b
c
+ + +
+
+

b a c a+b b+c c+a 2


15
b+c a+b c+a
a
b
c

+
+
+
+
+

b
a
c
a+b b+c c+a 2
S dng bt ng thc AM GM cho 2 s dng ta c

b+c
b
b+c b
+
2
.
=
1.
4b
b+c
4b b + c
a+b
a
c+a
c
Tng t ta c:
+
1;
+
1.
4a
a+b
4c
c+a
Cng theo v 3 bt ng thc trn ta c
1a+b b+c c+a
a
b
c
+
+
+
+
3.

+
4 a
b
c a+b b+c c+a
Mt khc

3 a +b b+c c+ a 3
b c a
+
+

=
3 + + +
4 a
b
c 4
a b c

3
b c a 9
.
3 + 33 . . =

4
a b c 2

Cng theo v 2 bt ng thc trn ta c ngay iu phi chng minh. ng thc


xy ra khi v ch khi a= b= c .
Bi 4. Chng minh rng vi mi s thc dng x,y,z ta c
2( x + y + z )

x
y
z
.
1 + 1 + 1 + 2 +
3 xyz
y
z
x

Li gii
Khai trin v tri vit li bt ng thc cho di dng

200

www.TaiLieuLuyenThi.com
Cty TNHH MTV DVVH Khang Vit

x y z x+ y+z
+ +
=
3 xyz
y z x

x2 3 y 2 3 z 2
.
+
+
yz
zx
xy

S dng bt ng thc cho 3 s dng ta c


Tng t ta c:

x x y
x2
.
+ + 33
y y z
yz

y y z
y2 z z x
z2
.
+ + 33
; + + 33
z z x
zx x x y
xy

Cng theo v 3 bt ng thc trn ta c iu phi chng minh. ng thc xy ra


khi v ch khi x= y= z .
Bi 5. Chng minh rng vi mi s thc dng x,y,z ta c
x2 y 2 z 2
.

2
+
+

yz zx xy
y 3 x3 z 3 y 3 x3 z 3

Li gii
S dng bt ng thc AM GM cho 3 s dng ta c
x3

x3
y3
y3
z3
z3
x3

+
+
+

x3
z3
y3
x3
z3
y3

y3

y3

z3

z3

+ 1 33

x3 x3
x2
.
.1
3
=
yz
y3 z3

+ 1 33

y3 y3
y2
.
.1
3
=
zx
z 3 x3

+ 1 33

3z 2
z3 z3
.
.1
=
xy
x3 y 3

x3

Cng theo v 3 bt ng thc trn ta c iu phi chng minh. ng thc xy ra


khi v ch khi x= y= z .
Bi 6. Cho x,y,z l cc s thc khng m.
Chng minh rng ( x + y z )( y + z x )( z + x y ) xyz .
Li gii
Do tng 3 tha s v tri bng x + y + z 0 nn c cc kh nng sau
+ TH1: Nu c 1 tha s m v 2 tha s dng lc ny bt ng thc lun ng.
+ TH2: Nu c 2 tha s m v 1 tha s dng(iu ny v l) v tng ca 2 tha
s chng hn ( x + y z ) + ( y + z x ) = 2 y 0 .
+ TH3: Nu c 3 tha s u khng m lc ta s dng bt ng thc cho 2 s
dng ta c

201

www.TaiLieuLuyenThi.com
Khm ph t duy K thut gii bt T Bi ton Max Min ng Thnh Nam
2

x+ yz+ y+zx
y2
( x + y z )( y + z x )
=
2

y+zx+z+x y
z2
( y + z x )( z + x y )
=
2

z+x y+x+ yz
x2
( z + x y )( x + y z )
=
2

Nhn theo v 3 bt ng thc trn ta c

( x + y z )( y + z x )( z + x y ) x 2 y 2 z 2
.
( x + y z )( y + z x )( z + x y ) xyz
2

Bt ng thc c chng minh. ng thc xy ra khi v ch khi x= y= z .


Nhn xt. V cha bit tnh m dng ca mi tha s v tri nn cha th p
dng trc tip bt ng thc AM GM v ta cn phn chia cc trng hp.
+ y l mt dng ca bt ng thc Schur bc ba c rt nhiu ng dng trong
vic gii quyt mt lp bi ton(xem ch sau).
Chng hn vi x,y,z c tng bng 3 ta c bt ng thc
4 ( xy + yz + zx ) 3 xyz 9 .
Bi tp tng t
Cho x,y,z l cc s thc dng. Chng minh

x
y
z 1
1
1
.
1
y + z z + x x + y 8

Bi 7. Cho a,b,c l cc s thc dng tho mn iu kin a 2 + b 2 + c 2 =


3.
Chng minh rng

a3

b3

c3

3
.
2

c +3
a +3
Li gii
S dng bt ng thc AM GM cho 3 s dng ta c
a3
b2 + 3
b3
c2 + 3
c3
a2 + 3

+
+
+

b +3

a3
b2 + 3
b3
c2 + 3
c3
a2 + 3

b2 + 3 3 2
a
8
2

c2 + 3 3 2
b
8
2

a2 + 3 3 2
c
8
2

Cng theo v 3 bt ng thc trn v kt hp vi iu kin a 2 + b 2 + c 2 =


3 ta
c ngay iu phi chng minh. ng thc xy ra khi v ch khi a= b= c= 1 .
202

www.TaiLieuLuyenThi.com
Cty TNHH MTV DVVH Khang Vit

Bi 8. Cho a,b,c l cc s thc dng tho mn iu kin a + b + c =


3.
Chng minh rng

(1 + a )2 (1 + b )2 + (1 + b )2 (1 + c )2 + (1 + c )2 (1 + a )2 24 .
1 + c2

1 + a2
1 + b2
Li gii
Gi P l biu thc v tri ca bt ng thc.
S dng bt ng thc AM GM cho 2 s dng ta c:

(1 + a )2 (1 + b )2 = (1 + ab + a + b )2 4 (1 + ab )( a + b ) =
Suy ra

4a 1 + b 2 + 4b 1 + a 2 .

(1 + a )2 (1 + b )2 4a.1 + b2 + 4b. 1 + a 2
1 + c2

Tng t ta c:

1 + c2

(1 + b )2 (1 + c )2
1 + a2

(1 + c )2 (1 + a )2

4b.

a + c2

1 + c2
1 + a2

4a.

1 + c2

1 + b2
1 + b2
Cng theo v 3 bt ng thc trn ta c

+ 4c.

1 + b2
1 + a2

+ 4c.

1 + a2
1 + b2

1 + b2 1 + c2
1 + a2 1 + c2
1 + a 2 1 + b2
4
4
P 4a
b
c
+
+
+
+
+

2
1 + c2 1 + b2
1 + c2 1 + a2
1 + a 2

1+ b
1 + b2 1 + c2
1 + a2 1 + c2
1 + a 2 1 + b2
.
8
b
.
8
c
.
+
+
1 + c2 1 + b2
1 + c2 1 + a2
1 + b2 1 + a 2
= 8 ( a + b + c )= 24
8a

Bt ng thc c chng minh. ng thc xy ra khi v ch khi a= b= c= 1 .


a
b
c
Bi 9. Cho a,b,c l cc s thc dng tha mn iu kin
2.
+
+
=
a +1 b +1 c +1
Tm gi tr nh nht ca biu thc P = ab + bc + ca .
Li gii
Theo gi thit kt hp s dng bt ng thc AM GM cho 2 s dng ta c
a
b
c
1
1
=
1
+1
= +

a +1
b +1
c +1 b +1 c +1

Tng t ta c:

b +1
c

c +1

( b + 1)( c + 1)

( a + 1)( c + 1)
2

( a + 1)( b + 1)
203

www.TaiLieuLuyenThi.com
Khm ph t duy K thut gii bt T Bi ton Max Min ng Thnh Nam

Ln lt nhn theo v hai bt ng thc ta c:


ab

( a + 1)( b + 1) ( c + 1)

Tng t ta c: bc

( a + 1)( b + 1)

( b + 1)( c + 1)

ab

; ca

( a + 1)( b + 1)
c +1

( c + 1)( a + 1)

.
a +1
b +1
Cng li theo v 3 bt ng thc trn v s dng AM-GM ta c:

P 4

12 3

( a + 1)( b + 1)
c +1

( b + 1)( c + 1)
a +1

( a + 1)( c + 1)
b +1

( a + 1)( b + 1) ( b + 1)( c + 1) ( a + 1)( c + 1)

=
.
.
12
c +1
a +1
b +1
ng thc xy ra khi v ch khi a= b= c= 2 .
Bi 10. Cho x,y,z l cc s thc dng tho mn iu kin xy + yz + zx =
1.

Chng minh rng 3 3 +

x2 y 2 z 2
2
+
+
( x + y + z) .
y
z
x

Li gii
Trc ht ta chng minh bt ng thc sau

2
2
2
x2 y 2 z 2 ( x + y + z ) x + y + z
.
+
+

y
z
x
xy + yz + zx

Tht vy, bt ng thc tng ng vi:


x2 y 2 z 2
( xy + yz + zx) +
+ ( x + y + z )( x 2 + y 2 + z 2 ) .
y
z
x

x3 z y 3 x z 3 y
+
+
xz 2 + zy 2 + yx 2 .
y
z
x

S dng bt ng thc AM-GM ta c:

x3 z y 3 x
+
2 x2 y .
y
z

y3 x z3 y
+
2 y2 z .
z
x

x3 z z 3 y
+
2z2 x .
y
x

Cng theo v 3 bt ng thc trn ta c iu phi chng minh.


Gi P l biu thc v tri bt ng thc ban u ta c
P 3 3 + ( x + y + z )( x 2 + y 2 + z 2 ) .
204

www.TaiLieuLuyenThi.com
Cty TNHH MTV DVVH Khang Vit

Vy ta cn chng minh
3 3 + ( x + y + z )( x 2 + y 2 + z 2 ) ( x + y + z ) 2
( x + y + z )( x 2 + y 2 + z 2 ) x 2 + y 2 + z 2 + 3 1
( x 2 + y 2 + z 2 )( x + y + z 1) 3 1
2
( x + y + z ) 2 ( xy + yz + zx ) ( x + y + z 1) 3 1

2
( x + y + z ) 2 ( x + y + z 1) 3 1

Bt ng thc lun ng do x + y + z 3( xy + yz + zx) =3 .


Bt ng thc c chng minh. ng thc xy ra khi v ch khi
1
.
x= y= z=
3
Bi tp tng t
Cho x,y,z l cc s thc dng c tng bng 1.
Chng minh rng

x2 y 2 z 2
+
+
3 x2 + y 2 + z 2 .
y
z
x

C. BI TP RN LUYN
Bi 1. Chng minh rng vi mi s thc x ta c
x

12 15 20
x
x
x
+ + 3 +4 +5 .
5
4
3

Bi 2. Chng minh rng vi mi s dng x,y,z ta c
x3 y 3 z 3
+
+ x2 + y 2 + z 2 .
y
z
x

Bi 3. Chng minh rng vi mi x,y,z l cc s thc dng ta c


x5
y

y5
z

z5
x

x3 + y 3 + z 3 .

Bi 4. Chng minh rng vi mi s thc dng x,y,z ta c


x3 y 3 z 3
+
+
x+ y+z.
yz zx xy

Bi 5. Cho x,y,z l cc s thc dng tho mn iu kin x + y + z =


xyz .
Chng minh rng

x
y

y
z

z
x3

1.

205

www.TaiLieuLuyenThi.com
Khm ph t duy K thut gii bt T Bi ton Max Min ng Thnh Nam

Bi 6. Cho a,b,c l di 3 cnh mt tam gic. Chng minh rng


a
b
c
+
+
3.
a+bc b+ca c+ ab
Bi 7. Cho a,b,c l cc s thc dng tho mn iu kin a 2 + b 2 + c 2 =
3.
Chng minh rng

a3

b +1
2

b3
c +1
2

c3
a +1
2

3
.
2

Bi 8. Cho x,y,z l cc s thc dng tho mn iu kin 2 xz + yz + 3 xy =


1.
Chng minh rng

7 yz 8 zx 9 xy
+
+
4.
x
y
z

Bi 9. Cho x,y,z l cc s thc dng tho mn iu kin xyz = 1 .

1
1
1
Chng minh rng x + 1 y + 1 z + 1 1 .
y
z
x

1
1
1
Bi 10. Cho a,b,c l cc s thc dng tha mn
+
+
=
1.
a +1 b +1 c +1
Chng minh rng abc 8 .
Bi 11. Cho x,y,z l cc s thc dng. Chng minh

x
y
z 1
1
1
.
1
y + z z + x x + y 8

D. HNG DN GII P S
Bi 1. S dng bt ng thc AM GM cho 2 s dng ta c
x

12 15
12
+ 2
5 4
5

15
. =
2.3x
4
x

15 20
15 20
2.5 x
+ 2 . =
4 3
4 3
20 12
20
+ 2
3 5
3

12
. =
2.4 x
5

Cng li theo v 3 bt ng thc trn ta c iu phi chng minh. ng thc


xy ra khi v ch khi x = 0 .
Bi 2. S dng bt ng thc AM GM cho 3 s dng ta c
x3 x3
x3 x3
+
+ y 2 33 . . y 2 =
3x 2
y
y
y y

206

www.TaiLieuLuyenThi.com
Cty TNHH MTV DVVH Khang Vit

y3 y3
y3 y3 2
. .z =
3y2
+
+ z 2 33
z
z
z z
z3 z3
z3 z3
3z 2
+ + x 2 3 3 . .x 2 =
x
x
x x
Cng theo v 3 bt ng thc trn ta c iu phi chng minh. ng thc xy ra
khi v ch khi x= y= z .
Cch 2: S dng bt ng thc AM GM cho 2 s dng ta c
x3
x3
y3
z3
+ xy 2
=
.xy 2 x 2 ; + yz 2 y 2 ; + xz 2 z 2 .
y
y
z
x

Cng theo v 3 bt ng thc trn kt hp vi x 2 + y 2 + z 2 xy + yz + zx ta c


iu phi chng minh.
Bi 3. S dng bt ng thc AM GM cho 2 s dng ta c
x5
y2
y5
z2
z5

+ xy 2 2
+ yz 2 2

y2
y5
z2

.xy 2 =
2 x3
. yz 2 =
2 y3

z5

.zx 2 =
2z3
x2
x2
Cng theo v 3 bt ng thc trn ta c

x5
y2

+ zx 2 2

x5

y5
z2

z5
x2

+ xy 2 + yz 2 + zx 2 2 x3 + y 3 + z 3 .

Vy ta ch cn chng minh x3 + y 3 + z 3 xy 2 + yz 2 + zx 2 .
Bt ng thc ny hin nhin ng theo AM GM cho 3 s dng
V x3 + y 3 + y 3 3 xy 2 ; y 3 + z 3 + z 3 3 yz 2 ; z 3 + x3 + x3 3 zx 2 .
Cng li theo v 3 bt ng thc trn ta c iu phi chng minh. ng thc
xy ra khi v ch khi x= y= z .
Bi 4. S dng bt ng thc AM GM cho 3 s dng ta c
x3
x3
+ y + z 33 . y.z =
3x
yz
yz
y3
y3
+ z + x 33
. z. x =
3y
zx
zx
z3
z3
+ x + y 33 .x. y =
3z
xy
xy
207

www.TaiLieuLuyenThi.com
Khm ph t duy K thut gii bt T Bi ton Max Min ng Thnh Nam

Cng theo v 3 bt ng thc trn ta c iu phi chng minh. ng thc xy ra


khi v ch khi x= y= z .
1
1
1
+
+ =
1.
xy yz zx

Bi 5. Theo gi thit ta c

Vy bt ng thc cn chng minh c vit li di dng:


x
y
z
1
1
1
+ 3+ 3
+
+
3
xy yz zx
y
z
x

x 4 z 3 + y 4 x3 + z 4 y 3
3 3 3

x y z

x+ y+z
xyz

x 4 z 3 + y 4 x3 + z 4 y 3 ( x + y + z ) x 2 y 2 z 2
x2 z y 2 x z 2 y
x2 y 2 z 2 2 + 2 + 2 ( x + y + z ) x2 y 2 z 2
y
z
x

x2 z
y2

y2 x
z2

z2 y
x2

x+ y+z

S dng bt ng thc AM GM cho 3 s dng ta c


x2 z
y2

y2 x
z2
z2 y
x2

+
+

y2 x
z2
z2 y
x2
x2 z
y2

+ z 33

x2 z y 2 x
.
.z =
3x
y2 z2

+ x 33

y2 x z2 y
.
.x =
3y
z 2 x2

+ y 33

z 2 y x2 z
.
.y =
3z
x2 y 2

Cng theo v 3 bt ng thc trn ta c iu phi chng minh. ng thc xy ra


1
khi v ch khi x= y= z=
.
3
Bi 6. S dng bt ng thc AM GM cho 3 s dng ta c
a
b
c
a
b
c
.
+
+
33
.
.
a+bc b+ca c+ab
a+bc b+ca c+ab

Ch . ( a + b c )( b + c a )( c + a b ) abc .
Ta c iu phi chng minh. ng thc xy ra khi v ch khi a= b= c .
Bi 7. S dng bt ng thc AM GM cho 3 s dng ta c
a3
b2 + 1
208

a3
b2 + 1

b2 + 1 3 2

a ;
2 2
2

www.TaiLieuLuyenThi.com
Cty TNHH MTV DVVH Khang Vit

b3
c2 + 1
c3
a2 + 1

+
+

b3
c2 + 1
c3
a2 + 1

c2 + 1 3 2

b ;
2 2
2

a2 + 1 3 2

c .
2 2
2

Cng theo v 3 bt ng thc trn v kt hp vi iu kin a 2 + b 2 + c 2 =


3 ta
c ngay iu phi chng minh. ng thc xy ra khi v ch khi a= b= c= 1 .
Bi 8. Gi P l biu thc v tri ca bt ng thc.
yz zx
yz xy xy zx
Ta vit li P di dng P = 3 + + 4 + + 5 + .
y
z z
y
x
x
S dng bt ng thc AM GM ta c
yz zx
yz zx
. =
2z
+ 2
x
y
x y
yz xy
yz xy
. =
2y
+
2
x
z
x z
xy zx
xy zx
. =
2x
+ 2
z
y
z y

Suy ra P 6 z + 8 y + 10 x = 4 ( x + z ) + 2 ( z + y ) + 6 ( x + y )
8 xz + 4 yz + 12 xy =
4
x= y= z
1
ng thc xy ra khi v ch khi
x=y=z= .
6
1
2 xz + yz + 3 xy =
Bi 9. V x,y,z l cc s thc dng tho mn xyz = 1 nn tn ti cc s thc dng
a
b
c
khi bt ng thc c vit li di dng:
a,b,c tho mn=
x =
,y =
,z
b
c
a
( a + b c )( b + c a )( c + a b ) abc .

y l bt ng thc c chng minh. ng thc xy ra khi v ch khi


x= y= z= 1 .
Cch 2: Theo gi thit ta c

1
1 1
1

x + 1= x 1 +
= x 1 + z
y
xy x
x

1
1
1
1

x + 1 z + 1= x 1 + z z + 1
y
x
x
x

1
= x z 2 1 xz 2 .
x

209

www.TaiLieuLuyenThi.com
Khm ph t duy K thut gii bt T Bi ton Max Min ng Thnh Nam

1
1

Tng t ta c: y + 1 x + 1 yx 2 .
z
y

1
1

2
z + 1 y + 1 zy
x
z

Nhn theo v 3 bt ng thc trn kt hp iu kin xyz = 1 ta c iu phi


chng minh. ng thc xy ra khi v ch khi x= y= z= 1 .

a
1
1
1
.
=
1
= +
a +1
a +1 b +1 c +1
S dng bt ng thc AM-GM cho 2 s dng ta c
a
2
.

a +1
( b + 1)( c + 1)

Bi 10. Theo gi thit ta c

Tng t ta c:

b +1
c

c +1

( a + 1)( c + 1)

( a + 1)( b + 1)

Nhn theo v 3 bt ng thc trn ta c ngay iu phi chng minh.


ng thc xy ra khi v ch khi a= b= c= 2 .
Nhn xt. Vi cch lm tng t trn ta x l c bi ton tng qut cho n s dng.
1
1
1
Bi ton. Vi x1 , x2 ,..., xn > 0 tha mn
+
+ ... +
=
1 ta lun c
1 + x1 1 + x2
1 + xn
x1 x2 ...xn ( n 1) .
n

Bi 11. Bt ng thc c vit di dng tng ng:

8 ( x + y z )( y + z x )( z + x y ) ( x + y )( y + z )( z + x ) .

Ch ta c ( x + y z )( y + z x )( z + x y ) xyz .
8 xyz ( x + y )( y + z )( z + x )

T suy ra pcm. ng thc xy ra khi v ch khi x= y= z .

210

www.TaiLieuLuyenThi.com
Cty TNHH MTV DVVH Khang Vit

CH 3:

K THUT S DNG BT NG THC AM GM


DNG CNG MU S
1
1
1
n2
.
+
+ ... +

A1 A2
An A1 + A2 + ... + An

A. NI DUNG PHNG PHP


Xut pht t bt ng thc
1 1
1
( a1 + a2 + ... + an ) + + ... +
an
a1 a2
Do

n
=
n2 .
n n a1a2 ...an . n
a1a2 ...an

1 1
1
n2
.
+
+ ... +

a1 a2
an a1 + a2 + ... + an

ng thc xy ra khi v ch khi a1= a2= ...= an .


Hai bt ng thc dng trn hay c s dng nht l
1 1
4
+
a b a+b
1 1 1
9
+ +
a b c a+b+c
Di y ti s trnh by mt lp bi ton dng trn.
B. BI TP MU
Bi 1. Cho a,b,c l cc s thc dng tho mn iu kin a + b + c =
3 .
1
1
1
1
1
1
Chng minh rng
.
+
+

+
+
a + 3b b + 3c c + 3a a + 3 b + 3 c + 3
Li gii
1 1
4
ta c
S dng bt ng thc +
x y x+ y
1
1
4
2
+

=.
a + 3b c + 3 a + 3b + c + 3 b + 3
1
1
4
2
Tng t ta c
+

=
b + 3c a + 3 a + b + 3c + 3 c + 3
1
1
4
2
+

=
c + 3a b + 3 3a + b + c + 3 a + 3
Cng theo v 3 bt ng thc trn ta c iu phi chng minh. ng thc xy ra
khi v ch khi a= b= c= 1 .
Bi 2. Cho a,b,c l di ba cnh mt tam gic tha mn iu kin 2c + b =
abc .
Tm gi tr nh nht ca biu thc

211

www.TaiLieuLuyenThi.com
Khm ph t duy K thut gii bt T Bi ton Max Min ng Thnh Nam

3
4
5
.
+
+
b+ca a +cb a +bc
Li gii
2
1
T iu kin ta c:
+
=
1.
ab ac
S dng bt ng thc AM-GM ta c:
P=

12 1 1 32 1 1 3 2 1
3 2 1
=
1
. + + + + + 2 3.
+ =

a b c 3 a b c 12 a b c
a b c

Ta c:
1
1
1
1
1
1

P = 2
+
+
+
+
+ 3
.
b+ca a +bc b+ca a +cb a +cb a +bc

S dng bt ng thc
P

1 1
4
+
, x, y > 0 ta c:
x y x+ y
8
4 12
3 2 1
+
+ = 2 + + 4 3 .
2b 2c 2a
a b c

ng thc xy ra khi v ch khi a= b= c=

3.

Vy gi tr nh nht ca P bng 4 3 t ti a= b= c= 3 .
Nhn xt. im quan trng nht ca bi ton l d on du bng ti a= b= c .
3 2 1
Ngoi ra ta c th nh gi P nh sau: P 2 + + .
a b c
Theo gi thit ta c:

2 1
3

+ =
a suy ra P 2 + a 4 3 . ng thc xy ra
a
b c

khi v ch khi a= b= c= 3 .
Bi tp tng t
Cho a,b,c l di ba cnh mt tam gic tho mn iu kin
2ab + 3ac + 4bc =
9abc .
5
6
7
.
Tm gi tr nh nht ca biu thc P =
+
+
b+ca a +cb a +bc
Bi 3. Cho a,b,c l cc s thc dng tho mn iu kin a 2 + b 2 + c 2 =
3.
1
1
1
4
4
4
.
Chng minh rng
+
+
2
+ 2
+ 2
a+b b+c c+a a +7 b +7 c +7
Li gii
1
1
4
1 1
4
S dng bt ng thc +
ta c
.
+

a + b b + c a + 2b + c
x y x+ y
212

www.TaiLieuLuyenThi.com
Cty TNHH MTV DVVH Khang Vit

Theo AM GM ta c a 2 + 1 2a

2 b 2 + 1 4b
c 2 + 1 2c
Cng theo v 3 bt ng thc trn ta c

(a

2 ( a + 2b + c ) a 2 + 2b 2 + c 2 + 4 =

Suy ra

+ b2 + c2 + 4 + b2 = b2 + 7 .

1
1
8
.
+
2
a+b b+c b +7

1
1
8
+
2
b+c c+a c +7
1
1
8
+
2
c+a a+b a +7
Cng theo v 3 bt ng thc trn ta c iu phi chng minh. ng thc xy ra
khi v ch khi a= b= c= 1 .
Bi 4. Cho a,b,c l cc s thc dng chng minh rng

Tng t ta c:

ab
a +b +a c+b c
3

11 1 1
+ + .
b +c +b a +c a c +a +c b+a b 4 a b c
Li gii
bc

ca

Ta c a3 + b3 ab ( a + b ) .

ab
a +b +a c+b c
3

ab

ab ( a + b ) + c a 2 + b 2

1
1
ab

+
4 ab ( a + b ) c a 2 + b 2

ab 1 1 1 1 1
1 1
1
= .
+ . 2
. + + c
4 a + b 4 a + b 2 c 16 a b 8

Tng t ta c

bc
b +c +b a+c a
ca
3

1 1 1 1
.
+ +
16 b c 8a

1 1 1 1
+ +
16 c a 8b

c +a +c b+a b
Cng theo v ba bt ng thc trn ta c pcm. ng thc xy ra khi v ch khi
a= b= c .
Bi 5. Cho a,b,c l cc s thc dng c tch bng 1. Chng minh
1
1
1
3
+
+
.
ab + a + 2 bc + b + 2 ca + c + 2 4
3

213

www.TaiLieuLuyenThi.com
Khm ph t duy K thut gii bt T Bi ton Max Min ng Thnh Nam

Li gii
1
1
1 1
1 1 c
1
=

+
+
Ta c
=

ab + a + 2 (ab + 1) + (a + 1) 4 ab + 1 a + 1 4 c + 1 a + 1
1
1
1 1
1 1 a
1
=

+
+
=

bc + b + 2 (bc + 1) + (b + 1) 4 bc + 1 b + 1 4 a + 1 b + 1
1
1
1 1
1 1 b
1
=

+
+
=

ca + c + 2 (ca + 1) + (c + 1) 4 ca + 1 c + 1 4 b + 1 c + 1
Cng theo v ba bt ng thc trn ta c cpm.
ng thc xy ra khi v ch khi a= b= c= 1 .

C. BI TP RN LUYN
Bi 1. Chng minh rng vi mi x,y,z dng ta c
1
1
1
xy + yz + xz
.
+
+

2x + y + z 2 y + z + x 2z + x + y
4 xyz
Bi 2. Cho x,y,z l cc s thc dng tho mn iu kin xy + yz + zx + xyz =
4.
Chng minh rng

1
1
1
1
+
+

2x + y + 6 2 y + z + 6 2z + x + 6 3

Bi 3. Cho a,b,c l cc s thc dng. Chng minh


6(a + b + c)
a
2b
3c
.
+
+

a +1 b + 2 c + 3 a + b + c + 6
Bi 4. Cho a,b,c l cc s thc dng. Chng minh
1
1
1
4
4
4
.
+
+

+
+
a + 2b b + 2c c + 2a 3a + 4b + 5c 3b + 4c + 5a 3c + 4a + 5b
Bi 5. Cho x,y,z l cc s thc dng tha mn iu kin x + y 2 + z 3 =
13 .
Tm gi tr nh nht ca biu thc P=

1
4 12
+ + .
1+ x y z

Bi 6. Chng minh rng vi mi s thc dng ty a,b,c,d ta lun c


a d d b bc ca
+
+
+
0.
b+d c+b c+a d +a
Bi 7. Cho a,b,c l di ba cnh mt tam gic tho mn iu kin
2ab + 3ac + 4bc =
9abc .
5
6
7

Tm gi tr nh nht ca biu thc P= 2


+
+
.
b+ca a +cb a +bc
Bi 8. Cho a,b,c l cc s thc dng c tch bng 1. Chng minh
a
b
c
1
+
+
a 2 + b2 + c2 .
ca + 1 ab + 1 bc + 1 2

214

www.TaiLieuLuyenThi.com
Cty TNHH MTV DVVH Khang Vit

1 1 1
+ + =
1.
a b c
1
8
1
Chng minh rng
+
.
( a 1)( b 1)( c 1) ( a + 1)( b + 1)( c + 1) 4

Bi 9. Cho a,b,c l cc s thc dng tho mn iu kin

HNG DN GII P S
1
1
1
1
16
Bi 1. S dng bt ng thc
ta c:
+
+ +

a1 a2 a3 a4 a1 + a2 + a3 + a4
1
1 2 1 1
+ +
2 x + y + z 16 x y z
1
1 2 1 1
+ +
2 y + z + x 16 y z x
1
1 2 1 1
+ +
2 z + x + y 16 z x y
Cng theo v 3 bt ng thc trn ta c iu phi chng minh. ng thc xy ra
khi v ch khi x= y= z .

Bi 2. Gi P l biu thc v tri ca bt ng thc cho.


1
1
1
9
S dng bt ng thc
ta c:
+
+

a1 a2 an a1 + a2 + a3
1
=
2x + y + 6

1
1 2
1
.

+
( x + 2 ) + ( x + 2 ) + ( y + 2 ) 9 x + 2 y + 2

Tng t ta c
1
1 2
1

+

2y + z + 6 9 y + 2 z + 2
1
1 2
1

+

2z + x + 6 9 z + 2 x + 2
Cng theo v 3 bt ng thc trn ta c
1 1
1
1
P
+
+
.
3 x + 2 y + 2 z + 2
Ch iu kin bi ton c vit li di dng:
1
1
1
+
+
=
1.
x+2 y+2 z+2

T suy ra iu phi chng minh. ng thc xy ra khi v ch khi x= y= z= 1 .


215

www.TaiLieuLuyenThi.com
Khm ph t duy K thut gii bt T Bi ton Max Min ng Thnh Nam

Bi 3. Bt ng thc c vit li di dng

6(a + b + c)
2b
3c
a
1+
2+
3
6
a +1
b+2
c+3
a+b+c+6
1
4
9
36

+
+

a +1 b + 2 c + 3 a + b + c + 6
Bt ng thc cui lun ng. ng thc xy ra khi v ch khi
b+2 c+3
.
a+
=
1
=
2
3

Bi 4.
Ta c
16
3a + 4b + 5c

(1 + 1 + 1 + 1)2
1
2
1
.

+
+
+
+
+
+
+
+
+
+
+
+
a
2
b
b
2
c
b
2
c
c
2
a
a
2
b
b
2
c
c
2a
(
) (
) (
) (
)

Xy dng tng t 2 bt ng thc nh trn ri cng li ta c iu phi chng minh.


Bi 5. im ri ca bi ton l=
x 1,=
y 2,=
z 2.
S dng bt ng thc AM-GM ta c
33 =x + y 2 + z 3 + 20 =x + y 2 + 4 + z 3 + 8 + 8 x + 4 y + 12 z .

S dng bt ng thc C-S dng phn thc ta c:


1
1
1 (1 + 4.1 + 12.1)
17 2
17 2 289 17
.
+ 4. + 12.
=

=
=
1+ x
y
z 1 + x + 4 y + 12.z x + 4 y + 12 z + 1 34
34
2
2

P=

ng thc xy ra khi v ch khi =


x 1,=
y 2,=
z 2.
17
t ti x= 1, y= z= 2 .
2
Bi 6. Cng 4 vo 2 v bt ng thc tng ng vi:
a+b c+d b+a c+d
+
+
+
4.
b+d c+b c+a d +a

Vy gi tr nh nht ca P bng

1
1
1
1
(c + d )
+
+
+ ( a + b)
4.
b+c a+d
b+d c+a
1
1
4
Ch
+

;
b+c a+d a+b+c+d
1
1
4
+

b+d c+a a+b+c+d


T suy ra iu phi chng minh. ng thc xy ra khi v ch khi a= b= c= d .
4 3 2
Bi 7. Theo gi thit ta c: 2ab + 3ac + 4bc
= 9abc + + = 9 .
a b c
S dng bt ng thc AM GM dng cng mu, ta c
216

www.TaiLieuLuyenThi.com
Cty TNHH MTV DVVH Khang Vit

1
1
1
1
1
1

+
+
+
P= 2.
+ 4.
+ 3.

b+ca a +cb
a +cb a +bc
a+bc b+ca
.
4
4
4
4 3 2
P 2. + 4. + 3. P 2 + + =
18
2c
2a
2b
a b c

Du bng xy ra khi v ch khi tam gic ABC u a= b= c= 1 .


x
y
z
.
Bi 8. Tn ti cc s thc dng x,y,z sao cho=
a =
,b =
,c
y
z
x
Bt ng thc tr thnh

x2
y

y2
z

z2
x

2x
2y
2z
.
+
+
y+z z+x x+ y

Ch s dng bt ng thc AM GM ta c
x2
y

x2
y

+
+

y2
z

y2
z

+
+

z2
x

z2
x

x y z
+ +
y z x

x y z
+ +
z x y

Cng theo v hai bt ng thc trn v ch


1 1
1 1
x y z x y z
1 1
+ + + + + = x + + y + + z +
y z x z x y
x z
y z
x y
4x
4y
4z

+
+
y+z z+x x+ y

Bt ng thc c chng minh. ng thc xy ra khi v ch khi a= b= c= 1 .


Bi 9. Gi thit ta c ab + bc + ca =
abc .
Suy ra
1
8
1
8
.
+
=
+
( a 1)( b 1)( c 1) ( a + 1)( b + 1)( c + 1) a + b + c 1 2abc + a + b + c + 1
Ch a + b + c

9
=
9;
1 1 1
+ +
a b c

abc = ab + bc + ca 3abc ( a + b + c ) 3 3abc abc 27

T ta c ngay iu phi chng minh. ng thc xy ra khi v ch khi


a= b= c= 3 .

217

www.TaiLieuLuyenThi.com
Khm ph t duy K thut gii bt T Bi ton Max Min ng Thnh Nam

CH 4:

K THUT S DNG BT NG THC


CAUCHY SCHWARZ

A. NI DUNG PHNG PHP

Cho hai dy s thc ( a1 , a2 ,..., an ) ; ( b1 , b2 ,..., bn ) khi ta c

( a1b1 + a2b2 + ... + anbn )2 ( a12 + a22 + ... + an2 )( b12 + b22 + ... + bn2 ) .
ng thc xy ra khi v ch khi=
ai kb
=
i , i 1, n, k .
Chng minh.
Xt tam thc bc hai
f ( x)=
=

(a

2
1

+ a22 + ... + an2 x 2 2 ( a1b1 + a2b2 + ... + an bn ) x + b12 + b22 + ... + bn2

( a1 x b1 )2 + ( a2 x b2 )2 + ... + ( an x bn )2 0

Theo nh l v du ca tam thc bc hai ta c

)(

).

' 0 ( a1b1 + a2b2 + ... + an bn ) a12 + a22 + ... + an2 b12 + b22 + ... + bn2 .
2

H qu(Bt ng thc Mincopski) Vi 2 dy s thc a1 , a2 ,.., an v b1 , b2 ,..., bn ta c


a12 + b12 + a22 + b22 + ... + an2 + bn2

( a1 + a2 + ... + an )2 + ( b1 + b2 + ... + bn )2

ng thc xy ra khi v ch khi a1 , a2 ,.., an v b1 , b2 ,..., bn l 2 b s t l.


B. BI TP MU
47
Bi 1. Cho x,y,z l cc s thc dng tho mn iu kin x + y + z = .
12
235
Chng minh rng 3 x 2 + 4 y 2 + 5 z 2
.
12
Li gii
Phn tch tm li gii:
Ta cn tm cc s a,b,c dng sao cho

( 3x

)(

) (

+ 4 y 2 + 5 z 2 a 2 + b 2 + c 2 a 3 x + 2by + c 5 z .

Vi v phi bt ng thc l mt hng s v vy ch cn chn a,b,c sao cho


a 3 x + 2by + c 5 z = x + y + z .
1 1 1
Ta c ngay ( a; b; c ) =
; ;
.
3 2 5

218

www.TaiLieuLuyenThi.com
Cty TNHH MTV DVVH Khang Vit

Li gii chi tit:

2 47
1 1 1
3x 2 + 4 y 2 + 5 z 2 + + ( x + y + z ) =

3 4 5
12 .
235
3x 2 + 4 y 2 + 5 z 2
12

5 5
ng thc xy ra khi v ch khi ( x; y; z ) = ; ;1 .
3 4
Bi 2. Cho x,y,a,b,c l cc s thc tho mn iu kin

c2 .
( x + a )2 + ( y + b )2 + ( x + y )2 =

Chng minh rng ( a + b ) 3c 2 .


2

Li gii
Phn tch tm li gii:

Ta cn chng minh ( a + b ) 3 ( x + a ) + ( y + b ) + ( x + y )
2

Ch a + b =

).

( x + a) + ( y + b) + (x y ) .

Nn da vo bt ng thc trn ta s dng bt ng thc C S cho hai b s

(1;1;1) ; ( x + a; y + b; x y ) .

Li gii chi tit:


S dng bt ng thc C S ta c

( a + b )2 =
( x + a ) + ( y + b ) + ( x y )

2
2
2
3 ( x + a ) + ( y + b ) + ( x + y ) =
3c 2 .

Bt ng thc c chng minh.


Bi 3. Cho x,y,z l cc s thc tho mn x 5, y 6, z 7; x 2 + y 2 + z 2 125 .
Tm gi tr nh nht ca biu thc P = x + y + z .
Li gii
x + y + z=

x + y + z 2 + 2 yz + 2 zx + 2 xy
2

125 + 2 xy + 2( x + y ) z

.
125 + 2 xy + 2( x + y ) 125 x y
2

125 + 60 + 22 64 =
19

C s nh nht ta p v 2 bin c chn di nh nht l x v y khi tch xy


nh nht trong cc tch.

219

www.TaiLieuLuyenThi.com
Khm ph t duy K thut gii bt T Bi ton Max Min ng Thnh Nam

x2 + y 2 =
9

Bi 3. Cho x,y,z,t l cc s thc tho mn iu kin z 2 + t 2 =


16 .
xt + yz =
12

Chng minh rng x + z 5 .


Li gii

Ch h iu kin ta c ng thc sau: ( xt + yz ) = x 2 + y 2


2

)( z

+ t 2 = 144 .

Do =
x kt=
, y kz .

2
2
V vy x 2 + y =
k 2 t2 + z2 k=

9
16

)(

2
k 2 + 1 x2 + y 2
kx + y )
(
y kx + y
2
S = x+z= x+ =
S =

= 25 .
k
k
k2
k2
Bt ng thc c chng minh.
Bi tp tng t

a 2 + b2 + c 2 =
9

2
2
2
16 .
Cho a,b,c,x,y,z l cc s thc tho mn iu kin x + y + z =
ax + by + cz =
12

Chng minh rng a + y + z 41 .


Bi 4. Cho x,y,z l cc s thc khng m. Chng minh
x3 y + y 3 z + z 3 x + x3 z + y 3 x + z 3 y

( x + y + 2 z )2
.
2

Li gii
S dng bt ng thc C S ta c:

x3 y + y 3 z + z 3 x + x3 z + y 3 x + z 3 y 2 x3 y + y 3 z + z 3 x + x3 z + y 3 x + z 3 y
=

2 xy ( x 2 + y 2 )

2( xy + yz + zx)( x 2 + y 2 + z 2 )

x 2 + y 2 + z 2 + 2( xy + yz + zx)
2

( x + y + z )2 ( x + y + 2 z )2

2
2
ng thc xy ra khi v ch khi=
x y=
,z 0.
=

220

www.TaiLieuLuyenThi.com
Cty TNHH MTV DVVH Khang Vit

Bi 5. Cho x1 , x2 , y1 , y2 , z1 , z2 l cc s thc tho mn iu kin


x1 , x2 > 0, x1 y1 > z12 , x2 y2 > z22 .

Chng minh rng

x1 y1

z12

1
x2 y2

z22

( x1 + x2 )( y1 + y2 ) ( z1 + z2 )2

Li gii
Bt ng thc tng ng vi:

1
1
( x + x )( y + y ) ( z + z )2 8 .
+

2
1
2
2
2

1 2 1
x1 y1 z1 x2 y2 z2
S dng bt ng thc C S ta c
2

z2 z2
z
z
z2 ) x1 . 1 + x2 . 2 ( x1 + x2 ) 1 + 2 .
( z1 +=
x

x1
x2
1 x2

Kt hp s dng bt ng thc AM GM ta c:
2

z12

( x1 + x2 )( y1 + y2 ) ( z1 + z2 )2 ( x1 + x2 )( y1 + y2 ) ( x1 + x2 )

x1

z22

x2

z2
z2
= ( x1 + x2 ) y1 1 + y2 2

x1
x2

x y z2 x y z2
=
( x1 + x2 ) 1 1 1 + 2 2 2
x1
x2

x1 y1 z12 x2 y2 z22
.
x1
x2

2 x1 x2 .2
=4

Mt khc

1
x1 y1

z12

1
x2 y2

z22

( x y z )( x y
2
1

1 1

2 2

z22

( x y )( x y
1 1

z12

2 2

z22

)
.

Nhn theo v hai bt ng thc trn ta c ngay iu phi chng minh.


ng thc xy ra khi v ch khi
=
x1 x=
y=
2 ; y1
2 ; z1 z2 .
Tng qut vi xk > 0, xk yk > zk2 , k =
1, n ta c
n

k =1 k yk

zk2

n3

n
n
n

x
y
z
k

k
k


=
k 1=
k 1 =
k 1

221

www.TaiLieuLuyenThi.com
Khm ph t duy K thut gii bt T Bi ton Max Min ng Thnh Nam

Bi 3. Cho a,b,c,x,y,z l cc s thc dng tho mn iu kin ax + by + cz =


xyz .
Chng minh rng x + y + z > a + b + b + c + c + a .
Li gii
Theo gi thit ta c

a
b
c
+ +
=
1.
yz zx xy

t a= myz , b= nzx, c= pxy m + n + p= 1 .


Bt ng thc tr thnh
x + y + z > z ( my + nx ) + x ( nz + py ) + y ( mz + px ) .

S dng bt ng thc C S cho v phi ta c


z ( my + nx ) + x ( nz + py ) + y ( mz + px )

( x + y + z )( my + mz + nx + nz + px + py )

<

( x + y + z )( m + n + p )( x + y + z ) = x + y + z

Bt ng thc c chng minh.


Nhn xt. Ta c th tm c gi tr nh nht ca tng x + y + z bng
M ( M + a )( M + b )( M + c ) .

Chnh l nghim dng duy nht ca phng trnh


2
1
1
1
.
=
+
+
M a+M b+M c+M
Bi 3. Cho x,y l 2 s thc dng tho mn iu kin x 2 + y 3 x3 + y 4 .
Chng minh rng x3 + y 3 2 .
Li gii
Phn tch tm li gii:
Bng cch s dng C S lm xut hin cc bc x 2 , y 4 tn dng iu kin bi ton.
S dng bt ng thc C S, ta c

(x

) (x
2

+=
y3

x .x x + y 2 . y

x +y

Suy ra x3 + y 3

)( x

+y

+ y4

)( x

+ y2

x 2 + y 3 + x3 + y 2

x 2 + y 3 + x3 + y 2
x3 + y 3 x 2 + y 2 .
2

Vy ta ch cn chng minh x 2 + y 2 2 .
222

) (x

2.

www.TaiLieuLuyenThi.com
Cty TNHH MTV DVVH Khang Vit

Tht vy s dng bt ng thc C S ta c

(x

) (x
2

+=
y2

x. x + y y. y

) ( x3 + y 3 ) ( x + y ) ( x 2 + y 2 ) ( x + y )
2

x2 + y 2 x + y 2 x2 + y 2 x2 + y 2 2

Bt ng thc c chng minh. ng thc xy ra khi v ch khi x= y= 1 .


Bi 5. Cho x,y,z l cc s thc dng tha mn iu kin 4 xy + 2 yz zx =
25
Tm gi tr nh nht ca biu thc=
P

x2 + 4 y 2
z + 4 xy
2

2 2
z + 4 xy .
5

Li gii
p dng bt ng thc C S ta c
=
25

4 xy . 4 xy + z (2 y x)

(4 xy + z 2 )[4 xy + (2 y x)2 ]
=( z 2 + 4 xy )( x 2 + 4 y 2 )
x2 + 4 y 2

V vy P

25
z + 4 xy
2

25
z 2 + 4 xy

2 2
z + 4 xy .
5

p dng BT AM-GM ta c:
25
z + 4 xy
2

2 2
25
1 2
1 2
+
z + 4 xy= 2
z + 4 xy +
z + 4 xy
5
5
z + 4 xy 5
33

25

1 2
1 2
z + 4 xy .
z + 4 xy =
3
5
z + 4 xy 5
2

Vi=
x 3,=
y 2,=
z 1 th P bng 3.
Vy gi tr nh nht ca P bng 3.
2
Bi 4. Cho x,y,z l cc s thc tho mn iu kin x + y + =
z 0; x 2 + y 2 + z=
6.

Tm gi tr ln nht ca biu thc P =( x y )( y z )( z x ) .


Li gii
Khng mt tnh tng qut gi s x y z .
Khi P =
( x y )( y z )( x z ) .
S dng bt ng thc C S ta c
223

www.TaiLieuLuyenThi.com
Khm ph t duy K thut gii bt T Bi ton Max Min ng Thnh Nam

x z 2 x2 + z 2 2 x2 + y 2 + z 2 =
2 3.

S dng bt ng thc AM GM ta c
x y+ yz
P

( x z )=

ng thc xy ra khi v ch khi x =

1
( x z )3 6 3 .
4

3, y = 0, z = 3 hoc cc hon v.

Bi 5. Cho x,y,z l cc s thc tho mn iu kin x 2 + y 2 + z 2 =


2.
Chng minh rng x + y + z 2 + xyz .
Li gii
Phn tch tm li gii:
Bt ng thc vit li di dng x (1 yz ) + y + z 2 .
Ta nh gi thng qua tng x 2 + y 2 + z 2 v vy p dng C S cho 2 b s

( x; y; z ) v (1 yz;1;1)

hoc ( x; y + z ) ; (1 yz;1) .

Ta la chn 2 b s th 2 v khi ta c mt biu thc tch ca yz .


Li gii chi tit:
S dng bt ng thc C S ta c
x (1 yz ) + y + z

(x

+ ( y + z)

) ((1 yz )

= ( 2 + 2 yz ) 2 + y 2 z 2 2 yz

( 2 + 2 yz ) ( 2 + y 2 z 2 2 yz ) 2

Vy ta ch cn chng minh

+ 12

(1 + yz ) 2 + y 2 z 2 2 yz 2 y 3 z 3 y 2 z 2 .

Bt ng thc cui lun ng v


2 = x 2 + y 2 + z 2 y 2 + z 2 2 yz yz 1 y 3 z 3 y 2 z 2 .

Bt ng thc c chng minh ng thc xy ra chng hn 1 s bng 0 v 2 s


bng 1.
Bi tp tng t
Cho x,y,z l cc s thc khng m tho mn iu kin x 2 + y 2 + z 2 =
1.
Chng minh rng x + y + z 2 +

224

9
xyz .
4

www.TaiLieuLuyenThi.com
Cty TNHH MTV DVVH Khang Vit

Bi 6. Cho x,y,z l cc s thc tho mn iu kin x 2 + y 2 + z 2 =


1 . Chng minh
rng xy + yz + 2 zx

3 +1
.
2

Li gii
Ch vai tr i xng ca x v z v vy s dng bt ng thc C S ta c

y ( x + z ) 2 y 2 x2 + y 2=
z2

2 y2 1 y2

).

2 zx x 2 + z 2 =
1 y2

Cng theo v hai bt ng thc a v chng minh

3 +1
.
2
n y c th a v kho st hm s hoc s dng trc tip C S nh sau
1
1
1
2 y 2 1 y 2 +=
1 y2
4 y2 4 y4 + 1 2 y2 +
2
2
2
.
1 1 2
4
2 2 1 1+ 3
+ 4y 4y + 1 2y
+ =
2
2
2 4
2 y2 1 y2 + 1 y2

1
1
1
1
ng thc xy ra khi v ch khi x =
.
z=
1
,y=

+
2
2
3
2 3

Bi 5. Chng minh rng nu phng trnh x 4 + bx3 + cx 2 + bx + 1 =


0 c nghim th
b2 + ( c 2 ) > 3 .
2

Li gii
Nu x0 l nghim ca phng trnh r rng x0 0 vit li phng trnh di dng
x02

1
1
1
+ 2 + b x0 + + c =
0 x0 + + b x0 + + c 2 =
0.
x0
x0
x0
x0

t X =
x0 +

1
, X 2 phng trnh tr thnh
x0

X 2 + bX + c 2 = 0 ( bX + c 2 ) = X 4 .
2

S dng bt ng thc C S ta c

( bX + c 2 )2 b2 + ( c 2 )2 ( X 2 + 1) .
X4

1
= X 2 1+ 2
> X 2 1 3 .
X +1
X +1
Bt ng thc c chng minh.

Suy ra b 2 + ( c 2 )
2

225

www.TaiLieuLuyenThi.com
Khm ph t duy K thut gii bt T Bi ton Max Min ng Thnh Nam

Bi tp tng t
Ch phng trnh x 4 + ax3 + bx 2 + 2ax + 4 =
0.
16
Tm nghim ca phng trnh bit a 2 + b 2 = .
9
Bi 3. Cho x,y,z l cc s thc tho mn iu kin x, y, z 1 v
Chng minh rng

1 1 1
+ + =
2.
x y z

x + y + z x 1 + y 1 + z 1 .

Li gii
Vit li iu kin gi thit di dng
1
1
1
1 = 1 +1 +1
x
y
z
x 1 y 1 z 1
1

+
+
=
x
y
x
S dng bt ng thc C S ta c
x 1 y 1 z 1
x + y + z = ( x + y + z)
+
+

y
z
x
2

x 1
y 1
z 1
x .
+ y.
+ z.

x
y
z .

x 1 + y 1 + z 1

x + y + z x 1 + y 1 + z 1

Bt ng thc c chng minh. ng thc xy ra khi v ch khi x= y= z=

3
.
2

Bi 4. Cho x,y,z l cc s thc thuc on [ 1;1] v tho mn iu kin


x + y + z + xyz =
0.

Chng minh rng

x +1 + y +1 + z +1 3 .

Li gii
+ Nu x + y + z 0 ta c ngay iu phi chng minh v
x +1 + y +1 + z +1
=

(1 + 1 + 1)( x + 1 + y + 1 + z + 1)
3( x + y + z ) + 9 3

+ Ta xt vi x + y + z > 0 xyz =
( x + y + z ) < 0 do phi tn ti t nht 1 s
m v 1 s dng.
226

www.TaiLieuLuyenThi.com
Cty TNHH MTV DVVH Khang Vit

xy > 0
Khng mt tnh tng qut gi s z < 0
x, y > 0 .
x + y > z > 0
Khi vai tr ca x v y nh nhau nn ta p dng C S cho 2 cn thc u tin
ta c

x + 1 + y + 1 + z + 1 2 ( x + 1 + y + 1) + z + 1
2( x + y) + 4 + z +1

Bt ng thc c chng minh nu ta chng minh c


2( x + y) + 4 + z +1 3
2( x + y) + 4 2 1 z +1
2( x + y)

2( x + y) + 4 + 2

Ch x + y = z ( xy + 1) ;1 + z =
Vy ta i chng minh

z
1+ z +1

(1 x )(1 y ) .
x+ y
+1 =
xy + 1
1 + xy

2 z ( xy + 1)

2( x + y) + 4 + 2

xy +

1+

(1 x )(1 y )
1 + xy

(1 x )(1 y )(1 + xy ) 1 +

1+

x+ y
2

Bt ng thc lun ng v
xy +

xy )
(1 x )(1 y )(1 + =

x . xy 2 +

(1 x ). (1 y )(1 + xy )

( x + 1 x ) ( xy 2 + (1 y )(1 + xy ) )

1 + xy y =

1 + y ( x 1) 1 1 + 1 +

x+ y
2

Bt ng thc c chng minh. ng thc xy ra ti x= y= z= 0 .


Bi 6. Cho a,b,c l cc s thc dng. Chng minh rng
5
2
a 2 + 1 b 2 + 1 c 2 + 1 ( a + b + c + 1) .
16
Li gii

)(

)(

D on du bng t ti a= b= c thay vo bt ng thc ta tm c a= b= c=

1
.
2

S dng bt ng thc C S ta c
227

www.TaiLieuLuyenThi.com
Khm ph t duy K thut gii bt T Bi ton Max Min ng Thnh Nam
2

5
5
5 2
b + c +1
2
2
1)
a + 1 1 + ( b + c + 1) .
( a + b + c +=
a.1 + 1.

16
16
1
16

)(

Vy ta ch cn chng minh b 2 + 1 c 2 + 1

5
2
1 + ( b + c + 1)

16

16b 2 c 2 + 11b 2 + 11c 2 + 6 10bc + 10b + 10c .


Bt ng thc trn l tng ca cc bt ng thc sau
16b 2 c 2 + 1 8bc
b 2 + c 2 2bc
1

10 b 2 + 10b
4

10 c 2 + 10c
4

Bt ng thc c chng minh. ng thc xy ra khi v ch khi a= b= c=

1
.
2

Cch 2: Ch . Vi mi s thc a1 , a2 ,..., an > 1 v cng du ta lun c

(1 + a1 )(1 + a2 ) ...(1 + an ) 1 + a1 + a2 + ... + an .


1
1
1
Trong 3 s a 2 , b 2 , c 2 c t nht 2 s cng du khng mt tnh tng
4
4
4
1
1
qut gi s a 2 ; b 2 cng du ta c:
4
4

( a + 1)(b + 1)=
2

2 1 5 2 1 5
a 4 + 4 . b 4 + 4

25 4
1 4
1
= 1 + a 2 . 1 + b 2
16 5
4 5
4

25 4 2 1 2 4 2 1
1 + a a + b
16 5
4
5
4

Bt ng thc c chng minh nu ta chng minh c:

25 4 2 1 4 2 1 2
5
2
1 + a + b c + 1 ( a + b + c + 1) .

16 5
4 5
4
16

4
1 4
1
2
5 1 + a 2 + b 2 c 2 + 1 ( a + b + c + 1) .
4 5
4
5

)(

4a 2 + 4b 2 + 3 c 2 + 1 ( a + b + c + 1) .

228

www.TaiLieuLuyenThi.com
Cty TNHH MTV DVVH Khang Vit

Theo C-S ta c

( 4a

)(

) ( 4a

+ 4b 2 + 3 c 2 +=
1

1 1
1 1
+ 4b 2 + 1 + 1 + 1 + + c 2 + +
4 4
4 4
2

1
1
1
1

2a. + 2b. + 1.c + 1. + 1. =


2
2
2
2

( a + b + c + 1)2

Bt ng thc c chng minh. ng thc xy ra khi v ch khi a= b= c=

1
.
2

Bi tp tng t
Cho a,b,c l cc s thc dng chng minh

(a

)(

)(

+ 3 b 2 + 3 c 2 + 3 4 ( a + b + c + 1) .
2

Bi 10. Cho a,b,c l cc s thc dng tho mn iu kin a + b + c + 1 =


4abc .
1
1
1
3
.
Chng minh rng 4
+ 4
+ 4

a +b+c b +c+a c +a+b a+b+c


Li gii
S dng bt ng thc C S ta c

(a

Tng t ta c

)(

) (

+ b + c 1 + b3 + c 3 a 2 + b 2 + c 2
1

a4 + b + c
1

b4 + c + a

1 + b3 + c 3

(a

+ b2 + c2

1 + c3 + a 3

a 2 + b2 + c2

;
2

1
c4 + a + b

3 + 2 a 3 + b3 + c 3

(a

+ b2 + c

Cng theo v ba bt ng thc trn ta c


P

2 2

1 + a 4 + b4

a 2 + b2 + c2

).

Ta ch cn chng minh

3 + 2 a 3 + b3 + c 3

(a

+ b2 + c

2 2

3
a+b+c

) (

a 4 + b 4 + c 4 + 6 a 2b 2 + b 2 c 2 + c 2 a 2 2 a3b + b3a + b3c + c3b + c3a + a3c + 3 ( a + b + c )

) (

a 2 + b 2 + c 2 ab bc ca + 3 a 2b 2 + b 2 c 2 + c 2 a 2 a b c 0

.
229

www.TaiLieuLuyenThi.com
Khm ph t duy K thut gii bt T Bi ton Max Min ng Thnh Nam

Ch

a 2b 2 + b 2 c 2 + c 2 a 2 3a 2b 2 c 2 a 2 + b 2 + c 2 abc ( a + b + c ) a + b + c

bi v 4abc = 1 + a + b + c 1 + 3 3 abc abc 1 .


Bt ng thc c chng minh. ng thc xy ra khi v ch khi a= b= c= 1 .
Bi tp tng t
Cho a, b, c l cc s thc dng tho mn iu kin
1
1
1
+
+
1.
a + b +1 b + c +1 c + a +1
Chng minh rng a + b + c ab + bc + ca .
Bi 11. Cho a, b, c l cc s thc dng. Chng minh

( a + b + c )

1 1 1
+ + 1+ 3 5 +
a b c

(a

1
1
1
+ b3 + c 3 3 + 3 + 3 .
b
c
a

Li gii
Ta c

31

1 1
( a + b + c ) + +
a b c

3 a + b )( b + c )( c + a )

a3 + b3 + c3 + 3 ( a + b )( b + c )( c + a ) 1 + 1 + 1 + (

a 3 b3 c 3
a 2b 2 c 2

(a

1
1 3 ( a + b )( b + c )( c + a )
1
+ b3 + c 3 3 + 3 + 3 +
abc
b
c
a

Ch

( a + b )( b + c )( c + a ) = ( a + b + c )( ab + bc + ca ) 1 =
abc

abc

( a + b + c )

1 1 1
+ + 1
a b c

Do
1 1 1
1 1 1
3( a + b + c ) + + 3 3( a + b + c ) + + 3
a b c
a b c
1 1 1
V ( a + b + c ) + + 9 .
a b c

230

( a + b + c )

1 1 1
+ + +6
a b c

www.TaiLieuLuyenThi.com
Cty TNHH MTV DVVH Khang Vit

T suy ra

1 1 1
( a + b + c ) + + 1 5 +
a b c

(a

( a + b + c )

1 1 1
+ + 1+ 3 5 +
a b c

(a

1
1
1
+ b3 + c 3 3 + 3 + 3
b
c
a
3

1
1
1
+ b3 + c 3 3 + 3 + 3
b
c
a

Bt ng thc c chng minh. ng thc xy ra khi v ch khi a= b= c .


Bi tp tng t
Cho a,b,c l cc s thc dng chng minh P =

(a

1
1
1
+ b2 + c2 2 + 2 + 2 .
b
c
a

C. BI TP RN LUYN

(1 + a )(1 + b )

1
( a + b + ab 1) .
2
Bi 2. Cho x,y,z l cc s thc tho mn iu kin 1 x y z 4 .

Bi 1. Cho a,b l cc s thc chng minh

4
2 y
z
Tm gi tr nh nht ca biu thc P =( x 1) + 1 + 1 + 1 .
x y z
Bi 2. S dng bt ng thc C S ta c
2

1
y
z
4
P x 1 + 1 + 1 + 1 .
4
x
y
z

Ch s dng bt ng thc AM GM ta c
x+

2
y z 4
y z 4
1
+ + 4 4 x. . . = 4 2 P 4 2 4 = 4
x y z
x y z
4

ng thc xy ra khi v ch khi x =

y z 4
= = x=
x y z

2 1 .

2, y = 2, z = 2 2 .

Bi 2. Chng minh rng vi mi x,y,z l cc s thc dng ta c


x
y
z
9
.
+
+

2
2
2
( y + z ) ( z + x) ( x + y) 4( x + y + z )
Bi 3. Cho x,y,z l cc s thc khng m.
Chng minh rng

x2 + 1 + y 2 + 1 + z 2 + 1 6 ( x + y + z ) .

Bi 4. Chng minh rng vi mi x,y,z l cc s thc dng ta c


x3
y2

y3
z2

z3
x2

x2 y 2 z 2
.
+
+
y
z
x
231

www.TaiLieuLuyenThi.com
Khm ph t duy K thut gii bt T Bi ton Max Min ng Thnh Nam

Bi 5. Cho cc s thc a,b,c thay i tha mn iu kin a 2 + b 2 + c 2 =


1.
Tm gi tr ln nht, gi tr nh nht ca biu thc P = a3 + b3 + c3 abc .
Bi 6. Cho a,b,c l cc s thc tho mn iu kin a 2 + b 2 + c 2 =
2.
Chng minh rng a3 + b3 + c3 abc 2 2 .
Bi 7. Cho a,b,c l cc s thc tho mn iu kin a 2 + b 2 + c 2 =
9.
Chng minh rng 2 ( a + b + c ) abc 10 .
Bi 8. Cho a,b,c l cc s thc tho mn iu kin x 2 + y 2 + z 2 = k , ( k 0 ) .
Chng minh rng 18k ( x + y + z ) xyz 270k k .
Bi 9. Cho x,y,z l cc s thc khng m tho mn iu kin x 2 + y 2 + z 2 =
1.
9
xyz .
4
Bi 10. Chng minh rng vi mi a,b,c l cc s thc dng ta c

Chng minh rng x + y + z 2 +

4(a + b + c)

b+c
c+a
a+b
+
+

a
b
c

( a + b )( b + c )( c + a )

Bi 11. Chng minh vi mi a,b,c l cc s thc dng ta c

a 2 + b2

2 a 2 + b2 + c ( a + b )

b2 + c2

2 b2 + c2 + a ( b + c )

Bi 12. Cho a,b,c l cc s thc dng. Chng minh

c2 + a2

2 c2 + a2 + b ( c + a )

2a
2b
2c
+
+
3
a+b
b+c
c+a

Bi 13. Cho a,b,c l cc s thc dng. Chng minh


a2
b + (c + a)
2

b2
c + (a + b)
2

c2
a + (b + c )
2

3
.
5

Bi 14. Cho a,b,c l cc s thc dng chng minh

(a

)(

)(

+ 3 b 2 + 3 c 2 + 3 4 ( a + b + c + 1) .
2

Bi 15. Cho a,b,c l cc s thc dng tha mn iu kin


1
1
1
+
+
1.
a + b +1 b + c +1 c + a +1
Chng minh rng a + b + c ab + bc + ca .
Bi 16. Cho a,b,c l cc s thc dng c tch bng 1. Chng minh

232

1.

www.TaiLieuLuyenThi.com
Cty TNHH MTV DVVH Khang Vit

a5 a 2

b5 b 2

c5 c 2

a 5 + b 2 + c 2 b5 + c 2 + a 2 c 5 + a 2 + b 2
Bi 17. Chng minh rng vi mi s thc a,b,c ta c

0.

2(1 + abc) + 2(1 + a 2 )(1 + b 2 )(1 + c 2 ) (1 + a )(1 + b)(1 + c) .

Bi 18. Cho x,y,z,t l cc s thc khng m tho mn iu kin


| x y | + | y z | + | z t | + | t x |=
4.
Chng minh rng x 2 + y 2 + z 2 + t 2 2 .
Bi 19. Cho a,b,c l cc s thc dng tho mn iu kin a 2 + b 2 + c 2 =
3.
Chng minh rng
5a 2

5b 2

5c 2

5a 2 + 4bc + 2 bc
5b 2 + 4ca + 2 ca
5c 2 + 4ab + 2 ab
Bi 20. Cho a, b, c khng m tho mn iu kin a + b + c =
1.

3.

Tm gi tr nh nht ca biu thc P = a + b + b + c + c + a .


Bi 21. Cho a,b,c l di 3 cnh mt tam gic.
a +bc + b+ca + c+ a b a + b + c .

Chng minh rng

Bi 22. Cho a,b,c l cc s thc khng m tho mn iu kin a 2 + b 2 + c 2 =


8.
Chng minh rng 4 ( a + b + c ) abc 16 .
Bi 23. Cho x,y,z l cc s thc tho mn iu kin x 2 + y 2 + z 2 =
1.
Tm gi tr ln nht v gi tr nh nht ca biu thc
P = x3 + y 3 + z 3 3 xyz .

Bi 24. Chng minh rng vi mi a,b,c l di 3 cnh mt tam gic ta c


a 2b ( a b ) + b 2 c ( b c ) + c 2 a ( c a ) 0 .

Bi 25. Cho x,y,z l cc s thc dng chng minh


1 + yz + zx 1 + xy + zx 1 + xy + yz
+
+
1.
(1 + x + y )2 (1 + y + z )2 (1 + z + x )2
Bi 26. Cho a,b,c l cc s thc dng tho mn iu kin
1 1 1
( a + b c ) + =4 .
a b c

1
1
1
Chng minh rng a 4 + b 4 + c 4 4 + 4 + 4 2304 .
b
c
a
Bi 27. Cho a,b,c l cc s thc dng tho mn iu kin
233

www.TaiLieuLuyenThi.com
Khm ph t duy K thut gii bt T Bi ton Max Min ng Thnh Nam

( a + b c )

1 1 1
+ =
4.
a b c

Tm gi tr nh nht ca biu thc P =

(a

1
1
1
+ b3 + c 3 3 + 3 + 3 .
b
c
a

D. HNG DN GII P S
Bi 1. Ta c

(1 + a )(1 + b ) =( a + b ) + ( ab 1) 12 ( a + b + ab 1)
1
(1 + a )(1 + b )
( a + b + ab 1)
2
2

iu phi chng minh.


Bi 2. S dng bt ng thc C S ta c

( x + y + z)

( y + z )2

Mt khc

( z + x )2

2
x
y
z

+
+

.
( x + y )2 y + z z + x x + y

1
x
y
z
1
1
+
+
=
+
+
( x + y + z )
3
y+z z+x x+ y
y+z z+x x+ y

1
1
1
1
= ( x + y ) + ( y + z ) + ( z + x )
+
+
3
2
y+z z+x x+ y
1
3
(1 + 1 + 1)2 3 =
2
2
Ta c iu phi chng minh. ng thc xy ra khi v ch khi x= y= z .

Bi 3. S dng h qu bt ng thc C S (Mincopski) ta c


x2 + 1 + y 2 + 1 + z 2 + 1

( x + y + z )2 + (1 + 1 + 1)2 .

Vy ta cn chng minh ( x + y + z ) + 9 6 ( x + y + z ) ( x + y + z 3) 0
2

Bt ng thc lun ng. ng thc xy ra khi v ch khi x= y= z= 1 .


Bi 4. S dng bt ng thc C S ta c
2

x3 y 3 z 3
x2 y 2 z 2
+
2 + 2 + 2 ( x + y + z ) +
z
x
z
x
y
y
x2 y 2 z 2
2
+ ( y + z + x) ( x + y + z )
+

y
z
x

Nhn theo v 2 bt ng thc trn ta c iu phi chng minh. ng thc xy ra


khi v ch khi x= y= z .
234

www.TaiLieuLuyenThi.com
Cty TNHH MTV DVVH Khang Vit

x k +1

Tng qut ta c

yk

y k +1
zk

z k +1
xk

xk
y k 1

yk

z k 1

zk
x k 1

,k 1.

Bi 5. S dng bt ng thc C-S ta c:

P 2 = a3 + b3 + c c 2 ab = a.a 2 + b.b 2 + c c 2 ab

a 2 + b 2 + c 2 a 4 + b 4 + c 2 ab

= a 4 + b 4 + c 4 2abc 2 + a 2b 2

a 4 + b 4 + c 4 + a 2b 2 + c 2 a 2 + b 2

) (a

a 4 + b 4 + c 4 + 2 a 2b 2 + b 2 c 2 + c 2 a 2 =

+ b2 + c2

= 1

Vi a= 1, b= c= 0 th P bng 1; vi a =
1, b =
c=
0 th P bng -1.
Vy gi tr ln nht ca P bng 1 t ti a= 1, b= c= 0 hoc cc hon v.
Gi tr nh nht ca P bng 1 t ti a =
1, b =
c=
0 hoc cc hon v.
Bi 6. Gi P l biu thc v tri.
S dng bt ng thc C-S ta c

P 2 = a3 + b3 + c c 2 ab = a.a 2 + b.b 2 + c c 2 ab

a 2 + b 2 + c 2 a 4 + b 4 + c 2 ab

= 2 a 4 + b 4 + c 4 2abc 2 + a 2b 2

2 a 4 + b 4 + c 4 + a 2b 2 + c 2 a 2 + b 2

4
4
4
2
2
2
2
2
2 a + b + c + 2 a b + b c + c a2

=2 a 2 + b 2 + c 2

=8 P 2 2

Bt ng thc c chng minh. ng thc xy ra khi a =


2, b =
c=
0 hoc
cc hon v.
Bi tp tng t
Cho x,y,z l cc s thc khng m tho mn iu kin x 2 + y 2 + z 2 =
3.
Chng minh rng x3 + y 3 + z 3 xyz 3 3 .
Bi 7. S dng bt ng thc C S ta c

235

www.TaiLieuLuyenThi.com
Khm ph t duy K thut gii bt T Bi ton Max Min ng Thnh Nam
2
2
a ( 2 bc ) + 2 ( b + c ) a 2 + ( b + c ) ( 2 bc ) + 22

( 2bc + 9 ) ( b2c 2 4bc + 8)

Vy ta cn chng minh

( 2bc + 9 ) ( b2c 2 4bc + 8) 100 ( bc + 2 )2 ( 2bc 7 ) 0 .

Bt ng thc cui ng nu ta gi s a 2 = max a 2 , b 2 , c 2 .


Tht vy ta c 2bc b 2 + c 2 = 9 a 2 9 3 = 6 2bc 7 1 .
Bt ng thc c chng minh. ng thc xy ra chng hn ti a =
1, b =
c=
2.
Bi 9. Khng mt tnh tng qu=
gi s x max { x, y, z} x

1
.
3

S dng bt ng thc C S ta c
9
x 1 yz + y + z
4
=

(x

+ ( y + z)

9 2
1 yz + 1
4

( 2 yz + 1)

81 2 2 9

y z yz + 2
2
16

9
81

Ta cn chng minh ( 2 yz + 1) y 2 z 2 yz + 2 2
2
16

yz 162 y 2 z 2 63 yz 8 0
y 2 + z 2 1 x2 1
=
.
2
2
3
Bt ng thc c chng minh. ng thc xy ra khi v ch khi
1
x= y=
, z= 0 hoc cc hon v.
2
Bi 10. Bt ng thc cn chng minh tng ng vi
b+c
a ( a + b )( a + c ) 4 ( a + b + c ) .
cyc

Bt ng thc lun ng v yz

S dng bt ng thc C S ta c

236

( a + b )( a + c ) a +

bc

( b + c )( b + a ) b +

ca

( c + a )( c + b ) c +

ab

www.TaiLieuLuyenThi.com
Cty TNHH MTV DVVH Khang Vit

p dng v ta cn chng minh


b+c
c+a
a+b
bc +
ca +
ab 2 ( a + b + c ) .
a
b
c
Khng mt tnh tng qut ta gi s
bc
ca
ab
.

a
b
c
S dng bt ng thc Chebyshev ta c
a b c b + c c + a a + b;

bc
b+c
c+a
a+b
1
ca
ab
+
+
bc +
ca +
ab ( b + c + c + a + a + b )

a
b
c
3
b
c
a
bc ca ab
.
.
= 2(a + b + c)
a
b
c
Bt ng thc c chng minh. ng thc xy ra khi v ch khi a= b= c .
Bi 11. S dng bt ng thc C S ta c
2 ( a + b + c ) .3

a 2 + b2

2 a +b
2

Tng t ta c

1
1
a+b
.
=

=
+
+
c
a
b
c
a
b
) 2+ (
) 2(a + b + c)
+ c (a + b) 2 + (
1
a 2 + b2
( a + b )2
2

b2 + c2

2 b2 + c2 + a ( b + c )

c2 + a2

2 c2 + a2 + b ( c + a )

b+c
.
2(a + b + c)

c+a
2(a + b + c)

Cng theo v 3 bt ng thc trn ta c iu phi chng minh.


ng thc xy ra khi v ch khi a= b= c .
Bi 12. Gi P l biu thc v tri v s dng bt ng thc C S ta c
P=

2a ( a + c )

( a + b )( a + c )

2b ( b + a )

( b + c )( b + a )

2c ( c + b )

( c + a )( c + b )

2a
( a + c )

( a + b )( a + c )

Ta ch cn chng minh

8 ( a + b + c )( ab + bc + ca )

( a + b )( b + c )( c + a )

8 ( a + b + c )( ab + bc + ca )

( a + b )( b + c )( c + a )

237

www.TaiLieuLuyenThi.com
Khm ph t duy K thut gii bt T Bi ton Max Min ng Thnh Nam

8
( a + b + c )( ab + bc + ca ) ( a + b )( b + c )( c + a ) 8abc
9
Bt ng thc cui lun ng theo AM GM.
Bi 13. S dng bt ng thc C S ta c
( a + b )( b + c )( c + a )

(1 + 4 ) b2 + ( c + a )2 b + 2 ( c + a )

a2

b + (c + a)
2

a
.
5 ( b + 2 a + 2c )

Tng t cho 2 cn thc cn li bi ton a v chng minh


3
a
b
c
+
+

b + 2a + 2c c + 2b + 2a a + 2b + 2c 5
.
9
b + 2c
c + 2a
a + 2b

+
+

b + 2a + 2c c + 2b + 2a a + 2b + 2c 5
S dng bt ng thc C S ta c
b + 2c
c + 2a
a + 2b
+
+

b + 2a + 2c c + 2b + 2a a + 2b + 2c
2
.
2
( b + 2c ) )
9(a + b + c)
(
9

= =
( b + 2c )( b + 2a + 2c ) 5 ( a + b + c )2 5
Bt ng thc c chng minh. ng thc xy ra khi v ch khi a= b= c .
Bi 14. D on ng thc t ti khi 3 s bng nhau thay vo bt ng thc ta c
a= b= c= 1 .
Ta s dng dng bt ng thc C S
2

b + c +1

2
4 ( a + b + c + 1=
)2 4 a.1 + 3.
4 a +3
3

( b + c + 1)2
1 +
.

( b + c + 1)2
b2 + 3 c2 + 3
4 1 +

Vy ta chng minh

)(

3b 2 c 2 + 5b 2 + 5c 2 + 11 8bc + 8b + 8c
Bt ng thc cui l tng ca 3 bt ng thc sau

3 b 2 c 2 + 1 6bc
b 2 + c 2 2bc

4 b 2 + c 2 + 2 4 ( 2b + 2c ) = 8b + 8c

Bt ng thc c chng minh. ng thc xy ra khi v ch khi a= b= c= 1 .


Cch 2: Xem bi tp mu.
Bi 15. S dng bt ng thc C S ta c
238

www.TaiLieuLuyenThi.com
Cty TNHH MTV DVVH Khang Vit

( a + b + 1) ( a + b + c 2 ) ( a + b + c )2
Tng t ta c

1
a + b + c2
.

a + b + 1 ( a + b + c )2

1
b + c + a2
1
c + a + b2
.

b + c + 1 ( a + b + c )2 c + a + 1 ( a + b + c )2

Cng theo v ba bt ng thc trn v kt hp vi iu kin ta c:


a 2 + b2 + c2 + 2 ( a + b + c )

( a + b + c )2

1 a + b + c ab + bc + ca .

Bt ng thc c chng minh. ng thc xy ra khi v ch khi a= b= c= 1 .


Bi 16. Bt ng thc tng ng vi:
1
1
1
3
.
+ 5
+ 5
2
5
2
2
2
2
2
2
a +b +c
b +c +a
c +a +b
a + b2 + c2
S dng bt ng thc C S ta c

a5 + b2 + c 2 + b2 + c 2 a 2 + b2 + c 2
a

1
a5 + b2 + c 2

1
+ b2 + c2
a
a 2 + b2 + c2

Tng t ta c
1
b5 + c 2 + a 2

1 2
+ c + a2
b
a 2 + b2 + c2

1
c5 + a 2 + b 2

1
+ a 2 + b2
c
a 2 + b2 + c2

Cng theo v ba bt ng thc trn v a v chng minh

) (

1 1 1
+ + + 2 a 2 + b2 + c2 3 a 2 + b2 + c2
.
a b c
a 2 + b 2 + c 2 ab + bc + ca
Bt ng thc cui ng vy ta c pcm. ng thc xy ra khi v ch khi
a= b= c= 1 .
Bi 17. Bt ng thc tng ng vi:

2(1 + a 2 )(1 + b 2 )(1 + c 2 ) a + b + c + ab + bc + ca abc 1 .

)(

Ch 2(1 + a 2 )(1 + b 2 )(1 + c 2 ) = (a + b) 2 + (1 ab) 2 (1 + c) 2 + (c 1) 2 .


S dng bt ng thc C S ta c

239

www.TaiLieuLuyenThi.com
Khm ph t duy K thut gii bt T Bi ton Max Min ng Thnh Nam

2(1 + a 2 )(1 + b 2 )(1 + c 2 )=

[(a + b)2 + (1 ab)2 ][(1 + c)2 + (c 1)2 ]

( a + b )(1 + c ) + ( c 1)(1 ab )
= a + b + c + ab + bc + ca abc 1
Bt ng thc c chng minh.
Bi 18. S dng bt ng thc C S ta c
x2 + y 2 + z 2 =
+ t2

x2 + y 2 y 2 + z 2 z 2 + t 2 t 2 + x2
+
+
+
2
2
2
2
( x y ) 2 ( y z ) 2 ( z t ) 2 (t x) 2
+
+
+
2
2
2
2

(| x y | + | y z | + | z t | + | t x |)2
8

2
=

Bt ng thc c chng minh.


Du bng xy ra chng hn ti x = z = 1, y = t = 0 .

Bi 19. S dng bt ng thc C S ta c


5a 2 + 4bc
=
+ 2 bc

3.

5a 2 + 4bc
+ 2. 2bc
3
5a 2 + 4bc

( 3 + 2 )

5
5a 2 + 5 b 2 + c 2
3

5a 2

+ 2bc=

5
5a 2 + 10bc
3

a +b +c
5
)) =
3
2

a2

a + b2 + c2
5a 2 + 4bc + 2 bc
Tng t ri cng li theo v ta c pcm.
Bt ng thc c chng minh. ng thc xy ra khi v ch khi a= b= c .

Bi 20. Ta c P 2= 2 ( a + b + c ) + 2

( a + b )( a + c ) .

Ch s dng bt ng thc C S ta c

( a + b )( a + c ) ( a +

Do P 2 4 ( a + b + c ) + 2

bc = a + b + c + ab + bc + ca .

ab + bc + ca 4 P 2 .

Du bng t ti a= 1, b= c= 0 hoc cc hon v.


Vy gi tr nh nht ca P bng 2.
240

www.TaiLieuLuyenThi.com
Cty TNHH MTV DVVH Khang Vit

Bi 21. S dng bt ng thc C S ta c


a + b c + b + c a 2(a + b c + b + c a) =
2 b
b + c a + c + a b 2 (b + c a + c + a b) =
2 c
c + a b + a + b c 2(c + a b + a + b c) 2 a

Cng theo v 3 bt ng thc trn ta c iu phi chng minh.


ng thc xy ra khi v ch khi a= b= c .
Bi 22. Khng mt tnh tng qut =
gi s a max {a, b, c} a

8
.
3

S dng bt ng thc C S ta c
a ( 4 bc ) + 4 ( b + c )
=

(a

+ (b + c )

) (( 4 bc )

+ 42

(8 + 2bc ) ( b2c 2 8bc + 32 )

Ta chng minh ( 8 + 2bc ) b 2 c 2 8bc + 32 162 2 ( bc 4 ) b 2 c 2 0 .


b2 + c2 8 a 2
Bt ng thc lun ng v bc
=
2
2

8
8
3=
< 4.
2
3

Bt ng thc c chng minh.


ng thc xy ra khi v ch khi a= b= 2, c= 0 hoc cc hon v.
Bi 23. Ta c xy + yz + zx x 2 + y 2 + z 2 =
1.

Khi P = ( x + y + z ) x 2 + y 2 + z 2 xy yz zx

= ( x + y + z )(1 xy yz zx )
P 2 =(1 + 2 ( xy + yz + zx ) ) (1 xy yz zx )

= 2 ( xy + yz + zx ) 3 ( xy + yz + zx ) + 1
3

( xy + yz + zx )2 ( 2 ( xy + yz + zx ) 3) + 1 1

Vy gi tr ln nht ca P bng 1 t ti x= 1, y= z= 0 hoc cc hon v.


Gi tr nh nht ca P bng -1 t ti x =
1, y =
z=
0 hoc cc hon v.
Bi 24. t a =+
y z , b =+
z x, c =+
x y, x, y, z > 0 bt ng thc tr thnh

241

www.TaiLieuLuyenThi.com
Khm ph t duy K thut gii bt T Bi ton Max Min ng Thnh Nam

x3 z + y 3 x + z 3 y xyz ( x + y + z )

x2 y 2 z 2
+
+
x+ y+z
y
z
x

x2 y 2 z 2
2
+
+ ( y + z + x) ( x + y + z ) .
y

z
x

Lun ng theo C S.
Bt ng thc c chng minh. ng thc xy ra khi v ch khi a= b= c .
Bi 25. S dng bt ng thc C S ta c
x+ y
2

1 + z ( x + y ) 1 +
(1 + x + y )
z

z
1 + yz + zx
1

=
2
x
y
+
x+ y+z
(1 + x + y ) 1 +
z
Tng t cho hai biu thc cn li ri cng li theo v ta c pcm. ng thc
xy ra khi v ch khi x= y= z= 1 .

Bi 26. Theo gi thit kt hp s dng bt ng thc AM GM ta c

( a + b c )

1 1

+ c=
a b

1 1
1 1
+ + 1 c ( a + b ) + c +
a b
a b

( a + b )

1 1
(a + b) + + 1 2
a b

( a + b )

1 1
+
a b

1 1
( a + b ) + 1
a b

( a + b )

1 1
a b
a b
+ 1 2
+ +2 3 + 7.
b a
b a
a b

Suy ra

Khi s dng bt ng thc C S ta c

1
1
1
a +b +c 4 + 4 + 4
b
c
a
4

1
1
a + b 4 + 4 + 1
b
a
4

a b 2
a 2 b2

= 2 + 2 + 1 = + 1 7 2 1
b

a
b a

= 2304

Bt ng thc c chng minh. ng thc xy ra khi v ch khi ab = c 2 v


a b
+ =
7.
b a
242

www.TaiLieuLuyenThi.com
Cty TNHH MTV DVVH Khang Vit

K THUT S DNG BT NG THC


CAUCHY SCHWARZ DNG PHN THC

CH 5:

A. NI DUNG PHNG PHP


B. BI TP MU
m u ta xt bi ton quen thuc sau y
Bi 1. Chng minh rng vi mi s thc dng a,b,c ta c
a
b
c
3
+
+
.
b+c c+a a+b 2
Li gii
y l bt ng thc Nebits ni ting c rt nhiu li gii dnh cho n. Di
y ti cp 2 phng php tip cn cng chnh l k thut chnh chng ta cn
p dng trong ch ny.
Cch 1: Cng thm vo mi hng t v tri vi 1 ta c
a
b
c
a
b
c

+
+ =
+ 1 +
+ 1 +
+ 1 3
b+c c+a a+b b+c c+a a+b
1
1
1
= (a + b + c)
+
+
3
b+c c+a c+a
1
1
1
1
= ( a + b ) + ( b + c ) + ( c + a )
+
+
3
2
a+b b+c c+a
2

1
1
1
1
3
+ b + c.
+ c + a.
a + b.
3=
2
2
a+b
b+c
c+a

Bt ng thc c chng minh. ng thc xy ra khi v ch khi a= b= c= 1 .


Cch 2: Nhn thm vo mi hng t v tri tng ng vi a,b,c xut i lng
bnh phng ta c:
a
b
c
a2
b2
c2
+
+
=
+
+
b + c c + a a + b a (b + c ) b (c + a ) c (c + a )

( a + b + c )2
( a + b + c )2 .
=
a ( b + c ) + b ( c + a ) + c ( c + a ) 2 ( ab + bc + ca )
3 ( ab + bc + ca ) 3

=
2 ( ab + bc + ca ) 2

ng thc xy ra khi v ch khi

a
b
c
=
=
a =b =c .
a (b + c ) b (c + a ) c (c + a )
243

www.TaiLieuLuyenThi.com
Khm ph t duy K thut gii bt T Bi ton Max Min ng Thnh Nam

Nhn xt. Nh vy 2 k thut ta cn p dng l:


+ Cng thm vo mi phn thc 1 h s thch hp tt c cc phn thc c dung
t thc.
+ Nhn thm vo mi phn thc mt bin thch hp nhm to bnh phng trnh
trn t thc mi phn thc.
+ Khi c thm iu kin rng buc gia cc bin ta cn kho lo vn dng C S
a v bin chung.
a
3
Bi 2. Cho a,b,c l cc s thc dng tho mn iu kin
.
b+c 2
a
b
c
Chng minh rng
+
+
2.
b+c c+a a+b
Li gii
S dng bt ng thc C S ta c

(b + c )
(b + c )
b
c
+

=
c + a a + b b ( c + a ) + c ( a + b ) a ( b + c ) + 2bc
2

( b + c )2

1
( b + c )2
2
Gi P l biu thc v tri ta c
a (b + c ) +

1
=
1
a
+
b+c 2

x ( 2 x 3)
a
1

=
+ 2=
2, x
.
a
1
+
+
x
b
c
2
1

+
b+c 2
Bt ng thc c chng minh. ng thc xy ra khi v ch khi =
a 3=
b 3c .
Bi 3. Cho x,y,z l cc s thc tho mn iu kin x + y + z =
0.
P

a
+
b+c

Chng minh rng

x ( x + 2)
2x + 1
2

y ( y + 2)
2y +1
2

z ( z + 2)
2z2 + 1

0.

Li gii

Gi s z = max { x , y , z } ta c bt ng thc cho tng ng vi:


x ( x + 2)
2 x2 + 1

1 y ( y + 2) 1 z ( z + 2)
+
+ +
1 0
2 2 y2 + 1 2 2z2 + 1

2
2
2
2 x + 1)
2 y + 1)
2 ( z 1)
(
(

2 x2 + 1
2 y2 + 1
2z2 + 1
S dng bt ng thc C S cho v tri ta c

( 2 x + 1)2 + ( 2 y + 1)2 ( 2 x + 2 y + 2 )2
2 x2 + 1

244

2 y2 + 1

2 ( z 1)
.
=
2 x2 + 2 y 2 + 2 x2 + y 2 + 1
2

www.TaiLieuLuyenThi.com
Cty TNHH MTV DVVH Khang Vit

Vy ta ch cn chng minh
Bt ng thc cui ng ta c pcm.
1
ng thc xy ra khi v ch khi x =
y=
,z =
1 hoc cc hon v.
2

Bi 4. Cho cc s thc x, y, z 1 v tha mn 3 ( x + y + z ) = x 2 + y 2 + z 2 + 2 xy .


Tm gi tr nh nht ca biu
thc P
=

x2
x
.
+ 2
2
( x + y) + x z + x

Li gii

Ta c x 2 x suy ra: P x

( x + y )2 + x

1
4x
.

2
2
z + x ( x + y ) + z 2 + 2 x

Theo gi thit ta c:

( x + y)

2
2
+ z 2 = 3 ( x + y ) + z 3 2 ( x + y ) + z 2 ( x + y ) + z 2 18 .

2 ( 2 x + 18 ) 36
4x
18
18
1
=
= 2
2
= .
18 + 2 x
2 x + 18
1+ 9 5
x+9
1
Vy gi tr nh nht ca P bng t ti=
x 1,=
y 2,=
z 3.
5
Suy ra P

Bi 5. Cho a,b,c l cc s thc dng tha mn iu kin a 2 + b 2 + c 2 =


3.

a2
b2
c2
.
Tm gi tr nh nht ca biu thc P =
+
+
b + 2c c + 2a a + 2b
Li gii
Phn tch tm li gii:
Mi phn thc c dng bnh phng trn t thc tuy nhin ta cn s dng trit
.
iu kin gi thit
Ta x l nh sau:

245

www.TaiLieuLuyenThi.com
Khm ph t duy K thut gii bt T Bi ton Max Min ng Thnh Nam

Theo gi thit ta c:

S dng bt ng thc AM GM cho 3 s dng ta c

Cng theo v 3 bt ng thc trn ta c

.
.

Suy ra
Mt khc

Suy ra

.
.

ng thc xy ra khi v ch khi


.
Bi 6. Cho a,b,c l cc s thc dng. Chng minh rng

.
Li gii
Vit li bt ng thc di dng
.
Gi P l biu thc v tri ta c

Bt ng thc c chng minh. ng thc xy ra khi v ch khi


Bi 7. Cho x,y,z l cc s thc dng tho mn iu kin

246

www.TaiLieuLuyenThi.com
Cty TNHH MTV DVVH Khang Vit

Chng minh rng

.
Li gii

Theo gi thit ta c

S dng bt ng thc C-S ta c

Tng t ta c

Cng theo v 3 bt ng thc trn a v chng minh


.

a v bt ng thc

.
Bt ng thc ny c chng minh. ng thc xy ra khi v ch khi

.
Cch 2: S dng bt ng thc C-S ta c

Tng t ta c

247

www.TaiLieuLuyenThi.com
Khm ph t duy K thut gii bt T Bi ton Max Min ng Thnh Nam

Cng theo v 3 bt ng thc trn ta c iu phi chng minh.


Nhn xt. Nh vy linh hot s dng iu kin cng nh s dng C-S hp l ta c
li gii ngn gn cho bi ton nh cch 2.
Bi 8. Cho x,y,z l cc s thc dng c tng bng 1.
.

Chng minh rng

Li gii
Bt ng thc c dng mnh hn ti cp trong chng 1 bt ng thc
AM GM
.
Di y ta chng minh bng C S cho bi ton trn.
Ta c

Ch

T suy ra iu phi chng minh.


ng thc xy ra khi v ch khi

Bi 9. Cho x,y,z l cc s thc dng tho mn iu kin

Chng minh rng

.
Li gii

S dng bt ng thc C S ta c

.
Vy ta chng minh
t
248

.
.

www.TaiLieuLuyenThi.com
Cty TNHH MTV DVVH Khang Vit

(lun ng).

Bt ng thc tr thnh

Bt ng thc c chng minh. ng thc xy ra khi v ch khi

Bi 10. Cho a,b,c l cc s thc dng tho mn iu kin

.
.

Chng minh rng

Li gii
Theo gi thit ta c

Suy ra

Tng t ta c

Cng theo v 3 bt ng thc trn ta cn chng minh bt ng thc

.
Gi P l biu thc v tri vit li P v s dng bt ng thc C S dng phn
thc ta c

Bt ng thc c chng minh. ng thc xy ra khi v ch khi


Bi 11. Cho a,b,c l cc s thc dng c tch bng 1 chng minh

.
Li gii
Theo gi thit tn ti cc s dng x, y, z tho mn

.
249

www.TaiLieuLuyenThi.com
Khm ph t duy K thut gii bt T Bi ton Max Min ng Thnh Nam

Bt ng thc tr thnh
.
S dng bt ng thc C S ta c

Vy ta ch cn chng minh

Bt ng thc ng ta c pcm. ng thc xy ra khi v ch khi


Bi 12. Cho a,b,c l cc s thc dng c tng bng 3 chng minh

.
Li gii
S dng bt ng thc AM GM ta c

.
Tng t ta c

.
Ta ch cn chng minh
.
S dng bt ng thc C S cho v tri ta c

Vy ta chng minh

250

www.TaiLieuLuyenThi.com
Cty TNHH MTV DVVH Khang Vit

Bt ng thc cui ng. ng thc xy ra khi v ch khi


Bi 13. Cho a,b,c l cc s thc dng tho mn iu kin

.
.

Chng minh rng

Li gii
nn tn ti cc s dng tho mn

Do a,b,c dng v
.

Bt ng thc c vit li di dng

.
Gi P l biu thc v tri ca bt ng thc trn vit li P v s dng bt ng
thc C S ta c

S dng bt ng thc C S ta c

Mt khc s dng bt ng thc quen thuc ta c


.
Do

.
251

www.TaiLieuLuyenThi.com
Khm ph t duy K thut gii bt T Bi ton Max Min ng Thnh Nam

Bt ng thc c chng minh. ng thc xy ra khi v ch khi


Bi 14. Cho a,b,c l cc s thc dng tho mn iu kin

.
.

Chng minh rng

Li gii
Gi P l biu thc v tri vit li P v s dng bt ng thc C S ta c

Vy ta cn chng minh
Tht vy gi s ngc li
S dng bt ng thc Schur bc 3

.
.

Ta c

V l. Vy
.
Bt ng thc c chng minh. ng thc xy ra khi v ch khi
Bi 15. Cho a,b,c l cc s thc khng m tho mn iu kin
Chng minh rng
Li gii
Gi P l biu thc v tri s dng bt ng thc C- S ta c
252

.
.
.

www.TaiLieuLuyenThi.com
Cty TNHH MTV DVVH Khang Vit

P=

a3
b3
c3
+
+
a 2 ( b2 + c2 ) b2 ( c2 + a 2 ) c2 ( a 2 + b2 )

a 3 + b3 + c 3

2 ( a 2b 2 + b 2 c 2 + c 2 a 2 )

Vy ta ch cn chng minh

S dng bt ng thc C S ta c

.
Vy ta ch cn chng minh

t
Ta cn chng minh

Khng mt tnh tng qut gi s

Ch kho st hm s

.
ta c

trn on

ta c

iu phi chng minh.


Bt ng thc c chng minh. ng thc xy ra khi v ch khi
hoc cc hon v.
253

www.TaiLieuLuyenThi.com
Khm ph t duy K thut gii bt T Bi ton Max Min ng Thnh Nam

Bi 16. Cho a,b,c l cc s thc khng m tho mn iu kin


Chng minh rng

.
Li gii
Gi P l biu thc v tri v s dng bt ng thc C S ta c:

Vy ta ch cn chng minh

2 1 + 3abc + 4 ( a 2b + b 2 c + c 2 a + ab 2 + bc 2 + ca 2 )

(a + b + c)3 3abc + ab ( a + b ) + bc ( c + a ) + ca ( c + a )
Bt ng thc cui chnh l bt ng thc Schur bc 3.
Bi ton c chng minh. ng thc xy ra khi v ch khi

hoc

v cc hon v.

C. BI TP RN LUYN
Bi 1. Cho x,y l hai s thc dng tha mn iu kin x + y =
1.

Tm gi tr nh nht ca biu =
thc P

y +1
2

y
x +1
2

Bi 2. Cho x,y l hai s thc dng thay i tha mn iu kin 2 x + 3 y =


5.
Tm gi tr nh nht ca biu thc

(1 + x )(1 + y ) 1
2

=
P
Bi 3. Cho

2y

(1 + x )(1 + y ) 1
3

3x 2

. Tm gi tr nh nht ca biu thc


.

254

www.TaiLieuLuyenThi.com
Cty TNHH MTV DVVH Khang Vit

Bi 4. Cho x, y, z l cc s thc dng. Chng minh rng

3x + 7 ( y + z )

3y + 7 ( z + x)

3z + 7 ( x + y )

3
.
17

Bi 5. (TSH Khi A 2007) Cho x, y, z l cc s thc dng tho mn iu kin


.
Tm gi tr nh nht ca biu thc
.
Bi 6. Chng minh rng vi mi a,b,c l cc s thc dng ta c

.
Bi 7. Cho a,b,c l cc s thc dng tho mn iu kin

.
.

Chng minh rng


Bi 8. Cho x,y,z l cc s thc dng tho mn iu kin

Tm gi tr nh nht ca biu thc


.
Bi 9. Cho x,y,z l cc s thc dng tho mn iu kin

Chng minh rng

.
Bi 10. Cho x,y,z l cc s thc dng tho mn iu kin

Chng minh rng


.
Bi 11. Cho a,b,c l cc s thc dng. Chng minh rng

.
Bi 12. Cho a,b,c l cc s thc dng. Tm gi tr nh nht ca biu thc
.

255

www.TaiLieuLuyenThi.com
Khm ph t duy K thut gii bt T Bi ton Max Min ng Thnh Nam

Bi 13. Cho a,b,c l cc s thc dng. Tm gi tr nh nht ca biu thc


.
Bi 14. Cho a,b,c l cc s thc dng. Tm gi tr nh nht ca biu thc
.
Bi 15. Cho a,b,c l cc s thc dng. Chng minh rng
.
Bi 16. Cho a,b,c l cc s thc dng. Tm gi tr nh nht ca biu thc

.
Bi 17. Cho a,b,c l cc s thc dng. Tm gi tr nh nht ca biu thc
.
Bi 18. Cho a,b,c,x,y,z l cc s thc dng.
Chng minh rng
a
b
c
3( xy + yz + zx)
( y + z) +
( z + x) +
( x + y)
b+c
c+a
a+b
x+ y+z

Bi 19. Cho x,y,z l cc s thc dng v k s thc,

Chng minh rng

Bi 20. Chng minh vi a,b,c l cc s thc dng tho mn iu kin

ta c

.
Bi 21. Chng minh vi mi a,b,c l cc s thc dng ta c
.
Bi 22. Cho a,b,c l cc s thc dng c tng bng 3.
Chng minh rng

Bi 23. Chng minh vi mi a,b,c dng ta c


.
256

www.TaiLieuLuyenThi.com
Cty TNHH MTV DVVH Khang Vit

Bi 24. Cho a,b,c l cc s thc dng tho mn iu kin

.
.

Chng minh rng


Bi 25. Cho a,b,c l cc s thc dng c tch bng 1.

Chng minh rng

Bi 26. Cho a,b,c l di 3 cnh mt tam gic. Chng minh


.

1.
Bi 27. Cho a,b,c l cc s thc dng tha mn iu kin ab + bc + ca =
Chng minh rng

1
a b +c
2

1
b c +a
2

1
c a +b
2

9
.
2 2 3 6abc

Bi 28. Cho a,b,c l cc s thc khng m tho mn iu kin


.
1
1
1
Chng minh rng (a + b + c)
+
+
a + b + c +1
a+b b+c c+a

Bi 29. Cho a,b,c l cc s thc dng tho mn iu kin

Chng minh rng

Bi 30. Cho a,b,c l cc s thc dng. Chng minh


.
Bi 31. Cho a,b,c l cc s thc dng c tch bng 1. Chng minh

.
Bi 32. Cho a,b,c l cc s thc dng. Chng minh
.
Bi 33. Cho a,b,c l 3 s thc dng.
Chng minh rng

.
257

www.TaiLieuLuyenThi.com
Khm ph t duy K thut gii bt T Bi ton Max Min ng Thnh Nam

Bi 34. Cho x,y l hai s thc dng c tng bng 1.


Chng minh rng

Bi 35. Cho x,y l hai s thc dng. Chng minh

32 ( x 2 + y 2 )
1
1
4
.
+ +

4
x2 y 2 x2 + y 2
( x + y)

Bi 36. Cho x,y l cc s thc dng tho mn iu kin

.
Tm gi tr nh nht ca biu thc

Bi 37. Cho a,b,x,y,z l cc s thc dng. Chng minh


.
Bi 38. Cho x,y,z l cc s thc dng. Chng minh
.

1.
Bi 39. Cho a,b,c l cc s thc dng tha mn a + b + c =
Tm gi tr nh nht ca biu thc
P=

a 3 + 2b3
b 3 + 2c 3
c 3 + 2a 3
.
+
+
4ab + 3a 2 4bc + 3b 2 4ca + 3c 2

Bi 40. Chng minh rng vi mi a, b, c dng ta c

.
A. HNG DN GII P S
Bi 1. S dng bt ng thc C-S dng phn thc ta c:

P=

x
y2 +1

y
x2 + 1

x2
x y2 +1

1
x . xy 2 + x + y . yx 2 + y

1
1
=
=
1 + xy
xy ( x + y ) + x + y

258

y2
y x2 + 1

( x + y)

x y 2 + 1 + y x2 + 1
1

( x + y ) ( xy 2 + x + yx 2 + y )
1

2
=
5
x+ y
1+

2
2

www.TaiLieuLuyenThi.com
Cty TNHH MTV DVVH Khang Vit

ng thc xy ra khi v ch khi x= y=


Vy gi tr nh nht ca P bng

1
.
2

1
2
t ti x= y=
.
2
5

Bi 2. S dng bt ng thc C-S ta c:

(1 + x )(1 + y ) 1 + xy, (1 + x )(1 + y ) 1 + xy


2

Suy ra P

xy .

xy xy xy 1
y
+
=
3 x + 2 y
.
2
2y
3x
6
x

Nhn xt. V phi ca bt ng thc bc 1 v iu kin bc 1 nn s dng k thut

y
y y
3x + 2 y
3+ 2
5
5
x =
x x .
ng bc: P .
.
6 2x + 3y
6 2 + 3y
x
t t
=

y
, ( t > 0 ) khi
x

5 3 + 2t 3 5 3 + 2t 3 5 5 ( t 1) ( 2t + 1) 5 5
P .=
=
1 +
.
+ .

6 2 + 3t 2 6 2 + 3t 2 6 6
2 + 3t 2
6 6
ng thc xy ra khi v ch khi t =1 x = y x = y =1 .
2

5
t ti x= y= 1 .
6
Bnh lun. D on c im ri x= y= 1 ta c th nh gi nhanh bng bt
Vy gi tr nh nht ca P bng

ng thc AM-GM nh sau:

1
y 1
y
y 1
y y 2x + 3y 5

P 3x + 2 y =
= .
2 x + x + y + y 2 x + 3 3 xy y =
6
x 6
x
x 6
x x
6
6

Bi 3. S dng bt ng thc C-S ta c


.
ng thc xy ra khi v ch khi

Suy ra

.
.

S dng bt ng thc C-S ta c


259

www.TaiLieuLuyenThi.com
Khm ph t duy K thut gii bt T Bi ton Max Min ng Thnh Nam

.
Suy ra

ng thc xy ra khi v ch khi

.
Vy gi tr nh nht ca P bng 24 t ti

Bi tp tng t
1) Cho
. Tm gi tr nh nht ca biu thc

.
Bi 4. S dng bt ng thc C-S dng phn thc ta c

x2
y2
z2
+
+
VT = 2
3x + 7 ( y + z ) x 3 y 2 + 7 ( z + x ) y 3z 2 + 7 ( x + y ) z

( x + y + z)
( x + y + z)

=
2
2
2
2
3 ( x + y + z ) + 14 ( xy + yz + zx ) 3 ( x + y + z ) + 8 ( xy + yz + zx )
2

( x + y + z)

3
=
8
2
3 ( x + y + z ) + ( x + y + z ) 17
3
ng thc xy ra khi v ch khi x= y= z .

Bi 5. S dng bt ng thc AM GM cho 2 s dng ta c

.
Suy ra
260

www.TaiLieuLuyenThi.com
Cty TNHH MTV DVVH Khang Vit

Tng t ta c

Cng theo v 3 bt ng thc trn ta c


.
t

Ta c

2a
2b
2c
+
+
b + 2c c + 2a b + 2b

a2
b2
c2
= 2
+
+

a ( b + 2c ) b ( c + 2a ) c ( a + 2b )

2(a + b + c)

a ( b + 2c ) + b ( c + 2a ) + c ( a + 2b )
2

2(a + b + c)
6 ( ab + bc + ca )
=

=2
3 ( ab + bc + ca ) 3 ( ab + bc + ca )
2

.
Vy gi tr nh nht ca P bng 1 t ti
Bi 6. S dng bt ng thc AM GM cho 2 s dng ta c

Tng t ta c

Cng theo v 3 bt ng thc trn v gi P l biu thc v tri ca bt ng thc


ta c
261

www.TaiLieuLuyenThi.com
Khm ph t duy K thut gii bt T Bi ton Max Min ng Thnh Nam

.
Bt ng thc c chng minh. ng thc xy ra khi v ch khi
Bi 8. S dng bt ng thc AM GM cho 2 s dng ta c

Tng t ta c

Cng theo v 3 bt ng thc trn ta c

.
ng thc xy ra khi v ch khi

. Vy gi tr nh nht ca P bng 2.

Cch 2: Vit li P di dng

.
Ch s dng AM GM ta c

Cng theo v 2 bt ng thc trn c kt qu tng t trn.


suy ra
Bi 9. Gi P l biu thc v tri, vi gi thit
.
S dng bt ng thc C S dng phn thc ta c
262

www.TaiLieuLuyenThi.com
Cty TNHH MTV DVVH Khang Vit

.
Ta cn chng minh

Bt ng thc trn l tng ca cc bt ng thc

Chng minh (1).


S dng bt ng thc AM GM cho 3 s dng ta c

Chng minh (2).


S dng bt ng thc AM GM cho 4 s dng ta c
.
Cng theo v 3 bt ng thc trn ta c iu phi chng minh.
Chng minh (3).
Bt ng thc tng ng vi

.
Bi ton c chng minh. ng thc xy ra khi v ch khi

Bi 10. Gi P l biu thc v tri ca bt ng thc.


Vit li P v s dng bt ng thc C S ta c

263

www.TaiLieuLuyenThi.com
Khm ph t duy K thut gii bt T Bi ton Max Min ng Thnh Nam

Vy ta ch cn chng minh
.
Gi Q l biu thc v tri ca bt ng thc vit li Q v s dng bt ng thc
C-S ta c

Ch
2
x 2 y 2 + z 2 + y 2 z 2 + x 2 + z=
x2 + y 2

( x + y + z ) x ( y
2 ( x + y + z )( x y
2

x2 x2 ( y 2 + z 2 )

+ z 2 ) + y 2 ( z 2 + x 2 ) + z 2 ( x 2 + y 2 )

+ y2 z 2 + z 2 x2 )

2
2 2
x + y 2 + z 2 )( x 2 + y 2 + z 2 )
(
3

Suy ra
.

Bt ng thc c chng minh. ng thc xy ra khi v ch khi

Bi 11. Vit li bt ng thc di dng


.
264

www.TaiLieuLuyenThi.com
Cty TNHH MTV DVVH Khang Vit

Gi P l biu thc v tri ca bt ng thc ta c

Bt ng thc c chng minh. ng thc xy ra khi v ch khi


Bi 12. Ta c:

ng thc xy ra khi v ch khi

Bi 13.
Phn tch tm li gii:
Tng t bi ton trn ta cn thm vo mi phn thc cc hng s thch hp
c c 3 phn thc c chung t thc.
Ta chn x,y dng sao cho:

265

www.TaiLieuLuyenThi.com
Khm ph t duy K thut gii bt T Bi ton Max Min ng Thnh Nam

Vy ta thm vo

Vy ta c P

Bi 14. Ta c:

3(b + c )
4a + 3c 12 ( b c )
=
+ 2 +
+ 1 +
+ 8 11
P
2a + 3c
2a
3b

1
4
1
= ( 4a + 3b + 3c ) + +
11
2a 3b 2a + 3c
1
4
1
= 2a + 3b + ( 2a + 3c ) + +
11
2a 3b 2a + 3c
(1 + 1 + 2 ) 11 =5
2

ng thc xy ra khi v ch khi

Bi 15. Gi P l biu thc v tri ca bt ng thc.


Ta c

b+c
2a + c 4 ( a + b )
=
P
+ 2 +
+ 1 +
+ 4 7
a
b
a+c

( 2a + b + c )

1 1
4
+ +
7
a b a+c

4
2
1 1
( a + b + ( a + c )) + +
7 (1 + 1 + 2 ) 7 =9
a b a+c
ng thc xy ra khi v ch khi
Bi 16. Ta c
266

www.TaiLieuLuyenThi.com
Cty TNHH MTV DVVH Khang Vit

ng thc xy ra khi v ch khi

Bi 17. Ta c

64 ( 4a + 9b )

9b + 16c
25 ( 4a + 16c )
=
P
+ 4 +
+ 9.25 +
+ 64.16 1253
a
b
c

( 4a + 9b + 16c )

1 25 64
2
+ + 1253 ( 2 + 3.5 + 4.8 ) 1253 = 1148
c
a b

.
ng thc xy ra khi v ch khi
Bi 18. Gi P l biu thc v tri ca bt ng thc.
Ta vit li P v s dng bt ng thc C S ta c

Vy ta chng minh

267

www.TaiLieuLuyenThi.com
Khm ph t duy K thut gii bt T Bi ton Max Min ng Thnh Nam

Ta c

x 2 + xy + yz + zx
=

( x + y + z)

+ 3 xy + yz + zx

( x + y + z)

+ 9 ( xy + yz + zx )

Vy ta chng minh

Bt ng thc lun ng.


Bt ng thc c chng minh. ng thc xy ra khi v ch khi
.
Bi 19. Gi P l biu thc v tri, vit li P v s dng bt ng thc C S ta c

Vy ta chng minh

( k + 1)( x + y + z )

9 x ( x 2 + kyz ) + y ( y 2 + kzx ) + z ( z 2 + kxy )

( k 8 ) ( x3 + y 3 + z 3 ) + 3 ( k + 1)( x + y )( y + z )( z + x ) 27 kxyz
Bt ng thc cui luon ng da vo AM GM.
Bt ng thc c chng minh. ng thc xy ra khi v ch khi
.
Bi 20. Gi P l biu thc v tri, vit li P v s dng bt ng thc C S ta c

268

www.TaiLieuLuyenThi.com
Cty TNHH MTV DVVH Khang Vit

Bt ng thc c chng minh. ng thc xy ra khi v ch khi


Bi 21. t

bt ng thc tr thnh
.

S dng bt ng thc AM GM ta c

Cng theo v 3 bt ng trn kt hp s dng C S ta c

.
Bt ng thc c chng minh. ng thc xy ra khi v ch khi
Bi tp tng t
Cho a,b,c l cc s thc dng. Chng minh rng

.
Bi 22. Gi P l biu thc v tri ca bt ng thc v s dng AM GM ta c

Cng theo v 3 bt ng thc trn ta c

.
Vy ta cn chng minh
269

www.TaiLieuLuyenThi.com
Khm ph t duy K thut gii bt T Bi ton Max Min ng Thnh Nam

.
S dng bt ng thc C S ta c

Vy ta chng minh

Bt ng thc c chng minh. ng thc xy ra khi v ch khi


Bi tp tng t
Cho a,b,c l cc s thc dng c tng bng 3.
Chng minh rng

.
Bi 23. Gi P l biu thc v tri s dng bt ng thc C S ta c

(a

+ b2 + c2 )

a b 2 bc + c 2 + b c 2 ca + a 2 + c a 2 ab + b 2

(a

+ b2 + c2 )

( a + b + c ) a ( b2 bc + c 2 ) + b ( c 2 ca + a 2 ) + c ( a 2 ab + b2 )

Ta cn chng minh

(a

+ b 2 + c 2 ) ( a + b + c ) a ( b 2 bc + c 2 ) + b ( c 2 ca + a 2 ) + c ( a 2 ab + b 2 )
2

a 4 + b 4 + c 4 + abc ( a + b + c ) ab ( a 2 + b 2 ) + bc ( b 2 + c 2 ) + ca ( c 2 + a 2 )
270

www.TaiLieuLuyenThi.com
Cty TNHH MTV DVVH Khang Vit

Bt ng thc cui lun ng(xem ch chng minh tng ng).


Bt ng thc c chng minh. ng thc xy ra khi v ch khi
Bi 24. t

Bt ng thc tr thnh

Gi P l biu thc v tri v s dng bt ng thc C S ta c

Mt khc s dng bt ng thc AM GM ta c

Tng t ta c

Cng theo v 3 bt ng thc trn t suy ra

.
Bt ng thc c chng minh. ng thc xy ra khi v ch khi
Bi 25. S dng bt ng thc C S ta c

.
Tng t ta suy ra
271

www.TaiLieuLuyenThi.com
Khm ph t duy K thut gii bt T Bi ton Max Min ng Thnh Nam

Bt ng thc c chng minh. ng thc xy ra khi v ch khi


Bi 26. Gi P l biu thc v tri ta c

Bt ng thc c chng minh. ng thc xy ra khi v ch khi


Bi 27.
S dng bt ng thc C-S ta c

1
a b +c
2

1
b c +a
2

1
c a +b
2

1
a b +c
2

1
b c +a
2

a b + c + b c + a 2 + c a 2 + b2

1
c a +b
2

9
6 12abc ( a + b + c )

Bt ng thc c chng minh nu ta chng minh c

272

9
2

Tng t ta c

Vy

www.TaiLieuLuyenThi.com
Cty TNHH MTV DVVH Khang Vit

9
6 12abc ( a + b + c )

9
2 2 3 6abc 6 12abc ( a + b + c ) .
2 2 3 6abc

Lun ng do a + b + c 3 ( ab + bc +=
ca )
ng thc xy ra khi v ch khi a= b= c=

3; abc

1
3 3

1
.
3

Bi 28. Bt ng thc tng ng vi

S dng bt ng thc C S ta c

Bt ng thc c chng minh ng thc xy ra khi v ch khi


hoc cc hon v.
Bi 29. S dng bt ng thc C S ta c

Vy ta ch cn chng minh
S dng bt ng thc AM GM ta c

.
.

273

www.TaiLieuLuyenThi.com
Khm ph t duy K thut gii bt T Bi ton Max Min ng Thnh Nam

a+b+c =

( a + b + c ) ( a 2 + b2 + c2 )

= ( a 3 + ac 2 ) + ( b3 + ba 2 ) + ( c 3 + cb 2 ) + ab 2 + bc 2 + ca 2
2 ( a 2 c + b 2 a + c 2b ) + ab 2 + bc 2 + ca 2= 3 ( ab 2 + bc 2 + ca 2 )
.

Suy ra

Cng theo v 2 bt ng thc trn ta c iu phi chng minh. ng thc xy ra

khi v ch khi

Bi 30. Do tnh thun nht ca bt ng thc chun ho cho

Ta cn chng minh

S dng bt ng thc C S v gi P l biu thc v tri ta c

.
Vy ta ch cn chng minh

.
Tht vy s dng bt ng thc C S ta c

a b+c +b c+a +c a+b


=

( a + b + c ) a ( b + c ) + b ( c + a ) + c ( a + b )
3 ( ab + bc + ca ) a + b=
+c

3
2

Bt ng thc c chng minh. ng thc xy ra khi v ch khi


Bi 31. Bt ng tng ng vi

.
Gi P l biu thc v tri v s dng bt ng thc C S ta c
.
Vy ta cn chng minh

274

www.TaiLieuLuyenThi.com
Cty TNHH MTV DVVH Khang Vit

S dng bt ng thc C S ta c

Bt ng thc c chng minh. ng thc xy ra khi v ch khi


Bi 32. t

Bt ng thc tr thnh:

Theo bt ng thc C S ta c:
.
V vy ta ch cn chng minh

( x + y + z )2
3
2
2
2
2
2
k ( x + y + z ) + (k + 1)( xy + yz + zx) k + k + 1
(k 1) 2 ( x 2 + y 2 + z 2 xy yz zx) 0
2
2
2
2
( k 1) ( x y ) + ( y z ) + ( z x ) 0

Bt ng thc cui ng ta c pcm.


Bi 33. t

ta c:

Bt ng thc cho tr thnh:


.
p dng bt ng thc C S ta c:

.
Bt ng thc c chng minh. ng thc xy ra khi v ch khi
Bi 34. Ta c

.
275

www.TaiLieuLuyenThi.com
Khm ph t duy K thut gii bt T Bi ton Max Min ng Thnh Nam

Bt ng thc c chng minh. ng thc xy ra khi v ch khi

Bi 35.
Ta c:

(x

1 1
4
+=
+ 2
2
2
x
y
x + y2
Ta chng minh

( x + y)
2

2x y

(x

+y

( x + y + 2 xy )
=
) 2x y ( x + y )

+ y 2 ) + 4 x2 y 2
2

x2 y 2 ( x2 + y 2

32 ( x 2 + y 2 )

( x + y)

( x + y)

2x2 y 2 ( x2 + y 2 )

( x + y ) 64 x 2 y 2 ( x 2 + y 2 ) .
2

( x + y ) 8 xy ( x 2 + y 2 )

(( x

+ y 2 ) + 2 xy

8 xy ( x 2 + y 2 ) .

( x 2 + y 2 ) 4 xy ( x 2 + y 2 ) + 4 x 2 y 2 0 ( x y ) 0 (lun ng).
2

Bt ng thc c chng minh. ng thc xy ra khi v ch khi x = y .


Nhn xt. Ta c th bin i tng ng bt ng thc nh sau:

2
2
1
1
4 32 ( x + y )
.
( x + y) 2 + 2 + 2 2
2
y
x +y
( x + y)
x
2

x
y

t t =+

x2 + 2x +

y
, ( t 2 ) v a v chng minh bt ng thc:
x

8
32 x
2
+4
( x 2 ) ( x 2 + 8 x + 4 ) 0 (lun ng).
x
x+2

Bi 36. Ta c
.
Vy ta c

S dng bt ng thc C S ta c
.
Bi 37. S dng bt ng thc C S ta c

276

www.TaiLieuLuyenThi.com
Cty TNHH MTV DVVH Khang Vit

Bt ng thc c chng minh. ng thc xy ra khi v ch khi


Bi 38. Gi P l biu thc v tri v t

.
.

Suy ra
.

Ch :

Thay t ln lt bi a,b,c v s dng bt ng thc quen thuc:

.
. ng thc xy ra khi v ch khi

V vy

Bi 39. S dng bt ng thc AM-GM ta c:

a 3 + b3 ab ( a + b ) a 3 + 2b3 ab ( a + b ) + b3 = b ( a 2 + ab + b 2 ) 3ab 2 .
Suy ra

a 3 + 2b3
3ab 2
3b 2
.

=
4ab + 3a 2 4ab + 3a 2 3a + 4b

Tng t ta c:

b3 + 2c3
3c 2
c 3 + 2a 3
3a 2
.

4bc + 3b 2 3b + 4c 4ca + 3c 2 3c + 4a

a2
b2
c2
+
+
.
4a + 3c 4b + 3a 4c + 3b

Suy ra P 3

S dng bt ng thc C-S ta c:

(a + b + c) = a + b + c = 1 .
a2
b2
c2
+
+

4a + 3c 4b + 3a 4c + 3b 7 ( a + b + c )
7
7
2

1
3
. ng thc xy ra khi v ch khi a= b= c=
.
3
7
3
1
.
Vy gi tr nh nht ca P bng t ti a= b= c=
7
3
Vy P

277

www.TaiLieuLuyenThi.com
Khm ph t duy K thut gii bt T Bi ton Max Min ng Thnh Nam

Bi 40. Gi P l biu thc v tri bt ng thc ta c

.
S dng bt ng thc C-S dng phn thc ta c
.
Bt ng thc c chng minh. ng thc xy ra khi v ch khi

CH 6:

K THUT THAM S HA

A. NI DUNG PHNG PHP


Khi p dng cc bt ng thc c bn nh AM-GM, C-S ci kh nht ca bi
ton l d on du bng. khc phc iu ny ta la chn k thut tham s ha
tc gi s du bng ca bt ng thc t ti cc im cho trc v i tm cc im
xy ra du bng bng cch gii phng trnh v h phng trnh ta c kt qu ca
bi ton.
Ta xt bi ton sau y
Cho x,y,z l cc s thc tho mn iu kin xy + yz + zx =
1.
Tm gi tr nh nht ca biu thc P = kx 2 + ly 2 + z 2 vi k,l l cc s thc
khng m.
Ta c 2 cch cn bng h s cho bi ton trn cng chnh l 2 phng php c
bn ta hay s dng x l cc bi ton phi s dng n tham s ho.
B. BI TP CHN LC
Bi 1. Cho x,y,z l cc s thc tho mn iu kin xy + yz + 3 zx =
1.
Tm gi tr nh nht ca biu thc P = x 2 + y 2 + z 2 .
Li gii
Phn tch tm li gii:
Vi iu kin cho ta thy vai tr ca x v z nh nhau vy gi s P t min ti

x= z= k . y, ( k ) .

Vi a dng s dng bt ng thc AM-GM ta c

a ( x 2 + z 2 ) 2azx

2
2 2
x + k y 2kxy .
z 2 + k 2 y 2 2kyz

278

www.TaiLieuLuyenThi.com
Cty TNHH MTV DVVH Khang Vit

Cng theo v cc bt ng thc trn ta c:

( a + 1) ( x 2 + z 2 ) + 2k 2 y 2 2ky ( z + x ) + 2azx
x2 + z 2 +

2k 2 2
2a 3k

y
( yz + xy ) + 3zx
a +1
3 ( a + 1) a

Ta cn chn a v k tho mn

2k 2
a + =1

3 3 + 33
a > 0
1
a =
3k

= 1 a = 3k
a

3 + 33
0
2k 3k 3 =
a > 0
k =

Vi a v k chn c t trn ta c P

2a
15 33
=.
3 ( a + 1)
16

Li gii chi tit:


S dng bt ng thc AM-GM ta c:

3 3 + 33
4

) (x

+ z2 )

3 3 + 33
2

) zx

3 + 33 3 + 33
x 2 +
yz

2
4
3 + 33 2 3 + 33
z 2 +
xy
y
4
2

Cng theo v ba bt ng thc trn ta c

2
3 3 + 33

3 + 33 2
2
2

+ 1 ( x + z ) + 2.
y

4
4

hay

3 + 33
3 + 33
( xy + yz + 3zx ) =
2
2

2 3 + 33 15 33
10 + 3 33
3 + 33
.
.P
P
=
4
2
16
10 + 3 33
3 + 33
y=
6 33 .
2

ng thc xy ra khi v ch khi x ==


z

279

www.TaiLieuLuyenThi.com
Khm ph t duy K thut gii bt T Bi ton Max Min ng Thnh Nam

Bi 2. Cho x,y,z l cc s thc dng c tng bng 3.


Tm gi tr nh nht ca biu thc P = x 2 + y 2 + z 3 .
Li gii
Vai tr ca x v y nh nhau cho php ta gi s P t min ti x= y= a, z= b .
Theo gi thit ta c 2a + b =
3.
S dng bt ng thc AM-GM ta c

x 2 + a 2 2ax
y 2 + a 2 2ay
z 3 + b3 + b3 2b 2 z
Cng theo v 3 bt ng thc trn ta c

x 2 + y 2 + z 3 + ( 2a 2 + 2b3 ) 2a ( x + y ) + 3b 2 z .

Ta chn cc h s a v b dng sao cho 2a = 3b 2 .

19 37
3b
a =
3 a =
2a + b =

12
Vy ta c h phng trnh
.

2
2
a
=
3
b
2

1
+
37

3b b 3 =
0 b =

6
2

19 37 1 + 37
Suy ra P =
2a + b 2
+
.
6
12

Bi 3. Cho x,y l hai s thc khng m tho mn iu kin x 2 + y 2 =


5.
Tm gi tr nh nht ca biu thc P
= x3 + y 6 .
Li gii
Gi s P t min ti=
5.
x a=
, y b theo gi thit ta c a 2 + b 2 =
S dng bt ng thc AM-GM ta c

x3 + x 3 + a 3 3ax 2
2 ( y 6 + b 6 + b 6 ) 2.3b 4 y 2
Cng theo v 2 bt ng thc trn ta c

2 x3 + 2 y 6 + a 3 + 4b 6 3 ( ax 2 + 2by 2 ) .

Ta chn cc s dng a,b tho mn a = 2b .

a = 2b

a = 2
.

5 b = 1
a + b =

Khi ta c h phng trnh

Do P a 3 + b 6 =
9 . ng thc xy ra khi v ch khi=
x 2,=
y 1.
280

www.TaiLieuLuyenThi.com
Cty TNHH MTV DVVH Khang Vit

Bi 4. Cho x,y,z l cc s thc khng m tho mn iu kin x + y + z =


3.
Tm gi tr nh nht ca biu thc P =x 4 + 2 y 4 + 3 z 4 .
Li gii
Gi s P t min ti=
3.
x a=
, y b=
, z c theo gi thit ta c a + b + c =
S dng bt ng thc AM-GM ta c

x 4 + a 4 + a 4 + a 4 4a 3 x
2 ( y 4 + b 4 + b 4 + b 4 ) 8b3 y
3 ( z 4 + c 4 + c 4 + c 4 ) 12c 3 z
Cng theo v 3 bt ng thc trn ta c

x 4 + 2 y 4 + 3 z 4 + 3a 4 + 6b 4 + 9c 4 4 ( a 3 x + 2b3 y + 3c3 z ) .

Ta chn cc s dng a,b,c sao cho =


a 3 2=
b3 3c3 .

3
a =

1
1
1+ 3 + 3

2
3

3
a + b + c =
3

.
Vy ta c h phng trnh 3
b =
3
3
1
1

a 2=
b 3c
3
=

2 1 + 3 + 3

2
3

3
9

=
c

1
1
1+ 3 + 3
2
3

Suy ra P a 4 + 2b 4 + 3c 4 vi a,b,c xc nh nh trn.


Tng qut. Vi cc s thc khng m x1 , x2 ,..., xn c tng bng n. V n s thc
dng k1 , k2 ,..., kn . Tm gi tr nh nht ca biu thc P= k1 x1m + k2 x2m + ... + kn xnm
vi m > 1 l s nguyn dng cho trc.
Bi 5. Cho x,y,z l cc s thc dng tho mn iu kin 2 x + 4 y + 3 z 2 =
68 .
Tm gi tr nh nht ca biu thc P = x 2 + y 2 + z 3 .
Li gii
Gi s P t min ti=
68 .
x a=
, y b=
, z c theo gi thit ta c 2a + 4b + 3c 2 =
S dng bt ng thc AM-GM ta c

281

www.TaiLieuLuyenThi.com
Khm ph t duy K thut gii bt T Bi ton Max Min ng Thnh Nam

x 2 + a 2 2ax
y 2 + b 2 2by
z 3 + z 3 + c3 3cz 2 z 3 +

c3 3cz 2

2
2

Cng theo v 3 bt ng thc trn ta c

c3
3cz 2
.
2ax + 2by +
2
2
3c
2a 2b 2
Chn cc s dng a,b,c sao cho
=
=
b = c = 2a .
2
4
3
x2 + y 2 + z 3 + a 2 + b2 +

2a + 4b + 3c 2 =
68
a = 2
Vy ta c h phng trnh
.

b= c= 4
b= c= 2a
Suy ra P a 2 + b 2 + c 3 =
84 .
Bi 6. Cho x,y,z l cc s thc khng m tha mn x + y + z =
3.
Tm gi tr nh nht v ln nht ca biu thc

P=

x2 x + 1 + y 2 y + 1 + z 2 z + 1 .
Li gii

Tm gi tr ln nht ca P
Nhn xt. Tm mt bt ng thc dng x 2 x + 1 mx + n .
Cng vic ca ta l i tm m v n. Bt ng thc i xng vi 3 bin x,y,z nn ta
khng th quyt nh xem bin no bng 0 v bin no bng 3. V vy cch tt nht
l tm m v n sao cho du bng t ti 0 v 1 u tha mn. Hay cch khc ta cn
tm m,n sao cho c 0 v 3 l nghim ca phng trnh:

x 2 x + 1= mx + n .

7 1
n = 1
m =
Vy ta c h phng trnh

3
3m + n =7
n = 1

Cui cng kim tra xem

x2 x + 1 1 +

7 1 x
3

(1 + 2 7 ) x ( x 3) 0
( 7 1) x
x x +1 +1+
2

vi x [ 0;3] ).
282

(lun ng

www.TaiLieuLuyenThi.com
Cty TNHH MTV DVVH Khang Vit

Vy P 3 +

7 1 ( x + y + z )
3

= 3 + 7 1 = 2 + 7 .

ng thc xy ra khi v ch khi 2 bin bng 0 v mt bin bng 3.


Vy gi tr ln nht ca P bng 2 + 7 t ti a= b= 0, c= 3 hoc cc hon v.
Tm gi tr nh nht ca P

3 ( x 1) x + 1
x +1
.
Ta c x=
x +1
+

4
2
2
y +1
z +1
.
Tng t ta c: y 2 y + 1
; z2 z +1
2
2
x+ y+ z +3
Cng theo v 3 bt ng thc trn ta c: P
=
3.
2
ng thc xy ra khi v ch khi x= y= z= 1 .
2

Vy gi tr nh nht ca P bng 3 t ti x= y= z= 1 .
Bi 7. Cho x,y,z l cc s thc tha mn iu kin xy + yz + zx =
1.
Tm gi tr nh nht ca biu thc P =x 2 + 2 y 2 + 5 z 2 .
Li gii
Ta chng minh bt ng thc:

x 2 + 2 y 2 + 5 z 2 2k ( xy + yz + zx ) =
2k vi k > 0 l s chn sau.

Cng thm vo hai v bt ng thc trn vi k x 2 + y 2 + z 2 ta c:

( k + 1) x 2 + ( k + 2 ) y 2 + ( k + 5) z 2 k ( x + y + z )

S dng bt ng thc C-S ta c:

( x + y + z)
x2
y2
z2
+
+
+
+
+
=
+
+

k
1
x
k
2
y
k
5
z
( ) (
) (
)
1
1
1
1
1
1
+
+
k +1 k + 2 k + 5 k +1 k + 2 k + 5
2

Vy ta ch cn chn k dng sao cho:

1
k >0
= k ( k 1) ( k 2 + 5k + 5 )= 0
k= 1 .
1
1
1
+
+
k +1 k + 2 k + 5
Li gii.
Ta c:

283

www.TaiLieuLuyenThi.com
Khm ph t duy K thut gii bt T Bi ton Max Min ng Thnh Nam

x2 y 2 z 2 ( x + y + z )
+ +
=
2x + 3y + 6z =
1
1
1
1 1 1
+ +
2
3 6
2 3 6
P = x 2 + 2 y 2 + 5 z 2 2 ( xy + yz + zx ) = 2
2

ng thc xy ra khi v ch khi x : y : =


z

( x + y + z)

1 1 1
: : = 3 : 2 :1 =
x 3z, =
y 2z
2 3 6

thay vo iu kin ta c

3 z 2 + 6 z 2 + 2 z 2 =
1 z =

1
2
1
3
;
;
( x; y; z ) =

.
11
111
11
11

Vy gi tr nh nht ca P bng 2 t ti ( x; y; z ) =

3
2
1
;
;
.
11
111
11

Bi 8. Cho a,b,c l cc s thc dng tha mn iu kin a 2 + b 2 + c 2 =


1.
Tm gi tr ln nht ca biu thc P = a 2b3c 3 .
Li gii
Phn tch chn im ri:
Gi s P t gi tr nh nht ti=
a x=
, b y=
,c z .
Khi theo iu kin ta c: x 2 + y 2 + z 2 =
1.
S dng bt ng thc AM-GM ta c:

2a 3b 4c
a 2b 3c 4
+ +
99 2 3 4 .
x
y
z
x y z
S dng bt ng thc C-S ta c:
2

2a 3b 4c
9 16 4
9 16
2
2
2 4
+ + (a + b + c ) 2 + 2 + 2 = 2 + 2 + 2 .
y
z
y
z x
y
z
x
x
ng thc xy ra khi v ch khi

2 3 4
ax by cz
x2 y 2 z 2
a :b:c = : :
= =
=
= .
x y z
2
3
4
2
3
4

2
x =
3

x2 y 2 z 2

3
= =
Vy ta chn x, y, z > 0 sao cho 2
3
4 y = .
3
x2 + y 2 + z 2 =
1

z = 3

284

www.TaiLieuLuyenThi.com
Cty TNHH MTV DVVH Khang Vit

Li gii chi tit:


S dng bt ng thc AM-GM ta c:

2a
3b 4c
a 2b3c 4
.
+
+
9
2
3
4
2
3 2
9
2 3 2



3
3
3
3 3 3
S dng bt ng thc C-S ta c:
2
2

2a
3b 4c
2 3 4
+
+ =
+
+
( a 2 + b 2 + c 2 )
9.
2 3 2
2
3 2

3
3
3
3
3 3

2
Suy ra: P =
a 2b3c 4

3 2 4 32 3
.

=
6561
3 3

ng thc xy ra khi v ch khi


=
a

2
=
,b
3

3
2
.
=
,c
3
3

Bi 9. Cho x,y, z l cc s thc dng. Tm gi tr ln nht ca biu thc

P=

2 xy + 4 yz + 3 zx

( x + y + z)

Li gii
Ta c
x
y
y
z
z
x
.
.
+ 4.
.
+ 3.
.
x+ y+z x+ y+z
x+ y+z x+ y+z
x+ y+z x+ y+z
x
y
z
t a =
1.
=
,b =
,c
, ( a, b, c > 0 ) ta c a + b + c =
x+ y+z
x+ y+z
x+ y+z
Ta cn tm gi tr ln nht ca biu thc
P = 2ab + 4bc + 3ca vi a + b + c =
1.
Ta chn cc hng s m,n,p dng sao cho
P = 2.

= ma ( b + c ) + nb ( c + a ) + pc ( a + b )
2ab + 4bc + 3ca

( m + n ) ab + ( n + p ) bc + ( p + m ) ca = 2ab + 4bc + 3ca


1

m = 2
2
m + n =

n + p = 4 n =
2
p+m =

p = 2

285

www.TaiLieuLuyenThi.com
Khm ph t duy K thut gii bt T Bi ton Max Min ng Thnh Nam

Khi

1
3
5
a (b + c ) + b (c + a ) + c ( a + b)
2
2
2
1
3
5
=
a (1 a ) + b (1 b ) + c (1 c )
2
2
2
2
2
2
9 1
1
1
1

= a + 3 b + 5 c
8 2
2
2
2

=
P

2
2
2

1
1
1

a
b
c




9 1
2
2
2
=

+
+
1
1

8 2
1

3
5

1
1
1

a +b +c

9
24
2
2
2

=
8
23
1 1
2 1 + +
3 5
ng thc xy ra khi v ch khi

1
a + b + c =

1
1
1
1 1 x = 3 y 1= 5z 2 .

1: :
a 2 : b 2 : c 2 =
3 5

Bi tp tng t
1) Cho x,y,z l cc s thc
Bng cch tng t ta tng qut bi ton sau y

c k , ( k > 0 ) . Tm gi tr
Cho a,b,c l cc s thc khng m tho mn a + b +=

ln nht ca biu thc

P = m.ab + n.bc + p.ca, ( m, n, p > 0 ) .

Bi 10. Chng minh vi mi 0 x 1 ta c x 9 1 + x 2 + 13 1 x 2 16 .


Li gii
Ta c

2 2
2
2 2
2
9ax 1 + x 2 13bx 1 x 2 9 ( a x + 1 + x ) 13 ( b x + 1 x )
.
VT =
+

+
a
b
2a
2b
Ta chn a v b dng sao cho h s ca x 2 trit tiu v tho mn du bng ca

bt ng thc AM GM

286

www.TaiLieuLuyenThi.com
Cty TNHH MTV DVVH Khang Vit

9 ( a 2 + 1) 13 ( b 2 1)

+
=
0
3

a 2 1 = b 2 + 1
2b
a=
2a

2 2

2.
9 ( a 2 + 1) 13 ( b 2 1)

1 + x2
a x =
+
=
0
b 2 x 2 = 1 x 2

b = 1
2b
2a

Li gii chi tit:

x
3
x 9 1 + x 2 + 13 1 =
x2
6. x 1 + x 2 + 26. 1 x 2
2
2
2
9x

x2

3
+ 1 + x 2 + 13 + 1 x 2 =
16
4

9x2
2
4 = 1 + x
2
ng thc xy ra khi v ch khi 2
x=
5
x = 1 x2
4
C. BI TP RN LUYN
Bi 1. Cho x,y,z l cc s thc khng m tho mn iu kin xy + yz + zx =
1.
Chng minh rng 10 x 2 + 10 y 2 + z 2 4 .
Bi 2. Cho x,y,z l cc s thc dng c tng bng 3. Tm gi tr nh nht ca biu
thc P =

15 5 3
+ + .
x y3 z5

Bi 3. Cho ba s thc x,y,z thuc on [ 0;1] v x + y + z =


2.
Tm gi tr ln nht v nh nht ca biu thc

P=

x2 4 x + 5 + y 2 4 y + 5 + z 2 4z + 5 .

Bi 4. Cho a,b,c l cc s thc dng. Chng minh rng

10a 2 + 10b 2 + c 2 4 ( ab + bc + ca ) .

Bi 5. Cho x,y,z l cc s thc khng m tho mn iu kin x 2 + y 2 + z 2 =


1.
Tm gi tr nh nht ca biu thc P =x 3 + 2 y 3 + 3 z 3 .
Bi 6. Cho x,y,z l cc s thc khng m tha mn x + y + z =
1.
Tm gi tr nh nht ca biu thc P = x 3 + y 3 +

1 3
z .
2

287

www.TaiLieuLuyenThi.com
Khm ph t duy K thut gii bt T Bi ton Max Min ng Thnh Nam

D. HNG DN GII P S
Bi 1. Cch 1: Ta cn chng minh

10 x 2 + 10 y 2 + z 2 4 ( xy + yz + zx )
12 x 2 + 12 y 2 + 3 z 2 2 ( x + y + z )

x2 y 2 z 2
2
+
+ 2( x + y + z)
1
1
1
12 12 3

Bt ng thc lun ng theo C-S, tht vy

x2 y 2 z 2 ( x + y + z )
2
+
+
= 2( x + y + z)
1
1
1
1 1 1
+ +
12 12 3 12 12 3
2

1 1 1
x= y=

x : y : z = : :

3
.
ng thc xy ra khi v ch khi
12 12 3
4
xy + yz + zx =

1
z=

3
Cch 2: S dng bt ng thc AM-GM ta c

2 x 2 + 2 y 2 4 xy
z2
4 xz
2
z2
8 y 2 + 4 yz
2
8x2 +

Cng theo v ba bt ng thc trn ta c

10 x 2 + 10 y 2 + z 2 4 ( xy + yz + zx ) =
4.
x = y
1

x= y=
4 x = z

3
ng thc xy ra khi v ch khi
.

4 y = z
z = 4
xy + yz + zx =
3
1
Bi 2. Gi s P t min ti=
3.
x a=
, y b=
, z c theo gi thit ta c a + b + c =
S dng bt ng thc AM-GM ta c

288

www.TaiLieuLuyenThi.com
Cty TNHH MTV DVVH Khang Vit

1 x 30
15 + 2 ;
x a a
1
y
y
y 20
5 3 + 4 + 4 + 4 3 ;
y b b b b
z
z
z
z 18
1 z
3 5 + 6 + 6 + 6 + 6 + 6 5 .
z c c c c c c
Cng theo v 3 bt ng thc trn ta c

15 5 3 15 x 15 y 15 z 30 20 18
+ + +
+ 4 + 6
+ + .
x y3 z5 a2
b
c
a b3 c 5
1
a
3
a + b + c =

Vy ta c h phng trnh 1
1
1
=
a=
2
4
b
c6
15 5 3
Suy ra P
+ + =
23 .
a b3 c 5

Ta chn cc s dng a,b,c tho mn =


2

1
1
.
=
4
b
c6

a = b = c =1 .

Bi 4. Cng thm vo hai v ca bt ng thc i lng 2 a 2 + b 2 + c 2 ta cn


chng minh: 12a 2 + 12b 2 + 3c 2 2 ( a + b + c ) .
2

S dng bt ng thc C-S ta c:

a 2 b2 c2 ( a + b + c )
2
+
+
= 2(a + b + c) .
1
1
1
1 1 1
+ +
12 12 3 12 12 3
1 1 1
1
ng thc xy ra khi v ch khi a : b : c = : : a = b = c .
12 12 3
4
2

12a 2 + 12b 2 + 3c 2 =

Bi 6. Phn tch tm im ri
Biu thc ca P i xng vi x,y nn du bng xy ra ti x= y= a, z= b . Khi
theo iu kin ta c: 2a + b =
1.
Khi p dng bt ng thc AM-GM ta c:

b 2 ( a 3 + a 3 + x 3 ) 3a 2b 2 x

b 2 ( a 3 + a 3 + y 3 ) 3a 2b 2 y
a 2 ( b3 + b3 + z 3 ) 3a 2b 2 z
289

www.TaiLieuLuyenThi.com
Khm ph t duy K thut gii bt T Bi ton Max Min ng Thnh Nam

Cng theo v 3 bt ng thc trn ta c:

b ( x3 + y 3 ) + a 2 z 3 + 4a 3b 2 + 2a 2b3 3a 2b 2 ( x + y + z ) =
3a 2b 2
2

b 2 ( x3 + y 3 ) + a 2 z 3 3a 2b 2 4a 3b 2 2a 2b3 (1)
Ta chn a,b khng m sao cho:

b2 a 2
=
b 2 = 2a 2 .
1
1
2

2 2
b 2 = 2a 2
a =
Ta c h phng trnh:

2 .
1
2a + b =
b
2 1
=
Li gii.
Thay a,b vo bt ng thc (1) ta c:

(3 2 2 ) ( x

+ y3 ) +

3 2 2 3 17 12 2
1
3 2 2
.
z
x3 + y 3 + z 3
2
2
2
2

ng thc xy ra khi v ch khi x= y=

2 2
, z=
2

2 1.

Cch 2: y l dng ton quen thuc s dng tnh n iu hm s.


Ta c:

P = ( x + y ) 3 xy ( x + y ) +
3

1 3
3
1
3
2
z ( x + y ) ( x + y ) ( x + y ) + z3
2
4
2

1
1
3
(1 z ) + z 3
4
2
1
1
3
Xt hm s f (z) = (1 z ) + z 3 lin tc trn on [ 0;1] ta c:
4
2
3
3
2
z[ 0;1]
z = 2 1.
f '( z ) = (1 z ) + z 2 ; f '(z) = 0
4
2
f (z) =

Ta c f(z) i du t m sang dng khi i qua=


z
ti=
z

3 2 2
.
2 1 hay P f ( z ) f ( 2 1) =
2

Vy gi tr nh nht ca P bng

290

2 1 nn f(z) t cc tiu

3 2 2
2 2
t ti x= y=
, z=
2
2

2 1.

www.TaiLieuLuyenThi.com
Cty TNHH MTV DVVH Khang Vit

CH 7: BT NG THC HOLDER V NG DNG


A. NI DUNG PHNG PHP
Di y ti trnh by bt ng thc Holder cho 3 dy s mi dy gm 3 s dng.
Cho a, b, c, x, y, z, m, n, p l cc s thc dng ta c

(a

+ b3 + c 3

)( x

)(

+ y 3 + z 3 m3 + n3 + p3 ( axm + byn + czp ) .


3

Chng minh. S dng bt ng thc AM GM ta c


a3

a 3 + b3 + c 3

(a

x3

m3

x 3 + y 3 + z 3 m3 + n 3 + p 3
3axm

+ b3 + c 3

)( x

)(

+ y 3 + z 3 m3 + n3 + p 3

Tng t ta c
b3
a 3 + b3 + c 3

(a

a 3 + b3 + c 3
3

(a

y3
x3 + y 3 + z 3
3byn

+ b3 + c 3

c3

)( x

x3 + y 3 + z 3
3czp

)( x

n3
m3 + n 3 + p 3

)(

+ y 3 + z 3 m3 + n 3 + p 3

z3

+ b3 + c 3

p3
m3 + n 3 + p 3

)(

+ y 3 + z 3 m3 + n 3 + p 3

Cng li theo v 3 bt ng thc trn ta c iu phi chng minh.


H qu 1. Cho a,b,c l cc s thc dng ta c

(1 + a )(1 + b )(1 + c ) (1 + 3 abc )

B. BI TP MU
Bi 1. Chng minh vi mi a,b,c l cc s thc dng ta c

)(

)(

2 a 2 + 1 b 2 + 1 c 2 + 1 ( a + 1)( b + 1)( c + 1)( abc + 1) .

Li gii
Nhn xt. Vi a= b= c bt ng thc tr thnh

) (
3

2 a 2 + 1 a3 + 1 ( a + 1) ( a 1) a 2 + a + 1 0 (lun ng).
3

Vy ta c
291

www.TaiLieuLuyenThi.com
Khm ph t duy K thut gii bt T Bi ton Max Min ng Thnh Nam

( )
2 ( b + 1)
2 ( c + 1)

( )
( b + 1) ( b + 1)
( c + 1) ( c + 1)

2 a 2 + 1 ( a + 1) a3 + 1
2

Nhn theo v 3 bt ng thc trn ta c

) (b + 1) ( c + 1)

8 a2 + 1

)(

)(

( a + 1) ( b + 1) ( c + 1) a3 + 1 b3 + 1 c3 + 1 .
3

)(

)(

Vy ta ch cn chng minh a3 + 1 b3 + 1 c3 + 1 (1 + abc ) .


3

y chnh l bt ng thc Holder. Bt ng thc c chng minh ng thc


xy ra khi v ch khi a= b= c= 1 .
Bi 2. Chng minh vi mi a,b,c l cc s thc dng ta c
a
b
c
+
+
3.
2b 2 + 2c 2 a 2
2c 2 + 2a 2 b 2
2a 2 + 2b 2 c 2
Li gii
Gi P l biu thc v tri v t

) (

) (

S= a 2b 2 + 2c 2 a 2 + b 2c 2 + 2a 2 b 2 + c 2a 2 + 2b 2 c 2 .

S dng bt ng thc Holder ta c P.P.S ( a + b + c ) P


3

3
a + b + c)
(
.

Vy ta chng minh

( a + b + c )3 3

( a + b + c ) 3 a 2b + 2c a

) + b ( 2c

) (

+ 2a 2 b 2 + c 2a 2 + 2b 2 c 2

4 a3 + b3 + c3 + 6abc 4 ab ( a + b ) + bc ( b + c ) + ca ( c + a )

Ch a3 + b3 + c3 + 3abc ab ( a + b ) + bc ( b + c ) + ca ( c + a )
a3 + b3 + c3 3abc
Cng theo v 2 bt ng thc trn ta c iu phi chng minh. ng thc xy ra
khi v ch khi a= b= c .
Bi 3. Cho a,b,c l di 3 cnh mt tam gic tho mn iu kin

Chng minh rng


292

a 2 + b2 + c2 =
3.
a
b
c
+
+
3.
b+ca
c+ab
a+bc

www.TaiLieuLuyenThi.com
Cty TNHH MTV DVVH Khang Vit

Li gii
Gi P l biu thc v tri v t S= a ( b + c a ) + b ( c + a b ) + c ( a + b c ) .
S dng bt ng thc Holder ta c P.P.S ( a + b + c ) .
3

Vy ta cn chng minh ( a + b + c ) 9 S
3

( a + b + c ) 9 2 ( ab + bc + ca ) 3 .
3

t x = a + b + c, x 3;3 ta c 2 ( ab + bc + ca ) = x 2 3 ta cn chng minh

x3 9 x 2 6 ( x 3) x 2 6 x 18 0, x 3;3 .
Bt ng thc c chng minh. ng thc xy ra khi v ch khi a= b= c= 1 .
Bi 4. Cho a,b,c l cc s thc dng c tch bng 1.
a
b
c
Chng minh rng
+
+
1.
b+c+7
c+a+7
a+b+7
Li gii

Gi P l biu thc v tri v t S = a ( b + c + 7 ) + b ( c + a + 7 ) + c ( a + b + 7 ) .


S dng bt ng thc Holder ta c P.P.S ( a + b + c ) .
3

Vy ta cn chng minh ( a + b + c ) S
3

( a + b + c ) 2 ( ab + bc + ca ) + 7 ( a + b + c ) .
3

Ta c

2
a + b + c)
(
.
ab + bc + ca

Vy ta chng minh ( a + b + c )
3

2
( a + b + c )2 + 7 ( a + b + c )
3

3 ( a + b + c ) 2 ( a + b + c ) 21 0
2

( a + b + c 3) 3 ( a + b + c ) + 7 0

Bt ng thc cui lun ng do a + b + c 3 3 abc =


3.
Bt ng thc c chng minh. ng thc xy ra khi v ch khi a= b= c= 1 .
Bi 5. Cho x,y,z l di 3 cnh mt tam gic.
1
1
1
x+ y+z
Chng minh rng
.
+
+

x+ yz
y+zx
z+x y
xyz
Li gii
Gi P l biu thc v tri v t =
S x3 ( y + z x ) + y 3 ( z + x y ) + z 3 ( x + y z ) .
293

www.TaiLieuLuyenThi.com
Khm ph t duy K thut gii bt T Bi ton Max Min ng Thnh Nam

S dng bt ng thc Holder ta c P.P.S ( x + y + z ) .


3

Vy ta cn chng minh

( x + y + z )3 x + y + z

xyz

xyz ( x + y + z ) x3 ( y + z x ) + y 3 ( z + x y ) + z 3 ( x + y z )

x 4 + y 4 + z 4 + xyz ( x + y + z ) xy x 2 + y 2 + yz y 2 + z 2 + zx z 2 + x 2

Bt ng thc cui lun ng(xem thm ch bin i tng ng).


Bt ng thc c chng minh. ng thc xy ra khi v ch khi x= y= z .
Bi 6. Cho a,b,c l cc s thc khng m tho mn ab + bc + ca > 0 .
a
b
c
+
+
2.
b+c
c+a
a+b
Li gii
Nhn xt. Bt ng thc trn c chng minh n gin bng bt ng thc
AM GM di y ta tip cn bi ton theo bt ng thc Holder.

Chng minh rng

Gi P l biu thc v tri v t S= a 2 ( b + c ) + b 2 ( c + a ) + c 2 ( a + b ) .


S dng bt ng thc Holder ta c P.P.S ( a + b + c ) .
3

Vy ta cn chng minh ( a + b + c ) 4 S
3

a3 + b3 + c3 + 6abc ab ( a + b ) + bc ( b + c ) + ca ( c + a )

Bt ng thc cui lun ng do


a3 + b3 + c3 + 3abc ab ( a + b ) + bc ( b + c ) + ca ( c + a )
3abc 0

Bt ng thc c chng minh. ng thc xy ra khi v ch khi c 1 s bng 0


v 2 s cn li bng nhau.
Bi 7. Cho a,b,c l cc s thc dng tho mn iu kin a 2 + b 2 + c 2 =a + b + c .
Chng minh rng a 2b 2 + b 2 c 2 + c 2 a 2 ab + bc + ca .
Li gii
Ta c

(a

+ b2 + c2

( a + b + c )2

2 ab + bc + ca a 2b 2 b 2 c 2 c 2 a 2 = a 4 + b 4 + c 4 a 2 b 2 c 2

Vy ta chng minh a 4 + b 4 + c 4 a 2 + b 2 + c 2 .
S dng bt ng thc Holder ta c
294

www.TaiLieuLuyenThi.com
Cty TNHH MTV DVVH Khang Vit

( a + b + c )2 ( a 4 + b 4 + c 4 ) ( a 2 + b 2 + c 2 )

a 4 + b4 + c4 a 2 + b2 + c2 .

Bt ng thc c chng minh. ng thc xy ra khi v ch khi a= b= c= 1 .


Bi 8. Cho a,b,c l cc s thc dng tho mn iu kin a 2 + b 2 + c 2 =
3.
a5

Chng minh rng

a3 + 2bc

b5
b3 + 2ca
Li gii

c5
c3 + 2ab

3.

) (

) (

Gi P l biu thc v tri v t S = a a3 + 2bc + b b3 + 2ca + c c3 + 2ab .

S dng bt ng thc Holder ta c P 2 S a 2 + b 2 + c 2


Vy ta ch cn chng minh

) (

) (
+ 6abc ( a + b

=
27 .

a a3 + 2bc + b b3 + 2ca + c c3 + 2ab 9


a 4 + b4 + c4

+ c2

a 2b 2 + b 2 c 2 + c 2 a 2 3abc
Bt ng thc cui ng bi v theo AM GM ta c

a 2b 2 + b 2 c 2 + c 2 a 2 3a 2b 2 c 2 a 2 + b 2 + c 2 =
3abc .

Bt ng thc c chng minh. ng thc xy ra khi v ch khi a= b= c= 1 .


C. BI TP RN LUYN
Bi 1. Chng minh vi mi a,b,c l cc s thc dng ta c

3 a 3 + b3 + c 3

) (a
2

+ b2 + c2

Bi 2. Cho a,b,c l cc s thc dng c tng bng 1.


a
b
c
3
.
+
+

2
b+c
c+a
a+b
Bi 3. Chng minh vi mi a,b,c l cc s thc dng ta c
a
b
c
+
+
1.
2
2
2
a + 8bc
b + 8ca
c + 8ab
Bi 4. Cho a,b,c l cc s thc dng c tch bng 1.
a
b
c
Chng minh rng
+
+
1.
2
2
2
2
2
b +c +7
c +a +7
a + b2 + 7
Bi 5. Cho a,b,c l cc s thc khng m c tng bng 3.

Chng minh rng

295

www.TaiLieuLuyenThi.com
Khm ph t duy K thut gii bt T Bi ton Max Min ng Thnh Nam

a
b
c
+
+
3.
1 + b + bc
1 + c + ca
1 + a + ab

Chng minh rng

Bi 6. Cho a,b,c l cc s thc dng chng minh rng

(a

)(

)(

a 2 + 3 b5 b 2 + 3 c 5 c 2 + 3 ( a + b + c ) .
3

Bi 7. Cho x,y,z l cc s thc dng tho mn iu kin x + y + z = xy + yz + zx .


Chng minh rng ( x + y + z )

x+ y+ z

27 .

Bi 8. Cho a,b,c l cc s thc dng c tng bng 3.


Chng minh rng a + b + c ab + bc + ca .
Bi 9. Cho x,y,z l cc s thc dng.
x2 y 2 z 2
x4 + y 4 + z 4
.
+
+
34
y
z
x
3

Chng minh rng

Bi 10. Cho a,b,c l cc s thc dng. Chng minh rng

(1 + a )(1 + b )(1 + c ) (1 + ab )(1 + bc )(1 + ca ) .


3

Bi 11. Cho a,b,c,x,y,z l cc s thc dng. Chng minh


a 3 b3 c 3 ( a + b + c )
.
+ +
x
y
z 3( x + y + z )
3

Bi 12. Cho a,b,c l cc s thc dng.


1
1
1
3
Chng minh rng
.
+
+

3
3
3
(1 + a ) (1 + b ) (1 + c ) 4 (1 + abc )
Bi 13. Cho a,b,c l cc s thc dng c tng bng 1. Chng minh
a
b
c
+3
+3
1.
3
a + 2b
b + 2c
c + 2a
Bi 14. Cho a,b,c l cc s thc dng. Chng minh

)(

)(

) (

9 a 4 + 1 b 4 + 1 c 4 + 1 8 a 2b 2 c 2 + abc + 1 .

Bi 15. Cho a,b,c l cc s thc dng. Chng minh


a+b
b+c
c+a
a+b+c
.
+
+
4(a + b + c)
c
a
b
3 ( a + b )( b + c )( c + a )

Bi 16. Cho a,b,c l cc s thc dng. Chng minh


a
b
c
27
.
+
+

3
3
2
(b + c)
(a + c)
( a + b)
8(a + b + c) 2
296

www.TaiLieuLuyenThi.com
Cty TNHH MTV DVVH Khang Vit

Bi 17. Cho a,b,c l cc s thc dng. Chng minh

( a + b )3
+
8ab ( 4a + 4b + c )

( b + c )3
+
8bc ( 4b + 4c + a )

( c + a )3
1.
8ca ( 4c + 4a + b )

Bi 18. Cho a,b,c l cc s thc khng m. Chng minh


a
b
c
+
+
1.
2
2
2
2
2
2
b
+
c
+
5
c
c
+
a
+
5
a
a
+
b
+
5
b
(
)
(
)
(
)
Bi 19. Cho a,b,c l cc s thc dng tho mn iu kin ab + bc + ca =
1.

1
1
1
Chng minh rng abc 3 6a + + 3 6b + + 3 6c + 1 .
c
a
b

Bi 20. Cho a,b,c l cc s thc dng tho mn iu kin

a 3 + b3 + c 3 = a 4 + b 4 + c 4 .
a
b
c
Chng minh rng 2
+ 3
+ 3
1.
3
3
2
3
a +b +c
a +b +c
a + b3 + c 2

Bi 21. Cho a,b,c l cc s thc dng tho mn iu kin


1
1
1
a + b + c= 2 + 2 + 2 .
a
b
c
Chng minh rng

3
3
7 a 2b + 1 + 7b 2 c + 1 + 7c 2 a + 1 2(a + b + c) .

Bi 22. Cho a,b,c l cc s thc dng c tng bng 1. Chng minh


a
b
c
13
+ 1 ( ab + bc + ca ) .
4abc
+
+
2
2
2
4
( a + 1)
( b + 1) ( c + 1)

D. HNG DN GII P S
Bi 1. S dng bt ng thc Holder ta c

3 a 3 + b3 + c 3

=(1 + 1 + 1) a3 + b3 + c3

) (a
2

+ b2 + c2

).
3

Bt ng thc c chng minh. ng thc xy ra khi v ch khi a= b= c .


Bi 2. Gi P l biu thc v tri v t S = a ( b + c ) + b ( c + a ) + c ( a + b ) .
S dng bt ng thc Holder ta c P.P.S ( a + b + c ) =
1.
3

Mt khc S = 2 ( ab + bc + ca )

2
2
3
( a + b + c )2 = P .
3
3
2

Bt ng thc c chng minh. ng thc xy ra khi v ch khi a= b= c=

1
.
3
297

www.TaiLieuLuyenThi.com
Khm ph t duy K thut gii bt T Bi ton Max Min ng Thnh Nam

Bi 3. Gi P l biu thc v tri v t

) (

) (

S = a a 2 + 8bc + b b 2 + 8ca + c c 2 + 8ab .

S dng bt ng thc Holder ta c P.P.S ( a + b + c )

Vy ta cn chng minh ( a + b + c ) S
3

) (

) (

( a + b + c ) a a 2 + 8bc + b b 2 + 8ca + c c 2 + 8ab


3

( a + b )( b + c )( c + a ) 8abc

Bt ng thc hin nhin ng theo AM GM. ng thc xy ra khi v ch khi


a= b= c .
Bi 4. Gi P l biu thc v tri v t

) (

) (

S= a b 2 + c 2 + 7 + b c 2 + a 2 + 7 + c a 2 + b 2 + 7 .

S dng bt ng thc Holder ta c P.P.S ( a + b + c ) .


3

Vy ta cn chng minh ( a + b + c ) S
3

( a + b + c ) 7 ( a + b + c ) + ab ( a + b ) + bc ( b + c ) + ca ( c + a )
3

( a + b + c ) 7 ( a + b + c ) + ( a + b + c )( ab + bc + ca ) 3abc
3

( a + b + c ) 7 ( a + b + c ) + ( a + b + c )( ab + bc + ca ) 3
3

Lun ng. Bt ng thc c chng minh. ng thc xy ra khi v ch khi


a= b= c= 1 .
Bi 5. Gi P l biu thc v tri v t
S= a (1 + b + bc ) + b (1 + c + ca ) + c (1 + a + ab ) .
3

2
2
2
S dng bt ng thc Holder ta c P.P.S a 3 + b 3 + c 3 .

2
2
2
Vy ta cn chng minh a 3 + b 3 + c 3 3 ( 3 + ab + bc + ca + 3abc )

2 2 2
2
2
a 2 + 3 a 3 b 3 a 3 + b 3 + 6 ( abc ) 3 9 + 3 ( ab + bc + ca ) + 9abc .

S dng bt ng thc AM GM ta c

298

2 2 2
2
3 b 3 a 3 + b 3 2 ( ab + bc + ca ) .
a

www.TaiLieuLuyenThi.com
Cty TNHH MTV DVVH Khang Vit
2

Vy ta cn chng minh ab + bc + ca + 6 ( abc ) 3 9abc .


Bt ng thc cui lun ng do
3

a+b+c
1.
ab + bc + ca + 6 ( abc ) 9 ( abc ) 9abc v abc
=
3

Bt ng thc c chng minh. ng thc xy ra khi v ch khi


a= b= c= 1 hoc a= 3, b= c= 0 v cc hon v.
2
3

2
3

Bi 6. Ch a5 a 2 + 3 a3 + 2 =

( a 1)2 ( a + 1) ( a 2 + a + 1) 0 .

Thit lp tng t v ta chng minh

(a

)(

)(

+ 1 + 1 1 + b3 + 1 1 + 1 + c 3 ( a + b + c ) .
3

y chnh l bt ng thc Holder.


Bi 7. S dng bt ng thc Holder ta c

(x

+ y2 + z2

Vy ta cn chng minh

)(

x+ y+ z

( x + y + z )4
x2 + y 2 + z 2

( x + y + z) .
3

27

( x + y + z ) 27 ( x + y + z )( x + y + z 2 )
4

( x + y + z ) + 54 27 ( x + y + z )
3

( x + y + z 3)

( x + y + z + 6) 0

Bt ng thc c chng minh. ng thc xy ra khi v ch khi x= y= z= 1 .


Bi 8. S dng bt ng thc Holder ta c

(a

+ b2 + c2

)(

a+ b+ c

(a + b + c) =
27 .
3

Vy ta cn chng minh 27 ( ab + bc + ca ) a 2 + b 2 + c 2 .
2

Bt ng thc cui ng theo AM GM


3

2 ( ab + bc + ca ) + a 2 + b 2 + c 2
27 .
=
( ab + bc + ca ) a + b + c

Bt ng thc c chng minh. ng thc xy ra khi v ch khi a= b= c= 1 .


2

Bi 9. Gi P l biu thc v tri v t S = x 2 y 2 + y 2 z 2 + z 2 x 2 .

S dng bt ng thc Holder ta c P.P.S x 2 + y 2 + z 2 .


Ta cn chng minh
299

www.TaiLieuLuyenThi.com
Khm ph t duy K thut gii bt T Bi ton Max Min ng Thnh Nam

(x

+ y2 + z2

x2 y 2 + y 2 z 2 + z 2 x2

x2 + y 2 + z 2

x4 + y 4 + z 4
3

3 x2 y 2 + y 2 z 2 + z 2 x2

) 3( x

+ y4 + z4

t a = x 2 + y 2 + z 2 , b = x 2 y 2 + y 2 z 2 + z 2 x 2 x 4 + y 4 + z 4 = a 2 2b .

Bt ng thc tr thnh a3 3b 3 a 2 2b

a 6 27 a 2b + 54b3 0 a 2 3b

) (a
2

+ 6b 0 .

Lun ng. Bt ng thc c chng minh. ng thc xy ra khi v ch khi


x= y= z .
Bi tp tng t
Chng minh x,y,z l cc s thc dng tho mn x 4 + y 4 + z 4 =
3 ta c
x2 y 2 z 2
+
+
3.
y
z
x

Bi 10. S dng bt ng thc Holder ta c

(1 + a )(1 + b )(1 + b ) (1 + ab )
(1 + b )(1 + c )(1 + c ) (1 + bc ) .
(1 + c )(1 + a )(1 + a ) (1 + ca )
3

2 3

2 3

2 3

Nhn theo v 3 bt ng thc trn ta c iu phi chng minh.


Bi 11. S dng bt ng thc Holder ta c
a3

(1 + 1 + 1)( x + y + z )

b3 c 3
3
+ (a + b + c)

y
z

a 3 b3 c 3 ( a + b + c )

+ +
x
y
z 3( x + y + z )
3

Bt ng thc c chng minh.


Bi 12. Bt ng thc tng ng vi:
1 + abc 1 + abc 1 + abc 3
+
+
.
(1 + a )3 (1 + b )3 (1 + c )3 4
S dng bt ng thc Holder ta c:
300

www.TaiLieuLuyenThi.com
Cty TNHH MTV DVVH Khang Vit

(1 + abc ) 1 +

Tng t:

a a
1 + abc
bc
3
.

1 + (1 + a )
3
b c
(1 + a ) ( a + b )( a + c )

1 + abc

(1 + b )

ac
1 + abc
ab
.
;

3
b
+
a
b
+
c
c
+
a
(
)(
) (1 + c ) (
)( c + b )

Cng theo v 3 bt ng thc trn v cn chng minh


bc
ac
ab
3
+
+
.
( a + b )( a + c ) ( b + a )( b + c ) ( c + a )( c + b ) 4
3
( a + b )( b + c )( c + a ) .
4
ab ( a + b ) + bc ( b + c ) + ca ( c + a ) 6abc .

ab ( a + b ) + bc ( b + c ) + ca ( c + a )

Bt ng thc cui theo AM-GM.


Bi ton c chng minh ng thc xy ra khi v ch khi a= b= c .
Bi 13. Gi P l biu thc v tri v t
S =a ( a + 2b ) + b ( b + 2c ) + c ( c + 2a ) =( a + b + c ) =1 .
2

S dng bt ng thc Holder ta c P.P.P.S ( a + b + c ) P3 1 P 1 .


4

Bt ng thc c chng minh ng thc xy ra khi v ch khi a= b= c=

1
.
3

Bi 14. Vi a= b= c bt ng thc tr thnh

9 a 4 + 1 8 a 6 + a3 + 1

3
2.
1
1

9 a 2 + 2 8 a 3 + 3 + 1
a
a

1
t x =
a + , ( x 2 ) bt ng thc tr thnh
a

9 x2 2

8 x3 3 x + 1

( x 2)

( x ( x 8) + 4 ( x 5) + 6 x ) 0 .
3

Bt ng thc lun ng vi x 2 .

( )
9 ( b + 1)
9 ( c + 1)

(
8(b
8(c

)
+ b + 1)
+ c + 1)

p dng ta c 9 a 4 + 1 8 a 6 + a3 + 1
4

Nhn theo v 3 bt ng thc trn ta c


301

www.TaiLieuLuyenThi.com
Khm ph t duy K thut gii bt T Bi ton Max Min ng Thnh Nam

) (b + 1) ( c + 1)

93 a 4 + 1

) (b
2

83 a 6 + a3 + 1

) (c

+ b3 + 1

+ c3 + 1 .

Mt khc theo bt ng thc Holder ta c

(a

)(

)(

) (

+ a3 + 1 b6 + b3 + 1 c 6 + c3 + 1 a 2b 2 c 2 + abc + 1 .

T ta c iu phi chng minh.


ng thc xy ra khi v ch khi a= b= c= 1 .
Bi 15. Gi P l biu thc v tri v t S = c ( a + b ) + b ( c + a ) + a ( b + c ) .
2

S dng bt ng thc Holder ta c P.P.S ( a + b + b + c + c + a ) = 8 ( a + b + c ) .


3

Vy ta cn chng minh
8( a + b + c)

16 ( a + b + c )

a (b + c ) + b (c + a ) + c ( a + b)
2

3 ( a + b )( b + c )( c + a )

2
2
2
3 ( a + b )( b + c )( c + a ) 2 a ( b + c ) + b ( c + a ) + c ( a + b )

.
( a + b + c )( ab + bc + ca ) 9abc

Lun ng theo AM GM.


Bi 16. Gi P l biu thc v tri v t S = a + b + c .
S dng bt ng thc Holder ta c
3

b
c 3
27
a
.
P.S .S
+
+
P
2
b+c c+a a+b 2
8( a + b + c)

Bt ng thc c chng minh. ng thc xy ra khi v ch khi a= b= c .


Bi 17. Gi P l biu thc v tri v t
=
S 8ab ( 4a + 4b + c ) + 8bc ( 4b + 4c + a ) + 8ca ( 4c + 4a + b )
= 32 ( a + b + c )( ab + bc + ca ) 72abc

S dng bt ng thc Holder ta c


P.P.S ( a + b + b + c + c + a ) = 8 ( a + b + c ) .
3

Vy ta chng minh 8 ( a + b + c ) S
3

8 ( a + b + c ) 32 ( a + b + c )( ab + bc + ca ) 72abc
3

( a + b + c ) + 9abc 4 ( a + b + c )( ab + bc + ca )
3

y chnh l bt ng thc Schur bc 3. Bt ng thc c chng minh. ng


thc xy ra khi v ch khi a= b= c .
Bi 18. Gi P l biu thc v tri v t
302

www.TaiLieuLuyenThi.com
Cty TNHH MTV DVVH Khang Vit
2
2
2
S = a ( b + c ) + 5c 2 + b ( c + a ) + 5a 2 + c ( a + b ) + 5b 2 .

S dng bt ng thc Holder ta c P 2 .S ( a + b + c ) .


3

Vy ta cn chng minh

( a + b + c )3 a ( b + c )2 + 5c 2 + b ( c + a )2 + 5a 2 + c ( a + b )2 + 5b2
a ( a b) + b (b c ) + c (c a ) 0
2

Bt ng thc cui lun ng ta c pcm. ng thc xy ra khi v ch khi


a= b= c .
Bi 19. S dng bt ng thc Holder ta c
3

6a +

1 3
1
1
+ 6b + + 3 6c=
+
c
a
b

1
1
1
( 6ab + 1) + 3 ( 6bc + 1) + 3 ( 6ca + 1)
a
b
c

1 1 1
3 3 + + ( 6ab + 1 + 6bc + 1 + 6ca +=
1)
a b c

3
3

abc

1
abc

1
ab + bc + ca
V theo AM GM ta c abc
.
=
3
3 3

Bt ng thc c chng minh. ng thc xy ra khi v ch khi a= b= c=

1
.
3

Bi 20. S dng bt ng thc C S ta c


a
b
c
+ 3
+ 3
2
3
3
2
3
a +b +c
a +b +c
a + b3 + c 2
=

a2
a3 + ab3 + ac3

b2
a3b + b3 + bc3

c2
ca3 + cb3 + c3

( a + b + c )2

a3 + b3 + c3 + ab a 2 + b 2 + bc b 2 + c 2 + ca c 2 + a 2

( a + b + c )2

a 4 + b 4 + c 4 + ab a 2 + b 2 + bc b 2 + c 2 + ca c 2 + a 2
2
a + b + c)
(
=
( a + b + c ) ( a 3 + b3 + c 3 )

a+b+c
a + b3 + c 3
3

Ta chng minh a + b + c a3 + b3 + c3

( a + b + c ) a 4 + b4 + c4

) (a
2

+ b3 + c 3

ng theo bt ng thc Holder.


303

www.TaiLieuLuyenThi.com
Khm ph t duy K thut gii bt T Bi ton Max Min ng Thnh Nam

Bi 21. i P l biu thc v tri s dng bt ng thc Holder ta c


7 a 2b + 1

7b 2 c + 1
7c 2 a + 1
3
3

.
.
.
.
.
.
P = 3
a
a
+
b
b
+
c
c
2
2
2

a
b
c

7 a 2 b + 1 7b 2 c + 1 7 c 2 a + 1
3
2

+
+
(a + b + c) = 8 ( a + b + c )
2
2
a2
b
c

Bt ng thc c chng minh. ng thc xy ra khi v ch khi a= b= c= 1 .


Bi 22. S dng bt ng thc Holder ta c
3
a + b + c)
(
1
.
+
+

=
2
2
2
2
( a + 1) ( b + 1) ( c + 1) ( a ( a + 1) + b ( b + 1) + c ( c + 1) ) ( t + 1)2

Vi t = a 2 + b 2 + c 2

1
1
( a + b + c )2 = .
3
3

p2 t 1 t
=
; r = abc .
2
2
Mt khc theo bt ng thc Schur bc ba ta c
abc ( a + b c )( b + c a )( c + a b )

t p = a + b + c = 1; q = ab + bc + ca =

abc (1 2a )(1 2b )(1 2c ) 9r + 1 4q 0 r

4q 1 1 2t .
=
9
9

Vy ta ch cn chng minh
1 2t
1
13
4.
.
+ 1 (1 t ) ( 3t 1) 39t 2 + 76t + 13 0 .
2
9 ( t + 1)
8

Bt ng thc cui ng ta c pcm. ng thc xy ra khi v ch khi a= b= c=


CH 8: K THUT S DNG BT NG THC CHEBYSHEV
A. NI DUNG PHNG PHP
1. Bt ng thc Chebyshev vi hai dy n iu cng chiu
a1 a2 ... an
a1 a2 ... an
Nu c
hoc
th ta c
b1 b2 ... bn
b1 b2 ... bn
n ( a1b1 + a2b2 + ... + an bn ) ( a1 + a2 + ... + an )( b1 + b2 + ... + bn ) .

a1= a2= ...= an


ng thc xy ra khi v ch khi
.
b1= b2= ...= bn

Chng minh. Ta c ng thc


304

1
.
3

www.TaiLieuLuyenThi.com
Cty TNHH MTV DVVH Khang Vit

n ( a1b1 + a2b2 + ... + anbn ) ( a1 + a2 + ... + an )( b1 + b2 + ... +=


bn )

)(

( ai a j )( bi b j ) 0 .

1i j

V ai a j bi b j 0, i, j =
1, n .
2. Bt ng thc Chebyshev vi hai dy n iu ngc chiu
a1 a2 ... an
a1 a2 ... an
hoc
th ta c
Nu c
b1 b2 ... bn
b1 b2 ... bn
n ( a1b1 + a2b2 + ... + an bn ) ( a1 + a2 + ... + an )( b1 + b2 + ... + bn ) .

a1= a2= ...= an


ng thc xy ra khi v ch khi
.
b1= b2= ...= bn
Chng minh tng t cho dy n iu cng chiu.
Bt ng thc Chebyshev c p dng cho cc bt ng thc i xng ba
bin v bi ton c gii theo cch ny kh n gin.

B. BI TP MU
Bi 1. Cho a,b,c l cc s thc dng tho mn iu kin a 2 + b 2 + c 2 =
1.
a3
b3
c3
1
+
+
.
b+c a+c a+b 2
Li gii
Do a,b,c i xng, khng mt tnh tng qut gi s a b c

Chng minh rng

a 2 b2 c2

a
b
c tc l hai dy n iu cng chiu.

b + c a + c a + b
p dng bt ng thc Chebyshev ta c :

a2 .

a
b
c
a 2 + b2 + c2 a
b
c 1 3 1
+ b2 .
+ c2 .

+
+
.
. =
b+c
a+c
a+b
3
b+c a+c a+b 3 2 2

Bt ng thc c chng minh. Du bng xy ra khi a = b = c=

1
3

Ch . Ta c th chng minh bng C S

a 2 + b2 + c2
a3
b3
c3
+
+

b + c a + c a + b a (b + c ) + b (c + a ) + c ( a + b)
=

1
1
1

=
2
2
2
2 ( ab + bc + ca ) 2 a + b + c
2

305

www.TaiLieuLuyenThi.com
Khm ph t duy K thut gii bt T Bi ton Max Min ng Thnh Nam

Bi 2. Chng minh vi mi s thc dng x,y,z ta c


1
1
1
x+ y+z
3
.
+
+

+
x + y y + z z + x xy + yz + x 2 ( x + y + z )
Li gii
Bt ng thc cho tng ng vi:

3 ( xy + yz + zx )
1
1
1
+
+
x+ y+z+
2( x + y + z )
x+ y y+z z+x

( xy + yz + zx )

3 ( xy + yz + zx )
xy
yz
zx
+
+

x+ y y+z z+x
2( x + y + z )

1
1
1
3 ( xy + yz + zx )
( x + y ) + ( y + z ) + ( z + x ) .
+
+
1+1 1+1 1+1
x y y z z x

Khng mt tnh tng qut gi s


x y z x + y x + z y + z;

1
1 1
+
x y

1 1
+
x z

1 1
+
z y

S dng bt ng thc cho hai dy n iu cng chiu trn ta c ngay iu


phi chng minh.
Bt ng thc xy ra khi v ch khi x= y= z .
Bi 3. Cho x,y,z l cc s thc c tng bng 1. Chng minh
x
y
z
9
+ 2
+ 2
.
2
x + 1 y + 1 z + 1 10
Li gii
Trc ht chuyn bi ton v nh gi vi cc s dng
x
x +1
2

y
y +1
2

z
z +1
2

x
x +1
2

y
y +1
2

z
z +1
2

Nn ta ch cn chng minh bt ng thc trong trng hp c ba s u khng m.


V bt ng thc i xng vi ba bin nn khng mt tnh tng qut gi s
x y z khi
x 2 + 1 y 2 + 1 z 2 + 1;

x
x +1
2

y
y +1
2

z
z +1
2

Tht vy ta ch cn chng minh


x
y
2
xy 2 + x x 2 y + y ( x y )(1 xy ) 0 .
2
x +1 y +1
306

www.TaiLieuLuyenThi.com
Cty TNHH MTV DVVH Khang Vit
2

1 3
x+ y
Bt ng thc ng do x y 0; 1 xy 1
1 = > 0 .
4 4
2
Vy ta c hai dy n iu cng chiu
x
y
z
x 2 + 1; y 2 + 1; z 2 + 1 ; 2
; 2
; 2
.
x +1 y +1 z +1
p dng bt ng thc Chebyshev ta c
x
y
z
+ 2
+ 2
x 2 + 1 + y 2 + 1 + z 2 + 1 2

x +1 y +1 z +1

x
y
z
3 x2 + 1 . 2
+ y2 + 1 . 2
+ z2 + 1 . 2
x +1
y +1
z + 1

3( x + y + z )
x
y
z
2
+ 2
+ 2
2
x + 1 y + 1 z + 1 x + y2 + z2 + 3

Vy ta chng minh

3( x + y + z )

9
1
x2 + y 2 + z 2 .
3
x + y + z + 3 10
2

Bt ng thc ng v x 2 + y 2 + z 2

1
1
( x + y + z )2 = .
3
3

1
.
3
1 1 1
Bi 3. Cho a,b,c l cc s thc dng tho mn iu kin a + b + c + + .
a b c

Bi ton c chng minh. ng thc xy ra khi v ch khi x= y= z=

Chng minh rng 2 ( a + b + c ) a 2 + 3 + b 2 + 3 + c 2 + 3 .


Li gii
Bt ng thc cho tng ng vi

( 2a

)(

)(

a 2 + 3 + 2b b 2 + 3 + 2c c 2 + 3 0

3 a2 1

2a + a 2 + 3

3 b2 1

2b + b 2 + 3

3 c2 1

2c + c 2 + 3

1 1 1
a 2 1 b2 1 c2 1
+ +
+
+
0.
a b c
a
b
c
Nn ta bin i bt ng thc v dng

Ch iu kin a + b + c
a2 1
a
3
2 + 1+ 2
a

b2 1
b
3
2 + 1+ 2
b

c2 1
c
3
2 + 1+ 2
c

0.

307

www.TaiLieuLuyenThi.com
Khm ph t duy K thut gii bt T Bi ton Max Min ng Thnh Nam

Khng mt tnh tng qut gi s a b c , khi


1
2 + 1+

3
a2

2 + 1+

3
b2

2 + 1+

3
c2

a 2 1 b2 1 c2 1

a
b
c
Khi s dng bt ng thc Chebyshev ta c
a2 1
a
3
2 + 1+ 2
a

b2 1
b
3
2 + 1+ 2
b

c2 1
c

2 + 1+

3
c2

1 a 1 b 1 c 1
1
1
1
0

+
+
+
+

3 a
b
c
3
3
3
2 + 1+ 2 2 + 1+ 2 2 + 1+ 2
a
b
c

Bt ng thc c chng minh. ng thc xy ra khi v ch khi a= b= c= 1 .


Bi tp tng t
1
1
1
Cho n s thc dng tho mn iu kin a1 + a2 + ... + an +
.
+ ... +
a1 a2
an
2

Chng minh rng 2 ( a1 + a2 + ... + an ) a12 + 3 + a22 + 3 + ... + an2 + 3 .


Bi 4. Cho x,y,z l cc s thc dng. Chng minh
x+ y
+
z

y+z
+
x

6 ( x + y + z)
z+x
.

y
xy + yz + zx

Li gii
Ta c

x+ y
+
z

y+z
+
x

z+x
=
y

x+ y
z ( x + y)

x+z
y( x + z)

y+z
x( y + z)

Khng mt tnh tng qut gi s x y z x + y x + z z + y v


x+ y
z ( x + y)

x+z
y( x + z)

y+z
x( y + z)

z ( x + y) y ( z + x) x ( y + z )

Khi s dng bt ng thc Chebyshev cho hai dy n iu ngc chiu nn


ta c

308

www.TaiLieuLuyenThi.com
Cty TNHH MTV DVVH Khang Vit

x+ y

+
z ( x + y)

z ( x + y) + y ( z + x) + x ( y + z )
y( x + z)
x ( y + z )
3 ( x + y + x + z + y + z )= 6 ( x + y + z )
x+z

y+z

Kt hp s dng bt ng thc C S ta c
x+ y
z ( x + y)

x+z

y+z

y( x + z)

x( y + z)

6( x + y + z )

z ( x + y) + y ( z + x) + x ( y + z )
6( x + y + z )

6 (x + y + z)
=
xy + yz + zx
6 ( xy + yz + zx )

Bt ng thc c chng minh. ng thc xy ra khi v ch khi x= y= z .


Bi tp tng t
Chng minh rng vi mi x,y,z dng ta c
8( x + y + z )
z+x
.

y
( x + y )( y + z )( z + x )
Bi 5. Cho a,b,c l cc s thc khng m chng minh
3 x+ y
+

2
z

y+z
+
x

3 ( a + b + c ) a 2 + 8bc + b 2 + 8ca + c 2 + 8ab .

Li gii
+ Nu tn ti mt s bng 0 bt ng thc c dng 2 ( x + y ) 2 2 xy lun ng.
+ Ta xt c ba s u dng khi vit li bt ng thc di dng

(3a

)(

8 a 2 bc

3a + a 2 + 8bc

)(

a 2 + 8bc + 3b b 2 + 8ca + 3c c 2 + 8ab 0

(a

8 b 2 ca

3b + b 2 + 8ca

bc ( b + c )

3a + a 2 + 8bc ( b + c )

(b

+
2

8 c 2 ab

3c + c 2 + 8ab

ca ( c + a )

3b + b 2 + 8ca ( c + a )

(c

ab ( a + b )

3c + c 2 + 8ab ( a + b )

Ch a 2 bc ( b + c ) + b 2 ca ( c + a ) + c 2 ab ( a + b ) =
0.
Do vy s dng bt ng thc Chebyshev ta ch cn chng minh hai dy sau
n iu ngc chiu

(3a +

(a

bc ( b + c ) , b 2 ca ( c + a ) , c 2 ab ( a + b )

a 2 + 8bc ( b + c ) , 3b + b 2 + 8ca ( c + a ) , 3c + c 2 + 8ab ( a + b )


309

www.TaiLieuLuyenThi.com
Khm ph t duy K thut gii bt T Bi ton Max Min ng Thnh Nam

Khng mt tnh tng qut gi s a b c ta c

(a
(b

)
(
ca ) ( c + a ) ( c

)
(
ab ) ( a + b ) = ( b c ) ( a

)
+ ab + bc + ca ) 0

bc ( b + c ) b 2 ca ( c + a ) =( a b ) c 2 + ab + bc + ca 0

V 3a + a 2 + 8bc ( b + c ) 3b + b 2 + 8ca ( c + a )
3c ( a b ) + ( b + c ) a 2 + 8bc ( c + a ) b 2 + 8ca 0
3c ( a b ) +

( b + c )2 ( a 2 + 8bc ) ( c + a )2 ( b2 + 8ca )
( b + c ) a 2 + 8bc + ( c + a ) b2 + 8ca

8 a 2 + b 2 + c 2 + 6ab + 15c ( a + b )

0
3c ( a b ) 1
( b + c ) a 2 + 8bc + ( c + a ) b 2 + 8ca

Bt ng thc cui ng v
a 2 + 8bc a + b + c; b 2 + 8ca a + b + c
( b + c ) a 2 + 8bc + ( c + a ) b 2 + 8ca ( a + b + c )( a + b + 2c )

< 8 a 2 + b 2 + c 2 + 6ab + 15c ( a + b )

Bt ng thc c chng minh. ng thc xy ra khi v ch khi a= b= c .


C. BI TP RN LUYN
Bi 1. Cho a,b,c l cc s thc dng tho mn iu kin a + b + c 3abc .
1
1
1
Chng minh rng
+
+
1.
1 + 2bc 1 + 2ca 1 + 2ab
1
1
1
Bi 2. Cho cc s thc a, b, c > 1 tho mn iu kin 2
1.
+ 2
+ 2
=
a 1 b 1 c 1
1
1
1
Chng minh rng
+
+
1.
a +1 b +1 c +1
Bi 3. Cho x,y,z l cc s thc dng c tng bng 1. Chng minh
x
y
z
27
.
+
+

2
2
2
xyz + x + 1 xyz + y + 1 xyz + z + 1 31
Bi 4. Cho a,b,c l cc s thc dng tho mn iu kin ab + bc + ca =
3.
1
1
1
3
.
Chng minh rng
+
+

2
2
2
1 + a ( b + c ) 1 + b ( c + a ) 1 + c ( a + b ) 1 + 2abc
310

www.TaiLieuLuyenThi.com
Cty TNHH MTV DVVH Khang Vit

Bi 5. Cho x,y,z l cc s thc dng. Chng minh


1+

16 x
16 y
16 z
+ 1+
+ 1+
9.
y+z
z+x
x+ y

D. HNG DN GII P N
Bi 1. Bt ng thc cho tng ng vi
a
b
c
+
+
1.
a + 2abc b + 2abc c + 2abc
Khng mt tnh tng qut gi s a b c. Khi ta c
a + 2abc b + 2abc c + 2abc
a
b
c
1
1
1

1 + 2bc 1 + 2ca 1 + 2ab


a + 2abc b + 2abc c + 2abc
S dng bt ng thc Chebyshev cho hai dy n iu cng chiu

( a + 2abc; b + 2abc; c + 2abc ) ;

a
b
c

;
;

a + 2abc b + 2abc c + 2abc

ta c
a
b
c

3 ( a + b + c ) ( a + b + c + 6abc )
+
+

+
+
+
a
abc
b
abc
c
abc
2
2
2

a
b
c
3
+
+
(
) 1
a
b
c

+
+

a + 2abc b + 2abc c + 2abc a + b + c + 6abc


Bi v a + b + c 3abc .
Bt ng thc c chng minh. ng thc xy ra khi v ch khi a= b= c= 1 .
Bi tp tng t
1
1
1
Cho a,b,c l cc s thc dng tho mn iu kin
+
+
1.
1 + 2bc 1 + 2ca 1 + 2ab
Chng minh rng a + b + c 3abc .
Bi 2. Bt ng thc cho tng ng vi
1
1
1
1
1
1
a2 b2 c2

+
+
0
+
+
0.
3 a +1 3 b +1 3 c +1
a +1 b +1 c +1
Ch iu kin c vit li thnh
1
1
1
1
1
1
0
+ 2
+ 2
=
2
a 1 3 b 1 3 c 1 3

4 a2

4 b2

4 c2

0
=
a 2 1 b2 1 c2 1
a2 a+2 b2 b+2 c2 c+2
.
.
.
0

+
+
=
a +1 a 1 b +1 b 1 c +1 c 1
311

www.TaiLieuLuyenThi.com
Khm ph t duy K thut gii bt T Bi ton Max Min ng Thnh Nam

a 2 b 2 c 2
a + 1 b + 1 c + 1
Khng mt tnh tng qut a b c
.
2
2
2
a
b
c
+
+
+

a 1 b 1 c 1
S dng bt ng thc Chebyshev cho hai dy n iu ngc chiu trn ta c
ngay pcm.
Bt ng thc c chng minh. ng thc xy ra khi v ch khi a= b= c= 2 .
Tng qut cho n s thc a1 , a2 ,..., an > 1 tho mn iu kin

1
a12

Chng minh rng

1
a22

+ ... +

1
an2

=
1.

1
1
1
+
+ ... +
n +1 1.
a1 + 1 a1 + 1
an + 1

Bi 3. Khng mt tnh tng qut gi s


xyz + x 2 + 1 xyz + y 2 + 1 xyz + z 2 + 1

x yz
x
y
z

2
2
2
xyz + x + 1 xyz + y + 1 xyz + z + 1

S dng bt ng thc Chebyshev cho hai dy n iu cng chiu trn ta c


3( x + y + z )
x
y
z
+
+

2
2
2
xyz + x + 1 xyz + y + 1 xyz + z + 1 3xyz + 3 + x 2 + y 2 + z 2
3
=
3 xyz + 3 + x 2 + y 2 + z 2
Do ta ch cn chng minh
3
3 xyz + 3 + x + y + z
2

27
4
3 xyz + x 2 + y 2 + z 2 .
31
9

Ch theo bt ng thc Schur bc ba ta c


9 xyz
2
4 ( xy + yz + zx ) ( x + y + z )
x+ y+z
4
1
3 xyz ( xy + yz + zx )
3
3
4
1
3 xyz + x 2 + y 2 + z 2 ( xy + yz + zx ) + x 2 + y 2 + z 2
3
3
1
2
3 ( x + y + z ) 2 ( xy + yz + zx ) 1 3 2. 3 1 4
=

3
3
9

312

www.TaiLieuLuyenThi.com
Cty TNHH MTV DVVH Khang Vit

1
.
3

Bt ng thc c chng minh. ng thc xy ra khi v ch khi x= y= z=

Bi 4. Bt ng thc cho tng ng vi


1
1
1
1
1
1

0
1 + 2abc 1 + a 2 ( b + c ) 1 + 2abc 1 + b 2 ( c + a ) 1 + 2abc 1 + c 2 ( a + b )

a ( ab + ac 2bc )
1 + a2 (b + c )
a (1 bc )

b ( bc + ab 2ca )
1 + b2 ( c + a )

b (1 ca )

c ( ca + bc 2ab )
1 + c2 ( a + b )

c (1 ab )

1 + a ( 3 bc ) 1 + b ( 3 ca ) 1 + c ( 3 ab )

1 bc
1 ca
1 ab
+
+
0
bc + abc ( 3 bc ) ca + abc ( 3 ca ) ab + abc ( 3 ab )

1 bc
1 ca
1 ab

bc + abc ( 3 bc ) ca + abc ( 3 ca ) ab + abc ( 3 ab )


Gi s a b c
.
bc + abc ( 3 bc ) ca + abc ( 3 ca ) ab + abc ( 3 ab )

S dng bt ng thc Chebyshev cho hai dy n iu ngc chiu trn ta c


ngay iu phi chng minh. ng thc xy ra khi v ch khi a= b= c= 1 .
Bi 5. Bt ng thc cho tng ng vi:

16 x
16 y
16 z
3 + 1 +
3 + 1 +
3 0
1 +
y+z
z+x
x+ y

2x

2z

2y

8
8
1
1
8
1
y+z
x+ y
z+x

+
+
0
16 x
16 y
16 z
1+
1+
1+
+3
+3
+3
y+z
z+x
x+ y

Khng mt tnh tng qut gi s


2y
2z
2x
y + z 1 z + x 1 x + y 1

1
1
1
x yz

16 x
16 y
16 z
1+
+3
1+
+3
1+
+3
y+z
z+x
x+ y

S dng bt ng thc Chebyshev cho hai dy n iu cng chiu trn v a


v chng minh
2x
2y
2z
x
y
z
3
1+
1+
1 0
+
+
.
y+z
z+x
x+ y
y+z z+x x+ y 2

313

www.TaiLieuLuyenThi.com
Khm ph t duy K thut gii bt T Bi ton Max Min ng Thnh Nam

Bt ng thc cui ng. Bi ton c chng minh. ng thc xy ra khi v


ch khi x= y= z .
Bi tp tng t
Cho x,y,z l cc s thc khng m tho mn iu kin xy + yz + zx > 0 .
Chng minh rng 1 +

48 x
48 y
48 z
+ 1+
+ 1+
15 .
y+z
z+x
x+ y

CH 9: BT NG THC BERNOULLI V NG DNG

A. NI DUNG PHNG PHP

Vi mi s thc a 1, 1 ta lun c (1 + a ) 1 + .a (1).


Chng minh.
+ Vi = 1 bt ng thc tr thnh ng thc.

+ Vi > 1 Xt hm s f (a ) = (1 + a ) 1 .a vi a 1 ta c
1
f '(a ) = ( a + 1) 1 ; f '(a ) = 0 a = 0 .

Ta c f(a) i du t m sang dng khi i qua a = 0 nn f(a) t cc tiu ti


a=0.

V vy f (a ) f (0) (1 + a ) 1 + .a .
Bt ng thc c chng minh. ng thc xy ra khi v ch khi a = 0 hoc
= 1.
Ch . Vi 0 < 1 bt ng thc i chiu.
Mt dng pht biu khc ca bt ng thc Bernoulli hay c s dng
a
a
x a xb + 1 vi x 0, a b > 0 .
b
b
Chng minh.

a
a b
a
x = 1 + xb 1 b 1 + xb 1 =
x +1 .

b
b
b
p dng ta chng minh c bt ng thc AM GM dng lu tha
a

a1m + a2m + ... + anm a1 + a2 + ... + an

.
n
n

Chng minh.
Vit li bt ng thc di dng

314

www.TaiLieuLuyenThi.com
Cty TNHH MTV DVVH Khang Vit
m

nan
na1
na2

+
+ ... +
n.
a1 + a2 + ... + an
a1 + a2 + ... + an
a1 + a2 + ... + an
Ch
m

n
n

( n 1) ak ai
( n 1) ak ai
m

nak

1 m.
1i k
1i k
, k = 1, n

= 1 +

+
+
+
+
+
+
+
+
+ an
...
...
...
a
a
a
a
a
a
a
a
n
n
1
2
1
2
1 2

Ly tng tt n bt ng thc trn ta c iu phi chng minh.


ng thc xy ra khi v ch khi a1= a2= ...= an .

Xt hai bt ng thc AM GM
a1 + a2 + ... + an n
a1a2 ...an .
n
Chng minh.
a1 + a2 + ... + an
t An =
=
; Gn n a1a2 ...an .
n
S dng bt ng thc Bernoulli ta c
An

A
nAn ( n 1) An 1
An
a
1 n + n n 1 =
= n

= 1 +
An 1
An 1
An 1 .
An 1
An 1
n

Ann an Ann11

S dng lin tip bt ng thc trn ta c


Ann an Ann11 an an 1 Ann22 ... an an 1...a2 . A1 = a1a2 ...an = Gnn An Gn .

Vy bt ng thc c chng minh.


Ta c mt dng suy rng ca Bernoulli hay c s dng nh sau
Vi n s thc cng du v ln hn hoc bng -1 ta c

(1 + a1 )(1 + a2 ) ...(1 + an ) 1 + a1 + a2 + ... + an (1).

Ch . (1) d chng minh bng quy np ton hc.

B. BI TP MU
Bi 1. Cho a, b l cc s thc thay i tha mn iu kin 0 < a, b < 1 .
Chng minh rng ab + b a > 1 .
Li gii
S dng bt ng thc Bernoulli ta c
b

a
1 a a + b ab
1 a
= 1 +
=
ab
1 + b.
b
a
a
a
a + b ab
a

315

www.TaiLieuLuyenThi.com
Khm ph t duy K thut gii bt T Bi ton Max Min ng Thnh Nam
a

1 b a + b ab
b
1 b
.
= 1 +
=
ba
1 + a.
a
b
b
b
a + b ab
b

a+b
a+b
Cng theo v hai bt ng thc trn ta c ab + b a
1.
>
=
a + b ab a + b
Bt ng thc c chng minh.
Ch . Bt ng thc trn ng vi mi a,b dng.
Bi 2. Cho a,b l cc s thc dng thay i tha mn iu kin a + b =
ab .
1

Chng minh rng ab + b a > 6 .


Li gii
T gi thit ta c:

a
1 1
.
+ =1 a > 1, b > 1 v b =
a 1
a b

Ta cn chng minh

a
a
a 1

a
+
>6.
a 1

a
a
a
a a 1 = (1 + a 1) a 1 1 + ( a 1) .
=1+ a

a 1
Ta c:
.
a
a
1
a
a
a 1 = 1 + a 1 1 + a 1

Suy ra:

a
a
a 1

1
a
a
+
= 4 + ( a 1) +
4+2
2+a+
a 1
a 1
a 1

( a 1) .

1
=6.
a 1

Du ng thc khng xy ra nn ab + b a > 6 .


Bi 3. Cho hai s thc a,b tho mn iu kin 0 < a, b 1 .
Chng minh rng ab a + b a b 2 .
Li gii
S dng bt ng thc Bernoulli ta c
ba

1 + ( a 1)( b a )

a b

1 + ( b 1)( a b )

ab a = (1 + a 1)
b a b = (1 + b 1)

Cng theo v hai bt ng thc trn ta c


ab a + b a b 2 + ( b 1)( a b ) + ( a 1)( b a ) = 2 ( a b ) 2 .
2

Bt ng thc c chng minh. ng thc xy ra khi v ch khi a = b .


Bi tp tng t
Cho a,b,c l cc s thc dng c tng bng 1. Chng minh
1
a1 a b1b c1c .
3
316

www.TaiLieuLuyenThi.com
Cty TNHH MTV DVVH Khang Vit

Bi 4. Chng minh vi mi s thc a, b, c thuc on [ 0;1] ta lun c


a 2b + b 2 c + c 2 a

3
.
4

Li gii
S dng bt ng thc Bernoulli ta c:
y

1 x x + y xy
x
1 x
.
= 1 +
=
xy
1 + y.
y
x
x
x
x + y xy
x

p dng bt ng thc trn suy ra:


1

a
b
c

a 2b + b 2 c + c 2 a
+
+

a
b
ab
b
c
bc
c
a
ac
+

3
a b c

+
+

4
a+b b+c c+a
Bi ton c chng minh.
n

3
1
.
Bi 5. Chng minh vi mi s nguyn dng n ta c 1 + 3
n+2
n
Li gii
Ta chng minh bi ton bng quy np kt hp s dng bt ng thc Bernoulli.
+ Vi n = 1 bt ng thc tr thnh ng thc.
k

3
1
+ Gi s bt ng thc ng vi n = k tc 1 + 3
.
k+2
k
1

+ Ta cn chng minh 1 +

k +1

k +1

3
.
( k + 1) + 2

Ch
k +1

k
1 +

1
k + 2 k 2 + 2k
k+2
k +1

=
=
.
2

k
k
k + 1 k + 2k + 1
k +1
1
1
+
1
+
1

k
k 2 + 2k

( k + 2)
k+2
1
=
.
k
k +1 1+
( k + 1)( k + 3)
2
k + 2k
2

Suy ra
1

1 +

k +1

k +1

1
1 +
k

( k + 2 )2 3 3 ( k + 2 )2 =3
3
.

( k + 1) + 2
( k + 1)( k + 3) k + 2 ( k + 1)( k + 3)
317

www.TaiLieuLuyenThi.com
Khm ph t duy K thut gii bt T Bi ton Max Min ng Thnh Nam

iu chng t bt ng thc ng vi n= k + 1 .
Vy theo nguyn l quy np ta c iu phi chng minh.
ng thc xy ra khi v ch khi =
n 1,=
n 2.
Bi tp tng t
n +1

1
Chng minh vi mi s nguyn dng n ta c 1 +
> 1 + .
n +1
n
Ta cng xt cc bi ton c cp.
Bi 6. Cho a,b,c l cc s thc khng m tho mn iu kin ab + bc + ca > 0
chng minh vi mi s nguyn dng n 2 ta c
a
b
c
+n
+n
2.
b+c
c+a
a+b
Li gii
+ Nu tn ti mt s bng 0 gi s l khi s dng bt ng thc AM GM cho
hai s ta c ngay iu phi chng minh.
+ Xt c 3 s u dng s dng bt ng thc Bernoulli ta c
n

n
a
1
1
=
=

1
1
( n 1)( b + c )
( n 1)( b + c ) n ( n 1)( b + c ) a n

1 +
(1)
a
a

na
1

n
b
c
a
1

n
1
)( )
1 (
( )( a + b + c )
1+ .
n
a

a
=
n
( n 1)( b + c )

Tng t ta c

nb
( n 1)( a + b + c )

nc
( n 1)( a + b + c )

( n 1)( c + a )
( n 1)( a + b )

Gi P l biu thc v tri v cng theo v ba bt ng thc trn ta c


n n
P
n 1 2 .
n 1
Bi ton c chng minh.
Ch trong (1) ta dng php nghch o trc khi s dng Bernoulli bi vi s
m nh hn 1 nu nh gi ngay s c chiu bt ng thc ngc li. V vy
cn nghch o c chiu bt ng thc cn chng minh.

318

www.TaiLieuLuyenThi.com
Cty TNHH MTV DVVH Khang Vit

Bi 7. Cho a,b,c l cc s thc dng. Chng minh


3

a
b
c
+3
+3
1.
2b + 25c
2c + 25a
2a + 25b

Li gii
p dng BT Bernoulli ta c
3

27 a
1
1
81a
.
=

=
1
1 2b + 25c 2b + 25c + 54a
2b + 25c
1
2b + 25c 3 1 +
3 27 a

27 a

27b
81b
27c
81c
.

;3

2c + 25a 2c + 25a + 54b 2a + 25b 2a + 25b + 54c


Cng theo v ba bt ng thc trn v kt hp s dng bt ng thc C S ta

Tng t ta c

a
b
c
(a + b + c)2
+3
+3
27.
1
2b + 25c
2c + 25a
2a + 25b
81(ab + bc + ac)

Bt ng thc c chng minh. ng thc xy ra khi v ch khi a= b= c .


Bi 8. Cho a,b,c l cc s thc dng. Chng minh
7

2a
2b
2c

+3
+3
3.
a+b
b+c
c+a

Li gii
S dng bt ng thc Bernoulli dng x a 1 +

a b
x 1 ta c
b

7
2

2a
7 2a
2a 3
3 =

1
+

6 b + c
a+b
a+b

7
2

2b
7 2b
2b 3
3 =

1
+

; .

6 b + c
b+c
b+c

7
2

2c
7 2c
2c 3
3 =

1
+

1 .
6 c + a
c+a
c+a

Cng theo v ba bt ng thc trn v gi P l biu thc v tri ta c


2
2
2

7 2a 2b 2c
+
+
P 3 +


3 .
6 a + b b + c c + a

Ch .

319

www.TaiLieuLuyenThi.com
Khm ph t duy K thut gii bt T Bi ton Max Min ng Thnh Nam
2

1
1
1
b
c
a
a b c

, x =
,y=
,z =
+

+
+
= 2+

2
2
b
c
a+b b+c c+a
(1 + x ) (1 + y ) (1 + z ) a

1
1
1
3
z
+
=
+

2
2
1 + xy (1 + z )
4
z + 1 ( z + 1)

V vy P 3 . ng thc xy ra khi v ch khi a= b= c .


Ch . Cu hi t ra l ti sao khng s dng trc tip Bernoulli dng suy rng
ban u bi nh vy ta c
7

2a
2a
7 2a

3
1 1 +
1 .

= 1 +
3 a +b
a+b
a+b
a
b
c
3
+
+
.
a+b b+c c+a 2
Tuy nhin bt ng thc ny khng phi ng vi mi a,b,c dng chng hn
=
a 1,=
b 2,=
c 3.

Khi a v chng minh

Bi tp tng t
1) Cho a,b,c l cc s thc khng m tho mn iu kin ab + bc + ca > 0 .
a
b
c
+3
+3
2.
b+c
c+a
a+b
2) Cho bn s thc dng a, b, c, d. Chng minh

Chng minh rng

2a
2b
2c
2d

+3
+3
+3
4.
a+b
b+c
c+d
d +a

Bi 9. Cho a,b,c l cc s thc dng chng minh ab + c + bc + a + c a +b 1 .


Li gii
Ta cn so snh cc s (b + c),(c + a ),(a + b) vi s 1.
+ Nu tn ti mt trong ba s ln hn 1 th r rng bt ng thc lun ng.

+ Ta ch cn xt khi a,b,c cng thuc na khong ( 0;1] , khng mt tnh tng qut
ta gi s a b c suy ra min {(a + b),(b + c),(c + a )} =+
b c.

n y ta li chia trng hp x l
- Nu b + c 1 s dng bt ng thc Bernoulli ta c
b+c

1 + ( a 1)( b + c )

c+a

1 + ( b 1)( c + a ) .

ab+c =
1 + ( a 1)
bc + a =
1 + ( b 1)

a +b

c a +b =
1 + ( c 1)
1 + ( c 1)( a + b )
Cng theo v ba bt ng thc trn a v chng minh
320

www.TaiLieuLuyenThi.com
Cty TNHH MTV DVVH Khang Vit

1 + ab + bc + ca a b c 0 (1 a )(1 b )(1 c ) + abc 0 .

Bt ng thc ng.
- Nu b + c < 1 ta li chia thnh cc trng hp sau y:
Kh nng 1: b + c c + a a + b 1 s dng bt ng thc Bernoulli ta c
a
a
a
a
.

ab+c =
1( b + c )
+

+
+

+
+
+
1
1
1
a
b
c
a
b
c
a
b
c
a
b
c
(
)
(
)(
)
+

1
1
a
( ( ))
Tng t cho 2 bt ng thc cn li ri cng li theo v ta c iu phi
chng minh.
Kh nng 2: b + c c + a 1 a + b ta c
a b + c a; b c + a b a b + c + b c + a + c a + b a + b + c a + b
a + b + 1 + ( c 1)( a + b=
) c (a + b) + 1 1

Ta c iu phi chng minh.


Kh nng 3: b + c 1 a + c a + b ta c
ab+c a
bc + a 1 + ( b 1)( c + a )
c a +b 1 + ( c 1)( a + b )

Cng theo v ba bt ng thc trn v a v chng minh

1 + bc + ( ab + bc + bc a b c ) 0 (1 a )(1 b )(1 c ) + abc + bc 0 .

Bt ng thc ng.
Vy vi mi s thc dng a,b,c ta lun c ab + c + bc + a + c a +b 1 .
Bi tp tng t
Cho a,b,c l cc s thc dng chng minh ( a + b ) + ( b + c ) + ( c + a ) 2 .
c

Bi 10. Cho a,b l cc s thc khng m c tng bng 2 chng minh


2

a b
a +b +
2.
2
Li gii
Khng mt tnh tng qut gi s a b 0 a 1 1;0 b 1 s dng bt ng
thc Bernoulli ta c
2b

2a

1 + ( a 1) 1 + ( a 1) b =
b 2 + b + 1
ab =
b

b a = b. 1 + ( b 1)

a 1

b 1 + ( a 1)( b 1) = b 2 ( 2 b )

Vy ta ch cn chng minh

321

www.TaiLieuLuyenThi.com
Khm ph t duy K thut gii bt T Bi ton Max Min ng Thnh Nam

( b

+ b +1 + b

(2 b)

a b
+
2
2

b 2 + b + 1 + b 4 ( 2 b ) + ( b 1) 2 b3 ( b 1) ( b 2 ) 0
2

Bt ng thc cui ng do ta c pcm.


ng thc xy ra khi v ch khi a= b= 1 .
Bi tp tng t
Cho a,b l cc s thc khng m c tng bng 2. Chng minh
4

a b
a3b + b3a +
2.
2

C. BI TP RN LUYN
Bi 1. Cho n s thc tho mn iu kin 0 < x1 , x2 ,..., xn 1 . Chng minh rng
1

(1 + x1 ) x2 (1 + x2 ) x3 ...(1 + xn ) x1 2n .
Bi 2. Chng minh rng vi mi s thc 0 < x <

( cos x )
2

cot x

Bi 3. Chng minh vi mi s thc 0 < x <

ta c

> sin 2 x .

ta c

( 2 + sin x )2+ tan x > ( 3 + tan x )1+sin x .


Bi 4. Cho a,b l cc s thc thuc khong (0;1). Chng minh
a

a b
+ 2.
b a

4
Bi 5. Cho a,b l cc s thc dng tho mn iu kin b , a + b =
2. Chng
3

minh ab b a + 2 3ab .
Bi 6. Cho x,y l hai s thc dng. Chng minh x y + y x > 1 .
Bi 7. Cho a,b,c l cc s thc khng m tho mn iu kin ab + bc + ca > 0 .
Chng minh rng

322

a
b
c
+3
+3
2.
b+c
c+a
a+b

www.TaiLieuLuyenThi.com
Cty TNHH MTV DVVH Khang Vit

D. HNG DN GII P S
1

Bi 1. Ch (1 + xk ) xk +1 1 +

xk
xk
2 =
, k 1, n=
; xn +1 x1 .
xk +1
xk +1

Nhn theo v n bt ng thc trn ta c iu phi chng minh.


ng thc xy ra khi v ch khi x1= x2= ...= xn= 1 .
Bi 2. Ch vi 0 < x <

ta c cot x > 1 khi s dng bt ng thc

Bernoulli ta c

( cos x )
2

cot x

= (1 sin x )

cot x

(1 + sin x )cot x (1 sin x cot x )(1 + sin x cot x )

1 cos 2 x =
sin 2 x
=
(1 cos x )(1 + cos x ) =

ng thc khng xy ra nn bi ton c chng minh.


Bi 3. t a = 1 + sin x, b = 2 + tan x 1 < a < b ta cn chng minh
b

(1 + a )b > (1 + b )a (1 + a ) a

>1+ b .

Bt ng thc cui ng theo Bernoulli.


Bi 4. S dng bt ng thc Bernoulli ta c:
a

ba
1
b ba
a
= b +1 a
.
= 1 +
1 + a.
a
a
b +1 a
b
a
b

1
b
.
Tng t ta c:
a +1 b
a
Cng theo v hai bt ng thc trn v ch
1
1
4
+

=
2.
b +1 a a +1 b b +1 a + a +1 b
Bt ng thc c chng minh. ng thc xy ra khi v ch khi a = b .
Bi 5. S dng bt ng thc Bernoulli ta c
ab b a =1 + ( a 1) .
b

Vy ta chng minh

1
1
1 + 1
b

1 + b ( a 1)
.
1
1 + a 1
b

1 + b (1 b )
1 + b ( a 1)
+ 2 3b ( 2 b )
+ 2 3ab b.
2 b (2 b)
1
1 + a 1
b

( b 1)( 3b 4 ) 0 ng.

Bt ng thc c chng minh.


323

www.TaiLieuLuyenThi.com
Khm ph t duy K thut gii bt T Bi ton Max Min ng Thnh Nam

Bi 6. Ta xt ba trng hp sau:
+ Nu x > 1, y > 1 x y + y x > x + y > 2 , bt ng thc lun ng.
+ Nu x > 1, y < 1 x y + y x > 1 + y x > 1 , bt ng thc lun ng.
+ Nu x, y 1, p dng bi tp mu ta c pcm.
Bi 7. Nu mt s bng 0 s dng bt ng thc AM GM ta c ngay iu phi
chng minh.
Ta xt c ba s u dng khi s dng bt ng thc Bernoulli ta c

a
1
1
2a 2/3
2a 2/3
.
= 1
=

2
2/3
2/3
b+c
a 2/3 + b 2/3 + c 2/3
b
c
a
+
+
(
)
b + c 3 1 b + c 3 1

+
2 a
2
a
b
2b 2/3
c
2c 2/3
3
.
2/3
;

c + a a + b 2/3 + c 2/3 a + b a 2/3 + b 2/3 + c 2/3


Cng li theo v ba bt ng thc trn ta c pcm.
ng thc xy ra khi v ch khi c mt s bng 0 v hai s bng nhau.

Tng t ta c

324

www.TaiLieuLuyenThi.com
Cty TNHH MTV DVVH Khang Vit

Chng 3:
PHNG PHP HM S TRONG GII TON
BT NG THC V CC TR
CH 1: K THUT S DNG TNH N IU VI BI TON

CC TR V BT NG THC MT BIN S
A. TM TT L THUYT
1. nh ngha 1. Gi s K l mt khong, na khong, mt on.
Hm s y = f ( x) xc nh trn D c gi l:
+ ng bin trn K nu vi mi x1 , x2 K , x1 < x2 f ( x1 ) < f ( x2 ) .
+ Nghch bin trn K nu vi mi x1 , x2 K , x1 < x2 f ( x1 ) > f ( x2 ) .
2. iu kin cn hm s n iu
Gi s hm s y = f ( x) c o hm trn K
+ Nu hm s y = f ( x) ng bin trn K th f '( x) 0 vi mi x thuc K.
+ Nu hm s y = f ( x) nghch bin trn K th f '( x) 0 vi mi x thuc K.
3. iu kin hm s n iu
nh l. Gi s K l mt khong, na khong, mt on. Hm s y = f ( x) lin
tc trn K v c o hm ti mi im trong ca K(tc cc im thuc K nhng
khng phi u mt ca K). Khi
+ Nu f '( x) > 0, x K th hm s y = f ( x) ng bin trn K.
+ Nu f '( x) < 0, x K th hm s y = f ( x) nghch bin trn K.
+ Nu f '( x) = 0, x K th hm s y = f ( x) khng i trn K.
Ch . Nu hm s

y = f ( x) lin tc trn on [a;b] v c o hm

f '( x) > 0 ( f '( x) < 0 ) vi mi x thuc khong (a;b) th hm s y = f ( x) ng bin

(nghch bin) trn on [a;b].


4. nh ngha 2.
Cho hm s y = f ( x) xc nh trn K.
+ S M c gi l gi tr ln nht ca hm s y = f ( x) trn K khi v ch khi
f ( x) M , x K
.

M
x0 K | f ( x0 ) =
+ S m c gi l gi tr nh nht ca hm s y = f ( x) trn K khi v ch khi
f ( x) m, x K
.

m
x0 K | f ( x0 ) =
325

www.TaiLieuLuyenThi.com
Khm ph t duy K thut gii bt T Bi ton Max Min ng Thnh Nam

B. NI DUNG PHNG PHP


1) K thut tm gi tr ln nht, gi tr nh nht ca hm s
tm gi tr ln nht, gi tr nh nht ca hm s y = f ( x) trn D ta tnh y
ri tm cc im ti hn(cc im m ti y trit tiu hoc khng tn ti) lp
bng bin thin. T bng bin thin suy ra max, min.
Ch . Nu hm s y = f ( x) lun tng hoc lun gim trn on [a;b] th
ymax

max
=
{ f (a); f (b)} ; ymin min { f (a); f (b)} .

+ Nu hm s y = f ( x) lin tc trn on [ a; b ] lun tn ti gi tr ln nht, gi tr


nh nht trn on v ta tm max, min n gin nh sau:
Bc 1. Tnh o hm y, tm cc im ti hn ca hm s gi s l x1 , x2 ,..., xn .
Bc 2. Tnh cc gi tr f ( x1 ), f ( x2 ),..., f ( xn ) .
Bc 3. Kt lun
ymax

max
=
{ f ( x1 ), f ( x2 ),..., f ( xn )}; ymin min { f ( x1 ), f ( x2 ),..., f ( xn )} .

V d 1. Tm gi tr ln nht, gi tr nh nht ca hm s y =x 6 + 4 1 x 2
on [ 1;1] .
Li gii

Xt hm s y =x 6 + 4 1 x

lin tc trn on [ 1;1] .Ta c

) =0

; y' =
0 6 x x4 4 1 x2

x = 0
x = 0

2
2
x =
x =

2 1 x

2 1 x 2
x =
x =

6 x5 24 x 1 x 2
y' =
x = 0
4
x= 4 1 x 2

2 3

2
3
2

2 4
Suy ra y (0)= 4; y =
; y (1)= 1 .

3 9
2 4
V vy ymin =y
4.
= ; ymax =y (0) =
3 9
V d 2. Tm gi tr ln nht v nh nht ca hm s

y=

326

1 + x 2015 + 1 + (1 x )

2015

vi x [ 0;1] .

trn

www.TaiLieuLuyenThi.com
Cty TNHH MTV DVVH Khang Vit

Li gii
Ta c y '
=

2015 x 2014
2 1 + x 2015

2015 (1 x )

2014

2 1 + (1 x )

2015

Suy ra
y ' = 0 x 2014 1 + (1 x )

1 + x 2015
x 2014

Xt hm s f (t ) =

2015

=(1 x )

1 + (1 x )
=
(1 x )2014

2014

1 + x 2015

2015

1 + t 2015
t 2014

(1)

trn on [0;1] ta c

2013t 2015 + 4028

<0.
f '(t ) =
2t 2015 1 + t 2015
V vy f(t) l hm nghch bin trn on [0;1], do
1
(1) f ( x) = f (1 x) x =1 x x = .
2

1
1
Tnh cc gi tr: y (0) =
y (1) =
1 + 2; y =
2 1+
2
2
1
Vy gi tr nh nht ca hm s bng 2 1 +
2

2015

2015

; gi tr ln nht bng 1 + 2 .

Bi tp tng t
Cho a,b l hai s thc khng m c tng bng 1. Tm gi tr ln nht, gi tr nh
nht ca biu thc P = 1 + a 2014 + 1 + b 2014 .
V d 3. Tm gi tr ln nht v gi tr nh nht ca hm s
y= 5 | cos x + sin x | + | 7 cos x + sin x | .
Li gii
Ta a v chung mt biu thc bng cch s dng ng thc:
a + b=

a 2 + b 2 + 2 ab .

Khi

327

www.TaiLieuLuyenThi.com
Khm ph t duy K thut gii bt T Bi ton Max Min ng Thnh Nam

25 ( sin x + cos x ) + ( 7 cos x + sin x ) + 10 ( sin x + cos x )( 7 cos x + sin x )


2

y
=

50 + 32sin 2 x + 24cos 2 x + 10 sin 2 x + 7 cos 2 x + 4sin 2 x


=
= 50 + 32sin 2 x + 24cos 2 x + 10 4 + 3cos 2 x + 4sin 2 x
=
=

8 ( 3cos 2 x + 4sin 2 x ) + 10 4 + 3cos 2 x + 4sin 2 x + 50


40sin ( 2 x + ) + 40 + 50sin ( 2 x + ) + 50

Vi=
cos

4
3
.
=
,sin
5
5

Gi ch cn kho st hm s y =
4sin t + 4 + 5sin t ymax =
13; ymin =

16
.
5

T suy ra gi tr ln nht v nh nht ca hm s cho l


=
ymax 6=
5; ymin 3 2 .

2) K thut i bin
S dng khi hm mt bin c cha cn phc tp; dng lng gic; m hoc a
thc c cha bc cao.
V d 1. Tm gi tr ln nht v nh nht ca hm s y =

x x 1=

1
x + x 1

Ta
c y '
=

t2 +1
.
2

t2 + 5
.
2t + 2

t 2 + 2t 5
2 ( t + 1)

= y=
(1)
< 0, t ( 0;1] . Do ymin

tr ln nht.
V d 2. Chng minh rng vi mi s t nhin n > 1 ta c
n

328

1, x 1 t ( 0;1] .

Ch . t 2 = 2 x 2 x ( x 1) 1 x x ( x 1) =
Khi y =

x x 1 +1

Li gii

Tp xc nh: D= [1; + ) .
t t=

x x ( x 1) + 2

1+

n n nn
+ 1
< 2.
n
n

3
. Khng tn ti gi
2

www.TaiLieuLuyenThi.com
Cty TNHH MTV DVVH Khang Vit

Li gii
n

n
( 0;1) , n N * . BT cn chng minh tr thnh
n
n 1 + x + n 1 x < 2, x ( 0;1) .
Xt hm s f ( x) = n 1 + x + n 1 x lin tc x [ 0;1) c

t x=

1
1
1
< 0, x 0;1
f '( x)
=

( )
n n (1 + x)n1 n (1 x)n1
Vy f(x) nghch bin [0; 1) nn f(x) < f(0) = 2, x ( 0;1) (pcm).
V d 3. Cho hm s f xc nh trn tp s thc, ly gi tr trn R v tha mn iu
kin f ( cot=
x ) sin 2 x + cos 2 x, x ( 0; ) .
Hy

tm

gi

tr

ln

) (

nht

gi

tr

nh

nht

ca

hm

g ( x ) = f sin x . f cos x trn R.


2

Li gii
Ta c f ( cot=
x ) sin 2 x + cos 2 x, x ( 0; )
cot 2 x 1 cot 2 x + 2cot x 1
=
, x ( 0; )
cot 2 x + 1 cot 2 x + 1
cot 2 x + 1
T vi ch rng vi mi t R u tn ti x ( 0; ) sao cho cot x = t ta
2cot x

f=
( cot x )

c
=
f (t )

t 2 + 2t 1
t2 +1

, t R .

) (

Suy ra g ( x ) f =
=
sin 2 x . f cos 2 x

sin 4 2 x + 32sin 2 2 x 32
sin 4 2 x 8sin 2 2 x + 32

, x R (1) .

1
1
t u = sin 2 2 x ta c u thuc on 0; .
4
4
V vy t (1) ta c
min g ( x) = min h(u ) v m ax g ( x) = m ax h(u )
xR

1
0; 4

trong h ( u ) =
Ta c h ( u ) =

xR

u 2 + 8u 2
u 2 2u + 2

(
(u

2 5u 2 + 4u + 6
2

2u + 2

1
0; 4

).
329

www.TaiLieuLuyenThi.com
Khm ph t duy K thut gii bt T Bi ton Max Min ng Thnh Nam

1
D dng chng minh c h ( u ) > 0, u 0; .
4
1
Do hm h(u) ng bin trn 0; .
4
1
1 1
.
V vy trn 0; ta c min h ( u ) = h ( 0 ) = 1 v max =
h ( u ) h=

4
4 25
1
Vy min g ( x ) = 1 , t c chng hn khi x = 0 v max g ( x ) =
, t c
25

chng hn khi x =

3) K thut kho st trc tip


Trong mt s bi ton c th phi o hm nhiu ln lin tip thm ch phi
kho st thm hm s ph. Ta thng s dng
f(x) ng bin trn [a; b] th f(x) > f(a) vi mi x > a.
f(x) nghch bin trn [a; b] th f(x) > f(b) vi mi x < b.
Ch . Lp bng bin thin nu cn thit hoc s dng tnh cht v im cc i,
cc tiu.
V d 1. Chng minh rng vi mi x > 0 ta c x

x3
< sinx < x .
6

Li gii
Xt hm s f ( x) = x +

x
+ sinx trn na khong [ 0;+ ) . Ta c
6

x2
1 + cos x
2
f ( x)= x sinx
f ( x) = 1 cos x
f ( x)=

Ta c f ( x) = 1 cos x 0, x [ 0; + ) f ( x) f (0) = 0 , nn f(x) ng


bin trn [ 0;+ ) . Suy ra f ( x) f (0) =
0 f ( x) ng bin trn [ 0;+ ) .
Do f ( x) f (0) = 0, x 0 v f ( x) > f (0) = 0, x > 0.
Tc l

x3
x3
x + sin x > 0 x
< sin x vi x > 0
6
6

Lu f ( x) > f (0) =
0 vi x > 0 ta c sinx < x

(1)
(2)

T (1) v (2) ta c pcm.


V d 2. Chng minh vi mi x dng v n l s nguyn dng ta c
330

www.TaiLieuLuyenThi.com
Cty TNHH MTV DVVH Khang Vit

x 2 x3
xn
.
+ ... +
2! 3!
n!
Li gii

ex > 1 + x +

x 2 x3
xn
Xt hm s f n ( x) = e x 1 + x +
+ ... + trn khong ( 0;+ ) ta c

2! 3!
n !

x 2 x3
x n 1
+
+ ... +
f 'n ( x) = e x 1 + x +
.

2!
3!
1
!
n

(
)

T suy ra f ( n ) n ( x) = e x 1 > 0, x > 0 f ( n 1) n ( x) > f ( n 1) n (0) = 0 .


S dng lin tip ta c f 'n ( x) > f 'n (0) =
0 f n ( x) > f n (0) =
0 (pcm).
Ch . S thc dng a bt ng thc sau lun ng vi mi s thc x l e.
x2
xn
+ ... + , x ; n * .
2
n!
Tht vy, Bt ng thc cho tng ng vi:
ax 1+ x +

x2
xn
+ ... +
x ln a ln 1 + x +

2
n !

ln 1 + x +

ln a

ln 1 + x +

ln a

Ch

x2
xn
+ ... +
n!
2
,x > 0.
x
x2
xn
+ ... +
n!
2
,x < 0
x

x2
xn
ln 1 + x +
+ ... +
2
n!

lim
=
x
x 0+

x2
xn
ln 1 + x +
+ ... +
2
n!

lim
1.
=
x
x 0

ln a 1
Do
a=
e.
ln a 1
Ngc li vi a l e th bt ng thc ng.
V d 3. Cho n l s l ln hn 3. Chng minh rng vi mi x 0 ta c

x2
x n
x 2 x3
xn
x
x
1
...
1
...
+
+
+
+

+
+

< 1 .

2!
n
!
2!
3!
n
!

331

www.TaiLieuLuyenThi.com
Khm ph t duy K thut gii bt T Bi ton Max Min ng Thnh Nam

Li gii

x
x
+ ... +
u ( x) =1 + x +

2!
n!
t
.
2
3
x x
xn

v( x) =1 x + 2! 3! + ... + n !
Ta cn chng minh
=
f ( x) u ( x).v( x) < 1.
2

Ta c

x2 + ... + xn 1 = u ( x) xn
u '( x) =1 + x +
2!
(n 1)!
n!

x2 + x3 + ... + xn 1 =
xn
(
)
v
x

'(
)
1
=

v
x
x

2! 3!
(n 1)!
n!

V vy

).v( x) u( x)v( x) + u ( x).v( x)


f ( x) u ( x=
=

=
u ( x )

xn
xn
xn
u ( x) + v( x)
.v( x) + u ( x) v( x) =
n!
n!
n!

x n1
2 xn x2 x4
1
...
=

+
+
+
+

(n 1)!
n! 2! 4!
V n l s l ln hn 3 nn f ( x) i du t dng sang m khi i qua x = 0 .
Do ti x = 0 th hm s t cc i v vy f ( x) < f (0) = 1, x 0. (pcm).
a+x
V d 4. Chng minh vi mi a, b, x > 0 v a b ta c

b+ x
Li gii
a+x
Xt=
hm s f ( x)

b+ x

b+ x

x 0. Khi ln f ( x=
)

b+ x

f ( x)
a+x
b + x a + x
a+ x ba
.
= ln
+ (b + x)
=
+
ln
f ( x)
b+x
a+x b+x
b+x a+x

a+ x ba
=
f ( x) f ( x) ln
+=

b+ x a+x
a+x
=

b+ x
332

b+ x

a+ x ba a+ x
+
ln =

b+ x a+ x b+ x

a+x
.
b+ x

( b + x ) ln

a + x
Suy ra [ ln f ( x)=
] (b + x) ln
b + x

a
> .
b

b+ x

.g ( x )

www.TaiLieuLuyenThi.com
Cty TNHH MTV DVVH Khang Vit

a+ x ba
. Ta c
+
b+ x a+ x
b+ x ba
a b
( a b) 2
+
=

< 0.
g ( x) =
.
a + x (b + x) 2 (a + x) 2
(a + x) 2 (b + x)

trong =
g ( x) ln

Do g(x) nghch bin trn ( 0;+ ) . Suy ra


a+ x ba
+ =
g ( x) > lim =
g ( x) lim ln
0.
x
x
b+ x a+x

Vy f ( x) > 0, x > 0 , nn f(x) ng bin trn ( 0;+ ) .


Suy ra f(x) > f(0) (pcm).
V d 5. Cho a, b [ 0;1] . Chng minh rng
x
b
a
+
+
+ (1 x )(1 a )(1 b ) 1, x [ 0;1] .
a + b +1 x + a +1 x + b +1
Li gii
x
b
a
Xt hm s =
f ( x)
+
+
+ (1 x )(1 a )(1 b ) lin tc
a + b +1 x + a +1 x + b +1
trn on [0;1] ta c
1
b
a
( x)
f=

(1 a )(1 b )
2
a + b + 1 ( x + a + 1)
( x + b + 1)2
f ( x )
=

2b

( x + a + 1)

2a

( x + b + 1)3

0, [ 0;1]

Nn hm s f(x) ng bin trn [0; 1], suy ra phng trnh f(x) = 0 c nhiu
nht mt nghim trn (0; 1).
Nu phng trnh f(x) = 0 v nghim th f(x) n iu trn [0; 1], th
f ( x ) max f ( x ) =
max { f ( 0 ) ; f (1)}.
[0;1]

Ta c
f ( 0=
)

f (1=
)

b
a
a 2b 2 + a + b + 1 ab + a + b + 1
+
+ (1 a )(1 b=

= 1;
)
a +1 b +1
( a + 1)( b + 1) ( a + 1)( b + 1)

1
a
b
1
a
b
+
+

+
+
= 1.
a + b +1 b + 2 a + 2 a + b +1 b + a +1 a + b +1

Suy ra f ( x ) max=
f ( x ) max { f ( 0 ) ; f (1)} 1 .
[0;1]
Nu phng trnh f(x) = 0 c nghim duy nht x = x0 . Khi do f(x) ng
bin trn [0; 1], nn
333

www.TaiLieuLuyenThi.com
Khm ph t duy K thut gii bt T Bi ton Max Min ng Thnh Nam

f ( x ) < 0, x [ 0; x0 ] v f ( x ) > 0, x [ x0 ;1] .

Do x0 l im cc tiu ca hm s, m f(x) lin tc trn [0; 1] nn


f ( x ) max=
f ( x ) max { f ( 0 ) ; f (1)} 1 .
[0;1]
T hai trng hp ta c iu phi chng minh.
V d 6. Cho tam gic ABC c A > B > C . Tm gi tr nh nht ca biu thc
=
P

iu kin xc nh: D =
Xt hm
s f ( x)
=
f '( x)

x sin A
x sin B
.
+
x sin C
x sin C
Li gii

( ;sin C ] [sin A; + ) .

x sin A
+
x sin C

x sin B
lin tc trn D ta c
x sin C

1 x sin C sin A sin C 1 x sin C sin B sin C


.
+
.
> 0, xD .
2 x sin A ( x sin C )2 2 x sin B ( x sin C )2

Do f(x) ng bin trn D v c =


f (sin A)
lim f ( x) 2;=
x

Vy gi tr nh nht ca P bng

sin A sin B
t ti x = sin A .
sin A sin C
+n

1
V d 7. (VMC 2011) Cho , tha mn 1 +
n
n nguyn dng. Tm min .

1
< e < 1 +
n

Li gii
1
Ly logarit t nhin suy ra <
n < , n * .
1
ln 1 +
n
1
Xt hm s =
f ( x)
x, x 1 . Khi
1
ln 1 +
x

max f ( x) min f ( x) .
min =
x 1

Mt khc =
f '( x)

334

sin A sin B
<1.
sin A sin C

x 1

1
1 < 0, x 1 .
x ( x + 1)

+n

vi mi

www.TaiLieuLuyenThi.com
Cty TNHH MTV DVVH Khang Vit

1
1
1 lim
x .
Do max f ( x) min f ( x=
) f (1) lim f ( x=
)
x 1
x +
x +
1
x 1
ln 2

ln 1 + x

1
1
1
1 x ln 1 +
ln 1 + +
1
1
x
x x +1

.
1 lim
=
1 lim
1
x +
x +
ln 2
ln 2
1

ln 1 +
x ( x + 1)
x

1
1

x ( x + 1) ( x + 1)2
x ( x + 1) x 2
1
1
1
3
=
1 lim
==
1 lim
=
.
2x + 1
x +
x +
ln 2
ln 2
2x + 1
ln 2 2
2
2
x ( x + 1)

1
3
.
ln 2 2
V d 8. [ MR No.4 2009] Cho cc s thc .. tha mn iu kin

Vy min =

a1x + a2x + ... + anx b1x + b2x + ... + b nx vi mi x .

Chng minh rng f : ( 0, + )


x

a
a a
f (=
x) 1 + 2 + ... + n
b1 b2
bn
Li gii
x

tng trn ( 0, + ) .

a a
a a a a
Ta=
c f '( x) 1 ln 1 + 2 ln 2 + ... + n ln n ,
b1 b1 b2 b2
bn bn
x

a2
an an
ln + ... + ln 0 .
b2
bn bn
Suy ra f '( x) l hm tng trn . Ta chng minh f '(0) = 0 , tht vy.

a
=
f ''( x) 1
b1

a1 a2
ln +
b1 b2

Theo gi thit ta c
a x
a x
a x
b1x 1 1 + b2x 2 1 + ... + bnx n 1 0, x .
b

an
a1
a2
1
1
1
x b1
x b2
x bn
Vi x > 0 ta c b1 .
+ b2 .
+ ... + bn .
0, x > 0 .
x
x
x

Cho x 0+ ta c
335

www.TaiLieuLuyenThi.com
Khm ph t duy K thut gii bt T Bi ton Max Min ng Thnh Nam

ln

a
a1
a
+ ln 2 + ... + ln n 0
b1
b2
bn

(0.1)

Vi x < 0 ta c
x

an
a1
a2
1
1
1
x b1
x b2
x bn
b1 .
+ b2 .
+ ... + bn .
0, x < 0 .
x
x
x

Cho x 0 ta c
a
a
a
ln 1 + ln 2 + ... + ln n 0
b1
b2
bn
T (0.1) v (0.2) suy ra f '(0)
= ln

(0.2)
a
a1
a
+ ln 2 + ... + ln n= 0
b1
b2
bn

f '( x) f '(0) =
0 iu phi chng minh.

V d 9. Cho cc s thc a1 , a2 ,..., an > 0 v hm s f : ( 0, + ) ( 0, + ) xc nh


1/ x

a x + a2x + ... + anx


bi f ( x) = 1
.

Chng minh rng f l hm tng v tnh gii hn ca f ( x) khi x + .

Li gii
Xt hm s g : ( 0, + ) ( 0, + ) xc nh bi g ( x) = xt l hm li.
Xt =
t

y
> 1 tc x < y, khi ta c
x

a1x + a2x + ... + anx


g =

a1x + a2x + ... + anx

1 y
a + a2y + ... + any
n 1
Bt ng thc trn tng ng vi
=

x 1
x
x
x
g a1 + g a2 + ... + g an
n

(( ) ( )

)
1/ y

1/ x

a y + a2y + ... + any


1
f ( x) f ( y ) .

Vy f l hm tng. Ta c iu phi chng minh.


a x + a2x + ... + anx
1
n


V d 10. (IMC 2006) So snh sin ( tan x ) v tan ( sin x ) vi x 0; .
2

Li gii
336

( ))

www.TaiLieuLuyenThi.com
Cty TNHH MTV DVVH Khang Vit


Xt hm=
s f ( x) tan ( sin x ) sin ( tan x ) trn 0; .
2

Ta c f '( x) =

cos x
cos 2 ( sin x )

Nu 0 < x < arctan

cos ( tan x )
cos 2 x

cos3 x cos ( tan x ) cos 2 ( sin x )


cos 2 x cos 2 ( sin x )

cos ( tan x ) > 0 s dng bt ng thc c si ta c


3

cos(tan x) + 2cos(sin x) tan x + 2sin x


3
cos ( tan x ) cos 2 ( sin x )
< cos x
cos
3
3


do cos x l hm lm v tan x + 2sin x > 3 x, x 0; .
2

Do f '( x) > 0 hay f ( x) l hm tng trn 0;arctan .


2

V vy f ( x) > f (0) =
0 tan ( sin x ) > sin ( tan x ) .
Nu x = arc tan

th




/2


arc tan tan
tan sin =
> tan > 1 > sin tan arc tan .
2
4
2


1+ 2 / 4

Nu x arc tan ; tan ( sin x ) > 1 > sin x ( tan x ) .


2 2


Vy vi mi x 0; th tan ( sin x ) > sin ( tan x ) .
2
4) Vn dng kt hp cc bt ng thc khc nh AM GM; C S
Trc khi kho st tnh n iu ca hm s ta s dng cc nh gi c bn
thng qua cc bt ng thc c bn nh AM GM; C S s a v chng minh
mt bt ng thc c bn hn thun li cho vic kho st hm s.

V d 1. Chng minh rng nu 0 x <

ta c 2s inx + 2 t anx 2 x+1 .

2
Li gii

S dng bt ng thc AM GM ta c 2s inx + 2 t anx 2. 2s inx.2 t anx .


Ta chng minh 2 2s inx.2 t anx 2 x +1 2s inx + t anx 22 x sinx + t anx 2 x .

Xt hm s f ( x) = sinx + tan x 2 x lin tc trn 0; , ta c
2

337

www.TaiLieuLuyenThi.com
Khm ph t duy K thut gii bt T Bi ton Max Min ng Thnh Nam

f ( x) =cos x +

1
2

cos x

2 > cos 2 x +


(v vi x 0; th
2
1
cos 2 x +
2 ).
cos 2 x


2 0, x 0; .
cos x
2
1

cos x > cos 2 x

v theo BT AM-GM ta c


Do f(x) ng bin trn 0; .
2

Suy ra f ( x) f (0) =
0 , hay sinx + t anx 2x vi mi x 0;
2
Bi tp tng t
sin x

Chng minh rng 4

+2

tan x

3x+2
2 2 ,

(pcm).


x 0; .
2

sinx

V d 2. Chng minh rng
> cos x, x 0; .
x
2
Li gii
Bt ng thc tng ng vi:
3

sin x
sinx
2
3

> cos x 3 > cos x sin x.tan x- x > 0 (1) .


x
x

Xt hm s f ( x) = sin 2 x.tan x- x3 vi x 0; .
2

Ta c f ( x ) = 2sin 2 x + tan 2 x 3 x 2 .
p dng BT 3(a 2 + b 2 + c 2 ) ( a + b + c ) ta c
2

f ( x )= sin 2 x + sin 2 x + tan 2 x 3 x 2

t g ( x) = 2sin x + tan x 3 x,
g ( x=
) 2cos x +

1
2

cos x

1
( 2sin x + tan x )2 3x 2 .
3


x 0; , th
2

3 cos x + cos x +
=

1
cos x


vi mi x 0; , nn g(x) ng bin trn
2
338

3 33 cos 2 x.


0; .
2

1
cos 2 x

3 0
=

www.TaiLieuLuyenThi.com
Cty TNHH MTV DVVH Khang Vit


Suy ra g ( x ) g ( 0 ) = 0, x 0; .
2

Do f ( x )

1
0 nn f(x) ng bin trn
( 3 x )2 3 x 2 =
3


0; 2 . Suy ra

f ( x ) > f ( 0=
) 0, x 0; .
2
Nhn xt: Khi trong BT c cha cc loi hm s khc nhau ta thng c lp
mi loi hm s d xt du ca o hm, hoc ta c th o hm lin tip
kh bt mt loi hm s.
Cch 2: Ta c x

x3
< sin x, suy ra
6
3

3
2
x 2 x 4 x6
x2 x4
sin x x
>

=
+

>

+
1
1
1

6
2 12 216
2 24
x

Ta chng minh c cos x < 1

x2 x4
+
2 24

( 2)

( 2)

T (1) v (2) ta c pcm.


5) K thut vn dng nh l Lagrange
nh l. Nu hm s y = f ( x) l hm lin tc trn on [a; b] v c o hm trn
khong (a; b) th tn ti c (a; b) sao cho f '(c) =
1
1 x +1
V d 1. Chng minh (1 + ) x < (1 +
) ,
x
x +1
Li gii

f (b) f (a )
.
ba

x (0; +) .

1
1 x +1
Ta c: (1 + ) x < (1 +
) x[ln( x + 1) - ln x] < ( x + 1)[ln( x + 2) - ln( x + 1)]
x
x +1
Xt hm s =
f ( x) x[ln( x + 1) - ln x] ta c
x
1= ln( x + 1) ln x
x +1
p dng nh l Lagrange i vi hm s: y =
1
c (x; x+1) sao cho: f '(c)= ln( x + 1) ln x =
c
f '( x)= ln( x + 1) ln x +

Mt khc 0 < x < c < x + 1

1
.
x +1
lnt trn [x; x+1], th tn ti
ln( x + 1) ln x.

1 1
1
> >
x c x +1
339

www.TaiLieuLuyenThi.com
Khm ph t duy K thut gii bt T Bi ton Max Min ng Thnh Nam

1
1
1
> ln( x + 1) ln x >
ln( x + 1) ln x
>0
x
x +1
x +1
f '( x) > 0, x (0;+) hm s f ( x) ng bin trn (0;+).

T (1) suy ra: f '( x) > 0, x (0; +) f ( x) ng bin trn (0; +).
Suy ra: f ( x + 1) > f ( x), x (0; +) iu phi chng minh.
t

1
Ch . Ta c th chng minh trc tip hm s y= 1 + ng bin trn khong
t
( 0;+ ) .
x +1

1
1
+ Vi mi x dng ta c 1 + < e < 1 + .
x
x

V d 2. Cho s nguyn dng n. Chng minh rng

1
vi mi x ( 0;1) .
2n.e

xn . 1 x <

Li gii
Bt ng thc tng ng vi

1
1
(*)
x 2n (1 x) <
2ne
2n.e

xn . 1 x <

Xt hm s f=
( x) x 2n (1 x) vi x ( 0;1) , ta c

f =
( x) x 2n1 2n (2n + 1) x .
Suy ra f(x) i du t dng sang m khi i qua x =

2n
.
2n + 1

(2n) 2 n
2n
.
V vy =
max f ( x) f=

(0;1)
2n + 1 (2n + 1) 2 n +1

Ta chng minh

(2n) 2 n
(2n + 1)

2 n +1

2n + 1

2n

<

1
2ne

2 n +1

2n

2n + 1

2 n +1

<

1
e

> e (2n + 1) [ ln(2n + 1) ln(2n) ] > 1 .

1
(2) .
2n + 1
p dng nh l Lagrange cho hm s f ( x) = ln x trn [2n; 2n+1] suy ra tn ti
ln(2n + 1) ln(2n) >

c [ 2n;2n + 1] thuc sao cho f ( x) =


340

f (2n + 1) f (2n)
.
2n + 1 2n

www.TaiLieuLuyenThi.com
Cty TNHH MTV DVVH Khang Vit

1
1
>
(3) .
e 2n + 1
T (1), (2) v (3) suy ra pcm.
C. BI TP RN LUYN
Bi 1. Chng minh cc bt ng thc sau

Suy ra ln(2n + 1) ln(2n) =

a) x

x3
x3 x5
vi mi x > 0 .
< sin x < x +
6
6 120

b) 1

x2
x2 x4
vi mi x l s thc dng.
< cos x < 1
+
2
2 24

c)

1
2

sin x

<

1
x

+1

d) 1 < x.cos 2 x <


vi mi x 0; .

+2
4


vi mi x 0; .
4

e x
x4
e) 1 x
vi mi x thuc on [0;1].
1 x +
2 (1 + x )
x +1

Bi 2. Chng minh rng


4
4

a) Vi mi x 0; ta c sin x x 2 x 2 .
2

b) 3 x x3 <

2

vi x 0; .
sin 2 x
2

Bi 3. Tm gi tr nh nht ca hm s =
f ( x)

sin 3 x cos3 x

trn khong 0; .
+
cos x
sin x
4

Bi 4. Cho s nguyn dng n. Chng minh rng

k (2k 1) < 2ln 2 .


k =1

n
1
Bi 5. Chng minh rng: 1 + ln n > > ln(n + 1), n N * .
i =1 i

Bi 6. Tm GTNN ca hm s f ( x=
)
x

1
x x

x2 + x + 1 + x2 x + 1 .

1
x x

2
> , x ( 0;1) .
e
Bi 8. Chng minh rng vi mi x ta c

Bi 7. Chng minh rng

17 cos 2 x + 4cos x + 6 + cos 2 x 2cos x + 3 2 + 11 .


341

www.TaiLieuLuyenThi.com
Khm ph t duy K thut gii bt T Bi ton Max Min ng Thnh Nam

Bi 9. Tm gi tr ln nht, gi tr nh nht ca hm s y =
khong (1;6] .

1+ x + 3
trn na
x 1

Bi 10. Tm gi tr ln nht v gi tr nh nht ca hm s y =


( x + 1) 1 x 2 .
Bi 11. (TSH Khi D 2010) Tm gi tr nh nht ca hm s
y = x 2 + 4 x + 21 x 2 + 3 x + 10 .

Bi 12. Tm gi tr nh nht ca hm s y =x +

11
7
+ 2 1 + 2 trn khong
2x
x

( 0;+ ) .
Bi 13. (TSH Khi B 2003) Tm gi tr ln nht, gi tr nh nht ca hm s
y =x + 4 x 2 .

Bi 14. (TN THPT 2014) Tm gi tr ln nht v nh nht ca hm s


1 2
=
y
x x 4 x x2 .
4

Bi 15. (TSH Khi D 2003) Tm gi tr ln nht, gi tr nh nht ca hm s


x +1
trn on [ 1;2] .
y=
x2 + 1
Bi 16. Tm gi tr ln nht, gi tr nh nht ca hm s y = e x 1 x
on [ 0;1] .
Bi 17. Tm gi tr nh nht, gi tr ln nht ca hm s y =

ln x
x2 + 1

x2
trn
2

trn on

1
2 ;1 .

Bi 18. (TSH Khi B 2004) Tm gi tr nh nht ca hm s y =

ln 2 x
trn on
x

1;e3 .

Bi 19. Tm gi tr ln nht v gi tr nh nht ca hm =


s y sin 5 x + 3 cos x .
Bi 20. Chng minh rng e x + cos x 2 + x

342

x2
vi mi s thc x.
2

www.TaiLieuLuyenThi.com
Cty TNHH MTV DVVH Khang Vit

Bi 21. Chng minh rng 1 + x ln x + x 2 + 1 x 2 + 1 vi mi s thc x.

Bi 22. Chng minh rng ln 1 + 1 + x 2 < ln x +


Bi 25. Cho hm s

1
vi mi s thc dng x.
x

2
x + 1) 2
(
. Tm gi tr ln nht v gi tr nh nht
f ( x) =

x2 + 1
ca hm
s y f ( x). f (1 x) trn on [ 1;1] .
=

x
2
Bi 26. Tm gi tr ln nht v gi tr nh nht ca hm s y =
trn
x
cos x + 2sin
2

on 0; .
2
Bi 27. Tm gi tr ln nht v gi tr nh nht ca hm s
sin x + 2cos

x +1 + 2 3 x + 2
.
2 x +1 + 3 x +1
Bi 28. Tm gi tr ln nht v gi tr nh nht ca hm s
y=

y = x 2013 + 2015 x 2 .
sin x

1+

cos x

1
2

.
Bi 29. Chng minh rng vi mi s thc x ta c 3 2
+2
2
Bi 30. Tm tt c cc s thc a bt ng thc sau lun ng vi mi x khng
m ln (1 + x ) x ax 2 .
Bi 31. Tm tt c cc gi tr ca k bt ng thc sau ng vi mi s thc a v
b khng m e a +b 1 + a + b + k ( a b ) .
2

Bi 31. p s: k= e 2 .
Bi 32. Chng minh rng vi mi x > 2 ta c ( x + 1) cos
Bi 33. Chng minh rng vi mi 0 < x < 1 ta c

x +1

x cos

> 1.

1
x x2
x
ln
ln

< 4 .
2
2
1 x
x x x + 1

Bi 34. Chng minh rng vi mi s nguyn dng n 3 ta c n n +1 ( n + 1) .


n

Bi 35. Cho tam gic ABC c A B C < 900 .


2cos3C 4cos 2C + 1
Chng minh
2.
cos C
343

www.TaiLieuLuyenThi.com
Khm ph t duy K thut gii bt T Bi ton Max Min ng Thnh Nam

Bi 36. Cho a,b,c l cc s thc tho mn iu kin a 6, b 8, c 3 . Chng


minh vi mi x 1 ta c x 4 ax 2 bx c 0 .

1
Bi=
37. Cho I n x tan xdx, n 1 . Chng minh I n >

n+ 2 4
0
4

n+2

Bi 38. (i hc khi A, 2004) Cho tam gic ABC khng t, tha mn iu kin
cos 2 A + 2 2 cos B + 2 2 cos C =
3 (1) .

Tnh cc gc ca tam gic ABC.


D. HNG DN GII P
1
1 sin 2 2 x
2 sin 2 2 x
2
.
=
1
sin 2 x
sin 2 x
2
(1 sin 2 x )( 2 + sin 2 x ) + 1 1, x 0;
=

sin 2 x
2
sin 4 x + cos 4 x
Bi 3. =
Ta c f ( x)
=
sin x cos x

Vy gi tr nh nht ca f(x) bng 1 t ti x =

x 2 x3
x 2 n 1 x 2 n
Bi 4. Xt hm s f ( x)= ln(1 + x) x
+
... +
+

2
3
2n 1 2n

trn [ 0;+ ) . Hm s ng bin trn [ 0;+ ) suy ra f ( x) > f (0) , pcm.


1
Bi 5. Xt f ( x) =
ln x f '( x) = v f '( x) nghch bin trn (0 : +) .
x
Tng t bi ton trn ta c:
n

f (n) f (1) + f '(1) > f '(i ) > f (n + 1) f (1), n N *


i =1

1
1 + ln n > > ln(n + 1), n N *
i =1 i

Bi 6. Xt hm s f ( x)=

344

x 2 + x + 1 + x 2 x + 1 trn R. Ta c

www.TaiLieuLuyenThi.com
Cty TNHH MTV DVVH Khang Vit

x+

f ( x) =

1
2

1
3

x+ +
2
4

1
2

x+

1
3

x +
2
4

1
2
2

1
3

x+ +
2
4

1
x
2
2

3
1

x +
4
2

= g x + + g x
2
2

vi mi x, trong=
g (t )

, t R.
3
t +
4
V hm g ng bin trn R nn f ( x) = 0 x = 0
2

f ( x) > 0 g x + > g x x > 0


2

iu chng t f(x) t cc tiu ti x = 0. V vy min f ( x) = 2 x = 0 .

Bi tp tng t
Tm tp gi tr ca hm s f ( x)=

x2 + 2 x + 4 x2 2 x + 4 .

Bi 7. HD: Xt hm s
x

x
x
f (=
x) x1 x + x1=
x1 x + x.x1=
(1 + x) x1 x , x ( 0;1) .

Bi
9. Ta c y '
=

x 7 2 x + 3
2 x + 3 ( x 1)

< 0, x (1;6] .

Do hm s nghch bin trn (1;6] .


V vy y=
y=
(6)
min

4
, khng tn ti max.
5

Bi 10. Tp xc nh: D =
Ta c y ' =

2 x 2 x + 1
1 x2

[ 1;1] .
1

x=

; y ' = 0 2x + x 1 = 0
2 .

1
x

1 3 3
Ta c y (1)= y (1)= 0; y =
.
4
2
1 3 3
Vy ymax = y =
; ymin = y (1) = 0 .
4
2
x 2 + 4 x + 21 0
Bi 11. iu kin:
2 x 5 .
2
x + 3 x + 10 0
345

www.TaiLieuLuyenThi.com
Khm ph t duy K thut gii bt T Bi ton Max Min ng Thnh Nam

Xt hm s y = x 2 + 4 x + 21 x 2 + 3 x + 10 trn on [-2;5] ta c
y'

2 x + 4

2 x + 4 x + 21
2

2 x + 3
2 x 2 + 3x + 10

y ' =0 ( 2 x + 4 ) x 2 + 3 x + 10 =
( 2 x + 3) x 2 + 4 x + 21
( 2 x + 3)( 2 x + 4 ) 0
1

x=
2
2
2
2
3
( 2 x + 3) x + 4 x + 21
( 2 x + 4 ) x + 3 x + 10 =

1
Ta c y (=
2) 3; y =

3
1
Vy =
ymin y=

3

2
Ch . y > 0; y=

2; =
y (5) 4 .

2.

( x + 3)( 5 x ) ( x + 2 )( 7 x ) )

+22 y 2.

Kho st hm s cho ta c kt qu max.


Bi 12. Ta c
1
y' =

11
2 x2

14

7
1+ 2
x

y ' = 0 2 x 2 11

( 2x
=

11

x 2 + 7 28

2 x2 x2 + 7

x 2 + 7 28 = 0 x = 3

Ta c y i du t m sang dng khi i qua x = 3 nn y t cc tiu ti x = 3 .


15
.
V vy y=
y=
(3)
min
2
Bi tp tng t
Tm gi tr nh nht ca hm s y = 2 x +
Bi 13. Tp xc nh: D =
Ta c y ' =1

x
4 x

1
1

+ 2 1 + 2 vi x > 0 .
x
x

[ 2;2] .
; y ' =0 4 x 2 =x x = 2 .

Suy ra y (2) =
2; y ( 2) =
2 2; y (2) =
2.
Vy ymax =
y ( 2) =
2 2; ymin =
y (2) =
2 .
Bi 14. Tp xc nh: D = [ 0;4] .

346

www.TaiLieuLuyenThi.com
Cty TNHH MTV DVVH Khang Vit

Ta c y ' =

1
1
2 x
1
x 1
= ( x 2) +

2
4 x x2
4 x x2
2

; y ' = 0 x = 2 .

Suy ra y (0) =
0; y (2) =
3; y (4) =
0.
V vy y=
y=
(0) y=
(4) 0; y=
y=
(2) 3 .
max
min
Bi 15. Ta c y ' =

1 x

x2 + 1

x2 + 1

Suy ra y (=
1) 0; y=
(1)
V vy ymax = y (1) =

; y ' = 0 x =1.

2; y (2)
=

3
.
5

2; ymin = y (1) = 0 .

Bi 16. Ta c
y '= e x 1 x; y ''= e x 1 0, x [ 0;1]

.
5
y ' y '(0) =0 ymax =y (1) =e ; ymin =y (0) =0
2
Ch . e x 1 + x +

x2
xn
+ ... + , x 0, n N * .
2
n!

Bi 17. Ta c
1 2
1
x + 1 2 x ln x x + 2 x ln x
2 2 x ln x
1
x
x
=
y'
=

> 0, x ;1 .
2
2
2
2
x2 + 1
x2 + 1
x2 + 1

4ln 2
1
Suy ra ymin =
y =

; ymax =
y (1) =
0.
5
2

x =1
ln x ( 2 ln x )
ln x = 0
.
Bi 18. Ta c y ' =2
; y' =
0

2
x
ln x = 2
x = e

Suy ra
=
y (1) 0;=
y (e 2 )
2
V vy ymax
= y (e=
)

4
e
4

=
; y (e3 )

9
e3

; ymin
= y=
(1) 0 .
e2
Bi 19. V hm s cho tun hon vi chu k T = 2 nn ta ch cn tm gi tr
ln nht v gi tr nh nht trn on [ 0;2 ] .

Ta c =
y ' 5sin 4 x cos x 3 sin=
x sin x 5sin 3 x cos x 3 ;

y' =
0 sin x 5sin 3 x cos x 3 (1)

Ch .
347

www.TaiLieuLuyenThi.com
Khm ph t duy K thut gii bt T Bi ton Max Min ng Thnh Nam

1
1 3cos 2 x + 3sin 2 x
3
.
sin x cos=
x
3cos 2 x.sin 2 x.sin 2 x.sin 2 x
<
3
3
4
27

V vy (1) sin x = 0 x = k x = 0, x = , x = 2 .
Suy ra y (0) = y (2 ) =

3; y ( ) = 3 .

Vy ymax = y (0) = y (2 ) =

3; ymin = y ( ) = 3 .

Bi 20. Xt hm s y = e x + cos x +

x2
x 2 ta c
2

y ' = e x sin x + x 1; y '' = e x cos x + 1 > 0, x .

Mt khc y '(0) = 0 nn y ' > 0, x ( 0; + ) ; y ' < 0, x ( ;0 ) .


Do hm s t cc tiu ti x = 0 . V vy y y (0) =
0 (pcm).

Bi 21. Xt hm s y =1 + x ln x + x 2 + 1 x 2 + 1 ta c

y' =
ln x + x 2 + 1 ; y ' =
0 x + x2 + 1 =
1 x =
0.

Ta c y i du t m sang dng khi i qua x = 0 nn t cc tiu ti x = 0 .


V vy y y (0) =
0 (pcm).
x 2 (1 x ) + 8 x (1 x ) 2
2

Bi 25. Tnh c y =

x 2 (1 x ) 2 x (1 x ) + 2
2

t 2 + 8t 2
1
t =
.
t x (1 x ) 2; , x [ 1;1] khi y = 2
4
t 2t + 2

Ta c y ' =

2 5t 2 4t 6

(t

2t + 2

) ; y ' = 0 5t

1
t 2;
4

4t 6 = 0 t =

2 34
.
5

2 34
1 1
.
Tnh cc gi tr suy ra ymin =
y
4 34; ymin =
y =
=
5
4 25

3x
sin 1

2
Bi
26. Ta c y '
=
0, x 0; .
2
2
x

cos x + 2sin
2

2

V vy ymin = y (0) = 2; ymax = y = 1 +
.
2
2
348

www.TaiLieuLuyenThi.com
Cty TNHH MTV DVVH Khang Vit

[ 1;3] .

Bi 27. Tp xc nh: D =
Do

x +1

) +(

Khi y =

3 x

2t 2 4t 6
t 2 8t 3

= 4 t [ 0;1] | x + 1 =

4t
t2 +1

, 3 x=

2 1 t2
t2 +1

).

.
4
; ymax
= y=
(0) 2 .
5

T d c ymin
= y=
(1)

Bi 28. Tp xc nh: D = 2015; 2015 .


Ta c y ' =
2013 +

2015 2 x 2
2015 x

; y ' =
0
2 x 2 2015 =
2013 2015 x 2 .

2 x 2015 0

2
x=
2014
= 20132 2015 x 2
2 x 2015
2

Suy ra ymax =
y ( 2014) =
2014 2014; ymin =
y ( 2014) =
2014 2014 .
Bi 29. t
=
y 2

sin x

Ta c y ' =2t ln 2

+2

v t sin x , t [ 0;1] khi =


=
y 2t + 2

cos x

t
1 t2

1t 2

ln 2; y ' =0 2t =

t
1 t2

1t 2

1t 2

0 < t < 1

2t 2 1t 2
(1)
=
1 t2
t

Xt hm s f (a ) =
=
f '(a )

2a
trn khong ( 0;1) ta c
a

2a ( a ln 2 1)
a2

V vy (1) f (t ) = f

< 0, a ( 0;1) . Do f(a) nghch bin trn khong ( 0;1) .

( 1 t ) t =

1 1+
Ta c y=
(0) y=
(1) 3; y =
2
2

1
2

1
.
2

1 t2 t =

1 1+
Suy ra ymin
= y=
= y =
(0) y=
(1) 3; ymax
2
2

1
2

(pcm).

349

www.TaiLieuLuyenThi.com
Khm ph t duy K thut gii bt T Bi ton Max Min ng Thnh Nam

Bi tp tng t
Chng minh rng vi mi s thc x ta c 8 4

sin x

+ 4.2

cos x

5.4

1
5

Bi 30. Xt hm s f ( x)= ax 2 x + ln (1 + x ) trn [ 0;+ ) .


Ta cn tm a f ( x) 0, x 0 .
x
( 2ax + 2a 1) .
x +1
+ Nu a 0 f '( x) 0 f ( x) f (0) =
0 khng tho mn.

Ta c =
f '( x)

1
1 2a
.
f '( x) = 0 x = 0 hoc x =
2
2a
Khi f ( x) f (0) =
0 khng tho mn.

+ Nu 0 < a <

1
2ax + 2a 1 0 f '( x) 0 f ( x) f (0) =
0 tho mn.
2
1
Vy gi tr cn tm ca a l : a .
2

+ Nu a

Bi 32. HD: S dng nh l Lagrange cho hm s y = t cos

x
.
1 x
n +1
n
Bi 34. Ly logarit t nhin hai v ta cn chng minh
.

ln ( n + 1) ln n

Bi 33. Hng dn s dng nh l Lagrange cho hm s y = ln

ln x 1
x
vi x 3 ta c f=
'( x)
> 0, x 3 .
ln x
ln 2 x
T suy ra f (n + 1) > f (n) .

Xt hm s f ( x) =

1
Bi 35. a v chng minh y = 8t 3 8t 2 8t + 5 0, t = cos C 0; .
2

1

Bi 37. S dng tan x x, x 0; I n > 4 x n +1dx =

0
n+ 2 4
4
thm ch K thut nh gi bt ng thc Tch phn).
A 1

.
Bi 38. T gi thit 0 < A , suy ra 0 < sin
2
2
2

Ta c (1) 1 2sin 2 A + 2 2 ( cos B + cos C ) =


3
sin 2 A + 2 2 sin

350

A
BC
.cos
=1
2
2

( 2)

n+2

. (Xem

www.TaiLieuLuyenThi.com
Cty TNHH MTV DVVH Khang Vit

A
B C
A
.cos
2 2 sin
( 3)
2
2
2
A
T (2) v (3) ta c sin 2 A + 2 2 sin 1 0 .
2
A
t t = sin th bt ng thc tr thnh: 4t 2 1 t 2 + 2 2t 1 0
2

Li c 2 2 sin

( 4)

Xt hm f ( t ) = 4t 4 4t 2 + 2 2t 1 .
Ta phi c f ( t ) 0 vi 0 < t

1
5

(5).

f ( t ) = 16t 3 8t + 2 2; f ( t ) = 48t 2 8 f ( t ) = 0 t =

Suy ra f ( t ) < 0 0 < t <

1
6

; f ( t ) < 0

1
6

<t <

1
V vy f ( t ) f
> 0 nn f(t) ng bin trn
6

1
6

1
2

1
0;
.
2

1
Do f ( t ) f
0.
=
2

V vy du bng xy ra khi v ch khi t =

1
2

sin

A
=
2

1
2

A=

y = x + x 12, y 0 x + x 12 0 4 x 3
2

Thay vo ta c cos

BC

=1 B =C = .
2
4

CH 2: K THUT S DNG TNH N IU CHO BI TON

CC TR V BT NG THC HAI BIN S


A. NI DUNG PHNG PHP
Nguyn tc chung ca k thut ny l dng php th(rt bin x theo y hoc
ngc li) thay vo biu thc cn tm cc tr a v kho st hm mt bin. Tuy
nhin nu tinh t s dng cc bt ng thc c bn(tng v tch i xng vi hai
bin) v nhn ra cc php t c bit a v mt bin mi chung gia hai bin s
gip ta x l bi ton n gin hn.
1. K thut th bin a v kho st hm mt bin
Rt x theo y(hoc rt y theo x) thay vo biu thc cn tm cc tr(bt ng
thc) a v kho st hm mt bin(bt phng trnh).
Nh vy k nng cn c l bi ton kho st tnh n iu ca hm mt bin s.
351

www.TaiLieuLuyenThi.com
Khm ph t duy K thut gii bt T Bi ton Max Min ng Thnh Nam

V d 1. Cho x,y l hai s thc tha mn iu kin y 0, x 2 + x y 12 =


0.
Tm gi tr nh nht ca biu thc P = xy + x + 2 y + 17 = 0 .
Li gii
T gi thit ta c: y = x + x 12, y 0 x 2 + x 12 0 4 x 3 .
2

Khi P = x x 2 + x 12 + x + 2 x 2 + x 12 + 17 = x3 + 3 x 2 9 x 7 .
Xt hm s f ( x) = x3 + 3 x 2 9 x 7 lin tc trn on [ 4;3] ta c:
x =1
f '( x) =3 x 2 + 6 x 9; f '( x) =0
x = 3
Ta c f (4) =
13, f (3) =
20, f (1) =
12, f (3) =
20

max f ( x) =f (3) =f (3) =


20, min f ( x) =
f(1) =
12 .
x[ 4;3]
x[ 4;3]

Vy gi tr ln nht ca P bng 20 t ti ( x=
; y)
nht ca P bng 12 t ti ( x; y=
)

( 3;0 ) ; ( 3; 6 ) v gi tr nh

(1; 10 ) .

V d 2. Cho x,y l hai s thc tha mn iu kin 2 x y =


2.
Tm gi tr nh nht ca biu thc P =

x 2 + ( y + 1) + x 2 + ( y 3) .
2

Li gii
Theo gi thit ta c: =
y 2 x 2 thay vo biu thc ca P ta c
=
P

Xt hm s f (=
x)

5 x 2 4 x + 1 + 5 x 2 20 x + 25 .

5 x 2 4 x + 1 + 5 x 2 20 x + 25 lin tc trn ta c:

=
f '( x)

5x 2
5x 4x + 1
2

5 x 10
5 x 20 x + 25
2

f '( x) =
0 ( 5 x 2 ) 5 x 2 20 x + 25 =
( 5 x 10 ) 5 x 2 4 x + 1
( 5 x 2 )( 5 x 10 ) 0

2
2
2
2
( 5 x 2 ) 5 x 20 x + 25 = ( 5 x 10 ) 5 x 4 x + 1
2
2
x2
5
x=
3
24 x 2 16 x =
0

2
T suy ra P =f ( x) f =2 5 .
3
2
2
ng thc xy ra khi v ch khi x = , y = .
3
3
352

www.TaiLieuLuyenThi.com
Cty TNHH MTV DVVH Khang Vit

V d 3. Cho a,b l cc s thc dng tho mn iu kin a + b =


1.
Tm gi tr nh nht ca biu thc P =3 1 + 2a 2 + 2 40 + 9b 2 .
Li gii
Theo gi thit ta c a =1 b, ( 0 < b < 1) .
Khi P = 3 1 + 2 (1 b ) + 2 40 + 9b 2 .
2

Xt hm s f (b) = 3 1 + 2 (1 b ) + 2 40 + 9b 2 vi 0 < b < 1 ta c


2

=
f '(b)

6 ( b 1)
2b 4b + 3
2

18b
9b 2 + 40

f '(b) =0 (1 b ) 9b 2 + 40 =3b 2b 2 4b + 3

(1 b ) 9b +=
40 9b
2

( 2b

4b + 3

2
3
1
2
Vy gi tr nh nht ca P bng 5 11 t ti=
.
a =
;b
3
3
( b + 2 )( 3b 2 ) 3b 2 10b + 10 = 0 b =

V d 4. Cho a,b l hai s thc tha mn a 2 + b 2 =


1.
3 3
.
4
Li gii

Chng minh rng ab + max {a, b}

Nu a v b tri du khi ab + max {a, b} max {a, b} 1 <

3 3
ta c ngay iu
4

phi chng minh.


Vy a v b cng du v nu c a v b u m khi :
ab + max {a, b} ab

a 2 + b2 1 3 3
, ta c ngay iu phi chng minh.
= <
2
2
4

Xt vi a, b 0 khi khng mt tnh tng qut gi s a = max {a, b} ta cn


chng minh: ab + a

3 3
.
4

T iu kin ta c: a =

1 b 2 ab + a = f (b) = ( b + 1) 1 b 2 .

Xt hm s f (b) =
( b + 1) 1 b2 trn on [0;1] ta c:
f '(b) =

(1 + b )(1 2b ) ; f '(b) = 0 b = 1 .
1 b2

353

www.TaiLieuLuyenThi.com
Khm ph t duy K thut gii bt T Bi ton Max Min ng Thnh Nam

Ta c f(b) i du t dng sang m khi i qua b =


tr cc i hay f (b)

1
1 3 3
nn f =
l gi
4
2
2

3 3
. Ta c ngay iu phi chng minh. ng thc xy
4

3
1
=
,b
.
2
2
Cch 2: Thc hin lp lun nh trn v s dng bt ng thc AM-GM ta c:

ra khi v ch khi
=
a

ab + a= a ( b + 1)=

1
3 1

. 2. .a 2. .b + 1
3 2 2

5
2 3
a + + b2 +
1 2 3 2 5 1
3 3
4
4 =

a + b +
4
4
2
4
3
3

2. K thut x l vi biu thc i xng hai bin


Bc 1: T iu kin t t= x + y (hoc t = xy ) rt t theo xy(hoc x+y).
Bc 2: Thay vo biu thc ca P ta c mt hm s vi t.
Bc 3: Tm min gi tr ca t da vo bt ng thc c bn ( x + y ) 4 xy (hoc
2

iu kin rng buc ca bin bi cho). Ta tm c min gi tr ca t l [ a; b ] .


Bc 4: Xt hm s f(t) trn on [ a; b ] t tm c gi tr ln nht, nh nht
ca f(t) v suy ra gi tr ln nht, nh nht ca P.
Ch . i khi cn nh gi P thng qua cc bt ng thc c bn trc tin
gim s cng knh ca hm f(t) gip bi ton thc hin n gin hn. Mt s bi
ton c th thc hin theo nh gi iu kin c nghim ca phng trnh bc 2.
Du hin nhn bit:
- Rt c bin ny theo kin kia v a biu thc v dng mt bin.
- Biu thc c dng i xng gia tng v tch.
- Biu thc c dng i xng gia cc bin.
- Biu thc l tng hp ca cc hm(hm a thc,cn thc, lng gic, m v
logarit).
Mt s nh gi c bn:
a) Vi hai s thc khng m x,y ta c:

t2
,0 t 2 x 2 + y 2 .
4
a
b
2
.
b) Vi a,b dng ta c
+

b + ab a + ab 1 + ab
t = x + y xy

354

www.TaiLieuLuyenThi.com
Cty TNHH MTV DVVH Khang Vit

V d 1. Cho x,y l hai s thc khng m thay i tha mn iu kin

4 x 2 + y 2 + xy 1 + 2 ( x + y ) .

Tm gi tr ln nht ca biu thc P = xy + x + y x 2 y 2 .


Li gii
2

1
2
x+ y
2
2
Nhn xt. Ta c: xy
, x y ( x + y) .
2
2
Do vy tm gi tr ln nht ca P ta quy v n t= x + y v vic u tin cn
lm l tm khong gi tr ca t= x + y .
2

1
1
2
2
x+ y
Ta c: P
( x + y) + x + y .
+ x + y ( x + y) =
2
4
2
1
t =
t
x + y , (t 0) P t 4 + t .
4
Xut pht t iu kin bi ton ta c:
2
2
2
4 ( x + y ) xy 1 + 2 ( x + y ) 4 ( x + y ) 2 ( x + y ) 1 4 xy ( x + y ) .

1
Suy ra 3t 2 2t 1 0 t 1 do t > 0 nn 0 < t 1 .
3
1
Xt hm s f (t ) =
t 4 + t vi 0 < t 1 ta c:
4
3
3
f '(t ) =t 3 + 1 0, t ( 0;1] f (t ) f (1) = P .
4
4
3
1
Vy gi tr ln nht ca P bng t ti x= y=
.
4
2
V d 2(TSH Khi D 2014) Cho hai s thc x,y tho mn cc iu kin
1 x 2;1 y 2 .

Tm gi tr nh nht ca biu thc P =

x + 2y
x + 3y + 5
2

y + 2x
y + 3x + 5
2

1
.
4 ( x + y 1)

Li gii
Do 1 x 2 nn ( x 1)( x 2 ) 0 x 2 + 2 3 x .
Tng t ta c y 2 + 2 3 y .
Suy ra P
=

1
x + 2y
y + 2x
+
+
3x + 3 y + 3 3x + 3 y + 3 4 ( x + y 1)
1
x+ y
+
x + y + 1 4 ( x + y 1)
355

www.TaiLieuLuyenThi.com
Khm ph t duy K thut gii bt T Bi ton Max Min ng Thnh Nam

t t = x + y, ( 2 t 4 ) ta c P f (t ) =

t
1
.
+
t + 1 4 ( t 1)

V d 3. Cho x,y l hai s thc dng.


Tm gi tr nh nht ca biu thc P =

1
x

1
y

5
+ ln(xy) .
2
x +y
2

Li gii
Nhn xt. Biu thc i xng vi x v y d on du bng xy ra ti x = y, khi
1
x

1
y

1
x +y
2

5
2x

5
1
1
1
5
nn ta nh gi 2 + 2 + 2
vi
.
2
2 xy
2xy
x
y
x +y

Ta c:
1
x

1
y

1
x +y
2

5 1
1
2 1
1
= 2 + 2 + 2

2
xy x + y
2 xy x
2 xy
y

1
1
=
( x y ) 2 2
2 2
2
2
x y
x y
2 xy x + y
2 xy x 2 + y 2

( x y )2 2 x 2 + y 2 xy ( x y )2 [ 2.2 xy xy ]
=

0
2 x2 y 2 x2 + y 2
2 x2 y 2 x2 + y 2
2
2
x y)
x y)
(
(
=

(
(

Vy P
t
=
t

)
)

5 1
1
5 1
+ ln( xy ) = ln .

2 xy
2 xy
xy

5
1
, ( t > 0 ) khi P f (t ) = ( t ln t ) .
2
xy

Xt hm s f =
(t )
f '(t ) =

5
( t ln t ) vi t > 0 ta c:
2

5 1
1 ; f '(t ) = 0 t = 1 .
2 t

Ta c f(t) i du t m sang dng khi i qua t = 1 nn f(t) t cc tiu ti


5
.
t = 1 hay P f (t ) f (1) =
2
ng thc xy ra khi v ch khi x= y= 1 .
Vy gi tr nh nht ca P bng

356

5
t ti x= y= 1 .
2

www.TaiLieuLuyenThi.com
Cty TNHH MTV DVVH Khang Vit

V d 4. Cho x,y l hai s thc dng tha mn iu kin 3 xy + 3 = x 4 + y 4 +


Tm gi tr ln nht ca biu thc
=
P x2 y 2 +

16
x + y2 + 2
2

2
.
xy

Li gii
Xut pht t gi thit ta c: 3 ( xy + 1) = x 4 + y 4 +

2
2
.
2 x2 y 2 +
xy
xy

2 x3 y 3 + 2 3 x 2 y 2 + 3 xy 2 x3 y 3 3 x 2 y 2 3 xy + 2 0
1
( xy + 1)( 2 xy 1)( xy 2 ) 0 xy 2
2
16
8
Khi P x 2 y 2 +
.
= x2 y 2 +
2 xy + 2
xy + 1

8
1

.
t
=
t xy, t 2 khi P f (t ) =
t2 +
t +1
2

Xt hm s f (t=
) t2 +
f '(t ) = 2t

( t + 1)2

8
lin tc trn
t +1

1
2 ;2 ta c:

; f '(t ) = 0 t = 1 .

20
11
20
1 67
Ta c=
suy ra P f (t ) f(2) =.
f =
, f (1) =
, f (2)
3
3
3
2 12
20
Vy gi tr ln nht ca P bng
t ti x= y= 2 .
3
V d 5. Cho x, y l cc s thc dng tha mn iu kin x + y + xy =
3.

Tm gi tr nh nht ca biu thc:=


P 4(

x +1 3
y +1 3
) + 4(
) + x2 + y 2 .
y
x

Li gii
S dng bt ng thc a3 + b3 ab ( a + b ) ta c
x +1 y +1 x +1 y +1
2
2
P 4
+

+ x + y

y
x
y
x

16 x 2 + y 2 + x + y
2 2

x y

)+

x 2 + y 2 64 + 2

357

www.TaiLieuLuyenThi.com
Khm ph t duy K thut gii bt T Bi ton Max Min ng Thnh Nam

3. a thc c dng ng cp
Dng ton ny cp n trong ch k thut s dng tnh ng cp l
t x = t. y hoc y = t.x a v kho st hm mt bin vi t.
Dng ton ny vi nm tr li y thng xut hin trong thi TSH ca B v
k thut kh n gin nn cc em nm chc phng php c th hon ton x l c.
V d 1. Cho x,y l hai s thc khc 0. Tm gi tr nh nht ca biu thc
P=

x4
y

y4
x

2(

x2
y

y2
x

)+

x y
+ .
y x

Li gii
4

x y
x y
x y
Ta c P = + 6 + + + + 6 .
y x
y x
y x
x y
t t =+ , t 2 khi P = t 4 6t 2 + t + 6 .
y x

Xt hm s f (t ) = t 4 6t 2 + t + 6 trn ( ; 2] [ 2; + ) , ta c
f '(=
t ) 4t 3 12t + 1; f ''(=
t ) 12t 2 12 > 0, t ( ; 2] [ 2; + ) .

Do f '(t ) f '(2) = 7, t ( ; 2]; f '(t ) f '(2) = 9, t [ 2; + ) .


V vy f(t) nghch bin trn ( ; 2] v ng bin trn [ 2;+ ) .
Suy ra f (t ) min { f (2), f (2)} =f (2) =
4 .
Vy gi tr nh nht ca P bng -4 t ti x = y .
V d 2. Cho x,y l hai s thc dng phn bit tha mn iu kin x 2 + 2 y =
12 .
Tm gi tr nh nht ca biu thc P =

4
x

4
y

5
8( x y )

Li gii
Ta thy P cha c dng ng cp tuy nhin kho lo s dng iu kin
Ta c: 12 = x 2 + 2 y = x 2 + y + y 3 3 x 2 y 2 xy 8 .
Suy ra
P

1 2 2 4
4 xy
5
1 x2 y 2 5
xy
x y 4 + 4 + .
=
2 + 2 + .
2
64
16 y
y 8 8( x y )
x 64 ( x y )2
x

2
5
1 x y
1

=
+ 2 + . x
16 y x
64 2 + y

y
x

358

www.TaiLieuLuyenThi.com
Cty TNHH MTV DVVH Khang Vit

x y
t t =+ , ( t > 2 ) , do x,y dng phn bit.
y x

1 2
5 1
.
t 2 + .
64 t 2
16
1 2
5 1
trn ( 2;+ ) ta c
Xt hm s f (=
t)
t 2 + .
64 t 2
16

Khi P f (=
t)

f '(=
t)

t
5
5
2
t >2
.

=
; f '(=
t ) 0 8t ( t 2 ) =
5 0
t
2
8 64 ( t 2 )
2

5
5
f(t) i du t m sang dng nn ti t = f(t) t cc tiu hay
2
2
5 27
.
f (t ) f =
2 64
27
t ti=
Vy gi tr nh nht ca P bng
x 2,=
y 4.
64
V d 3. Cho x,y l hai s thc khc 0. Chng minh rng

Ti t =

48 x 2 y 2

(x

+ y2

x2 y 2
+ 1001 2 + 2 2014 .
y
x

Li gii
Bt ng thc cho tng ng vi:
2
x y 2

x y
48
+ 2 2014
+
+
+
1001
1001

4016 .
2
2
y x

y x
x y
x y
+
+
y x
y x
Ch s dng bt ng thc AM GM ta c

48

x y
x y
48
.3 + =
24
+ 3 + 2
2
2
y x
x y
x y y x
+
+
y x
y x
48

x y
998 + 998.22 =
3992
y x
Cng theo v hai bt ng thc trn ta c pcm.
4. K thut kho st hm c trng
Nu bt ng thc cn chng minh c dng hm c trng f (a ) > f (b) hoc
f (a ) < f (b) .
359

www.TaiLieuLuyenThi.com
Khm ph t duy K thut gii bt T Bi ton Max Min ng Thnh Nam

Khi ta chng minh hm s f ( x) lun ng bin hoc lun nghch bin trn
min D thuc iu kin ca bi ton.
V d 1. (TSH Khi D 2007) Cho a b > 0 . Chng minh rng
b

a 1 b 1
2 + a 2 + b .
2
2

Li gii
Bt ng thc cho tng ng vi:

(1 + 4 ) (1 + 4 )
a b

b a

Xt hm s f ( x) =

b ln 1 + 4

ln 1 + 4 x
x

f '( x)

) (
(1 + 4 )

x2

ln 1 + 4a
a

) vi x > 0 ta c

4 ln 4 1 + 4 ln 1 + 4 x
x

) a ln (1 + 4 )
b

) ln (1 + 4 ) .
b

) < 0, x > 0 .

Do f(x) l hm nghch bin trn ( 0;+ ) . V vy vi a b f (a ) f (b) .


Bt ng thc c chng minh. ng thc xy ra khi v ch khi a = b .
V d 2. Chng minh rng vi mi x, y ( 0;1) , x y ta c
1
y
x
ln
ln
>4
y x 1 y
1 x

(1) .

Li gii

Nu y > x th
y
x
y
x
ln
> 4 ( y x ) ln
4 y > ln
4x .
(1) ln
1 y
1 x
1 y
1 x

Nu y < x th
y
x
y
x
ln
< 4 ( y x ) ln
4 y < ln
4x .
(1) ln
1 y
1 x
1 y
1 x
Xt hm s=
f ( t ) ln
=
f (t )

t
4t vi t ( 0;1) . Ta c
1 t

1 t t
(2t 1) 2
=

4
> 0, t ( 0;1) .

t 1 t
t (1 t )

Suy ra f(t) tng trn (0;1). Suy ra f(y) > f(x) nu y > x v f(y) < f(x) nu y < x

360

1
y
x
ln
ln
> 4, x, y ( 0;1) , x y .
y x 1 y
1 x

www.TaiLieuLuyenThi.com
Cty TNHH MTV DVVH Khang Vit

Bt ng thc c chng minh.


Ch . Theo nh l Lagrange cho hm s y = ln

t
ta c
1 t

f ( y ) f ( x)
1
1
4
VT =
=
f '(c) =+

=
4 (pcm).
yx
c 1 c c +1 c

5. K thut xt ring l tng bin


Ni dung k thut ny ti s cp chi tip hn trong ni dung ch kho st
hm nhiu bin. Loi ton ny thng p dng vi cc bi ton cc tr khi cc bin
thuc on gi tr cho trc hoc c iu kin rng buc gia cc bin vi nhau.
V d 1. Xt hm s f ( x, y ) =
(1 x )( 2 y )( 4 x 2 y ) trn min

{( x, y ) | 0 x 1,0 y 2} .

=
D

Tm gi tr nh nht ca hm f trn min D.


Li gii
Bin i hm s cho thnh

f ( x, y ) = 2 (1 x )( 2 y )( (2 y ) 2(1 x) )

t u= 1 x, v = 2 y, ta chuyn v tm GTNN ca hm s:
=
E
F ( u, v ) =
2uv 2 + u 2 v trn min

{( u, v ) ,0 u 2,0 v 1} , ngha l

min F =
(u , v) min min (2uv 2 + u 2 v) .
0v1
0u 2
E

Xt hm s g ( v ) =
2uv 2 + u 2 v vi 0 v 1 , coi u l tham s. Ta c
g(v) =
4uv + u 2 =
u (4v + u ) .

Ta thy g(v) = 0 khi v0 =


dng sang m, suy ra

u 1
u
u
, m 0 v qua v0 = th g(v) i du t
4 2
4
4

m=
ax g ( v ) min { g (=
0 ) , g (1)} min 0; u 2 2u

0v 1

( do u 2 2u 0 ).

Vy min F (u , v) =
1 khi u = 1, v = 1. T
min (u 2 2u ) =
0u 2

min f ( x, y ) = 2 min F (u , v) = 2 khi x = 0, y = 1.


D

V d 2. Cho 0 < a, b 1 . Chng minh rng tan a.tan b tan ab .


Li gii
Gi s a b . t
=
f ( x ) tan b.tan x tan bx vi b x 1 . Ta c
( x)
f=

tan b
2

cos x

b
cos 2bx

.
361

www.TaiLieuLuyenThi.com
Khm ph t duy K thut gii bt T Bi ton Max Min ng Thnh Nam

suy ra f ( x ) > 0 , nn f
cos x cos 2bx
ng bin trn [b; 1]. V vy vi a b ta c f ( a ) f ( b ) . Suy ra
Do 0 < a, b 1 nn tan b > b > 0 v

tan a.tan b tan ab tan 2 b tan b 2

(1) .

t =
g ( x ) tan 2 x tan x 2 , c
( x)
g=

2 tan x
2x

> 0; x ( 0;1)
cos x cos 2 x

Suy ra g ( b ) > g ( 0 ) =
0 tan 2 b tan b 2 > 0

( 2)

T (1) v (2) suy ra pcm.


V d 3. Chng minh rng vi mi 0 < x <
cos ( x y )

v 0 < y <

4
4cos x cos y

( cos x + cos y )2

ta lun c

Li gii
x+ y
x y
Ta c cosx + cos y =
.
2cos
cos
2
2
Do vai tr x, y nh nhau nn ta c th gi s x y .
Khi vit li bt ng thc c vit li di dng
x+ y
x y
cos 2
cos 2
.cos ( x y ) cos x.cos y .
2
2
Ly logarit t nhin hai v ta c
x+ y
x y
2ln cos
+ 2ln cos
+ ln cos ( x y ) ln cos x + ln cos y .
2
2
n y ta xt hm s
f ( x) ln cos x + ln cos y 2ln cos
=

x+ y
x y

2ln cos
ln cos ( x y ) , x y; x, y 0;
2
2
4

Ta c f '( x) =
tan x + tan ( x y ) + 2 tan
=

x y
x+ y
.
+ 2 tan
2
2

sin y
x y
x+ y

+ 2 tan
+ 2 tan
> 0, x y; x, y 0;
cos x.cos ( x y )
2
2
4

Do f l hm tng. Do f ( x) f ( y ) =
0 , iu ny tng ng vi
x+ y
x y
+ 2ln cos
+ ln cos ( x y ) ln cos x + ln cos y .
2
2
Bi ton c chng minh hon ton. ng thc xy ra khi v ch khi x = y .
2ln cos

362

www.TaiLieuLuyenThi.com
Cty TNHH MTV DVVH Khang Vit

6. K thut s dng nh l Lagrange


V d 1. Cho hai s thc dng a, b tha mn a < b. Chng minh rng:
ba
b ba
< ln <
b
a
a
Li gii
1
Xt hm s f ( x=
) ln x f '( x=
)
, x (0; +).
x
f (b) f (a )
.
Theo nh l Lagrange lun tn ti c (a; b) sao cho f '(c) =
ba
1 ln b ln a
a b
b
V vy=
= ln
c
ba
c
a
ba
b ba
1 1 1
Mt khc 0 < a < b < c < <
(pcm).
< ln <
b
a
a
b c a
7. Kt hp s dng cc bt ng thc c bn AM GM ; Cauchy Schwarz
vic kho st hm s thun tin v n gin trc khi chng minh bt ng
thc hoc tm Min, Max ta nh gi qua cc i lng trung bnh ca hai bin s
x+ y
x2 + y 2
, xy ,
) hoc kt hp s dng mt s bt ng thc ph quen bit.
2
2
Sau y l mt s nh gi hay c s dng:
1
1
2
3
+
x 2 + y 2 ( x + y ) ; x3 + y 3 ( x + y ) .
2
4

+
+

e x x + 1, x 0;ln ( x + 1) x, x 0 .
x2 + y 2 + a 2 + b2

( a + x )2 + ( b + y )2 .

V d 1. (TSH Khi B 2006) Cho x,y l hai s thc thay i.


Tm gi tr nh nht ca biu thc P =

( x 1)2 + y 2 + ( x + 1)2 + y 2 +

y2 .

Li gii
Trn mt phng ta Oxy, xt cc im M(1-x; y) v N(1+x; y).
Ta c OM + ON MN, suy ra

( x 1)2 + y 2 + ( x + 1)2 + y 2

4 + 4 y2 .

ng thc xy ra khi x = 0, ta c P 2 1 + y 2 + y 2 .
Xt hm s f ( y )= 2 1 + y 2 + y 2 .
*) Vi y 2 th f ( y )= 2 1 + y 2 + y 2 l hm ng bin.
*) Vi y < 2 th f ( y )= 2 1 + y 2 + 2 y c =
f ( y )

2y
1 + y2

1 .
363

www.TaiLieuLuyenThi.com
Khm ph t duy K thut gii bt T Bi ton Max Min ng Thnh Nam

Khi f ( y ) = 0 y =

1
.
3

1
Lp bng bin thin ca hm s f(y) ta c min P =2 + 3 ( x; y ) =0; .
3

Ch . Ta c th s dng bt ng thc Mincopski:

( x 1)2 + y 2 + ( x + 1)2 + y 2 (1 x + x + 1)2 + 4 y 2=

2 y2 + 1 .

V d 2. (TSH Khi B 2011) Cho a,b l s thc dng tha mn iu kin

2 a 2 + b 2 + ab = ( a + b )( ab + 2 ) .

a 3 b3 a 2 b 2
Tm gi tr nh nht ca biu thc P = 4 3 + 3 9 2 + 2 .
b
a b
a

Li gii
a b
Nhn xt. P c th biu din theo t=
+ nn ta ngh n vic nh gi
b a
a b
t=
+ da vo iu kin bi ton.
b a
a b
t t=
+ , khi
b a
a b 3
a b 2

a b
P = 4 + 3 + 9 + 2 = 4t 3 9t 2 12t + 18 .
b a
b a
b a

Xut pht t iu kin ta c:


a
b
a b
1 1
1 1
2 + + 1= + ( ab + 2 ) 2 2ab + = 2 2
+
.
a
b a
a b
a b
b

Suy ra 2t + 1 2 2. t + 2 t

5
.
2

Xt hm s f (t ) = 4t 3 9t 2 12t + 18 trn ; + ta c:
2

f '(=
t ) 12t 2 18t 12 > 0, t

5
nn f(t) ng bin trn
2

2 ; + suy ra

23
5
P=
f (t ) f =
.
4
2

Vy gi tr nh nht ca P bng

364

23
t ti=
a 2,=
b 1 hoc =
a 1,=
b 2.
4

www.TaiLieuLuyenThi.com
Cty TNHH MTV DVVH Khang Vit

Bi tp tng t
1) Cho x,y l 2 s thc dng tha mn iu kin

6 x 2 + y 2 + 20 xy = 5 ( x + y )( xy + 3) .

Tm gi tr nh nht ca biu thc


x4 y 4
x3 y 3
x2 y 2
P = 9 4 + 4 16 3 + 3 + 25 2 + 2 .
y
y
y
x
x
x

2) Cho a,b l hai s thc dng tha mn iu kin

(a

+ 2b 2

)(

+ 3a 2b 2 =2 a 2 + b 2 a 2 + 2b 2 .

Tm gi tr nh nht ca biu thc


=
P

a 3 + b3
b3

( a + b )2 + 2a 2 + 5b 2 ( a b )2 + 2a 2 + 5b 2

.
+ 3 +
2
2
a
ab a + 2b
8b3

V d 3. Cho a,b l cc s thc dng tha mn iu kin


a+b+2

12 .
( a + 2 )( b + 2 ) =
a3
b3
48
.
+
+
b+2 a+2 a+b
Li gii

Tm gi tr nh nht ca biu thc P =

Theo gi thit ta c:

1
(12 a b )2 =
4

a+2+b+2
.
2

( a + 2 )( b + 2 )

t t = a + b, ( 0 < t < 12 ) suy ra (12 t ) ( t + 4 ) t + 4 12 t t 4 .


2

a 4 + b 4 + 2 a 3 + b3
a 3 ( a + 2 ) + b3 ( b + 2 )
48
48
.
Khi P
=
=
+
+
a+b
a+b
( a + 2 )( b + 2 )
( a + 2 )( b + 2 )

S dng bt ng thc AM-GM ta c a 4 + b 4

1
1
( a + b )4 ; a3 + b3 ( a + b )3 .
8
4

1
1
( a + b )4 + 2. ( a + b )3 48
4
Suy ra P 8
+
a+b
( a + 2 )( b + 2 )
=

( a + b )4 + 4 ( a + b )3 + 48
=
2
a+b
12 a b )
(
8.

t 4 + 4t 3
2 (12 t )

48
t

365

www.TaiLieuLuyenThi.com
Khm ph t duy K thut gii bt T Bi ton Max Min ng Thnh Nam

t 4 + 4t 3

Nhn xt. Nu ta xt hm
s f (t )
=

2 (12 t )

48
ny r rng y l mt hm s
t

phc tp nu o hm ln khng xc nh tnh dng m ca o hm trn khong


[ 4;+ ) . Do ta cn n gin hm s i bng cch nh gi
2 (12 t ) 2 (12 4 ) = 128
2

Vy ta c nh gi P f (t=
)

=
f (t )
Xt hm s

1
2 (12 t )

1
.
128

t 4 + 4t 3 48
+ .
128
t

t 4 + 4t 3 48
t 5 + 3t 4 1536
+ vi t =
4 ta c f '(t )
> 0, t 4 .
128
t
32t 2

Do f(t) ng bin trn [ 4;+ ) suy ra P f (t ) f (4) =


16 . ng thc xy ra
khi v ch khi a= b= 2 .
Vy gi tr nh nht ca P bng 16 t ti a= b= 2 .
B. BI TON CHN LC
Bi 1. Cho x,y l hai s thc tha mn iu kin x + y =
1

2x 4 + y + 1 .

Tm gi tr ln nht v gi tr nh nht ca biu thc


1
2
.
P = ( x + y) 9 x y +
x+ y
Li gii
Nhn xt. P l mt biu thc biu din theo t= x + y nn ta tm cch a v kho
st hm vi t= x + y nhng trc tin phi tm min gi tr ca t tha mn iu
kin bi ton.
t t= x + y xut pht t iu kin v bt ng thc C-S ta c:
x + y =
1

2 ( x 2) + y + 1

( 2 + 1)( x 2 + y + 1=)

3 ( x + y 1) .

t 1

1 t 4.
2
( t 1) 3 ( t 1)

Khi P = f (t ) = t 2 9 t +
Ta c f '(t )= 2t +

1
.
t

1
1
1
4t t 1

=
+
> 0, t [1;4] .
2 9 t 2t t 2 9 t
2t t

33
Suy ra max f (t ) =
5; min f (t ) =
f(4) =
f(1) =
22 2.
t[1;4]
t[1;4]
2
366

www.TaiLieuLuyenThi.com
Cty TNHH MTV DVVH Khang Vit

33
5 t ti=
x 4,=
y 0 . Gi tr nh nht
2
ca P bng 2 2 2 t ti x = 2, y = 1 .

Vy gi tr ln nht ca P bng

Bi 2. Cho x,y l hai s thc dng tha mn iu kin xy + x + y =


3.
Tm gi tr nh nht ca biu thc P = x 2 + y 2

xy
3x
3y
.

x + y y +1 x +1

Li gii
Vit li P =( x + y )

( x + y ) 2 xy + 3 ( x + y ) .
xy
2 xy
3.
x+ y
xy + x + y + 1
2

t >0
t t = x + y xy = 3 t . Ta c ( x + y ) 4 xy t 2 4 ( 3 t )
t 2 .
2

12 3
.
t 2
12 3
Xt hm s f (t ) = t 2 t trn [ 2;+ ) ta c
t 2

Khi P = t 2 t

f '(t) = 2 t 1 +

12
t2

t 2 ( 2t 1) + 12

t2

> 0, t 2 .

3
Do f(t) l hm ng bin trn [ 2;+ ) suy ra P =
f (t ) f (2) =
.
2
3
Vy gi tr nh nht ca P bng t ti x= y= 1 .
2
Bi 3. Cho x,y l hai s thc tha mn iu kin

(x

+ y 2 + 1 + 3x 2 y 2 + =
1 4 x2 + 5 y 2 .

Tm gi tr ln nht, gi tr nh nht ca biu thc P =

x 2 + 2 y 2 3x 2 y 2
x2 + y 2 + 1

Li gii
Nhn xt. n i lng x + y 2 + 1 t iu kin v biu thc ca P ta ngh
2

n vic bin i P theo x 2 + y 2 + 1 da vo iu kin bi ton.

T gi thit ta c: x 2 + y 2 + 1 + 3 x 2 y 2 + 1= 4 x 2 + y 2 + 1 + y 2 4

x 2 + y 2 + 1 4 x 2 + y 2 + 1 + 5 = y 2 3x 2 y 2 .
x + y + 1) + ( y 3 x y 1) ( x
(=
2

Do P

x2 + y 2 + 1

2 2

+ y 2 + 1 3 x2 + y 2 + 1 + 4
x2 + y 2 + 1

.
367

www.TaiLieuLuyenThi.com
Khm ph t duy K thut gii bt T Bi ton Max Min ng Thnh Nam

t t = x 2 + y 2 + 1 khi P =

t 2 3t + 4
. Ta cn tm min gi tr ca t.
t

Xut pht t iu kin ta c:

(x

+ y 2 + 1 + 3 x 2 y 2 + 1= 5 x 2 + y 2 + 1 x 2 5

(
(x

x2 + y 2 + 1 5 x2 + y 2 + 1 + 6 =
x 2 3x 2 y 2 0
2

)(

Vy xt hm s f (t ) =
f=
'(t)

+ y 2 + 1 2 x 2 + y 2 + 1 3 0 t [ 2;3]

t2 4
t2

t 2 3t + 4
lin tc trn [ 2;3] ta c:
t

0, t [ 2;3] . Vy f(t) l hm ng bin trn

[ 2;3] suy

ra

4
4
hay 1 P .
3
3
Vy gi tr nh nht ca P bng 1 t ti x = 0, y = 1 v gi tr ln nht ca P
1 = f (2) f (t ) f (3) =

bng

4
t ti x = 0, y = 2 .
3

Bi 4. Xt phng trnh ax3 x 2 + bx 1 =


0 vi a, b l cc s thc, a 0 , a b
sao cho cc nghim u l s thc dng. Tm gi tr nh nht ca biu thc:
P=

5a 2 3ab + 2
a2 (b a )

Li gii
Gi u, v, s l ba nghim thc dng ca a thc ax3 x 2 + bx 1 .
1
b
1
Theo nh l Viete ta c u + v +=
s
=
=
; uv + vs + su
; uvs
(1) .
a
a
a
T suy ra a > 0, b > 0.
1
t c = . p dng BT AM GM cho ba s dng ta c
a
c = uvs = u + v + s 3 3 uvs = 3 3 c c3 27c c 3 3

Mt khc ( u + v + s ) 3 ( uv + vs + su=
)
2

Do c 2 =

( u + v + s )2 3 ( uv + vs + su ) =

T (1), (2) v (3) ta c

368

( 2) .

1
( u v )2 + ( v s )2 + ( s u )2 0 .

2
3bc

( 3)

www.TaiLieuLuyenThi.com
Cty TNHH MTV DVVH Khang Vit

b
1
5 3 + 2 3 c 5 3bc + 2c 2
5a 3ab + 2 1
a
a
P =
.=
=
2
b
a
bc 1
a (b a )
1
a
2

c 5 c 2 + 2c 2

5c ( c + 5 )
)=
( 4)
2

c2 3

c2
1
3

Xt hm s f ( c ) =

5c c 2 + 5
c2 3

) vi

c 3 3. Ta c f ( c ) 12 3 .

Du bng xy ra khi c = 3 3. Suy ra P 12 3 .


ng thc xy ra khi u= v= s=

3 , tc=
l a

1
=
,b
3 3

3.

Vy min P = 12 3 .
Bi 5. Cho a,b,c l cc s thc khng m tha mn a > b > c v
3ab + 5bc + 7ca 9 .
32
1
1
Tm gi tr nh nht ca biu thc P =
.
+
+
4
4
( a b ) ( b c ) ( c a )4
Li gii
Theo gi thit ta c a > b > c 0 9 7ca + 5bc + 3ab 3ab ab 3 .
Khi :
32
1
1 a 2b 2 32
1
1

P
+
+

+
+
4
4
4
4
4
4
9
a
b
a
b

b
a

b
(
)
)
(

2
1
32a 2b 2
32
a 2 b2 1
a b
=
=
+
+
+ + 2
2

9 a 2 2ab + b 2 2 b 2 a 2 9 a b
b a

+ 2

b a

a b
1 32
2

t t =+ , ( t > 2 ) . Do a > b, khi P=


f (t )
+
t

2
b a
9 ( t 2 )2

Xt hm s
=
f (t )

f '(t ) =

1 32
2

vi t > 2 ta c:
+
t

2
9 ( t 2 )2

1
64
3

; f '(t ) = 0 2t ( t 2 ) = 64 t = 4 .
+
2
t
3
9 (t 2)

369

www.TaiLieuLuyenThi.com
Khm ph t duy K thut gii bt T Bi ton Max Min ng Thnh Nam

Ta c f(t) i du t m sang dng khi i qua t = 4 nn ti t = 4 th f(t) t


10
.
cc tiu hay P f (t ) f (4) =
9

=
a
6+3 3
c = 0

ng thc xy ra khi v ch khi: ab = 3


b = 2 + 3 2 3 3 .
a b

+ =
c = 0
4

b a
Vy gi tr nh nht ca P bng 10/9 t ti

a = 6 + 3 3 , b = 2 + 3 2 3 3 , c =0 .

C. BI TP RN LUYN
Bi 1. (TSC 2009) Cho a v b l hai s thc tha mn 0 < a < b < 1 . Chng minh
rng a 2 ln b b 2 ln a > ln a ln b .
Bi 2. Cho x,y l cc s thc khng m tha mn iu kin x 2 xy + y 2 =
1.
2 xy
.
( x + 1)( y + 1)

Tm gi tr nh nht ca biu thc P = x + y

Bi 3. Cho x,y l hai s thc dng tha mn iu kin xy + x + y =


3.
Tm gi tr nh nht ca biu thc P=

4x
4y
+
+ 2 xy 7 3 xy .
y +1 x +1

Bi 4. Cho x, y > 0 thoa man iu kin x + 2 y xy =


0.
Tim gia tri nho nht cua biu thc
=
P

x2
y2
.
+
4 + 8y 1+ x

Bi 5. Cho x,y l hai s thc dng c tng bng 1. Chng minh rng
x
y
+
2.
1 x
1 y
Bi 6. Cho x,y l hai s thc dng thay i tha mn 3 x 2 + 8 y 3 =
20 .
Tm gi tr nh nht ca biu thc P =

4
x

4
y

( x y )2

Bi 7. Cho x,y l hai s thc dng phn bit tha mn xy 4 .


Tm gi tr nh nht ca biu thc P =

2
x

2
y

( x y )4

Bi 8. Cho x,y l hai s thc dng tha mn xy 3 .


370

www.TaiLieuLuyenThi.com
Cty TNHH MTV DVVH Khang Vit

Tm gi tr nh nht ca biu thc=


P

( x y ) x y4
Bi 9. Cho x,y l hai s thc dng tha mn 8 ( x + 2 y ) xy =
5 x + 2 y ( 5 + 16 x ) .
4

Tm gi tr ln nht v nh nht ca biu thc


P =x 2 + 4 y 2 +

(1 + 2 xy )2 + 7 xy 6 .
xy

Bi 10. Cho x,y l 2 s thc dng tha mn iu kin

6 x 2 + y 2 + 20 xy = 5 ( x + y )( xy + 3) .

Tm gi tr nh nht ca biu thc


x4 y 4
x3 y 3
x2 y 2
P = 9 4 + 4 16 3 + 3 + 25 2 + 2 .
y
y
y
x
x
x

Bi 11. Cho a,b l hai s thc dng tha mn iu kin

(a

+ 2b 2

)(

+ 3a 2b 2 =2 a 2 + b 2 a 2 + 2b 2 .

Tm gi tr nh nht ca biu thc


=
P

a 3 + b3
b3

( a + b )2 + 2a 2 + 5b 2 ( a b )2 + 2a 2 + 5b 2

.
+ 3 +
2
2
a
ab a + 2b
8b3

Bi 12. Cho x,y l hai s thc tha mn x + y =


1.

)(

Tm gi tr ln nht ca biu thc P =x3 + 1 y 3 + 1 .


Bi 13. Cho x,y l hai s thc tha mn x 2 + y 2 =x + y .
Tm gi tr ln nht v nh nht ca biu thc P = x3 + y 3 + x 2 y + xy 2 .
Bi 14. Cho x,y l hai s thc tha mn x3 + y 3 =
2.
Tm gi tr nh nht ca biu thc =
P x2 + y 2 .
Bi 15. Cho x,y l hai s thc dng tha mn x 1, y 1 v 3 ( x + y ) =
4 xy .
1
1
Tm gi tr nh nht v ln nht ca biu thc P = x3 + y 3 + 3 2 + 2 .
y
x

Bi 16. Cho x,y l hai s thc khc 0 v tha mn xy ( x + y ) = x 2 + y 2 x y + 2 .


Tm gi tr ln nht ca biu thc P=

1 1
+ .
x y
371

www.TaiLieuLuyenThi.com
Khm ph t duy K thut gii bt T Bi ton Max Min ng Thnh Nam

Bi 17. Cho x,y l hai s thc tha mn x 2 + y 2 + xy =


3.
Tm gi tr ln nht v nh nht ca biu thc P = x3 + y 3 3 x 3 y .
Bi 18. Cho x,y l hai s thc tha mn x 2 + y 2 =
2.

Tm gi tr ln nht v gi tr nh nht ca biu thc P = 2 x3 + y 3 3 xy .


Bi 19. Cho x,y l 2 s thc khng m tha mn x + y =
1.
Tm gi tr ln nht ca biu thc P =

x 2 y + xy 2
x 2 + y 2 xy

Bi 20. Cho x,y l hai s thc tha mn 2 x 2 + y 2 xy =


1.

Tm gi tr ln nht v nh nht ca biu thc P = 7 x 4 + y 4 + 4 x 2 y 2 .


Bi 21. Cho a,b l hai s thc dng tha mn a + b + 1 =
3ab .
3a
3b
1
1
Tm gi tr ln nht ca biu thc=
P
+
2 2.
( a + 1) b ( b + 1) a a b
1
1
Bi 22. Cho cc s thc x, y 0; v x + y .
2
2

x+ y
Tm gi tr ln nht ca biu thc P =
.
1 x + 1 y

Bi 23. Cho a,b l hai s thc dng tha mn a + b =


6ab .
3a + 1
3b + 1
Tm gi tr nh nht ca biu thc =
P
+ 2
+ ( 3a + b )( 3b + a ) .
2
9b + 1 9a + 1
Bi 24. Cho x,y l hai s thc tha mn iu kin x 2 y 2 + y 2 x 2 =
2.
Tm gi tr ln nht ca biu thc P = ( x + y ) 12 ( x 1)( y 1) + xy .
3

Bi 25. Cho a,b l cc s thc khng m tha mn iu kin


a 2 + b 2 + ab 1 a b .
1
1
Tm gi tr ln nht ca biu thc
=
P
+
+ 1 + ab .
1 + a2
1 + b2

Bi 26. Cho x,y l hai s thc dng tha mn x 4 + y 4 +


Tm gi tr ln nht ca biu thc P =

372

2
1+ x

2
1+ y

1
= xy + 2 .
xy

3
.
1 + 2 xy

www.TaiLieuLuyenThi.com
Cty TNHH MTV DVVH Khang Vit

Bi 27. Cho x,y l hai s thc thuc khong ( 0;1) v

(x

+ y3

) ( x + y )=

xy ( x 1)( y 1) .
1

Tm gi tr ln nht ca biu thc=


P

1+ x

1+ y

+ 5 xy ( x + y ) .
2

Bi 28. (TSH Khi D 2009) Cho x,y l hai s thc khng m tha mn iu kin
x+ y=
1.
Tm gi tr ln nht v nh nht ca biu thc
P=

( 4x

)(

+ 3 y 4 y 2 + 3 x + 25 xy .

Bi 29. (TSH Khi D 2012) Cho x,y l hai s thc tha mn iu kin

( x 4 )2 + ( y 4 )2 + 2 xy 32 .
Tm gi tr nh nht ca biu thc P = x3 + y 3 + 3 ( xy 1)( x + y 2 ) .
Bi 30. (TSH Khi B 2009) Cho x,y l hai s thc tha mn iu kin

( x + y )2 + 4 xy 2 .

) (

Tm gi tr nh nht ca biu thc P= 3 x 4 + y 4 + x 2 y 2 2 x 2 + y 2 + 1 .


Bi 31. Cho x,y l 2 s thc khng m tha mn xy ( x + y ) =
2.

) (

Chng minh rng 3 x3 + y 3 2 x 2 + y 2 + x + y + 4 0 .


Bi 32. Cho x,y l hai s thc dng tha mn

( xy + 1) ( 9

) (

xy 2 xy = 7 x 2 + y 2 2 xy + 2 .

Tm gi tr ln nht v nh nht ca biu thc P = xy + xy +

1
1
.
+
xy
xy

Bi 33. Cho x,y l hai s thc tha mn 2 x 2 + y 2 =xy + 1 .


Tm gi tr nh nht v ln nht ca biu thc P =

x4 + y 4
.
2 xy + 1

Bi 34. Cho x,y l hai s thc tha mn x 2 xy + y 2 =


1.
Tm gi tr ln nht v nh nht ca biu thc P =

x4 + y 4 + 1
x2 + y 2 + 1

Bi 35. Cho x,y l 2 s thc tha mn x 4 + y 4 =


2 xy .
Tm gi tr ln nht v nh nht ca biu thc
373

www.TaiLieuLuyenThi.com
Khm ph t duy K thut gii bt T Bi ton Max Min ng Thnh Nam

P = xy + 3 x 2 y 2 + 2 xy x 2 + y 2 ( x + y ) .

Bi 36. Cho x 2 + y 2 > 0 v x 4 + y 4 +=


2 2 x 2 + y 2 + xy .
Tm gi tr ln nht v nh nht ca biu thc =
P 2 x 2 y 2 + xy

4
x + y2
2

Bi 37. Cho x,y l cc s thc dng. Tm gi tr nh nht ca biu thc


=
P

x + y + x2 y 2
13xy
.
+
xy
1 + xy (1 + x + y )

Bi 38. Cho x,y l hai s thc dng tha mn x 4 x 2 y 2 + y 4 =


1.
Tm gi tr nh nht ca biu thc P =

2 xy
x +1
y +1
.
+

3 x + 1 3 y + 1 xy ( x + y ) + 2

Bi 39. Cho x,y l hai s thc dng tha mn xy + x + y =


3.
Tm gi tr ln nht ca biu thc P=
Bi 40. Cho x,y l hai s thc tha mn

3x
3y
xy
+
+
x2 y 2 .
y +1 x +1 x + y

x +1 + y +1 =
4.

Tm gi tr ln nht v nh nht ca biu thc P


= xy +
Bi 41. Cho cc s thc x,y tha mn x + y=

64
.
4 x y

x 1 + 2 y + 2 .

Tm gi tr ln nht v nh nht ca biu thc


P = x 2 + y 2 + 2 ( x + 1)( y + 1) + 8 4 x y .
x 1 y 1
Bi 42. Cho x,y l hai s thc dng tha mn x + y=
+ 2 3
+
.
x
y
2
y2 3
2 x
Tm gi tr nh nht ca biu thc P =( x y ) 4 + 4 .
y
xy
x

Bi 43. Cho x > 1, y > 1 . Tm gi tr nh nht ca biu thc

=
P

x3 + y 3 x 2 y 2
+ 2 x 2 + y 2 16 xy .
( x 1)( y 1)

Bi 44. Cho x,y l hai s thc dng. Tm gi tr ln nht ca biu thc


P=

374

x4 + y 4

( x + y)

x2 + y 2

( x + y)

5 xy
.
x+ y

www.TaiLieuLuyenThi.com
Cty TNHH MTV DVVH Khang Vit

Bi 45. Cho x,y l hai s thc khng m. Tm gi tr nh nht ca biu thc


P=

4 e

3x

+e

3y

2 4 ( 2 x + 1) ( 2 y + 1)
3

E. HNG DN GII P S
Bi 1. Bt ng thc tng ng vi:

( a + 1) ln b > (b + 1) ln a bln+b1 > aln+a1 .


2

Vi 0 < a < b < 1 vy ta cn chng minh hm s f ( x) =


khong ( 0;1) .

ln x
x2 + 1

ng bin trn

1 2
x + 1 2 x ln x
x 2 + x 2 x ln x
Ta c f '( x) x
=
=
> 0, x ( 0;1) .
2
2
2
2
x +1
x x +1

Vy f(x) ng bin trn khong ( 0;1) .


Do 0 < a < b < 1 ta c f (b) > f (a )
Bi ton c chng minh.
Bi 2. Theo gi thit ta c

( x + y)

ln b
b +1
2

>

ln a
a2 + 1

2
x + y ) 1 x + y 2
(
3 xy = 1 xy =

1 x + y 2 .

=x+ y

2 3
.
3

Khi
P=x+ y

2.

( x + y )2 1
3

2
x + y) 1
(
x + y +1+

( x + y )2 1
.
2
( x + y ) + 3( x + y ) + 2

2 3
t2 1
.
.
3
t 2 + 3t + 2
Ta chng minh P 1 , tht vy ta cn chng minh.

t t = x + y,1 t 2 ta c P = t

2 3
t2 1
.
1
3
t 2 + 3t + 2

( t 1) 3 t 2 + 3t + 2 2 3 ( t + 1) 0

( t 1) ( 3t 2 + 3t + 6 )

3 t 2 + 3t + 2 + 2 3 ( t + 1)

0 ( t 1) ( 2 t ) 0
2

375

www.TaiLieuLuyenThi.com
Khm ph t duy K thut gii bt T Bi ton Max Min ng Thnh Nam

Bt ng thc lun ng. ng thc xy ra khi v ch khi t = 1 hoc t = 2 tng


ng vi ( x; y ) = ( 0;1) ; (1;0 ) ; (1;1) .
Vy gi tr nh nht ca P bng 1.
Bi 3. Ta c:
P

4 x ( x + 1) + 4 y ( y + 1)

+ 2 xy 7 3 xy

( x + 1)( y + 1)
4 ( x2 + y 2 ) + 4 ( x + y )
xy + x + y + 1

+ 2 xy 7 3 xy

4 ( x + y ) + 4 ( x + y ) 8 xy
2

+ 2 xy 7 3 xy

=( x + y ) + x + y 7 3 xy 22 + 2 7 3.1 =4
2

Bi 4. Theo gia thit ta co : x + 2 y = xy =

1
1 x + 2y
.x.2 y
x + 2y 8
2
2 2

Theo bt ng thc C-S ta c :

(2y) ( x + 2y) = ( x + 2y) .


x2
y2
x2
+
=
+
4 + 8 y 1 + x 4 + 8 y 4 + 4x 4 + 8 y + 4 + 4x 8 + 4( x + 2 y )
2

Vy t t= x + 2 y va xet ham s
=
f (t )
Ta co
=
f '(t )

4t 2 + 8t

(8 + 4t )2

Suy ra min f=
(t )
t[8; + )

t2
,t 8 .
8 + 4t

> 0 vi t 8 .

(8)
f=

8
5

T o suy ra gia tri nho nht cua P bng

8
, khi=
x 4;=
y 2.
5

Bi 5. Ta c y = 1 x nn BT cn chng minh l
x
1 x
+
2, x (0; 1) .
1 x
x
Xt =
f ( x)

376

x
1 x
+
, x (0; 1). Ta c
1 x
x

www.TaiLieuLuyenThi.com
Cty TNHH MTV DVVH Khang Vit

1
2 x
1+ x
x + 1 1 1 + (1 x)
=

2 (1 x ) 1 x x x 2 (1 x ) 1 x x x
1
=
h 1 x h x

=
f ( x )

) ( )

trong

1+ t2 1 1
h ( t=
= 3+
)
t3
t t

f ( x ) = 0 h

nghch bin trn

) ( x)

1 x = h

V f ( x ) > 0 h

( 0;+ ) , nn

1
2
1
x 1 x < x x ;1
2
1 x = x x =

) ( )

1 x > h

1
2

Do vy: f ( x ) < 0 x 0;
Vy min =
f ( x)
( 0;1)

1
f=

2

2 . Suy ra f ( x ) 2, x ( 0;1) .

Ch . Ta c th nh gi n gin bng C S .
Bi 6. tng l a P v dng hm s c nhn t chung t l t. Mun vy ta to
cho P dng ng bc nn thm vo P mt lng xy.

1
y x
xy
1
1
.
+ 2 +
= 4 + +
2
2
x
y
y ( x y)
x
x y
2+
y
x

Ta c: P.xy = 4 xy

x
y

t t =+

1
y
.
, ( t > 2 ) a v P.xy= 4t +
t 2
x

Mun tm gi tr nh nht ca P vy trc tin ta tm gi tr ln nht ca xy.

1
vi t > 2 ta c:
t 2
1
5
; f '(t ) = 0 t = x = 2 y .
f '(t ) = 4
2
2
(t 2)

Kho st hm s f (t=
) 4t +

Vy khi x = 2 y 12 y 2 + 8 y 3 = 20 y = 1, x = 2 tc P t min ti

=
x 2,=
y 1 . y l c s cho ta nh gi lng ln nht ca xy.
Xut pht t iu kin ta c:

377

www.TaiLieuLuyenThi.com
Khm ph t duy K thut gii bt T Bi ton Max Min ng Thnh Nam

20= 3 x 2 + 8 y 3= 3 x 2 + 4 ( y 3 + y 3 + 1) 4 3 x 2 + 12 y 2 4 .

8 = x 2 + 4 y 2 4 xy xy 2 .

5
f =
6.
2
Vy gi tr nh nht ca P bng 6 t ti=
x 2,=
y 1.
1
1
f (t )
2
2

Do P

Bi 10. Ta c

x
P = 9 +
y

x
y
4 +
x
y

x
y
+

2
16

x
y
3 +
x
y

x
y
+ 25 +
x
y

Bi 11. T iu kin v s dng bt ng thc AM-GM ta c:

(a

+ 2b 2 ) + 3a 2b 2 = 2 ( a 2 + b 2 )( a 2 + 2b 2 ) 4ab ( a 2 + 2b 2 )
2

a 2b
a 2b
a 2b
+ + 3 4 + +
3
b a
b a
b a
4
a 2b
Khi t t=
+ v ta c P = f (t ) = t 3 + 3t + 1 .
t
b a
4
Xt hm s f (t ) = t 3 + 3t + 1 trn [3; + ) ta c
t
4
f '(=
t ) 3t 2 + 2 + 3 > 0, t [3; + ) nn f(t) ng bin trn [3; + ) .
t
97
.
Do P = f (t ) f(3) =
3
97
t ti a= b= 1 .
Vy gi tr nh nht ca P bng
3
Bi 25. Xut pht t iu kin ta c:

(a + b)

+ a + b + ab 1 1 ab + 2 ab + 4ab 0 ab

p dng bt ng thc c bn ta c:
Suy ra P

1+ a

2
+ 1 + ab .
1 + ab

t t = 1 + ab , t 1;

378

2 2
10
ta c P + t .
t
3

1
1+ b

1
< 1.
9

2
.
1 + ab

y
50
x

www.TaiLieuLuyenThi.com
Cty TNHH MTV DVVH Khang Vit

Bi 45. HD: S dng bt ng thc quen thuc: a + b


3

(a + b)

S dng bt ng thc AM-GM ta c:


6

+
+
+
x
y
2
1
2
1
3
3

4
( 2 x + 1) ( 2 y + 1) 4
=
2

x+ y

e x + e y 2 e x .e y =
2e 2

2
Suy ra P e + e
3
x

( x + y + 1)

2e

x+ y
2

( x + y + 1)

2
3

( x + y + 1)

CH 3: K THUT S DNG TNH N IU CHO BI TON

CC TR V BT NG THC BA BIN S
A. NI DUNG PHNG PHP
1) S dng php th a v bi ton mt bin s

p
x + y + z =

q th lun biu din c S n = x n + y n + z n theo


Vi iu kin xy + yz + zx =
xyz = r

p,q,r.
Do vy vi bi ton gi thit cho 2 trong 3 iu kin (p,q,r) ta hon ton c th
a S n v mt a thc ca mt bin.

0
x + y + z =
.
1
xy + yz + zx =

V d 1. Cho x,y,z l cc s thc tha mn


Tnh P = x 3 + y 3 + z 3 theo x.

Li gii

y + z = x
.
2
yz =1 x ( y + z ) =1 x ( x ) =x 1

Theo gi thit ta c:
Ta c

P =x3 + y 3 + z 3 =( y + z ) 3 yz ( y + z ) + x3 =
( x)3 + 3x ( x 2 1) + x3 =3x3 3x .
3

Nhn xt. Ta chn gi tr ca bin x nh sau

( y + z)

4 yz x 2 4 ( x 2 1) 3 x 2 4

2
2
x
3
3
379

www.TaiLieuLuyenThi.com
Khm ph t duy K thut gii bt T Bi ton Max Min ng Thnh Nam

Vy bi ton tm gi tr ln nht v nh nht ca P a v kho st hm mt


2 2
bin f (=
x) 3 x3 3 x trn on
; .
3 3
a b c
+ + =
5.
b c a
b c a
Tm gi tr ln nht v gi tr nh nht ca biu thc P = + + .
a b c
Li gii

V d 2. Cho a,b,c l cc s thc dng tho mn iu kin

t x=

xyz = 1
a
b
c
.
, y= , z=

5
b
c
a
x + y + z =
1 1 1
+ + .
x y z

Ta cn tm gi tr ln nht nh nht ca biu thc P =


1

4
yz =
2
2
Ta c
( y + z ) 4 yz ( 5 x ) .
x
x
y + z = 5 x
x 3 + 2 2
.
( x 4) x2 6 x + 1 0
3 2 2 x 4

x2 (5 x ) + 1
1 y+z 1
Khi P = +
.
= + x (5 x ) =
x
yz
x
x

Xt hm s f ( x) =

x2 (5 x ) + 1
x

lin tc trn

D= 3 2 2;4 3 + 2 2; + d c

17
1
f =
f (4) =;max f ( x) =
f 3 2 2 =
min f ( x) =
1+ 4 2 .
xD
4 xD
2

V d 3. (TSH Khi B 2012) Cho cc s thc x,y,z tha mn x + y + z =


0 v
x2 + y 2 + z 2 =
1 . Tm gi tr ln nht ca biu thc P = x5 + y 5 + z 5 .

Li gii
Ta c:
0
x + y + z =
0
0
x + y + z =
x + y + z =

2
1.
2
2
2

1 ( x + y + z ) 2 ( xy + yz + zx ) =
1 xy + yz + zx =
x + y + z =
2

1
1
Suy ra y + z = x v x ( y + z ) + yz =
yz =
+ x2 .
2
2
380

www.TaiLieuLuyenThi.com
Cty TNHH MTV DVVH Khang Vit

6
6
2
1

Mt khc ( y + z ) 4 yz x 2 4 + x 2
.
x
3
3
2

Khi :

P = x5 + ( y + z ) 5 yz y 3 + z 3 10 y 2 z 2 ( y + z )
5

5
3
= x5 + ( y + z ) 5 yz ( y + z ) 3 yz ( y + z ) 10 y 2 z 2 ( y + z )

10 x3 5 x
1

= x5 + ( x)5 5 + x 2 x3 3 + x 2 ( x) 10 + x 2 ( x) =
4
2

Xt hm s f ( x) =

10 x3 5 x
lin tc trn
4

6 6
;

ta c:
3 3

30 x 2 5
6
.
f '( x) =
; f '( x) =
0 x=

4
6

6
6
5 6
Ta c f
f

,f
=
=
3
6
36

Suy ra=
P f ( x)

6 5 6
f

=
= .
3
6

36

5 6
.
36

6
6
5 6
t ti x =
.
,y = z =
3
6
36
Bnh lun. Biu thc ca P l mt i vi x nn theo kt qu trn ta tm c c
gi tr nh nht ca P. Ngoi ra c th bin i P nh sau:

Vy gi tr ln nht ca P bng

P = x5 + y 5 + z 5 = x5 + y 3 + z 3

)( y

+ z2 y2 z2 ( y + z ) .

V d 4. Cho a,b,c l cc s thc thuc on [1;4] v a + b + 2c =


8.
Tm gi tr ln nht ca biu thc P = a3 + b3 + 5c3 .
Li gii
Theo gi thit ta c 2 + c a + b + 2c =8 8 + 2c c [1;3]
Ta c P = ( a + b ) 3ab ( a + b ) + 5c3 = ( 8 2c ) 3ab ( 8 2c ) + 5c3
3

=
3c3 + 96c 2 384c + 512 3ab ( 8 2c )

Vi a, b 1 ( a 1)( b 1) 0 ab a + b 1 = 7 2c > 0
Khi
P 3c3 + 96c 2 384c + 512 3 ( 7 2c )( 8 2c ) =
3c3 + 84c 2 294c + 344

Xt hm s f (c) =
3c3 + 84c 2 294c + 344 lin tc trn on [1;3] , ta c
381

www.TaiLieuLuyenThi.com
Khm ph t duy K thut gii bt T Bi ton Max Min ng Thnh Nam

28 7 10
f '(c) =9c 2 + 168c + 512; f '(c) =0 c =c0 =
[1;3] .
3
Ta c f(c) i du t m sang dng khi i qua c0 nn f(c) t cc tiu ti c0 .

Do P f (c) max { f (1); f (3)} = f (3) = 137 .


ng thc xy ra khi v ch khi a= b= 1, c= 3 .
Vy gi tr ln nht ca P bng 137 t ti a= b= 1, c= 3 .
2) nh gi thng qua cc i lng trung bnh ca ba bin s
Vi mi s thc x,y,z ta lun c

3 x2 + y 2 + z 2 ( x + y + z ) ;
2

( x + y + z )2 3 ( xy + yz + zx ) ;
( xy + yz + zx )2 3xyz ( x + y + z ) .
Vi x,y,z l cc s thc khng m ta lun c

x+ y+z 3
xyz ;
3

xyz ( x + y z )( y + z x )( z + x y )
8
9

( x + y )( y + z )( z + x ) ( x + y + z )( xy + yz + zx )
Nhn xt. Vi cc bi ton c x, y, z [ a; b ] v x + y + z =
s ta thng s dng bt
ng thc tm mi rang buc gia cc i lng i xng xy + yz + zx v xyz .
Mt s ng thc nh ch

( a + b + c )3 = a3 + b3 + c3 + 3 ( a + b )( b + c )( c + a )
( a + b )( b + c )( c + a ) = ( a + b + c )( ab + bc + ca ) abc
2 ( a 2b 2 + b 2 c 2 + a 2 a 2 ) ( a 4 + b 4 + c 4 ) = ( a + b + c )( a + b c )( b + c a )( c + a b )
V d 1. (TSH Khi B 2011) Cho a,b,c l cc s thc khng m tha mn iu
kin a + b + c =
1.
Tm gi tr nh nht ca biu thc

=
P 3 a 2b 2 + b 2 c 2 + c 2 a 2 + 3 ( ab + bc + ca ) + 2 a 2 + b 2 + c 2

Li gii
Theo gi thit ta c:
a 2 + b 2 + c 2 =( a + b + c ) 2 ( ab + bc + ca ) =1 2 ( ab + bc + ca ) .
2

Mt khc: 3 a 2b 2 + b 2 c 2 + c 2 a 2 ( ab + bc + ca ) .

382

www.TaiLieuLuyenThi.com
Cty TNHH MTV DVVH Khang Vit

Suy ra P ( ab + bc + ca ) + 3 ( ab + bc + ca ) + 2 1 2 ( ab + bc + ca ) .
2

t t = ab + bc + ca khi : P f (t ) = t 2 + 3t + 2 1 2t .
Vi 0 ab + bc + ca

1
1
1
( a + b + c )2 = t 0; .
3
3
3

1
Xt hm s f (t ) = t 2 + 3t + 2 1 2t lin tc trn on 0; ta c:
3

f '(t ) = 2t + 3

2
; f ''(t ) = 2
1 2t

(1 2t )

1
0, t 0; .
3

1 11
Do f '(t ) f ' = 2 3 > 0 .
3 3
1
V vy f(t) ng bin trn on 0; . Suy ra P f (t ) f (0) =
2.
3
ng thc xy ra khi v ch khi a= b= 0, c= 1 hoc cc hon v.
Vy gi tr nh nht ca P bng 2 t ti a= b= 0, c= 1 hoc cc hon v.
V d 2. Cho x,y,z l cc s thc thuc on [0;2] v tho mn iu kin
x+ y+z=
3.

Tm gi tr ln nht ca biu thc P=

1
1
1
+
+
+ xy + yz + zx .
x+ y y+z z+x

Li gii
Ch bt ng thc
8
9

( x + y )( y + z )( z + x ) ( x + y + z )( xy + yz + zx ) .
Khi
P

( x + y )( x + z ) + ( y + z )( y + x ) + ( z + x )( z + y ) + xy + yz + zx
( x + y )( y + z )( z + x )
( x + y + z )2 + xy + yz + zx + xy + yz + zx
( x + y )( y + z )( z + x )
2
x + y + z ) + xy + yz + zx
(

+ xy + yz + zx

8
( x + y + z )( xy + yz + zx )
9
27
3
=
+ xy + yz + zx +
8 ( xy + yz + zx )
8

Ch iu kin x, y, z [ 0;2] nn
383

www.TaiLieuLuyenThi.com
Khm ph t duy K thut gii bt T Bi ton Max Min ng Thnh Nam

4 + xyz
2.
2
1
2
Khi t t = xy + yz + zx v xy + yz + zx ( x + y + z ) =
3 nn t [ 2;3] .
3
27
3
Do P f (t )=
+t + .
8t
8
27
27
3
Xt hm s f (t )=
+ t + trn on [2;3] ta c f '(t ) = 1 2 > 0, t 2 .
8t
8
8t

( 2 x )( 2 y )( 2 z ) 0 xy + yz + zx

9
.
Do f(t) ng bin trn on [2;3] suy ra f (t ) f (3) =
2
Du bng xy ra khi v ch khi x= y= z= 1 .

Vy gi tr ln nht ca P bng 9/2.


Nhn xt. Ta c th nh gi thng qua bt ng thc(xem chng 2).
1
1
1
3
x+ y+z
.
+
+

+
x + y y + z z + x 2 ( x + y + z ) xy + yz + zx
V d 3. Cho x, y, z l cc s thc thuc khong (0, 1) tho mn iu kin
xyz =
(1 x )(1 y )(1 z ) .

Tm gi tr nh nht ca biu thc P = x + y + z +

1 1 1
+ + .
x y z

Li gii
Theo gi thit ta c: x + y + z xy yz zx + 2 xyz 1 =
0.
D on du bng xy ra khi x = y = z x3 = (1 x ) x = y = z =
3

Do ta xt hai kh nng sau:


3
3
1
+ Nu xy + yz + zx < > xy + yz + zx 3 3 x 2 y 2 z 2 xyz < .
4
4
8
Khi s dng bt ng thc AM GM ta c
1
1
1 3 1 1 1

P = x+ + y+
+z+ + + +
4x
4y
4z 4 x y z

2 x.
= 3+

384

1
1
1
9
+ 2 y.
+ 2 z. +
4x
4y
4 z 4 3 xyz
9

43

xyz

> 3+

9
15
=
1 2
4.
2

1
.
2

www.TaiLieuLuyenThi.com
Cty TNHH MTV DVVH Khang Vit

3
khi bin i P theo xy + yz + zx bng cch rt
4
xy + yz + zx + 1 x y z
.
xyz =
2

+ Nu xy + yz + zx

V ch x + y + z 3 ( xy + yz + zx ) .
xy + yz + zx
xyz
xy + yz + zx
= x + y + z + 2.
xy + yz + zx + 1 x y z
xy + yz + zx
3 ( xy + yz + zx ) + 2.
xy + yz + zx + 1 3 ( xy + yz + zx )

Ta c P = x + y + z +

t
=
t

3
3 ( xy + yz + zx ) , t < 3 ta c
2

P t + 2.

t2
3
t2
+1 t
3

=t+

2t 2
t 2 3t + 3

( 2t 3) ( t 2 7t + 15)

2 t 3t + 3
2

15 15
3
, t ;3
2
2
2

1
.
2
Nhn xt. L do xt hai trng hp do suy ngh lm theo hng hai u tin tuy
3
nhin bt ng thc cui ch ng vi xy + yz + zx . Do vy cn phn chia lm
4
3
hai trng hp, vi xy + yz + zx < ta kho lo kt hp AM GM ch ra P ln
4
hn 15/2.
Bi tp tng t
Cho x, y, z l cc s thc thuc khong (0, 1) tho mn iu kin

So snh hai trng hp ta c Min P bng 15/2 t ti x= y= z=

xyz =
(1 x )(1 y )(1 z ) .

Tm gi tr nh nht ca biu thc P = x 2 + y 2 + z 2 .


S: Pmin =

3
.
4

385

www.TaiLieuLuyenThi.com
Khm ph t duy K thut gii bt T Bi ton Max Min ng Thnh Nam

V d 4. Cho a,b,c l cc s thc dng. Tm gi tr nh nht ca biu thc


P=

a b c
8abc
.
+ + 2 +
b c a
( a + b )( b + c )( c + a )

Li gii
S dng bt ng thc AM-GM ta c:
8abc
8
8.27
.
=

3
a
b
c
a
b
b
c
c
a
+
+
+

(
)(
)(
)
a
b
c

+ 1 + 1 + 1 + + + 3
b c a b c a

t t =

a b c
+ + + 3, ( t 6 ) ta c P f (t ) =
b c a

Xt hm s f (t ) =

t 5 +

216
t3

t 5 +

216
t3

vi t 6 ta c:

1
648 t 4 1296 t 5
=
0, t 6 .
2 t 5 t4
2t 4 t 5
Suy ra P f (t ) f (6) =
2 . ng thc xy ra khi v ch khi a= b= c .

f '(t)
=

Vy gi tr nh nht ca P bng 2 t ti a= b= c .
V d 5. Cho x,y,z l cc s thc khng m tha mn iu kin x 2 + y 2 + z 2 =
3.
16

Tm gi tr nh nht
ca biu thc P
=

x y +y z +z x
2 2

2 2

2 2

xy + yz + zx
.
x+ y+z

Li gii

(x

+ y 2 + z 2 x4 y 4 z 4 9 x4 y 4 z 4
2 2
2 2
2 2
.
Ta=
c x y + y z + z x
=
2
2
S dng bt ng thc AM-GM ta c :
2

x 4 + x + x 3x 2 ; y 4 + y + y 3 y 2 ; z 4 + z + z 3z 2

Cng theo v 3 bt ng thc trn ta c :

x4 + y 4 + z 4 3 x2 + y 2 + z 2 2 ( x + y + z ) = 9 2 ( x + y + z ) .
=
xy + yz + zx

2
2
y + z ) x2 y 2 z 2 ( x + y + z ) 3
( x +=
.

( x + y + z) 1 .
16
+
2( x + y + z )
x + y + z +1
2

Do P

t t = x + y + z , t 3;3 . Khi P f (t ) =
386

16
t2 1
.
+
2t
t +1

www.TaiLieuLuyenThi.com
Cty TNHH MTV DVVH Khang Vit

Xt hm s=
f (t )
f '(t )=

1
2t

16
t2 1
lin tc trn on 3;3 ta c
+
2t
t +1

( t + 1)

1 1
8
+
< 0 nn f(t) l hm nghch bin trn
6 2
43

on 3;3 .
28
Do P f (t ) f (3) =
. ng thc xy ra khi v ch khi x= y= z= 1 .
3
V d 6. Cho a,b,c l cc s thc dng tha mn iu kin ab + bc + ca =
1.
Tm gi tr nh nht ca biu thc P = a + b + c + abc .
Li gii
1 ab
Theo gi thit ta c:
=
c
, c > 0 ab < 1 .
a+b
Thay vo biu thc ca P ta c:
P = a+b+

1 ab
1 ab
1 a 2b 2
.
+ ab.
= a+b+
a+b
a+b
a+b
4

a+b
S dng bt ng thc AM-GM ta c: a 2b 2
.
2
4

a+b
1
4
2

( a + b ) + 16 ( a + b ) + 16
2

Suy ra P a + b +
.
=
a+b
16 ( a + b )
t 4 + 16t 2 + 16
t t =+
.
a b, ( t > 0 ) khi P f (t ) =
16t

Xt hm s f (t ) =

t 4 + 16t 2 + 16
vi t > 0 ta c:
16t

)(

t = 2
4 t 2 3t 2 4
3t 4 16t 2 + 16
.

=
f '(t ) =
;f'(t) =
0
2
2
t = 2
16t
16t

3
10 3
2 10 3
. ng thc
Lp bng bin thin suy ra Min f (=
P
t) f =

t >0
27
27
3
1
.
xy ra khi v ch khi a= b= c=
3

Vy gi tr nh nht ca P bng

10 3
t ti a= b= c=
27

1
.
3
387

www.TaiLieuLuyenThi.com
Khm ph t duy K thut gii bt T Bi ton Max Min ng Thnh Nam

V d 7. Cho a,b,c l cc s thc tha mn iu kin a 2 + b 2 + c 2 =


2.
Tm gi tr ln nht v nh nht ca biu thc P = a3 + b3 + c3 3abc .
Li gii
Ta c:
2 = a 2 + b 2 + c 2 = ( a + b + c ) 2 ( ab + bc + ca ) ab + bc + ca =
2

( a + b + c )2 2 .
2

Khi :
P=
=

( a + b + c ) ( a 2 + b2 + c 2 ab bc ca )
( a + b + c ) ( a + b + c )2 3 ( ab + bc + ca )

a + b + c) 2 .
(
2

= ( a + b + c ) ( a + b + c ) 3.
2

( a + b + c ).

(a + b + c) + 6
2

2
t t = a + b + c t 2 = ( a + b + c ) 3 a 2 + b 2 + c 2 = 6 t 6; 6 .

1
Khi P =
f (t ) =
t 3 + 3t .
2
1
Xt hm s f (t ) =
t 3 + 3t lin tc trn 6; 6 ta c
2
3
f '(t ) = t 2 + 3; f '(t ) =0 t = 2 .
2
Bng bin thin:
t
6
2
2

f(t)
f(t)

2 2

2 2

Da vo bng bin thin suy ra f(t) t gi tr ln nht bng 2 2 ti t = 2 v


t gi tr nh nht bng 2 2 t ti t = 2 .
Vy gi tr ln nht ca P bng 2 2 t ti a=

2, b= c= 0 hoc cc hon v.

gi tr nh nht ca P bng 2 2 t ti a =
2, b =
c=
0 hoc cc hon v.
388

www.TaiLieuLuyenThi.com
Cty TNHH MTV DVVH Khang Vit

3) Bt ng thc c hai bin i xng


Ghp cp hai bin i xng vi nhau v nh gi bt ng thc c bn nh AM
GM v Cauchy Schwarz hoc mt s bt ng thc ph a v bin cn li
v hon tt bng kho st hm s (xem ch sau).
V d 1. Cho x,y,z l cc s thc dng tho mn iu kin x 2 + y 2 + z 2 =
1.
Tm gi tr ln nht ca biu thc P= 6 ( y + z x ) + 27 xyz .
Li gii
S dng bt ng thc AM GM v C S ta c

y + z 2 y 2 + z 2=
yz

2 1 x2

y 2 + z 2 1 x2
=
2
2

27
Suy ra P 6 2 1 x 2 x +
x 1 x2 .

2
27
Xt hm s f =
x 1 x 2 trn khong (0;1) ta c
( x) 6 2 1 x 2 x +

f '( x) =

3
4 2x
27 x 2 + 5
2
1 x2

4 2x
=0
; f '( x) = 0 27 x 2 + 5

1 x2

0 < x < 1

4 2x

=
5 27 x 2 5 27 x 2 > 0
1 x2

2
2
32 x =
5 27 x 2 =
0
2
1 x
0 < x < 1

5 27 x 2 > 0
x=
3
2
2
2
x
x
x

=
3
1
9
1
27
25
0

)(

)(

1
1
nn f(x) t cc i ti .
3
3
1
1

V vy P f ( x) f =
.
10 . Du bng t ti x= y= z=
3
3
Vy gi tr ln nht ca P bng 10.
1
V d 2. Cho x,y,z l cc s thc tha mn iu kin x 2 + y 2 z 2 .
2
1
1
1
Tm gi tr nh nht ca biu thc P = x 4 + y 4 + z 4 4 + 4 + 4 .
y
z
x

Ta c f(x) i du t dng sang m khi i qua

389

www.TaiLieuLuyenThi.com
Khm ph t duy K thut gii bt T Bi ton Max Min ng Thnh Nam

Li gii
1
2
S dng bt ng thc c bn: a 2 + b 2 ( a + b ) ta c:
2
1 2
1
2
z x2 + y 2 ( x + y ) x + y z .
2
2
2
1
1
1
2
32
4 1
.
x4 + y 4 x2 + y 2 ( x + y ) ; 4 + 4 2 2
2
8
x
y
x y
( x + y )4

4
4
( x + y )4
32
z
1 1 x + y
4

Suy ra P
+z .
+=
+5.
+ 32
8
( x + y )4 z 4 8 z
x+ y
4

1 32
x+ y
t t
=
+5.
, ( 0 < t 1) ta c: P f (t ) = t +
8
t
z
1 32
Xt hm s f (t ) = t +
+ 5 vi 0 < t 1 ta c:
8
t
1 32
f '(t ) =

< 0, t ( 0;1] v vy f(t) nghch bin trn ( 0;1] .


8 t2
z
297
Do P f (t ) f (1) =. ng thc xy ra khi v ch khi x= y=
.
2
8
297
Vy gi tr nh nht ca P bng
t ti =
z 2=
x 2y .
8
V d 3. Cho a, b, c l cc s thc dng tha mn iu kin
1 1
1
1
a ;
+
+
=
2.
2 a +1 b +1 c +1

Tm gi tr nh nht ca biu thc


=
P

32 b3 + c3 + 10

.
8a 2 + 1 64b 2 c 2 + 16bc + 1
Li gii
Ch . Ta cn nh gi bc theo a mun vy xut pht t iu kin ta c:
2abc + ab + bc + ca =1 bc < 1 .
1
1
1
1 + 2a
V
.
+
=
2
=
b +1 c +1
1+ a 1+ a
1
1
2
1 + 2a
2
1
Vi bc < 1 ta c
.
+

bc
b + 1 c + 1 1 + bc
1 + a 1 + bc
( 2a + 1)2

32 b3 + c3 + 10

Mt khc

390

64b c + 16bc + 1
2 2

8bc + 1

6
.
8
+1
(2a + 1) 2

www.TaiLieuLuyenThi.com
Cty TNHH MTV DVVH Khang Vit

6 ( 2a 1) 4a 2 + 8a + 1
3
6
Suy ra P =
+
+ 3 3.
2
2
8
8a 2 + 1
8
a
+
1
2
a
+
1
+
8
(
)
+1
(2a + 1) 2
2

)(

1
1
1
1
1
Du bng t ti a =; b =
c;
+
+
=
2a=
b=
c =.
2
a +1 b +1 c +1
2
V d 4. Cho a,b,c l cc s thc dng tha mn iu kin abc = 1 .
1
1
2
.
Tm gi tr nh nht ca biu thc P =
+
+
2a + 1 2b + 1 (2c + 1) 6c + 3

Li gii
1
1
2
1
Ta bit bt ng thc ph
ng vi ab . Do vy ta
+

1 + 2a 1 + 2b 2 ab
4
chia trng hp x l.
1
1
1
1
1
1
+ TH1: Nu ab < a < , khi P >
1.
+
>
+
=
1
2a + 1 2b + 1 1 +
2b + 1
4
4b
2b
1
+ TH2: Nu ab c 4, khi vn dng bt ng thc ph trn ta c:
4
P

2
2
c
2
.
+
=
+
1 + 2 ab ( 2c + 1) 6c + 3
c + 2 (2c + 1) 6c + 3
c
2
trn khong ( 0;4] ta c
+
c + 2 (2c + 1) 6c + 3

Xt hm s f (c) =
min f=
(c) f=
(1)

8
.
9

Du bng t ti a= b= c= 1 . Vy gi tr nh nht ca P bng

8
.
9

4) nh gi xoay quanh cc i lng (a b),(b c),(c a ) .


V d 1. Cho cc s thc khng m a,b,c c tng bng 1.
Tm gi tr ln nht, gi tr nh nht ca biu thc P =
( a b )( b c )( c a ) .
Li gii
Ta c: P 2 =
( a b) (b c ) (c a ) .
2

Khng mt tnh tng qut ta c th gi s a b c 0 khi


P 2 ( a + c b ) b2 ( a + c ) .
2

t t= a + c ta c P 2 = ( 2t 1) t 2 (1 t ) = 2t 3 3t 2 + t
2

.
391

www.TaiLieuLuyenThi.com
Khm ph t duy K thut gii bt T Bi ton Max Min ng Thnh Nam

Ta c: t = a + c =1 b 1, t = a + c
Xt hm s f (t ) =

( 2t

3t 2 + t

a+b+c 1
1
= . Vy t ;1 .
2
2
2

1
lin tc trn ;1 ta c:
2

1
t =
1
t ;1
2
2
.
f '(t )= 2 2t 3 3t 2 + t 6t 2 6t + 1 ; f '(t )= 0
t = 1

t = 3 + 3

)(

3+ 3 1
1
1
1
Ta c f =
. Suy ra 0 f (t )
hay P 2
.
(1) 0, f
=

f=
6
108
108
108
2

Do
=
a

3
3
3+ 3
3 3
3
. Ti =
th P =
P
a
=
c
, b 0,=
. Ti
6
6
18
18
18

3
3+ 3
3 3
.
=
,b =
, c 0 th P =
18
6
6

Vy gi tr nh nht ca P bng

3
3+ 3
3 3
t
ti a =
=
,b =
, c 0 v gi
108
6
6

3
3+ 3
3 3
t ti=
.
a
=
c
, b 0,=
18
6
6
Nhn xt. Xut pht t ng thc

tr ln nht ca P bng

x3 + y 3 + z 3 3 xyz = ( x + y + z ) x 2 + y 2 + z 2 xy yz zx .

Ta thay x =
a b, y =
b c, z =
c a ta c x3 + y 3 + z 3 =
3 xyz .
Thay v yu cu tm gi tr ln nht, nh nht ca P c th yu cu tm gi tr
ln nht, nh nht ca biu thc
P = ( a b) + (b c ) + (c a ) .
3

V d 2. Cho a,b,c l cc s thc khng m i mt phn bit tha mn


ab + bc + ca =
4.
1
1
1
.
Tm gi tr nh nht ca biu thc P =
+
+
2
2
( a b ) ( b c ) ( c a )2
Li gii
Khng mt tnh tng qut gi s a > b > c 0 .
1
1
1
Khi P
+ 2+ 2.
2
(a b) a b
392

www.TaiLieuLuyenThi.com
Cty TNHH MTV DVVH Khang Vit

Ta c: 4 = ab + bc + ca ab ab 4 .
1
1
1
1
a b

Khi 4 P =
+
=
+
+ + .
P.ab ab
2
2
2
a
b a 2 + b b a
( a b )
b
a
1
1
a b
t t =+ , ( t > 2 ) . Khi P f (t ) = t +
.
4 t 2
b a

Xt hm s f =
(t )

1
1
t +
vi t > 2, ta c
4 t 2

1
1
t >2
1
; f '(=
=
t ) 0
t 3.
4 ( t 2 )2

Ta c f(t) i du t m sang dng khi i qua 3 nn ti t = 3 th f(t) t cc


tiu hay f (t ) f (3) =
1.
f '(=
t)

Vy gi tr nh nht ca P bng 1 t ti
a b
c = 0, ab = 4, + = 3 a = 5 1, b = 5 + 1, c = 0 v cc hon v.
b a
Cch 2: Khng mt tnh tng qut gi s a > b > c 0 . Khi vit li v tri ca
bt ng thc v s dng bt ng thc AM-GM ta c:
2

1
1
1
1
1
1
1
1
1
P=
.
.
.
+
+
+
+
2

a b bc ca
a b bc bc ca ca a b
2

1
1
4 1
1
1
=
+
+
+
=

a bbc ca
a b bc ca

Nhn xt. Ch ng thc quen thuc


1
1
1
1
1
1
.
+
.
+ =
.
a b bc bc ca ca a b

( b c )( a c )

4
1
ab

ca + a b+bc
= 0.
( a b )( b c )( c a )

Vi gi thit a,b,c biu thc sau c ngha v cch chng minh tng t ta c
cc bt ng thc tng t dng sau
1) Vi a, b, c 0 ta c
1
1
1 11 + 5 5

.
+ b2 + c2
+
+
2
( a b )2 ( b c )2 ( c a )2
2) Vi a, b, c 0 ta c

(a

1
1 59 + 11 33
( a + b ) 2 + ( b + c ) 2 + ( c + a ) 2 1

.
+
+

( a b )2 ( b c )2 ( c a )2
4

393

www.TaiLieuLuyenThi.com
Khm ph t duy K thut gii bt T Bi ton Max Min ng Thnh Nam

3) Vi a b c ta c
1
1
1 27

.
a 2 + b 2 + c 2 ab bc ca
+
+
( a b )2 ( b c )2 ( c a )2 4
4) Vi a b c ta c

1
1
1 9
.
a 2 + b2 + c2
+
+
( a b )2 ( b c )2 ( c a )2 2
Sau y ta cng xt mt s bi ton dng trn
V d 3. Cho a,b,c l cc s thc khng m i mt phn bit.
Tm gi tr nh nht ca biu thc

(a

P=

1
1
1
.
+ b2 + c2
+
+
( a b )2 ( b c )2 ( c a )2
Li gii

Khng mt tnh tng qut gi s c = min {a, b, c} .


Khi ( a c ) a 2 , ( b c ) b 2 , a 2 + b 2 + c 2 a 2 + b 2 .
2

Do
a b
2
+
1
1 1 a 2 + b2 a 2 b2
b
a +a + b .

2
P a +b
+
+
=
+
+
=
+
2
2
2
2
2
a
b b a

a2
2+
( a b) a b ( a b) b
b
a
a b
t
t t =+ , ( t > 2 ) do a b . Khi P f (t ) =
+ t2 .
b a
t2

Ta c: f '(t )=

(t 2)

t >2

+ 2t ; f '(t )= 0
t=

3+ 5
.
2

Bng bin thin:


t
2
f(t)
f(t)

3+ 5
2
0

11 + 5 5
2

394

www.TaiLieuLuyenThi.com
Cty TNHH MTV DVVH Khang Vit

3 + 5 11 + 5
Da vo bng bin thin suy ra f (t ) f
.
=
2
2
Du bng xy ra khi v ch khi
=
t

3+ 5
a b 3+ 5
3+ 5 6 5 2
+
=

=
a
b.
2
b a
2
4

Vy gi tr nh nht ca P bng

3+ 5 6 5 2
11 + 5 5
t ti=
c 0,=
a
b
4
2

hoc cc hon v.
V d 4. Cho a,b,c l cc s thc phn bit tha mn a + b + c =
1 v
ab + bc + ca > 0 .
2
2
2
5
.
Tm gi tr nh nht ca biu thc P =
+
+
+
a b bc ca
ab + bc + ca
Li gii
Khng mt tnh tng qut gi s a > b > c khi :
2
2
2
5
8
2
5
+
+
+

+
+
P=
ab bc ac
ab + bc + ca a b + b c a c
ab + bc + ca
10
5
=
+
ac
ab + bc + ca
t =
t

1
1
1

2
.
ab + bc + ca , 0 < t <
do ab + bc + ca < 3 ( a + b + c ) =
3
3

Ta c: ( a b ) + ( b c )
2

2
1
1
( a b ) + ( b c ) =
( a c )2 .
2
2

Suy ra:
3
2 6t 2
( a c )2 ( a b )2 + ( b c )2 + ( c a )2 =2 ( a + b + c )2 3 ( ab + bc + ca ) =
2
ac

2 1 3t 2
3

Xt hm =
s f (t )
f '(t ) =

15 3t

(1 3t )

2 3

P =
f (t )

5 3
1 3t 2

5
t

5
1
+ vi t 0;
ta c:
t
3

1 3t
5 3

5
t

; f '(t ) = 0 3 3t 3 =

(1 3t )

2 3

t=

1
.
6

395

www.TaiLieuLuyenThi.com
Khm ph t duy K thut gii bt T Bi ton Max Min ng Thnh Nam

Ta c f(t) i du t m sang dng khi i qua t =


t=

1
nn f(t) t cc tiu ti
6

1
1
. Do P f (t ) f
10 6 .
=
6
6

1 1
1
1 1
.
ng thc xy ra ti a =
+
,b =
,c =

3
3
3
6
6
1 1
1
1 1
hoc
Vy gi tr nh nht ca P bng 10 6 t ti a =
+
,b =
,c =

3
3
3
6
6
cc hon v.
Cch 2: nh gi thng qua bt ng thc AM-GM.
Khng mt tnh tng qut ta gi s a > b > c ta c:
2
2
2
5
.
P=
+
+
+
a b bc a c
ab + bc + ca

S dng bt ng thc:
P 2.

1 1
4
+
, x, y > 0 ta c:
x y x+ y

4
2
5
1
2

+
+
= 5
+

a b+bc a c
ab + bc + ca
ab + bc + ca
ac
5.2 2

20
=
4 a c 2 ab + bc + ca
(
) (
) 4 ( a c )2 ( 4ab + 4bc + 4ca )
20

(a c)

20 2
20 2
=
=
2
+ 4ab + 4bc + 4ca
( a + c ) + 4b ( a + c ) ( a + c )( a + c + 4b )
2

20 2

(1 b )(1 + 3b )

20 6

( 3 3b )(1 + 3b )

40 6
= 10 6
3 3b + 1 + 3b

2+ 6
a =
a b = b c
6

1
1

2
ng thc xy ra khi v ch khi
.
=
=
b

3
ab + bc + ca
a c

3 3b =+

1 3b
2 6
c =
6

1 1
1
1 1
hoc
Vy gi tr nh nht ca P bng 10 6 t ti a =
+
,b =
,c =

3
3
3
6
6
cc hon v.
396

www.TaiLieuLuyenThi.com
Cty TNHH MTV DVVH Khang Vit

V d 5. Cho a,b,c l cc s thc khng m tha mn iu kin

( ab + bc + ca + 1)( ab + bc + ca ) .

2 a 2 + b2 + c2 =

Tm gi tr ln nht ca biu thc =


P 2 a 2 + b2 + c2 a b b c c a .
Li gii
Khng mt tnh tng qut gi s a b c =
P 2 a 2 + b2 + c2 2 ( a c ) .
t ( t + 1)
t t = ab + bc + ca, ( t 0 ) theo gi thit ta c: a 2 + b 2 + c 2 =
.
2
Nu t = 0 a = b = c = 0 P = 0 .
Xt t > 0 khi s dng bt ng thc c bn ta c:
a 2 + b 2 + c 2 ab + bc + ca

Ta c

t ( t + 1)
2

t t 1.

( a b )2 + ( b c )2 + ( c a )2 =

2 a 2 + b 2 + c 2 2 ( ab + bc + ca ) = t ( t + 1) 2t = t ( t 1) .

( a b ) + ( b c )
( a c )2 =

=
( a b ) + ( b c ) + 2 ( a b )( b c ) ( a b ) + ( b c ) .

Suy ra 2 ( a c ) ( a b ) + ( b c ) + ( c a ) = t ( t 1) a c
2

Do P f (=
t)

2t ( t + 1) 2t ( t 1=
)

2t + 1
2t ( t + 1)

t ( t 1)
2

2 2t
.
t +1 + t 1

2t ( t + 1) 2t ( t 1) vi t 1 ta c:

Xt hm s f (=
t)
f '(t ) =

t 1

2
( 2t + 1) ( t 1) =

2t 1
2t ( t 1)

; f '(t ) =0 ( 2t + 1) t ( t 1) =( 2t 1) t ( t + 1) .

(v nghim).

( 2t 1)2 ( t + 1)
Vy f(t) khng i du trn (1;+ ) ta c
Vy f(t) l hm nghch bin trn [1;+ ) .

f '(2) < 0 nn f '(t ) < 0, t > 1 .

Suy ra P f (t ) f (1) =
2 . ng thc xy ra khi v ch khi a= b= c=
Vy gi tr ln nht ca P bng 2 t ti a= b= c=

1
.
3

1
.
3

397

www.TaiLieuLuyenThi.com
Khm ph t duy K thut gii bt T Bi ton Max Min ng Thnh Nam

V d 6. Cho a,b,c l cc s thc tha mn iu kin a 2 + b 2 + c 2 =


5.
Tm gi tr ln nht v nh nht ca biu thc
P = ( a b )( b c )( c a )( ab + bc + ca ) .

Li gii
Ta c: P 2 = ( a b ) ( b c ) ( c a ) ( ab + bc + ca ) .
2

tm gi tr ln nht v nh nht ca P ta tm gi tr ln nht ca P 2 .


Khi ta c th gi s a b c khi :
S dng bt ng thc AM-GM ta c:

( a b) (b c )
2

Suy ra P 2

( a b) + (b c )
(a c) .
=( a b )( b c )
=
2
16

1
( c a )6 ( ab + bc + ca )2 .
16

t t = ab + bc + ca, ( t 5 ) do ab + bc + ca a 2 + b 2 + c 2 =
5.
Ta c: ( a b ) + ( b c )
2

2
1
1
( a b ) + ( b c ) =
( a c )2 .
2
2

Suy ra:
3
( a c )2 ( a b )2 + ( b c )2 + ( c a )2 =2 a 2 + b2 + c 2 ab bc ca =10 2t
2

(a c)

20 4t
1 20 4t 2 4 2
3

P
t
t (5 t )
=
3
16 3
27

4 2
3
t ( 5 t ) vi t 5 ta c:
27
t = 0
20
2
t =
f '(t ) =
0
t ( t 2 )( t 5 ) ; f '(t) =
2.
27
t = 5

Xt hm s
=
f (t )

Bng bin thin:


t

f(t)
+
f(t)

0
0

2
0
16

0
Da vo bng bin thin suy ra P 2 f (t ) f (2)= 16 4 P 4 .
398

5
0

www.TaiLieuLuyenThi.com
Cty TNHH MTV DVVH Khang Vit

Ti =
a 2,=
b 1,=
c 0 th P = 4 ; ti a =
2, b =
1, c =
0 th P = 4 .
Vy gi tr nh nht ca P bng 4 ; gi tr ln nht ca P bng 4.
Nhn xt. Php ton bnh phng a v a thc i xng vi ba bin a,b,c gip ta
c th gi s c a b c . y l mt cch rt hay c p dng khi gi tr ln
nht v nh nht ca bi ton c gi tr i nhau.
Bi tp tng t
Vi mi a,b,c tm s thc M nh nht tho mn bt ng thc

ab a 2 b 2 + bc b 2 c 2 + ca c 2 a 2 M a 2 + b 2 + c 2

V d 7. Cho a,b,c l cc s thc tho mn iu kin a 2 + b 2 + c 2 =


1.
Tm gi tr ln nht, gi tr nh nht ca biu thc
P=

( a b )( b c )( c a )( a + b + c ) .
Li gii

Chuyn P v dng i xng


P=
=

( a b )2 ( b c )2 ( c a )2 ( a + b + c )2
.
( a b )2 ( b c )2 ( c a )2 1 + 2 ( ab + bc + ca )

1
Khng mt tnh tng qut gi s a b c v t x= ab + bc + ca, x ;1 .
2
Ta co :
2

a b+bc a c
1
3
=

( a b )( b c )( a c ) ( a c )
2
4

( a b )( b c )
Mt khac lai co

1
1
1
( a b )2 + ( b c )2 + ( a c )2
2
2
2
1
1
2
2
(a c) + (a b + b c)
2
4

a 2 + b 2 + c 2 ab bc ca=

Suy ra 1 x

3
4
1
( a c )2 a c (1 x ) ; x 1
4
3
2
2

3
1 4

4
2

Suy ra P (1 + 2 x )
(1 x ) =
( 2 x + 1)(1 x )3
4 3

27

.

)
Xt hm s f ( x=

4
( 2 x + 1)(1 x )3 trn on
27

1
2 ;1 ta c

399

www.TaiLieuLuyenThi.com
Khm ph t duy K thut gii bt T Bi ton Max Min ng Thnh Nam

x =1
4
2
f '( x) =
( x 1) ( 8 x + 1) ; f '( x) =
0
x = 1
27

8.

1 81
T d c max f ( x) = f =
.
1
8 512
x ;1
2

Suy ra P 2

81
9
9

P
512
16 2
16 2

Vy gi tr nh nht ca P bng

9
t ti
16 2

3 3+ 6
1
=
,b =
,c
6 2
2 3

=
a

6 3 3
6 2

V d 8. Cho a,b,c l cc s thc tha mn iu kin a 2 + b 2 + c 2 =


8.
Tm gi tr ln nht, nh nht ca biu thc
P = ( a b) + (b c ) + (c a ) .
5

Li gii
Ta c:

( a b )5 + ( b c )5 + ( c a )5 = 5 ( a b )( b c )( c a ) ( a 2 + b2 + c 2 ab bc ca )
= 5 ( a b )( b c )( c a )( 8 ab bc ca )
Vy tm gi tr ln nht v nh nht ca P ta tm gi tr ln nht ca biu thc
P 2 = ( a b ) ( b c ) ( c a ) ( 8 ab bc ca ) .
2

Khi ta c th gi s a b c .
S dng bt ng thc AM-GM ta c:

( a b) (b c )
2

Suy ra P 2

( a b) + (b c )
(a c) .
=( a b )( b c )
=
2
16

25
( c a )6 (8 ab bc ca )2 .
16

t t = ab + bc + ca, ( t 8 ) do ab + bc + ca a 2 + b 2 + c 2 =
8.
Ta c: ( a b ) + ( b c )
2

Suy ra:

400

2
1
1
( a b ) + ( b c ) =
( a c )2 .
2
2

www.TaiLieuLuyenThi.com
Cty TNHH MTV DVVH Khang Vit

3
( a c )2 ( a b )2 + ( b c )2 + ( c a )2 =2 a 2 + b2 + c 2 ab bc ca =16 2t
2
(a c)
2

32 4t
25 32 4t
100
2
P
(8 t )5
( 8 t ) = f (t )=
3
16 3
27

Mt khc:
a 2 + b 2 + c 2 + 2 ( ab + bc + ca ) =+
8 2t 0 t 4 4 t 8 .
( a + b + c )2 =

Xt hm s=
f (t )

100
(8 t )5 trn on [ 4;8] ta c:
27

500
f '(t ) =

( t 8)2 0, do f(t) l hm nghch bin trn on [ 4;8] .


27
Suy ra P f (t ) =
f (4) 921600 960 P 960 .

Vi a =
2, b =
0, c =
2 P=
960 . Vi a =
2, b =
2, c =
0 th P = 960 .
Vy gi tr ln nht ca P bng 960 v gi tr nh nht ca P bng 960 .
V d 9. Cho a,b,c l cc s thc khng m tho mn iu kin ab + bc + ca =
1.
1
1
1
.
Tm gi tr nh nht ca biu thc P =
+
+
a 2 + b2
b2 + c2
c2 + a2
Li gii
Khng mt tnh tng qut gi s c = min {a, b, c} , khi ta c:
2

c
c

a 2 + b2 a + + b + ;
2
2

a + c a + ;
2

b + c b + ;
2

c
c

1 = ab + bc + ca a + b +
2
2

c
c
t x =
a+ ,y=
b + suy ra
2
2
P

t t
=

1
x +y
2

1 1
+
x y

1 1
1
x y
+ xy + =
+ + .
x
y
y x
x +y
x y
+
y x
xy

1
1

khi P f (t ) =
t+ 2 .
, 0 < t

x y
2
t
+
y x
1

401

www.TaiLieuLuyenThi.com
Khm ph t duy K thut gii bt T Bi ton Max Min ng Thnh Nam

Ta c f '(t ) = 1

2
t

t3 2
t

1
< 0, t 0;
nn f(t) l hm nghch bin trn
2

1
0;
.
2

1
1
Do P f (t ) f
.
2+
=
2
2

t ti a= b= 1, c= 0 hoc cc hon
2
v(xem thm k thut s dng tnh ng cp).
Vy gi tr nh nht ca P bng 2 +

2
2
Cch 3: t=
x a=
, y b=
, z c 2 khi xy + yz + zx 1 . Bi ton a v tm gi

tr nh nht ca biu thc P =

1
+
x+ y

1
+
y+z

1
. Bi ton ny ti trch
z+x

ring v trnh by di y.
V d 10. Cho a,b,c l cc s thc khng m thay i tha mn iu kin
ab + bc + ca =
1.
1
1
1
Tm gi tr nh nht ca biu thc P =
.
+
+
a+b
b+c
c+a
Li gii
Nhn xt. Vi iu kin a,b,c khng m v ab + bc + ca =
1 ngh n P t gi tr
nh nht khi mt s bng 0 v hai s bng 1. Vy ta c ghp cp hai cn li vi
nhau xem c nh gi c qua cn thc cn li hay khng.
2

1
1
1
1
Ta c:
+
= a+b + a+c +
a+c
a+b

( a + b )( a + c )

2a + b + c
2
2a + b + c
2
.
=
+
=
+
2
2
2
2
a + ab + bc + ca
a +1
a + ab + bc + ca
a +1

Suy ra

a+b

2a + b + c + 2 a 2 + 1
.
= 2
a+c
a +1
1

Khng mt tnh tng qut gi s a = max {a, b, c} khi :


=
P

2a + b + c + 2 1 + a 2

.
b+c
1 + a2
Coi y l hm s vi bin a v b,c l cc hng s ta c:
f '(a)

bc a 2 a a 2 + 1

( a + 1) ( 2a + b + c + 2
2

402

a +1
2

0.

www.TaiLieuLuyenThi.com
Cty TNHH MTV DVVH Khang Vit

Do f(a) l hm nghch bin nn =


t a

1 bc
1
suy ra

b+c b+c

1
f (a) f
.
b+c

1
Mt khc: f
=
b+c

b+c +

1
+
b+c

b+c

( b + c )2 + 1

1
t
1
1
1
1
t t = b + c P t + +
.
=t + +
=t + +
1
4
2
t
t
t
t +1
t2 + 2
1
t + 2
t
t
1
1
trn [ 2;+ ) ta c:
t u =
u+
t + , ( u 2 ) . Khi P g (u ) =
2
t
u 2

g '(u ) = 1

(u

u
u3

u 1
> 0, u 2 .
u

Do g(u) l hm ng bin trn [ 2;+ ) nn P g (u ) g(2) =


2+
Vy gi tr nh nht ca P bng 2 +

1
.
2

1
t ti a= b= 1, c= 0 hoc cc hon v.
2

1
t
trn ( 0;+ ) ta c:
Nhn xt. Ta c th kho st trc tip hm s g (t ) = t + +
t
t4 +1

t4 +1
g '(t ) =
1

1
t2

2t 4

2
1 t4
t 4 + 1 =t 1 +
t4 +1
t2
t4 +1 t4 +1

1
t2 +1
2

t 1 2
=
t

t4 +1 t4 +1

(t 1) (t + 1)
2

t2

(t + 1)
4

t4 t2
t >0
; g '(=
t ) 0
=t 1

Ta c g(t) i du t m sang dng khi i qua t = 1 nn g(t) t cc tiu ti


1
.
t = 1 trn khong ( 0;+ ) hay min g (t )= g(1)= 2 +
t( 0;+ )
2
Vy gi tr nh nht ca P bng 2 +

1
t ti a= b= 1, c= 0 hoc cc hon v.
2

Cch 2: t=
x 2 a=
, y 2 b=
, z 2 c. Khi x 2 y 2 + y 2 z 2 + z 2 x 2 =
1.
403

www.TaiLieuLuyenThi.com
Khm ph t duy K thut gii bt T Bi ton Max Min ng Thnh Nam

Suy ra

( xy + yz + zx )2 = x 2 y 2 + y 2 z 2 + z 2 x 2 + 2 xyz ( x + y + z ) x 2 y 2 + y 2 z 2 + z 2 x 2 = 1 .
Bi ton a v tm gi tr nh nht ca biu thc
1
1
1
vi xy + yz + zx 1 .
P=
+
+
x2 + y 2
y2 + z2
z 2 + x2
y chnh l bi ton va trnh by trc .
B. BI TP RN LUYN
Bi 1. Cho x,y,z l ba s thc dng tha mn iu kin x + y + z
Tm gi tr nh nht ca biu thc P = x3 + y 3 + z 3 +

3
.
2

1 1 1
+ + .
x y z

Bi 2. Cho x,y,z l cc s thc khng m tha mn iu kin x 2 + y 2 + z 2 =


3.
Tm gi tr ln nht v nh nht ca biu thc
P = xy + yz + zx +

5
.
x+ y+z

Bi 3. Cho x,y,z l cc s thc khng m tha mn iu kin x + y + z =


1.
Tm gi tr nh nht ca biu thc

=
P 3 x 2 y 2 + y 2 z 2 + z 2 x 2 + 3 ( xy + yz + zx ) + x 2 + y 2 + z 2 .

Bi 4. Cho cc s thc x,y,z tha mn iu kin x 2 + y 2 + z 2 =


1.
Tm gi tr ln nht v nh nht ca biu thc
P = xy + yz + zx +

4
.
xy + yz + zx + 2

Bi 5. Cho x,y,z l cc s thc khng m tha mn iu kin

3 x 2 + y 2 + z 2 + xy + yz + zx =
12 .

Tm gi tr ln nht v nh nht ca biu thc


=
P

x2 + y 2 + z 2
+ xy + yz + zx .
x+ y+z

Bi 6. Cho x,y,z l cc s thc dng tha mn iu kin x + y + z =


3.
Tm gi tr nh nht ca biu thc P = x 2 + y 2 + z 2 +

xy + yz + zx
x + y2 + z2 + 3
2

Bi 7. Cho x,y,z l cc s thc dng tha mn iu kin x + y + z =


3.
404

www.TaiLieuLuyenThi.com
Cty TNHH MTV DVVH Khang Vit

Chng minh rng

x3 + y 3 + z 3
+ 8 3 xyz 9 .
3

Bi 8. Cho x,y,z l cc s thc dng tha mn iu kin x 2 + y 2 + z 2 =


3.
Tm gi tr nh nht ca
biu thc P
=

( x + y + z 1)2

x y+ y z+z x
2

1
.
x+ y+z

Bi 9. Cho x,y,z l cc s thc dng tha mn iu kin x + y + z =


3.
Tm gi tr nh nht ca biu thc P = x 2 + y 2 + z 2 +

xy + yz + zx
x y + y2 z + z2 x
2

Bi 10. Cho cc s thc dng x,y,z tha mn iu kin xy + yz + zx =


3.

Tm gi tr nh nht ca biu thc P = x 2 + y 2 + z 2

324
.
x+ y+z

Bi 11. Cho x,y,z l cc s thc dng tha mn iu kin xyz = 1 .


Tm gi tr nh nht ca biu thc P = ( x + y )( y + z )( z + x ) +

72
x + y + z +1

Bi 12. Cho a,b,c l cc s thc dng tha mn iu kin

3 a 2 + b2 + c2 = 3( a + b + c ) + 4 .

Tm gi tr nh nht ca biu thc


P = a 3 + b3 + c 3

a+b+c
a + b2 + c2
2

6 ( ab + bc + ca ) .

Bi 13. Cho x,y,z l cc s thc tha mn iu kin x 2 + y 2 + z 2 =


3.
Tm gi tr nh nht ca biu thc P =
( x + 2 )( y + 2 )( z + 2 ) .
Bi 14. Cho cc s dng x, y, z . Tm gi tr nh nht ca biu thc
S=

x2 y
z

y2 z
x

z2 x
y

13 xyz

3 xy + yz 2 + zx 2
2

3
Bi 15. Cho x,y,z l cc s thc thuc on [ 0;1] v x + y + z = .
2

Tm gi tr ln nht ca biu thc P = x 2 + y 2 + z 2 .


Bi 16. Cho x,y,z l cc s thc dng tha mn x + y + z =
10 v xy + yz + zx =
xyz .
Tm gi tr ln nht v nh nht ca biu thc P = x 2 + y 2 + z 2 .
4
x + y + z =
Bi 17. Cho x, y, z l cc s thc tha mn iu kin
.
xyz = 2
405

www.TaiLieuLuyenThi.com
Khm ph t duy K thut gii bt T Bi ton Max Min ng Thnh Nam

Chng minh rng 183 165 5 x 4 + y 4 + z 4 18 .


Bi 18. Cho a,b,c l cc s thc dng. Chng minh rng
a + 2014 b + 2014 c + 2014 a b c
+
+
+ + .
b + 2014 c + 2014 a + 2014 b c a
Bi 19. Cho x,y,z l cc s thc tha mn 1 x y z 4 .
1 1 1
1 1
Chng minh rng ( x + y + z ) + + (1 + y + z ) 1 + + .
y z
x y z

Bi 20. Cho a,b,c l cc s thc dng tho mn iu kin a + b + c =


3.

Tm gi tr ln nht ca
biu thc: P
=

2
abc
.
+3
3 + ab + bc + ca
(1 + a )(1 + b)(1 + c)

1
Bi 21. Cho x,y,z l cc s thc thuc on ;1 .
2

Tm gi tr ln nht, nh nht ca biu thc P =

y+z z+x x+ y
.
+
+
x +1 y +1 z +1

Bi 22. Cho a,b,c l cc s thc tha mn a + b + c =


3.

Chng minh rng abc a 2 + b 2 + c 2 3 .


Bi 23. Cho x,y,z l cc s thc tha mn iu kin x + y + z =
0 v x 2 + y 2 + z 2 =
1.
Tm gi tr ln nht ca biu thc P = x 2 y 2 z 2 .
Bi 24. Cho a,b,c l cc s thc. Chng minh rng

6 ( a + b + c ) a 2 + b 2 + c 2 + 27 abc 10

(a

+ b2 + c2

Bi 25. Cho a,b,c l cc s thc dng tha mn iu kin ab + bc + ca =


3.
1
4
.
Tm gi tr nh nht ca biu thc=
P
+
abc ( a + b )( b + c )( c + a )
Bi 26. Cho a,b,c,d,e l cc s thc dng thay i tha mn a + b + c + d + e =
1.
Tm gi tr ln nht ca biu thc P = abc + bcd + cde + eda + eab .
Bi 27. Cho a,b,c l cc s thc dng tha mn iu kin ab + bc + ca =
3.
Tm gi tr ln nht ca biu thc
a
b
c
=
P
+
+
a 2 b2 c2 + 2 ( a + b + c ) .
( a + b )( a + c ) ( b + c )( b + a ) ( c + a )( c + b )
Bi 28. Cho a,b,c l cc s thc thuc on 0; 3 . Tm gi tr ln nht ca biu
8
thc P =

+ 31 a 2 ( a + 1) b 2 ( b + 1) c 2 ( c + 1) .
a + b + c +1
406

www.TaiLieuLuyenThi.com
Cty TNHH MTV DVVH Khang Vit

Bi 29. Cho a,b,c l cc s thc khng m tha mn iu kin a + b + c =


3.
Tm gi tr ln nht v nh nht ca biu thc
a 4 + b4 + c4

+ 3 ( ab + bc + ca ) .
a 3 + b3 + c 3
Bi 30. Cho a,b,c l cc s thc khng m v khng c hai s no ng thi bng
0. Tm gi tr nh nht ca biu thc
1
1
1
.
P=
+
+
a 2 ab + b 2
b 2 bc + c 2
c 2 ca + a 2
Bi 31. Cho a,b,c l cc s thc khng m tha mn iu kin ab + bc + ca > 0 .
P
=

Tm gi tr nh nht=
ca biu thc P

2 a 2 + b2 + c2
a+b+c

) + 3.

abc

4 a b + b2c + c 2 a
Bi 32. Cho x,y,z l cc s thc thuc khong ( 0;1) tha mn iu kin
2

xyz =
(1 x )(1 y )(1 z ) .

Tm gi tr nh nht ca biu thc P = x 2 + y 2 + z 2 .


Bi 33. Cho x,y,z l cc s thc tha mn iu kin x3 + y 3 + z 3 3 xyz =
1.
Tm gi tr nh nht ca biu thc P = x 2 + y 2 + z 2 .
0
x + y + z =

Bi 34. Cho x,y,z l cc s thc tho mn iu kin 2


8.
2
2
x + y + z =
3

Tm gi tr ln nht, gi tr nh nht ca biu thc P = xy 2 + yz 2 + zx 2 .


a 2 + c 2 =
1
Bi 35. Cho a,b,c l cc s thc tho mn iu kin
.
2
6
b + 2b ( a + c ) =

Chng minh rng b ( c a ) 4 .


Bi 36. Cho a,b,c s thc khng m tha mn iu kin
a + 2b + 1 + a + 2c + 1 =
4.

Tm gi tr ln nht ca biu thc: P = a (1 + b ) + b (1 + c ) + c (1 + a ) .


Bi 37. Cho x,y,z l cc s thc khng m tho mn iu kin x3 + y 3 + z 3 =2 + 3 xyz .
Tm gi tr nh nht ca biu thc P =

x2 + 2 y 2 + z 2
x2 + y 2 + z 2 + 1

407

www.TaiLieuLuyenThi.com
Khm ph t duy K thut gii bt T Bi ton Max Min ng Thnh Nam

Bi 38. Cho a,b,c l cc s thc khng m tha mn iu kin a + b + c =


3.
Tm gi tr ln nht v nh nht ca biu thc P =( a 1) + ( b 1) + ( c 1) .
3

Bi 3 9. Cho a,b,c l cc s thc khng m i mt khc nhau.


Tm gi tr nh nht ca biu thc
1
1
1
2
2
2
.
P = ( a + b ) + ( b + c ) + ( c + a )
+
+

( a b )2 ( b c )2 ( c a )2

D. HNG DN GII P S
3

1 1 1
9
x+ y+z
.
Bi 1. Ta c: x3 + y 3 + z 3 3
v + +
3
x y z x+ y+z

9
x+ y+z
Suy ra P 3
.
+
3
x+ y+z

t t = x + y + z , 0 < t

Xt hm s f (t=
)
f '(t )=

3
t3 9
ta
c
P

+ .

2
9 t

3
t3 9
+ vi 0 < t ta c
2
9 t

t 2 9 t 3 27
3
3 51
.
2=
< 0, t 0; P f (t ) f =
2
3 t
3t
2
2 8

Vy gi tr nh nht ca P bng

51
1
t ti x= y= z=
.
8
2

t2 3
.
Bi 2. t t =x + y + z t 2 =3 + 2 ( xy + yz + zx ) xy + yz + zx =
2

Do 0 xy + yz + zx x 2 + y 2 + z 2 = 3 0

t2 3
3 t 3;3 .
2

Khi =
P f=
(t )

t2 3 5
+ .
2
t

Xt hm s =
f (t )

t2 3 5
+ vi t 3;3 ta c
2
t

f '(t ) = t

5
t2

t3 5
t2

> 0, t 3;3 .

5
Vy f(t) l hm ng bin trn on 3;3 . Do P =f (t ) f ( 3) = .
3

ng thc xy ra khi v ch khi c mt s bng


408

3 v hai s cn li bng 0.

www.TaiLieuLuyenThi.com
Cty TNHH MTV DVVH Khang Vit

14
. ng thc xy ra khi v ch khi x= y= z= 1 .
P f (3) =
3
5
t ti x= 3, y= z= 0 hoc cc hon v ;
Vy gi tr nh nht ca P bng
3
14
gi tr ln nht ca P bng
t ti x= y= z= 1 .
3

Bi 3. Ta c
=
P 3 x 2 y 2 + y 2 z 2 + z 2 x 2 + xy + yz + zx + 1 1 .
ng thc xy ra khi v ch khi x= y= 0; z= 1 hoc cc hon v.
1
Bi 4. HD : nh gi t= xy + yz + zx ;1 .
2

Bi 5. HD : nh gi t = x 2 + y 2 + z 2 [3;4] .
Bi 6. HD : t t = x 2 + y 2 + z 2 [3;9 ) .
Bi 7. HD : S dng ng thc kt hp AM GM ta c :
x3 + y 3 + z 3 = ( x + y + z ) 3 ( x + y )( y + z )( z + x )
3

= 27 3 ( x + y )( y + z )( z + x ) 27 24 xyz

a v kho st hm s vi t = xyz .
Bi 8. S dng bt ng thc C-S ta c :

( x + y + z ) ( x 2 + y 2 + z 2 )=

(x

) (

) (

+ xy 2 + y 3 + yz 2 + z 3 + zx 2 + x 2 y + y 2 z + z 2 x

2 x 2 y + 2 y 2 z + 2 z 2 x + x 2 y + y 2 z + z 2 x= 3 x 2 y + y 2 z + z 2 x

Do x y + y z + z
2

2
x + y + z 1)
(
+
x x+ y+zP

x+ y+z

1
.
x+ y+z

t t = x + y + z . Ta c

x2 + y 2 + z 2 < ( x + y + z ) 3 x2 + y 2 + z 2 t
2

Xt hm s=
f (t )
f '(t ) = 1

10
t2

( t 1)2 + t vi
t

t 2 10
t2

< 0, t

3;3 .

3;3 ta c

3;3 .

13
.
3;3 . Suy ra P f (t ) f (3) =
3
ng thc xy ra khi v ch khi x= y= z= 1 .

Do f(t) l hm nghch bin trn

409

www.TaiLieuLuyenThi.com
Khm ph t duy K thut gii bt T Bi ton Max Min ng Thnh Nam

Vy gi tr nh nht ca P bng
Bi 9. Ta c :

(
= (x

)
)+(y

13
t ti x= y= z= 1 .
3

3 x 2 + y 2 + z 2 =( x + y + z ) x 2 + y 2 + z 2
3

+ xy 2

) (

+ yz 2 + z 3 + zx 2 + x 2 y + y 2 z + z 2 x

2 x 2 y + 2 y 2 z + 2 z 2 x + x 2 y + y 2 z + z 2 x= 3 x 2 y + y 2 z + z 2 x

x2 y + y 2 z + z 2 x x2 + y 2 + z 2

Suy ra :
Px + y +z +
2

xy + yz + zx

= x2 + y 2 + z 2 +

x2 + y 2 + z 2

=x +y +z
2

2
x + y + z ) x2 y 2 z 2
(
+

2 x2 + y 2 + z 2

9 x2 y 2 z 2

2 x2 + y 2 + z 2

t t = x 2 + y 2 + z 2 , ( t 3) khi P f (t ) =
t+

9t
4.
f (3) =
2t

ng thc xy ra khi v ch khi x= y= z= 1 .


Bi 10. HD : t t = x + y + z , ( t 3) .
Bi 11. S dng ng thc kt hp bt ng thc AM GM ta c :

( x + y )( y + z )( z + x ) = ( x + y + z )( xy + yz + zx ) xyz
= ( x + y + z )( xy + yz + zx ) 1
.
( x + y + z ) 3 xyz ( x + y + z ) 1
= ( x + y + z ) 3( x + y + z ) 1

t t = x + y + z , ( t 3) khi P t 3t +
Xt hm s f (t ) =t 3t +
3 3t ( t + 1) 72
3

=
f '(t )

( t + 1)

72
1.
t +1

72
1 vi t 3 ta c
t +1

> 0, t 3 do f(t) l hm ng bin trn [3;+ ) .

V vy P f (t ) f (3) =
44 . ng thc xy ra khi v ch khi x= y= z= 1 .
Vy gi tr nh nht ca P bng 44 t ti x= y= z= 1 .
410

www.TaiLieuLuyenThi.com
Cty TNHH MTV DVVH Khang Vit

Bi 12. Ta c
2
2
6 ( ab + bc + ca )= 3 ( a + b + c ) a 2 b 2 c 2 = 3 ( a + b + c ) 3 ( a + b + c ) 4 .

V theo bt ng thc AM GM dng lu tha ta c :


1
3
a 3 + b3 + c 3 ( a + b + c ) .
9

Suy ra

3
a + b + c)
(

3( a + b + c )

3( a + b + c ) + 4

3( a + b + c ) + 3( a + b + c ) + 4 .
2

Theo gi thit ta c : 3 ( a + b + c ) +=
4 3 a 2 + b2 + c2 ( a + b + c )

( a + b + c + 1)( a + b + c 4 ) 0 a + b + c 4 .

Suy ra

3( a + b + c )

3( a + b + c ) + 4

Do P

=1 +

( a + b + c )3 3

4
4
3
1 +
= .
3( a + b + c ) + 4
3.4 + 4
4

( a + b + c )2 + 3 ( a + b + c ) +

9
t t = a + b + c, ( 0 < t 4 ) khi :

13
.
4

t3
13 1
923
923
3t 2 + 3t + = ( t 4 ) t 2 23t 65

, 0 < t 4 .
9
4 9
36
36
4
.
ng thc xy ra khi v ch khi a= b= c=
3
923
4
t ti a= b= c=
.
Vy gi tr nh nht ca P bng
36
3
P

Bi 13. Theo gi thit ta c 3 x, y, z 3 .


Do x + 2 > 0, y + 2 > 0, z + 2 > 0 .
V vy P t gi tr nh nht khi v ch khi c x,y,z u m.
Khng mt tnh tng qut gi s x = max { x, y, z} khi 1 x 0 v

P=

( x + 2 ) ( yz + 2 ( y + z ) + 4 ) =

1
1
( x + 2 ) ( y + z + 2 )2 + x 2 + 1 ( x + 2 ) x 2 + 1
2
2

1
Xt hm s f ( x) = ( x + 2 ) x 2 + 1 vi x [ 1;0] ta c
2
x = 1
3 2
1
.
f '( x) = x + 2 x + ; f '( x) =0
x = 1
2
2

3
1 25
.
Lp bng bin thin suy ra min f ( x) = f =
x[ 1;0]
3 27
411

www.TaiLieuLuyenThi.com
Khm ph t duy K thut gii bt T Bi ton Max Min ng Thnh Nam

ng thc xy ra khi v ch khi


1

x = 3

1 1 5 1 5 1
( x; y; z ) = ; ; ; ; ; .
y + z + 2 =0
3 3 3 3 3 3
2
2
2
x
+
y
+
z
=
3

25
1
5
t ti hai bin bng v mt bin bng .
27
3
3

Vy gi tr nh nht ca P bng
Bi 14. Ta c
+
+

xy 2 + yz 2 + zx 2 y z x
=
+ +
z x y
xyz
x2 y
z3

y2 z
x3

z2 x
y/z
z/x
x/ y
=
+
+
3
2
2
( z / x)
y
( x / y ) ( y / z )2
=

a
b

b
c

c
a

1 1 1
+ +
a b c

= ab + bc + ca 3abc ( a + b + c ) =

3( a + b + c )

13
x
y
z
, t = a + b + c 3, a = , b = , c =
3t
y
z
x
3
13
40
+ f '(=
2 > 0, t 3 S f (t ) f (3) =
t)
9
2 3t 3t
Bi tp tng t
Cho x,y,z l cc s thc dng. Tm gi tr nh nht ca biu thc
S f (t ) = 3t +

P=

x5
3 2

y z

y5
3 2

z x

z5
3 2

x y

9 xyz
2( x y + y 2 z + z 2 x)
2

1
Bi 15. Khng mt tnh tng qut gi s x = max { x, y, z} , khi x ;1 .
2

Ta c P = x + y + z = x + ( y + z ) 2 yz x + ( y + z )
2

= x + x .
2

1
Xt hm s f ( x) = x 2 + x trn on ;1 ta c
2

2
5 3 9
3
1
.
(1)
; f =
f '( x) = 4 x 3; f '( x) = 0 x = . Ta c f =
f=

4 4 8
4
2
1
5
Do P ng thc xy ra khi v ch khi=
x
=
, y 0,=
z 1 v cc hon v.
2
4
412

www.TaiLieuLuyenThi.com
Cty TNHH MTV DVVH Khang Vit

Bi 16. Ta c : P = ( x + y + z ) 2 ( xy + yz + zx ) = 100 2 xyz .


2

y + z = 10 x
y + z = 10 x

T iu kin ta c :

x (10 x ) .
yz ( x 1)= x ( y + z )= x (10 x ) yz =
x 1

Suy ra =
P f (=
x) 100

2 x 2 (10 x )
x 1

Mt khc : ( y + z ) 4 yz (10 x ) 4.
2

x (10 x )
x 1

( x 1)(10 x ) 4 x x 7 x + 10 0 2 x 5 .
2

Xt hm s f (=
x) 100
=
f '(x)

2 x 2 (10 x )

2 x 2 x 2 13 x + 20

( x 1)2

Bng bin thin :


t
2
+

f(t)
f(t)

x 1

) ; f '( x=)
5
2
0
75
2

lin tc trn [ 2;5] ta c

x=
x[ 2;5]

0
2.

x = 4

5
+

75
2

36

36

5
75
t ti x= 5, y= z=
.
2
2
Gi tr nh nht ca P bng 36 t ti x= 2, y= z= 4 (ch ch c mt trng

T bng bin thin suy ra gi tr ln nht ca P bng

hp xy ra du bng bt ng thc ( y + z ) 4 yz trong mi trng hp max


2

v min).

Bi 17. Ta c P = x 4 + y 4 + z 4 = x 2 + y 2 + z 2

2 x2 y 2 + y 2 z 2 + z 2 x2

2
2
= ( x + y + z ) 2 ( xy + yz + zx ) 2 ( xy + yz + zx ) + 2 xyz ( xy + yz + zx )

4
x + y + z =
Theo gi thit ta c
, t t = xy + yz + zx P = 2 t 2 32t + 144
xyz
2
=

413

www.TaiLieuLuyenThi.com
Khm ph t duy K thut gii bt T Bi ton Max Min ng Thnh Nam

Ta c

( y + z )2 4 yz ( 4 x )2

8
, gii bt phng trnh ny ta suy ra
x

3 5 x 2.
2
2
, xt hm s f ( x) = x ( 4 x ) +
trn
x
x
5 5 1
on 3 5, 2 ta c t 5,

Ta c t = x ( y + z ) + yz = x ( 4 x ) +

5 5 1
Tng t xt hm s f (t ) = 2 t 2 32t + 144 trn on 5,
ta c iu
2

phi chng minh.


Bi 18. Nhn xt. hai v ca bt ng thc nu b i s 2014 ta c ng thc
v nu thay t = 2014 ta c ngay mt hm s m f (t ) f (0) .

t +a t +b t +c
trn [ 0;+ ) .
+
+
t +b t +c t +a
Bt ng thc c chng minh nu chng minh c f(t) l hm nghch bin
trn [ 0;+ ) .

Xt hm s f (t ) =

f '(t ) =

ba

(t + b)

cb

(t + c )

ac

(t + a )

bc+ca

(t + b)

cb

(t + c )

ac

( t + a )2

1
1
1
1

= (a c)

+
b

(
)
( t + a )2 ( t + b )2
( t + b )2 ( t + c )2

( a c )( b a )( 2t + a + b ) ( b c )2 ( 2t + b + c )
( t + b )2 ( t + c )2
( t + a )2 ( t + b )2
f '(t ) 0 ta ch cn gi s a = max {a, b, c} v bi ton c chng minh.

ng thc xy ra khi v ch khi f '(t ) = 0, t 0 a = b = c .


1 1 1
1 1
Bi 19. Xt hm s f ( x) = ( x + y + z ) + + (1 + y + z ) 1 + + trn
y z
x y z

[1;4] ta c:

2
1
1 1 1 x+ y+z
1 x yz ( y + z )
f '( x) = + +
= ( y + z ) 2 =
0.
x y z
x2
x 2 yz
yz x

Do f(x) l hm nghch bin trn [1;4] suy ra f ( x) f (1) =


0 hay

414

www.TaiLieuLuyenThi.com
Cty TNHH MTV DVVH Khang Vit

1 1 1
1 1
+ + (1 + y + z ) 1 + + .
y z
x y z

Bt ng thc c chng minh. ng thc xy ra khi v ch khi x = 1 .

( x + y + z)

1 1 1
Nhn xt. tm gi tr ln nht ca P = ( x + y + z ) + + vi cng iu
x y z
1 1
kin ta xt tip hm s g ( y ) = (1 + y + z ) 1 + + (xem thm ch k thut
y z

kho st hm nhiu bin).

Bi 20. Ta c ab + bc + ca 3 3 (abc)2 .

(1 + a )(1 + b )(1 + c ) =1 + ab + bc + ca + a + b + c + abc (1 + 3 abc )

Khi P

t
+ =
,t
t +1
3 + 3t
2

abc ,0 < t 1 .

t ( t + 1) 3t 2 + t 1
t
(t )
Xt hm s =
.
f (t )
+
, 0 < t 1 f=
2
2
3 + 3t 2 t + 1
3 ( t + 1) 1 + t 2
2

T suy ra MaxP =

5
t =1 a = b = c =1.
6

Bi 21. Ta c:
x + y + z +1 x + y + z +1 x + y + z +1
=
P+3
+
+
x +1
y +1
z +1
=

1
1
1
+
+

x +1 y +1 z +1

( x + y + z + 1)

Tm gi tr nh nht ca P.
S dng bt ng thc C-S ta c:
1
1
1
9
.
+
+

x +1 y +1 z +1 x + y + z + 3
Suy ra P + 3 ( x + y + z + 1) .

9
.
x+ y+ z+3

9 ( t + 1)
3
t t = x + y + z , t ;3 khi P f (t=
)
3.
t +3
2

Xt hm s
=
f (t )

9 ( t + 1)

3
3 lin tc trn on ;3 ta c:
t +3
2
415

www.TaiLieuLuyenThi.com
Khm ph t duy K thut gii bt T Bi ton Max Min ng Thnh Nam

=
f '(t )

18

( t + 3)

3
3
> 0, t ;3 do f(t) ng bin trn ;3 .
2
2

1
3
Suy ra P f (t ) f =
.
5 . ng thc xy ra khi v ch khi x= y= z=
2
2
1
Vy gi tr nh nht ca P bng 5 t ti x= y= z=
.
2
Tm gi tr ln nht ca P.
Ta c:

(1 x ) x

1
3
1
2
2
0 x + x 0 2 x + 3x 1 0
2
2
2

1
x 5
( x + 1)( 2 x + 5 ) 6
+
x +1
3 6
1
y 5 1
z 5
Tng t ta c:
+ ;
+ .
y +1
3 6 z +1
3 6

Cng theo v 3 bt ng thc trn ta c:


1
1
1
x+ y+z 5
+
+

+ .
x +1 y +1 z +1
3
2
x+ y+ z 5
Suy ra P ( x + y + z + 1)
+ .
3
2

3
t 5 1
t t = x + y + z , t ;3 ta c: P ( t + 1) + =
( 2t 7 )( 3 t ) + 6 6 .

2
3 2 6
ng thc xy ra khi v ch khi x= y= z= 1 .

Vy gi tr ln nht ca P bng 6 t ti x= y= z= 1 .
Bi 22. Gi biu thc v tri ca bt ng thc l P ta c
2
P= abc ( a + b + c ) 2 ( ab + bc + ca ) = 9abc 2abc ( ab + bc + ca ) .

Ta c ( ab + bc + ca ) 3abc ( a + b + c ) =
9abc .
2

Suy ra
P

( ab + bc + ca )2 9 2
9

ab + bc + ca )
(=

t t = ab + bc + ca suy ra P f (t ) =
Xt hm s =
f (t )
416

2 ( ab + bc + ca )

2t 3 2
+t .
9

2t 3 2
+ t vi t ( 0;3] ta c
9

+ ( ab + bc + ca ) .
2

www.TaiLieuLuyenThi.com
Cty TNHH MTV DVVH Khang Vit

2t 2
+ 2t ; f '(t ) =0 t =3 .
3
Ta c f(t) i du t m sang dng khi i qua t = 3 nn f(t) t cc i ti
t = 3 . Suy ra P f (t ) f (3) =
3 . ng thc xy ra khi v ch khi a= b= c= 1 .
f '(t ) =

Bi 23. Ta c: 2 yz = ( y + z ) y 2 + z 2 = x 2 1 x 2 = 2 x 2 1 .
2

2 2

1
1
Suy ra P = x x = x 6 x 4 + x 2 .
2
4

Mt khc: ( y + z ) 4 yz x 2 2 2 x 2 1 0 x 2
2

2
.
3

1
2
t
t x 2 , t 0; v P = f (t ) = t 3 t 2 + t .
=
4
3


1
Xt hm s f (t ) = t 3 t 2 + t lin tc trn on
4

2
0; 3 ta c

1
1
1
f '(t ) = 3t 2 2t + ; f '(t ) = 0 t = hoc t = .
4
6
2
1
1
2 1
.
Ta c =
f (0) f=
0, f=
f=

2
6
3 54
1
1
Suy ra max f (t ) =
. Do P ng thc xy ra khi
2
54
54
t 0;
3

x=

2
1
.
,y= z =
3
6

Vy gi tr ln nht ca P bng

1
t ti x =
54

2
1
,y= z =
.
3
6

Bi 24. Nu a 2 + b 2 + c 2 =0 a =b =c =0 bt ng thc tr thnh ng thc.


+ Nu a 2 + b 2 + c 2 > 0 vit li bt ng thc di dng :

6 ( a + b + c ) a 2 + b 2 + c 2 27 abc

(a
t x
=

+b +c
2

a
=
,y
a 2 + b2 + c2

2 3

10 .

b
=
,z
a 2 + b2 + c2

c
a +b +c
2

x2=
+ y2 + z2 1 .

Bt ng thc tr thnh: 6 ( x + y + z ) 27 xyz 10 2 ( x + y + z ) 9 xyz

10
.
3
417

www.TaiLieuLuyenThi.com
Khm ph t duy K thut gii bt T Bi ton Max Min ng Thnh Nam

x2 + y 2 + z 2 1
Khng mt tnh tng qut=
gi s x 2 max x 2 , y 2 , z 2 x 2 =
.
3
3
Ta c

( x ( 2 9 yz ) + 2 ( y + z ) )2 x2 + ( y + z )2 . ( 2 9 yz )2 + 4= ( 2 yz + 1) (81y 2 z 2 36 yz + 8) .
t t = yz t

y 2 + z 2 1 x2 1
=
.
2
2
3

1 1
Khi P f (t ) = ( 2t + 1) 81t 2 36t + 8 vi t ; ta c:
3 3

t=

9
.
f '(t ) =486t 2 + 18t 20; f '(t ) =0
t = 5
27
1 29 2 100 5 1369 1 25
Ta c f =
.
; f =
;f =
;f=

9
3 3
9
27 243 3 3

100
10
2 100
Suy ra Max f (t ) = f =
P2
P .
1 1
9
9
9
3

t ;
3 3

ng thc xy ra chng hn ti a =
1, b =
c=
2.
Bi 25. Ta c:

( a + b )( b + c )( c + a ) = ( a + b + c )( ab + bc + ca ) abc = 3 ( a + b + c ) abc .
Khi =
P

1
4
.
+
abc 3 ( a + b + c ) abc

Mt khc ( ab + bc + ca ) 3abc ( a + b + c ) a + b + c
2

3
.
abc

1
4
1
4abc
.
+
=
+
2
9
abc
abc abc 9 ( abc )
abc
1
4t
.
t
=
t abc, ( 0 < t 1) khi P f (t) =+
t 9 t2

Suy ra P

Xt hm s f (t)=

f '(t ) =

418

1
t2

1
4t
trn ( 0;1] ta c:
+
t 9 t2

4t 2 + 36

9 t2

) (
(9 t )

t 2 4t 2 + 36 9 t 2
t2

2 2

< 0, t ( 0;1]

www.TaiLieuLuyenThi.com
Cty TNHH MTV DVVH Khang Vit

3
nn f(t) l hm nghch bin trn ( 0;1] . Do f (t ) f (1) =
.
2
3
Vy gi tr nh nht ca P bng t ti a= b= c= 1 .
2
Cch 2: Ta c th nh gi nhanh thng qua bt ng thc AM-GM nh sau:
1
1
4
1
1
4
P=
+
+
+
+
2abc 2abc ( a + b )( b + c )( c + a ) 2 2abc ( a + b )( b + c )( c + a )

1
+2
2

2
1
2
3
+2
=
3
2
( ac + bc )( ab + ac )( bc + ab ) 2
2ab + 2bc + 2ca

3
xy ra khi a= b= c= 1 .
2
Bi 26. Khng mt tnh tng qut gi s e = min {a, b, c, d , e} vit li biu thc P

Vy gi tr nh nht ca P bng

di dng: P
= bc ( a + d e ) + e ( a + c )( b + d ) .
S dng bt ng thc AM-GM ta c:
3

b + c + a + d e 1 2e
bc ( a + d e )
=

v
3

3
a+c+b+d
1 e
e ( a + c )( b + d ) e
e
=
.
2

2
3

1 2e
1 e
Suy ra P
+ e
.
3
2

Vi a, b, c, d , e > 0 v a + b + c + d + e = 1, e = min {a, b, c, d , e} 0 < e


3

1
.
5

1 2e
1 e
1
Xt hm=
s f (e)
+ e
trn 0; ta c:
3
2
5
f '(e=
)

1
1
1
5e2 4e + 1=
(1 5e )( e + 1) 0, e 0; nn f(e) l hm
36
36
5

1 1
1
ng bin trn 0; . Do f (e) f =
.
5 25
5
1
1
t ti a= b= c= d= e=
.
Vy gi tr ln nht ca P bng
25
5
a
b
c
Bi 27. Ta c:
+
+
( a + b )( a + c ) ( b + c )( b + a ) ( c + a )( c + b )
419

www.TaiLieuLuyenThi.com
Khm ph t duy K thut gii bt T Bi ton Max Min ng Thnh Nam

2 ( ab + bc + ca )
=
( a + b )( b + c )( c + a )

6
.
( a + b )( b + c )( c + a )

V
2 ( a + b + c ) a 2 b 2 c 2= 2 ( a + b + c ) ( a + b + c ) + 2 ( ab + bc + ca )
2

= 2(a + b + c) (a + b + c) + 6
2

Mt khc:

( a + b )( b + c )( c + a ) = ( a + b + c )( ab + bc + ca ) abc
1
( a + b + c )( ab + bc + ca )
9
8
8
Suy ra: ( a + b )( b + c )( c + a ) ( a + b + c )( ab + bc + ca=
) (a + b + c) .
9
3
9
2
Suy ra P
+ 2(a + b + c) (a + b + c) + 6 .
4(a + b + c)
( a + b + c )( ab + bc + ca )

t t = a + b + c, t 3 ( ab + bc + ca ) = 3 P f (t ) =
Xt hm s f (t ) =
f '(t ) =

9
4t 2

9
+ 2t t 2 + 6 .
4t

9
+ 2t t 2 + 6 vi t 3 ta c:
4t

+ 2 (1 t ) < 0, t 3 . Do f(t) l hm nghch bin trn [3;+ ) .

15
. ng thc xy ra khi v ch khi a= b= c= 1 .
Suy ra P f (t ) f (3) =
4
15
t ti a= b= c= 1 .
Vy gi tr ln nht ca P bng
4
Bi 28. S dng bt ng thc AM-GM ta c:
1
3 1
2
a 2 ( a + 1) + b 2 ( b + 1) + c 2 ( c + 1) = a3 + b3 + c3 + a 2 + b 2 + c 2 ( a + b + c ) + ( a + b + c ) .
9
3

t3 t 2
8
t t =a + b + c, t 0;3 3 P f (t ) =
+ 31 .
9 3
t +1

Xt hm s f (t ) =

=
f '(t )

420

1
2

( t + 1)3

t3 t 2
8
+ 31 lin tc trn on 0;3 3 ta c:
9 3
t +1

.
3
2
t 3t + 279

t (t + 2)

www.TaiLieuLuyenThi.com
Cty TNHH MTV DVVH Khang Vit

Ta c: f '(t ) = 0 8 t 3 3t 2 + 279 t ( t + 2 )

( t + 1)3 t = 3 (do

v tri l

hm s nghch bin).
Ta c f '(t ) i du t dng sang m khi i qua t = 3 nn f(t) t cc i ti
t = 3 . Do P f (t ) f (3) =
3.

Vy gi tr ln nht ca P bng 3 t ti a= b= c= 1 .
Bi 29. Tm gi tr nh nht ca P
Ta c: x 4 3 x3 9 x ( x 3) = x ( x 3)
Nu ab + bc + ca

a +b +c
3

( x + 3) 0, x [0;3] .

9
9 27
P 3. =
> 3.
5
5 5

Xt vi ab + bc + ca
a 4 + b4 + c4

3 + 9.

9
ta c:
5

a ( a 3) + b ( b 3) + c ( c 3)
a 3 + b3 + c 3

a 2 + b2 + c2 3( a + b + c )
a 2 + b2 + c2 ( a + b + c )
=
3 + 9.
=
3
+
9.
a 3 + b3 + c 3
a 3 + b3 + c 3
ab + bc + ca
ab + bc + ca
ab + bc + ca
= 3 18. 3 3 3 3 18. 3 3 3
= 3 18.
3. 9 3 ( ab + bc + ca )
a +b +c
a + b + c 3abc
2

= 3 + 2.

ab + bc + ca
ab + bc + ca 3

Vy P 3 + 2.

ab + bc + ca
+ 3 ( ab + bc + ca ) .
ab + bc + ca 3

9
2t
6
t t = ab + bc + ca, t 0; khi P f (t ) = 3 +
.
+ 3t = 3t + 5 +
t 3
t 3
5

Xt hm s f (t ) =3t + 3 +
f '(t ) =

3
5+

6
t 3

6
ta c:
t 3

4t 6

3
; f '(t ) =0 t = .
2

6
+ 1
( t 3) 5 +
t 3

+3=

421

www.TaiLieuLuyenThi.com
Khm ph t duy K thut gii bt T Bi ton Max Min ng Thnh Nam

Bng bin thin:


t

3
2
0
11
2

0
+

f(t)
f(t)

9
5

27
5

Da vo bng bin thin suy ra P f (0) =


3.
3 t ti a= 3, b= c= 0 hoc cc hon v.

Vy gi tr nh nht ca P bng

Bi 30. Khng mt tnh tng qut gi s c = min {a, b, c} khi


b 2 bc + c 2 =b 2 + c ( c b ) b 2 , c 2 ca + a 2 =a 2 + c ( c a ) a 2 .

Suy ra:
P

t t
=

1 1
=
+ +
a 2 ab + b 2 a b

a+b+c

a+b

( a + b )2 3ab
ab

(a + b)

( a + b )2 3ab

2
a + b)
(
+
=

ab

2
a + b)
(
+

1
1 3

ab

( a + b)

)
, 0 < t khi P f (t=
4

Xt hm s=
f (t )

a+b
ab

ab

1
1
+ .
1 3t t

1
1
+ trn 0; ta c
1 3t t
4
1

3
1
3
2
2.
1 3t 2 (1 3t )
4 < 0, t 0; 1 .
=
=
2
16
f '(t )

3
3
3
t
4
2 (1 3t )
2t 2 (1 3t )
2t 2 (1 3t )
3

1
1
Suy ra f(t) nghch bin trn 0; do P f (t ) f =
6.
4
4
1
Vy gi tr nh nht ca P bng 6 t ti a= b=
, c= 0 hoc cc hon v.
2
Bi 31. Khng mt tnh tng qut gi s b l s nm gia a v c.
Khi :
422

www.TaiLieuLuyenThi.com
Cty TNHH MTV DVVH Khang Vit

( b a )( b c ) 0 b2 + ac ab + bc b2c + c 2 a abc + bc 2

a 2b + b 2 c + c 2 a a 2b + abc + bc 2 = b a 2 + ac + c 2

Suy ra

abc
a b+b c+c a
2

abc

b a + ac + c
2

2 a 2 + b2 + c2

Ta cn nh gi

Ta c:

a+b+c

2 a 2 + b2 + c2
=
a+b+c

) v

2 4.

ac
.
=
2
a + ac + c 2
ac

a + ac + c 2
2

ab + bc + ca

( a + b + c )2

S dng bt ng thc AM-GM ta c:

) (

4 ( ab + bc + ca ) a 2 + ac + c 2 ab + bc + ca + a 2 + ac + c 2
2

2
= b ( a + c ) + ( a + c ) =

Suy ra

ab + bc + ca

( a + b + c )2

( a + c )2

4 a 2 + ac + c 2

(a + b + c) (a + c)
2

.
2

Do
P 2

t t =

( a + c )2
a + ac + c
2

3ac

4 a 2 + ac + c 2

3ac
ac
.
=
+
1 2
2
a + ac + c
4 a 2 + ac + c 2

,
t 1 khi :
a + ac + c 3

P f (t ) =t +

ac

3
3
3
1 t 2 = t 2 + t + .
4
4
4

2
3
3
Xt hm s f (t ) = t 2 + t + lin tc trn on ;1 ta c
4
4
3
2
2
3
f '(t ) = t + 1 < 0, t ;1 . Do f(t) nghch bin trn ;1 .
2
3
3
Suy ra P f (t ) f (1) =
a b=
,c 0 .
1 . ng thc xy ra khi v ch khi=

Vy gi tr nh nht ca P bng 1 t ti=


a b=
, c 0 hoc cc hon v.
Bi 34. y l mt biu thc hon v cn tm cch nh gi chuyn v biu thc
i xng cch thc hin l xt phn tng ng vi P c th l
423

www.TaiLieuLuyenThi.com
Khm ph t duy K thut gii bt T Bi ton Max Min ng Thnh Nam

Q = x2 y + y 2 z + z 2 x .

Theo gi thit ta c

( x + y + z )2 x 2 y 2 z 2 =
4
xy + yz + zx =

2
3

3 3 3
3
.
x + y + z = ( x + y + z ) 3 ( x + y )( y + z )( z + x ) = 3xyz

x3 y 3 + y 3 z 3 + z 3 x3 = ( xy + yz + zx )3 3 xyz ( x + y + z )( xy + yz + zx ) x 2 y 2 z 2 = 3x 2 y 2 z 2 64

27

Khi
P + Q =
xy ( x + y ) + yz ( y + z ) + zx ( x + z ) =
3 xyz

64 .
3 3
3 3
3 3
3
3
3
2 2 2
2 2 2
P.Q= x y + y z + z x + xyz x + y + z + 3 x y z = 9 x y z
27

Theo nh l Vi-t P,Q l 2 nghim ca phng trnh


64
x 2 + 3tx + 9t 2
=
0 vi t = xyz .
27

256
256
27t 2
3t
27t 2
27
27
hoc P =
.
Suy ra P =
2
2
Kho st P nh trn ta tm c Max v Min.
Bi tp tng t
3t +

Cho x,y,z l cc s thc tha mn x + y + z =


5 v x 2 + y 2 + z 2 =
9 . Tm gi tr
ln nht v nh nht ca biu thc P = x 2 y + y 2 z + z 2 x .
( a + c )2 2ac =
1

Bi 35. Theo gi thit ta c


2ac
=
6 b2
a + c =
2b

6 b2

1 .
2b

2 2
6b

Do P = b ( c a ) = b c + a 2ca = b 1
1 16 .

2b

b+c
Bi 36. Ta c P =a + (a + 1)(b + c) + bc a + (a + 1)(b + c) +
.
2
Ta ch cn rt c b+c theo a.
x + 2 =
Theo gi thit ta c
2 x =
424

a + 2b + 1
a + 2c + 1

x [ 1;1] ta c

www.TaiLieuLuyenThi.com
Cty TNHH MTV DVVH Khang Vit

2
a + b + c = x + 3
a + b + c = x 2 + 3

b c 4 x
4x
b c =
2
2
a + 2 ( b + c + 1) a + ( 2b + 1)( 2c + 1) = 4 x
Ta c

4 ( a + b + c ) 3 ( b c ) = 12 ( ab + bc + ca ) ( 2a b c ) 12 ( ab + bc + ca ) .
2

Do
P = a + b + c + ab + bc + ca a + b + c +

1
1
( a + b + c )2 ( b c )2
3
4

x2 x2 3
2
x4
1 2
2
2
= x + 3 + x + 3 4x =
x + 6=
+ 6 6, x [ 1;1]
3
3
3
Du bng xy ra khi v ch khi a= b= c= 1 .
Bi 37. Theo gi thit ta c:

2
( x + y + z ) ( x 2 + y 2 + z 2 xy yz zx ) =

1
2
3
( x + y + z ) x2 + y 2 + z 2 ( x + y + z ) =
2.
2
2

x +=
y +z
2

Khi P =
Vi x=

( x + y + z )2 +
3

x2 + 2 y 2 + z 2
x + y + z +1
2

4
34
3( x + y + z )

x2 + y 2 + z 2
x + y + z +1
2

2, y= z= 0 th P = 1

1
1+ 3 4

=1

1
x + y + z +1
2

1
.
1+ 3 4

Bi tp tng t
Cho x,y,z l cc s thc khng m tho mn iu kin x3 + y 3 + z 3 =2 + 3 xyz .
Tm gi tr nh nht ca biu thc P =x 2 + 2 y 2 + 3 z 2 .
Bi 38. Tm gi tr nh nht
Khng mt tnh tng qut gi s a = min {a, b, c} a 1 b + c = 3 a 2 .
Khi

( b 1)3 + ( c 1)3 = ( b + c 2 ) ( b 1)2 ( b 1)( c 1) + ( c 1)2


( b 1 + c 1)2 ( b 1 + c 1)2 ( b + c 2 )3
=
(b + c 2)

2
4
4

425

www.TaiLieuLuyenThi.com
Khm ph t duy K thut gii bt T Bi ton Max Min ng Thnh Nam

Suy ra
1
1
3
3
( b + c 2 )3 =( a 1)3 + (1 a )3 = ( a 1)3 , a [0;1] .
4
4
4
4
3
3
hoc cc hon v.
Vy gi tr nh nht ca P bng t ti a= 0, b= c=
2
4
+ Tm gi tr ln nht.
P ( a 1) +
3

S dng ng thc x3 + y 3 + z 3 = ( x + y + z ) 3 ( x + y )( y + z )( z + x ) .
3

( a + b + c 3)3 3 ( a + b 2 )( b + c 2 )( c + a 2 ) .
=3 ( a + b 2 )( b + c 2 )( c + a 2 )
2
2
2
P 2= 9 ( b + c 2 ) ( a + b 2 ) ( c + a 2 )
Ta c b + c 2 0, ( a + b 2 ) + ( a + c 2 ) = a 1 0 .

Ta c: P =

+ Nu c a + b 2 0, a + c 2 0 P 0 .
+ Nu tn ti mt tha s dng 1 tha s m ta gi s a + b 2 0, a + c 2 0
khi
P= 3 ( b + c 2 )( 2 b a )( a + c 2 )
2

b+c2+a+c2
3( 2 b a )

3
3
3
3
= ( c 1) . ( 3 1) = 6
4
4
ng thc xy ra khi v ch khi a= b= 0, c= 3 .
Vy gi tr ln nht ca P bng 6.
Bi 39. Gi
s c min {a, b, c} 0 c < a, b khi
=
1
1
1
2
.
P ( a + b ) + a 2 + b 2
+
+

( a b )2 a 2 b 2

Ta c:
2

1
1
a 2 + ab + b 2
a b
a b
( a + b ) 2 + a 2 + b 2 1

+
=
+
2.
+
2
+
+ 2 +

2
2
2
2

(a b)
a
b
b a
b a
(a b)

a
b
2
+1+
a + 2 a + b + 2 a + b
= 2. b

a
b
b a
b a
2+
b
a

a b
t +1

t t =+ , ( t > 2 ) khi P f (=
t) 2 t 2 + t +
.
t 2
b a

426

www.TaiLieuLuyenThi.com
Cty TNHH MTV DVVH Khang Vit

t +1

Xt hm s f (=
t) 2 t2 + t +
trn ( 2;+ ) ta c
t 2

3
3
; f '(t ) = 0 ( 2t + 1)( t 2 ) 3 = 0 .
f '(t ) = 2 2t + 1
2

( t 2 )
t >2

t
2t 3 7t 2 + 4t +=
=
1 0

5 + 33
.
4

5 + 33
th f(t) i du t m sang dng nn f(t) t cc tiu ti
4
5 + 33 59 + 11 33
5 + 33
hay f (t ) f
=.
4
4
4

Ti t =

Vy gi tr nh nht ca P bng

a b 5 + 33
59 + 11 33
t ti =
+
=
,c 0 .
b a
4
4

CH 4: K THUT S DNG TNH THUN NHT


A. NI DUNG PHNG PHP

Xt mt biu thc P ( x1 ; x2 ;...; xn ) c gi l biu thc thun nht bc k khi

P ( tx1 ; tx2 ;...; txn ) = t k P ( x1 ; x2 ;...; xn ) .


Nh cp trong ch Bt ng thc AM GM vi bt ng thc thun nht
ta c th chun ho iu kin ca cc bin chng minh cho n gin. Vi dng
ton ny chng ta tip cn a v bi ton t bin s hn bt ng thc ban u v
kt hp s dng tnh n iu ca hm s.
PHNG PHP
Phng php chung l gim bi ton n bin v (n-1) bin hoc nh hn bng
cc php t n ph.
Bng cch t

x2 = t1 x1
x3 = t2 x2
...
xn = tn 1 xn 1

Ta a v bi ton vi (n-1) bin t1 , t2 ,..., tn 1 .


Du hin nhn din:
- Gi thit bi ton c dng thun nht.
- Biu thc ca P thun nht.
427

www.TaiLieuLuyenThi.com
Khm ph t duy K thut gii bt T Bi ton Max Min ng Thnh Nam

1) K thut t x = ty hoc y = tx cho trng hp bt ng thc v bi ton


cc tr hai bin s.
V d 1. Cho x,y l cc s thc dng tha mn iu kin x 2y .
Tm gi tr nh nht ca biu thc P =

x2 + y2
.
xy

Li gii
t x = ty t x 2y t 2 v ta c:

t 2 + 1 t 2 + 1 5 5 2t 2 5t + 2 5
=
+=
+
t
t
2 2
2t
2
( t 2 )( 2t 1) + 5 0, t 2
=
2t
2
=
P

Vy gi tr nh nht ca P bng

5
t ti x = 2y .
2

Nhn xt. Ta c th x l bng cch khc nh sau:


Vi d on du bng xy ra ti x = 2y ta vit li P di dng:

3x 2 x 2 + 4y 2 3 x x 2 + 4y 2 3
4xy 5
P=
+
= . +
.2 +
= .
4xy
4xy
4 y
4xy
4
4xy 2
Trong trng hp khng d on c du bng ta s dng k thut ng cp
nh trn ta d dng tm c gi tr ln nht, nh nht ca biu thc ca P vi
vic coi P l hm ca t.
V d 2. (TSH Khi B 2006) Cho hai s thc x, y thay i v tha mn iu
kin x 2 + y 2 =
1.
Tm gi tr ln nht v gi tr nh nht ca biu thc P =

2 ( x 2 + 6 xy )
1 + 2 xy + 2 y 2

Li gii
Nhn xt. Biu thc P cha ng bc nhng nu thay x 2 + y 2 =
1 vo biu
thc ca P th ta a P v dng ng bc.

2 ( x 2 + 6 xy )
2 ( x 2 + 6 xy )
.
Vit li biu thc
P di dng: P =
=
x 2 + y 2 + 2 xy + 2 y 2 x 2 + 3 y 2 + 2 xy
TH1: Nu y = 0 ta c P = 2 .
TH2: Nu y 0 ta t x = t.y ta c:=
P
428

f=
(t )

2 ( t 2 + 6t )
t 2 + 2t + 3

www.TaiLieuLuyenThi.com
Cty TNHH MTV DVVH Khang Vit

Xt hm s f (t ) =

2 ( t 2 + 6t )

trn ta c

t 2 + 2t + 3

4 ( 2t 2 3t 9 )
t=

f '(t ) =
; f '(t ) =
0
2.
2

( t 2 + 2t + 3)
t = 3
Bng bin thin:
t

f(t)
f(t) 2

3
2

0
3

Da vo bng bin thin suy ra P=


f(3)
= 3 t ti
max

3
1
x=
3y ( x; y ) =
;

.
10
10

3
6 t ti
Pmin =
P =
2

3
3
2 3
2
x=
y ( x; y ) =
;
;

;
.
2
13
13
13
13

V d 3. Gi s x, y l cc s thc dng tha mn iu kin x + y 2.


Tm gi tr ln nht ca biu thc P = 7( x + 2 y ) 4 x 2 + 2 xy + 8 y 2 .
Li gii
V x, y l cc s thc dng nn

7( x + 2 y ) 4 x 2 + 2 xy + 8 y 2
P= ( x + y )
x+ y

7 y 4 x 2 + 2 xy + 8 y 2
.
=
( x + y) 7 +

x+ y

t
=
t

(1)

7 y 4 x 2 + 2 xy + 8 y 2 7 4 t 2 + 2t + 8
x
, t > 0 khi
=
. (2)
x+ y
t +1
y
429

www.TaiLieuLuyenThi.com
Khm ph t duy K thut gii bt T Bi ton Max Min ng Thnh Nam

Xt hm s f (t ) =
Ta c f '(t ) =

7 4 t 2 + 2t + 8
vi t > 0.
t +1

7 t 2 + 2t + 8 + 28
(t + 1)

t + 2t + 8
2

Suy ra bng bin thin

; f '(t ) = 0 t 2 + 2t + 8 = 4 t = 2.

f '(t )

f (t )
T bng bin thin ta suy ra f (t ) 3 vi mi t > 0.
(3)
Du ng thc xy ra khi v ch khi t = 2.
T (1), (2) v (3) ta suy ra P ( x + y )(7 3) 8, du ng thc xy ra khi v

2
x + y =
4
2

ch khi
=
x
y
,=
.
x
=
t
=
2
3
3

4
2
=
,y
.
3
3
V d 4. (TSH Khi D 2013) Cho x,y l hai s thc dng tha mn iu kin
xy y 1 . Tm gi tr ln nht ca biu thc
Vy gi tr ln nht ca P l 8, t khi
=
x

P
=

x+y
x xy + 3y
2

x 2y
.
6 (x + y)

Li gii
Nhn xt. Nhn thy P ng bc nn ta tm cch nh gi min gi tr ca t =

x
v
y

a v kho st hm s vi t.
2

1 1 1 1
x y 1 1 1
Xut pht t gi thit ta c: 0 < 2 = 2 =
+ .
y
y y
y
y 2 4 4
t t =

Xt hm
s f(t)
=
430

t +1

x
1
0 < t=
v P
y
4
t +1
t2 t + 3

t2 t + 3

t 2
trn
6 ( t + 1)

t 2
.
6 ( t + 1)
1
0; ta c:
4

www.TaiLieuLuyenThi.com
Cty TNHH MTV DVVH Khang Vit

7 3t

f '(t)
=

(t

Vi 0 < t

t +3

(t

1
2 ( t + 1)

1
ta c t 2 t + 3= t ( t 1) + 3 < 3; 7 3t > 6; t + 1 > 1 .
4

7 3t

t +3

1
2 ( t + 1)

>

1
=
2.3 3 2

1
3

1
> 0.
2

1
4

Vy f(t) l hm ng bin trn 0; .

1
4

5
3

Do f(t) f = +

7
.
30

Vy gi tr ln nht ca P bng

5 7
1
t ti=
+
x =
,y 2 .
3 30
2

2) K thut gim bin vi bt ng thc v bi ton cc tr ba bin


Ta thng t
=
a x=
.b; b y.c .
V d 1. (Bt ng thc Schur) Cho a,b,c l cc s thc khng m ta lun c

a3 + b3 + c3 + 3abc ab ( a + b ) + bc ( b + c ) + ca ( c + a ) .

Chng minh.
Khng mt tnh tng qut gi s a b c khi tn ti hai s thc khng m

)
( )
( x + 1) + ( y + 1) + 1 + 3 ( x + 1)( y + 1) ( x + 1)( y + 1)( x + y + 2 ) + ( x + 1)( x + 2 ) + ( y + 1)( y + 2 )
( x y ) ( x + y + 1) 0 (lun ng).
V d 2. Cho x,y,z l cc s thc dng tha mn iu kin z ( x + y ) =
xy .
x,y sao cho a =+
x 1 c, b =+
y 1 c v bt ng thc tng ng vi:
3

Tm gi tr nh nht ca biu thc


=
P

x2 + y2
z
.
+
2
x+y
z

Li gii
t
ab .
=
x a.z,y
= b.z ta c a + b =
Khi

P =a2 + b2 +

2
2
1
1
1
.
=( a + b ) 2ab +
=( a + b ) 2 ( a + b ) +
a+ b
a+ b
a+ b

431

www.TaiLieuLuyenThi.com
Khm ph t duy K thut gii bt T Bi ton Max Min ng Thnh Nam

Mt khc a + b

4ab ( a + b ) 4 ( a + b ) a + b 4 .
2

1
t

t t =
a + b, t 4 khi P = f(t) = t 2 2t + .

1
t

t 3 2t 2 1
> 0, t 4 nn
t2
33
.
f(t) l hm ng bin trn 4; + ) suy ra P = f(t) f(4) =
4
c f '(t)
Xt hm s f(t) = t 2 2t + vi t 4 ta=

Vy gi tr nh nht ca P bng
Cch 2: T gii thit suy ra

33
t ti x= y= 2z .
4

xy x + y
.
=
z
z2

x+y
x+y
z
Khi P =
.
2
+
z
z x+y
2

x+y
xy x + y
t2
t t =
t= 2
=
t 4.

z
4
z
2z

1
t

Khi P = f(t) = t 2 2t + .

t 3 2t 2 1
> 0, t 4 nn
t2
33
.
f(t) l hm ng bin trn 4; + ) suy ra P = f(t) f(4) =
4

1
t

c f '(t)
Xt hm s f(t) = t 2 2t + vi t 4 ta=

Vy gi tr nh nht ca P bng

33
t ti x= y= 2z .
4

V d 3. Cho a,b,c l cc s thc dng. Chng minh

a b c a+b b+c
+ +
+
+1 .
b c a b+c a+b
Li gii

a+b
b+c

=
a x=
.b; c y.b ( x, y > 0 ) khi=
Bt ng thc tr thnh: x +

432

x +1 b + c y +1
.
=
;
y +1 a + b x +1

1 y x +1 y +1
+
+
+1
y x y +1 x +1

www.TaiLieuLuyenThi.com
Cty TNHH MTV DVVH Khang Vit

x3 y 2 + x 2 + x + y 3 + y 2 x 2 y + 2 xy 2 + 2 xy .
Bt ng thc cui ng v l tng ca ba bt ng

x3 y 2 + x
x3 y 2 + x + y 3 + y 3
x 2 y;
2 xy 2 ; x 2 + y 2 2 xy .
2
2
Bt ng thc c chng minh. ng thc xy ra khi v ch khi a= b= c .

V d 4. Cho x,y,z l cc s thc thuc on [1; 2] .


Tm gi tr gi tr ln nht ca biu thc
3

x 2 + z 2 + 4 xz y 2 + 2 yz 5 z 2
2
=
P
+
3x x .
2
2
2
x +z
yz 4 z

Li gii

t
=
x a=
.z; y b.z; a, b ; 2 .
2
3

a 2 + 4a + 1 b 2 + 2b 5
2
Khi
P
=
+
3x x .
2
a +1 b 4
Xt hm s

=
f (a)

a 2 + 4a + 1
b 2 + 2b 5
trn on
=
;
g
(
b
)
a2 + 1
b4

1
2 ; 2

v h( x) =
3 x x 2 trn on [1; 2] ta c

f '(a ) =

4 ( a 2 1)

1
; f '(a ) =
0 a =
1 ; 2 .
2
( a + 1)
2

Suy ra max f =
(a)

=
g '(b)

b 2 8b 3

(b 4)

1 13
.
f=
(1) 3; min f =
(a ) f =
(2) f =

2 5

1
< 0, b ; 2
2

3
1 15
; min g (b) =
max g (b) =
g =
g (2) =

2
2 14
h '( x) =

3 2x

3
; h '( x) =
0 x=
2
2 3x x 2

3
3
max h( x) =
h(1) =
h(2) 2; min h( x) =
h =

2
2
433

www.TaiLieuLuyenThi.com
Khm ph t duy K thut gii bt T Bi ton Max Min ng Thnh Nam

Do vy P = ( f (a ))3 + ( g (b))3 + h( x) .
3

V vy Pmax

15
=
3 + 2 .
( max f (a) ) + ( max g (b) ) + max h( x) =
14
3

=
a 1=
x z
x = 2

1
1

Du bng xy ra khi v ch khi b = y = z y =1 .


2
2

z = 2

x 1=
x 1
=

=
x 2=
x 2

ac + bc =

V d 5. Cho 3 s thc dng a,b,c tha mn iu kin


Tm gi tr nh nht ca biu thc P =

a
b
c
c2
.
+
+
+ 2
b + c c + a a + b a + b2

Li gii

t
=
a x.c,
=
b y.c, x,y 1 khi theo gi thit ta c:

x 1 + y =
1

2
xy ( ( x 1)( y 1) 1)=
0

xy =+
x y 2 xy =
> xy 4
Ta vit li biu thc P di dng: P =

x
y
1
1
.
+
+
+ 2
y +1 x +1 x + y x + y2

S dng bt ng thc C-S ta c:

P=

x2
y2
1
1
+
+
+
xy + x xy + y x + y ( x + y ) 2 2 xy
( x + y)2
1
1
+ + 2 2
2 xy + x + y xy x y 2 xy

x2 y 2 1
1
+ + 2 2
3 xy xy x y 2 xy
xy 1
1
=
+ + 2 2
3 xy x y 2 xy

t t = xy, t 4 P f(t) =

434

t 1
1
.
+ + 2
3 t t 2t

ab
.
c

www.TaiLieuLuyenThi.com
Cty TNHH MTV DVVH Khang Vit

Xt hm s f(t) =

f '(t)

t 1
1
trn 4; + ) ta c:
+ + 2
3 t t 2t

t 2 t 2 4t + 1 + 6 ( t 1)
6t 2 ( t 2 )

> 0, t 4 .
41
.
24

V vy f(t) ng bin vi t 4 P f(t)= f(4)=

41
t ti x= y= 2 .
24
V d 6. Cho a,b,c l cc s thc tha mn iu kin a b c > 0 .
Vy gi tr nh nht ca P bng

Tm gi tr nh nht ca biu
thc P
=

( 3ab + bc )

b4

121b 2
.
a 2 + b 2 + c 2 + 8ac

Li gii
t =
x

a
c
,y
=
, ( x 1 y > 0 ) khi :
b
b

121
x + y + 8xy + 1
121
= 9x 2 + 6xy + y 2 + 2
2
x + y + 8xy + 1
121
x 2 + 8xy + y 2 + 6 + 2
2
x + y + 8xy + 1

P = ( 3x + y ) +
2

t t =
x 2 + 8xy + y 2 + 1 P f(t) =
t+
Ta c: f '(t) = 1

121
+ 5.
t

121
; f '(t) = 0 t = 11 .
t2

Ta c f(t) i du t m sang dng khi i qua t = 11 nn f(t) t cc tiu ti


t = 11 hay P f(t) f(11) =
27 . ng thc xy ra khi v ch khi a= b= c .
Vy gi tr nh nht ca P bng 27 t ti a= b= c .
V d 7. Cho x,y,z l cc s thc dng tha mn iu kin

7 ( x 2 + y 2 + z 2=
) 11( xy + yz + zx ) .

Chng minh rng

51
x
y
z

+
+
2.
28 y + z z + x x + y
435

www.TaiLieuLuyenThi.com
Khm ph t duy K thut gii bt T Bi ton Max Min ng Thnh Nam

Li gii
t P =

x
y
z
khi
+
+
y+z z+x x+ y

x
y
z
+1+
+1+
+1
y+z
z+x
x+ y

P +=
3

1
x+ y+z x+ y+z x+ y+z
1
1
+
+
= ( x + y + z)
+
+

y+z
z+x
x+ y
x+ y y+z z+x

( x + y + z ) ( x + y )( x + z ) + ( y + z )( y + x ) + ( z + x )( z + y )
( x + y )( y + z )( z + x )
2
( x + y + z ) ( x + y + z ) + xy + yz + zx
=
( x + y )( y + z )( z + x )
=

Theo gi thit ta c: 7 ( x + y + z=
) 25 ( xy + yz + zx ) . Suy ra
2

P+3
Do

( x + y + z ) ( x + y + z )

7
2
+ ( x + y + z)
3
x + y + z)
(
32
25

=
25 ( x + y )( y + z )( z + x )
( x + y )( y + z )( z + x )
2

( x + y )( y + z )( z + x ) .
32 1
.
=
3
25 P + 3
( x + y + z)

1
a + b + c =
x
y
z

t a
=
=
=

,b
,c
7 .
x+ y+z
x+ y+z
x+ y+z
ab + bc + ca =
25
32 1
.
=( a + b )( b + c )( c + a ) =(1 a )(1 b )(1 c )
25 P + 3
.
=1 ( a + b + c ) + ab + bc + ca abc
= 1 1 +

7
7
abc =
abc
25
25

Vy tm gi tr ln nht v nh nht ca P ta tm gi tr ln nht v nh nht


ca Q = abc .
Ta c

7
7

3
2
Q
= a ab ac=
a

a
1

a
=
a

a
+
a
(
)
a a ( b + c )=

25
25
25

25

Ta c:
436

www.TaiLieuLuyenThi.com
Cty TNHH MTV DVVH Khang Vit

b + c =1 a
7
1
3
2
2

2
7 ; ( b + c ) 4bc (1 a ) 4 a a + a .

2
25 15
5

bc = a a 25
Xt hm s f (a ) = a 3 a 2 +

7
1 3
a trn on ; ta c:
25
15 5

a=

7
5
.
f '(a ) =3a 2 2a + ; f '(a ) =0
25
a = 7

15
1
15

49
3
7
3
1
.
f=
; f=

f=

15 3375 5
5 125
3
49
51
.
=
; Qmin =
Pmax = 2; Pmin =
125
3375
28

Ta c f =

Suy ra Qmax

3) Mt nh gi hay s dng
Vi iu kin cc s thc a,b,c khng m ta cc bt ng thc thng t im
ri ti mt bin bng 0 nn ta thng gi s c = min {a, b,c} v tm cch nh
gi a bt ng thc v hai bin.
Ta c cc c lng hay s dng:
2

c
c
c
c
a + c a + ; b2 + c2 b + ;a2 + b2 a + + b + ;
2
2
2
2

c
c
a + b + c a+ +b +
2
2

c
c
ab + bc + ca a + b +
2
2

c
a ac + c = a + c ( c a ) a a + ;
2

c
b bc + c = b + c ( c b ) b b + ;
2

c
c
c
c
a ab + b a + a + b + + b +
2
2
2
2

437

www.TaiLieuLuyenThi.com
Khm ph t duy K thut gii bt T Bi ton Max Min ng Thnh Nam

V d 1. Cho a,b,c l cc s thc khng m tha mn iu kin a + b + c =


3.

)(

)(

Tm gi tr ln nht ca biu thc P = a2 ab + b2 b2 bc + c2 c2 ca + a2 .

Li gii

Gi s c = min a, b,c ta c:

b2 bc + c2 = b2 + c ( c b ) b2 ;

a2 ac + c2 = a2 + c ( c a ) a2
Suy ra

P a2 b2 a2 ab + b2

36

t t =

a2 b2 a2 ab + b2

( a + b + c)

) a b (a
2

ab + b2

(a + b)

a
b
b +2+ a

a b
t 1
.
+ , ( t 2 ) P f(t) = 36.
3
b a
+
t
2
( )

Xt hm s f(t) = 36.

f '(t) = 36.

5 2t

( t + 2)

t 1

( t + 2)

vi t 2 ta c:

; f '(t) = 0 t =

5
.
2

Ta c f(t) i du t dng sang m khi i qua t =

t=

) =b 1 + a

5
nn f(t) t cc i ti
2

5
5
hay P f(t) f =
12 .
2
2

Vy gi tr ln nht ca P bng 12 t ti =
a 2,=
b 1,c
= 0 hoc cc hon v.
Cch 2: Thc hin nh gi nh trn v s dng AM-GM ta c:

3
ab + ab + a2 ab + b2

4 3 3
4
2
2
P a2 b2 a=
. ab. ab a2 ab + b2 2
ab + b2

9 2 2
9
3

4 ( a + b ) 4 ( a + b + c)
12
=.

=
9 27
9.27
6

438

www.TaiLieuLuyenThi.com
Cty TNHH MTV DVVH Khang Vit

ng thc xy ra khi v ch khi

=
= 0
a 2,=
b 1,c
3
a + b + c = 3,c = 0, ab = a2 ab + b2
2
b 2,c
=
= 0
a 1,=

Cch 3: Gi s c = min a, b,c khi ta c:


2

c
a ac + c = a + c ( c a ) a a + ;
2

c
b bc + c = b + c ( c b ) b b + ;
2

c
c
c
c
a ab + b a + a + b + + b +
2
2
2
2

t x = a +

c
c
,y = b + , ( x,y 0 ) x + y = a + b + c = 3 khi :
2
2

P x 2 y 2 x 2 xy + y 2 = x 2 ( 3 x ) x 2 x ( 3 x ) + ( 3 x )
2

=3x 6 27x 5 + 90x 4 135x3 + 81x 2 12


V d 2. Cho a,b,c l cc s thc khng m khng c hai s no ng thi bng 0
tha mn a + b + c =
2.
Tm gi tr nh nht ca biu thc P =

1
4

a2 + b 2

Li gii

1
4

b 2 + c2

1
4

a 2 + c2

Khng mt tnh tng qut gi s c = min a, b,c khi ta c:


2

c
c
a + b a+ +b + ;
2
2

c
b +c b+ ;
2

c
a + c a+
2

Khi P

1
a+

c
2

1
b+

c
2

c
c
a+ 2 +b + 2

439

www.TaiLieuLuyenThi.com
Khm ph t duy K thut gii bt T Bi ton Max Min ng Thnh Nam

t x = a +

c
c
,y = b + , ( x,y > 0 ) x + y = a + b + c = 2 .
2
2
2

x+y
Theo bt ng thc AM-GM ta c: xy
1 v
=
2

1
x2 + y2

+2

=
4 2
x + y2

2xy

1 1
+ 2 .
4
4 2
2 4 xy
x + y2
1

+2

= 2+

2
2
2xy + x 2 + y 2

Du bng xy ra khi v ch khi x = y = 1 a = b = 1,c = 0 .


8

2xy x 2 + y 2

xy

1
4

Vy gi tr nh nht ca P bng 2 +

1
4

t ti a= b= 1,c= 0 hoc cc hon v.

B. BI TP CHN LC
Bi 1. (TSH Khi A, A1 2013) Cho cc s thc dng a,b,c tha mn iu kin

4c
( a + c)( b + c) =

Tm gi tr nh nht ca biu thc P =

32a3

32b3

( b + 3c) ( a + 3c)
3

a2 + b 2
.
c

Li gii

t
=
a cx,
=
b cy, x,y > 0 t iu kin bi ton ta c:

( x + 1)( y + 1) = 4 xy + x + y = 3 .
Ta c: =
P

32x3

32y3

( y + 3) ( x + 3)
3

x2 + y2 .

Trc tin ta chng minh bt ng thc quen thuc sau y:


Vi mi a,b dng ta lun c a3 + b3

3
1
a + b) .
(
4

)(

) (

Tht vy bt ng thc tng ng vi: 4 a + b a2 ab + b2 a + b .

4 a2 ab + b2 a2 + 2ab + b2 3 ( a b ) 0 .
440

www.TaiLieuLuyenThi.com
Cty TNHH MTV DVVH Khang Vit

Bt ng thc lun ung v bi ton ph c chng minh.


p dng ta c:

32x3

( y + 3)

x 2 + y 2 + 3x + 3y
x
y
8
+
=
8

y+3 x+3
xy + 3x + 3y + 9

32y3

( x + 3)

( x + y )2 + 3 ( x + y ) 2xy

= 8

xy + 3 ( x + y ) + 9

Thay xy = 3 x y vo biu thc trn ta c:


3

32x

32y

( y + 3) ( x + 3)
3

( x + y )2 2 ( 3 x y ) + 3 ( x + y )
=
8

3 x y + 3(x + y) + 9

3
3

( x + y 1) .
3

Do

( x + y 1)

P ( x + y 1) x 2 + y 2 =
3

( x + y 1)

( x + y 1)

(x + y)

2 (3 x y )

(x + y)

+ 2 (x + y) 6

(x + y)

2xy

t t= x + y ta c P t 1 t 2 + 2t 6 v
2

x+y
t2

( t 2 )( t + 6 ) 0 t 2 .
3 x y= xy
3
t

4
2

Xt hm s f(t) = t 1 t 2 + 2t 6 vi t 2 ta c:

t +1

f '(t) = 3 ( t 1)
2

t 2 + 2t 6

3 2 ( t 1) t 1
2

t 2 + 2t 6

3 t 2 + 2t 6 ( t 1) t 1
2

t 2 + 2t 6

4 ( t 1) t 1 4t 2 9t + 3
>
=
> 0, t 2
t 2 + 2t 6
t 2 + 2t 6
2

Vy f(t) l hm ng bin trn 2; + do

f(t) f(2) = 1 2 P 1 2 .
Vy gi tr nh nht ca P bng 1 2 t ti a= b= c .
441

www.TaiLieuLuyenThi.com
Khm ph t duy K thut gii bt T Bi ton Max Min ng Thnh Nam

Bi 2. Cho a,b,c l cc s thc dng i mt phn bit tha mn b 8a 4c v

ab + bc =
2c2 . Tm gi tr ln nht v nh nht ca biu thc
P=

a
b
c
.
+
+
ab bc ca
Li gii

t=
a x.c,=
b y.c, 0 <

1
1
2
.
y x v xy + y = 2 y =
8
2
x +1

Khi

2
1
+ x +1 +
2
2
1 x
x
1
x +1 x +1
2
2
x + x 3 ( x + 2 ) x 2 2x 6
x +x
3
= 2
+ =
=
x + x 2 1 x
x2 + x 2
x2 + x 2
x
y
1
P=
+
+
=
x y y 1 1 x

Ta c

1 + 2 1
1
1
y x x
;
8
2
2
2

Xt hm s f(x) =

=
f '(x)

x 2 2x 6
lin tc trn
x2 + x 2

3x 2 + 8x + 10

(x

+x2

1 + 2 1
; ta c

2
2

1 + 2 1
> 0, x
;
2
2
1 + 2 1
;
2
2

suy ra f(x) l hm ng bin trn

1 + 2
1
17 + 6 2
27
.
P
f(x) f hay

7
5
2
2

Suy ra f

Vy gi tr ln nht ca P bng
ca P bng

442

27
8
t ti=
b 2a,c
= a v gi tr nh nht
3
5

1 + 2
17 + 6 2
t =
ti a
=
c, b 4
2
7

2 1 c .

www.TaiLieuLuyenThi.com
Cty TNHH MTV DVVH Khang Vit

Bi 3. Cho a,b,c l cc s thc dng tha mn iu kin a3 + b3 =


c3 .
Tm gi tr nh nht ca biu thc P =

a 2 + b 2 c2
.
( c a )( c b )

Li gii
Nhn xt. iu kin bi cho ng bc 3 v biu thc P bc 0 nn ta t

=
a xc,
=
b yc, ( x,y > 0 ) .

( x + y ) 2xy 1 .
xy ( x + y ) + 1
(x + y) 1 .
= ( x + y ) 3xy ( x + y ) xy =
3(x + y)
2

x2 + y2 1
Khi x +
=
y =
1 v P =
(1 x )(1 y )
3

Xut pht t 1 = x + y
3

t t= x + y ta c t > 1 v x + y
Ta c P =

4 +2

4 1

t3 1
4xy t 4.
1< t 3 4 .
3t
2

t+2
d thy y l hm nghch bin vi 1 < t 3 4 suy ra
t 1
.

Vy gi tr nh nht ca P bng

4 +2

4 1

t ti=
a b,c
= a3 2 .

Bi 4. Cho a,b,c l cc s thc dng tha mn iu kin a b c .

(a
Tm gi tr nh nht ca biu thc M =
Li gii

+ c2

ab + bc + ca

ac ( a + b + c )

)(

Nhn xt. iu kin cho a b c b a b c 0 v M bc 0 nn t

=
a x.b,c
= y.b, ( x 1,0 < y 1) v

(1 x )(1 y ) 0 xy x y + 1 0 x + y 1 + xy .

(x
Khi : M =

+ y2

x + y + xy

xy ( x + y + 1)

443

www.TaiLieuLuyenThi.com
Khm ph t duy K thut gii bt T Bi ton Max Min ng Thnh Nam

y l biu thc i xng vi tng x + y v tch xy nn suy ngh ngay n vic


t S =+
x y,P =
xy t iu kin ta c ngay 0 < P S 1 .

(S
Ta c: M =

2P

S+ P
.

P ( S + 1)

Coi v phi l hm s vi P v tham s S ta c:

f '(P) =

2S3 + S2 P + 2P 2

2 ( S + 1) P 2 S + P

< 0, S,P > 0

do f(P) l hm nghch bin trn 0;S 1 .

Do f(P) f S 1

(S
Xt hm s g(S) =

g'(S)

(S
=

2S + 2

2S + 2

2S 1
.

S2 1
2S 1

S2 1

S4 + 2S3 13S2 + 12S 4


=
2
2S 1 S2 1

trn 1; + ta c:

(S 2) (S

+ 4S2 5S + 2

2S 1 S2 1

);

S>1
= 0
=
g'(S)
S 2

Ta c g(S) i du t m sang dng khi i qua S = 2 nn g(S) t cc tiu ti

2
S = 2 trn (1; + ) hay g(S) g(2) =.
3
Vy gi tr nh nht ca M bng

2
3

t ti a= b= c .

Bnh lun. Ngoi li gii trn ta c th xt hm s trc tip bng cch coi b l n
v a,c l tham s ta c kt qu tng t hoc chng minh

( a + c) b + 2ac
. Khi M

ac ( a + b + c )
( a + b + c) 3 ( a + c)
b2 + 2ac

4ac

2
3

Nhng r rng vi du hiu ng cp t iu kin cho n biu thc M vic s


dng k thut gim v hai bin x,y t ra hiu qu. y l mt bi ton hay i
hi phi t duy logic khi gp tng v tch i xng ca S =+
x y,P =
xy .

Bi 5. Cho a,b,c l cc s thc dng thay i tha mn iu kin a + b + c =


32abc .
444

www.TaiLieuLuyenThi.com
Cty TNHH MTV DVVH Khang Vit

Tm gi tr ln nht v nh nht ca biu thc P =

a 4 + b 4 + c4

( a + b + c)

Li gii
Theo gi thit ta c:

a
b
c
1
.
.
.
=
a + b + c a + b + c a + b + c 32

1
a
b
c
=
,x + y +=
z 1.
=
,y =
,z
,x,y,z > 0 v xyz
32
a+ b+c
a+ b+c
a+ b+c

=
t x

Khi P = x 4 + y 4 + z 4 v ta c:

P=

(x

+ y2 + z2

2 x2 y2 + y2z2 + z2 x2

2
2
= ( x + y + z ) 2 ( xy + yz + zx ) 2 ( xy + yz + zx ) 2xyz ( x + y + z )

2
2

1
= 1 2 ( xy + yz + zx ) 2 ( xy + yz + zx )
16

2
9
= 2 ( xy + yz + zx ) 4 ( xy + yz + zx ) +
8

t t = xy + yz + zx ta c P = 2t 2 4t +

Ta c: t =x y + z + yz =x 1 x +

9
.
8

1
1
.
= x 2 + x +
32x
32x

Mt khc:

3 5
1
x

2
2
4
2.
( 2x 1) 4x 2 6x + 1 0 4
( y + z ) 4yz (1 x ) 32x
3+ 5
x

Xt hm s f(x) = x 2 + x +

1
trn
32x

3 5 1 3 + 5

;
; + ta c:

2 4
4

1
x=

1
4
.

f '(x) = 2 x + 1
; f '(x) = 0
2
32x

1+ 5
x =
8

3 5 1 3+ 5

x
;
;+

4 2 4

445

www.TaiLieuLuyenThi.com
Khm ph t duy K thut gii bt T Bi ton Max Min ng Thnh Nam

Ta c:

3 5
f
=
4

Suy ra

1+ 5 5 5 1 1
f
,f =
=
8
32
4

1 5 3+ 5
5 5 +1
.
f =
,f
=
32
2 16 4

5
5 5 1
9
. Xt hm s g(t) = 2t 2 4t + trn
t
16
32
8

5 5 5 1
;

32
16

5 5 5 1
;
.
32
16

Ta c: g'(t)= 4t 4 < 0, t

5 5 1
5
383 165 5
9
.
P

g(t)

hay ta c

32
256
128
16

Suy ra g

Vy gi tr ln nht ca P bng

9
t ti a= 2, b= c= 1 hoc cc hon v v
128

1+ 5
383 165 5
t ti a =
3 5, b =
c= .
2
256
Bi 6. Cho a,b,c l cc s thc tha mn iu kin a b c 0 v
2b + 2c a > 0 .
gi tr nh nht ca P bng

b
c
2b + 2c a
.
+

c+a
a + b
a

Tm gi tr nh nht ca biu thc P = 2

Li gii

b
2b a
.

a
a

TH1: Nu c = 0 P = 2
t t
=

b 1
,
t 1 khi P =
f(t) =
2t 2t 2 1 .
a 2

;1 ta c
2

Xt hm s f(t) =
2t 2t 2 1 lin tc trn

f '(t) = 2

2t
2t 2 1

2t 2 1 t
2t 2 1

)=

2 t2 1
2t 2 1

1
0, t
;1 .
2
2

2t 1 + t

;1 , suy ra P = f(t) f(1) = 1 .


2

Do f(t) l hm nghch bin trn


446

www.TaiLieuLuyenThi.com
Cty TNHH MTV DVVH Khang Vit

Trng hp ny gi tr nh nht ca P bng 1 t ti=


a b,c
= 0.

TH2: Nu c > 0 a b c > 0 t c = x.a, b = y.a, 0 < x,y 1 .

x
y
+
2x + 2y 1 .
y +1
x + 1

Khi :
P 2
=

x
y
+
x+y .
y +1
x +1

Ta chng minh

Tht vy bt ng thc tng ng vi:

x2 + x + y2 + y

( x + y )( x + 1)( y + 1)

x 2 + x + y 2 + y + 2 xy ( x + 1)( y + 1) ( x + y )( x + 1)( y + 1) .
2 xy ( x + 1)( y + 1) xy ( x + y + 2 ) 4 ( xy + x + y + 1) xy ( x + y + 2 ) .
2

Bt ng thc lun ng do xy x + y + 2

xy + x + y + 1 xy ( x + y + 2 ) =

4xy ( x + y + 2 ) v

( x + y + 1)(1 xy ) 0 .

Vy P 2 x + y 2x + 2y 1
t t=

x + y, 0 < t 2 suy ra P f(t) =2t 2t 2 1 .

Ta c: f '(t) = 2

2t

; f '(t) = 0 t = 1 .

2t 1
Do P f(1) =
1 . Nhng trng hp ny du bng khng xy ra.
2

Kt hp hai trng hp suy ra gi tr nh nht ca P bng 1 t ti=


a b,c
= 0.
Bi 7. (TSH Khi A 2009) Chng minh rng vi mi s thc dng x,y,z tha

)
( x + y ) + ( x + z ) + 3 ( x + y )( y + z )( z + x ) 5 ( y + z )

mn iu kin x x + y + z =
3yz ta lun c
3

Li gii

)
Ta cn chng minh ( a + 1) + ( b + 1) + 3 ( a + 1)( b + 1)( a + b ) 5 ( a + b ) .
( a + b + 2 ) 3 ( a + 1)( b + 1)( a + b + 2 ) + 3 ( a + 1)( b + 1)( a + b ) 5 ( a + b ) .
t
3ab .
=
y a.x,
=
z b.x, a, b > 0 ta c a + b + 1 =
3

447

www.TaiLieuLuyenThi.com
Khm ph t duy K thut gii bt T Bi ton Max Min ng Thnh Nam

( a + b + 2 ) 6 ( a + 1)( b + 1) 5 ( a + b )
3

( a + b + 2 ) 6 ( ab + a + b + 1) 5 ( a + b )
3

Thay a + b + 1 =
3ab vo bt ng thc ta cn chng minh

( 3ab + 1)

6 ( ab + 3ab ) 5 ( 3ab 1) ( 3ab 1)( ab 1)( 6ab 1) 0


3

(lun ng)
Do 3ab = a + b + 1 2 ab + 1

)(

ab 1 3 ab + 1 0 ab 1 .

Bt ng thc c chng minh. ng thc xy ra khi v ch khi x= y= z .


Bi tp tng t
1) Cho a,b l hai s thc v c khng m tha mn iu kin a2 + b2 + ab =
3c2 .
Chng minh rng a3 + b3 + 4abc 6c3 .

Cho a,b,c l cc s thc dng tho mn iu kin a 2 + 2a ( b + c ) =


5bc .
Chng minh rng ( a + b ) + ( a + c ) + ( a + b )( a + c )( b + c ) 3 ( b + c ) .
3

Bi 8. Cho a,b,c l cc s thc dng tha mn a + b + c =


14 .
6 v a 2 + b 2 + c 2 =
Tm gi tr ln nht v nh nht ca biu thc P =
Li gii

4a + b
.
c

6
c ( x + y + 1) =
.
2
2
2
c
x
+
y
+
1
=
14
(
)

t
=
a x=
.c, b y.c, ( x, y > 0 ) theo gi thit ta c:

6c

x+ y =

6 c
( x + y ) c =
c

2
2
2
2
( 6 c ) (14 c 2 ) c 2 6c + 11
14 c
( x + y ) c 2 xyc =

=
xy =
c2
2c 2
Trc ht ta phi c:

( x + y)

c 2 6c + 11
62 3
6+2 3
6c
.
4 xy

c
4.
2
c
3
3
c
2

Suy ra x,y l hai nghim dng ca phng trnh:

6 c 3c 2 + 12c 8
t1 =
6 c c 6c + 11
2c
0
.
t2
t+
=
2

c
c
6 c + 3c 2 + 12c 8
t 2 =
2c

448

www.TaiLieuLuyenThi.com
Cty TNHH MTV DVVH Khang Vit

Ta c: =
P 4x + y .
Tm gi tr ln nht ca P.
tm gi tr ln nht ca P ta xt vi=
x t2=
, y t1 v t2 t1 .
Khi

6 c + 3c 2 + 12c 8 6 c 3c 2 + 12c 8 30 5c + 3 3c 2 + 12c 8


+
=
.
2c
2c
2c
6 2 3 6 + 2 3
30 5c + 3 3c 2 + 12c 8
lin tc trn on
Xt hm s f (c) =
;
.
2c
3
3
P= 4.

f '(c) =

3 5 3c 2 + 12c 8 + 3c 4

) ; f '(c) =0 5

c 3c + 12c 8
4 3c 0
6

2 c =.
2
7
25 ( 3c + 12c 8 ) = ( 4 3c )
2

3c 2 + 12c 8 + 3c 4 =0

Ta c f(c) i du t dng sang m khi i qua c =

6
nn f(c) t cc i ti
7

6
6 31
hay P f (c) f =
.
7
7 2
19
17
6
ng thc xy ra khi v ch khi
.
=
a =
,b =
,c
7
7
7
31
19
17
6
Vy gi tr ln nht ca P bng
t ti
.
a =
,b =
,c
=
2
7
7
7
c=

Tm gi tr nh nht ca P.
tm gi tr nh nht ca P ta xt vi=
x t1=
, y t2 .
Khi

P= 4.

6 c 3c 2 + 12c 8 6 c + 3c 2 + 12c 8 30 5c 3 3c 2 + 12c 8


+
=
.
2c
2c
2c

30 5c 3 3c 2 + 12c 8
lin tc trn on
2c
6 2 3 6 + 2 3
;

.
3
3

Xt hm s g(c) =

g'(c) =

3 5 3c 2 + 12c 8 3c + 4
c

3c + 12c 8
2

) ;g'(c) =0 5

3c 2 + 12c 8 3c + 4 =0 .
449

www.TaiLieuLuyenThi.com
Khm ph t duy K thut gii bt T Bi ton Max Min ng Thnh Nam

3c 4 0

2
25 ( 3c + 12c 8 ) =

( 3c 4 )

c=
3.

Ta c g(c) i du t m sang dng khi i qua c = 3 nn g(c) t cc tiu ti


c = 3 hay P g (c) g (3) =
2.
ng thc xy ra khi v ch khi =
a 1,=
b 2,=
c 3.
Vy gi tr nh nht ca P bng 2 t ti =
a 1,=
b 2,=
c 3.
Bi 9. Cho a,b,c l cc s thc dng tha mn iu kin

( 3a + 2b + c )

1 2 3
+ + =
30 .
a b c

Tm gi tr ln nht ca biu thc P =

b + 2c 7 72a 2 + c 2
.
a

Li gii
t
=
x

b
c
. Suy ra: 30 =
=
,y
2a
3a

1 1
+ + 1
x y

( 4 x + 3 y + 3)

1 1
1 1 x ( y + 1)
= x + + 1 + 3 ( x + y + 1) + + 1
+ 28
2y
x y
x y
2y
4y
x
P = 2 x + 6 y 21 y 2 + 8
+ 6 y 21 y 2 + 8 55
y +1
y +1
ng thc xy ra khi v ch khi 6=
a 3=
b 2c .
Cch 2: Trc tin ta bin i iu kin bi ton

( 3a + 2b + c )

1
1 1
5
+ + =
6a 3b 2c

1 1 1
1 1
1
10
( 6a + 3b + 2c ) + + + b + + =
6a 3b 2c 6a 3b 2c
1 1
1 2
4ac
1
1
b + + 1 b + b
3a + c
6a 3b 2c
6 a 2c 3
4ac
2
+ 2c 7 72a 2 + c 2
4c
2c
c
3
+
a
c
P
=
+ 7 72 +
3a + c a
a
a
4t
Xt hm s f (t ) =
+ 2t 7 t 2 + 72 55, ( t = c / a ) ta c
3+t

450

www.TaiLieuLuyenThi.com
Cty TNHH MTV DVVH Khang Vit

f '(t ) =
2+

4
4t
7t
.

2
t + 3 ( t + 3)
t 2 + 72

S: Pmax =
55; 2a =
3b =
6c .
Di y ti trnh by mt bi ton tng t nhng i theo mt hng khc
Bi 10. Cho a,b,c l cc s thc dng tho mn iu kin

( a + 5b + 3c )

1 1
1 39
+ + =.
a 4b 3c 4

Tm gi tr nh nht ca biu thc

2 ( a + 3c )( a + 3c 4b ) 7c 2 + 4c a 2 + 7c 2
.
P=
2ac
Li gii
Theo gi thit ta c

39
1 1 1
1
1
1 1
1 1
=( a + 4b + 3c ) + + + b + + 9 + b + +
4
a 4b 3c a 4b 3c
a 4b 3c

3ac 2b ( a + 3c )

1
1 1
+
+ 9.
a 4b 3c

Ch AM GM ta c ( a + 4b + 3c )
Khi

2 ( a + 3c ) 8b ( a + 3c ) 7c 2 + 4c a 2 + 7c 2
P=
2ac
2

2 ( a + 3c ) 12ac 7c 2 + 4c a 2 + 7c 2 2a 2 + 11c 2 + 4c a 2 + 7c 2

=
.
2ac
2ac
2

a
2 x 2 + 11 + 4 x 2 + 7
=
,x
c
2x
Xt hm s f ( x) =

( 2x
f '( x) =

2 x 2 + 11 + 4 x 2 + 7
vi x dng ta c
2x

11) x 2 + 7 28
2 x2 x2 + 7

; f '( x) = 0 x = 3 .
15
2

Ta c f(x) i du t m sang dng khi i qua x=3 nn f ( x) f (3) =


.
Du bng xy ra khi v ch khi =
a 4=
b 3c .
15
Vy gi tr nh nht ca P bng
.
2

451

www.TaiLieuLuyenThi.com
Khm ph t duy K thut gii bt T Bi ton Max Min ng Thnh Nam

Bi tp tng t
Cho a,b,c l cc s thc dng tho mn iu kin

( a + 3b + c )

1 1 1 21
+
+ =.
a 2b c 2

Tm gi tr nh nht ca biu thc

a 2 + ( a + c )( a + c 2b ) + 2a 2 + 2c 2
.
P=
ac
S: Pmin
= 5;=
a 2=
b c.
C. BI TP RN LUYN
Bi 1. Cho x,y l hai s thc dng. Tm gi tr ln nht ca biu thc

P=

(x +

4xy 2
x + 4y
2

Bi 2. Cho x,y l hai s khng m . Tm gi tr ln nht ca biu thc

xy + x 4 + 9x 2 y 2
.
P=
x 2 + 8y 2
Bi 3. Cho x,y l hai s thc dng tha mn xy y 1 .

y3 2x 2
.
Tm gi tr nh nht ca biu thc =
P
+
x3 y 2
Bi 4. Cho a,b,c l cc s thc dng thay i tha mn

( a + 2b ) 1b + 1c =

2
2
a + 2b
.
P=
ac

4,c 3a . Tm gi tr ln nht v gi tr nh nht ca biu thc

Bi 5. Cho x,y l cc s thc dng. Tm gi tr nh nht ca biu thc

=
P

x3
4y3
+
.
3
x3 + 8y3
y3 + ( x + y )

Bi 6. Cho a,b,c l cc s thc dng thay i tha mn

( a + b + c) 1a + 1b + 1c =16 .

Tm gi tr ln nht v nh nht ca biu thc P =


452

a2 + 2b2
.
ab

www.TaiLieuLuyenThi.com
Cty TNHH MTV DVVH Khang Vit

Bi 7. (TSH Khi A 2011) Cho cc s thc x, y, z [1;4] , x y, z z .


x
y
z
.
+
+
2x + 3y y + z z + x

Tm gi tr nh nht ca biu thc P=

Bi 8. Cho a,b,c l cc s thc dng. Tm gi tr nh nht ca biu thc


bc
4abc
.
=
P

( b + a )( c + a ) ( a + b )( b + c )( c + a )
Bi 9. Cho cc s thc dng tha mn iu kin ( a + c )( b + c ) =
4c 2 .
a
b
ab
.
+
+
b + 3c a + 3c bc + ca
Bi 10. Cho a,b,c l cc s thc dng tha mn a 2b .

Tm gi tr ln nht ca biu thc P =

Chng minh: 14 a 2 + b 2 + c 2 5 ( a + b + c ) .
2

Bi 11. Cho a,b l hai s thc v c khng m tha mn a 2 + b 2 + ab =


3c 2 .
Chng minh rng a3 + b3 + 4abc 6c3 .
Bi 12. Cho a,b,c l cc s thc dng thay i tha mn ( a + b c ) =
ab .
2

ab
c2
c

Tm gi tr nh nht ca biu thc P =


+ 2
+
.
2
a+b a +b a+bc
Bi 13. Cho cc s thc khng m a,b,c tha mn a + b + c > 0 .

Tm gi tr ln nht v nh nht ca biu thc P =

a3 + b3 + 16c3

( a + b + c )3

Bi 14. Cho cc s thc a,b,c.


Chng minh rng 8a 4 + 8b 4 + 27c 4

27
( a + b + c )4 .
64

Bi 15. Cho a,b,c l cc s thc tha mn ( a + b + c )= 2 a 2 + b2 + c 2 v a + b + c 0 .


2

Tm gi tr ln nht v nh nht ca biu thc P =

a 3 + b3 + c 3
.
( a + b + c )( ab + bc + ca )

Bi 16. Cho x,y,z l cc s thc khng m i mt phn bit. Chng minh rng
x+ y
y+z
z+x
9
.
+
+

2
2
2
x
+
y+z
( x y) ( y z ) ( z x)
Bi 17. Cho a,b,c l cc s thc khng m i mt phn bit tha mn a + b + c =
1.
453

www.TaiLieuLuyenThi.com
Khm ph t duy K thut gii bt T Bi ton Max Min ng Thnh Nam

Tm gi tr nh nht ca biu thc P =

(a b)

(b c )

( c a )2

Bi 18. Cho a,b,c l cc s thc khng m khng c hai s no ng thi bng 0.


Tm gi tr nh nht ca biu thc

.
b +c
c +a
a +b
Bi 19. Cho a,b,c l cc s thc khng m v khng c 2 s no ng thi bng 0.
Tm gi tr nh nht ca biu thc
P=

( a + b + c )2

.
b +c
c +a
a +b
Bi 20. Cho a,b,c l cc s thc khng m khng c 2 s no ng thi bng 0 tha
mn a + b + c =
1.
1
1
1
.
Tm gi tr nh nht ca biu thc P = 2
+ 2
+ 2
2
2
a +b
b +c
c + a2
Bi 21. Cho cc s thc a > 0, b,c 0 . Tm gi tr nh nht ca biu thc
P=

( ab + bc + ca )

.
a +b
a +b +c
a
Bi 22. Cho a,b,c l cc s thc khng m v c < a, c < b . Chng minh rng
P=

( ab + bc + ca )

a 2 + b2 + c2
a b
.
+

ab + bc + ca
bc ca

Bi 23. Cho x,y,z l cc s thc thuc on [1;2] .


Tm gi tr ln nht v nh nht ca biu thc P =

x2 y 2 + y 2 z 2 + z 2 x2
.
xyz ( x + y + z )

Bi 24. Cho a,b,c l cc s thc dng.


Tm gi tr nh nht ca biu thc P =

4a3 + 3b3 + 2c3 3b 2 c

( a + b + c )3

Bi 25. Cho a,b,c l di mt tam gic khng nhn. Chng minh rng

(a

1
1
1
+ b 2 + c 2 2 + 2 + 2 10 .
b
c
a

Bi 26. Cho x,y,z l cc s thc dng tha mn x 2 + y 2 + 6 z 2= 4 z ( x + y ) .


Tm gi tr nh nht ca biu thc P =
454

x3
y( x + z)

y3
x( y + z)

x2 + y 2
.
z

www.TaiLieuLuyenThi.com
Cty TNHH MTV DVVH Khang Vit

Bi 27. Cho a,b,c l cc s thc tha mn ( a + b + c )= 4 ( ab + bc + ca ) v a + b + c 0 .


2

Tm gi tr ln nht v nh nht ca biu thc P =

a 2b + b 2 c + c 2 a

( a + b + c )3

Bi 28. Cho a, b 0, c > 0 v tho mn iu kin a3 + b3 = c ( c 1) .


Tm gi tr ln nht v nh nht ca biu thc P =

a 2 + b2 + c2

( a + b + c )2

Bi 29. Cho x,y l cc s thc dng thay i tha mn x + 2 xy 8 y .


x3 y 3 x 2 y 2
Tm gi tr nh nht ca biu thc P = 4 3 + 3 9 2 + 2 .
y
x y
x

Bi 30. Cho a,b,c l cc s thc dng tha mn 0 < 4a c 9a .


12a
12b
25c
.
Tm gi tr nh nht ca biu thc P =
+
+
a+b b+c c+a
4
3
Bi 31. Cho a,b,c l cc s thc dng tha mn b a c b .
5
5

Tm gi tr ln nht ca biu thc P =

12 ( a b ) 12 ( b c ) 25 ( c a )
.
+
+
c
a
b

Bi 32. Cho a,b,c l cc s thc khng m thay i tha mn 2a3 ( b + c ) 4abc .


3

Chng minh

7 4
a + 3 b 4 + c 4 2a 2b 2 3b 2 c 2 + 2c 2 a 2 .
16

Bi 33. Cho a,b,c l cc s thc dng thay i tha mn ab + bc + ca =


3b 2 .
Tm gi tr ln nht v nh nht ca biu thc

abc ( a + c ) a 2 + c 2
a
c
.
P=
+
+
b+c a+b
( a + c ) b2

Bi 34. Cho a,b,c l cc s thc dng thay i tha mn

( a + c )

1 10
, c 4b .
=
b2 b
a
a +cb
Tm gi tr ln nht v nh nht ca biu thc P =
.
b
Bi 35. Cho a,b,c l cc s thc khng m. Chng minh rng

(a

+ b2 + c2

27 ( a b ) ( b c ) ( c a ) .
2

Bi 36. Cho a,b,c l cc s thc khng m tha mn ab + bc + ca > 0 .


455

www.TaiLieuLuyenThi.com
Khm ph t duy K thut gii bt T Bi ton Max Min ng Thnh Nam

Chng minh rng

1
a +b
2

1
b +c
2

1
c +a
2

11
a +b +c
2

32

( a + b + c )2

Bi 37. Cho a,b,c l cc s thc khng m i mt khc nhau tha mn a + b + c =


0.

33
.
( a b )4 ( b c )4 ( c a )4 16
Bi 38. Tm hng s k tt nht sao cho bt ng thc sau ng vi mi s thc
khng m a,b,c

Chng minh rng

( ab + bc + ca )2

( a + b + c )5 k ( a 2 + b2 + c 2 ) ( a b )( b c )( c a ) .

Bi 39. Cho a,b,c l cc s thc i mt phn bit. Chng minh rng


2

a b bc ca

+
+
5.
bc ca a b
Bi 40. Cho a,b,c l cc s thc dng. Chng minh rng
1
a + ab + b
2

1
b + bc + c
2

1
c + ca + a
2

( a + b + c )2

Bi 41. (Iran 1996) Cho a,b,c l cc s thc khng m. Chng minh rng
1
1
1
9
.
+
+

2
2
2
4
ab
+
bc + ca )
(
( a + b) (b + c ) (c + a )
Bi 42. Cho a,b,c l cc s thc dng tha mn

1
2

1
=2 .
b
2c
c
2

a
a
b
.
Tm gi nh nht ca biu thc P =
+
+
b+c c+a
a 2 + b2 + c2
Bi 43. Cho a,b,c l cc s thc dng. Chng minh rng

27 ( a + b + c ) a 2 + b 2 + c 2 + 45abc 32 ( a + b + c )( ab + bc + ca ) .

Bi 44. Cho a,b,c l cc s thc dng. Tm gi tr ln nht ca biu thc


ab
bc
ca
4abc
.
P=
+
+

2
2
2
( a + b ) ( b + c ) ( c + a ) ( a + b )( b + c )( c + a )
Bi 45. Cho x,y,z l cc s thc khng ng thi bng 0 tha mn

2 x 2 + y 2 + z 2 = 3 ( xy + yz + zx ) .

Tm gi tr nh nht v ln nht ca biu thc P =

456

( y 2 z )2
xy + yz + zx

www.TaiLieuLuyenThi.com
Cty TNHH MTV DVVH Khang Vit

Bi 46. Cho a,b,c l cc s thc tha mn a 2 + b 2 + c 2 + 4ab + 4bc + 4ca =


0 v
a + b + c 0 . Tm gi tr ln nht v gi tr nh nht ca biu thc
P=

a3 + b3 + c3 abc

( a + b + c )3

Bi 47. Cho a,b,c l cc s thc tha mn a 2 + b 2 + c 2= 4 ( ab + bc + ca ) v


ab + bc + ca 0 . Tm gi tr ln nht v gi tr nh nht ca biu thc

P=

( a b )( b c )( c a ) ( a 2 + b2 + c 2 ab bc ca )
( a + b + c )5

Bi 48. Cho x,y,z l cc s thc dng tha mn 5 x 2 + y 2 + z 2 = 6 ( xy + yz + zx ) .


Tm gi tr ln nht v gi tr nh nht ca biu thc
1 1 1
P = ( x + y + z) + + .
x y z
Bi 49. Cho x,y,z l cc s thc khng ng thi bng 0 v tha mn

8 x 2 + y 2 + z 2= 11( xy + yz + zx ) .

Tm gi tr ln nht, gi tr nh nht ca biu thc P =

x3 + y 3 + z 3

( x + y + z ) ( x2 + y 2 + z 2 )

Bi 50. Cho a,b,c l cc s thc tha mn a 2 + b 2 + c 2 > 0 .


Tm gi tr ln nht v gi tr nh nht ca biu thc
P=

2 ( a + b + c ) a 2 + b 2 + c 2 9abc

(a

+b +c
2

2 3

C. HNG DN GII P S
Bi 1. t
=
x t. y, ( t > 0 ) ta c=
P f=
(t )

f '(t ) =

t 2 + 4 3t

t2 + 4 t + t2 + 4

(t +

4t
t +4
2

ta c

; f '(t ) = 0 t 2 + 4 = 3t t =

1
.
2

457

www.TaiLieuLuyenThi.com
Khm ph t duy K thut gii bt T Bi ton Max Min ng Thnh Nam

Ta c f(t) i du t m sang dng khi i qua t =


ti t =

1
trn khong ( 0;+ ) nn
2

1
1 1
f(t) t cc tiu hay P =
f (t ) f
=.
2
2 8

1
t ti y = 2 x .
8
Bi 2. Nu x =0 P =0 . Xt x > 0 ta t=
y t .x ( t 0 ) .

Vy gi tr nh nht ca P bng

Khi =
P f=
(t )
Xt hm s f (t ) =

t + t 4 + 9t 2
t2 + 8

t + t 4 + 9t 2

trn [ 0;+ ) ta c

t2 + 8

7t 2 + 72 + 8 t 2 t 2 + 9
=
f '(t ) =
; f '(t ) 0
2
4
2 2
t + 9t t + 8

7t 2 + 72 + 8 t 2

( )
) t +9 = 0t = 6
2

3 2
Lp bng bin thin suy ra P =
f (t ) f (6 2) = .
4

Vy gi tr ln nht ca P bng

3 2
t ti x = 6 2 y .
4
2

Bi 3. Theo gi thit ta c:
Khi P= f (t=
) t3 +

2
t

x y 1 1 1 1
1
y
2 = t = 4 .
y
4 y 2
4
x
y

. Xt hm s f (t=
) t3 +

2
t2

vi t 4 ta c

4 3t 5 4
f '(t=
) 3t 2 3=
> 0, t 4 do f(t) l hm ng bin trn
t
t3
513
[ 4;+ ) suy ra P = f (t ) f (4) = .
8
513
1
Vy gi tr nh nht ca P bng
t ti=
x =
,y 2.
8
2

Bi 4. t =
a x.b,=
c y.b, ( 3 x y > 0 ) khi t iu kin ta c:

( x + 2 ) 1 +

458

2 ( y 1)
1
.
=4 x =
y
y +1

www.TaiLieuLuyenThi.com
Cty TNHH MTV DVVH Khang Vit

Mt khc: 3 x y 3.

2 ( y 1)
y +1

y y2 5 y + 6 0 2 y 3 .

2 ( y 1)

+2
2
x + 2 y +1
3y2 2 y + 2
.
Khi
=
=
= f=
P
( y)
2 ( y 1)
xy
y3 y
.y
y +1
2

3y2 2 y + 2

Xt hm s f ( y ) =

y3 y

lin tc trn on [ 2;3] ta c:

3
3
3 y 4 + 4 y 3 + 3 3 y ( y 2 ) 2 y + 3
=
f '(y)
=
< 0, y [ 2;3] .
2
2
y3 y
y3 y

Do f(y) l hm nghch bin trn [ 2;3] .


11
2
t ti
=
a =
b, c 2b
6
3
a b=
, c 3b .
P=
f=
(3) 1 t ti=
min

Suy ra P=
f=
(2)
max

1 1
Bi 6. t
=
c x=
.a, b y.a, ( x, y > 0 ) theo gi thit ta c: (1 + x + y ) 1 + + =
16 .
x y

Khi
=
P f=
( y)

1 + 2 y2
.
y

kho st hm f(y) ta cn tm min gi tr ca y trc tin.


1

1
Vit li ng thc trn di dng: 1 + x 2 + y + 13 x + y + 1 =
0 (1) .
y
y

Phng trnh (1) phi c nghim x,y dng do iu kin l:

y > 0

1
1
73 5
7+3 5

=
y
+

13
y

4 1 + ( y + 1) 0
y
y
2
2

1
y + 13 > 0
y

7 3 5 7 + 3 5
Vic cn li ta ch kho st hm s f(y) trn
;
ta c:
2
2
7 3 5 21 3 5
1
=
=
Pmax f =
, Pmin f=

2 2.
2
2
2

459

www.TaiLieuLuyenThi.com
Khm ph t duy K thut gii bt T Bi ton Max Min ng Thnh Nam

Bi 7. Nhn xt. Biu thc P ng cp bc 0 v c x y, x z nn ta t n ph


gim bin bng cch biu din y theo x v z theo x.
1
t
=
y a=
.x, z b.x do x, y, z [1;4] , x y, x z a, b ;1 .
4
1
a
b
Khi P =
.
+
+
2 + 3a a + b b + 1
Ta cha c mi lin h gia a v b nn c mnh dn xt hm s vi a hoc b
xem sao v hy vng c mt hm lun ng bin hoc lun ngch bin.

Xt hm s f (a) =

1
a
b
1
lin tc trn ;1 ta c:
+
+
2 + 3a a + b b + 1
4

( 2 + 3a ) b 3 ( a + b ) .
3
b
f '(a ) =

+
= 2
2
2
( 2 + 3a ) ( a + b )
( 2 + 3a ) ( a + b )2
2

Ta c:

( 2 + 3a )2 b 3 ( a + b )2=

9a 2b + 6ab + 4b 3a 2 3b 2

9a 2b + 6a 2b + 4b 3a 2 3b 2

1
= 3a 2 ( 5b 1) + b ( 4 2b ) > 0, a, b ;1
4
1
Do f(a) l hm ng bin trn ;1 suy ra
4
1
b
1 4
.
P=
f (a ) f =+
+
4 11 1 + 4b 1 + b

4
1
b
1
lin tc trn on ;1 ta c:
Xt hm s g (b) =+
+
11 1 + 4b 1 + b
4
g '(b)=

( b + 1)

(1 + 4b )

Ta c g(b) i du t m sang dng khi i qua b =


b=

1
b ;1
4

b=
; g '(b)= 0 (1 + 4b ) = 4 ( b + 1)
2

1
.
2

1
nn g(b) t cc tiu ti
2

1
34
1 34
hay g (b) g =
. Do P .
2
33
2 33

34
1
1
t ti=
hay
=
x 4=
y, x 2 z v do
a =
,b
33
4
2
x, y, z [1;4] x = 4, y = 1, z = 2 .

Vy gi tr nh nht ca P bng

Bi 8. t
=
b x=
.a, c y.a, ( x, y > 0 ) khi :
460

www.TaiLieuLuyenThi.com
Cty TNHH MTV DVVH Khang Vit

xy
xy
4 xy
4
.
P=

=
1
( x + 1)( y + 1) ( x + y )( x + 1)( y + 1) ( x + 1)( y + 1) x + y

Vi x + y > 4 P > 0 ta xt vi x + y 4 .
Khi P
=

xy
4
1
khi nu coi v phi lm mt hm ca xy
xy + x + y + 1 x + y
2

x+ y
ta c mt hm nghch bin mt khc xy
suy ra:
2
2

x+ y

( x + y )( x + y 4 ) .
4
2
P
1
=2
2
x + y ( x + y) + 4( x + y) + 4
x+ y

+ x + y +1
2

t t = x + y, ( 0 < t 4 ) P f (t ) =
Xt hm s f (t ) =

f '(t ) =

t (t 4)

( t + 2 )2

t (t 4)

( t + 2 )2

trn ( 0;4] ta c:

( 2t 4 )( t + 2 ) 2 ( t 2 4t )
(t + 2)

8t 8

( t + 2 )3

; f '(t ) = 0 t = 1 .

Ta c f(t) i du t m sang dng khi i qua t = 1 nn f(t) t cc tiu ti


1
t = 1 do P f (t ) f(1) =
.
3
1
1
Vy gi tr nh nht ca P bng t ti x = y = a = 2b = 2c .
3
2
Bi 9. t
=
a x=
.c, b y.c, ( x, y > 0 ) theo iu kin bi ton ta c:
4.
( x + 1)( y + 1) =
x2 + y 2 + 3( x + y )
x
y
xy
xy
Khi : P =
+
+
=
+
y+3 x+3 x+ y
xy + 3 ( x + y ) + 9 x + y
( x + y )2 + 3 ( x + y ) 2 xy xy
+
xy + 3 ( x + y ) + 9
x+ y
Do ( x + 1)( y + 1) =4 xy =3 ( x + y ) .

461

www.TaiLieuLuyenThi.com
Khm ph t duy K thut gii bt T Bi ton Max Min ng Thnh Nam

t t = x + y, ( 0 < t < 3) xy = 3 t v
2

t2
x+ y
t >0
3=
t xy
=
t 2 + 4t 12 0
t 2 .

4
2
t 2 + 3t 2 ( 3 t )

3t t 3 3
= + .
3 t + 3t + 9
t
2 t 2
Bng
bin thin:
t 3 3
Xt hm s f (t ) = + vi
t
2 t 2
2
ta
c
t [ 2;3)
f(t)

Khi P =

1 3
; f '(t ) = 0 t = 6 .
2 t2
Da vo bng bin thin suy ra
max f =
(t ) f(2)
= 1.
t[ 2;+3)
f '(t ) =

f(t)

6
+

0
6

3
2

Vy gi tr ln nht ca P bng 1 t ti a= b= c .
Bi 10. t a= x.c, b= y.c, ( 0 < 2 y x ) ta cn chng minh

14 x 2 + y 2 + 1 5 ( x + y + 1) 9 y 2 10 ( x + 1) y + 9 x 2 10 x + 9 0 .
2

Coi v tri l hm ca y v x l tham s


f ( y ) = 9 y 2 10 ( x + 1) y + 9 x 2 10 x + 9 ta c
f '( y ) =
18 y 10 ( x + 1) 9 x 10 ( x + 1) =
x 10 < 0 nn f(y) l hm nghch
2

x 5

bin suy ra f ( y ) f = x 3 0 .
2
2

Bt ng thc c chng minh.


ng thc xy ra khi v ch khi
=
a 2=
b,5a 6c .
2

b 3b 2

Bi 11. Nu c = 0 a 2 + ab + b 2 = 0 a + +
= 0 a = b = 0 bt ng
2
4

thc tr thnh ng thc.


Nu c > 0 t
=
a x=
.c, b y.c khi theo iu kin ta c:
x 2 + y 2 + xy =
3 v bt ng thc cn chng minh tr thnh:

x3 + y 3 + 4 xy 6 ( x + y ) 3 xy ( x + y ) + 4 xy 6 .
3

t = x + y 3 = x 2 + y 2 + xy =

462

( x + y )2 xy =

t 2 xy t 2 3 = xy

t2
2 t 2 .
4

www.TaiLieuLuyenThi.com
Cty TNHH MTV DVVH Khang Vit

Thay xy= t 2 3 vo bt ng thc trn ta cn chng minh:

)
9t + 18 0, t [ 2;2] ( t 2 ) ( 2t

t 3 3 t 2 3 t + 4 t 2 3 6, t [ 2;2] .

2t 3 4t 2

9 0, t [ 2;2] (lun ng).

Bt ng thc c chng minh. ng thc xy ra khi v ch khi a= b= c .

xy
1
1
2
Bi 12. t
xy v P = + 2 2 + .
=
a x=
.c, b y.c t iu kin ta c: ( x + y 1) =
x + y x + y xy
2

2
2
2
x+ y

Ta c: ( x + y 1) =xy
( x + y ) 4 ( x + y ) 2 ( x + y ) + 1 0 .
2

2
2
3( x + y ) 8 ( x + y ) + 4 0 x + y 2 .
3
2
2

t t = x + y, t 2 thay xy= ( t 1) vo biu thc ca P ta c:


3

( t 1)2

P=

t
=

( t 1)2
t

1
t 2 ( t 1)
2

1
2 ( t 1)

( t 1)

1
t + 4t 2
2

( t 1)2

t
+

t + 4t 2
2

( t 1)2

1
2 ( t 1)

2
4
2
+ 2, t ;2
t t 1 t2
3

2
4
+
=
t t 1 t 2 + 4t 2 + 2 ( t 1)2

Vy gi tr nh nht ca P bng 2 t ti a= b= c= 1 .
Nhn xt. Ta c th x l theo cch khc sau y:
ab
c2
c2
ab
c2
c2
c2
+ 2
+
=
+
+
+
P=
a + b a + b 2 ab a + b 2ab a 2 + b 2 2ab
4c 2
2c 2c
c 2 ab
2
+ 2

2
ab ( a + b ) a + b + 2ab a + b a + b
2

Xut pht t gi thit ta c: 1 =

a+b

ab

(a + b)

1
1
c
3

.
4
2 a+b 2

c
1 3
. Xt hm s g (t=
) 2t + 4t 2 trn ; ta c kt qu tng t nh
a+b
2 2
li gii trn. Qua bi ton ny ta c nhn xt l i khi s dng k thut gim
bin i vi dng ng bc i hi chng ta phi bin i v vn dng tinh t
cc bt ng thc.

t t =

463

www.TaiLieuLuyenThi.com
Khm ph t duy K thut gii bt T Bi ton Max Min ng Thnh Nam
3

a
b
c

Bi 13. Ta c: P =
+
+ 16
.
a+b+c a+b+c
a+b+c
a
b
c
t x =
=
,y =
,z
, ( x, y, z 0 ) v x + y + z =
1.
a+b+c
a+b+c
a+b+c

Khi P = x3 + y 3 + 16 z 3 .
tm gi tr ln nht ta thc hin nh sau:
P =+
x3 y 3 + z 3 + 15 z 3 ( x + y + z ) + 15 z 3 =
1 + 15 z 3 16 .
3

Vy gi tr ln nht ca P bng 16 t ti a= b= 0 .
tm gi tr nh nht ca P ta thc hin nh sau:
2
P = x3 + y 3 + 16 z 3 = ( x + y ) ( x + y ) 3xy + 16 z 3

3
1
2
2

( x + y ) ( x + y ) ( x + y ) + 16 z 3 =
(1 z )3 + 16 z 3
4
4

1
3
Xt hm s f ( z ) = (1 z ) + 16 z 3 lin tc trn on [ 0;1] ta c:
4
3
1
z[ 0;1]
2
.
f '( z )= (1 z ) + 48 z 2 ; f '( z )= 0
z=
4
9

Ta c
=
f (0)

1 1 16
1 16
.
=
,f =
, f (1) 16 . Suy ra P f ( z ) f =
4 9 81
9 81

16
t ti
81
1
c
1
x = y , z = a = b,
= , a = b = 4c .
9
a+b+c 9
Bi 14. Nu a + b + c =
0 , bt ng thc hin nhin ng.
Nu a + b + c 0 , khi vit li bt ng thc di dng:

Vy gi tr nh nht ca P bng

a
b
c
27

.
8
+ 8
+ 27

64
a+b+c
a+b+c
a+b+c
a
b
c
suy ra x + y + z =
t x =
=
,y =
,z
1.
a+b+c
a+b+c
a+b+c

Khi v tri ca bt ng thc bng P = 8 x 4 + 8 y 4 + 27 z 4 .


S dng bt ng thc C-S ta c:

) (

8 x4 + y 4 4 x2 + y 2

Suy ra P 27 z 4 + (1 z ) .
4

464

4
4
= 2 x 2 + y 2 ( x + y ) =(1 z ) .

www.TaiLieuLuyenThi.com
Cty TNHH MTV DVVH Khang Vit

Xt hm s f ( z=
) 27 z 4 + (1 z ) ta c:
4

1
3
3
f '( z ) =4 (1 z ) + 27.4 z 3 ; f '( z ) =
0 (1 z ) =27 z 3 z = .
4
1
1
Ta c f(z) i du t m sang dng khi i qua z = nn ti z = th f(z) t
4
4
1
27

.
cc tiu hay P f ( z ) f =
4 64

Vy bt ng thc c chng minh. ng thc xy ra khi v ch khi a= b=

3c
.
2

Nhn xt. Trong trng hp a,b,c dng v cc h s ca a 4 , b 4 , c 4 i mt khc


nhau ta s dng k thut cn bng h s cho bt ng thc AM-GM hoc dng AM
GM dng lu tha.

Bi 15. Ta c: ( a + b + c ) = 2 a 2 + b 2 + c 2 a 2 + b 2 + c 2= 2 ( ab + bc + ca ) .
2

( a + b + c )= 4 ( ab + bc + ca ) .
2

Khi P =

4 a 3 + b3 + c 3

( a + b + c )3

) = 4

a
b
c

+ 4
+ 4
.
a+b+c
a+b+c
a+b+c

a
b
c
ta c x + y + z =
1 v
=
,y =
,z
a+b+c
a+b+c
a+b+c
ab + bc + ca 1
.
xy +=
yz + zx
=
( a + b + c )2 4

t x
=

y + z =1 x
y + z =1 x

Suy ra
1
1.
2
yz + x ( y + z ) = 4
yz = x x + 4
1
2
2
2

Mt khc ( y + z ) 4 yz (1 x ) 4 x 2 x + 0 x .
4
3

Khi :

3
P= 4 x3 + y 3 + z 3 = 4 x3 + ( y + z ) 3 yz ( y + z )

1
3

3
2
= 4 x3 + (1 x ) 3 x 2 x + (1 x )=
12 x 12 x + 3x + 1
4

2
Xt hm s f ( x)= 12 x3 12 x 2 + 3 x + 1 lin tc trn 0; ta c:
3

465

www.TaiLieuLuyenThi.com
Khm ph t duy K thut gii bt T Bi ton Max Min ng Thnh Nam

x=

6
.
f '( x)= 36 x 2 24 x + 3; f '( x)= 0
x = 1

2
2
x0;
3

11
1
2
1 11
Ta c =
. Suy ra 1 f ( x) =
.
f (0) f=
P
1, f=
f=

9
2
3
6 9

Vy gi tr nh nht ca P bng 1 t ti=


a 0,=
b c . Gi tr ln nht ca P
bng

a
11
5
t ti b= c=
hoc b= c=
a.
2
9
2

Bi 16. Khng mt tnh tng qut =


gi s z min { x, y, z} x > y > z 0 .
x+ y
y+z
z+x
9.
cn chng minh: ( x + y + z )
+
+
( x y )2 ( y z )2 ( z x )2

t P l biu thc v tri ca bt ng thc.


Ta c
x+ y
y
x

P ( x + y)
+
+
=
( x y )2 y 2 x 2

( x + y )2 + x + y=
+2
( x y )2 y x

x
+2+
y
x
2+
y

y
x x y
+ + +2.
y y x
x

t+2
x y
t t =+ , ( t > 2 ) do x > y khi P f (=
t)
+t +2.
t2
y x

Xt hm s f (=
t)
f '(t )=

(t 2)

t+2
+ t + 2 trn khong ( 2;+ ) ta c
t2
+ 1; f '(t )= 0 ( t 2 ) = 4
t= 4 .
2

t >2

Ta c f(t) i du t m sang dng khi i qua t = 4 nn f(t) t cc tiu ti


t = 4 hay f (t ) f (4) =
9 do P 9 .
Bt ng thc c chng minh.
ng thc xy ra khi v ch khi

x y
+ = 4, z = 0 hoc cc hon v.
y x

Bi 17. Khng mt tnh tng qut =


gi s c min {a, b, c} a, b > c 0 .
Khi :

466

www.TaiLieuLuyenThi.com
Cty TNHH MTV DVVH Khang Vit

P=

( a + b + c )2

( a b )

(b c )

(c a)

.
a
b
+2+
2
2

1
1
1
2
a + a + b + 2 a + b + 2
b
( a + b)
+ 2 +=

2
2
a
b
a
b
b2 a 2
b a
2+
( a b )
b
a
a b
t t =+ , ( t > 2 ) do a,b dng phn bit.
b a
t+2
2
Khi P f (t=
)
+ ( t + 1) + 1 .
t2
t+2
2
Xt hm s f (t=
)
+ ( t + 1) + 1 vi t > 2
t2
4
Ta c: f '(t ) =

+ 2 ( t + 1) ;
( t 2 )2
t >2

f '(=
t ) 0
t 1+ 3 .
=

Bng bin thin:


t
2
f(t)
f(t)

1+ 3

+
+

11 + 6 3

Da vo bng bin thin suy ra Pmin =


min f (t ) =
f(1 + 3) =
11 + 6 3 t ti
t >2

a b
+ =1 + 3, c =0 hoc cc hon v.
b a
Bi 18. Gi s c = min {a, b, c} khi a, b > c 0 ta c:
2

c
c

a 2 + b2 a + + b + ;
2
2

b2 + c2 b + ;
2

a2 + c2 a + ;
2

467

www.TaiLieuLuyenThi.com
Khm ph t duy K thut gii bt T Bi ton Max Min ng Thnh Nam

(a + b + c)

c
c
c
c

= a + + b + 4 a + b + .
2
2
2
2

c
c
1
1
1

.
Suy ra: P 4 a + b +
+
+
2
2
2
2
2
2

c
c
c
c

a +
b +
a + + b +
2
2
2
2

1
1
1 x
c
c
t x =
a+ ,y=
b + khi P 4 xy 2 + 2 + 2
= 4 +
2
2
y
x + y 2 y
x
t t =

y
4
.
+
x x + y
y x

4
4
x y
) 4t + vi t 2 ta c f '(t ) = 4 2 > 0, t 2
+ 2 xt hm s f (t=
t
y x
t

nn f(t) l hm ng bin vi t 2 . Do P f (t ) f (2) =


10 . ng thc xy
ra khi v ch khi x = y, c = 0 a = b, c = 0 .
Vy gi tr nh nht ca P bng 10 t ti=
a b=
, c 0 hoc cc hon v.
Bi 19. Gi s c = min {a, b, c} ta c:
2

c
c

a 2 + b2 a + + b + ;
2
2

b2 + c2 b + ;
2

a2 + c2 a + ;
2

ab + bc + ca = ab +

c
c2 c
c
c
c2
c
c
a
b
a
b
ab
a
b
+
+
+
+

+
+
+
= a + b +
( )
( )

2
4 2
2
2
4
2
2

c
c
1
1
1

Suy ra: P a + b +
.
+
+
2
2
2
2
2
2

c
c
c
c

a +
b +
a + + b +
2
2
2
2

1
1
1 x y
1
c
c
.
t x =
a+ ,y=
b + khi P xy 2 + 2 + 2
= + +
2
2
2
y
x +y y x x + y
x
y x

468

www.TaiLieuLuyenThi.com
Cty TNHH MTV DVVH Khang Vit

t t =

1
1
x y
+ 2 xt hm s f (t ) = t + vi t 2 ta c f '(t ) = 1 2 > 0, t 2
t
y x
t

5
. ng thc xy
nn f(t) l hm ng bin vi t 2 . Do P f (t ) f (2) =
2
ra khi v ch khi x = y, c = 0 a = b, c = 0 .
5
t ti=
a b=
, c 0 hoc cc hon v.
2
Cch 2: Coi P l hm s ca c ta chng minh P t gi tr nh nht khi c = 0 .
Tht vy ta cn chng minh:
1
1
1
1
1
1
( ab + bc + ca ) 2 2 + 2 2 + 2 2 ab 2 2 + 2 + 2
b +c
c +a
a
b
a +b
a +b
1
1
1
1
1
1
1
c (a + b) 2
+ 2
+ 2
ab 2 + 2 2
2
.
2
2
2
2
b +c
c +a
b
b +c
a + c2
a +b
a

Vy gi tr nh nht ca P bng

1
1
c2
c2
1

c (a + b) 2
+
+

ab
+

a2 a2 + c2
a + b2 b2 + c2 c2 + a 2
b2 b2 + c2

Bt ng thc cui ng do:

c (a + b)
a 2 + b2

0;

ac a 2 + b ( a c )
abc 2

=
2
2
2
2
2
2
2
2
a +c
a a +c
a a +c

c (a + b)

bc b 2 + a ( b c )
abc 2

=
b2 + c2 b2 a 2 + c2
b2 b2 + c2

c (a + b)

1
1
1
Vy ta tm gi tr nh nht=
ca Q ab 2
+ 2 + 2 .
2
a
b
a +b
a b 5
1
Tng t trn ta c: =
Q
+ + .
a b b a 2
+
b a
5
Vy gi tr nh nht ca P bng t ti=
a b=
, c 0 hoc cc hon v.
2
Bi 20. Nhn xt. Bi ton ny thc cht l mt trng hp ring ca hai bt ng
thc trn:
Ta c:

469

www.TaiLieuLuyenThi.com
Khm ph t duy K thut gii bt T Bi ton Max Min ng Thnh Nam

P=

( a + b + c )2

c +a
1

b +c
a +b

2
1
1
1
c
c

.
a + +b +
+
+
2
2
2
2
2
2

c
c
c
c

a + + b +
b +
a +
2
2
2
2

1
1
2 1
=( x + y ) 2 + 2 + 2
10
y
x + y 2
x

c
c
Vi x = a + , y = b + , ( x, y > 0, x + y =1) v c = min {a, b, c} .
2
2
1
Vy gi tr nh nht ca P bng 10 t ti a= b=
,c= 0 hoc cc hon v.
2

Bi 21. Gi s c = min {a, b, c} khi a 2 + b 2 a 2 + ( b + c ) v


2

a 2 + b2 + c2 a 2 + ( b + c ) .
2

Suy ra:
1

1
1

+
P ( ab + bc + ca ) 2 +
a ( b + c )2 a 2 + ( b + c )2
.
1

1
1
1
1
1
a (b + c ) 2 +

=
+
+
bc + a ( b + c ) 2 +
2
2
2
2
2
a ( b + c ) a + ( b + c )
a ( b + c ) a + ( b + c )2
t x= a, y= b + c a v tm gi tr nh nht ca biu thc quen thuc

1
1
1
=
Q xy 2 + 2 + 2
.
y
x + y 2
x

Ta c:
2

x y
x y
2 + 5 + + 2
y x
1
5
x y
y x
=
+
Q= + +
2
y x x+y
x y
2 +
y x
y x
x y
2 x 2 y
+
1
+ 2
y x
y
x

+55
=
2 2
x y
2 +
y x
470

www.TaiLieuLuyenThi.com
Cty TNHH MTV DVVH Khang Vit

ng thc xy ra khi v ch khi x = y, c = 0 a = b, c = 0 .


5
t ti=
a b=
, c 0 hoc=
a c=
,b 0 .
2
Bi 22. Bt ng thc cn chng minh tng ng vi:

Vy gi tr nh nht ca P bng

2
2
2
a b a +b +c
0.

+

ab + bc + ca
bc ca
Coi v tri l hm s ca c ta c:

f '(c) =

>

2a 2

2b 2

( b c )3 ( a c )3

( b + c ) ( a 2 + b2 + c 2 ) 2c ( ab + bc + ca )
( ab + bc + ca )2

2c a 2 + b 2 + c 2 2c ( ab + bc + ca )

( ab + bc + ca )2

2
2
2
c ( a b ) + ( b c ) + ( c a )

0
=
2
( ab + bc + ca )

Vy v tri l hm ng bin vi c vy ta ch cn chng minh vi c = 0 .


Khi ta cn chng minh:

a2
b2

b2

a2

a 2 + b2
ab

a 4 + b 4 ab a 2 + b 2 ( a b ) a 2 + ab + b 2 0 (lun ng).
2

Bt ng thc c chng minh. ng thc xy ra khi v ch khi=


a b=
,c 0 .
2

a 2 b2 a b a 2 + b2 a 2 + b2 + c2
a b
Cch 2: Ta c
.
+

+
+=

ab
ab + bc + ca
b2 a 2 b a
bc ca
Bt ng thc c chng minh.

Bi 23. Gi s x y z. t =
x a.z,=
y b.z, (1 b a 2 ) , khi
=
P f=
(a)

Ta c f '(a )

a 2b 2 + a 2 + b 2
.
ab ( a + b + 1)

( a + b + ab ) b2 ( a 1) + a b
a 2b ( a + b + 1)

0, a b 1 nn f(a) l hm

ng bin trn [1;2] suy ra f (b) f (a ) f (2) .


b 4 + 2b 2
b2 + 2
Mt khc f=
(b)
=
=
b 2 ( 2b + 1) 2b + 1

=
f (2)

5b 2 + 4
=
2b ( b + 3)

( b 1)2 + 1 1 v
2b + 1

(13b 10 )( b 2 ) + 6 6 .
10b ( b + 3)
5 5
471

www.TaiLieuLuyenThi.com
Khm ph t duy K thut gii bt T Bi ton Max Min ng Thnh Nam

6
.
5
Vy gi tr nh nht ca P bng 1 t ti x= y= z v gi tr ln nht ca P bng

Do 1 P

6
t ti x= y= 2, z= 1 hoc cc hon v.
5
Nhn xt. Ta c th tm nhanh Min ca P nh sau:

x 2 y 2 + y 2 z 2 + z 2 x 2 xy. yz + yz.zx + zx.=


xy xyz ( x + y + z ) suy ra P 1 .

Nu thay iu kin x, y, z [1;2] bng on [ 2;4] ta c kt qu tng t.


Bi 24. S dng bt ng thc AM-GM ta c: b3 + b3 + c3 3b 2 c .
Suy ra:

4a3 + b3 + c3 + 2b3 + c3 3b 2 c
P

( a + b + c )3

) 4a

+ b3 + c 3

( a + b + c )3

1
( b + c )3 16a3 + ( b + c )3 a 3 1 b + c 3
4
4
=
=
+

3
a +b+c 4 a +b+c
4(a + b + c)
( a + b + c )3

4a 3 +

a
b+c
=
,y
, ( y > x > 0 ) khi x + y =
1 v
a+b+c
a+b+c
1
3 1
P 4 x3 + y 3 = f ( y ) = 4 (1 y ) + y 3 .
4
4
3 1
Xt hm s f ( y ) = 4 (1 y ) + y 3 vi y ( 0;1) ta c
4
3
4
2
0< y <1
.
f '( y )= 12 (1 y ) + y 2 ; f '(y)= 0
y=
4
5
4
Ta c f(y) i du t m sang dng khi i qua y = nn f(y) t cc tiu ti
5
4
4 4
.
y = hay P f ( y ) f =
5
5 25

t x
=

4
t ti b= c= 2a .
25
Bi 25. Khng mt tnh tng qut gi s a l di cnh ln nht khi gc A ln nht:

Vy gi tr nh nht ca P bng

Tam gic khng nhn nn cos A 0 b 2 + c 2 a 2 0 b 2 + c 2 a 2 .


t
=
b 2 x=
.a 2 , c 2 y.a 2 , ( x, y > 0 ) v ta c: x + y 1 .
Khi t P l biu thc v tri ca bt ng thc.
472

www.TaiLieuLuyenThi.com
Cty TNHH MTV DVVH Khang Vit

1 1
xy + x + y
P = (1 + x + y ) 1 + + = ( x + y + 1)

x y
xy

( x + y + 1) .

( x + y )2 + x + y
4

( x + y)

( x + y + 1)( x + y + 4 )
=
x+ y

+ y + 4)
( x + y + 1)( x =
( x + y 1)( x + y 4 ) + 10 0
10 + 10

x+ y

x+ y

Bt ng thc c chng minh.


ng thc xy ra khi v ch khi tam gic vung.
Bi 26. t
=
y a=
.z, x b.z, ( a, b > 0 ) khi ta c a 2 + b 2 + 6= 4 ( a + b ) v
a3

=
P

Mt khc a 2 + b 2

b ( a + 1)

b3
a ( b + 1)

+ a 2 + b2 .

1
1
( a + b )2 ( a + b )2 + 6 4 ( a + b ) .
2
2

( a + b 2 )( a + b 6 ) 0 2 t = a + b 6 v
4 ( a + b ) = a 2 + b 2 + 6 2ab + 6 ab 2 ( a + b ) 3 .

S dng bt ng thc AM-GM ta c:


a3
b ( a + 1)

b3
a ( b + 1)

a 2 + b2

1
( a + b ) 2 v
2

a + 1 b ( a + 1) 3a
+

8
8
4

b + 1 a ( b + 1) 3b
+

8
8
4

a + b ab 1
a + b 2(a + b) 3 1 1

2+

= + 2.
2
4 4
2
4
4 2
1
Vy gi tr nh nht ca P bng + 2 t ti x= y= z .
2
Nhn xt. Ta c th nh gi nh sau

Suy ra P 2 +

b ( a +=
1)

1
1 2a + 2b + 2 1
3
.2b ( a + 1)( a + 1)
( a + b ) v
2
2
3
2

a ( b + 1)

1
( a + b )3 .
2

473

www.TaiLieuLuyenThi.com
Khm ph t duy K thut gii bt T Bi ton Max Min ng Thnh Nam

a3
1
1
b3
a 3 + b3

Do P 2
+
+
=
+ 2 2. + 2 = + 2 .
2
2.
3
3
3
4
2
( a + b )
( a + b )
(a + b)
Bi 27. HD: T gi thit ta c:
a
b
b
c
c
a
1
.
.
+
.
+
.
=
a+b+c a+b+c a+b+c a+b+c a+b+c a+b+c 4
a
b
c
t x =
ta c
=
,y =
,z
a+b+c
a+b+c
a+b+c
1
khi P = x 2 y + y 2 z + z 2 x .
x + y + z= 1, xy + yz + zx=
4

Bi 28. Ta c:

2
a + b + c ) 2 ( ab + bc + ca )
(
ab + bc + ca
1 2.
P=
=
1.
2
( a + b + c )2
(a + b + c)

Vy gi tr ln nht ca P bng 1 t ti a= b= 0, c= 1 .
2

c3
1+ c 1
T iu kin ta c: a3 + b3 = c ( c 1) c
.
=

4
2
Mt khc
3
2
2
2 (a + b)

.
a3 + b3 =( a + b ) ( a + b ) 3ab ( a + b ) ( a + b ) ( a + b ) =

4
4

Suy ra ( a + b ) c3 a + b c .
3

t
=
a x=
.c, b y.c ( x, y 0 ) ta c x + y 1 v
2
2
x + y ) 2 xy + 1 ( x + y ) + 1
(
2 xy
=

P=
2
2
2
( x + y + 1)
( x + y + 1)
( x + y + 1) ( x + y + 1)2

x2 + y 2 + 1

1
( x + y + 1)2 xy 1 1 x + y 2 1 1 3
2


=
( x + y + 1)2 2 2 2 2 2 8 8

Vy gi tr nh nht ca P bng

3
t ti a= b= 1, c= 2 .
8

Bi 35. Gi s c = min {a, b, c} khi ( a c ) a 2 , ( b c ) b 2 .


2

(
) 27 ( a b ) a b .
( a + b + c ) ( a + b ) . Vy bt ng thc c chng minh nu bt
ng thc sau ng: ( a + b ) 27 ( a b ) a b .

Vy ta i chng minh: a 2 + b 2 + c 2
2

2 3

474

2 3

2 3

2 2

2 2

www.TaiLieuLuyenThi.com
Cty TNHH MTV DVVH Khang Vit

Nu b = 0 bt ng thc tr thnh ng thc.


Nu b > 0 t
=
a x.b, ( x 0 ) a v chng minh bt ng thc.

( x + 1)
2

) (x

27 ( x 1) x 2 x 2 3 x + 1
2

ng thc xy ra khi x =

+ 6 x + 1 0 (lun ng).

3 5
3 5
b,c = 0 .
,c = 0 a =
2
2

Bi ton c chng minh. ng thc xy =


ra khi a

3 5
=
b,c 0 hoc cc
2

hon v.
Nhn xt. Vit li bt ng thc di dng.
f (c ) =

(a

+ b2 + c2

27 ( a b ) ( b c ) ( a c ) 0 .
2

Coi v tri bt ng thc l hm s ca c.

Ta c f '(=
c ) 6c a 2 + b 2 + c 2

+ 54 ( a b ) [b c + a c ] 0 (do
2

=
c min {a, b, c} 0 ).

Vy f (c) f (0) = a 2 + b 2

27 a 2b 2 ( a b ) .
2

Bi ton a v chng minh bt ng thc a 2 + b 2

27 a 2b 2 ( a b ) 0.
2

y chnh l kt qu ca cch trn. Trong trng hp a,b,c l cc s thc bt k


ta c bt ng thc sau:

(a

+ b2 + c2

2 ( a b) (b c ) (c a ) .
2

chng minh bt ng thc trn ta tm cch nh gi ( a c ) ( b c ) a 2b 2 .


2

Khng mt tnh tng qut gi s c l s nm gia a v b.


TH1: Nu c 0 ta c ngay iu phi chng minh.
TH2: Nu c 0 v a, b 0, ta t x =
a, y =
b, z =
c a bt ng thc v
trng hp 1 v c ngay iu phi chng minh.
TH3: Nu c 0, ab 0, ta c th gi s c [ a; b ] , khi xt

f (c) = ( c a )( c b ) + ab = c 2 ( a + b ) c + 2ab trn on [ a; b ] ta c:


f (c) max { f (a), f (b)} =
ab tc ( c a )( c b ) ab . Mt khc

( c a )( c b )= c ( c a b ) + ab ab .
Vy ( c a )( c b ) ab ( c a ) ( c b ) a 2b 2 .
2

475

www.TaiLieuLuyenThi.com
Khm ph t duy K thut gii bt T Bi ton Max Min ng Thnh Nam

Vy a v chng minh bt ng thc: a 2 + b 2 + c 2

2 ( a b ) a 2b 2 .
2

t a = x.b v a v chng minh bt ng thc:

( x + 1)

2 ( x 1) x 2 ( x + 1)
2

(x

2 x3 + 4 x 2 2 x + 1 0 .

2
2

( x + 1) x 2 x + 3 x 2 2 x + 1 0 (lun ng).

Bi ton c chng minh. ng thc xy ra khi a =


b, c =
0 hoc cc hon v.

Bi 36. Khng mt tnh tng qut gi s c = min {a, b, c} , khi ta c:


2

c
c

a 2 + b2 a + + b + ;
2
2

b2 + c2 b + ;
2

a2 + c2 a + ;
2

c
c

a 2 + b2 + c2 a + + b + .
2
2

c
c
t x =
a+ ,y=
b + , ( x, y > 0 ) bt ng thc c chng minh nu ta chng
2
2
1
1
12
32
minh c: 2 + 2 + 2
.

2
x
y
x +y
( x + y )2

S dng bt ng thc AM-GM ta c:


1
x

1
y

1
1
4
8
4
8
= 2 + 2 + 2
+ 2

+ 2
2
2

xy x + y 2
x +y
y
x +y x +y
x
8
8
4
32
= + 2
8.
=
2
2
2
2 xy x + y
2 xy + x + y
( x + y )2
12

Bi ton c chng minh. ng thc xy ra khi v ch khi=


a b=
, c 0 hoc cc
hon v.
Bi 39. Khng mt tnh tng qut gi s a > b > c, t
a = x + y + c, b = y + c, ( x, y > 0 ) .

Gi P l biu thc v tri ca bt ng thc cn chng.

( x + y) .
y2
x2
Ta c P =
+
+
x 2 ( x + y )2
y2
2

476

www.TaiLieuLuyenThi.com
Cty TNHH MTV DVVH Khang Vit

t y = t.x ta c P =
t +

Ta cn chng minh: t +

2
t + 1)
(
.
+

( t + 1)2

t2

2
t + 1)
(
+
5.

( t + 1)2

t2

t 6 + 2t 5 3t 4 6t 3 + 2t 2 + 4t + 1 0 t 3 + t 2 2t 1 0 (lun ng).

Cch 2:=
t x

a b
bc
ca
=
,y =
,z
, khi :
bc
ca
a b

0
x + y + 1 =

0
xy + y + 1 =
1

y + + 1 = 0 yz + z + 1 = 0 xy + yz + zx + x + y + z + 3 = 0 .
z

zx + x + 1 =
0

1
0
z
+
+
=

Ta cn chng minh x 2 + y 2 + z 2 5 ( x + y + z ) 2 ( xy + yz + zx ) 5 0
2

( x + y + z ) 2 ( 3 x y z ) 5 0 ( x + y + z + 1) 0 (lun ng).
2

Nhn xt. T cch 2 ta c mt ng thc p mt sau y:


2

a b bc ca
a b bc ca
5+
+
+
+ 1 .

+
+
=
bc ca a b
bc ca a b
Bi 41. Nhn xt. y l mt bi ton kh v c nhiu li gii dnh cho bt
ng thc ny.
Di y trnh by mt li gii khc da vo du hiu ng bc ca bt ng
thc trn.

Gi s c max {a, b, c} c > 0 .


=
t
=
a c=
.x, b c.y, ( x, y [ 0;1]) khi ta cn chng minh
1

( x + y)

( x + 1)

( y + 1)

9
.
4 ( xy + x + y )

Quy ng hai v v rt gn a v bt ng thc:

(
+ 6( x

(
)
+ y + x y ) + 2 xy x + y + x

x 6 x = x ( x 1) ( 4 x

4 x + y + x5 + y 5 + xy x 4 + y 4 + 2 xy 1 + x3 + y 3 + 6 x 2 y 2

x2 + y 2 + x4 + y 4

Ta c: 4 x + x5 x 2

3 3

+ y 2 + xy ( x + y ) + x 2 y 2 x 2 + y 2

+ 7 x + 4 0, x [ 0;1] do

477

www.TaiLieuLuyenThi.com
Khm ph t duy K thut gii bt T Bi ton Max Min ng Thnh Nam

4 x + y + x5 + y 5 x 2 + y 2 + x 4 + y 4 + 6 x3 + y 3 .

Vy ta ch cn chng minh

) (

4 x 4 + y 4 + 2 1 + x3 + y 3 + 6 xy 2 x + y + x 2 + y 2 + xy ( x + y ) + 6 x 2 y 2 + xy x 2 + y 2 .

4 x +y
4

) = 3( x

+y

)+ x

Ta c:
4

+ y 3.2 x 2 y 2 + xy x 2 + y 2 = 6 x 2 y 2 + xy x 2 + y 2 .
4

Vy ta chng minh 1 + x3 + y 3 + 3 xy x + y + x 2 + y 2 + xy ( x + y ) .
Ta c th gi s x y 0 y x 1 v bt ng thc tng ng vi:

y ( x 1) + (1 x )( x y ) + (1 y ) + (1 x ) 1 x + 2 ( x y ) 0 (lun ng).

Vy bt ng thc c chng minh.


2

Bi 42. t
=
a x=
.c, b y.c, ( x, y > 0 ) , khi theo gi thit ta c:
1
x2

1
x2 y 2
2
2
.
=

x
+
y
=
2
y2 2
1

2
x2 y 2
2
2
x
+
y

xy

2 xy xy 4

2
S dng bt ng thc AM-GM ta c:
.
2 2
2
x y
2
2 1
2 = x + y 2 ( x + y ) x + y xy

Khi : P =

x
y
+
+
y +1 x +1

1
x2 + y 2 + 1

x2 y 2 + 2 ( x + y )
x
y
x2 + y 2 + x + y
x 2 y 2 + 2 xy
x 2 y 2 + 2 xy
=
+
=

=
y +1 x +1
xy + x + y + 1 2 xy + 2 ( x + y ) + 2 2 xy + 2 xy + 2
4 xy + 2
(hm nghch bin vi x+y).

Do P

x 2 y 2 + 2 xy
+
4 xy + 2

1
x2 y 2
+1
2

t xy = t , ( t 4 ) P f (t ) =

Xt hm s=
f (t )

478

t 2 + 2t
+
4t + 2

t 2 + 2t
+
4t + 2

1
2

t
+1
2

1
2

t
+1
2

vi t 4 ta c:

www.TaiLieuLuyenThi.com
Cty TNHH MTV DVVH Khang Vit

f '(t )
=

t2 + t +1

( 2t + 1)2

(t

2t
=
3
t2 + 2

) (t

+ t +1

) 2t ( 2t + 1)
(t + 2)

+2

( 2t + 1)

> 0, t 4 .

5
Do f(t) l hm ng bin vi t 4 P f (t ) f (4) = .
3
5
Vy gi tr nh nht ca P bng t ti a= b= 2c .
3

Bi 43. Chia hai v bt ng thc cho ( a + b + c ) a v chng minh bt ng


3

thc: 27 x 2 + y 2 + z 2 + 45 xyz 32 ( xy + yz + zx )

=
P 27 x 2 + y 2 + z 2 + xy ( 45 z 32 ) 32 z ( x + y ) 0 .
a
b
c
=
,y
=
,z
; x=
+ y + z 1.
a+b+c
a+b+c
a+b+c
1
Khng mt tnh tng qut =
gi s z min { x, y, z} z 45 z 32 < 0 .
3
Khi s dng bt ng thc AM-GM ta c:

Trong
x
=

27
2 ( x + y)
P 27 z + ( x + y ) +
2
4

( 45 z 32 ) 32 z ( x + y )

27
1
2 (1 z )
= 27 z + (1 z ) +
( 45 z 32 ) 32 z (1 z=) ( 3z 1)2 ( 5 z + 22 ) 0
2
4
4
Bi ton c chng minh. ng thc xy ra khi v ch khi a= b= c .
2

Bi 44. t
=
a x=
.c, b y.c, ( x, y > 0 ) ta c:
P=

xy

( x + y)

( x + 1)

( y + 1)

4 xy
.
x
+
y
(
)( x + 1)( y + 1)

Nhn xt. Do biu thc ca P i xng vi a,b,c nn d on ng thc xy ra khi


a = b = c x = y =1 .
Vy ta chng minh P
xy

( x + y)

1
.
4
x

( x + 1)

( y + 1)

4 xy
1

x
+
y
x
+
1
y
+
1
(
)( )( ) 4

xy ( x + 1) ( y + 1) + x ( y + 1) ( x + y ) + y ( x + 1) ( x + y ) 4 xy ( x + y )( x + 1)( y + 1) 1

4
( x + y )2 ( x + 1)2 ( y + 1)2
2

479

www.TaiLieuLuyenThi.com
Khm ph t duy K thut gii bt T Bi ton Max Min ng Thnh Nam

(x

y + xy + x + y 2

)( xy

+ xy + y + x 2

( x + y )2 ( x + 1)2 ( y + 1)2

) 1 ( x 1)
4

( y 1)2 ( x y )2 0
( x + y )2 ( x + 1)2 ( y + 1)2
2

(lun ng).
1
t ti a= b= c .
4
Cch 2: Ta bin i a P v dng:
ab
bc
ca
3
1
1
1
4abc
P= +
+
+

2
2
2
4 (a + b)
4 (b + c )
4 (c + a)
4 ( a + b )( b + c )( c + a )

Vy gi tr ln nht ca P bng

( b c )2 + ( c a )2 +
3 1 (a b)
16abc

=

+
4 4 ( a + b )2 ( b + c )2 ( c + a )2 ( a + b )( b + c )( c + a )

t=
x

a b
bc
ca
2a
2b
2c
.
=
,y
=
,z
x +=
1
, y +=
1
, z +=
1
a+b
b+c
c+a
a+b
b+c
c+a

Do :
3 1
P = x 2 + y 2 + z 2 + 2 ( x + 1)( y + 1)( z + 1)

4 4
3 1
= x 2 + y 2 + z 2 + 2 (1 + x + y + z + xy + yz + zx + xyz )

4 4
3 1
1
1
2
= x 2 + y 2 + z 2 + 2 xy + 2 yz + 2 zx + 2 = ( x + y + z )
4 4
4
4
Nhn xt. Li gii trn ta s dng ng thc quen thuc:
a b bc ca ab bc ca
x + y + z + xyz
=
+
+
+
.
. = 0
a+b b+c c+a a+b b+c c+a
Bi 45. HD: Theo gi thit ta c:
xy + yz + zx 2
2
.
2 ( x + y + z )= 7 ( xy + yz + zx )
=
( x + y + z )2 7

t a
=

1
a + b + c =
x
y
z

,b
,c
=
=

2 (1) .
x+ y+z
x+ y+z
x+ y+z
ab + bc + ca =
7

7 ( y 2z )
7
Khi =
P
=
( b 2c )2 .
2
2
2( x + y + z )
2

Bi 46. Theo gi thit ta c: ( a + b + c ) =2 ( ab + bc + ca )


2

480

ab + bc + ca

(a + b + c)

1
= .
2

www.TaiLieuLuyenThi.com
Cty TNHH MTV DVVH Khang Vit

1
x + y + z =
a
b
c

t x
,y
,z
=
=
=

1.
a+b+c
a+b+c
a + b + c xy + yz + zx =

1 10
1 + 10
2
2
1

.
Ta c: ( y + z ) 4 yz (1 x ) 4 x (1 x )
x
3
3
2

Khi :
P = x3 + y 3 + z 3 xyz = x3 + ( y + z ) 3 yz ( y + z ) xyz
3

3
1

= x3 + (1 x ) 3 x (1 x ) (1 x ) x x (1 x )
2
2

4 x3 4 x 2 2 x + 5
2

Xt hm s f ( x) =

1 10 1 + 10
4 x3 4 x 2 2 x + 5
lin tc trn on
;
ta c:
3
2
3

2 10
x =
6
.
f '( x) =6 x 2 4 x 1; f '( x) =0

2 + 10
x =
6

Bi 47. HD: Ta c: P =

( a b )5 + ( b c )5 + ( c a )5 .
5
5( a + b + c)

a b
bc
ca
=
,y
=
,z
x +=
y+z 0.
a+b+c
a+b+c
a+b+c
Theo gi thit ta c:

t
=
x

( a b )2 + ( b c )2 + ( c a )2 = ( a + b + c )2 x 2 + y 2 + z 2 =

1.

Vy bi ton a v tm gi tr ln nht v nh nht ca biu thc


0
x + y + z =
.
Q = x5 + y 5 + z 5 vi 2
2
2
1
x + y + z =
Bi 48. Theo gi thit ta c: 5 ( x + y + z=
) 16 ( xy + yz + zx )
2

x
y
y
z
z
x
5
.
.
+
.
+
.
=
x + y + z x + y + z x + y + z x + y + z x + y + z x + y + z 16

t a
=

x
y
z
ab + bc + ca =
=
;b
=
;c

16 (1) .
x+ y+z
x+ y+z
x+ y+z
a + b + c =
1
481

www.TaiLieuLuyenThi.com
Khm ph t duy K thut gii bt T Bi ton Max Min ng Thnh Nam

Khi P
=

+ yz + zx )
( x + y + z )( xy
=
xyz

5( x + y + z )
5
.
=
16 xyz
16abc
3

Bi ton a v tm gi tr ln nht, gi tr nh nht ca biu thc P vi iu


kin (1).
b + c =1 a

Gi thit ta c:
5
5
5 .
2
bc = 16 a ( b + c ) = 16 a (1 a ) = a a + 16
5
5
5
Ta c: =
Q abc
= a a ( b + c=
) a a (1 a=
) a a 2 a + .
16
16
16

Mt khc:
5
1
1
2
2

4bc ( b + c ) 4 a 2 a + (1 a ) 12a 2 8a + 1 0 a .
16
6
2

1 1
Xt hm s f (=
a ) a a 2 a + lin tc trn on ; ta c:
16

6 2

a=

5
4 .
f '(a ) =3a 2 2a + ; f '(a ) =0
16
a = 5

12
1
5 25
1
1 1
Ta c f=
.
; f=
f=

f=

6
12 864 2
4 32
1
25
54
Suy ra MaxQ = MinP =
10 v MinQ = MinP = .
32
864
5
Vy gi tr nh nht ca P bng 10 t ti y= z= 2 x .

Gi tr ln nht ca P bng

54
5
t ti y= z=
x.
5
2

Bi 49. Theo gi thit ta c:


8 ( x + y + z=
) 27 ( xy + yz + zx )
2

t a
=

(x + y + z)

yz

(x + y + z)

zx

8
.
=
2
( x + y + z ) 27

x
y
z
ab + bc + ca =
=
;b
=
;c

27 (1) .
x+ y+z
x+ y+z
x+ y+z
a + b + c =
1

Khi P
=

482

xy

3
3
3
8
27 x3 + y 3 + z 3 27 a + b + c
x3 + y 3 + z 3
.
.
.
= =
13 ( x + y + z )( xy + yz + zx ) 13 ( x + y + z )3
13

www.TaiLieuLuyenThi.com
Cty TNHH MTV DVVH Khang Vit

8
8
8

a (b + c ) =
a (1 a ) = a 2 a +
bc =
T h iu kin (1) ta c:
27
27
27 .
b + c =1 a
8
1
5
2
2

Mt khc: ( b + c ) 4bc (1 a ) 4 a 2 a + a .
27
9
9

Khi :

27 3
27 3
8
3
3

a + ( b + c ) 3bc ( b + =
c )
a + (1 a ) 3 (1 a ) a 2 a +

13
13
27

3
=
27 a3 27 a 2 + 8a + 1
13
P
=

Xt hm s =
f (a)

3
1 5
27 a3 27 a 2 + 8a + 1 lin tc trn on ; ta c:
13
9 9

a=

3
9
.
f '(a ) = 81a 2 54a + 8 ; f '(a ) =
0

13
a = 4

1
4 43 5
2 47
Ta c f=
.
; f=
f=

f=

9
9
117
9



9 119
47
43
Suy=
ra Pmax =
.
;Pmin
117
117
Bi 50. t
a
b
c
=
x
=
,y
=
,z
x2=
+ y2 + z2 1 .
2
2
2
2
2
2
2
2
2
a +b +c
a +b +c
a +b +c

Khi P= 2 ( x + y + z ) 9 xyz= x ( 2 9 yz ) + 2 ( y + z ) .

x2 + y 2 + z 2 1
Khng mt tnh tng qut=
gi s x 2 max x 2 , y 2 , z 2 x 2 =
.
3
3
Ta c
2
2
2

x ( 2 9 yz ) + 2 ( y + z ) x 2 + ( y + z ) . ( 2 9 yz ) + 4=

t t = yz t

( 2 yz + 1) (81y 2 z 2 36 yz + 8)

y 2 + z 2 1 x2 1
=
.
2
2
3

1 1
Khi P f (t ) = ( 2t + 1) 81t 2 36t + 8 vi t ; ta c:
3 3

483

www.TaiLieuLuyenThi.com
Khm ph t duy K thut gii bt T Bi ton Max Min ng Thnh Nam

t=

9
.
f '(t ) =486t 2 + 18t 20; f '(t ) =0
t = 5
27
1 29 2 100 5 1369 1 25
Ta c f =
.
; f =
;f =
;f=

9
3 3
9
27 243 3 3

2 100
5 1369
.
Suy ra Max f (t ) = f =
; Min f (t ) = f =
1
1
1 1

9
9

27 243
t ;
t ;
3 3

3 3

Vy gi tr ln nht ca P bng
nh nht ca P bng

10
t ti chng hn a =
1, b =
c=
2 v gi tr
3

10
.
3

CH 5: K THUT S DNG BT NG THC TIP TUYN


A. NI DUNG PHNG PHP
nh l 1(Bt ng thc tip tuyn) Cho hm s f(x) lin tc v c o hm cp
2 trn on [ a; b ] .
Nu f ''( x) 0, x [ a; b ] ta lun c f ( x) f '( x0 ) ( x x0 ) + f ( x0 ), x0 [ a; b ] .
Nu f ''( x) 0, x [ a; b ] ta lun c f ( x) f '( x0 ) ( x x0 ) + f ( x0 ), x0 [ a; b ] .
ng thc xy ra trong hai bt ng thc trn x =
x0 .
Chng minh.
Xt hm s g ( x=
) f ( x) f '( x0 ) ( x x0 ) f ( x0 ), x [ a; b ] , x0 [ a; b ] .
Ta c g=
'( x) f '( x) f '( x0 ) do f ''( x) 0, x [ a; b ] nn f(x) l hm ng
bin trn [ a; b ] . Do g(x) i du t m sang dng khi i qua x0 nn g(x)
t cc tiu ti x0 hay g ( x) g ( x0 ) .
f ( x) f '( x0 ) ( x x0 ) f ( x0 ) 0 f ( x) f '( x0 ) ( x x0 ) + f ( x0 ), x [ a; b ] .

Chng minh tng t.

Nhn xt. H thc


=
y f '( x0 ) ( x x0 ) + f ( x0 ) chnh l phng trnh tip tuyn ca
th hm s f(x) ti im x0 . Do vy nu f ''( x) 0, x [ a; b ] th tip tuyn vi
th hm s ti im bt k trn on [ a; b ] lun nm pha di th hm s.
Nu f ''( x) 0, x [ a; b ] th tip tuyn vi th hm s ti im bt k trn on

[ a; b] lun nm pha trn th hm s.


484

www.TaiLieuLuyenThi.com
Cty TNHH MTV DVVH Khang Vit

Du hiu nhn bit


Thng thng bi ton c pht biu di dng
Cho cc s thc x1 , x2 ,..., xn tha mn g ( x1 ) + g ( x2 ) + ... + g ( xn ) k bi ton
yu cu chng minh
f ( x1 ) + f ( x2 ) + ... + f ( xn ) M hoc f ( x1 ) f ( x2 )... f ( xn ) M .
Ta c th p dng trc tip cc nh l 1 v 2 hoc ly logarit ha t nhin hai
v bt ng thc.
K nng cn vn dng
Vit phng trnh tip tuyn ca hm s f(x) ti im x0 (im du bng xy ra
k
) c phng trnh:=
y f '( x0 )(x x 0 ) + f ( x0 ) .
n
Xt tnh dng m ca f ( x) [ f '( x0 )(x x 0 ) + f ( x0 ) ] .

thng l x0 =

Thay x bi x1 , x2 ,..., xn cui cng cng li theo v n bt ng thc trn ta c


ngay iu phi chng minh.
Ch . Trong mt s trng hp khng p dng trc tip c cch trn ta cn xt
trng hp hoc xy dng mt hm ph ph hp vi iu kin( f ''(x) 0,f''(x) 0 )
v iu kin du bng xy ra.
V d 1. Cho a,b,c l cc s thc tha mn iu kin a + b + c =
6.

Chng minh rng a 4 + b 4 + c 4 2 a3 + b3 + c3 .


Li gii
Nhn xt. Du bng xy ra khi a= b= c= 2 nn ta vit phng trnh tip tuyn vi
th hm s f ( x=
) x 4 2 x3 ti im x = 2 .
Phng trnh tip tuyn l =
y 8 x 16 .

( x 2 )2 ( x 2 2 x + 4 ) 0, x .

Ta c: f ( x) 8 x + 16=

Do f (a ) + f (b) + f (c) 8 ( a + b + c ) 3.16 =


0.

V vy a 4 + b 4 + c 4 2 a3 + b3 + c3 .
Bi ton c chng minh. ng thc xy ra khi v ch khi a= b= c= 2 .
V d 2. Cho a,b,c l cc s thc khng m tha mn iu kin a + b + c =
1.
a
b
c
9
Chng minh rng 2
+ 2
+ 2
.
10
a +1 b +1 c +1
Li gii
Xt hm s f ( x) =

x
x +1
2

vi x [ 0;1] ta c
=
f ''( x)

2 x x2 3

( x + 1)
2

) 0, x [0;1] .
485

www.TaiLieuLuyenThi.com
Khm ph t duy K thut gii bt T Bi ton Max Min ng Thnh Nam

p dng bt ng thc ii.nh l 1 ta c:


1
1
1
f (a) f' a + f
3
3
3
1
1
f (b) f ' b +
3
3
1
1
f (c) f' c + f
3
3

1
f
3
1

3

Cng theo v 3 bt ng thc trn ta c:


1
VT
= f (a ) + f (b) + f (c) f ' ( a + b + c 1) + 3 f
3
1
.
ng thc xy ra khi v ch khi a= b= c=
3

1 9
.
=

3 10

3
Nhn xt. Bt ng thc vn ng trong trng hp a, b, c .
4
1
Tht vy xt tip tuyn ca f(x) ti im x = c phng trnh l:
3
18
3
.
=
y
x+
25
50

( 3x 1) ( 4 x + 3) 0, x 3 .
18
3
Ta c: f ( x) x
=
25
50
4
50 x 2 + 1
2

Do VT
= f (a ) + f (b) + f (c)

18
3
9
.
( a + b + c ) + 3. =
50
50 10

nh l 2 (Bt ng thc ct tuyn) Cho hm s f(x) lin tc v c o hm cp 2


trn on [ a; b ] .
f (a) f (b)
( x a ) + f (a), x [ a; b] .
a b
f (a) f (b)
Nu f ''( x) 0, x [ a; b ] ta lun c f ( x)
( x a ) + f (a), x [ a; b] .
a b
ng thc xy ra trong hai bt ng thc trn x =
a hoc x = b .
Vic chng minh hai bt ng thc trn ch cn chuyn v v kho st hm
trc tip.
Ta c mt bt ng thc khc hay c s dng

Nu f ''( x) 0, x [ a; b ] ta lun c f ( x)

486

www.TaiLieuLuyenThi.com
Cty TNHH MTV DVVH Khang Vit

nh l 3.(Bt ng thc Jensen) Cho hm s f ( x) lin tc v c o hm cp


hai trn khong (a;b) v n s thc dng k , k = 1, n c tng bng 1 ta c
+ Nu f ''( x) > 0, x ( a; b ) th ta c

1 f ( x1 ) + 2 f ( x2 ) + ... + n f ( xn ) f (1 x1 + 2 x2 + ... + n xn ) .
Vi mi xk ( a; b ) , k =
1, n . ng thc xy ra khi v ch khi x1= x2= ...= xn .
+ Nu f ''( x) < 0, x ( a; b ) th ta c

1 f ( x1 ) + 2 f ( x2 ) + ... + n f ( xn ) f (1 x1 + 2 x2 + ... + n xn ) .
Vi mi xk ( a; b ) , k =
1, n . ng thc xy ra khi v ch khi x1= x2= ...= xn .
Chng minh.
a) t y= 1 x1 + 2 x2 + ... + n xn . V f ''( x) > 0 nn p dng bt ng thc tip
tuyn ta c
f ( xk ) f '( y ) ( xk y ) + f ( y ), k =
1, n

.
k f ( xk ) f '( y ) ( k xk k y ) + k f ( y ), k =
1, n
Cng li theo v n bt ng thc trn ta c
n

k f ( xk ) ( f '( y) ( k xk k y ) + k f ( y)=) f ( y=) f (1x1 + 2 x2 + ... + n xn ) .


k 1=
k 1

Bt ng thc c chng minh.


Nhn xt. Ta c th vit li bt ng thc di dng
1 x1 + 2 x2 + ... + n xn

1 + 2 + ... + n

1 f ( x1 ) + 2 f ( x2 ) + ... + n f ( xn ) (1 + 2 + ... + n ) f
Trong 1 , 2 ,..., n l cc s thc dng bt k.

b) Chng minh tng t.


T y ta c ngay kt qu ca bt ng thc AM GM suy rng.
(Bt ng thc AM GM suy rng) Cho 1 , 2 ,..., n l cc s thc dng c
tng bng 1, khi vi mi s thc khng m bt k x1 , x2 ,..., xn ta c

1 x1 + 2 x2 + ... + n xn x11 x2 2 ...xn n .


Chng minh.
Bt ng thc tng ng vi:

ln (1 x1 + 2 x2 + ... + n xn ) 1 ln x1 + 2 ln x2 + ... + n ln xn .

Xt hm s =
f ( x) ln x, x > 0 ta c f ''( x) =

< 0, x > 0 .
x2
Khi p dng bt ng thc Jensen ta c ngay iu phi chng minh.
487

www.TaiLieuLuyenThi.com
Khm ph t duy K thut gii bt T Bi ton Max Min ng Thnh Nam

V d 1. Cho x,y,z,t l cc s thc dng tho mn iu kin


x 2, x + y 6, x + y + z 12, x + y + z + t 24 .
Chng minh rng

1 1 1 1
+ + + 1.
x y z t

Li gii
Xt hm s f (t ) =

1
2
vi t dng ta c f ''(t )= 3 > 0, t > 0 .
t
t

Do ta c
1 1 1 1 1 x 1 y 1 z 1 t
+ +=
+
f + f + f +
f
x y z t 2 2 4 4 6 6 12 12
144
1 x 1 y 1 z 1 t
f . + . + . + . =
2 2 4 4 6 6 12 12 36 x + 9 y + 4 z + t
144
=
27 x + 5 ( x + y ) + 3 ( x + y + z ) + ( x + y + z + t )
144
1
=
27.2 + 5.6 + 3.12 + 24
Bt ng thc c chng minh. ng thc xy ra khi v ch khi
=
x 2,=
y 4,=
z 6,=
t 12 .

Bi tp tng t
Cho x,y,z l cc s thc dng tho mn iu kin x 4, y 9, x + y + z 49 .
Chng minh rng

1
1
1
+
+
1.
x
y
z

V d 2. Cho a,b,c l cc s thc dng tho mn iu kin abc = 1 .


a

Chng minh rng a b .b c .c a 1 .


Li gii
a
b
c
Bt ng thc cho tng ng vi ln a + ln b + ln c 0 .
b
c
a
1
Xt hm s=
f ( x) x ln x, x > 0 ta c f '( x) = ln x + 1; f ''( x) =
> 0, x > 0 .
x
1
1
1
a
b
c
V vy ln a + ln b + ln c =
f (a ) + f (b) + f (c)
b
c
a
b
c
a
a
b c

+ +

1 1 1
+ + f b c a
b c a 1 + 1 + 1
a b c
488

www.TaiLieuLuyenThi.com
Cty TNHH MTV DVVH Khang Vit

a b c 1 1 1
+ + + + .
b c a a b c
Tht vy theo bt ng thc AM GM ta c

Ch f(x) ng bin v

a
b
a b2 3
+ 2. 33 . 2 =
b
c
b c
c
b
c
b c2 3
+ 2. 33 . 2 =
c
a
c a
a
c
a
c a2 3
+ 2. 33 . 2 =
a
b
a b
b
Cng li theo v ba bt ng thc trn ta c pcm.
T suy ra
a
b
c
1 1 1
ln a + ln b + ln c + + f (1) =
0.
b
c
a
b c a

Bt ng thc c chng minh. ng thc xy ra khi v ch khi a= b= c= 1 .


Ch . Bt ng thc trn vn ng khi abc 1 .
B. BI TON CHN LC
Bi 1. Chng minh rng vi mi s thc dng a,b,c ta c

( b + c a )2 + ( c + a b )2 + ( a + b c )2 3 .
2
2
2
5
a2 + (b + c )
b2 + ( c + a )
c2 + ( a + b )
Li gii
Bt ng thc c dng thun nht chun ho a + b + c =
1.
Bt ng thc tr thnh

(1 2a )2 + (1 2b )2 + (1 2c )2 3
2
2
2
5
a 2 + (1 a )
b 2 + (1 b )
c 2 + (1 c )

.
27
2
+
+

2a 2a + 1 2b 2 2b + 1 2c 2 2c + 1 5
1
1
54 x + 27
Tip tuyn ca hm s y = 2
ti im x = l y =
.
3
25
2x 2x + 1
1
54 x + 27
.
Vy ta chng minh:

2
25
2x 2x + 1
1

2 ( 3 x 1) ( 6 x + 1)
2

Tht vy bt ng thc tng ng vi:

25 2 x 2 2 x + 1

0 , lun ng.

489

www.TaiLieuLuyenThi.com
Khm ph t duy K thut gii bt T Bi ton Max Min ng Thnh Nam

Thay x bi a,b,c ri cng li theo v ta c pcm.


Bi tp tng t
1) Cho a,b,c l cc s thc dng. Chng minh

( 2a + b + c )2 + ( 2b + c + a )2 + ( 2c + a + b )2 8 .
2
2
2
2a 2 + ( b + c )
2b 2 + ( c + a )
2c 2 + ( a + b )
2) Cho a,b,c l cc s thc dng c tng bng 3. Chng minh
a2 + 9
2a 2 + ( b + c )

b2 + 9

2b 2 + ( c + a )

c2 + 9
2c 2 + ( a + b )

5.

3) Cho a,b,c l cc s thc khng m tho mn iu kin a + b + c > 0 .


Chng minh rng

a2
5a + ( b + c )
2

b2
5b + ( c + a )
2

c2
5c + ( a + b )
2

1
.
3

4) Cho a,b,c l cc s thc khng m tho mn iu kin a + b + c > 0 .


Chng minh rng
1
a2
b2
c2
2

+
+
.
2
2
2
2
2
2
2 2a + ( b + c )
3
2b + ( c + a )
2c + ( a + b )

5) Cho a,b,c l cc s thc dng. Chng minh


a (b + c )
a + (b + c )
2

b (c + a)
b + (c + a)
2

c (a + b)
c + (a + b)
2

6
.
5

Bi 2. Cho a,b,c l cc s thc dng tha mn iu kin


a 2 ab + b 2 + b 2 bc + c 2 + c 2 ca + a 2 =
12 .

Tm gi tr ln nht ca biu thc P =

a2 + 9
Li gii

b
b2 + 9

c
c2 + 9

Ch : a 2 ab + b 2 + b 2 bc + c 2 + c 2 ca + a 2 a + b + c .
Chng minh.
3( a b ) + ( a + b )
(a + b) a + b .
=

4
4
2
2

Ta c:
a ab + b
=
2

b+c
c+a
.
, c 2 ca + a 2
2
2
Cng li theo v 3 bt ng thc trn ta c ngay iu phi chng minh.
T suy ra: a + b + c 12 .

Tng t:

490

b 2 bc + c 2

www.TaiLieuLuyenThi.com
Cty TNHH MTV DVVH Khang Vit

Biu thc P c dng P = f (a ) + f (b) + f (c) trong f ( x) =


x

Xt hm s f ( x) =
f '(x) =

(x

+9

x2 + 9

x
x2 + 9

trn ( 0;+ ) ta c:

; f ''( x) =

27 x

(x

+9

< 0, x > 0 .

Do p dng bt ng thc ii ta c:
f (a ) f'(4) ( a 4 ) + f (4)
f (b) f '(4) ( b 4 ) + f (4)
f (c) f '(4) ( c 4 ) + f (4)

Cng li theo v 3 bt ng thc trn ta c:


=
P f (a) + f (b) + f (c) f '(4) ( a + b + c 12 ) + 3 f (4) 3f(4)
=

12
.
5

ng thc xy ra khi v ch khi a= b= c= 4 .


12
t ti a= b= c= 4 .
Vy gi tr ln nht ca P bng
5
Bi tp tng t
Cho a,b,c l cc s thc dng tho mn iu kin a 2 + b 2 + c 2 =
3.
1
1
1
Chng minh rng
+
+
1.
1 + 8a
1 + 8b
1 + 8c
Bi 3. Cho a,b,c l cc s thc khng m tha mn iu kin a + b + c =
3.
Tm gi tr ln nht ca biu thc

P =a + a 2 + 1

) (b +
b

b2 + 1

) (c +

Li gii
Ly logarit ha t nhin P ta c:

c2 + 1 .

ln=
P b ln a + a 2 + 1 + c ln b + b 2 + 1 + a ln c + c 2 + 1 .

Xt hm s f ( x=
) ln x + x 2 + 1 , x 0 ta c
1+

x +1 = 1
f '( x) =
> 0, f ''( x) =
x + x2 + 1
x2 + 1
2

( x + 1)
2

0, x 0 .

491

www.TaiLieuLuyenThi.com
Khm ph t duy K thut gii bt T Bi ton Max Min ng Thnh Nam

p dng nh l 1.ii ta c:
f (a ) f '(1) ( a 1) + f (1);
f (b) f '(1) ( b 1) + f (1);
f (c) f '(1) ( c 1) + f (1).

Ly tng ba bt ng thc trn ta c


ln P = b. f (a ) + c.f(b) + a .f(c)
b ( f '(1) ( a 1) + f (1) ) + c ( f '(1) ( b 1) + f (1) ) + a ( f '(1) ( c 1) + f (1) )
=

( ab + bc + ca a b c ) f '(1) + ( a + b + c ) f (1)

2
1

( a + b + c ) a b c f '(1) + 3 f (1)
= 3f(1)
= 3ln 1 + 2
3

Do P 1 + 2

Vy gi tr ln nht ca P bng 1 + 2

t ti a= b= c= 1 .

Bi 4. Cho tam gic ABC nhn tm gi tr ln nht ca biu thc


P = sin A.sin 2 B.sin 3 C .
Li gii
Ta c ln P =ln(sin A) + 2ln(sin B ) + 3ln(sin C ) .

Xt hm =
s f ( x) ln(sin x), x 0; ta c
2
f '( x) =cot x; f ''( x) =


< 0, x 0; .
sin x
2
1

V vy
f ( A) f '( M ) ( A M ) + f ( M ) =( A M ) cot M + ln(sin M )
f ( B ) f '( N ) ( B N ) + f ( N ) = ( B N ) cot N + ln(sin N )

f (C ) f '( P ) ( C P ) + f ( P ) = ( C P ) cot P + ln(sin P )

Vi M,N,P l ba gc nhn trong mt tam gic.


Khi
tan M . f ( A) + tan N . f ( B) + tan P. f (C ) ( A M + B N + C P )
+ tan M .ln(sin M ) + tan N .ln(sin N ) + tan P.ln(sin P)
= tan M .ln(sin M ) + tan N .ln(sin N ) + tan P.ln(sin P)

Do vy ta ch cn chn cc gc M,N,P sao cho


Mt khc
492

tan M tan N tan P


= = = k.
1
2
3

www.TaiLieuLuyenThi.com
Cty TNHH MTV DVVH Khang Vit

tan M + tan N + tan P =


tan M .tan N .tan P
k + 2k + 3k= 6k 3 k= 1
tan M
1
2
;sin N
;sin =
P
sin M
=
=
=
2
2
5
1 + tan M
Thay vo bt ng thc trn ta c ngay
1
2
ln P
= f ( A) + 2 f ( B) + 3 f (C ) ln
+ 2ln
+ 3ln
2
5

Du bng t ti sin
=
A sin=
M

1
;sin
=
B sin
=
N
2

.
3
10

3
27
27
= ln
P
.
5
25 5
25 5
2
;sin
=
C sin
=
P
5

3
.
10

Tng qut
Cho tam gic ABC nhn tm gi tr ln nht ca biu thc
P = sin m A.sin n B.sin n C .
Vi m,n,p l cc s thc dng.
Bi tp tng t
Cho tam gic ABC nhn, tm gi tr nh nht ca biu thc
P =tan A + 2 tan B + 3tan C .
Bi 5. Cho a,b,c l cc s thc khng m tha mn iu kin a + b + c =
3.
Tm gi tr ln nht v nh nht ca biu thc

P=

a 2 + a + 4 + b2 + b + 4 + c2 + c + 4 .
Li gii
Tm gi tr nh nht ca P

Xt hm s f ( x=
)
=
f '( x)

x 2 + x + 4 vi x 0 ta c:

2x + 1
=
; f ''( x)
2 x2 + x + 4
4

15

(x

+x+4

> 0, x 0 .

Theo bt ng thc i.nh l 1 ta c:


f (a ) f '(1) ( a 1) + f (1)
f (b) f'(1) ( b 1) + f (1)
f(c) f'(1) ( c 1) + f (1)

Cng theo v ba bt ng thc trn ta c:


=
P f (a ) + f (b) + f (c) f '(1) ( a + b + c 3) + 3 f (1)
= 3 6.

Vy gi tr nh nht ca P bng 3 6 t ti a= b= c= 1 .

493

www.TaiLieuLuyenThi.com
Khm ph t duy K thut gii bt T Bi ton Max Min ng Thnh Nam
2

2
1 15

Cch 2: Ta phn tch c tha s a + a + 4 = a + +


thnh tng ca
2 2

hai bnh phng nn ngh ngay n vic p dng bt ng thc Mincopski khi
a= b= c= 1 .
S dng bt ng thc Mincopski ta c:
2

2
2
2
1 15
1 15
1 15

P = a + +
+
b
+
+
+
c
+

+
2 2
2 2
2 2

2
3
15

a + b + c + + 3.
3 6
=
2
2

ng thc xy ra khi v ch khi a= b= c= 1 .


Cch 3: Ta c

a 2 + a + 4=

( a 1)2 + 3 ( a + 1)2

Tm gi tr ln nht ca P
Nh ti nhn xt nhng bt ng thc vi iu kin cc s thc khng m
thng thng nu khng nh gi qua cc bin i xng(du bng khng t ti
tm) th du bng ca bt ng thc thng xy ra khi c mt bin t ti bin.
Vi bi ton ny du bng xy ra khi c hai bin bng 0 khi bin cn li
bng 3.
Vy tm gi tr ln nht ca P lc ny ta cn xy dng hm f(x) c
f ''( x) 0 p dng bt ng thc ii.nh l 2.
Vi hm s lc u f ( x=
)

15

ta c f ''( x)
x 2 + x + 4=
4

(x

+x+4

> 0, x 0

vy to ra mt hm c o hm cp 2 m ta x l nh no. iu ny xut
15
pht t pht hin du bng xy ra nh trn v f ''( x)
< 1, x 0 .
4.8
Gi s du bng xy ra khi a= 3, b= c= 0 khi xt hm s nh sau:
g ( x=
)

15

Ta c: g ''( x)
x 2 + x + 4 ( x 3) x. =
4

p dng bt ng thc ii.nh l 2 ta c:


4
g (3) g (0)
g (a)
( a 3) + g (3)= ( a 3) + 4
30
3
g (3) g (0)
4
g (b)
( b 0 ) + g (0) = b + 2
30
3
494

(x

+x+4

2 < 0, x 0 .

www.TaiLieuLuyenThi.com
Cty TNHH MTV DVVH Khang Vit

g (3) g (0)
4
( c 0 ) + g (0) = c + 2
30
3
Cng theo v 3 bt ng thc trn ta c:
g (c )

P= g (a ) + g (b) + g (c) + a ( a 3) + b ( b 3) + c ( c 3)

4
8
( a + b + c 3) + 8 =
3
ng thc xy ra khi v ch khi a= 3, b= c= 0 .
g (a ) + g (b) + g (c)

Vy gi tr ln nht ca P bng 8 t ti a= 3, b= c= 0 hoc cc hon v.


Cch 2: Ngoi ra c th p dng bt ng thc

( x + y )2 + x + y + 4, x, y 0; x + y 1 .

x2 + x + 4 + y 2 + y + 4 2 +

Chng minh.
Bt ng thc cho tng ng vi

)(

x2 + y 2 + x + y + 8 + 2 x2 + x + 4 y 2 + y + 4 4 + 4 ( x + y ) + x + y + 4 + ( x + y ) + x + y + 4

(x

)(

+ x + 4 y 2 + y + 4 xy + 2

)(

x 2 + x + 4 y 2 + y + 4 x 2 y 2 + 4 xy

( x + y )2 + x + y + 4
( x + y )2 + x + y + 4 + 4 ( x + y )2 + x + y + 4

2
xy 4 ( x + y ) + x + y + 4 + x + y 7 0 (lun ng do x + y 1 ).

p dng vo bi ton
Trong ba s a,b,c lun tn ti hai s c tng khng nh hn 1 gi s hai s l
a v b khi p dng bt ng thc ph trn ta c

P2+

( a + b )2 + a + b + 4 +

c2 + c + 4 .

Mt khc a + b + c = 3 > 1 nn suy ra

( a + b )2 + a + b + 4 +

c2 + c + 4 2 +

6.
( a + b + c )2 + a + b + c + 4 =

Do P 8 . ng thc xy ra khi v ch khi a= 3, b= c= 0 hoc cc hon v.


Bi 6. Cho a,b,c l cc s thc khng m tha mn iu kin a + b + c =
1.

) (

Chng minh rng 10 a3 + b3 + c3 9 a5 + b5 + c5 1 .


Li gii
Bi ton ny bt ng thc xy ra c hai trng hp ca du bng l
1
hoc a= 1, b= c= 0 v cc hon v.
a= b= c=
3
Xt hm s f=
( x) 10 x3 9 x5 vi x [ 0;1] ta c:
495

www.TaiLieuLuyenThi.com
Khm ph t duy K thut gii bt T Bi ton Max Min ng Thnh Nam

f '( x) =
30 x 2 45 x 4 ; f ''( x) =
60 x 180 x3 =
60 x 1 3 x 2 .
1
1
Suy ra f ''( x) 0, x 0; , f ''( x) 0, x ;1 .
3

3
Vy ta cn phn chia trng hp ca cc bin, khng mt tnh tng qut gi s
abc.
1
TH1: Nu a
khi p dng bt ng thc i.nh l 1 ta c:
3

1
1
1
f (a) f ' a + f ;
3
3
3
1
1
1
f (b) f ' b + f ;
3
3
3
1
1
1
f (c) f ' c + f .
3
3
3
Cng theo v 3 bt ng thc trn ta c:
1
1
VT
= f (a ) + f (b) + f (c) f ' ( a + b + c 1) + 3 f =
1.
3

3
1
ng thc xy ra khi v ch khi a= b= c=
.
3
1
2
1
TH2: Nu a
khi nu b
th a + b + c
> 1 v l.
3
3
3

Vy a

1
b c . Khi xt hm g ( x) = 10 x3 9 x5 + 20 x 2 ( x 1) ta c
3

g ''(=
x) 180 1 x 2 0, x [ 0;1] .

p dng bt ng thc i.nh l 1 ta c:


g (a ) g '(1) ( a 1) + g (1) =
a
g (1) g (0)
b
( b 0 ) + g (0) =
1 0
g (1) g (0)
g(c)
c
( c 0 ) + g (0) =
1 0
Cng theo v 3 bt ng thc trn ta c:
g (b)

VT= g (a ) + g (b) + g (c) a 2 ( a 1) b 2 ( b 1) c 2 ( c 1)


1
g (a ) + g (b) + g (c) a + b + c =

ng thc xy ra khi v ch khi a= 1, b= c= 0 .


Bi ton c chng minh.
496

www.TaiLieuLuyenThi.com
Cty TNHH MTV DVVH Khang Vit

1
hoc a= 1, b= c= 0 hoc cc hon v.
3
1
1
1
1
.
Bi 7. Cho a, b, c > 2 tho mn iu kin 2
+ 2
+ 2
=
a 4 b 4 c 4 7
1
1
1
3
Chng minh rng
+
+
.
a+2 b+2 c+2 7
Li gii
1
9
1
Vi mi t > 2 ta c

+ .
t + 2 10 t 2 4 10

ng thc xy ra khi v ch khi a= b= c=

Tht vy bt ng thc tng ng vi:

( t 5 )2

10 t 2 4

0.

p dng bt ng thc trn ta c


1
1
1
9 1
1
1 3 3
+
+
2
+ 2
+ 2
+ =.
a + 2 b + 2 c + 2 10 a 4 b 4 c 4 10 7

Bt ng thc c chng minh. ng thc xy ra khi v ch khi a= b= c= 5 .


Ch . Bt ng thc trn c th chng minh bng BT Chebyshev.
C. BI TP RN LUYN
Bi 1. Cho x,y,z l cc s thc dng tho mn iu kin
x 4, y 9, x + y + z 49 .
Chng minh rng

1
1
1
+
+
1.
x
y
z

Bi 2. Cho x,y,z l cc s thc dng tho mn iu kin x 1, y 2, x + y + z 6 .


Chng minh rng ( x + 1)( y + 1)( z + 1) 4 xyz .
Bi 3. Cho a,b,c l cc s thc dng tha mn a 2 + b 2 + c 2 =
1.
3 3
.
2
b +c
c +a
a +b
Bi 4. Cho a,b,c l cc s thc khng m tha mn iu kin a + b + c =
1.
Tm gi tr ln nht v nh nht ca biu thc

Chng minh rng

P=

a 2 + 11a + 4 + b 2 + 11b + 4 + c 2 + 11c + 4 .

Bi 5. Cho a,b,c l cc s thc dng tho mn iu kin a 2 + b 2 + c 2 =


1.
Chng minh rng

1
1
1
3 3+9
.
+
+

1 a 1 b 1 c
2
497

www.TaiLieuLuyenThi.com
Khm ph t duy K thut gii bt T Bi ton Max Min ng Thnh Nam

Bi 6. Cho a,b,c l cc s thc dng c tch bng 1 chng minh


a
b
c
3 2
.
+
+

2
1+ a
1+ b
1+ c

Bi 7. Cho a,b,c l cc s thc dng tho mn iu kin a + b + c

)(

)(

Chng minh rng 1 + a 2 1 + b 2 1 + c 2

125
.
64

Bi 8. Cho a,b,c l cc s thc dng tho mn iu kin a + b + c


Tm gi tr ln nht ca biu thc

)(

3
.
2

)(

9
.
4

P =a + a 2 + 1 b + b 2 + 1 c + c 2 + 1 .

Bi 9. Cho x,y,z l cc s thc dng tho mn iu kin x + y + z =


3.
Chng minh rng

4+ x 4+ y 4+ z
+
+
5.
4 x
4 y
4 z

Bi 10. Cho a,b,c l di ba cnh mt tam gic chng minh


1 1 1
9
1
1
1
+ + +
4
+
+
.
a b c a+b+c
a+b b+c c+a

Bi 11. Cho a,b,c l cc s thc dng tho mn iu kin a + b + c =


3.
1
1
1
Chng minh rng 2 + 2 + 2 a 2 + b 2 + c 2 .
a
b
c
Bi 12. Cho a,b,c l cc s thc dng tho mn iu kin a 4 + b 4 + c 4 =
3.
1
1
1
Chng minh rng
+
+
1.
4 ab 4 bc 4 ca
Bi 13. Cho a,b,c l cc s thc dng. Chng minh

( 3a + b + c )3 + ( 3b + c + a )3 + ( 3c + a + b )3 375 .
3
3
3
11
3a3 + ( b + c )
3b3 + ( c + a )
3c3 + ( a + b )
Bi 14. Cho a,b,c l cc s thc dng. Chng minh

3+ 3 2
1 1 1
a + b2 + c2 + + a + b + c + a 2 + b2 + c2 .
9
a b c
Bi 15. Cho a,b,c l cc s thc dng. Chng minh

3( a + b + c )
a
b
c
+
+

.
2
b+c
c+a
a+b

498

www.TaiLieuLuyenThi.com
Cty TNHH MTV DVVH Khang Vit

D. HNG DN GII P S
Bi 1. Xt hm s=
''(t )
f (t ) 1 / t , t > 0 ta c f=

3
4t

> 0, t > 0 .

Do ta c
1
x

1
1 x 1
+ =
f +
y
z 2 4 3

y 1
f +
9 6

z
f
36

216
1 x 1 y 1 z
f . + . + . =
27 x + 8 y + z
2 4 3 9 6 36
=

216
26 x + 7 y + ( x + y + z )

216
= 1
26.4 + 7.9 + 49

Bt ng thc c chng minh.


ng thc xy ra khi v ch khi=
x 4,=
y 9,=
z 36 .
Bi 2. Bt ng thc cho tng ng vi
1
1
1
ln 1 + + ln 1 + + ln 1 + ln 4 .
x
y
z

1
2t + 1
1
Xt hm s f (t ) = ln 1 + , t > 0 ta c f '(t ) = 2 ; f ''(t ) =
> 0, t > 0 .
2
2
t
t +t
t +t

( )

Do s dng bt ng thc tip tuyn ta c


1
1
( x 1) + ln 2
ln 1 + f '(1) ( x 1) + f (1) =
2
x

1
1
3
( y 2 ) + ln
ln 1 + f '(2) ( y 2 ) + f (2) =
y
6
2

1
4
1
( z 3) + ln
ln 1 + f '(3) ( z 3) + f (3) =
z
12
3

Cng theo v ba bt ng thc trn ta c


1
1
1
1
1
1
ln 1 + + ln 1 + + ln 1 + ln 4 ( x 1) ( y 2 ) ( z 3)
x
y
z
2
6
12

6 x + 2 y + z 13
=
ln 4
+
12

4
5 x + y + ( x + y + z ) 13
= ln 4
+ ln 4
12
4
Bt ng thc c chng minh. ng thc xy ra khi v ch khi=
x 1,=
y 2,=
z 3.

499

www.TaiLieuLuyenThi.com
Khm ph t duy K thut gii bt T Bi ton Max Min ng Thnh Nam

Bi 3. Vit li bt ng thc di dng:

a
1 a

Ta c:

b
1 b

c
1 c

3 2
.
2

a
a
a
3 2a
.
=
=

2
2
2
2
b +c
1 a
a 1 a

3 2b 2
c
3 2c 2
.
;

2
2
c2 + a2
a 2 + b2
Cng theo v 3 bt ng thc trn ta c:
b

Tng t ta c:

a
b +c
2

b
c +a
2

c
a +b
2

3 2 2
3 2
a + b2 + c2 = .
2
2

ng thc xy ra khi v ch khi a= b= c=

1
3

Bi 4. Tm gi tr nh nht ca P
Xt hm s f ( x) =

Ta c f ''( x)
x 2 + 11x + 4 vi x 0.=

105

4 x + 11x + 4
2

p dng bt ng thc i.nh l 1 ta c:


1
1
1
f (a ) f ' a + f
3
3
3
1
1
1
f (b) f ' b + f
3
3
3
1
1
1
f (b) f ' c + f
3
3
3
Cng theo v 3 bt ng thc trn ta c:
1
1
=
P f (a ) + f (b) + f (c) f ' ( a + b + c 1) + 3 f =
2 10 .
3
3
1
Du bng xy ra khi v ch khi a= b= c=
.
3
Tm gi tr ln nht ca P

Xt hm s g ( x=
)

x 2 + 11x + 4 2 x ( x 1) vi x 0 ta c:

105

=
g ''( x)
4

(x

+ 11x + 4

4 < 0, x 0 .

p dng bt ng thc ii.nh l 2 ta c:

500

> 0, x 0 .

www.TaiLieuLuyenThi.com
Cty TNHH MTV DVVH Khang Vit

g (1) g (0)
( a 1) + g (1)= 2 ( a 1) + 4
1 0
g (1) g (0)
g (b)
( b 0 ) + g (0) = 2b + 2
1 0
g (1) g (0)
g (c )
( c 0 ) + g (0) = 2c + 2
1 0
Cng theo v 3 bt ng thc trn ta c:
g (a)

P= g (a ) + g (b) + g (c) + 2a ( a 1) + 2b ( b 1) + 2c ( c 1)
g (a ) + g (b) + g (c) 2 ( a + b + c 1) + 8 =
8

ng thc xy ra khi v ch khi a= 1, b= c= 0 .


Vy gi tr ln nht ca P bng 8 t ti a= 1, b= c= 0 hoc cc hon v.
Bi 5. HD: Chng minh
Bi 6. HD: Chng minh

1
9+6 3 2 3

x + , x ( 0;1) .
1 x
4
4

x
3 2
2

x
, x > 0 .
4
4
1+ x

Bi 7. HD: Chng minh ln 1 + x 2

4
5 2
3
x + ln , x 0; .
5
4 5
2

Bi ton tng qut xem chng 4.


Bi 8. Ly Logarit t nhin P ta c

) (
) (
Xt hm s f ( x) = ln ( x + x + 1 ) , x > 0 ta c

ln P= ln a + a 2 + 1 + ln b + b 2 + 1 + ln c + c 2 + 1 .
2

f '( x) =

x2 + 1

; f ''( x) =

( x + 1)
2

3
3
Do f ( x) f ' x +
4
4
T suy ra

) (

x2 + 1

< 0, x > 0 .

3
3 4
f = x + ln 2 .
5
4 5

) (

ln P= ln a + a 2 + 1 + ln b + b 2 + 1 + ln c + c 2 + 1

4
9
( a + b + c ) + 3ln 2 3ln 2
5
5
P8
3
th P bng 8. Vy gi tr ln nht ca P bng 8.
Vi a= b= c=
4

501

www.TaiLieuLuyenThi.com
Khm ph t duy K thut gii bt T Bi ton Max Min ng Thnh Nam

Bi 9. Phng trnh tip tuyn ca hm s y =


=
y

4+ x
ti im x = 1 l
4 x

13
17
x+ .
18
18

4 + x 13
17
x+ .
4 x 18
18
Tht vy bt ng thc tng ng vi:

Ta i chng minh:

17
2
13
4 + t 4 t 2 t 2 + , t = x 0; 3 ( t 1) 13t 2 + 26t + 4 0 .
18
18
Ta c iu phi chng minh.
Xy dng hai bt ng thc tng t cho y,z ri cng li theo v ta c pcm.
Bi 10. Bt ng thc c dng thun nht chun ho a + b + c =
1.
1
4
1
4
1
4
Bi ton a v chng minh
+
+
9 .
a 1 a b 1 b c 1 c
1
Ch do a,b,c l di ba cnh mt tam gic nn a, b, c 0; .
2
1
1
4
Tip tuyn ca hm s y=
ti im x = l y= 3 18 x .

3
x 1 x

Vy ta chng minh:

1
4
1

3 18 x, x 0; .
x 1 x
2

Tht vy bt ng thc tng ng vi:

( 3x 1)2 (1 2 x ) 0
x (1 x )

lun ng.

1
4
1
4
1
4

3 18a;
3 18b;
3 18c .
a 1 a
b 1 b
c 1 c
Cng li theo v ba bt ng thc trn ta c pcm.
ng thc xy ra khi v ch khi a= b= c .

Do

CH 6: K THUT KHO ST HM NHIU BIN


A. NI DUNG PHNG PHP
Bi ton. Tm gi tr ln nht v nh nht ca biu thc P ( x, y, z ) tha mn iu
kin K cho trc.
Xut pht t gi thit bi ton tm c mi lin h(iu kin x so vi y,z) gia
bin x vi hai bin cn li. Coi f ( x) = P ( x, y, z ) l hm s vi bin x v y,z l
tham s. Kho st hm s vi bin x tm ra gi tr nh nht, ln nht ca f(x) chng
502

www.TaiLieuLuyenThi.com
Cty TNHH MTV DVVH Khang Vit

hn t ti x = x0 . Khi tip tc kho st hm s g ( y ) = P ( x0 , y, z ) v tm ra gi


tr ln nht, nh nht ca g(y) t suy ra kt qu bi ton.
Cc im cn lu
- Bi ton x l c bng phng php ny thng cho mi rng buc cc bin
cng thuc on [ a; b ] cho trc v du bng ca bt ng thc t ti bin.
- Vi bi ton c hai bin s x,y ta thc hin tng t.
- Nu bt ng thc c dng i xng 3 bin ta thng gi s x y z thun
tin cho vic nh gi tnh n iu ca hm s.
- nh gi cui cng l hm mt bin ta c th kho st hm s hoc bin i
tng ng c nhanh kt qu.
Mt bi ton s dng phng php ny hay gp trong thi
Bi ton. Cho a,b,c v x,y,z l cc s thc dng tho mn iu kin
ax + by + cz =
xyz .
Tm gi tr nh nht ca biu thc P = x + y + z .
PHNG PHP
Theo gi thit ta c:
ax + by + cz = xyz z =

ax + by
ax + by
.
, ( xy > c ) P = f ( y ) = x + y +
xy c
xy c

Xt hm s f ( y ) = x + y +
f '( y ) =
1

bc + ax 2

( xy c )

ax + by
ta c
xy c

; f '( y ) =
0 ( xy c ) =
bc + ax 2
2

c + bc + ax 2
x
Ta c f(y) i du t m sang dng khi i qua y0 nn f(y) t cc tiu ti y0 .
y = y0 =

V vy
c + bc + ax 2
P f ( y0 ) =
x+
+
x
=x +
=

ax + b.

c + bc + ax 2
x
bc + ax 2

c + bc + ax 2 b
+ + bc + ax 2
x
x

x 2 + b + c + ( x + 1) bc + ax 2
x
503

www.TaiLieuLuyenThi.com
Khm ph t duy K thut gii bt T Bi ton Max Min ng Thnh Nam

Thc hin xt tnh n iu ca hm s g ( x) =

x 2 + b + c + ( x + 1) bc + ax 2
x

ta

c kt qu bi ton.
V d 1. Cho x,y,z l cc s thc dng tho mn iu kin 2 x + 4 y + 7 z =
2 xyz .
Tm gi tr nh nht ca biu thc P = x + y + z .
Li gii
Theo gi thit ta c z =

2x + 4 y
2x + 4 y
. Khi P = f ( y ) = x + y +
.
2 xy 7
2 xy 7

Xt hm s f ( y ) = x + y +
f '( y ) =
1

4 x 2 + 28

( 2 xy 7 )

7
2x + 4 y
vi y >
ta c
2x
2 xy 7

; f '( y ) =
0 ( 2 xy 7 ) =
4 x 2 + 28
2

7 + 2 x2 + 7 7
>
2x
2x
Ta c f(y) i du t m sang dng khi i qua y0 nn f(y) t cc tiu ti y0 .
y = y0 =

V vy
P f ( y0 ) =x +

7 + 2 x2 + 7
+
2x

7 + 2 x2 + 7
11 2 x 2 + 7
2x
.
=x +
+
2x
x
2 x2 + 7

2 x + 4.

11 2 x 2 + 7
vi x > 0 ta c
+
2x
x
28

Xt hm s g ( x) =x +
2 x 2 11
g '( x) =

2x

11
x
2

2
2 x 11

) (x
2

x 2 + 7 ; g '( x) =0 2 x 2 11 =

3
x=

15
15
P . ng thc xy ra khi v ch khi
2
2

5
=
,z 2 .
2

Vy gi tr nh nht ca P bng 15/2 t ti=


x 3,=
y
504

x2 + 7

0
+ 7 282 =

T suy ra g min = g (3) =


=
x 3,=
y

28

5
=
,z 2 .
2

www.TaiLieuLuyenThi.com
Cty TNHH MTV DVVH Khang Vit

Ch . S dng bt ng thc AM GM ta c:

x2 + 7 (9 + 7 )
11 2 x 2 + 7
11
x+
+
=x +
+
2x
2x
2x
x
11 3 x + 7
9 3
9 3 15
x+
+
= x + + 2 x. + =
2x
2x
x 2
x 2 2
Ta c kt qu tng t trn.
K thut c nh bin s
V d 1. Cho a,b,c l cc s thc khng m tho mn iu kin ab + bc + ca > 0 .
4a
4b
4c

Chng minh rng 1 +


1 +
1 +
25 .
b
+
c
c
+
a
a
+b

Li gii
Bt ng thc c vit di dng
( 4a + b + c )( 4b + c + a )( 4c + a + b ) 25 0 f =
ab +
(ab)
25 0 .
ab +
( a + b )( c + a )( c + b )

Trong , , , l cc hng s khi ta c nh c v a + b .


R rng f(ab) l hm lun ng bin hoc lun nghch bin do vi
( a + b )2
th f(ab) t gi tr nh nht hoc ti 0(khi c t nht mt
ab 0;
4

s bng 0) hoc ti

( a + b )2 (khi 2 s bng nhau).

4
Vy ta ch cn chng minh bt ng thc trong hai trng hp.
+ TH1: Nu c mt s bng 0 gi s l a. Ta cn chng minh
4b 4c
1 + 1 + 25 .
c
b

Bt ng thc ny l hin nhin theo CS, tht vy


2

4b 4c
4b 4c
. =
25 .
1 + 1 + 1 +
c
b
c b

ng thc xy ra khi v ch khi b = c .


+ TH2: Khi 2 s bng nhau gi s l a v b, khi ta cn chng minh.
4a
4a
4c

1 +
1 +
1 +
25
a + c a + c 2a
2

4 a 2c
4 2c

1 +
1 + 25 0
1 + 25 1 +
a
a
a+c
1+ c
a

505

www.TaiLieuLuyenThi.com
Khm ph t duy K thut gii bt T Bi ton Max Min ng Thnh Nam

t x=

c
0 ta phi chng minh
a
2

4
2

1 +
(1 + 2 x ) 25 0 x x 2 x + 5 0 x ( x 1) + 4 0 .
x +1

Bt ng thc cui lun ng.


Bt ng thc c chng minh. ng thc xy ra khi v ch khi 1
=
a b=
, c 0 hoc cc hon v.

2a
2b
2c
, y=
,=
z
xy + yz + zx + xyz= 4 .
b+c
c+a
a+b
Theo bt ng thc c chng minh ta c
x + y + z xy + yz + zx .

Cch 2: t x=

Vy bi ton a v chng minh

(1 + 2 x )(1 + 2 y )(1 + 2 z ) 25 2 xyz + x + y + z 4 ( x + y + z + xyz ) + xyz 4 .

Bt ng thc cui ng bi v x + y + z + xyz xy + yz + zx + xyz =


4.
V d 2. Cho a,b,c l cc s thc dng c tng bng 3. Chng minh
a
1 + (b + c) 2

b
1 + (a + c)2

c
1 + ( a + b) 2

3(a 2 + b 2 + c 2 )
a 2 + b 2 + c 2 + 12abc

Li gii
Bt ng thc cho tng ng vi:

3(a 2 + b 2 + c 2 )
a
b
c
3
b
c
+

2
2
2
a 2 + b 2 + c 2 + 12abc
1 + (b + c) 1 + (c + a ) 1 + (a + b)

a (b + c)2
1 + (b + c)

b (c + a ) 2
1 + (c + a )

Xt hm s y =

c ( a + b) 2
1 + (a + b)

36abc
a + b + c 2 + 12abc
2

x
2
2
2
tng do vi ( b + c ) 4bc; ( c + a ) 4ca; ( a + b ) 4ab
x +1

Ta c:
a (b + c )

1 + (b + c )

4abc b ( c + a )
4abc c ( a + b )
4abc
.
;
;

2
2
1 + 4bc 1 + ( c + a )
1 + 4ca 1 + ( a + b )
1 + 4ab
2

Do ta ch cn chng minh
4abc
4abc
4abc
36abc
36abc
.
+
+
2
=
2
2
1 + 4bc 1 + 4ca 1 + 4ab a + b + c + 12abc a 2 (1 + 4bc)
cyc

Bt ng thc cui ng bi theo C S ta c

506

www.TaiLieuLuyenThi.com
Cty TNHH MTV DVVH Khang Vit

1 + 4bc a 2 (1 + 4bc) (a + b + c)2 =9 .


cyc

cyc

Bt ng thc c chng minh. ng thc xy ra khi v ch khi a= b= c= 1 .

V d 3. Cho x,y,z l cc s thc thuc on [1;2] .

Tm gi tr ln nht v gi tr nh nht ca biu thc P =

x2 y + y 2 z + z 2 x
x4 + y 4 + z 4

Li gii
Tm gi tr ln nht
Do iu kin bi ton x, y, z [1;2] x 2 y x 2 y 2 ; y 2 z y 2 z 2 ; z 2 x z 2 x 2 .
Do P

x2 y 2 + y 2 z 2 + z 2 x2

= 1

x4 + y 4 + z 4
x4 + y 4 + z 4 x2 y 2 y 2 z 2 z 2 x2
x4 + y 4 + z 4

(x y ) + ( y z ) + (z x )
=
1
2( x + y + z )
2

2 2

2 2

2 2

ng thc xy ra khi v ch khi x= y= z= 1 .


Vy gi tr ln nht ca P bng 1 t ti x= y= z= 1 .
Tm gi tr nh nht
Vi gi thit bi ton x,y,z thuc on nn thng t cc tr ti bin. Th x,y,z
chn trong 2 s (1,2) thay vo P ta thy gi tr nh nht
7
7
x = y =1; z =2 P =
= .
2 + 16 18
7
Vy ta i chng minh P .
18
Trc tin chuyn P v biu thc i xng ca x,y,z.
Theo gi thit ta c: ( x 1)( x 2 ) 0 x 2 3 x 2; y 2 3 y 2; z 2 3z 2 .
y2 + 2
z 2 + 2 2 x2 + 2
Suy ra: x 2 y + y 2 z + z 2 x x 2
+ y2
+z

3
3
3

x2 y 2 + y 2 z 2 + z 2 x2 + 2 x2 + y 2 + z 2

Do vy ta ch cn chng minh.

507

www.TaiLieuLuyenThi.com
Khm ph t duy K thut gii bt T Bi ton Max Min ng Thnh Nam

x2 y 2 + y 2 z 2 + z 2 x2 + 2 x2 + y 2 + z 2
3

7 x +y +z
4

) 6( x

y + y2 z2

2 2

) 7

(x + y + z )
18
+ z x ) 12 ( x + y + z ) 0
2 2

cho n gin ta t a =
x2 , b =
y2 , c =
z 2 a, b, c [1;4] v bt ng thc tr

thnh: 7 a 2 + b 2 + c 2 6 ( ab + bc + ca ) 12 ( a + b + c ) 0 .
V tri k hiu l P (a, b, c) l tam thc bc hai ca a vi h s ca a2 dng
nn t max ti 1 hoc 4.
Do P (a, b, c) max { P (1, b, c); P(4, b, c)} .
Bng cch tng t P (1, b, c); P(4, b, c) l tam thc bc hai vi h s b2; c2
dng nn
P (1, b, c) max { P (1,1, c); P (1, 4, c)}

max {max { P(1,1,1); P(1,1, 4)} ;max { P(1, 4,1); P(1, 4, 4)}} =
0

V
P (4, b, c) max { P (4,1, c); P (4, 4, c)}

max {max { P(4,1,1); P(4,1, 4)} ;max { P(4, 4,1); P(4, 4, 4)}} =
0

T suy ra iu phi chng minh.


Vi x= y= 1, z= 2 th P bng 7/18. Vy gi tr nh nht ca P bng 7/18.
Nhn xt. Thc cht ta c nh gi trn da vo tnh cht ca hm li, l nu
f ''( x) 0; x [ a; b ] th f(x) t max ti a hoc ti b.
B. BI TON CHN LC
1
Bi 1. Cho a,b l hai s thc thuc on ;1 . Tm gi tr nh nht ca biu thc
4
1
a
b
.
P=
+
+
2 + 3a a + b b + 1
Li gii
1
a
b
1
Xt hm s f (a) =
trn on ;1 ta c:
+
+
2 + 3a a + b b + 1
4

b ( 2 + 3a ) 3 ( a + b )
3
b
f '(a ) =

+
= 2
> 0 v
2
2
( 2 + 3a ) ( a + b )
( 2 + 3a ) ( a + b )2
2

b ( 2 + 3a ) 3 ( a + b ) = 9a 2b + 6ab + 4b 3a 2 3b 2 9a 2b + 6a 2b + 4b 3a 2 3b 2
2

1
= 3a 2 ( 5b 1) + b ( 4 3b ) > 0, a, b ;1
4
508

www.TaiLieuLuyenThi.com
Cty TNHH MTV DVVH Khang Vit

1
Vy f(a) l hm ng bin trn on ;1 suy ra
4

( 2b 1)
4
1
b
34 34
1
.
P = f (a) f = g (b) = +
+
=
+

11 1 + 4b b + 1 3 ( b + 1)( 4b + 1) 33 33
4
2

ng thc xy ra khi v ch khi=


a

1
1
.
=
,b
4
2

34
1
1
t ti=
.
a =
,b
33
4
2
34
thc cht ta xt hm mt bin vi b tm ra
Nhn xt. c bin i g (b)
33
1
g(b) t cc tiu ti b = .
2

Vy gi tr nh nht ca P bng

Bi 2. Cho x,y l hai s thc thuc on [1;4] . Tm gi tr ln nht ca biu thc


P= x y+

y 1 1 x
.
+
x
y

Li gii
y 1 1 x
Ta c: x =
y+
+
x
y

y 1)( x 1)
( x y )( =
( x y )( y 1)( x 1)
P
xy

xy

Do P l biu thc i xng vi x v y nn khng mt tnh tng qut ta gi s


y )( x 1)( y 1) y 1 x 2 ( y + 1) x + y
( x =
=
x yP

.
xy
y
x

2
y 1 x ( y + 1) x + y
Xt hm s f ( x) =

lin tc trn on [1;4] ta c:


y
x

y 1
y
f=
'( x)
1 =

y x2

( y 1) ( x 2 y )
x2 y

0, x y 1 .

Do f(x) l hm ng bin trn on [1;4] .


Suy ra P = f ( x) f (4) = g ( y ) =
Xt hm s g ( y ) =

( y 1)(12 3 y ) .
4y

( y 1)(12 3 y ) lin tc trn on


4y

[1;4] ta c:

509

www.TaiLieuLuyenThi.com
Khm ph t duy K thut gii bt T Bi ton Max Min ng Thnh Nam

12 3 y 2
g '( y ) =
; g '( y ) =0 y =2 [1;4] . Ta c g(y) i du t dng sang
4 y2

3
.
m khi i qua y = 2 nn g(y) t cc i ti y = 2 . Do P g ( y ) g (2) =
4
ng thc xy ra khi v ch khi=
x 4,=
y 2.
3
t ti=
x 2,=
y 4 hoc=
x 4,=
y 2.
4
Bi 3. Cho hai s thc x 1 y > 0 . Tm gi tr nh nht ca biu thc

Vy gi tr ln nht ca P bng

(x
A=

+ y2

xy + x + y

xy ( x + y + 1)

Li gii
y l biu thc i xng vi tng x + y v tch xy nn suy ngh ngay n vic
t S =+
x y, P =
xy t iu kin (1 x )(1 y ) 0 x + y 1 xy ta c ngay
0 < P S 1 .

(S
Ta c: A =

2P

S+P

P ( S + 1)

Coi v phi l hm s vi P v tham s S ta c:


f '( P ) =

2S 3 + S 2 P + 2 P 2
2 ( S + 1) P 2 S + P

< 0, S , P > 0 do f(P) l hm nghch bin trn

( 0; S 1] .
Do

(S
f ( P ) f ( S 1) =

(S
Xt hm s g ( S ) =
g '( S )

2S + 2

2S + 2

S 2 1

S 2 1

S 4 + 2 S 3 13S 2 + 12 S 4
=
2
2S 1 S 2 1

2S 1

2S 1

.
trn (1;+ ) ta c:

( S 2 ) ( S 3 + 4 S 2 5S + 2 )

2S 1 S 2 1

S >1

g '( S=
=
S 2.
) 0

Ta c g(S) i du t m sang dng khi i qua S = 2 nn g(S) t cc tiu ti


2
S = 2 hay g ( S ) g (2) =.
3
2
t ti x= y= 1 .
Vy gi tr nh nht ca A bng
3
510

www.TaiLieuLuyenThi.com
Cty TNHH MTV DVVH Khang Vit

Bi 4. Cho x,y,z l cc s thc tha mn 1 x y z 4 .


1 1 1
Tm gi tr ln nht ca biu thc P = ( x + y + z ) + + .
x y z
Li gii
1 1 1
Xt hm s f ( x) = ( x + y + z ) + + trn [1;4] ta c:
x y z

2
1
1 1 1 x+ y+z
1 x yz ( y + z )
f '( x) = + +
= ( y + z ) 2 =
0.
x y z
x2
x 2 yz
yz x

Do f(x) l hm nghch bin trn [1;4] suy ra f ( x) f (1) hay


1 1
P = f ( x) f (1) = g ( y ) = (1 + y + z ) 1 + + .
y z

1 1
Xt hm s g ( y ) = (1 + y + z ) 1 + + trn on [1;4] ta c:
y z

g '( y ) =1 +

1 1 1 + y + z z + 1 1 + z ( y z )( z + 1)
+
=
2 =
0.
y z
yz
y2
y
y2 z

Do g(y) l hm nghch bin trn on [1;4] suy ra


1
2

P g ( y ) g(1) =( 2 + z ) 2 + =h( z ) =5 + 2 z + .
z
z

2
Xt hm s h( z ) =5 + 2 z + trn on [1;4] ta c:
z

2 z2 1
h '( z ) =2 2 = 2
0 nn h(z) l hm ng bin trn on [1;4] .
z
z
27
.
Do P h( z ) h(4) =
2
ng thc xy ra khi v ch khi x= y= 1, z= 4 .
2

27
t ti x= y= 1, z= 4 .
2
Bi 5. Cho a,b,c l cc s thc dng tho mn iu kin abc + a + c =
b.
2
2
3
.
Tm gi tr ln nht ca biu thc P = 2
2
+ 2
a +1 b +1 c +1
Li gii
1
a+c
Theo gi thit ta c b (1 ac ) = a + c > 0 1 ac > 0 a < ; b =
.
c
1 ac

Vy gi tr ln nht ca P bng

511

www.TaiLieuLuyenThi.com
Khm ph t duy K thut gii bt T Bi ton Max Min ng Thnh Nam

Khi P =

a +1 a + c
c +1

+1
1 ac
2

2 a 2 + 2ac + 2c 2 + 1

Xt
hm s f (a )
=

( a + 1)( c + 1)
2

3
c +1
2

( a + 1)( c + 1) c
4c ( a + 2ac 1)
f '(a ) =
; f '(a ) = 0 a =
( a + 1) ( c + 1)
2

2 a 2 + 2ac + 2c 2 + 1

3
+1

2 ta c

c2 + 1 c .

Ta c f(a) i du t dng sang m khi i qua


V vy f (a ) f

2c

Xt hm s
=
g (c )

g '(c) =

c2 + 1

2 1 8c 2

2c

c 2 + 1=
c

c2 + 1

c2 + 1

3
c +1
2

3c + c 2 + 1

3
c +1
2

c2 + 1 c .

vi c dng ta c

; g '(c) = 0 c =

1
2 2

Ta c g(c) i du t dng sang m khi i qua c =

, ( c > 0) .

1
2 2

1 10
.
V vy g (c) g
=
2 2 3

ng thc xy ra khi v ch khi c=

, a=

2 2
Vy gi tr ln nht ca P bng 10/3 t ti

1
, b=
2

2.

1
, b= 2 .
2 2
2
Nhn xt. Ta c th gii bi ton bng lng gic xem chng sau.
Bi 6. Cho a,b,c l cc s thc khng m tha mn a + b + c =
3.
c=

1
1
+1
=
8
2 2

, a=

1
1
+1
=
8
2 2

Tm gi tr nh nht ca biu thc P =a + b 2 + c3 .


Li gii
Thay a = 3 b c vo biu thc ca P ta c:
2

1
11
11

P = b 2 b + c 3 c + 3 = b + c 3 c + f (c ) = c 3 c + .
2
4
4

512

2.

www.TaiLieuLuyenThi.com
Cty TNHH MTV DVVH Khang Vit

11
vi c [ 0;3] ta c:
4
1
f '(c) = 3c 2 1; f '(c) = 0 c =
[ 0;3] .
3

Xt hm s f (c) = c3 c +

Ta c f(c) i du t m sang dng khi i qua c =


c=

1
3

1
nn f(c) t cc tiu ti
3

1 11 2 3
Suy ra P f (c) f
= 9 .
3 4
5 1
1
1
ng thc xy ra khi v ch khi a =

,b =
,c =.
2
2
3
3
5 1
1
1
11 5 3
t ti a =

,b =
,c =.

2
2
4
9
3
3
Bi 7. Cho a, b, c l di ba cnh mt tam gic. Chng minh rng

Vy gi tr nh nht ca P bng

a b c
a c b
3 + + 2 + + + 3 .
b c a
c b a
Li gii
Khng mt tnh tng qut gi s a = max {a, b, c}

- Nu a c b th
a b c a b c
a b c a b c
a b c
+ + + + 3 3 + + 3 + + 2 + + + 3
b c a c a b
b c a c a b
c a b
- Nu a b c th
a b c
a c b
Xt hm s f (a )= 3 + + 2 + + 3
b c a
c b a

Ta c f '(a ) =

a 2 bc ( 3c 2a )
3 3c 2 2b
2 + 2 =
b a
c a
a 2bc

Do a b c a 2 bc
Kh nng 1. Nu 3c > 2a hm s ng bin suy ra f (a ) f (b) =

b c
+ 20
c a

Kh nng 2. Nu 3c < 2a hm s nghch bin suy ra


f (a ) f (b =
+ c)

( b c )2
bc

3c 2b 2 ( b 2c ) + c ( b c )
+ =
0
b+c
( b + c )2
2

Bt ng thc c chng minh.


513

www.TaiLieuLuyenThi.com
Khm ph t duy K thut gii bt T Bi ton Max Min ng Thnh Nam

Bi 8. (TSH Khi A 2011) Cho x,y,z l ba s thc thuc on [1;4] v


x y, x z .

P=

Tm gi tr nh nht ca biu thc

x
y
z
.
+
+
2x + 3y y + z z + x

Li gii
x
y
z
Xt hm s f ( z )=
trn on [1;4] ta c:
+
+
2x + 3y y + z z + x

( x y ) z 2 xy
y
x
.
f '(z) =

+
=
( y + z )2 ( x + z )2 ( x + z )2 ( y + z )2
1
y
z
6
+ Nu x = y P = +
+
= .
5 y+z z+ y 5

+ Nu x > y khi f '(z) = 0 z = xy ta c f(z) i du t m sang dng khi


i qua z = xy nn f(z) t cc tiu ti z = xy .
x
Do P =
f (z) f( xy ) =
+
2x + 3y

t
=
t

x
2 y
y
=
+
x
x + y 2 +3
y

2
.
x
+1
y

x
, (1 < t 2 ) ta c
y

t2

2
P g=
(t)
+ =
2
2t + 3 t + 1

( 2 t ) ( 35t 2 27t + 48) 34 34


+
.
33 33
33 ( t + 1) ( 2t 2 + 3)

ng thc xy ra khi v ch khi


z = xy , t = 2 x = 4 y, z = 2 y x = 4, y = 1, z = 2 .

34
t ti =
x 4,=
y 1,=
z 2.
33
Nhn xt. Do P dng ng cp bc 0 nn ta c th t
=
y a=
.x, z b.x,

Vy gi tr nh nht ca P bng

1
a, b ;1 a bi ton v tm gi tr nh nht ca biu thc
4
1
a
b
c trnh by trn(xem thm ch s dng bt
P=
+
+
2 + 3a a + b b + 1
ng thc ph).

514

www.TaiLieuLuyenThi.com
Cty TNHH MTV DVVH Khang Vit

1
Bi 9. Cho cc s thc a, b, c ;3 .
3

a
b
c
.
+
+
a+b b+c c+a
Li gii

Tm gi tr ln nht ca biu thc S =

t f ( a ) =

a
b
c
. Xt hai trng hp sau:
+
+
a+b b+c c+a

TH1: a b c . Ta c f ( a ) =
Suy ra f ( a ) f ( 3) =
Mt khc g ( c ) =

( b c ) ( a 2 bc )

( a + b )2 ( a + c )2 ( a + b )2 ( a + c )2

3
b
c
+
+
= g (c) .
3+b b + c c +3

( b 3) ( 3b c 2 )

( c + b )2 ( c + 3)2 ( c + 3)2 ( b + c )2

3
3b
1
1
Suy ra g ( c ) g =
+
+ = h (b)
3 3 + b 3b + 1 10

Ta c h ( b ) =

( 3b + 1)

( b + 3)

(1 b )(1 + b ) 0

( 3b + 1)( b + 3)

1 8

T bng bin thin suy ra f ( a; b; c ) f 3;1; =


.
3 5

8
TH2: c b a . T TH1 ta c f ( c; b; a ) .
5

Mt khc f ( a; b; c ) =
f ( c; b; a )

( a b )( b a )( a c ) 0 .
( a + b )( b + a )( a + c )

8
Suy ra f ( a; b; c ) .
5
8

1 1
1
Vy max S = , t c khi v ch khi ( a, b, c ) 3,1, , ,3,1 , 1, ,3 .
5
3 3
3

Bi 10. Cho a, b, c [ 0;1] . Tm gi tr ln nht ca biu thc


S=

a
b +c +6

c + a + 6 a + b3 + 6
Li gii
a
b
c
t f ( c ) = 3 3
.
+ 3
+ 3
3
b + c + 6 c + a + 6 a + b3 + 6
3

515

www.TaiLieuLuyenThi.com
Khm ph t duy K thut gii bt T Bi ton Max Min ng Thnh Nam

1
3ac 2
Ta c f ( c ) =3

a + b3 + 6 b3 + c 3 + 6

3c 2

) ( a + c + 6)
6ac ( 6 + b 2c ) 6bc ( 6 + a 2c )

0
f ( c ) =
(b + c + 6)
( a + c + 6)
3

3b

Nn f(c) gim trn [0; 1]. Suy ra


f ( c )=
f (1)

1
6+a +b
3

3a

(7 + b ) (7 + a )
3 2

3 2

1
3
2. > 0
8
49

Suy ra f(c) tng trn [0; 1].


a

Do S = f ( c ) f (1) =

b +7

1
2a 2 b
Ta c g ( a ) =3

b + 7 a3 + 7

a +7
3

a + b3 + 6
3

= g (a) .

3a 2

) (a + b + 7)
6ab ( 7 2a ) 6a ( b + 6 2a )

0
g ( a ) =
(a + 7)
(a + b + 7)
2

Nn g(a) gim trn [0; 1]. Suy ra


1

g ( a ) g (1=
)

b +7
3

3b
3
1 5
3 5 3b
1

=
3
>0
3
+
2
64 7 + b3
64
b + 7 8 8 b + 7

Suy ra g(a) tng trn [0; 1]. Do =


S g ( a ) g (1=
)
1
6b 2
Ta c h ( b ) =
8 b3 + 7

(b

+7

48b 2

8 b3 + 7

b
+ = h(b) .
b +7 8
3

> 0, b [ 0;1] .

3
3
Suy ra h(b) tng trn [0; 1], nn h ( b ) h (1) = S .
8
8

Vi a = b = c = 1 th max S =

516

3
8

www.TaiLieuLuyenThi.com
Cty TNHH MTV DVVH Khang Vit

Bi 11. Xt cc s thc dng x, y, z tha mn h iu kin


2
5 z min { x, y} (1)

( 2)
xz
15

( 3)
yz 5

Hy tm gi tr ln nht ca biu thc P ( x, y, z ) =

1 2 3
+ + .
x y z

Li gii
4
T iu kin (1) v (2) suy ra x max z ,
(4)
15 z
2
1 1
a) Xt hm s f ( x )=
+ vi x > 0 v tham s z . Xt hai trng hp
5
x z
2
4
1 1 2
th x z
theo (4) nn f ( x ) + = 15 (5)
Nu z
15 z
z z z
15

Nu

2
2
4
15 z 1
th x
z
z theo (4) nn f ( x )
+ =
g ( z) .
5
15 z
4
z
15

Xt hm s g(z) vi
g( z ) =

2
2
. Ta c
z
5
15

15 1
2
.
2 <0 z<
4 z
15

2
Do g(z) l hm gim v f ( z ) g ( z ) g =
4 (6)
5
So snh (5) v (6) ta c
1 1
1 1
2
2
(7)
+ 4 v
+ =4 x = ,z =
x z
x z
3
5
2
1 1
b) Xt hm s h ( y =
) + vi tham s z
5
y z

1
T iu kin (1) v (3) suy ra y max z ,
5z
Lp lun tng t phn a) ta c
1
th h ( y ) 2 5 (9)
Nu z
5

(8)

517

www.TaiLieuLuyenThi.com
Khm ph t duy K thut gii bt T Bi ton Max Min ng Thnh Nam

2
1
9
th h ( y )
(10)
z
5
2
5
So snh (9) v (10) ta c
1 1 9
2
1
1 1 9
+ = x= ,y=
+ v
x z 2
5
2
y z 2

Nu

(11)

So snh kt qu phn a) v b) ta c
1 1
1 1
9
P ( x, y, z ) = + + 2 + 4 + 2. = 13
2
x y
y z
2
1
2
ng thc xy ra khi v ch khi=
x =
,y =
,z
3
2
5
Vy gi tr ln nht ca P bng 13.
1
Bi 12. Cho x,y,z l cc s thc thuc on ;2 . Tm gi tr nh nht ca biu
2

x y z y x z
thc P= 8 + + 5 + + .
y z x x z y
Li gii
Nhn xt. Ta th o hm P theo bin x ta c:

1 z y 1 8 x 2 z 8 z 2 y + 5 yz 5 x 2 y
2 5 2 +
=
8 =
z
x 2 yz
y x x

(x

yz ( 8 z 5 y )
x 2 yz

P min suy ngh n o hm khng m. Vy trc tin cn c x 2 yz ta ch


cn gi s x = max { x, y, z} .
Khng mt tnh tng qut gi s x = max { x, y, z} .
Vit li P di dng: P= f ( x)=

Ta c

(x
f '( x) =

yz ( 8 z 5 y )
x 2 yz

8z 5 y
8z 5 y 8 y 5z
vi
.x +
+

yz
x
z
y

yz x 2

TH1: Nu 8 z 5 y > 0 f '( x) 0 nn f(x) l hm ng bin trn yz ;2 do


f ( x) f ( yz ) =

t
=
t

518

8 y 5z
z
y
.
+ 16
10
z
y
y
z

5 16
y 1
, t ;2 . Khi P = f ( x) f ( yz ) = g (t ) = 8t 2 2 + 10t .
t
z 2
t

www.TaiLieuLuyenThi.com
Cty TNHH MTV DVVH Khang Vit

Xt hm s g (t ) = 8t 2

5
t

16
1
10t lin tc trn ;2 ta c:
t
2

( t 1)(8t 5) ( t 2 + t + 1)

10 16
) 16t + 3 2 10
g '(t=
=
t
t

t3

t = 1
; g '(t=
) 0
5 .
t =
8
1
t ;2
2

75
1
5 387
.
Ta c g =
(1) 9, g =
, g=
(2)
g=

4
2
8 40
1
Suy ra min g (t ) = 9 xy ra ti t = 1 hoc t = hay x= y= z hoc
=
z 4=
y, x 2 y .
2
TH2: Nu 8 y 5 z 0 f '( x) 0 suy ra

2 y z y z 2
f ( x) f (2)= 8 + + 5 + + = g ( y ) .
y z 2 2 y z
Thc hin tng t.
1
Bi 13. Cho a,b,c l cc s thc thuc on ;1 . Tm gi tr ln nht ca biu
2
a b bc ca
.
thc P =
+
+
c
a
b

Li gii
Ta c:

P=

a b b c c a ab ( a b ) + bc ( b c ) + ca ( c a )
+
+
=
=
c
a
b
abc

( a b )( b c )( c a )
abc

Do P l biu thc i xng vi ba bin a,b,c nn khng mt tnh tng qut gi


s a b c . Khi :
P

a b )( b c )( a c )
(=
abc

Xt hm s f (a ) =

2
b c a ( b + c ) a + bc

.
bc
a

2
1
b c a ( b + c ) a + bc

vi c b a 1 ta c:
2
bc
a

b c bc
f '(a=
)
1 =

bc a 2

( b c ) ( a 2 bc )
a 2bc

0.

1
Do f(a) l hm ng bin trn ;1 .
2

Suy ra P
= f (a ) f (1)
=

bc
1 b
1
1
+ c 1 + b .
(1 b c + bc=) g (c=)
bc
c
b
b
519

www.TaiLieuLuyenThi.com
Khm ph t duy K thut gii bt T Bi ton Max Min ng Thnh Nam

Xt hm s g (c=
)
g '(c) =

1 b
c

1 b
1
1
1
+ c 1 + b trn on ;1 ta c:
c
b
b
2

2
1
1 1 (1 b ) c b (1 b )( c b )
+ 1 =(1 b ) 2 =

0.
b
bc 2
bc 2
b c

1
Do g(c) l hm nghch bin trn ;1 .
2
3
1
1
.
Suy ra P g (c) g = h(b) = b
2
2b
2

3
1
1
lin tc trn on ;1 ta c:
b
2
2b
2
1
1
h '(b) =1 + 2 ; h '(b) =0 b = .
2
2b
1
nn h(b) t cc i ti
Ta c h(b) i du t dng sang m khi i qua b =
2

Xt hm s h(b) =

1
1 3 2 2
. Do P h(b) h
.
=
2
2
2
1
1
ng thc xy ra khi v ch khi =
v cc hon v.
a 1,=
b
=
,c
2
2
b=

1
1
3 2 2
a 1,=
b
=
,c
t ti =
v cc hon v.
2
2
2
Nhn xt. im quan trng trong li gii trn l pht hin ng thc
( a b )( b c )( c a ) .
a b bc ca
+
+
=

c
a
b
abc
Ngoi ra d thy P l biu thc ng cp bc 0 v dng i xng nn khng mt
tnh tng qut ta c th gi s a b c . Ta t =
a x.c,=
b y.c, ( 2 x y 1) .
Vy gi tr ln nht ca P bng

Bi ton a v tm gi tr ln nht ca biu thc P =

( x 1)( y 1)( x y ) .
xy

y chnh l bi ton 2 ti trnh by.


Bi 14. Cho a,b,c l cc s thc thuc on [1;2] v 4a + 2b + c =
11 .
Tm gi tr ln nht v nh nht ca biu thc P =

1 2 3
+ + .
a b c

Li gii
Theo gi thit ta c 4a =11 2b c suy ra P= f (b=
)
520

4
2 3
+ + .
11 2b c b c

www.TaiLieuLuyenThi.com
Cty TNHH MTV DVVH Khang Vit

Xt hm s
=
f (b)

4
2 3
+ + lin tc trn [1;2] ta c
11 2b c b c

2
2 8b 2 (11 2b c ) 2 ( 4b + c 11)(11 c )
=
f '(b)
=

=
< 0, b, c [1;2] .
2
b 2 (11 2b c )
(11 2b c )2 b2 b2 (11 2b c )2
2

Do f(b) l hm nghch bin trn on [1;2] suy ra f (2) f (b) f (1) .


Tm gi tr ln nht ca P.
Xt hm s g (c)= f(1)=
=
g '(c)

4
3
+ 2 + ta c:
9c
c

3 c 2 + 54c 243
=

< 0, c [1;2] .
c2 ( 9 c )
( 9 c )2 c 2
4

Vy g(c) l hm nghch bin trn on [1;2] P= f (b) f (1)= g(c) g(1)=

11
.
2

ng thc xy ra khi v ch khi b= c= 1, a= 2 .


Vy gi tr ln nht ca P bng

11
t ti a= 2, b= c= 1 .
2

Tm gi tr nh nht ca P.
Xt hm s h(=
c) f (2)
=
=
h '(c)

4
3
+ 1 + ta c:
7c
c

3 c 2 + 42c 147
=

< 0, c [1;2] .
2
c2 ( 7 c )
( 7 c )2 c 2
4

Do h(c) l lm nghch bin trn on [1;2] .


Suy ra P = f (b) f (2) = h(c) h(2) =

33
.
10

5
.
4
33
5
t ti a=
Vy gi tr nh nht ca P bng
, b= c= 2 .
10
4
Bi tp tng t

ng thc xy ra khi v ch khi b= c= 2, a=

Cho a,b,c l cc s thc thuc on [1;3] tha mn a + 2b + 3c =


12 .

3 2 1
+ + .
a b c
Bi 15. Cho a,b,c l cc s thc thuc on [1;2] . Tm gi tr nh nht ca biu

Tm gi tr nh nht ca biu thc P =

thc P =

1
1
1
.
+
+
4 + a ab 4 + b bc 4 + c ca
521

www.TaiLieuLuyenThi.com
Khm ph t duy K thut gii bt T Bi ton Max Min ng Thnh Nam

Li gii
Coi P l hm s ca a v b,c l tham s ta c:
1
1
1
.
P=
f (a) =
+
+
4 + a ab 4 + b bc 4 + c ca
b 1
c
=
f '(a )
+
0, a, b, c [1;2] .
2
( 4 + a ab ) ( 4 + c ca )2
Do f(a) l hm ng bin trn on [1;2] .
1
1
1
+
+ .
5 b 4 + b bc 4
1
1
1
Xt hm s g (b) = +
+ lin tc trn on [1;2] ta c:
5 b 4 + b bc 4
1
c 1
g=
'(b)
+
0, b, c [1;2] . Do g(b) l hm ng bin
2
( 5 b ) ( 4 + b bc )2

Suy ra f (a ) f (1) = g (b) =

trn on [1;2] .
1
1
1
1
3
+
+
= .
2 5 c 2 5 1 4
3
Vy gi tr nh nht ca P bng t ti a= b= c= 1 .
4
Bi 16. (Vit Nam TST 2001) Cho a,b,c l cc s thc dng tha mn
21ab + 2bc + 8ca 12 .
1 2 3
Tm gi tr nh nht ca biu thc P = + + .
a b c
Li gii
2 8 21 12
Ta c: 21ab + 2bc + 8ca 12 + +
.
a b c abc
1
1
1
t=
x =
,y =
, z , ( x, y, z > 0 ) v 2 x + 8 y + 21z 12 xyz . Khi P =x + 2 y + 3 z .
a
b
c
2x + 8 y
7
v x >
.
Ta c: z (12 xy 21) 2 x + 8 y > 0 z
4y
12 xy 21

Suy ra P g (b) g (1) =

Khi P x + 2 y + 3.

2x + 8 y
2x + 8 y
.
=x + 2 y +
12 xy 21
4 xy 7

Xt hm s f ( x) =x + 2 y +
f '( x) =1
522

14 + 32 y 2

( 4 xy 7 )

2x + 8 y
trn khong ; + ta c:
4 xy 7
4y

; f '( x) =0 ( 4 xy 7 ) =14 + 32 y 2 x =x0 =


2
2

7 + 32 y 2 + 14
.
4y

www.TaiLieuLuyenThi.com
Cty TNHH MTV DVVH Khang Vit

Ta c f(x) i du t m sang dng khi i qua x0 nn f(x) t cc tiu ti x0 .


Do f ( x) f ( x0 ) = 2 y +
Xt hm s g ( y ) = 2 y +

g '( y ) = 2

9
4y

32 y 2 + 14
9
.
+
4y
2y

32 y 2 + 14
9
trn ( 0;+ ) ta c:
+
4y
2y

7
y 2 32 y 2 + 14

; g '( y ) = 0 8 y 2 9

Ta c g(y) i du t m sang dng khi i qua

5
32 y 2 + 14 28 = 0 y = .
4
5
5
nn ti y = g(y) t cc
4
4

5 15
tiu. Do g ( y ) g =
.
4 2
5 15
Suy ra P f ( x) f ( x0 ) =g(y) g = .
4 2
15
5
2
Vy gi tr nh nht ca P bng
t ti =
hay
y
=
, x 3,=
z
2
4
3
1
4
3
.
=
a =
,b =
,c
3
5
2
Bi 17. Cho a,b,c l cc s thc thuc on [1;2] . Tm gi tr ln nht ca biu

thc P =

10a 11b 2014c


.
+
+
bc
ca
ab

Li gii
Ta ln lt coi P l hm ca c: f (c) =

2014c 1 10a 11b


+
+
.
ab
c b
a

Ta c:

f '(c) =

2014 1 10a 11b 2014c 2 10a 2 11b 2 2014.12 10.22 11.22



+

>0.
=
ab c 2 b
a
abc 2
abc 2

Do f(c) l hm ng bin trn [1;2] .


4028 5a 11b
.
+
+
ab
b 2a
4028 5a 11b
trn on [1;2] ta c:
Xt hm s g (b)=
+
+
ab
b 2a

Do f (c) f (2) = g (b) =

11 4028 11b 2 2.4028 11.22 2.4028


g '(b) =
=

<0.
2a ab 2
2ab 2
2ab 2
523

www.TaiLieuLuyenThi.com
Khm ph t duy K thut gii bt T Bi ton Max Min ng Thnh Nam

Do g(b) l hm nghch bin trn on [1;2] suy ra


4028
11
.
+ 5a +
a
2a
4028
11
Xt hm s h(a=
trn on [1;2] ta c:
)
+ 5a +
a
2a
4028
11
h '(a) = 2 + 5 2 < 0, a [1;2]. Do h(a) l hm nghch bin trn [1;2]
a
2a
8077
8077
V vy h(a ) h(1)
. ng thc xy ra khi v ch khi
=
P
2
2
a= b= 1, c= 2 .
g (b) g (1)= h(a )=

Vy gi tr ln nht ca P bng

8077
t ti a= b= 1, c= 2 .
2

C. BI TP RN LUYN
Bi 1. Cho 2 s thc x,y thuc [ 0;1] . Tm gi tr ln nht ca biu thc

P= 2 x3 + y 3 x 2 y y 2 x .

Bi 2. Cho x,y l hai s thc dng tha mn x, y 4 .


Tm gi tr nh nht ca biu thc P =

x
y 12
.
+ +
2 y x xy

Bi 3. Cho hai s thc x,y tho mn iu kin x y 1 .


Chng minh rng x 2 + x + 4

2 ( x + 1)
x+ y

2 x ( x + 1)
y +1

+ 2y .

Bi 4. Cho cc s thc 0 a b c 1 . Tm gi tr ln nht ca biu thc


P=

(a

b 2 ( b c ) + c 2 (1 c ) .

Bi 5. Cho a,b,c l cc s thc dng tha mn a + b + c =


8.
1
3
8
.
Tm gi tr nh nht ca biu thc P =
+
+
a
b
3c + 2a
Bi 6. Cho x,y,z l cc s thc dng tha mn x y z .
Chng minh rng

x z y x y z
+ + + + .
z y x y z x

Bi 7. Cho x,y,z l ba s thc thuc on [1;3] . Tm gi tr ln nht ca biu thc


P=
524

x y z y x z
+ + + + + .
y z x x z y

www.TaiLieuLuyenThi.com
Cty TNHH MTV DVVH Khang Vit

1
Bi 8. Cho x,y,z l cc s thc thuc on ;1 .
2
Tm gi tr ln nht v gi tr nh nht ca biu thc

1 1
21
P = ( x + y + z) + + .
x y z

Bi 9. Cho x,y,z l ba s thc thuc on [1;9] v x y, x z .


Tm gi tr gi tr nh nht ca biu thc P =

x
y
z
.
+
+
x + 2y y + z z + x

1
Bi 10. Cho x,y,z l cc s thc thuc on ;3 .
3

Tm gi tr ln nht v gi tr nh nht ca biu thc P =

x
y
z
.
+
+
x+ y y+z z+x

Bi 11. Cho x,y,z l cc s thc thuc on [1;3] . Tm gi tr nh nht ca biu


x y z x z y
thc P= 5 + + .
y z x z y x

Bi 12. Cho a,b,c l cc s thc thuc on [1;2] .


Chng minh rng a3 + b3 + c3 5abc .
Bi 13. Cho a,b,c l cc s thc thuc on [1;2] .
1 1 1
Chng minh ( a + b + c ) + + 10 .
a b c

Bi 14. Cho x,y,z l cc s thc thuc on [1;2] .


Tm gi tr ln nht ca biu thc P =

10 x 11 y 12 z
.
+
+
yz
zx
xy

Bi 15. Cho x,y,z l cc s thc thuc on [1;3] .


Tm gi tr nh nht ca biu thc P =

36 x 2 y z
.
+
+
yz
zx xy

Bi 16. Cho a,b,c l cc s thc thuc on [1;3] tha mn a + 2b + 3c =


12 .
Tm gi tr ln nht v nh nht ca biu thc P =

3 2 1
+ + .
a b c

Bi 17. Cho x,y l hai s thc thuc on [1;2] . Tm gi tr ln nht ca biu thc

525

www.TaiLieuLuyenThi.com
Khm ph t duy K thut gii bt T Bi ton Max Min ng Thnh Nam

P= x y+

y 1 1 x
.
+
x
y

1
Bi 18. Cho a,b,c l cc s thc thuc on ;2 .
2
a b bc ca
.
+
+
c
a
b
Bi 19. Cho a,b,c l cc s thc dng tho mn iu kin 21ab + 2bc + 8ca 12 .
1 2 3
Tm gi tr nh nht ca biu thc P = + + .
a b c
Bi 20. Cho x,y,z l cc s thc dng tho mn iu kin x + y + z xyz .

Tm gi tr ln nht ca biu thc P =

Tm gi tr nh nht ca biu thc P =x + 2 y + 5 z .


5
Bi 20. Cho x,y,z l cc s thc dng tho mn iu kin x + y + z = xyz .
3
Tm gi tr nh nht ca biu thc P = 6 ( x + y ) + z .

Bi 21. Cho x,y,z l cc s thc dng tho mn iu kin x 2 + y 2 + z 2 =


6.
Tm gi tr nh nht ca biu thc P =

x 2 y 5z
.
+
+
yz zx xy

Bi 22. Cho a,b,c l cc s thc dng tha mn iu kin ab >

7
v
3

100
.
a
Tm gi tr nh nht ca biu thc P = a + b + c .
Bi 23. Cho a, b, c l di ba cnh ca mt tam gic. Chng minh rng
3a + 57b + 7c= 3abc +

a ( b c ) + b ( c a ) + c ( a b ) + 4abc > a 3 + b3 + c3 .
2

Bi 24. Cho a, b, c l di ba cnh ca mt tam gic (c th suy bin).


( a b )( b a )( a c ) . Tm maxT v chng minh rng
1
.
t T =
max T <
21
( a + b )( b + a )( a + c )
Bi 25. Xt cc s thc dng x, y, z tha mn h iu kin

0 < z y x 3 (1)
3
2
( 2)
+ 2 1
xy
y

18
4
18
2 + 2 + 2 3
x y y z z x
526

( 3)

www.TaiLieuLuyenThi.com
Cty TNHH MTV DVVH Khang Vit

1
80 3 18 3
Hy tm GTLN ca biu thc P ( x, y, z ) = xyz +
x + y .
2
27
8
D. HNG DN GII P S

Bi 1. Xt hm s f ( x)= 2 x3 x 2 y x + 2 y 3 y 2 trn on [ 0;1] ta c:

y y2 + 6

x
=
[ 0;1]
6

.
f '( x) =
6 x 2 2 xy 1; f '( x) =
0

2
y
+
y
+
6
x
=
[ 0;1]
6

Ta c f(x) i du t m sang dng khi i qua x =


cc tiu ti x =

y + y2 + 6
.
6

y + y2 + 6
nn f(x) t
6

Do P Max { f (0); f (1)}= Max 2 y 3 y 2 ;2 y 3 y 2 y + 1 = 2 y 3 y 2 y + 1 v


2 y 3 y 2 y + 1 2 y 3 y 2 , y [ 0;1] .

1 y 2 y 2 y 1 +=
1 y ( y 1)( 2 y + 1) + 1 1, y [ 0;1] .
Mt khc 2 y 3 y 2 y +=
Suy ra P 1 . ng thc xy ra khi v ch khi=
x 1,=
y 0 hoc x= y= 1 .
Vy gi tr ln nht ca P bng 1 t ti=
x 1,=
y 0 hoc x= y= 1 .
Bi 2. Xt hm s f ( x) =
f '( x) =

x y 12
vi x ( 0;4] ta c:
+ +
2 y x xy

1
y
12 x 2 2 y 2 24 x 2 24
2 2 =
<
< 0.
2y x
yx
2 yx 2
2 yx 2

Do f(x) l hm nghch bin trn khong ( 0;4] suy ra


P = f ( x) f (4) =

y 5
+ =
4 y

( y 4 )( y 5) + 9 9 , 0 < y 4 .
4y

ng thc xy ra khi v ch khi x= y= 4 .


Vy gi tr nh nht ca P bng

9
t ti x= y= 4 .
4

Bi tp tng t
Cho a,b,x,y l cc s thc dng tha mn a3 + b3 =
1 v x, y 4 .
Tm gi tr nh nht ca biu thc P =

x 2 + 2 y 2 + 24

xy a 2 + b 2

.
527

www.TaiLieuLuyenThi.com
Khm ph t duy K thut gii bt T Bi ton Max Min ng Thnh Nam

Bi 3. Xt hm s f ( x) = x 2 + x + 4
f '( x) = 2 x + 1

2 ( 2 x + 1)
y +1

2 ( x + 1)
x+ y

2 x ( x + 1)
y +1

2x + 1

2 ( y 1)

( x + y)

( y 1)

=
2

y +1

2 y vi x y 1 ta c

0.
( x + y )2
2

1
V vy f(x) l hm ng bin suy ra f ( x) f ( y ) = y 2 3 y + 3 =
y

( y 1)3 0 .
y

Bt ng thc c chng minh. ng thc xy ra khi v ch khi x= y= 1 .


1 1
1 z y
Bi 6. Xt hm s f ( x)= x + ( y z ) . + ta c:
x y z
z y

( y z ) ( x 2 yz )

1 1 yz
f '( x) = 2 =
z y
x

x 2 yz

0, x y z > 0 .

Vy f(x) l hm ng bin suy ra f ( x) f ( y ) =


0 hay

x z y x y z
+ + + + .
z y x y z x

ng thc xy ra khi v ch khi x = y .


Bi 7. Khng mt tnh tng qut gi s x y z . Coi P l hm ca x ta c:
1 1
y+z y z
P = f ( x) = + x +
+ + vi x [1;3] ta c:
x
z y
y z

2
1 1 y + z ( y + z ) x yz
f '( x) = + 2 =
0.
y z
x
x 2 yz

Do f(x) l hm ng bin trn on [1;3] .


1 1 y+ z y z
Suy ra P= f ( x) f (3)= g ( z )= 3 + +
+ + .
3
z y
y z

1 1 y+ z y z
Xt hm s g ( z )= 3 + +
+ + trn on [1;3] ta c:
z y
3
y z

2
1 y 1 ( y + 3) z 3 y
g '( z ) = 2 + 2 + =
0, 1 z y 3 .
3 z
y
z
3 yz 2

Suy ra g(z) l hm nghch bin trn on [1;3] hay


1 y +1
1 4 ( y 1)( y 3) 26 26
= 3 + 1 +
+ y +=
+
, y [1;3] .
P g (z) g(1)
3
y
3y
3
3
y
ng thc xy ra khi v ch khi x= 3, y= z= 1 hoc x= y= 3, z= 1 .
528

www.TaiLieuLuyenThi.com
Cty TNHH MTV DVVH Khang Vit

Vy gi tr ln nht ca P bng

26
t ti x= 3, y= z= 1 hoc x= y= 3, z= 1 v
3

cc hon v.
Cch 2: Khng mt tnh tng qut gi s x y z khi :
x
z +1
2
( x y )( y z ) 0 xy + yz y + zx
1 + z
x

y
+
z
y
+
x

x
y
.
z
y

x z
Suy ra P 2 + + 2 .
z x
x
3
t=
t
, t [1;3] suy ra ( t 1)( t 3) 0 t 2 + 3 4t t + 4 .
z
t
2
4
4 26
1

.
Suy ra P 2 t + + 2 2 4 + 2 = 10 10 =
t
t
3 3
t

1 1 1 35
.
Nhn xt. Ta c th vit li bt ng thc di dng: ( x + y + z ) + +
x y z 3
Bt ng thc tng qut ca dng ton ny cp n trong ch trc.
Bi tp tng t
Cho x,y,z l cc s thc thuc on [1;2] . Chng minh rng
1 1 1
+ + 10 .
x y z
Bi 8. Tm gi tr nh nht ca P.

( x + y + z )

1 1 1
3
3
27
Ta c: x + y + z ; ( x + y + z ) + + 9 P .9 = .
2
2
2
x y z
1
.
ng thc xy ra khi v ch khi x= y= z=
2
27
1
t ti x= y= z=
.
Vy gi tr nh nht ca P bng
2
2
Tm gi tr ln nht ca P.
Khng mt tnh tng qut gi s x y z khi :
1 1
1
21
Xt hm s f ( x) = ( x + y + z ) + + trn on ;1 ta c:
2
x y z

529

www.TaiLieuLuyenThi.com
Khm ph t duy K thut gii bt T Bi ton Max Min ng Thnh Nam

2
xyz + 2 x 2 yz ( y + z )
1 1 1 ( x + y + z)
>0
= ( x + y + z)
f '( x) = 2 ( x + y + z ) + +
2
2

x
y
z
x
x
yz

v x 2 yz .
1
Do f(x) l hm ng bin trn ;1 suy ra
2

1 1
2
P = f (x) f(1) = g ( y ) = 1 + + (1 + y + z ) .
y z

1 1
1
2
Xt hm s g ( y ) = 1 + + (1 + y + z ) trn on ;1 ta c:
y z
2

g '( y ) =

(1 + y + z )2 + 2 1 + y + z 1 + 1 + 1 =1 + y + z yz + ( 2 y
(
)
)
(
2

z ( z + 1)
>0

y2 z

v 2 y = 2 y. y y z .
2

1
Vy g(y) l hm ng bin trn on ;1 suy ra
2

( z 1) 2 z 2 + 11z 4
1
2

1
P g ( y ) g (1)= 2 + ( 2 + z ) =
+ 27 27, z ;1 .
z
z

2
ng thc xy ra khi v ch khi x= y= z= 1 .
Vy gi tr ln nht ca P bng 27 t ti x= y= z= 1 .
Nhn xt. S dng bt ng thc AM-GM ta tm gi tr ln nht ca P nh sau:
3

1 1 1 1
P
+ + + 2 ( x + y + z ) .

27 x y z

1
Ta=
c: 2 x +
x

( x 1)( 2 x 1) + 3 3, x 1 ;1 .

Tng t ta c: 2 y +

1
1
1
3
27 .
3;2 z + 3 . Do P ( 3 + 3 + 3) =
27
y
z

ng thc xy ra khi v ch khi x= y= z= 1 .


Bi 9. Xt hm s f ( z ) =

x
y
z
trn on [1;9] ta c:
+
+
x + 2y y + z z + x

( x y ) z 2 xy
y
x
.
f '(z) =

+
=
( y + z )2 ( x + z )2 ( x + z )2 ( y + z )2
530

www.TaiLieuLuyenThi.com
Cty TNHH MTV DVVH Khang Vit

1
y
z
4
+ Nu x = y P = +
+
= .
3 y+z z+ y 3

+ Nu x > y khi f '(z) = 0 z = xy ta c f(z) i du t m sang dng khi


i qua z = xy nn f(z) t cc tiu ti z = xy .
x
Do P =
f (z) f( xy ) =
+
x + 2y

t
=
t

x
y

2 y

=
+
x + y x +2
y

2
x
+1
y

x
, (1 < t 3) ta c
y

t2

2
P g (t)= 2
+ =
t + 2 t +1

( 3 t ) ( 7t 2 16t + 10 ) 29 29
.
+

22 22
22 ( t + 1) ( t 2 + 2 )

ng thc xy ra khi v ch khi


z = xy , t = 3 x = 9 y, z = 3 y x = 9, y =1, z = 3 .

29
t ti =
x 9,=
y 1,=
z 3.
22
x
y
z
ta c:
Bi 10. Gi s z = min { x, y, z} khi xt hm s f ( z ) =
+
+
x+ y y+z z+x

Vy gi tr nh nht ca P bng

( x y ) z 2 xy
x = y
y
x
.
f '( z ) =

+
=
;
f
'(
z
)
=
0

( y + z )2 ( x + z )2 ( x + z )2 ( y + z )2
z = xy
1
y
z
3
+ Nu x = y P = +
+
= .
2 y+z z+ y 2

+ Nu x > y f '( z ) 0 nn f(z) l hm nghch bin do


x
P=
f ( z ) f ( xy ) = +
x+ y
2
t2
Mt khc:
=
+
2
t +1 t +1

2
t2
vi
=
+
=
t
2
t
+
1
x + y t +1
2 y

x
, (1 < t 3) .
y

( 3 t ) ( 2t 2 2t + 1) 7 7
+ , t (1;3] .
5 5
5 ( t + 1) ( t 2 + 1)

ng thc xy ra khi v ch khi


1
z = xy , t = 3 x = 9 y, z = 3 y x = 3, y = , z =1 .
3
531

www.TaiLieuLuyenThi.com
Khm ph t duy K thut gii bt T Bi ton Max Min ng Thnh Nam

1
7
t ti=
x 3,=
y
=
, z 1 hoc cc hon v.
3
5
Tm gi tr ln nht thc hin tng t
Bi 12. Khng mt tnh tng qut gi s 1 c b a 2 khi bt ng thc

Vy gi tr nh nht ca P bng

tng ng vi: a3 5bca + b3 + c3 0 .


Xt hm s f (a ) = a3 5bca + b3 + c3 trn on [1;2] ta c
f '(a ) =
3a 2 5bc; f ''(a ) =
6a > 0 .

Suy ra f (a ) max { f (1); f (2)=


} max 1 5bc + b3 + c3 ;8 10bc + b3 + c3 .
Ta ch cn ch ra rng f (1) 1, f (2) 0 .
+ Xt hm s g (b) = f (1) = b3 5cb + c3 + 1 trn on [1;2] ta c
g '(b) =
3b 2 5c; g ''(b) =
5b > 0 .

{
5c + 2 = ( c 2 ) ( c 1 + 2c ) 0, c [1;2]
10c + 9 = ( c 1) ( c + c 9 ) 0, c [1;2]

Suy ra g (b) max { g (1); g (2)


=
} max c3 5c + 2; c3 10c + 9 .
Ch c3
c3

Do f (1) 0 . Tng t ta c f (2) 0 .


Bi ton c chng minh. ng thc xy ra khi v ch khi a= 2, b= c= 1 v
cc hon v.
Bi tp tng t
Chng minh a,b,c l cc s thc thuc on [1;2] ta c

2 a3 + b3 + c3 ( a + b + c )( ab + bc + ca ) .

Bi 13. Bt ng thc cn chng minh tng ng vi:


a b c b c a
+ + + + + 7.
b c a a b c
Khng mt tnh tng qut gi s a b c . Khi
c
c b
a
a b
( a b )( b c ) 0 ab + bc ac + b2 + 1 + v + 1 + .
a
b a
c
b c
a b c b c a
a c
Do + + + + + 2 + 2 + .
b c a a b c
c a
a c 5
a
a
+ 2 1 0 lun ng do 1 c a 2 .
c a 2
c
c
Bt ng thc c chng minh. ng thc xy ra khi a= b= 2, c= 1 hoc
a= 2, b= c= 1 v cc hon v.

Vy ta chng minh

532

www.TaiLieuLuyenThi.com
Cty TNHH MTV DVVH Khang Vit

Tng qut. Cho cc s thc x1 , x2 ,..., xn [ p; q ] , ( p, q 0 ) . Chng minh


1
( p q) .
1
1
( x1 + x2 + ... + xn ) + + ... + n2 + kn
xn
4 pq
x1 x2
2

trong kn = n 2 nu n chn v k=
n 2 1 nu n l.
n
Bi 16. T iu kin ta c: 3c = 12 a 2b khi P =

3 2
3
.
+ +
a b 12 a 2b

3 2
3
trn on [1;3] ta c:
+ +
a b 12 a 2b
12 ( 6 b )( a + b 6 )
3
3
f '(a) =
2+
=
0, a, b [1;3] .
a
(12 a 2b )2 a 2 (12 a 2b )2

Xt hm s f (a) =

Do f(a) l hm nghch bin trn on [1;3] suy ra


2
3
2
3
.
+
= f (3) =P = f (a ) f (1) =h(b) =3 + +
b 9 2b
b 11 2b
1
2
3
Bi 19. t=
ta c 2 x + 4 y + 7 z 2 xyz v ta cn tm gi tr nh
x =
,y =
,z
a
b
c
nht ca biu thc P = x + y + z . Thc hin tng t bi ton trn ta c gi tr
1
4
3
.
nh nht ca P bng 15/2 t ti=
a =
,b =
,c
3
5
2
100
3a + 57b
100
a .
Bi 22. Theo gi thit ta c: c ( 3ab 7 ) = 3a + 57b
c=
a
3ab 7
100
3a + 57b
2
a = a + b + 3a + 57 ab 100
Khi P = a + b +
3ab 7
a ( 3ab 7 )
g (b) =1 +

= a+b+

3a 2 +

57
( 3ab 7 ) + 33
3a 2 + 33
57
3
= a+b+
+
a ( 3ab 7 )
a ( 3ab 7 ) 3a

=a +

1
3a 2 + 33
64
+
( 3ab 7 ) +
3a
a ( 3ab 7 ) 3a

a+

64
1
3a 2 + 33
64 2 a 2 + 11 35
+2
=a +
+

( 3ab 7 ) .
3a
3a
3a
3
a ( 3ab 7 )
a

Vic xut hin 3ab 7 ta suy ngh n vic kh ci khi biu thc ca P
bng cch tch nhm hp l v triu tiu bng AM-GM.
Ch . Kho st hm s f(a) sau cng ta c ngay kt qu bi ton.
35
35
5
Vi=
th P bng
. Vy gi tr nh nht ca P bng
.
a 5,=
b
3
3
3
533

www.TaiLieuLuyenThi.com
Khm ph t duy K thut gii bt T Bi ton Max Min ng Thnh Nam

CH 7: K THUT S DNG TNH CHT CA

NH THC BC NHT V TAM THC BC HAI


A. NI DUNG PHNG PHP
Ta m u k thut ny bng cc nh l sau y
1. nh l 1. Nu f(x) l hm bc nht tho mn iu kin f (a ) 0, f (b) 0 khi
f ( x) 0, x [ a; b ] .
2. nh l 2. Nu f(x) l hm bc nht khi

min { f (a ); f (b)} f ( x) max { f (a ); f (b)} , x [ a; b ] .

3. nh l 3. Nu f ( x) = ax 2 + bx + c, ( a 0 ) khi vi mi x [ ; ] .
Ta c f(x) t Max, Min ti x = hoc x = hoc x =

b
.
2a

B. BI TP MU
Bi 1. Cho x,y,z l cc s thc thuc on [ 0;2] .
Chng minh rng 2 ( x + y + z ) ( xy + yz + zx ) 4 .
Li gii
Phn tch tm li gii: Nu coi y v z l tham s th bt ng thc c dng nh
thc bc nht ca x nn ta nh gi theo nh l 2.
Vit li bt ng thc cho di dng:

( 2 y z ) x + 2 ( y + z ) yz 4 0 .
Do x [ 0;2] nn f ( x) max { f (0); f (2)} .
Ta c f (0) = 2 ( y + z ) yz 4 = ( y 2 )( 2 z ) 0, y, z [ 0;2] .
f (2) =2 ( 2 y z ) + 2 ( y + z ) yz 4 = yz 0 .
f ( x) =

Suy ra f ( x) 0 . Bt ng thc c chng minh ng thc t ti chng hn


x= 0, y= z= 2 .
Bi 2. Cho a,b,c,d l cc s thc thuc on [ 0;1] .
Chng minh rng (1 a )(1 b )(1 c )(1 d ) + a + b + c + d 1 0 .
Li gii
Ta cn chng minh P = (1 a )(1 b )(1 c )(1 d ) + a + b + c + d 1 0 .
R rng biu thc v tri l hm bc nht i vi mi bin a,b,c,d.
+ Nu coi l hm bc nht ca a th biu thc P t gi tr nh nht ti a = 0 hoc
a =1.
+ Tng t P t gi tr nh nht ti b, c, d {0;1} .
534

www.TaiLieuLuyenThi.com
Cty TNHH MTV DVVH Khang Vit

Nu mt trong 4 s bng 1 th P 0 .
Nu c 4 s bng 0 th P = 0 .
Vy P 0, a, b, c, d [ 0;1] ta c iu phi chng minh.
Bi 3. Cho 2015 s thc x1 , x2 ,..., x2015 thuc on [ 2015;2015] .
Tm gi tr nh nht ca biu thc P= x1 x2 + x2 x3 + ... + x2014 x2015 + x2015 x1 .
Li gii
Tng t bi ton trn ta c P t gi tr nh nht khi
xi {2015;2015} , i =1, 2015 .

Trong 2015 s cho lun tn ti t nht 2 s cng du khng mt tnh tng


qut gi s x1 x2 > 0 x1 x2 =
20152 .
Khi
P=
20152 + x2 x3 + ... + x2015 x1 20152 20152 ... 20152 =
2013.20152 .

Vi x1 =
x2 =
2015, x3 =
2015, x4 =
2015,..., x2014 =
2015, x2015 =
2015 th
P = 2013.20152 .

Vy gi tr nh nht ca P bng 2013.20152 .


Bi 4. Cho a,b,c l cc s thc thuc on [ 0;1] .
a
b
c
+
+
+ (1 a )(1 b )(1 c ) 1 .
b + c +1 c + a +1 a + b +1
Li gii
Gi P l biu thc v tri ca bt ng thc.
Khng mt tnh tng qut gi s
b
b
c
c
.
=
a max {a, b, c}

c + a +1 b + c +1 a + b +1 b + c +1
a+b+c
Suy ra P
+ (1 a )(1 b )(1 c ) .
b + c +1
Vy chng minh bt ng thc trn ta ch cn chng minh
a+b+c
=
f (a)
+ (1 a )(1 b )(1 c ) 1 0 .
b + c +1

Chng minh rng

Ta c f (a ) max { f (0); f (1)} .


Ta c f=
(1) 0; f =
(0)
=

b+c
+ (1 b )(1 c ) 1
b + c +1

bc ( b + c ) b 2 + c 2 bc 1
b + c +1
535

www.TaiLieuLuyenThi.com
Khm ph t duy K thut gii bt T Bi ton Max Min ng Thnh Nam

(b + c ).

(b

b2 + c2
b 2 + c 2 bc 1
2
b + c +1

+ c 2 ( b + c 2 ) 2 ( bc + 1)
2 ( b + c + 1)

Suy ra iu phi chng minh.


Cch 2: Ta c th chng minh theo cch khc nh sau:
a+b+c
+ (1 a )(1 b )(1 c ) 1
b + c +1
a+b+c
1 a
(1 a )(1 b )(1 c ) 1
=
b + c +1 b + c +1
(1 b )(1 c )( b + c + 1) 1
lun ng theo AM-GM.
Bi 5. Cho a,b,c l cc s thc khng m tha mn iu kin a + b + c =
1.
Tm gi tr ln nht ca biu thc P = ab + bc + ca 2abc .
Li gii
Do P l biu thc i xng vi a,b,c nn khng mt tnh tng qut ta c th gi
a+b+c 1
s c = min {a, b, c} c
= 1 2c > 0 .
3
3
Khi vit li P di dng: P = ab (1 2c ) + ( a + b ) c = ab (1 2c ) + c (1 c ) .
S dng bt ng thc AM-GM ta c:
2
2
1 c 2
a + b 1 c
0 ab
=

ab 0;
.
2 2
2

Coi ab l n th P l hm bc nht ca ab.


Suy ra
2

1 c 2

1 c
MaxP
= Max P(0);P
Max c (1 c ) ; (1 2c )
=
+ c (1 c ) .
2
2

1 c
1
1
1
1
Xt hm s f (c) =
c3 + c 2 + vi c 0;
(1 2c )
+ c (1 c ) =
2
2
4
4

3
ta c:
f '(c)=

1
1
1
3c 2 + c =
c (1 3c ) 0, c 0; .
2
2
3

1
Do f(c) l hm ng bin trn on 0; .
3
536

www.TaiLieuLuyenThi.com
Cty TNHH MTV DVVH Khang Vit

1 7
Suy ra Max =
.
f (c ) f=

1
3 27
c 0;
3

ng thc xy ra khi v ch khi a= b= c=

1
.
3

2c. (1 2c ) 1 2c + 1 2c 2 1 7
.
Xt Q =
c (1 2c ) =

=<
2
2
2
8 27

1 c
1 7
Vy Max c (1 c ) ; (1 2c )
.
+ c (1 c ) = f =

2
3 27

7
ng thc xy ra khi v ch khi
V vy gi tr ln nht ca P bng
27
1
.
a= b= c=
3
Nhn xt. V 1 2c > 0 nn ta c th nh gi trc tip:

a+b
1 c
P
c)
(1 2c ) + c (1=
(1 2c ) + c (1 c ) .
2
2
Bi 6. Cho x, y, z l cc s thc khng m c tng bng 3.

Chng minh rng x 2 + y 2 + z 2 + xyz 4 .


Li gii
1
Khng mt tnh tng qut ta gi s x = min { x, y, z} 3x x + y + z = 3 x .
3
t biu thc v tri bt ng thc l P, khi ta c:

P 4 = x 2 + ( y + z ) 2 yz + xyz 4 =
2

( x 2 ) yz + x 2 + ( 3 x )2 4

= f (t ) = ( x 2 ) t + 2 x 2 6 x + 5
2

y + z 3 x
Vi 0 t = yz
=

2 2 .
3 x 2
Vy ta tm gi tr nh nht ca f (t ) trn 0;
, ta c f (t ) l hm s
2
nghch bin do x 2 < 0 .
3 x 2 1
Vy P 4= f (t ) f

=
( x 1)2 ( x + 2 ) 0 P 4.
2 4

T ta c iu phi chng minh. ng thc xy ra khi v ch khi x= y= z= 1 .


537

www.TaiLieuLuyenThi.com
Khm ph t duy K thut gii bt T Bi ton Max Min ng Thnh Nam

Bi 7. Cho a, b, c l cc s thc dng tha mn iu kin a + b + c =


1.

) (

Chng minh rng 5 a 2 + b 2 + c 2 6 a3 + b3 + c3 + 1 .


Li gii
Khng mt tnh tng =
gi s a min {a, b, c} a

1
3

Bt ng thc cho tng ng vi:


2
3
5 a 2 + ( b + c ) 2bc 6 a3 + ( b + c ) 3bc ( b + c ) + 1

2
3
5 a 2 + (1 a ) 2bc 6 a3 + (1 a ) 3bc (1 a ) + 1

( 9a 4 ) bc + ( 2a 1) 0
2

b + c 1 a
Ta t t = bc 0 < t
=

2 2

Vy ta ch cn chng minh: f (t=


)

2
1 a
.
2

( 9a 4 ) t + ( 2a 1)2 0, t 0;

1 a 2 1
2
Do f (t ) l hm nghch bin nn f (t ) f

a ( 3a 1) 0.
=
2 4

1
.
Ta c pcm. ng thc xy ra khi v ch khi a= b= c=
3
C. BI TP RN LUYN

Bi 1. Cho a,b,c l cc s thc thuc on [ 0;1] .


Chng minh rng a + b + c ab bc ca 0 .
Bi 2. Cho l cc s thc khng m tha mn iu kin a + b + c =
1.
1
a) Chng minh rng a3 + b3 + c3 + 6abc .
4
b) 7 ( ab + bc + ca ) 2 + 9abc .
1
Bi 3. Cho a,b,c l cc s thc thuc on 0; v a + b + c =
1.
2
9
Chng minh rng a3 + b3 + c3 + 4abc
.
32

Bi 4. Cho a,b,c l cc s thc thuc on [1;2]. Chng minh a3 + b3 + c3 5abc .


Bi 5. Cho cc s thc khng m x,y,z c tng bng 1.
538

www.TaiLieuLuyenThi.com
Cty TNHH MTV DVVH Khang Vit

Chng minh rng 0 xy + yz + zx 2 xyz

7
.
27

D. HNG DN GII P S
Bi 2. a) Khng mt tnh tng qut =
gi s a max {a, b, c} a

1
.
3

Bt ng thc tng ng vi:


a3 + ( b + c ) 3 ( b + c ) bc + 6abc
3

1
4

a3 + (1 a ) 3 (1 a ) bc + 6abc
3

1
0.
4

4 ( 3a 1) bc + ( 2a 1) 0
2

Bt ng thc cui ng. ng thc xy ra khi a= b=

1
, c= 0 hoc cc hon v.
2

c) Khng mt tnh tng qut =


gi s a min {a, b, c} 7 9a > 0 .
Bt ng thc cho tng ng vi:

( 7 9a ) bc + 7a ( b + c ) 2 0
( 7 9a ) bc + 7 a (1 a ) 2 0

b+c
V tri l hm ng bin vi bc v bc
, do ta ch cn chng minh
2
2

b+c
2
( 7 9a )
7a + 7a 2 0
2
2

1 a
2
( 7 9a )
7a + 7a 2 0
2

( a + 1)( 3b 1)2
4

Bt ng thc cui ng. ng thc xy ra khi v ch khi a= b= c=

1
.
3

1
Bi 5. HD: Gi s x = min { x, y, z} 3 x x + y + z =1 x .
3
Khi ta c: P = xy + yz + zx 2 xyz = yz (1 2 x) + xy + zx 0

ng thc xy ra khi v ch khi x= 1, y= z= 0 .


Vy gi tr nh nht ca P bng 0 t ti x= 1, y= z= 0 hoc cc hon v.
2

1 x
Mt khc ta li c P= yz (1 2 x) + x( y + z ) x (1 x ) +
(1 2 x )= f ( x)
2
539

www.TaiLieuLuyenThi.com
Khm ph t duy K thut gii bt T Bi ton Max Min ng Thnh Nam

1
Ta tm gi tr ln nht ca f ( x) trn on 0; .
3

Ta c f '( x=
)

3 1

x x 0 , do f ( x) ng bin trnon
2 3

1
0; 3 .

1 7
Vy max
=
P max=
f ( x) f=
.

3 27

ng thc xy ra khi v ch khi x= y= z=

1
.
3

CH 8: BT NG THC PH NG CH

V P DNG GII THI


Ni dung ch ny ti cp n mt s bi ton c bn v kt hp s dng
bt ng thc c bn nh AM GM v Cauchy Schwarz trong chng minh bt
ng thc v tm cc tr. Hy vng ch ny s gip ch cho bn c nhn nhn
tip cn bi ton bt ng thc v cc tr linh hot v bao qut hn.
Bi ton 1. Vi mi s thc a,b tha mn iu kin ab 1 ta lun c
1
1
2
.
+

2
2
1 + ab
1+ a
1+ b
ng thc xy ra khi v ch khi a = b hoc ab = 1 .
Vi ab 1 th bt ng thc i chiu.
Chng minh.
Nhn xt. Bng cch chng minh tng t trn ta c cc bt ng thc cng dng
nh sau
Vi mi s thc dng a,b thay i tha mn ab 1 ta lun c
1
1
2
.
+

1 + ab
1 + a2
1 + b2
ng thc xy ra khi v ch khi a = b hoc ab = 1 .
Vi ab 1 bt ng thc i chiu.
Mt s bt ng thc ph khc cng dng
1
1
2

, ( 1 < xy 1) .
2
2
1 + xy
1+ x
1+ y

540

1
1 + x2

1
1 + y2

2
, ( xy 1) .
1 + xy

www.TaiLieuLuyenThi.com
Cty TNHH MTV DVVH Khang Vit

1
1 + x2

1 + y2

2
, ( 1 < xy 1) .
1 + xy

Cho hai s thc dng x,y ta lun c

1
x + xy
2

1
y + xy
2

2
x + y2
2

ng thc xy ra khi v ch khi x = y .


Chng minh.
S dng bt ng thc AM-GM ta c
1
1
2
+

2
2
x + xy
y + xy
x 2 + xy y 2 + xy

2
=
x 2 + xy + y 2 + xy
( x + y )2
2
2

)(

2
x2 + y 2

1
1
1
n
, xk 1, k =
1, n .
+
+ ... +

1 + x1 1 + x2
1 + xn 1 + n x1 x2 ...xn

T bt ng thc ny ta c th a ra cc bt ng thc cng dng chng hn


4
Chng minh rng vi mi s thc x,y tho mn iu kin xy ta c
5
1
1
2
+

2
2
4 + 5 xy
4 + 5x
4 + 5y
1
4 + 5x2

1
4 + 5 y2

Vi mi s dng a,b ta c

2
4 + 5 xy
a
b
2
.
+

b + ab a + ab 1 + ab

Chng minh.
S dng bt ng thc C S ta c
b)
( a + b )=
(a +=
a
b
+

2
2
b + ab a + ab ab + a b + ab + b a ab ( a + b ) + 2ab
2

1
ab
2ab
+
a + b ( a + b )2

1
2
=
ab
1 1 + ab
+
2 ab 2
Du bng xy ra khi v ch khi a = b .

541

www.TaiLieuLuyenThi.com
Khm ph t duy K thut gii bt T Bi ton Max Min ng Thnh Nam

CC V D
V d 1. Cho x,y,z l cc s thc dng tho mn iu kin y 2 xz; z 2 xy .
Tm gi tr nh nht ca biu thc P =

x
y
z
.
+
+ 2014.
x+ y y+z
z+x

Li gii
1

Ch z / x 1 khi s dng bt ng thc

1+ a
1
1
2
x
y
Ta c
.
+
=
+

x + y y + z 1+ y 1+ z
z
1+
x
y
x
z
t2
2
2
Do P
+ 2014. x = + 2014. 2
z
z
t +1
+1 1+ t
1+
x
x

1
1+ b

2
, ( ab 1)
1 + ab

2 ( t 1) 503t 2 + 1007t + 503


z
=
+ 1008 1008, t = 1
2
x
( t + 1) t + 1

Vy gi tr nh nht ca P bng 1008 t ti x= y= z .


V d 2. Cho x,y,z l cc s thc dng tha mn iu kin x z .
Tm gi tr nh nht ca biu thc: P =
2+

2 x2

( x + y)

2z ( 2 y + z )

( y + z)

3z
.
z+x

Li gii
Bin i biu thc trong cn v s dng bt ng thc C S ta c
2+

2 x2
( x + y )2

2 z (2 y + z )
( y + z )2

2 x2
( x + y )2

2 y2
( y + z )2


1
1
1

= 2
+

+
2
2
y
z

z
+
+
1
1
1+

1 +
x
y

x
y

Mt khc do:

y z z
. =
1.
x y x
2

1
2

Nn p dng bi ton 1 ta c
.
+
z
1+ y 1+ z
1+

x
y
x

542

www.TaiLieuLuyenThi.com
Cty TNHH MTV DVVH Khang Vit

T ta suy ra

2+

2 x2
( x + y)

2 z (2 y + z )
( y + z)

z
x

1+

z
2
3t 2
.
1 . Khi ta suy ra P
+ 2
x
1+ t t +1
2
6t
2
3t 2
f '(t )
+ 2 2 > 0 t [1; + )
Xt hm f=
trn [1; + ) ta c =
(t )
+ 2
2
1+ t t +1
(1 + t ) (t + 1)

t=
t

Do Min P = f (1) =

5
2

Bi tp tng t
Cho x,y,z l cc s thc dng tho mn iu kin x = min { x, y, z} .
Tm gi tr nh nht ca biu thc P =

x
y
3z
.
+
+
x+ y y+z z+x

1
x 1; y, z 1 tha mn iu kin xyz = 1 .
4
1
1
1
.
Tm gi tr nh nht ca biu thc P =
+
+
1+ x 1+ y 1+ z

V d 3. Cho cc s thc

Li gii
1
1
2
1
2
1
2
+

P
+
=
+
Vi yz 1 ta c
1 + y 1 + z 1 + yz
1 + x 1 + yz 1 + 1 1 + yz
yz
=
t

yz 1 =
t

2 . Suy ra P f (t ) =

t2

2
.
t +1 1+ t

Ta c =
: f '(t )

t2 +1

2t ( t + 1) 2 t 2 + 1
2
=
2
2
(1 + t )2
t 2 + 1 ( t + 1)
2

2t

0, t [1;2] .

22
f (t ) f (2) =
15
22
1
t ti x= ; y= z= 2 .
15
4
V d 4. Cho a,b,c l cc s thc dng.

Vy gi tr nh nht ca P bng

2a
2b
2c
+
+
3.
a+b
b+c
c+a
Li gii
Nhn xt. Ta chng minh bt ng thc trn bng bt ng thc C S di y
tip cn theo phng php hm s.

Chng minh rng

543

www.TaiLieuLuyenThi.com
Khm ph t duy K thut gii bt T Bi ton Max Min ng Thnh Nam

Ta cn chng minh:

b
,y=
a

t x =

1
b
1+
a

c
,z =
b

c
1+
b

1
a
1+
c

a
xyz = 1 .
c

Bt ng thc tr thnh:

1+ x

1+ y

1+ z

3
.
2

Khng mt tnh tng qut gi s z = max { x, y, z} z 1 xy =

1
1.
z

Khi s dng bt ng thc C-S v bi ton ph ta c:


1
1 + x2

1
1
4
2 z
2
+
=.

2
2
1 + xy
z +1
1+ y
1 + y2
1+ x
1

Chng minh hon tt nu ta chng minh c:


Tht vy
vi

2 z
1
3
.
+

z +1
2
1 + z2

2 z
1
2 z
2
3
(v bt ng thc tng ng
+

2
z +1
z +1 1+ z
2
1+ z

2z z + 1

0 ).

Bi ton c chng minh.


V d 5. Cho cc s thc dng a, b tha mn iu kin a + b =
2ab .

3 3 2 2
+3
a +b +a+b+4 .
4
a +1 b +1
Li gii

Q
Tm gi tr nh nht ca biu thc:=

T gi thit ta c: 2ab = a + b 2 ab ab 1 ab 1.
Vi ab 1 ta c BT:

2
.
a + 1 b + 1 1 + ab
2

Mt khc ta c:
3 3 2
a + b 2 + a + b=
+4
3
4

3 3 2
a + b 2 + 2ab + 4
4
3
2
= 3 3 (a + b) + 4
4
3
4 3 3 ab + 1
3 3 4ab +=
4

544

www.TaiLieuLuyenThi.com
Cty TNHH MTV DVVH Khang Vit

3
23 2 2 23 2
3
3
ab + 1 + ab + 1 + ab + 1 +
+
ab
+
1

ab + 1
p dng BT AM - GM ta c:

Do : Q

2
+ 33 ab +=
1
ab + 1

ab + 1 + 3 ab + 1 + 3 ab + 1 +

23 2
44 23 2 =
43 2
ab + 1

T gi thit ta c:
2ab = a + b 2 ab ab 1 ab 1 ab + 1 2

M 2 2 3 2 < 0

1
1

ab + 1 2

2 23 2 2 23 2

=1 3 2
ab + 1
2

Do Q 4 3 2 + 1 3 2 = 1 + 3 3 2 . ng thc xy ra khi a = b = 1.
Vy gi tr nh nht ca Q l bng 1 + 3 3 2 t c khi a = b = 1.
V d 6. Cho x,y,z l cc s thc dng tha mn iu kin
4

x y
z x
+ + = + 2.
x z
y z

Tm gi tr ln nht ca biu thc P =

2 y2
x +y
2

2z2
y +z
2

3z
.
2x + z

Li gii
4

x y
x y
x
z
z
x2 z
+=
2 + + 2 . + = 2 2 + .
z
x
x
x
z
y z
y z

Theo gi thit ta c:

x
1
, ( t > 0 ) t + 2 2t 2 + 2t 3 t 2 2t + 1 0
z
t
1
( t 1) 2t 2 + t 1 0 ( t 1)( t + 1)( 2t 1) 0 t 1 .
2
x y x
1
1
2
2
Khi . =
.
=
t 1
+

=
2
2
x y 1+ t
y z z
x
y
+
1
.
+1 z +1
y z

y

t=
t

Ta c:

y2
x +y
2

z2

1
= 2
+
y +z
x

+1

y
2

2
2
2
= = .
x
y
x
1+ t
y
1+ .
1+
+1
y
z
z
z
2

4
3
Suy ra P f (t ) =
.
t 2t + 1
545

www.TaiLieuLuyenThi.com
Khm ph t duy K thut gii bt T Bi ton Max Min ng Thnh Nam

Xt hm s f (t )=
f '(t ) =

( 2t + 1)

4
3
1
lin tc trn on ;1 ta c:

t 2t + 1
2
4

( t + 1)

2 5t 2 + 2t 1

( t + 1) ( 2t + 1)
2

1
0, t ;1 .
2

1
1 7
Do f(t) l hm nghch bin trn on ;1 . Suy ra P f (t ) f =
.
2
2 6
x y x 1
ng thc xy ra khi v ch khi = , = z = 2 x, y = 2 x .
y z z 2

Vy gi tr ln nht ca P bng

7
t ti
=
z 2=
x, y
6

2x .

V d 7. (TSH Khi A 2011) Cho cc s thc x, y, z [1;4] tha mn iu kin


x y, x z . Tm gi tr nh nht ca biu thc P=

x
y
z
.
+
+
2x + 3y y + z z + x

Li gii
1
1
1
Vit li P di dng: P =
.
+
+
3y
z
x
2+
1+
1+
x
y
z
Do x y
1
z
1+
y

t
=
t

z x x
. = 1. p dng bt ng thc trong bi ton 1, ta c
y z y

1
x
1+
z

1+

x
y

1
2
.
+
3y
x
2+
1+
x
y

x
1
2
t2
2
.
, (1 t 2 ) suy ra P
+
=
+
2
3
y
2 + 2 1 + t 2t + 3 1 + t
t

Xt hm s=
f (t )
=
f '(t )

6t

( 2t

+3

t2

2
trn [1;+ ) , ta c:
2t + 3 1 + t
2

< 0, t [1;2] .
(1 + t )2
2

34
.
Suy ra min f =
=
(t ) f(2)
t[ 2;2]
33

Vy gi tr nh nht ca P bng
546

34
t ti =
x 4,=
y 1,=
z 2.
33

www.TaiLieuLuyenThi.com
Cty TNHH MTV DVVH Khang Vit

Nhn xt. Ngoi ra c th chng minh bng tam thc bc hai, kho st hm nhiu
bin(chng 1 chng 3).
V d 8. Cho a,b,c l cc s thc dng tha mn iu kin abc = 1 v
max {a, b, c} 4 . Tm gi tr ln nht ca biu thc
P=

1
1 + a2

1
.
1+ c

1 + b2
Li gii
Nhn xt. Vi vai tr ca a v b nh nhau lm ta ngh n bt ng thc quen
1
1
2
.
thuc
+

1 + ab
1 + a2
1 + b2

1
1 . Vy ta cn chng
ab
minh rng P t gi tr ln nht trong trng hp c = max {a, b, c} .

c c iu ny ta cn c ab 1 . Do =
c

Tht vy gi s a = max {a, b, c} . Khi a 1 v ta cn chng minh


1

1 + a2

1
1 + b2

1
1
1
1

+
+
1+ c
1+ a
1 + c2
1 + b2

1
1
1
1

(1) .
1+ c
1+ a
1 + c2
1 + a2

+ Nu c < 1 VT(1) < 0 < VP(1) bt ng thc lun ng.


+ Nu c 1 khi xt hm s=
f ( x)

1
1+ x

1
1 + x2

2 x (1 + x ) 1 + x 2
=
3
3
2 (1 + x ) 1 + x 2
3

+
f '( x) =
3
2 (1 + x )

(1 + x )

2 3

trn on [1;4] ta c

>0.

Do
2 x (1 + x ) x x . (1 + x ) =
3

x3 (1 + x ) =
3

(x + x )

2 3

(1 + x ) , x [1;4] .
2 3

Do f(x) l hm ng bin trn on [1;4] . Nn c a f (c) f(a) . Ta c


(1) ng.
Vy P t gi tr ln nht khi v ch khi c = max {a, b, c} v ta c c 1, ab 1 .
p dng bt ng thc phn tch trn ta c
P

2
1
2
1
2 c +1
.
+
=
+
=
g (c ) =
1 + ab
1+ c
1
1+ c
1+ c
1+
c
547

www.TaiLieuLuyenThi.com
Khm ph t duy K thut gii bt T Bi ton Max Min ng Thnh Nam

Xt hm s g (c) =
=
g '(c)

2 c +1
lin tc trn on [1;4] ta c
1+ c

2 c
2 (1 + c ) c 2 + c

0, c [1;4] .

Do g(c) l hm ng bin trn on [1;4] suy ra P g (c) g (4) =


5.
1
ng thc xy ra khi v ch khi a =b, c =4, abc =1 a =b = , c =4 .
2
1
V d 9. Cho x,y,z l cc s thc thuc on ;3 .
3
Tm gi tr ln nht v gi tr nh nht ca biu thc
x
y
z
.
P=
+
+
x+ y y+z z+x

Li gii
Trong chng 1 ti trnh by bng cch s dng tam thc bc hai di y
chng ta p dng bi ton nu.
+ Tm gi tr nh nht.
z
Khng mt tnh tng qut =
gi s z max { x, y, z} 1 .
x
Khi s dng bt ng thc ph cho ta c
1
1
2
2
x
y
.
+
=
+

=
x + y y + z 1+ y 1+ z
y z
z
1+
.
1+
x
y
x y
x
2

Khi P
1+

z
x

Xt hm s f=
(t )

2
t2
=
+ 2
t
,=
x 1+ t t +1
1+
z
1

2
t2
lin tc trn on [1;3] ta c
+ 2
1+ t t +1

(
( t + 1) ( t

) 0, t [1;3] .

2 ( t 1) t 2 + t + 1
2

f '(t ) =

z
,1 t 3 .
x

+1

7
Do f(t) nghch bin trn on [1;3] v vy f (t ) f (3) =
.
5
1
ng thc xy ra khi v ch khi z = 9 x, x = y x = y = , z = 3 .
3
Vy gi tr nh nht ca P bng 7/5.
548

www.TaiLieuLuyenThi.com
Cty TNHH MTV DVVH Khang Vit

+ Tm gi tr ln nht
y x y
x
z
Ta c . =
1
+

x z z
x+ y z+x

Do P

y
+
y+z

2
1+

2
y
z

1+

2
t2
y 1
= + 2
, t = ;1 .
z 3
y t +1 t +1
z

Xt hm s tng t trn ta c P g (t ) =

2
t2
1 8
+ 2
g = .
t +1 t +1
3 5

Vy gi tr ln nht ca P bng 8/5 t ti x= z= 3, y=

1
.
3

1
tho mn iu kin abc = 1 .
9
a
b
1
Tm gi tr nh nht ca biu thc P =
.
+
+
b + ab c + ab 1 + c
Li gii
S dng bt ng thc C S ta c

V d 10. Cho cc s thc a, b, c

(a + b)
a
b
+

b + ab a + ab ab + a 2b + ab + b 2 a
2

2
a + b)
(
=
ab ( a + b ) + 2ab

1
2ab
ab
+
a + b ( a + b )2

2
=
1 1 + ab
ab
+
2 ab 2
Du bng xy ra khi v ch khi a = b .
2
1
2
ab
.
Khi P
+
=
+
1 + ab 1 + c 1 + ab ab + 1

t t = ab . Do a, b, c
Xt hm s f=
(t )

1
1
t=
9
9

ab=

1
2
t2
3; P
+ 2
.
1+ t t +1
c

2
t2
lin tc trn on [1/9;3] ta c
+ 2
1+ t t +1

(
( t + 1) ( t
2

f '(t ) =

) 0, t [1;3] .

2 ( t 1) t 2 + t + 1
2

+1

549

www.TaiLieuLuyenThi.com
Khm ph t duy K thut gii bt T Bi ton Max Min ng Thnh Nam

7
Do f(t) nghch bin trn on [1/9;3] v vy f (t ) f (3) =
.
5
1
.
ng thc xy ra khi v ch khi a= b= 3, c=
9
Vy gi tr nh nht ca P bng 7/5.

BI TP RN LUYN
Bi 1. Cho x,y,z l cc s thc dng tha mn iu kin x z .
Tm gi tr ln nht ca biu thc
P=

x
x +y
2

y +z
2

HD: P =

y
1+
x

z
.
x+z

z
2
z/x

+
5.
x+z
1+ z / x
1+ z / x

z
1+
y
Bi 2. Cho cc s thc khng m thay i tha mn iu kin a b c v abc = 1 .
1
1
2
.
Tm gi tr ln nht ca biu thc P =
+
+
2
2
1+ a
1+ b 1+ c

HD: P

2
2
+
=
2.
1 + ab 1 + c

Bi 3. Cho x,y,z l cc s thc dng tha mn iu kin xy 1, z 1 .


Tm gi tr nh nht ca biu thc P =

HD: P =

x
y
z3 + 2
.
+
+
y + 1 x + 1 3 ( xy + 1)

2 2 xy + 1
1
1
z3 + 2
1
3
+
2
+
2 .
+
xy + 1
2
1 + xy
x + 1 y + 1 3 ( xy + 1)

( x + y + 1)

Bi 4. Cho x,y,z l cc s thc dng tha mn iu kin x 2 + y 2 + z 2 =


1.
1

Tm gi tr nh nht ca biu thc P =


HD: S dng bt ng thc:
Suy ra P

550

1
x 2 + xy

2 3
2
8
.
+

1+ z
3
1 z2

x + xy
2

1
y 2 + xy

y + xy
2

2
x2 + y 2

2 3
.
1+ z

www.TaiLieuLuyenThi.com
Cty TNHH MTV DVVH Khang Vit

Bi 5. Cho a,b,c l cc s thc dng khng nh hn 1.


Tm gi tr nh nht ca biu thc
1
2
3
P=
+
+
+ 6 1 + abc ( abc 1) .
6
3
1 + a 1 + b 1 + c2
HD: S dng bt ng thc ph sau:
1
1
2
1
1
1
3
+

, ab 1 v
+
+

, a, b, c 1
2
2
3
3
3
1 + ab
1+ a
1+ b
1 + a 1 + b 1 + c 1 + abc
1
4
1
1
2
1
.
+

Ta c 2
;
3
2
6
2
3
1 + a 3c
1+ b 1+ c 1+ c b 1+ a 1+ c
Suy ra
1
1
1
P 2
+
+
3
1 + ca
1 + c b3 1 + c b3

6
+ 6 1 + abc ( abc 1) .
+ 6 1 + abc ( abc 1)

1 + abc

t
=
t abc, ( t 1) ta c: P f (t ) =

6
+ 6 1 + t ( t 1) vi t 1 ta c
t +1

Xt hm s f (t ) =
f '(t ) =

( t + 1)2

6
+ 6 1 + t ( t 1) .
t +1

3 ( 2t 1)

3 ( 2t 1)( t + 1) 6 t 2 t + 1
=
> 0, t 1 .
2
2
2
t t +1
( t + 1) t t + 1
2

Do f(t) l hm ng bin trn [1;+ ) .


Suy ra P f (t ) f (1) =
9 . ng thc xy ra khi v ch khi a= b= c= 1 .
Vy gi tr nh nht ca P bng 9 t ti a= b= c= 1 .
Bi 6. Cho a,b,c l cc s thc khng m tha mn iu kin a 2 + b 2 + c 2 =
1.
1
1
1
.
Tm gi tr ln nht ca biu thc P =
+
+
a +1
b +1
c+2
HD:
2

1
1
1
1
+

= a +1 + b +1 +
b +1
a +1

Suy ra

(1 + a )(1 + b )

a+b+2
2
+
ab + a + b + 1
ab + a + b + 1

a+b+2
2
1

+
=
1+

a + b +1
a + b +1
a + b +1

1
1
1
.
+
1+
a +1
b +1
a + b +1

Mt khc: ( a + b + c ) a 2 + b 2 + c 2 =
1 a + b 1 c .
2

551

www.TaiLieuLuyenThi.com
Khm ph t duy K thut gii bt T Bi ton Max Min ng Thnh Nam

Suy ra P f (c) =
1+

1
1
.
+
2c
c+2

1
1
lin tc trn on [ 0;1] ta c:
Xt hm s f (c) =
1+
+
2c
c+2

1
=
f '(c)
2

0, c [ 0;1]. Do f(c) l hm ng bin

3
( 2 + c )

( 2 c )3

trn [ 0;1] .

1
.
3
ng thc xy ra khi v ch khi c= 1, a= b= 0 .

Suy ra P f (c) f (1) =


2+

Vy gi tr ln nht ca P bng 2 +
Nhn xt. Ta c th nh gi
1
2c

1
c+2

1
t ti a= b= 0, c= 1 .
3

1
1
bng bin i tng ng nh sau:
+
2c
c+2
2

1
1
+

=
c+2
2c

1
1
2
+
+
2c 2+c
4 c2

4
2
4
2
1
= 2+

+
=
1+
4 1
4 1
3
4c
4 c2

Bi 7. Cho a,b,c,d l cc s thc dng tha mn iu kin a b c d v


abcd = 1 .
1
1
1
3
.
Tm gi tr nh nht ca biu th P =
+
+
+
1+ a 1+ b 1+ c 1+ d
HD: Do a b c d v abcd = 1 nn ab 1 cd d 2 .
1
1
2
2
2
2d
.
Vi ab 1 ta c:
+

=
1 1+ d
1 + a 1 + b 1 + ab 1 + 1
1+
d
cd
1
1
1
1
1
1
c

+
=
+
= 1.
c 1+ c 1+ d 1+ c 1+ 1 1+ c 1+ c
c
2d
2
Mt khc:
+
=
2.
1+ d 1+ d

V cd 1 d

552

www.TaiLieuLuyenThi.com
Cty TNHH MTV DVVH Khang Vit

1 1
1
2
2d
2
1
Suy ra P =
+
+

+1+
= 3.
+
+
1+ d
1+ a 1+ b 1+ c 1+ d 1+ d 1+ d
Vy gi tr nh nht ca P bng 3 t ti a= b= c= d= 1 .
Bi ton 2. Cho a,b,c l cc s thc khng m tho mn iu kin

a 2 + b2 + c2 =
k , trong k 1 .
Chng minh rng

a) k + 2bc 2a ( b + c ) .
b) ( a + b + c ) 2k (1 + bc ) .
2

2
c) a + b + c abc 2k .
k
Chng minh.

a) Theo gi thit ta c: k + 2bc =a 2 + b 2 + c 2 + 2bc =a 2 + ( b + c ) .


2

S dng bt ng thc AM GM ta c:
a 2 + ( b + c ) 2a ( b + c ) k + 2bc 2a ( b + c ) .
2

Bt ng thc c chng minh.


ng thc xy ra khi v ch khi a= b=

k
, c= 0 hoc a= c=
2

b) Ta cn chng minh: k + 2a ( b + c ) + 2bc 2k 1 + b 2 c 2 + 2bc

k
, c= 0 .
2

2kb 2 c 2 + 2 ( 2k 1) bc + k 2a ( b + c )

S dng bt ng thc chng cu a) ta ch cn chng minh


2kb 2 c 2 + 2 ( 2k 1) bc + k k + 2bc
kb 2 c 2 + 4 ( k 1) bc 0 bc ( kbc + 4 ( k 1) ) 0

Bt ng thc cui lun ng do k 1 .


Nhn xt. T b) ta c mt bt ng thc rt hay s dng l:

a
2k a
.

1 + bc a + b + c

c) S dng bt ng thc C S ta c
2
a 1 bc + b + c
k
=

( k + 2bc )

(a

+ (b + c )

2 2
1 bc + 1
k

b 2 c 2 bc + 2
k
k

Vy ta ch cn chng minh
553

www.TaiLieuLuyenThi.com
Khm ph t duy K thut gii bt T Bi ton Max Min ng Thnh Nam

( k + 2bc )

b 2 c 2 bc + 2 2k
k
k

4b 2 c 2
0 4b 2 c 2 ( 2bc k ) 0
2
k
k
Bt ng thc lun ng v theo AM GM ta c

8b3c3

2bc b 2 + c 2 a 2 + b 2 + c 2 =
k.
Cc bt ng thc dng ny trnh by trong chng 2 (K thut s dng bt
ng thc C-S).

CC V D
V d 1. (TSH Khi A,A1/2014) Cho x,y,z l cc s thc khng m tha mn
iu kin x 2 + y 2 + z 2 =
2.
Tm gi tr ln nht ca biu thc P=

x2
x + yz + x + 1
2

y+z
1 + yz
.

x + y + z +1
9

Li gii
Nhn xt. Vi cc bt ng thc v bi ton cc tr vi iu kin cc bin khng
m thng thng du bng xy ra ti mt bin bng 0 v P ln nht ta thy
yz = 0 l hp l nht. Vy ta tm cch nh gi P theo i lng 1 + yz v
y = 0 hoc z = 0 .
Ta c 2 (1 + yz ) =x 2 + ( y + z ) 2 x ( y + z ) .
2

Du bng xy ra khi v ch khi x= y + z .


Suy ra x 2 + yz + x + 1 x 2 + x + x ( y + z )= x ( x + y + z + 1) .
Do P

1 + yz
x2
y+z
+

9
x ( x + y + z + 1) x + y + z + 1

1 + yz
1
1 + yz
x+ y+z
=

=
+
1

9
9
x + y + z +1
x + y + z +1
Theo bt ng thc C-S ta c
2
x + y + z 2 x 2 + ( y + z ) = 2 1 + yz .

Suy ra P 1

1
1 + yz
.

9
2 1 + yz + 1

t t =1 + yz , ( t 1) ta c P f (t ) =
1

554

1
t2
.
2t + 1 9

www.TaiLieuLuyenThi.com
Cty TNHH MTV DVVH Khang Vit

1
t2
Xt hm s f (t ) =
1
vi t 1 ta c
2t + 1 9

f '(t ) =

( 2t + 1)2

2t 18 2t ( 2t + 1)
2
; f '(t) = 0 18 2t ( 2t + 1) = 0 t = 1 .
=
2
9
9 ( 2t + 1)
2

Ta c f(t) i du t dng sang m khi i qua t = 1 .


Do f(t) t cc i ti t = 1 .
5
. ng thc xy ra khi v ch khi
Do P f (t ) f (1) =
9
1
1 + yz =
x= y + z

x 1,=
y 1,=
z 0 hoc=
x 1,=
y 0,=
z 1.
=
2
2
2
2
x + y + z =
x, y , z 0

5
Vy gi tr ln nht ca P bng
t ti =
x 1,=
y 0,=
z 1 hoc
9
=
x 1,=
y 0,=
z 1.

V d 2. Cho a,b,c l cc s thc khng m tho mn iu kin a 2 + b 2 + c 2 =


1.
2a + c 2b + c a + b + c
.
Tm gi tr ln nht ca biu thc P =
+
+
1 + bc 1 + ca 1 + 2abc
Li gii
Ta c
2a + c
2a + c
2.
( a + b + c )2 2 (1 + bc )2
1 + bc
a+b+c
2b + c
2b + c
2.
( a + b + c )2 2 (1 + ca )2
1 + ca
a+b+c
2a + c 2b + c
2a + c
2b + c
2 2

+
2.
+ 2.
=
1 + bc 1 + ca
a+b+c
a+b+c
Theo bt ng thc chng minh ti cu c) bi ton ph ta c:
a + b + c 2abc 2 .
Tht vy s dng bt ng thc C S ta c
a + b + c 2abc= a (1 2bc ) + ( b + c )

) ((1 2bc ) + 1)

(a

(1 + 2bc ) ( 4b2c 2 4bc + 2 )

+ (b + c )

Vy ta ch cn chng minh
555

www.TaiLieuLuyenThi.com
Khm ph t duy K thut gii bt T Bi ton Max Min ng Thnh Nam

(1 + 2bc ) ( 4b2c 2 4bc + 2 ) 2 4b2c 2 ( 2bc 1) 0 .


Bt ng thc cui ng v theo AM GM ta c
2bc b 2 + c 2 a 2 + b 2 + c 2 =
1.

Bt ng thc c chng minh. T suy ra P 3 2 .


1
Du bng t ti a= b=
, c= 0 .
2
Bi tp tng t
Cho a,b,c l cc s thc khng m tho mn iu kin a 2 + b 2 + c 2 =
2.
2a + c 2b + c a + b + c
Chng minh rng
+
+
5.
1 + bc 1 + ca abc + 2
V d 3. Cho x,y,z l cc s thc khng m tho mn iu kin x 2 + y 2 + z 2 =
1.
Chng minh rng 1

x
y
z
+
+
2.
1 + yz 1 + zx 1 + xy

Li gii
Chng minh bt ng thc v phi
Ch ( x + y + z ) 2 (1 + yz )
2

Tng t ta c:

x
2x
.

1 + yz x + y + z

y
2y
z
2z
.

1 + zx x + y + z 1 + xy x + y + z

Cng theo v ba bt ng thc trn ta c pcm.


1
ng thc xy ra khi v ch khi x= y=
, z= 0 hoc cc hon v.
2
V d 4. Cho x,y,z l cc s thc khng m tho mn iu kin x 2 + y 2 + z 2 =
2.
Chng minh rng

x2
x 2 + yz + 1

y2
y 2 + zx + 1

z2
z 2 + xy + 1

1.

Li gii
Ta c 2 + 2 yz = x 2 + ( y + z ) 2 x ( y + z )
2

1 + yz x ( y + z ) x 2 + yz + 1 x ( x + y + z )

x2
x + yz + 1
2

Tng t ta c:
556

x
x+ y+z

y2
y 2 + zx + 1

y
z2
z
; 2

x + y + z z + xy + 1 x + y + z

www.TaiLieuLuyenThi.com
Cty TNHH MTV DVVH Khang Vit

Cng li theo v ba bt ng thc trn ta c pcm.


ng thc xy ra khi v ch khi x= y= 1, z= 0 hoc cc hon v.
V d 5. Cho x,y,z l cc s thc khng m tha mn iu kin x 2 + y 2 + z 2 =
2.
Tm gi tr ln nht ca biu thc
x2

M=

x + yz + x + 1
2

y+z
1
.
+
x + y + z + 1 xyz + 3

Li gii
S dng bt ng thc AM GM ta c:
2 + 2 yz =x 2 + y 2 + z 2 + 2 yz =x 2 + ( y + z ) 2 x ( y + z )
2

1 + yz x ( y + z ) x 2 + x + yz + 1 x ( x + y + z + 1)

Do

x2
x + x + yz + 1
2

x
.
x + y + z +1

Ta chng minh x + y + z xyz 2 .


Tht vy s dng bt ng thc C S ta c
x + y + z xyz = x (1 yz ) + ( y + z )
=

(x

+ ( y + z)

2 (1 + yz ) y 2 z 2 2 yz + 2=

) ((1 yz ) + 1)
2

y 2 z 2 ( yz 1) + 4 2

Bt ng thc cui ng v theo AM GM ta c


y 2 + z 2 x2 + y 2 + z 2

=
1.
2
2
x
y+z
1
T suy ra M
+
+
=
1.
x + y + z +1 x + y + z +1 x + y + z +1
yz

Vi x= y= 1, z= 0 th M bng 1. Vy gi tr ln nht ca M bng 1.


Bi tp tng t
Cho x,y,z l cc s thc khng m tho mn iu kin x 2 + y 2 + z 2 =
8.
Chng minh rng

x2
x + x + 4 + yz
2

y+z
2
+
1.
x + y + z + 1 xyz + 5 2

BI TP RN LUYN
Bi 1. Cho x,y,z l cc s thc khng m tho mn iu kin x 2 + 4 y 2 + z 2 =
8.
Tm gi tr nh nht ca biu thc
4 yz + 8
=
P
+2
xz + 2 yx + 2 yz

( y 1)2 + ( z 1)2 + 3 yz .
557

www.TaiLieuLuyenThi.com
Khm ph t duy K thut gii bt T Bi ton Max Min ng Thnh Nam

HD: Ch
4 yz + 8 = 4 yz + 4 y 2 + z 2 + x 2 = x 2 + ( 2 y + z ) 2 x ( 2 y + z )
2

2 yz + 4 x ( 2 y + z )

4 yz + 8
4 yz + 8 yz + 2 .

=
xz + 2 yx + 2 yz 4 yz + 4 yz + 1

Nhm thnh hng ng thc ta c:

( y 1)2 + ( z 1)2 + 3 yz =( y + z 1)2 + 1 + yz 1 + yz .


Suy ra P

yz + 2
+ 4 (1 + yz ) 4 .
yz + 1

Bi 2. Cho x,y,z l cc s thc khng m tho mn iu kin x 2 + y 2 + z 2 =


2.
Chng minh rng
HD: Chng minh

x
y
z
+
+
2.
1 + yz 1 + zx 1 + xy

x
2x
y
2y
z
2z
.

1 + yz x + y + z 1 + zx x + y + z 1 + xy x + y + z

Bi 3. Cho x,y,z l cc s thc khng m tho mn iu kin x 2 + y 2 + z 2 =


2.
Chng minh rng

x
y
z
+
+
2.
1 + yz
1 + zx 2 (1 + xy )

HD: S dng bt ng thc AM GM ta c:

x
y
1 x
y
+

+
+ 2 .
1 + yz
1 + zx 2 1 + yz 1 + zx

Chng minh

x
2x
y
2y
z
2z
.

1 + yz x + y + z 1 + zx x + y + z 1 + xy x + y + z

Bi 4. Cho a,b,c l cc s thc khng m tho mn iu kin a 2 + b 2 + c 2 =


2.
2a + c 2b + c a + b + c
Chng minh rng
+
+
5.
1 + bc 1 + ca abc + 2
a+b+c
a+b+c
HD: Chng minh a + b + c abc 2;1 + bc
.
;1 + ca
2
2
5
Bi 5. Cho a,b,c l cc s thc khng m tho mn iu kin a 2 + b 2 + c 2 =.
2
18
Tm gi tr ln nht ca biu thc P = ab + bc + ca
.
2
9 5 (1 + bc ) + 1
HD: ab
=
+ bc + ca
558

( a + b + c )2
2

5
2 5 (1 + bc )2 5
2
4

www.TaiLieuLuyenThi.com
Cty TNHH MTV DVVH Khang Vit

1
36
2
5 33 .
P 5 (1 + bc )

2
2
4
9 5 (1 + bc ) + 1 4

Bi 6. Cho x,y,z l cc s thc khng m tha mn iu kin x 4 + y 4 + z 4 =


2.
Tm gi tr ln nht ca biu thc
P

1 + yz ( 2 y
( y + z )2
+

2
x 4 + x 2 + yz ( 2 y 2 + 2 z 2 + 3 yz ) + 1 x 2 + 1 + ( y + z )
x4

HD: Ta c

2 + 2 yz 2 y 2 + 2 z 2 + 3 yz =x 4 + y 4 + z 4 + 2 yz 2 y 2 + 2 z 2 + 3 yz
= x4 + ( y + z ) 2 x2 ( y + z )
4

+ 2 z 2 + 3 yz

1 + yz 2 y 2 + 2 z 2 + 3 yz x 2 ( y + z )

x 4 + x 2 + yz 2 y 2 + 2 z 2 + 3 yz + 1
2
2
x4 + x2 + x2 ( y +=
z)
x2 x2 + 1 + ( y + z )

x4

x 4 + x 2 + yz 2 y 2 + 2 z 2 + 3 yz + 1

Mt khc: 1 + yz 2 y 2 + 2 =
z 2 + 3 yz

Suy ra P

x2 + ( y + z )

x +1+ ( y + z)
2

(x

x2
x2 + 1 + ( y + z )

x4 + ( y + z )
2

+ ( y + z)

t t = x 2 + ( y + z ) , ( t > 0 ) ta c P f (t ) =
Xt hm s f=
(t )
f '(t ) =

( t + 1)

1 2
2 2
x + ( y + z) .

2 2

36

.
t
t2
.

t + 1 36

t
t2
vi t > 0 ta c

t + 1 36

t
; f '(t ) = 0 t = 2 (do t > 0 ).
18

Ta c f(t) i du t dng sang m khi i qua t = 2 nn f(t) t cc i ti t = 2 .


5
.
Do P f (t ) f (2) =
9
ng thc xy ra khi v ch khi ( x; y; z ) = (1;1;0 ) ; (1;0;1) .
559

www.TaiLieuLuyenThi.com
Khm ph t duy K thut gii bt T Bi ton Max Min ng Thnh Nam

Vy gi tr ln nht ca P bng

5
.
9

ng thc xy ra khi v ch khi ( x; y; z ) = (1;1;0 ) ; (1;0;1) .


Cch 2: Ta c:
2 + 2 yz (2 y 2 + 2 z 2 + 3 yz ) =x 4 + y 4 + z 4 + 2 yz (2 y 2 + 2 z 2 + 3 yz )
= x 4 + ( y + z )4 2 x 2 ( y + z )2

Do :
P

x2
x 2 + ( y + z )2 + 1
x 2 + ( y + z )2
x 2 + ( y + z )2 + 1

( y + z )2
x 2 + ( y + z )2 + 1

x 4 + ( y + z )4
18

x 4 + ( y + z )4
18

1
x 4 + ( y + z )4
=
+
1 2

2
18
x + ( y + z ) + 1

2
x 4 + ( y + z )4 + 4 4 5
1 4
+

4
18
18 9
x + ( y + z ) + 4

Vy MaxP
=

5
khi =
x 1,=
y 1,=
z 0 hoc=
x 1,=
y 0,=
z 1.
9

Bi 7. Cho x,y,z l cc s thc khng m tha mn iu kin x3 + y 3 + z 3 =


2.
Tm gi tr ln nht ca biu thc
P
=

x2
x 2 + x + yz ( y + z ) + 1

2 (1 + yz ( y + z ) )
y+z
.

27
x + y + z +1

HD: Ta c
3 + 3 yz ( y + z ) =+
x3 y 3 + z 3 + 3 yz ( y + z ) + 1 =+
x3 ( y + z ) + 1 3x ( y + z )
3

x ( y + z ) 1 + yz ( y + z )
x 2 + x + yz ( y + z ) + 1 x 2 + x + x ( y + z )= x ( x + y + z + 1)

x2
x + x + yz ( y + z ) + 1
2

560

x
x + y + z +1

2 (1 + yz ( y + z ) )
x+ y+z

27
x + y + z +1

www.TaiLieuLuyenThi.com
Cty TNHH MTV DVVH Khang Vit

Mt khc ta c

( x + y + z )3 =

x3 + y 3 + z 3 + 3x ( y + z )( x + y + z )

= 2 + 3x ( y + z )( x + y + z )
2 + ( x + y + z ) ( 3 + 3 yz ( y + z ) )

3
x + y + z) 2
(
1 + yz ( y + z )
3( x + y + z )

x+ y+z
2 (x + y + z) 2
.
Do P
.
1 + x + y + z 81
x+ y+z
3

2 t3 2
t
t t = x + y + z v t x + y + z t 2 v P f (t ) =
.

t +1
81t
3

2 t3 2
t
Xt hm s f=
vi t 3 2 ta c

(t )
t +1
81t
f '(t ) =

( t + 1)

) ; f '(t ) = 0 t = 2 .

4 t3 + 1
81t

Ta c f(t) i du t dng sang m khi i qua t = 2 nn f(t) t cc i ti


t=2.
16
.
Do P f (t ) f (2) =
27
16
.
Vi x= y= 1, z= 0 th P =
27
16
Vy gi tr ln nht ca P bng
xy ra khi ( x; y; z ) = (1;1;0 ) ; (1;0;1) .
27
Bi 8. Cho a,b,c l cc s thc khng m tho mn iu kin a 2 + b 2 + c 2 =
1.
Tm gi tr ln nht ca biu thc P =

a
b
(1 + bc) 2
.
+

1 + bc 1 + ac 3 + 2 2

Kt hp nh gi bt ng thc AM GM v Cauchy Schwarz


Bt ng thc hoc bi ton cc tr c dng i xng gia hai bin s ta thng
s dng nh gi bng bt ng thc AM GM hoc Cauchy Schwarz gia hai
biu thc i xng v biu thc cn li a v kho st hm s.
CC V D
V d 1. Cho cc s thc tho mn iu kin x > y > z > 0 .
561

www.TaiLieuLuyenThi.com
Khm ph t duy K thut gii bt T Bi ton Max Min ng Thnh Nam

Tm gi tr nh nht ca biu thc P =

y
z
+
+
x y y z 8x

x2
xz z

Li gii
y
z
x
t a = , b = , c = abc =
1.
x
y
z

Khi P =

a
b
c2
.
+
+
1 a 1 b 8 c 1

S dng bt ng thc AM GM cho hai s dng ta c


a
b
+

1 a 1 b

Suy ra P

2 ab

(1 a )(1 b )

4 ab
4 ab
2 ab

= .
2 a b 2 2 ab 1 ab

2 ab
c2
c 2 + 16
+
= .
1 ab 8 c 1 8 c 1

c 1

c 1

3c 2 4c c 16
16 c

) (

c 2 + 16

Xt hm s f (c) =

f '(c) =

trn khong (1; +) ta c

; f '(c) = 0 3c 2 16 = 4c c c = 4 .

Lp bng bin thin ta c kt qu P=


min f=
(c) f=
(4) 4 .
min
ng thc xy ra khi v ch khi a = b, abc = 1, c = 4 x = 2 y = 4 z .
V d 2. (TSH Khi B 2013) Cho a,b,c l cc s thc dng.
Tm gi tr ln nht ca biu thc
4
9
.
=
P

2
2
2
a + b + c + 4 ( a + b ) ( a + 2c )( b + 2c )
Li gii
Nhn xt. Nhn thy a,b i xng vi nhau nn vai tr ca a + 2c, b + 2c nh nhau
ta s dng AM-GM cho hai s
S dng bt ng thc AM-GM ta c:
a + 2c + b + 2c
( a + b ) ( a + 2c )( b + 2c ) ( a + b ) .
2

( a + b )2 + 4c ( a + b ) ( a + b )2 + 4c 2 + ( a + b )2

=( a + b ) + 2c 2 2 a 2 + b 2 + 2c 2 =2 a 2 + b 2 + c 2
2

562

www.TaiLieuLuyenThi.com
Cty TNHH MTV DVVH Khang Vit

Vy P

a 2 + b2 + c2 + 4

2 a + b2 + c2
2

a 2 + b 2 + c 2 + 4, ( t > 2 ) P

t t=

Xt hm s f (t )=

f '(t ) =

4
t

4
9
.

2
t 2 t 4

4
9
trn khong ( 2;+ ) ta c:

2
t 2 t 4

9t

(t

)
( 4 t ) ( 4t
=

+ 7t 2 4t 16

t2 t2 4

) ; f '(t) =0 t =4 .

Ta c f(t) i du t dng sang m khi i qua t = 4 nn f(t) t cc i ti


t = 4 trn khong ( 2;+ ) .
5
Suy ra P f (t ) f (4) =
.
8
ng thc xy ra khi v ch khi a= b= c= 2 .
5
Vy gi tr ln nht ca P bng t ti a= b= c= 2 .
8
Bi tp tng t
Cho a,b,c l cc s thc dng. Tm gi tr nh nht ca biu thc
9
16
.
=
P

ac ( a + 2b )( c + 2b )
a 2 + b2 + c2 + 1

V d 3. Cho cc s thc x, y, z ( 0;1] tho mn iu kin x + y z + 1 .


Tm gi tr nh nht ca biu thc P =

x
y
z
.
+
+
y + z z + x xy + z 2

Li gii
S dng bt ng thc C-S v AM-GM ta c:

( x + y) + z
x2
y2
z
+
+

2
2 xy + z ( x + y ) xy + z 2
x ( y + z ) y ( z + x ) xy + z
2

P=

2
2
x + y)
2( x + y)
(
z
4z

+
=
+
2
2
2
( x + y ) + z x + y x + y + z 2 ( x + y ) + 2 z ( x + y ) ( x + y )2 + 4 z 2
(
) 2

t t = x + y, (1 + z t 2 ) khi P f (t=
)

2t 2
t + 2tz
2

4z
t + 4z2
2

.
563

www.TaiLieuLuyenThi.com
Khm ph t duy K thut gii bt T Bi ton Max Min ng Thnh Nam

Xt hm s
=
f (t )

2t 2
t + 2tz
2

f '(t=
) 4 zt
2
t + 2 zt

4z
t + 4z2
2

lin tc trn [1 + z;2] ta c

4 zt

2
2
2
t 2 + 4z2
t + 4 z

) (
2

=
0
2
2 2
t + 4z

) (
2

Do f(t) l hm nghch bin trn [1 + z;2] .


V vy
P f (t ) f (2) =

2
z
3z 2 + z + 2
+
=
1 + z 1 + z2
1 + z 2 (1 + z )

2
3 z 3 + 3 z 2 z + 1 3 (1 z ) 3 z + 1 3 3
=
=
+
+ , z ( 0;1]
2 1 + z 2 (1 + z ) 2 2 1 + z 2 (1 + z ) 2 2

3
t ti x= y= z= 1 .
2
V d 4. Cho cc s thc dng a, b, c tha mn iu kin a + b + 1 =
c.

Vy gi tr nh nht ca P bng

a2
b2
c 2 + 16
.
+
+
a + bc b + ca
c + ab
Li gii

Tm gi tr nh nht ca biu thc: F =


S dng bt ng thc C S ta c:
2
a + b)
(
F
+
( a + b )( c + 1)

c 2 + 16
c+

( a + b )2

c 1 2c 2 + 32 2c 2 + c + 31
.
= +
=
c +1
c +1
c +1

2c 2 + c + 31
, c > 1 f (c) = 0 c = 3 .
c +1
43
T suy ra : MinF =
c = 3, a = b = 1 .
4
V d 5. Cho x > 1, y > 0, z > 0 . Tm gi tr ln nht ca biu thc

Xt f ( c ) =

1
x2 + y 2 + z 2 2 x + 2

2
.
x ( y + 1)( z + 1)

Li gii
Nhn xt. D on du bng xy ra khi x = y + 1 = z + 1 nn ta s dng bt
ng thc AM-GM cho x ( y + 1)( z + 1) .
S dng bt ng thc AM-GM ta c:
564

www.TaiLieuLuyenThi.com
Cty TNHH MTV DVVH Khang Vit
3

x+ y+ z+2
x ( y + 1)( z + 1)
.
3

S dng bt ng thc C-S ta c:


x 2 + y 2 + z 2 2 x + 2 = ( x 1) + y 2 + z 2 + 1
2

Suy ra P

1
1
( x 1 + y + z + 1)2 = ( x + y + z )2 .
4
4

2
54
.

x + y + z ( x + y + z + 2 )3

2
54
.
t t = x + y + z , ( t > 1) khi P f (t ) =
t ( t + 2 )3
2
54
trn (1;+ ) ta c:

t ( t + 2 )3

Xt hm s f (t )=
f '(=
t)

162

(t + 2)

2
t

; f '(=
t ) 0 81=
t2

t >1

=
t
( t + 2 )4

4.

Ta c f(t) i du t dng sang m khi i qua t = 4 nn f(t) t cc i ti


1
.
t = 4 hay P f (t ) f (4) =
4
1
Vy gi tr ln nht ca P bng t ti x= 2, y= z= 1 .
4
Bi tp tng t
Cho x,y,z l cc s thc khng m tho mn iu kin x + y > 0 .
Tm gi tr ln nht ca biu thc
1
4
.
=
F

x 2 + y 2 + z 2 + 2 z + 2 3 ( x + y )3 ( z + 2)3
V d 6. Cho a,b,c l cc s thc dng tha mn iu kin
a 2 + b 2 + c 2 + 2ab= 3 ( a + b + c ) .

Tm gi tr nh nht ca biu thc P = 6 ( a + b ) + c 2 +

2014
2014
.
+
a+c
b+2

Li gii
Theo gi thit ta c: 3 ( a + b + c ) =
Ta c

1
2
4

( a + b )2 + c 2 ( a + b + c )2 a + b + c 6 .

1
1
4
4
.
+

=
a+c
b+2
a+c + b+2
2 ( a + c + b + 2)
2(a + b + c) + 4

6 ( a + b ) + c 2 6 ( a + b ) + 6c 9= 6 ( a + b + c ) 9 .
565

www.TaiLieuLuyenThi.com
Khm ph t duy K thut gii bt T Bi ton Max Min ng Thnh Nam

2014.4

Suy ra P 6 ( a + b + c ) +

2(a + b + c) + 4

9.
2014.4
.
2t + 4

t t = a + b + c, ( 0 < t 6 ) ta c P f (t ) =
6t +
2014.4

Xt hm s f (t=
) 6t +

f '(t ) = 6

2t + 4

8056

( 2t + 4 )3

vi t ( 0;6] ta c

( 2t + 4 )3 8056
( 2t + 4 )3

< 0, t ( 0;6] .

Do f(t) nghch bin trn ( 0;6] P f (t ) f (6) =


2050 .
a = 1
a + b = c = 3
ng thc xy ra khi v ch khi
b =
2.
a + c = b + 2
c = 3

Vy gi tr nh nht ca P bng 2050 t ti =


a 1,=
b 2,=
c 3.

Bi tp tng t
Cho a,b,c l cc s thc dng tha mn iu kin
a 2 + b 2 + c 2 = 5 ( a + b + c ) 2ab .
3
48
.
+3
a + 10
b+c
V d 7. Cho a,b,c l cc s thc dng tha mn iu kin a + b + 1 =
c.
Tm gi tr nh nht ca biu thc

Tm gi tr nh nht ca biu thc P = a + b + c + 48

P=

a3
b3
c3
+
+
+
a + bc b + ca c + ab ( c + 1)

14

( a + 1)( b + 1)

Li gii
S dng bt ng thc AM-GM ta c:
c + ab =a + b + 1 + ab =( a + 1)( b + 1)

( a + b + 2 )2 =( c + 1)2
4

Mt khc s dng bt ng thc C S ta c:

(a

+ b2

a
b
a
b
a 2 + b2
+
=
+
2
=
a + bc b + ca a ( a + bc ) b ( b + ac ) a + b 2 + 2abc 1 + 2abc
a 2 + b2 .
3

( c 1)
a 2 + b2 1 ( a + b )

=
1+ c
2 c +1
2 ( c + 1)
2

566

www.TaiLieuLuyenThi.com
Cty TNHH MTV DVVH Khang Vit

Do P f (c=
)

Ta c f '(c) =

( c 1)2
4c 3
28
+
+
2
2 ( c + 1) ( c + 1)
( c + 1)2

( 3c 5) ( 3c 2 + 14c + 23)
2 ( c + 1)

; f '(c) = 0 c =

5
.
3

5
Ta c f(c) i du t m sang dng khi i qua c = nn f(c) t cc tiu ti
3
5
5
53

.
c = hay P f (c) f =
3
3 8
1
5
ng thc xy ra khi v ch khi a= b= , c=
.
3
3
53
1
5
t ti a= b= , c=
.
Vy gi tr nh nht ca P bng
8
3
3

BI TP RN LUYN
Bi 1. Cho cc s thc dng a,b,c.
2

Tm gi tr nh nht ca biu thc P =

a
b
c
+
+
.
b+c c+a a+b

HD. S dng bt ng thc C-S ta c:

(a + b) =
a
b
a2
b2
+
=
+

b + c c + a a ( b + c ) b ( a + c ) c ( a + b ) + 2ab
2

Suy ra P

1
1
.

c
2ab
c
1
+
+
a + b ( a + b )2 a + b 2

c
.
+
1 a + b
c
+
a+b 2
1

1
c
+ t2 .
, ( t > 0 ) ta c: P f (t ) =
1
a+b
t+
2
1
1
Ta c: f '(t ) =
+ 2t ; f '(t ) =0 t = . Ta c f(t) i du t m sang
2
2
1
t +
2
1
1
dng khi i qua t = nn ti t = th f(t) t cc tiu trn ( 0;+ ) hay
2
2
1 5
.
f (t ) f =
2 4

t t
=

567

www.TaiLieuLuyenThi.com
Khm ph t duy K thut gii bt T Bi ton Max Min ng Thnh Nam

5
t ti a= b= 2c .
4
Bi 2. Cho a,b,c l cc s thc dng. Tm gi tr ln nht ca biu thc
1
1
.
=
P

2
2
2
a
+
1
b
)( + 1)( c + 1)
2 a + b + c +1 (

Vy gi tr nh nht ca P bng

HD. S dng bt ng thc C-S v AM-GM ta c:


1
2
a 2 + b 2 + c 2 + 1 ( a + b + c + 1)
4
a +b + c +3
( a + 1)( b + 1)( c + 1)

Suy ra P

1
27
.

a + b + c + 1 ( a + b + c + 3)3

1
27
.
t t = a + b + c + 1, ( t > 1) khi P f (t ) =
t ( t + 2 )3

Xt hm s f (t )=
f '(=
t)

1
t

1
27
vi t > 1 ta c:

t ( t + 2 )3
81

(t + 2)

t ) 0 81=
t2
; f '(=

2
t >1
=
=
t
9t ( t + 2 )
( t + 2 )4

4.

Ta c f(t) i du t dng sang m khi i qua t = 4 nn ti t = 4 f(t) t cc


1
.
i suy ra P f (t ) f (4) =
8
1
Vy gi tr ln nht ca P bng t ti a= b= c= 1 .
8
Bi 3. Cho a,b,c l cc s thc dng.
Tm gi tr ln nht ca
biu thc P
=

a 2 + b2 + c2 + 1
Bi 4. Cho a,b,c l cc s thc dng tha mn iu kin

( a + b + c + 3)2 .
3 ( a + 1)( b + 1)( c + 1)

a 2 + b 2 + c 2 = 5 ( a + b + c ) 2ab .

Tm gi tr nh nht ca biu thc P = a + b + c + 48


HD: T iu kin ta c: ( a + b ) + c 2= 5 ( a + b + c ) .
2

Mt khc:
568

3
48
.
+3
a + 10
b+c

www.TaiLieuLuyenThi.com
Cty TNHH MTV DVVH Khang Vit

1
1
2
2
D on c a + b = c, a + b + c = 10 c = 5, a + b = 5 ta s dng bt ng thc
AM-GM cho hai cn thc:
S dng bt ng thc AM-GM ta c:
a + 10
+4
a + 10 1 a + 10
a + 22
=
=
.4 3
3
2
3
4
22
b + c + 16
13
3
b+c
=
8.8 ( b + c )
4
12

( a + b )2 + c 2 ( a + b + c )2 ( a + b + c )2 5 ( a + b + c ) a + b + c 10 .

1
2304
1

.
Suy ra P a + b + c + 48.12
+
a+b+c+
a + b + c + 38
a + 22 b + c + 16
2304
t t = a + b + c, ( 0 < t 10 ) khi P f (t ) =
.
t+
t + 38

Ta c f '(t ) = 1

2304

( t + 38)2

( t + 38)2 2304 0, t 0;10 suy


( ]
( t + 38)2

ra f(t) nghch

bin trn ( 0;10] .


Do P f (t ) f (10) =
58 .
Vy gi tr nh nht ca P bng 58 t ti =
a 2,=
b 3,=
c 5.
Bi 5. Cho a,b,c l cc s thc dng. Tm gi tr ln nht ca biu thc
5
4
.
=
P

a 2 + 2b 2 + 5c 2 + 3 + 1 ab + bc + ca + 1
HD: S dng bt ng thc C-S ta c:
a 2 b2 c2 ( a + b + c )
+
+

=
1
1
1
1 1 1
+ +
2
3
6
2 3 6
2

2a 2 + 3b 2 + 6c 2 =

( a + b + c )2

a 2 + 2b 2 + 5c 2 2 ( ab + bc + ca )

ng thc xy ra khi v ch khi 2=


a 3=
b 6c .
5
4
.
Do P

2 ( ab + bc + ca ) + 3 + 1 ab + bc + ca + 1
Bi 8. Cho a,b,c l cc s thc thuc on [1;2] .
Tm gi tr nh nht ca biu thc P =

( a + b )2
.
c 2 + 4 ( ab + bc + ca )
569

www.TaiLieuLuyenThi.com
Khm ph t duy K thut gii bt T Bi ton Max Min ng Thnh Nam

HD : S dng bt ng thc AM-GM ta c 4ab ( a + b ) .


2

Khi P

Ta t t =

( a + b )2
2
c2 + 4 ( a + b ) c + ( a + b )

a b
+
c c
=
2
a b a b
1+ 4 + + +
c c c c

a+b
a+b
th do a, b, c [1;2] =
t
[1;4]
c
c

Xt hm s f (t ) =

4t 2 + 2t

t2

c f '(t )
=
1 + 4t + t 2

(t

+ 4t + 1

> 0, t [1;4] .

1
Do P f (t ) f (1) =
. ng thc xy ra khi v ch khi a= b= 1, c= 2 .
6
1
Vy gi tr nh nht ca P bng t ti a= b= 1; c= 2 .
6

Bi 9. Cho a,b,c l cc s thc dng. Tm gi tr nh nht ca biu thc


9
16
.
=
P

ac ( a + 2b )( c + 2b )
a 2 + b2 + c2 + 1
HD: S dng bt ng thc AM-GM ta c:
4

1
1 4a + 4c + 4b
1
4
ac ( a + 2b )( c + 2b ) .3a.3c. ( a + 2b )( c + 2b )
= (a + b + c) .
9
9
4
9

V s dng bt ng thc C-S ta c: ( a + b + c ) 9 a 2 + b 2 + c 2


4

Suy ra P
t t =

a +b +c
2

16
a 2 + b2 + c2 + 1

16
trn (1;+ ) ta c:
t 1 t

16
.
t 1 t
2

18t

(t

a 2 + b 2 + c 2 + 1, ( t > 1) khi P f (t ) =

Xt hm s =
f (t )
f '(t )=

).

3
2
16 2 ( t 2 ) 8t + 7t 2t 4
t >1
+ 2=
; f '(t )= 0
t= 2 .
2
2
2
t
t t 1

Ta c f(t) i du t m sang dng khi i qua t = 2 nn f(t) t cc tiu ti


t = 2, do P f (t) f(2) =
5 .
Vy gi tr nh nht ca P bng 5 t ti a= b= c= 1 .
570

www.TaiLieuLuyenThi.com
Cty TNHH MTV DVVH Khang Vit

Bi 10. Cho a,b,c l cc s thc dng tha mn iu kin a + b + c =


1.
Tm gi tr nh nht ca biu thc
a2

=
P

(b + c )

+ 5bc

b2

(c + a)

+ 5ca

3
( a + b )2 .
4

HD: S dng bt ng thc AM-GM ta c:


a2
a3
2a 3
=
=
( b + c )2 + 5bc a ( b + c )2 + 5abc 2a ( b + c )( b + c ) + 10abc

Tng t:

2a 3
3

2a + b + c + b + c
a+b+c

+ 10

3
3

b2

(c + a)

+ 5ca

=
3a3

3b3 .

3
3
3
( a + b )2 ( a + b )3 ( a + b )2 .
4
4
4
3
t t = a + b, ( 0 < t < 1) khi P f (t ) = t 3 t 2 .
4
3 3 2
Xt hm s =
f (t )
t t vi t ( 0;1) ta c:
4
3
2
f '(t ) = 3t 2 2t ; f '(t ) = 0 t = (t > 0) .
4
3
2
Ta c f(t) i du t m sang dng khi i qua t = nn f(t) t cc tiu ti
3
1
2
2
.
t = . Do P f (t ) f =
9
3
3

Suy ra P 3 a3 + b3

ng thc xy ra khi v ch khi

a= b= c=

1
.
3

1
1
Vy gi tr nh nht ca P bng t ti a= b= c=
.
9
3

Cch 2: Ta c ( b + c ) + 5bc 2 b 2 + c 2 + 5bc =( b + 2c )( c + 2b ) .


2

Suy ra
a2

( b + c )2 + 5bc

3a3
3a3
a2
=

= 3a3 .
3
( b + 2c )( c + 2b ) 3a ( b + 2c )( c + 2b ) 3a + b + 2c + c + 2b

571

www.TaiLieuLuyenThi.com
Khm ph t duy K thut gii bt T Bi ton Max Min ng Thnh Nam

Tng t ta c:

b2

(c + a)

+ 5ca

3b3 . Ta c kt qu tng t li gii trn.

Cch 3: S dng bt ng thc AM-GM ta c


bc

( b + c )2
4

a2

( b + c )2 + 5bc

4a 2
9 (b + c )

4 a
=

.
9 1 a

( 3a 1) 0 .
a
9
1
a
1 a 4
4
4 (1 a )
2

Mt khc

Suy ra

a2

4 9
1
a .
2
( b + c ) + 5bc 9 4 4

Tng t ta c:

Do P

b2

4 9
1
b .
2
( c + a ) + 5ca 9 4 4

2
2
4 9
1 9
1 3
2
a + b ( a + b )
9 4
4 4
4 4
2
2 9
1 9
1 3
2
a + b ( a + b )
9 4
4 4
4 4

3c 2 c 5
1

8
4 72
9
Bi ton 3. Cho a,b,c l cc s thc dng ta lun c
4
a + ab + 3 abc ( a + b + c ) .
3
Chng minh.
S dng bt ng thc AM-GM ta c:
1
1
a + ab + 3 abc =
a+
a.4b + 3 a.4b.16c
2
4
1 a + 4b 1 a + 4b + 16c 4
a+ .
+ .
=
(a + b + c)
2 2
4
3
3
Bt ng thc c chng minh. ng thc xy ra khi v ch khi =
a 4=
b 16c .
Nhn xt. y l ch l mt trng hp n gin ca nh gi AM GM t trung
bnh nhn sang trung bnh cng kt hp vi tham s ho ta c th vn dng bt
ng thc ny xy dng cc bi ton cc tr.
Bi tp tng t
1) Cho a,b,c l cc s thc khng m tha mn a + b + c =
3.
=

572

www.TaiLieuLuyenThi.com
Cty TNHH MTV DVVH Khang Vit

3
Chng minh rng 2a + b + ab + bc + 3 abc 7 .
4
2) Cho a,b,c l cc s thc dng. Chng minh
6 ab + 7c + 8 ca 9 ( a + b + c ) .

CC V D
V d 1. Cho a,b,c l cc s thc dng. Tm gi tr nh nht ca biu thc
1
3
.
=
P

3
a + ab + abc
a+b+c
Li gii
S dng bt ng thc AM-GM ta c:
1
a + 4b 3
13
a + 4b + 16c
.
ab =a.4b
, abc =
a.4b.16c
2
4
4
12
3
3
4
Suy ra ab + bc + 3 abc ( a + b + c ) do P
.

2(a + b + c)
3
a+b+c
2

3 3 1
3
3
=
1 .
2
t 2t
2
2
2t
16
4
1
3
.
Vy gi tr nh nht ca P bng t ti
=
a =
,b =
,c
21
21
21
2

t t =

a + b + c , ( t > 0 ) khi P

Bi tp tng t
Cho a,b,c l cc s thc dng. Tm gi tr nh nht ca biu thc
2
3
.
=
P

3
a + ab + abc
a+b+c
V d 2. Cho a,b,c l cc s thc dng tha mn iu kin a + b + c 1 .
Tm gi tr nh nht v ln nht ca biu thc
1
1
.
=
P

6 ab + 7c + 8 ca 9 a + b + c
Li gii
S dng bt ng thc AM-GM ta c
6 ab + 7c + 8 =
ca 2 a.9b + 7c + 4 c.4a
a + 9b + 7c + 2 ( c + 4a )
= 9(a + b + c)
2

1
1
1
1
1
1
1
Suy ra P
=

.
9 ( a + b + c ) 9 a + b + c 9 a + b + c 2 36
36
573

www.TaiLieuLuyenThi.com
Khm ph t duy K thut gii bt T Bi ton Max Min ng Thnh Nam

18

a = 23

a + b + c =
2
2

ng thc xy ra khi v ch khi


b = .
23
=
b; c 4a
a 9=

72

c = 23

Vy gi tr nh nht ca P bng

1
18
2
72
t ti
.
=
a =
,b =
,c
36
23
23
23

Bi tp tng t
Cho a,b,c l cc s thc dng. Tm gi tr nh nht ca biu thc
=
P

abc
6ab c + 7 abc3 + 8ca b

1
.
9 a+b+c

V d 3. Cho a,b,c l cc s thc dng. Tm gi tr ln nht ca biu thc


P=

8a + 3b + 4

ab + bc + 3 abc

1 + (a + b + c)

).

Li gii
S dng bt ng thc AM-GM ta c:
4=
ab 2 a.4b a + 4b
4=
bc 2 b.4c b + 4c
a + 4b + 16c
4 3 abc 3 a.4b.16c
=
3
Cng theo v 3 bt ng thc trn ta c:

8a + 3b + 4

Suy ra P

ab + bc + 3 abc
28 ( a + b + c )

3 + 3( a + b + c )

28
(a + b + c) .
3

14 14 ( a + b + c 1)
14
=

.
3 3 ( a + b + c )2 + 1
3
2

ng thc xy ra khi v ch khi a + b + c = 1, a = 4b = 16c a =


Vy gi tr ln nht ca P bng

574

14
.
3

16
4
1
.
,b =
,c =
21
21
21

www.TaiLieuLuyenThi.com
Cty TNHH MTV DVVH Khang Vit

BI TP RN LUYN
Bi 1. Cho a,b,c l cc s thc dng. Tm gi tr nh nht ca biu thc
1
8
.
=

P
2
2
2a + b + 2 2bc
2a + 2 ( a + c ) + 3
Bi 2. Cho a,b,c l cc s thc khng m tha mn a + b + c =
3.
3
Chng minh rng 2a + b + ab + bc + 3 abc 7 .
4
Bi 3. Cho a,b,c l cc s thc dng. Chng minh
6 ab + 7c + 8 ca 9 ( a + b + c ) .

Bi ton 4. Chng minh rng vi mi s thc x 1 ta c


x2 + 2
.
2
Chng minh.
S dng bt ng thc AM GM ta c
x3 + 1

( x + 1) ( x 2 x + 1)

x + 1 + x2 x + 1 x2 + 2
.
=
2
2
Bt ng thc c chng minh. Du bng xy ra khi v ch khi hoc
x = 0 hoc x = 2 .
Bt ng thc khc cng hay c dng
x3 + 1=

Chng minh vi mi s thc x ta c x 2 x + 1

x4 + 1
.
2

CC V D
V d 1. Cho a,b,c l cc s thc dng tha mn iu kin
2a 2 + 3b 2 + 5ab + 3bc + 2ac + c 3 + 5a + 8b .
1
1
1
Chng minh rng
+
+
1.
8a + 1
8b + 1
8c + 1
Li gii

Vit li iu kin ca bi ton thnh: ( a + b + c 3)( 2a + 3b + 1) 0 .


V a, b, c > 0 nn a + b + c 3 .
a
b
t
=
x 2=
, y 2=
, z 2c , ( x, y, z > 0 ) v=
xyz 2a +b + c 8 .

Ta cn chng minh

1
x +1
3

1
y +1
3

1
z +1
3

1.
575

www.TaiLieuLuyenThi.com
Khm ph t duy K thut gii bt T Bi ton Max Min ng Thnh Nam

( x + 1) ( x 2 x + 1)

x3 + 1=

Ta c

Suy ra

x3 + 1

x +2
2

x + 1 + x2 x + 1 x2 + 2
.
=
2
2
1
2
1
2
.
;
2
2
y3 + 1 y + 2 z3 + 1 z + 2

. Tng t:

ng thc xy ra khi v ch khi x= y= z= 2 .


1

Do

x3 + 1

y3 + 1

Ta ch cn chng minh

z3 + 1

2
x +2
2

x +2
2

y +2
2

2
y +2
2

2
z +2
2

2
z +2
2

1.

( )( ) ( )( ) ( )( ) (
2 ( x y + y z + z x ) + 8 ( x + y + z ) + 24
x y z + 4( x + y + z ) + 2( x y + y z + z x ) + 8
4 ( x + y + z ) + 16 x y z

)(

)(

2 y 2 + 2 z 2 + 2 + 2 x2 + 2 z 2 + 2 + 2 x2 + 2 y 2 + 2 x2 + 2 y 2 + 2 z 2 + 2
2 2

2 2

2 2 2

2 2

2 2

2 2

2 2

2 2 2

1
1
1
16
4 2 2 + 2 2 + 2 2 + 2 2 2 1 (lun ng).
y z
y z x y z
x y
V theo bt ng thc AM-GM ta c
1
1
1
16
1
16 12 16
4 2 2 + 2 2 + 2 2 + 2 2 2 12 3 4 4 4 + 2
+
=
1.
3 4
64
y z
y z x y z
x y z
8
8
x y
Bi ton c chng minh. ng thc xy ra khi v ch khi a= b= c= 1 .
V d 2. Cho x,y,z l cc s thc dng tha mn iu kin xyz = x + y + z + 2 .

Tm gi tr ln nht ca biu thc P =

1
x2 + 2

Li gii
Vit li ng thc bi cho di dng:

1
y2 + 2

1
z2 + 2

(1 + x )(1 + y )(1 + z ) = (1 + x )(1 + y ) + (1 + y )(1 + z ) + (1 + z )(1 + x ) .

1
1
1
+
+
=
1.
1+ x 1+ y 1+ z

S dng bt ng thc C-S ta c:

(x
576

2
1
+ 2 1 + ( x + 1)
2

1
x2 + 2

3 1
.
.
2 x +1

www.TaiLieuLuyenThi.com
Cty TNHH MTV DVVH Khang Vit

Tng t ta c:

y2 + 2

3 1
1
3 1
.
.
;

.
2 y + 1 z2 + 2
2 z +1

ng thc xy ra khi v ch khi x= y= z= 2 .


Cng theo v 3 bt ng thc trn ta c
1
x +2
2

y +2
2

z +2
2

3 1
1
1
3
.
+
+

=
2 1+ x 1+ y 1+ z
2

ng thc xy ra khi v ch khi x= y= z= 2 .


6
t ti x= y= z= 2 .
2

Vy gi tr ln nht ca P bng

V d 3. Cho a,b,c l cc s thc dng tha mn iu kin a 2 + b 2 = c 2 + 1 .


Tm gi tr nh nht ca biu thc
a3

P=

b3

a3 + ( b + c )

b3 + ( c + a )

2c 3 + 1
.
27

Li gii
S dng bt ng thc AM-GM ta c:

(a

+ b2 + c2

Do

) (

=a 4 + 2a 2 b 2 + c 2 + b 2 + c 2

)(

=a 4 + b 2 + c 2 b 2 + c 2 + 2a 2

a4 +

1
1
( b + c )2 ( b + c )2 + 2a 2
2
2

a4 +

1
1
( b + c )2 .2 ( b + c )2 .2a 2 = a 4 + a ( b + c )3
2
2

a3
a3 + ( b + c )

Tng t ta c:

a4

(a

+b +c
2

b3
b3 + ( c + a )

2 2

a3
a3 + ( b + c )

b2
a 2 + b2 + c2

a2
a 2 + b2 + c2

Cng theo v 2 bt ng thc trn ta c:


P

a 2 + b2
a 2 + b2 + c2

2c3 + 1 c 2 + 1 2c3 + 1
= 2
+
27
27
2c + 1

( c 1)2 ( 4c3 + 8c 2 + 14c + 7 )

27 2c + 1
2

7 7.

9 9

Du bng xy ra khi v ch khi a= b= c= 1 . Vy gi tr nh nht ca P bng 7/9.


577

www.TaiLieuLuyenThi.com
Khm ph t duy K thut gii bt T Bi ton Max Min ng Thnh Nam

V d 4. Cho a,b,c l cc s thc dng tha mn iu kin a + b = c + 1 .


Tm gi tr nh nht ca biu thc
P=3

9a 2

( b + c )2 + 5bc

9b 2

+3

( c + a )2 + 5ca

a + b + c2
.
9

Li gii
S dng bt ng thc C-S v AM-GM ta c:
9a 2

( b + c )2 + 5bc

Suy ra

9a 2

2 b 2 + c 2 + 5bc

27 a3
=
3
( a + b + c )3
3a + b + 2c + c + 2b

+ 5bc

3a
.
a+b+c

9b 2

Tng t ta c: 3
Do

( c + a ) + 5ca
3( a + b ) a + b + c2
P
+
2

a+b+c

Xt hm s
=
f (c )
f '(c) =

9a 2
27 a3
=
( b + 2c )( c + 2b ) 3a ( b + 2c )( c + 2b )

27 a3

9a 2

(b + c )

3 ( c + 1)
2c + 1

3b
.
a+b+c

3 ( c + 1) c 2 + c + 1
.
=
+
2c + 1
9

c2 + c + 1
vi c > 0 ta c
9

2c + 1 ( 2c + 1) 27
2
; f '(c) =0 ( 2c + 1) =27 c =
1.
+
=
2
9
9 ( 2c + 1)
3

( 2c + 1)2

Ta c f(c) i du t m sang dng khi i qua c = 1 nn f(c) t cc tiu ti c = 1 .


7
.
Do P f (c) f (1) =
3
ng thc xy ra khi v ch khi a= b= c= 1 .
7
Vy gi tr nh nht ca P bng t ti a= b= c= 1 .
3
BI TP RN LUYN
Bi 1. Chng minh rng vi mi s thc dng a,b,c ta c
a3
a3 + ( b + c )

578

b3
b3 + ( c + a )

c3
c3 + ( a + b )

1.

www.TaiLieuLuyenThi.com
Cty TNHH MTV DVVH Khang Vit

Bi 2. Cho cc s thc dng x, y, z tha mn iu kin x, y 1 v xyz = 1 .


Tm gi tr ln nht ca biu thc
1
1
P=
+
2
x2 x + 1
y2 y + 1

) (

HD: S dng bt ng thc: x 2 x + 1

) (
2

z2 z + 1

x4 + 1
, x .
2

2
z4 + 1
y4 + 1
v z 2 z + 1
.
2
2
2
2
2
2
2
2 x4 y 4
Suy ra P 4
.
+ 4
+ 4
= 4
+ 4
+ 4 4
x +1 y +1 z +1 x +1 y +1 x y +1
2

Tng t ta c: y 2 y + 1

Ta s chng minh

2 x4 y 4

x4 + 1 y 4 + 1 x4 y 4 + 1
Bin i tng ng ta th c:

)(

)(

3.

2 x4 y 4 1 x4 1 y 4 1 0 .

Bt ng thc ny lun ng do x, y 1 .
Vy gi tr ln nht ca P bng 3 , khi v ch khi x= y= z= 1 .
Bi ton 5. Chng minh rng vi mi s thc x,y cng du ta c
k2 + x + k2 + y k + k2 + x + y .

Chng minh.
Bnh phng hai v a v chng minh bt ng thc:

(k
(k

2
2

)(
+ x )( k

)
+ y) k (k

+ x k 2 + y 2k k 2 + x + y
2

+ x + y xy 0

Bt ng thc cui lun ng. ng thc xy ra khi v ch khi x = 0 hoc


y = 0.
Hay c dng: 1 + x + 1 + y 1 + 1 + x + y .
Ta c mt kt qu cng dng
Vi mi s thc khng m a,b,c tha mn iu kin a + b + c =
1 ta c
a + b2 + b + c2 a + b + c + b2 .

CC V D
V d 1. Cho a,b,c l cc s thc khng m c tng bng 1. Chng minh
4a + 1 + 4b + 1 + 4c + 1 5 + 2 .
579

www.TaiLieuLuyenThi.com
Khm ph t duy K thut gii bt T Bi ton Max Min ng Thnh Nam

Li gii
Ta c
1 + x + 1 + y 1 + 1 + x + y vi mi x,y khng m.

Khng mt tnh tng qut =


gi s a max {a, b, c} a

1
.
3

Khi
4b + 1 + 4c + 1 1 + 1 + 4a + 4b = 1 + 5 4a .
Gi P l biu thc v tri ta c
P 4a + 1 + 1 + 5 4a .
1
4a + 1 + 1 + 5 4a lin tc trn on ;1 ta c
3
2
2
1

; f '(a ) = 0 5 4a = 4a + 1 a = .
2
4a + 1
5 4a

Xt hm s f (a=
)
f '(a ) =

7
11 1
1
Ta c f =
1+
+
;f =
1 + 2 3; f (1) =5 + 2 .
3
3 2
3

V vy P f min =f (1) = 5 + 2 .
Bt ng thc c chng minh. ng thc xy ra khi v ch khi a = 1,
b= c= 0 hoc cc hon v.
V d 2. Cho a,b,c l cc s thc khng m thay i tha mn iu kin
1 + a 2 + 1 + 2b + 1 + 2c =
5.

Tm gi tr ln nht ca biu thc P = 2a3 + b3 + c3 .


Li gii
Theo gi thit v s dng bi ton ph trn ta c
1 + a 2 + 1 + 2b + 1 + 2c 1 + a 2 + 1 + 1 + 2b + 2c
1 + 1 + 1 + a 2 + 2b + 2c
=2 + 1 + a 2 + 2b + 2c

Suy ra: 1 + a 2 + 2b + 2c 3 a 2 + 2b + 2c 8 b + c 4

a2
.
2
3

Ta c: P = 2a + ( b + c ) 3bc ( b + c ) 2a + ( b + c )
3

T gi thit a,b,c khng m tha mn iu kin

1 + a 2 + 1 + 2b + 1 + 2c =
5

suy ra 5 1 + a 2 + 1 + 1 a 2 2 0 a 2 2 .
580

a2
2a + 4 .

www.TaiLieuLuyenThi.com
Cty TNHH MTV DVVH Khang Vit
3

a2
Xt hm s f (a ) = 2a + 4 lin tc trn 0;2 2 ta c:

a2
3
f '(a ) = 6a 3a 4 = a ( a 2 ) a 12 a 2 + 2 16 a 2 ;

2
4

.
a
0
=

0 a 2 2
f '(a )= 0

a = 2 2

) (

Ta c: f (0) =
64, f (2) =
24, f (2 2) =
32 2 max

a0;2 2

f (a) =
f(0) =
64 .

Vy gi tr ln nht ca P bng 64 t ti=


a 0,=
b 0,c
= 4 hoc=
a 0,=
b 4,=
c 0.
Bi tp tng t
Cho x,y,z l cc s thc khng m tho mn iu kin 1 + x + 1 + y + 1 + z =
5.
Tm gi tr ln nht ca biu thc P = x + y 2 z + 12 1 + z .
V d 3. Cho x,y,z l cc s thc khng m tho mn iu kin x 2 + y 2 + z 2 =
1.
1

Chng minh rng

Trong ba s

( x + y )2
( y + z )2
( z + x) 2
+ 1
+ 1
6.
4
4
4
Li gii

( x + y )2 1 ; ( y + z )2 1 ; ( z + x )2 1
4

lun tn ti hai s cng

( x + y )2 1 ( z + x )2 1
du, khng mt tnh tng qut gi s

0.
4
3 4
3

Ta c

2 ( x + y )2 1
2 ( z + x) 2 1


3 4
3
3 4
3

6
2 ( x + y )2 1 ( z + x)2 1
+

+

3
3 4
3
4
3

2
2
2
6
4 2x + 2x ( y + z ) + y + z
+

3
3
4

2
2
2
2
2
6
4 2x + 2x 2 y + z + y + z

3
3
4

581

www.TaiLieuLuyenThi.com
Khm ph t duy K thut gii bt T Bi ton Max Min ng Thnh Nam
2

2
2
6
4 y + z + 2 x
6
4 1 x2 + 2 x
.
=
+

=
+

3
3
2
3
3
2

S dng bt ng thc AM GM ta c
1

( y + z )2
4

y2 + z2
1 + x2
.
=
2
2
2

4 1 x2 + 2 x
1 + x2 2 6
+

.
Vy ta ch cn chng minh

3
2
2
3

Bt ng thc cui lun ng(bn c kim chng).


1
.
3

Bt ng thc c chng minh. ng thc xy ra khi v ch khi x= y= z=

V d 4. Cho a,b,c l cc s thc thuc on [ 1;1] v tho mn iu kin


a+b+c =
0.

Tm gi tr nh nht ca biu thc P = 1 + a + b 2 + 1 + b + c 2 + 1 + c + a 2 .


Li gii
Ta c: a + b 2 , b + c 2 , c + a 2 1 v trong 3 s ny c t nht hai s hng cng

)(

du, khng mt tnh tng qut ta gi s a + b 2 b + c 2 0 .


Khi p dng bi ton ph ta c
1 + a + b2 + 1 + b + c2 + 1 + c + a 2 1 + 1 + a + b2 + b + c2 + 1 + c + a 2
=+
1 1 + b2 + c2 c + 1 + c + a 2
=1 +
1+
=1 +

(
(
(

c2 c + 1

) +b
2

Rt gn v phi ta c: P 1 + 2c 2 + 2 1 c3 + 2 .
Ta chng minh 1 + 2c 2 + 2 1 c3 + 2 3
2c 2 + 2 1 c3 + 2 4 1 c3 1 c 2

582

1+ c

) + (a + b)
1+ c ) + c

c2 c + 1 + 1 + c
c2 c + 1 +

+
2

+ a2

www.TaiLieuLuyenThi.com
Cty TNHH MTV DVVH Khang Vit

1 c3 1 2c 2 + c 4 c 2 c 2 + c 2 0 c 2 ( c 1)( c + 2 ) 0 (lun ng

vi c [ 1;1] ).
Vy gi tr nh nht ca P bng 3 t ti a= b= c= 0 .
V d 5. Cho a,b,c l cc s thc khng m thay i tha mn iu kin
a+b+c =
1 . Tm gi tr nh nht ca biu thc
P=

a + b2 + b + c2 + c + a 2 .
Li gii

Ta c:

P a + b + c + b2 + c + a 2 a + b +
=1 c + 4c + (1 c ) =1 c +

c+ c

( c + 1)2

+ (a + b)

=2

Vy gi tr nh nht ca P bng 2 t ti a= b= c=

1
hoc a= 1, b= c= 0 v
3

cc hon v.
V d 6. Chng minh rng vi x,y l hai s thc khng m tha mn x + y 1 ta
x2 + x + 4 + y 2 + y + 4 2 +

lun c

( x + y )2 + x + y + 4 .

Li gii
Bt ng thc cho tng ng vi
x2 + y 2 + x + y + 8 + 2

(x

)(

+ x + 4 y2 + y + 4

( x + y )2 + x + y + 4 + ( x + y )2 + x + y + 4

4+4

(x

)(

+ x + 4 y 2 + y + 4 xy + 2

)(

x 2 + x + 4 y 2 + y + 4 x 2 y 2 + 4 xy

( x + y )2 + x + y + 4

( x + y )2 + x + y + 4 + 4 ( x + y )2 + x + y + 4

2
xy 4 ( x + y ) + x + y + 4 + x + y 7 0 (lun ng do x + y 1 ).

Ta c mt s kt qu cng dng
+ Chng minh rng vi mi s thc khng m x,y ta lun c

x2 x + 1 + y 2 y + 1 1 +

( x + y )2 x y + 1 .

ng thc xy ra khi v ch khi x = 0 hoc y = 0 .


+ Chng minh rng vi mi s thc khng m x,y ta lun c
x2 + x + k 2 + y 2 + y + k 2 k +

( x + y )2 + x + y + k 2

.
583

www.TaiLieuLuyenThi.com
Khm ph t duy K thut gii bt T Bi ton Max Min ng Thnh Nam

ng thc xy ra khi v ch khi x = 0 hoc y = 0 .


Bi ton p dng
Cho a,b,c l cc s thc khng m tha mn iu kin a + b + c =
3.
Tm gi tr ln nht v nh nht ca biu thc
P=

a 2 + a + 4 + b2 + b + 4 + c2 + c + 4 .

V d 7. Cho x,y,z l cc s thc khng m tho mn iu kin xy + yz + zx > 0 .


Tm gi tr nh nht ca biu thc
1
1
=
P
+
+ 4 z +1 + 2 x + 2y + 4 .
y( x + 4 z)
x( y + z ) + 2 z 2
Li gii
S dng kt qu bi ton ph ta c

4 z + 1 + 2 x + 2 y +=
4 4 z + 1 +

x
4 1 +
+
4

x y
+ + 1
4 2

y
+ z + 1
2

=4 + 2 x + 2 y + 4 z + 4

1
Ch =
x( y + z ) + 2 z 2

2 2
8 xy + 8 xz + 16 z 2

2 2
;
x + 2 y + 4z

1
2 2
2 2
v
.
=

y ( x + 4 z )) 2 2 y ( x + 4 z ) x + 2 y + 4 z

t t =x =2 y + 4 z > 0 ta suy ra: P


Xt hm f (t ) =

4 2
+ 2 t + 4 + 4.
t

4 2
+ 2 t + 4 + 4 ta c ngay Min f (t )= 4 + 5 2 .
t

Vy gi tr nh nht ca P bng 4 + 5 2 t ti ( x, y, z ) = ( 2;1;0 ) .


BI TP RN LUYN
Bi 1. Cho x,y,z l cc s thc khng m tha mn iu kin
x 2 + 4 + 3 y + 4 + 3z + 4 =
8.

Tm gi tr ln nht ca biu thc P =2 x3 + 9 y 3 + z 3 .


584

www.TaiLieuLuyenThi.com
Cty TNHH MTV DVVH Khang Vit

HD: Dng bt ng thc ph nh gi


8=

x 2 + 4 + 3 y + 4 + 3z + 4 x 2 + 4 + 2 + 4 + 3 y + 3z

4 + 4 + x 2 + 3 y + 3z y + z

12 x 2
3

3
1
12 x 2 576
3
Bi 2. Cho x,y,z l cc s thc khng m tho mn iu kin

P 2 x3 + 9 ( y + z ) 2 x3 +
3

1+ x + 1+ y + 1+ z =
5.

Tm gi tr ln nht ca biu thc P = x + y 2 z + 12 1 + z .


Bi 3. Cho x,y,z l cc s thc khng m tho mn iu kin
1 + 2 x2 + 1 + 2 y 2 + 1 + 2 z 2 =
5.

Chng minh rng 4 2 x3 + y 6 + z 6 64 .


Bi ton 6. Cho x,y,z l cc s thc khng m tha mn iu kin
xy + yz + zx > 0, z =
max { x, y, z}

Ta lun c

x
+
y+z

x+ y
.
z

z+x

Chng minh.
Tht vy ta c th gi s x y . Nu x = 0 bt ng thc tr thnh ng thc.
Vi z y x > 0 khi vit li bt ng thc di dng:
xz

( x + y )( y + z )

yz

( x + y )( x + z )

1.

Ta c
xz

( x + y )( y + z )

2 xz
2 z ( x + y ) .x ( y + z )

2 xz
2 xz
=
z ( x + y ) + x ( y + z ) xy + yz + 2 zx

2 yz

2 yz
2 yz
=

=
( y + z )( x + y ) 2 y ( x + z ) .z ( x + y ) y ( x + z ) + z ( x + y ) xy + xz + 2 yz
yz

Mt khc xy + xz + 2 yz xy + yz + 2 zx suy ra
xz

( x + y )( y + z )

yz

( x + y )( x + z )

2 ( xz + yz )
2 xz
2 yz
+
=
1.
xy + yz + 2 zx xy + yz + 2 zx xy + yz + 2 zx

V yz xz .
Bt ng thc c chng minh.
585

www.TaiLieuLuyenThi.com
Khm ph t duy K thut gii bt T Bi ton Max Min ng Thnh Nam

ng thc xy ra khi v ch khi x= y= z hoc x = 0 hoc y = 0 .


Ngoi ra ta c mt nh gi yu hn nh sau:
2( x + y)
y
.

z+x x+ y+z

x
+
y+z

Mt bt ng thc c dng tng t


Cho x,y,z l cc s thc khng
m v z min { x, y, z} ; xy + yz + zx > 0 ta c
=
y
x+ y
.
2
z+x
x + y + 2z

x
+
y+z

Chng minh.
S dng bt ng thc Holder ta c

x
+

y+z

x
+
y+z

z + x

(x

( y + z ) + y 2 ( z + x ) ) ( x + y )3
.

( x + y)
( y + z ) + y2 ( z + x)
3

z+x

Ch s dng bt ng thc AM GM ta c
x 2 ( y + z ) + y 2 ( z + x=
) xy ( x + y 2 z ) + z ( x + y )
x+ y

2
=

Suy ra

x
+
y+z

( x + y 2 z ) + z ( x + y )2 .

( x + y )4 + 2 z ( x + y )2

z+x

( x + y )3
( x + y )4 + 2 z ( x + y )2

x+ y
2
=
.
x + y + 2z

Bt ng thc c chng minh. ng thc xy ra khi v ch khi x = y .


CC V D
V d 1. Cho x,y,z l cc s thc khng m tha mn iu kin
xy + yz + zx > 0, z =
max { x, y, z}
x
y
z
.
Tm gi tr nh nht ca biu thc P = + 2
+ 33
y+z
z+x
x+ y
586

www.TaiLieuLuyenThi.com
Cty TNHH MTV DVVH Khang Vit

Li gii
S dng bt ng thc AM-GM ta c:

x
Suy ra P 2
+
y+z

Ta chng minh.

x
x
.
+1 2
y+z
y+z

y
z
1 .
+ 33
z+x
x+ y

x
+
y+z

2( x + y)
y
.

z+x x+ y+z

Tht vy nu x = 0 hoc y = 0 gi s y = 0 bt ng thc tr thnh:


x
2x
2
2

x ( x + z ) 4 x 2 z x ( x z ) 0 (lun ng).
z x+z

Vi x, y > 0 s dng bt ng thc AM-GM ta c:


x
+
y+z

Vy P

t=
t

y
=
z+x

4( x + y)
x+ y+z

x( y + z)

+ 33

y ( z + x)

2( x + y)
2x
2y
.
+
=
x+ y+z x+ y+z x+ y+z

4
z
z
1 =
+ 33
1 .
z
x+ y
x+ y
1+
x+ y

z 31
4
, t
t)
+ 3t 1 .
P f (=
2
x+ y
1 + t3

Xt hm s f (t=
)
f '(t ) =

12t 2

(1 + t )

3 2

4
1+ t

+ 3t 1 vi t 3

1
ta c:
2

)(
)

3 ( t 1) t 2 + t 1 t 3 + 2t + 1

1
3
+3=
;
f
'(
t
)
=
0

t
=
1,
t

1 + t3

Ta c f(t) i du t m sang dng khi i qua t = 1 nn f(t) t cc tiu ti


t = 1 hay P f (t ) f (1) =
4 . ng thc xy ra khi v ch khi=
y 0,=
x z.
Vy gi tr nh nht ca P bng 4 t ti=
x z=
,y 0.
Bi tp tng t
Cho x,y,z l cc s thc khng m tha mn iu kin
xy + yz + zx > 0, z =
max { x, y, z}

Tm gi tr nh nht ca biu thc P = 2

x
y
z
.
+2
+ 33
y+z
z+x
x+ y
587

www.TaiLieuLuyenThi.com
Khm ph t duy K thut gii bt T Bi ton Max Min ng Thnh Nam

V d 2. Cho x,y,z l cc s thc khng m tho mn iu kin xy + yz + zx > 0 .


Chng minh rng 1 +

48 x
48 y
48 z
+ 1+
+ 1+
15 .
y+z
z+x
x+ y

Li gii
Khng mt tnh tng qut gi s z = min { x, y, z} ta c
x
+
y+z

y
x+ y
.
2
z+x
x + y + 2z

S dng bt ng thc Mincopski v bt ng thc trn ta c

x
48 x
48 y
1+
+ 1+
4 + 48
+
y+z
z+x
y+z

z + x

x+ y
4 + 48 2
x + y + 2z

Vy ta ch cn chng minh 2 1 +

48 ( x + y )
x + y + 2z

1+

48 ( x + y )
= 2 1 +
x + y + 2z

+ 1 + 48

Bt ng thc kh n gin bng cch t t =

z
15 .
x+ y

48 z
,1 t 5 .
x+ y

Bn c t chng minh.
Bt ng thc c chng minh. ng thc xy ra khi v ch khi x= y= z hoc
=
x y=
; z 0 hoc cc hon v.

Bi tp tng t
Cho x,y,z l cc s thc dng chng minh rng
1+

16 x
16 y
16 z
+ 1+
+ 1+
9.
y+z
z+x
x+ y

V d 3. Xt cc s thc a b c > 0 tho mn iu kin a + b + c =


1.
Tm gi tr nh nht ca biu thc P =

a
b
24
.
+
+
b+c
c + a 5 5(a + b)

Li gii
Theo gi thit kt hp bt ng thc ph ta c

588

www.TaiLieuLuyenThi.com
Cty TNHH MTV DVVH Khang Vit

24
a
b
+
+
b+c
c + a 5 5(a + b)

P=
2
= 2

24
a+b
+
a + b + 2c 5 5 ( a + b )
1 c
24
+
1 + c 5 5 (1 c )

Xt hm s=
f (c ) 2

f '(c) =

1 c
24
trn na khong
+
1 + c 5 5 (1 c )

10 5 (1 c ) + 12 (1 + c ) 1 + c

5 5 1 c

1 c

1
0; ta c
3

f '(c) = 0 10 5 (1 c ) + 12 (1 + c ) 1 + c = 0

125 (1 c ) = 36 (1 + c ) 36c 17c + 358c 89= 0


2

( 4c 1) 9c 2 2c + 89 = 0 c =

1
4

78
1
T suy ra f (c) f =.
4 5 15

Du bng t ti a= b=

78
3
1
. Vy gi tr nh nht ca P bng
.
, c=
8
4
5 15

V d 4(TSH Khi B/2014) Cho a,b,c l cc s thc khng m tho mn iu


kin c ( a + b ) > 0 . Tm gi tr nh nht ca biu thc
a
b
c
.
+
+
b+c
c + a 2(a + b)

P=

Li gii
Ch

a
2a
b
2b
.

b+c a+b+c c+a a+b+c

T suy ra P

t t
=

2(a + b)
a+b+c

c
c
2
.
=
+
c
2(a + b) 1 +
2(a + b)
a+b

c
, t > 0 khi s dng bt ng thc AM GM ta c
a+b
589

www.TaiLieuLuyenThi.com
Khm ph t duy K thut gii bt T Bi ton Max Min ng Thnh Nam

2
t
2
t +1 1
2 t +1 1 3
.
+ =
+
2
.
=
1+ t 2 t +1
2
2
t +1 2
2 2

ng thc xy ra khi v ch khi=


a c=
, b 0 hoc=
a 0,=
b c.
Bi tp tng t
Cho a,b,c l cc s thc khng m tho mn iu kin a 2 + b 2 + c 2 =
2.
1 + bc

Tm gi tr nh nht ca biu thc P =

(b + c )

1 + ca

(a + c)

c
.
2(a + b)

V d 5. Cho a,b,c l cc s thc khng m tho mn iu kin ab + bc + ca > 0 .


Tm gi tr nh nht ca biu thc P =

a
b
c
.
+
+2
b+c
2c + a
a+b+c

Li gii
Ta s chng minh:

a
b
c
+
+2
2.
b+c
2c + a
a+b+c

+ Nu c bng 0 ta ch cn s dng bt ng thc AM GM cho hai s dng c


iu phi chng minh.
+ Nu a bng 0 khi ta cn chng minh
b
c
+2
2
2c
b+c
x+

b
1
+2
2
b
2c
+1
c

2, =
x
2 x2 + 1

b
0
2c

4 ( 2 x ) 2 x 2 + 1 x 2 x3 8 x 2 + 9 x 4 0
2

Bt ng thc cui ng vi x 2 .
+ Nu b bng 0 chng minh tng t trng hp trn.
+ Ta xt tt c cc s u dng khi s dng bt ng thc AM GM ta c
a
b
c
+
+2
=
2c + a
b+c
a+b+c

a
a (b + c )

b
b ( 2c + a )

2c
c (a + b + c)

2a
2b
4c
2a + 2b + 4c
2
+
+
>
=
a + b + c a + b + 2c a + b + 2 c
a + b + 2c

Bt ng thc c chng minh. ng thc xy ra khi v ch khi=


a b=
,c 0 .
25
V d 6. Cho cc s thc x, y, z [ 0;1) tha mn iu kin x + y + z = .
24
590

www.TaiLieuLuyenThi.com
Cty TNHH MTV DVVH Khang Vit

x
y
z
.
+
+
1 x
1 y
1 z

Tm gi tr nh nht ca biu thc P =

Li gii
Khng mt tnh tng qut gi s z = min { x, y, z} z

25
25
2
x + y=
z> .
72
24
3

S dng bt ng thc Holder ta c:

x
y
+

1 y
1 x

Suy ra

x
y
+

1 x
1 y

(x

(1 x ) + y 2 (1 y ) ) ( x + y )3 .

( x + y )3
.
x 2 (1 x ) + y 2 (1 y )

Ch . S dng bt ng thc AM GM ta c
x 2 (1 x ) + y 2 (1 y ) = x 2 + y 2 x3 y 3
= ( x + y ) ( x + y ) + xy ( 3 ( x + y ) 2 )
2

.
2
3 x+ y
( x + y) ( x + y) +
(3( x + y ) 2)
2
1
1
2
3
= ( x + y) ( x + y)
2
4
Suy ra

x
y
+

1 x
1 y

( x + y )3
1
1
( x + y )2 ( x + y )3
2
4

25 24 z
.
2
=
23 + 24 z

Khi P =
f ( z) 2

25 24 z
z
.
+
23 + 24 z
1 z

Xt hm
s f ( z ) 2
=

25 24 z
z
25
trn 0; ta c
+
23 + 24 z
1 z
72

=
f '( z )

1
2 ( z 1)

z
1 z

1152

( 23 + 24 z )

25 24 z
23 + 24 z

f '( z ) =
0 23042 (1 z ) z =
( 23 + 24 z ) ( 25 24 z )
3

25
25
Phng trnh cui c 2 nghim thuc on 0; l 0 < z1 < z2 = .
72
72

591

www.TaiLieuLuyenThi.com
Khm ph t duy K thut gii bt T Bi ton Max Min ng Thnh Nam

10
.
23

Lp bng bin thin suy ra min f=


( z ) f=
(0)
10

Vy gi tr nh nht ca P bng

25
, z= 0 hoc cc hon v.
48

t ti x= y=

23

Bi tp tng ng t
1) Cho a,b,c l cc s thc khng m tho mn iu kin
1
a + 47
2

1
b + 47
2

1
.
=
c + 47 24
2

47
.
23

Chng minh rng a + b + c 10

1
1 x
1
1 y
1
1 z
cho 2
.
HD: Tn ti x, y, z [ 0;1) sao =
=
; 2
=
; 2
47 c + 47 47
a + 47 47 b + 47

Khi x + y +=
z

25
;=
P
24

x
y
z
.
+
+
1 x
1 y
1 z

y chnh l bi ton trn.


2) Cho cc s thc a, b, c 1 v

a + 47
Chng minh rng a + b + c 15 .
2

1
b + 47
2

1
.
=
c + 47 24
2

BI TP RN LUYN
Bi 1. Cho x,y,z l cc s thc khng m tha mn iu kin
xy + yz + zx > 0, z =
max { x, y, z}

Tm gi tr nh nht ca biu thc P = 2


HD: Ta chng minh.

x
+
y+z

x
y
z
.
+2
+ 33
y+z
z+x
x+ y

2( x + y)
y
.

z+x x+ y+z

Tht vy nu x = 0 hoc y = 0 gi s y = 0 bt ng thc tr thnh:


x
2x
2
2

x ( x + z ) 4 x 2 z x ( x z ) 0 (lun ng).
z x+z

Vi x, y > 0 s dng bt ng thc AM-GM ta c:


x
+
y+z
592

y
=
z+x

x
x( y + z)

y
y ( z + x)

2( x + y)
2x
2y
.
+
=
x+ y+z x+ y+z x+ y+z

www.TaiLieuLuyenThi.com
Cty TNHH MTV DVVH Khang Vit

Vy P

t=
t

4( x + y)
x+ y+z

4
z
z
.
=
+ 33
x + y 1+ z
x+ y
x+ y

z 31
4
, t
t)
+ 3t .
P f (=

x+ y
2
1 + t3

Xt hm s =
f (t )
f '(t ) =

+ 33

12t 2

(1 + t )

3 2

4
1+ t

+ 3t vi t 3

1
ta c:
2

)(
)

3 ( t 1) t 2 + t 1 t 3 + 2t + 1

1
+3=
; f '(t ) =0 t =1, t 3 .
2
2

1 + t3

Ta c f(t) i du t m sang dng khi i qua t = 1 nn f(t) t cc tiu ti


t = 1 hay P f (t ) f (1) =
5 . ng thc xy ra khi v ch khi=
y 0,=
x z.
Vy gi tr nh nht ca P bng 5 t ti=
x z=
,y 0.
Bi 2. Cho x,y,z l cc s thc khng m tho mn iu kin xy + yz + zx > 0 .
Chng minh rng

x
+
y+z

y
+
z+x

z
xyz
2 1+
.
x+ y
( x + y )( y + z )( z + x )

HD: Gi s z = min { x, y, z} khi ta c


x
+
y+z

y
x+ y
.
2
z+x
x + y + 2z

Ch s dng bt ng thc AM GM ta c

( x + y) .
1
1
=

=
( x + z )( y + z ) z ( x + y + z ) + 1 4 z ( x + y + z ) + 1 ( x + y + 2 z )2
xy
( x + y )2
2

xy

Do vy ta ch cn chng minh 2

t t =

z ( x + y)
z
.
2 1+
x+ y
( x + y + 2 z )2

x+ y
+
x + y + 2z

z
a v chng minh bt ng thc mt bin.
x+ y

Bi 3. Cho a,b,c l cc s thc khng m tho mn iu kin a 2 + b 2 + c 2 =


2.
Tm gi tr nh nht ca biu thc P =

1 + bc

(b + c )

1 + ca

(a + c)

c
.
2(a + b)
593

www.TaiLieuLuyenThi.com
Khm ph t duy K thut gii bt T Bi ton Max Min ng Thnh Nam

HD: Ch
1 + bc a ( b + c ) ;1 + ca b ( a + c ) P

a
b
c
3
+
+
.
b+c
c + a 2(a + b) 2

Bi ton 7. Chng minh rng vi mi s thc x ta c e x 1 + x +

x2
xn
.
+ ... +
2!
n!

ng thc xy ra khi v ch khi x = 0 .


Chng minh. Xem ch 1 Kho st hm mt bin.

x2
xn
T bt ng thc trn ta c x ln 1 + x +
+ ... + .

2!
n1

Thng thng s dng bt ng thc x ln(1 + x), x 0 .


x2
xn
.
+ ... +
2!
n!
V d 1(TSH Khi A,A1/2012) Cho x,y,z l ba s thc tha mn iu kin
x+ y+z=
0 . Tm gi tr nh nht ca biu thc

Ch . Vi x l s thc khng m v a e ta c a x 1 + x +

P= 3

x y

+3

yz

+3

zx

6 x2 + 6 y 2 + 6 z 2 .

Li gii
Nhn xt. Biu thc P v iu kin cc bin c vai tr nh nhau nn d on du
bng xy ra ti x= y= z suy ra x= y= z= 0 khi P = 3 . Vy ta i chng
minh P khng nh hn 3.
Thay z = x y vo P ta c:
P =3

x y

+3

x+2 y

+3

y+2 x

6 x2 + 6 y 2 + 6 ( x y ) .
2

Suy ra :
P x y + 1 + x + 2 y + 1 + y + 2 x + 1 6 x2 + 6 y 2 + 6 ( x + y )

4 x + 2 y + 3 12 x 2 + 12 y 2 + 12 xy

Ta chng minh P 3 vy ta i chng minh:


4 x + 2 y + 3 12 x 2 + 12 y 2 + 12 xy 3 2 x + y 3x 2 + 3 y 2 + 3xy .

4 x 2 + 4 xy + y 2 3 x 2 + 3 y 2 + 3 xy
x 2 + xy 2 y 2 0 ( x y )( x + 2 y ) 0

594

www.TaiLieuLuyenThi.com
Cty TNHH MTV DVVH Khang Vit

bt ng thc cui ng nu ta c

x y z 2x + y 0

x + 2 y =x + y + y = z + y 0 (ch vic gi s x y z t u) do bt ng

thc c chng minh tc P 3 .


Vy gi tr nh nht ca P bng 3 t ti x= y= z= 0 .
Nhn xt. Nh vy bng vic d on du bng xy ra v bin i bt ng thc
tng ng ta ngh n vic gi s x y z y chnh l nt tht a n li gii
ht sc t nhin nh trn.
V d 2. Cho x,y,z l cc s thc dng tha mn iu kin xy + yz + zx =
3.
Tm gi tr nh nht ca biu thc
P = 10

x y

+ 10

yz

zx

+ 10

x2 y 2 + y 2 z 2 + z 2 x2 x2 + y 2 + z 2
.

3
3( x + y + z )

Li gii

1 2
x + y2 + z2
3

Ta c: x 2 y 2 + y 2 z 2 + z 2 x 2
4

x2 y 2 + y 2 z 2 + z 2 x2
4
x2 + y 2 + z 2
3
3

x + y + z 3 ( xy + yz + zx ) = 3

x2 + y 2 + z 2
x2 + y 2 + z 2
.

3( x + y + z )
9

Suy ra:
P 10

x y

+ 10

yz

+ 10

zx

= 10

x y

+ 10

yz

+ 10

zx

= 10

x y

+ 10

yz

+ 10

zx

Xt hm s f (=
t ) 10t
'(t ) 10t ln10
f=

ng bin trn

[0;+ ) do

13 2
x + y2 + z2
9
13
2
2
2
( x y ) + ( y z ) + ( z x ) + 6 .

18
13
13
2
2
2
( x y ) + ( y z ) + ( z x )
3
18

13 2
t vi t 0 ta c
18

13
13
13
''(t ) 10t ln 2 10 > ln 2 10 > 0 nn f(t) l hm
t; f =
9
9
9

[0; + )

f '(t ) f '(0) = 0 nn f(t) l hm ng bin trn

f (t ) f(0) =
1.

595

www.TaiLieuLuyenThi.com
Khm ph t duy K thut gii bt T Bi ton Max Min ng Thnh Nam

Hay 10t

13 2
t 1, t 0 .
18

p dng vi t =
x y , y z , z x ta c:
10

x y

+ 10

yz

+ 10

zx

13
( x y )2 + ( y z )2 + ( z x )2 3 .
18

4
Do gi tr nh nht ca P bng t ti x= y= z= 1 .
3
V d 3. Cho x,y,z l cc s thc tha mn iu kin x + y + z =
0.

Tm gi tr nh nht ca biu thc

P = 2 x y + 2 y z + 2 z x ln 14 x 2 + y 2 + z 2 + 1 .

Li gii
Nhn xt. D on du bng t ti x = y = z x = y = z = 0 P = 0 .
Vy ta chng minh P t gi tr nh nht bng 0.
Thay z = x y vo biu thc ca P ta c:

2
P= 2 x y + x + 3 y + 2 y + 3 x ln 14 x 2 + y 2 + ( x y ) + 1

.
2 x y + x + 3 y + 2 y + 3 x ln 28 x 2 + y 2 + xy + 1

= 6 x + 4 y ln 28 x 2 + y 2 + xy + 1

Ta cn chng minh: 6 x + 4 y ln 28 x 2 + y 2 + xy + 1 0 .

6 x + 4 y ln 28 x 2 + y 2 + xy + 1 .

S dng bt ng thc quen thuc: ln ( x + 1) x, x 0 ta c:

ln 28 x 2 + y 2 + xy + 1 28 x 2 + y 2 + xy .

Vy nhn nh c chng minh nu ta chng minh c bt ng thc sau ng:

6 x + 4 y 28 x 2 + y 2 + xy 36 x 2 + 48 xy + 16 y 2 28 x 2 + 28 y 2 + 28 xy .

8 x 2 + 20 xy 12 y 2 0 ( x + 3 y )( 8 x + 4 y ) 0 .

596

www.TaiLieuLuyenThi.com
Cty TNHH MTV DVVH Khang Vit

Bt ng thc cui lun ng nu ta c xy 0 . Tht vy vi ba s x,y,z lun c


hai s cng du v bi ton kt thc nu ta gi s t u x, y cng du.
Vy gi tr nh nht ca P bng 0 t ti x= y= z= 0 .
V d 4. Cho a,b l hai s thc dng tho mn iu kin a + b = a 4 + b 4 .
3

Chng minh rng a a bb 1 a a bb .


Li gii
Bt ng thc cho tng ng vi: a ln a + b ln b 0 a3 ln a + b3 ln b .
chng minh bt ng thc v tri ta s dng bt ng thc
ln x x 1, x > 0 ta c a ln a + b ln b a ( a 1) + b ( b 1) = a 2 + b 2 a b .

Vy ta ch cn chng minh a 2 + b 2 a + b .
Tht vy s dng bt ng thc Holder ta c

(a

+ b4 ( a + b ) a 2 + b2
2

a 2 + b2 a + b .

Ta c iu phi chng minh. ng thc xy ra khi v ch khi a= b= 1 .


Chng minh bt ng thc v phi
Theo gi thit kt hp s dng bt ng thc AM GM ta c
a + b = a 4 + b4

1
( a + b )4 a + b 2 a, b ( 0;2 ) .
8

Khi xt hm s f=
( x) 3ln x +

x x4

x3

trn khong (0;2) ta c

2
3
2 ( x 1) x + 2 x + 2
f '( x) = 1 3 =
; f '( x) = 0 x = 1 ( 0;2 ) .
x
x
x3
Khi f(x) i du t m sang dng khi i qua x = 1 nn f(x) t cc tiu ti
x =1.

V vy f ( x) f (1) =
0 x3 ln x

x4 x
.
3

a 4 a b4 b
+
=
0.
3
3
Bt ng thc c chng minh. ng thc xy ra khi v ch khi a= b= 1 .

Do a3 ln a + b3 ln b

597

www.TaiLieuLuyenThi.com
Khm ph t duy K thut gii bt T Bi ton Max Min ng Thnh Nam

BI TP RN LUYN
Bi 1. Cho a,b,c l cc s thc thay i tha mn a, b 0, c > 0 v a + b + c =
1.
Tm gi tr ln nht ca biu thc

1
1
=
P
ln 1 + a 2 +
ln 1 + b 2 +
2013
2014

8 1 c2 + 1
3c

).

HD:
Do a, b 0, c > 0, a + b + c =1 a, b [ 0;1] , c ( 0;1] .
S dng kt qu bi ton ph ta c
1
1
1 2
1 2
ln 1 + a 2 +
ln 1 + b 2
a +
b
2013
2014
2013
2014
.
1
1
4
4
4

a+
b a + b = (1 c )
2013
2014
3
3
3

4
Do : P (1 c ) +
3

8 1 c2 + 3
3c

4
Xt hm s g (c) =
(1 c ) +
3

).

8 1 c2 + 3
3c

c2 + 3

4
8 8
g '(c) =
2 . c +3 2
3 3c
3
c
2

4
8
8
.
=
2+
2
3 3c
c c2 + 3

24 4c 2 c 2 + 3 8 c 2 + 3

g '(c)

) lin tc trn ( 0;1] , ta c:

3c 2 c 2 + 3

0, c ( 0;1] .

8
Nn g (c) l hm ng bin trn ( 0;1] . Suy ra g (c) g (1) =
.
3
8
Vy gi tr ln nht ca P bng khi a= b= 0, c= 1 .
3
Bi 2. Cho x,y,z l cc s thc. Tm gi tr nh nht ca biu thc
P= 3

x y

+3

yz

+3

zx

2
2
2
x y) + ( y z ) + ( z x)
(

HD: S dng bt ng thc a x 1 + x +


3

x y

598

+3

yz

+3

zx

x 2 + y 2 + z 2 xy yz zx .

x2
, x, a e ta c
2

2
2
2
x y) + ( y z ) + ( z x)
(

3+

x y + yz + zx .

www.TaiLieuLuyenThi.com
Cty TNHH MTV DVVH Khang Vit

Do P 3 + x y + y z + z x 2 x 2 + y 2 + z 2 xy yz zx .
Ta chng minh x y + y z + z x 2 x 2 + y 2 + z 2 xy yz zx 0 .
Khng mt tnh tng qut gi s x y z bt ng thc tr thnh
1
1
1
( x y )2 + ( y z )2 + ( z x )2
2
2
2

( x y) + ( y z) + ( x z) 2

(
2 (( x y )

)
+ ( z x) )

2( x z ) 2 ( x y) + ( y z ) + ( z x)
4( x z )

+ ( y z)

( x z) ( x y) + ( y z)
2

( ( x y ) + ( y z ) ) ( x y ) + ( y z ) 2 ( x y )( y z ) 0
2

Bt ng thc cui lun ng ta c pcm. Suy ra P 3 .


Vy gi tr nh nht ca P bng 3 t ti x= y= z .
Bi 3. Cho x,y,z l cc s thc khng m tha mn iu kin

( x + y )2 + ( y + z )2 + ( z + x )2 18 .
Tm gi tr ln nht ca biu thc P =

x
3
4

y
3
+4

z
3
+4

1
( x + y + z )4 .
108

Bi 3. HD : Ta chng minh 4 3 x + 1, x [ 0;3] .


x

Xt hm s f ( x) = 4 3 x 1 vi x [ 0;3] ta c
x

1
3
.
f '( x) = ln 4.4 3 1; f '( x) = 0 x = x0 = 3log 4
3
ln 3

Ta c f(x) i du t m sang dng khi i qua x0 nn f(x) t cc tiu ti x0 .


Do f ( x) max { f (0); f (3)}

x
3
=
0 hay 4 x + 1 vi

mi x [ 0;3] .

ng thc xy ra khi v ch khi x = 0 hoc x = 3 .


Suy ra P ( x + y + z ) + 3

( x + y + z )4
108

599

www.TaiLieuLuyenThi.com
Khm ph t duy K thut gii bt T Bi ton Max Min ng Thnh Nam

Bi ton 7. Cho x,y l hai s thc khng m tha mn x + y

2
ta lun c
3

1 ( x + y)
1 x
1 y
.
+
1+
1+ x
1+ y
1+ ( x + y)

ng thc xy ra khi v ch khi x = 0 hoc y = 0 .


Chng minh.
Bt ng thc cho tng ng vi:
1 x 1 y
+
+2
1+ x 1+ y

(1 x )(1 y ) 1 1 x y 2
+
+
1+ x + y
(1 + x )(1 + y )

(1 x )(1 y )

(1 + x )(1 + y )

1 x y
1+ x + y

1 x y
1 x y 1 x 1 y
1+

1+ x + y
1+ x + y 1+ x 1+ y

y) 1 x y
2 xy ( x + y )
(1 x )(1 =

0.
(1 + x )(1 + y ) 1 + x + y (1 + x )(1 + y )(1 + x + y )
2 xy ( x + y + 2 )
1 x y 1 x 1 y
.
V 1 +

=
1 + x + y 1 + x 1 + y (1 + x )(1 + y )(1 + x + y )
2 xy ( x + y )
(1 x )(1 y ) 1 x y
(1 + x )(1 + y )(1 + x + y )
Suy ra

=
(1 + x )(1 + y ) 1 + x + y
(1 x )(1 y ) + 1 x y
(1 + x )(1 + y ) 1 + x + y
2 xy ( x + y )
(1 + x )(1 + y )(1 + x + y )

Ch

1 x y
1+ x + y

xy ( x + y )

(1 + x )(1 + y )(1 + x + y )
Vy ta ch cn chng minh
xy ( x + y )

(1 + x )(1 + y )(1 + x + y )

600

1 x y
1+ x + y

1 x y
1+ x + y

xy ( x + y + 2 )

(1 + x )(1 + y )(1 + x + y )

1 x y
x+ y

1+ x + y x + y + 2

www.TaiLieuLuyenThi.com
Cty TNHH MTV DVVH Khang Vit

Bt ng thc cui lun ng do


1 x y
x+ y
1 2 / 3
2/3

=
1+ x + y x + y + 2
1+ 2 / 3 2 / 3 + 2

1
5

1
>0.
4

CC V D
V d 1. Cho x,y,z l cc s thc khng m tha mn iu kin x + y + z =
1.
Tm gi tr ln nht v nh nht ca biu thc
P=

1 x
1 y
1 z
.
+
+
1+ x
1+ y
1+ z

Li gii
Tm gi tr nh nht

( (

Ch vi x, y, z [ 0;1] ta c (1 x ) 1 1 x 2

)) 0

1 x
1 x .
1+ x

Tng t ta c

1 y
1 z
1 y;
1 z .
1+ y
1+ z

Cng theo v ba bt ng thc trn ta c:


1 x
1 y
1 z
+
+
3 x y=
z 2.
1+ x
1+ y
1+ z

=
P

ng thc xy ra khi v ch khi x= 1, y= z= 0 hoc cc hon v.


Cch 2: Nu ta c
P=

y+z
z+x
x+ y
+
+
( x + y) + ( x + z)
( y + z) + ( y + z)
( z + x) + ( z + y )
2( y + z)

2( x + y + z)

2( z + x)

2( x + y + z )

2( x + y)

2( x + y + z )

2
=

Tm gi tr ln nht
1
2
Khng mt tnh tng qut=
gi s z max { x, y, z} z ; x + y .
3
3
Khi s dng bt ng thc ph ta c
1 x
1 y
1 x y
+
1+
=1+
1+ x
1+ y
1+ x + y

z
.
z+2

601

www.TaiLieuLuyenThi.com
Khm ph t duy K thut gii bt T Bi ton Max Min ng Thnh Nam

z
1 z
.
+
z+2
1+ z

Suy ra P 1 +

z
1 z
Xt hm s f ( z ) =
trn on
1+
+
z+2
1+ z

=
f '( z )

(2 z)

z
2 z

(1 + z )

1 z
1+ z

1
3 ;1 ta c

1 1
3
3
f '( z ) = 0 (1 + z ) (1 z ) = ( 2 z ) z z = ;1
2 3
1 1

1
So snh cc gi tr suy ra max f (=
z ) max f ; f ; f (1)=
f=

2
3 2

1
hoc cc hon v.
2

ng thc xy ra khi v ch khi x= 0, y= z=


Vy gi tr ln nht ca P bng 1 +

2
3

Bi tp tng t
Cho x,y,z l cc s thc khng m tho mn iu kin
=
z max { x, y, z=
}; x + y + z 1 .

Tm gi tr ln nht ca biu thc P =

1 x
1 y
2 z
.
+
+
1+ x
1+ y
z

BI TP RN LUYN
Bi 1. Cho a,b,c l cc s thc khng m tho mn iu kin
1

2.
=
1+ a
1 + b 1 + c2
Tm gi tr nh nht v ln nht ca biu thc P = a + b + c .
2

Bi ton 8. Cho x,y l hai s thc dng tha mn x + y < 1 ta lun c


2

1 1 2
1 .
1 1
x y x + y

ng thc xy ra khi v ch khi x = y .


Chng minh.
Bt ng thc cho tng ng vi:
602

1 1 1
4
4

2
xy x y ( x + y )
x+ y

2
+1 .
3

www.TaiLieuLuyenThi.com
Cty TNHH MTV DVVH Khang Vit

1
4
1 1
4

+
2
xy ( x + y )
x y x+ y

( x y )2
( x y )2

2
xy ( x + y )
xy ( x + y )
2
( x y ) (1 x y ) 0

ng thc xy ra khi v ch khi x = y .


CC V D
V d 1. Cho a,b,c l cc s thc dng tha mn iu kin a + b + c =
1.
Tm gi tr nh nht ca biu thc P=

18
1 1
.
1 1 +
a b a + b + 2c

Li gii
1 1 (1 a )(1 b ) 1 ( a + b ) + ab c
Ta c 1 1 =
=
=
+1.
ab
ab
ab
a b
2

a + b 1 c
Ta c ab
=

. ng thc xy ra khi v ch khi a = b .


2 2

Suy ra

c
1 1
=
+1
1 1
2
a b
1 c

Do P

c +1
c +1
.
=

1 c
1 c

c +1
18
c + 1 18
.
+
=
+
1 c a + b + 2c 1 c 1 + c

4 ( 2c 1)
c + 1 18 c + 1 18

+ =
+
15 + 15
=
+ 15 15 .
1 c 1+ c 1 c 1+ c
1 c2

Mt khc

ng thc xy ra khi v ch khi c=

1
1 c 1
.
=
, a= b=
2
2
4

Vy gi tr nh nht ca P bng 15 t ti a= b=

1
1
.
, c=
4
2

Cch 2: S dng bt ng thc ph ta c


2
18
2
18
1+
=
+
1.
a+b
a + b + 2c 1 c 1 + c
Thc hin nh gi nh li gii trn ta c kt qu bi ton.
Nhn xt. Li gii 2 cho ta nh gi trong trng hp tng ba s khc 1.
P

603

www.TaiLieuLuyenThi.com
Khm ph t duy K thut gii bt T Bi ton Max Min ng Thnh Nam

Bi ton 9. Chng minh rng vi mi s thc khng m a,b tho mn iu kin ta


1
1
1
.
c
+
1+
2
2
2
1 + kx
1 + ky
1+ k ( x + y)
Chng minh.
Bt ng thc cho tng ng vi:
1 k 2 x2 y 2

(1 + kx )(1 + ky )
2

1
1+ k ( x + y)

2kxy k 3 x 2 y 2 ( x + y ) 2k 2 x 2 y 2
2

)(

)(

1 + kx 2 1 + ky 2 1 + k ( x + y )

kxy 2 kxy k ( x + y ) + 2

)(

)(

))

1 + kx 2 1 + ky 2 1 + k ( x + y )

.
2

CC V D
V d 1. Chng minh rng vi mi s thc khng m a,b tho mn iu kin
2
1
1
1
ta c
.
a+b
+
1+
2
2
2
7
1 + 3a
1 + 3b
1 + 3( a + b )
Li gii

Bt ng thc cho tng ng vi:

Ta ch cn chng minh 1 3ab

2 3ab 2 + 3 ( a + b )

).

9ab ( a + b )

ab 1 3ab

(1 + 3a )(1 + 3b ) (1 + 3( a + b ) )

9ab ( a + b )
2

0.

Bt ng thc ng bi v

3ab 2 + 3 ( a + b )

1
2
a+b
3
2 + 3( a + b ) 3

2
7

2
2
2 + 3
<2.

Bt ng thc c chng minh. ng thc xy ra khi v ch khi a = 0 hoc b = 0 .

604

www.TaiLieuLuyenThi.com
Cty TNHH MTV DVVH Khang Vit

Bi tp tng t
Cho a,b l cc s thc khng m tho mn iu kin a + b 2 .
Chng minh rng

1
1+ a

1
1+ b

1+

1
1 + (a + b)

V d 2. Cho a,b,c l cc s thc khng m tho mn iu kin a + b + c =


1.
Tm gi tr nh nht ca biu thc
P=

1
1 + 3a

1 + 3b
Li gii

1
1 + 3c 2

Khng mt tnh tng qut gi s a = max {a, b, c}


1
2
a ;b + c .
3
3

Khi ta c
V vy P

1
1 + 3b
1

1 + 3a

1
1 + 3c

+1+

1+

1
1 + 3(b + c )

1
1 + 3(b + c )

1
1
= 2+
+1
2
1 + 3a
1 + 3 (1 a )
3 ( 2a 1) (1 + 6a (1 a ) ) 15 15
+
7
7
7 3a 2 + 1 3a 2 6a + 4
2

)(

Du bng xy ra khi v ch khi a= b=

1
, c= 0 hoc cc hon v.
2

Bi tp tng t
Cho a,b,c,d l cc s thc khng m tho mn iu kin a + b + c + d =
2.
Chng minh rng
1
1 + 3a

1
1 + 3b

1
1 + 3c

1
1 + 3d

16
.
7

605

www.TaiLieuLuyenThi.com
Khm ph t duy K thut gii bt T Bi ton Max Min ng Thnh Nam

MT S BT NG THC DNG LNG GIC HO GII BNG NH


GI I S
Di y cp mt s bt ng thc dng lng gic ho c th nh gi bng
cc bt ng thc c bn hoc s dng tnh n iu ca hm s (xem chng 4).
Bi 1. Cho a,b,c l cc s thc dng tha mn ab + bc + ca =
1 ta lun c
a
1+ a

b
1+ b

1 + c2

Li gii
V ab + bc + ca =
1 nn 1 + a 2 = ab + bc + ca + a 2 = ( a + b )( a + c )

1 + b 2 = ( b + c )( b + a ) ;1 + c 2 = ( c + a )( c + b )

(1 + a )(1 + b )(1 + c ) .

Suy ra ( a + b )( b + c )( c + a ) =

Khi s dng bt ng thc C S ta c


a

b
+=
1 + a 1 + b2

( a + b )( a + c )

=
( b + c )( b + a )

(1 + a )(1 + b )(1 + c
2

( a + b )( b + c )( c + a )

(1 + a )(1 + b ) =

) (1 + a )(1 + b )(1 + c )
2

1 + ab

a (b + c ) + b (c + a )

a = b
ng thc xy ra khi v ch khi
.
1
ab + bc + ca =

Bi 2. Cho a,b,c l cc s thc dng tha mn iu kin ab + bc + ca =


1.
Tm gi tr ln nht ca biu thc P =

a
1+ a

b
1+ b

3c
1 + c2

Li gii
S dng

a
1+ a

Suy ra P

b
1+ b

3c + 1
1 + c2

( c + 1)(3 + 1) =
10 .
2

1 + c2
2

c2 + 1

c = 3

a =b =3 + 10
ng thc xy ra khi v =
ch khi a b
.

c = 3
ab + bc + ca =
1

606

1
1 + c2

www.TaiLieuLuyenThi.com
Cty TNHH MTV DVVH Khang Vit

Vy gi tr ln nht ca P bng 10 .
Bi tp tng t
Cho a,b,c l cc s thc dng tha mn iu kin ab + bc + ca =
1.
Tm gi tr ln nht ca biu thc P =

a
1+ a

b
1+ b

3c

1 + c2

Bi 3. Cho cc s thc khng m x, y, z tha mn iu kin xz + yz + 1 =xy.


2x

Tm gi tr ln nht ca=
biu thc P

x2 + 1

2y
y2 + 1

z2 1
z2 + 1

Li gii
Theo gi thit ta c xz + yz = xy 1 0 xy 1 .
Khi chia hai v cho xy ta c:
t=
a

z z 1
+ +
=
1.
y x xy

2a
2b
c2 1
1
1
.
1 v P =
+
+
=
,b =
, c z ta c ab + bc + ca =
x
y
1 + a 2 1 + b2 c2 + 1

Ta c 1 + a 2 = ab + bc + ca + a 2 = ( a + b )( a + c )

1 + b 2 = ( b + c )( b + a ) ;1 + c 2 = ( c + a )( c + b )

Suy ra ( a + b )( b + c )( c + a ) =
Ta c
=

(1 + a )(1 + b )(1 + c ) .
2

b
a
b
=
+
+
2
2
1+ a
1+ b
ab + bc + ca + a
ab + bc + ca + b 2
2

a (b + c ) + b (c + a )

( a + b )( a + c ) ( b + c )( b + a ) ( a + b )( b + c )( c + a )

1 + ab
=

( a + b )( b + c )( a + c )

Vy P f (c=
)
Xt hm s=
f (c )

2
1 + c2

2
1 + c2

(1 + a )(1 + b ) =1
(1 + a )(1 + b )(1 + c ) 1 + c
2

c2 1
c2 + 1
+

c2 1
c2 + 1

vi c 0 ta c

607

www.TaiLieuLuyenThi.com
Khm ph t duy K thut gii bt T Bi ton Max Min ng Thnh Nam

2c c 2 + 1 2
c = 0
.
f '(c) =
;
f
'(
c
)
0

2 2
3
c
=

1+ c

3
.
2

Lp bng bin thin suy ra max=


f (c) f=
( 3)
c 0

ng thc xy ra khi v ch khi c = 3, a = b = 2 3 x = y = 2 + 3, z = 3 .


3
t ti x =
y=
2 + 3, z =
3.
2
Bi ton 10. Vi mi s thc dng a,b,c ta lun c

Vy gi tr ln nht ca P bng

(a + b)

(a + c)

1
a + bc
2

ng thc xy ra khi v ch khi a= b= c .


Chng minh.
S dng bt ng thc C-S cho hai s dng ta c:

) 1 + bc ( a + b )

) 1 + bc ( a + c )

2
a + bc

a 2 + bc

(a + b)

( a + c )2

c
=
2
b
a + bc ( b + c )
a 2 + bc 1 +
c
.
b
1

=
c
a 2 + bc ( b + c )
a 2 + bc 1 +
b

Cng theo v hai bt ng thc trn ta c iu phi chng minh.


ng thc xy ra khi v ch khi a= b= c .
c bit. Khi a = 1 ta c bt ng thc hay dng sau y:
1

+ Vi mi s thc dng a, b ta lun c:


+ Vi mi s thc dng a,b ta lun c:

( a + 1)

( b + 1)

(1 + a ) (1 + b )
2

1
.
1 + ab

Chng minh.
Ta c:

608

(1 + a ) (1 + b )
2

1
1 + ab

1
2
=.
a+b a+b+2
1+
2

2
.
a+b+2

www.TaiLieuLuyenThi.com
Cty TNHH MTV DVVH Khang Vit

CC V D
V d 1. Cho a,b,c l cc s thc dng thay i tha mn abc = 1 .
Chng minh rng

(1 + a )

(1 + b )

(1 + c )

3
.
4

Li gii
Bi ton ph. Vi mi s thc dng a, b ta lun c:
1
1
1
.
+

2
2
( a + 1) ( b + 1) 1 + ab
Chng minh. Ta c:

1
1
b
2
a

=
(1 + ab ) 1 + b (1 + a )
2

( a + 1) (1 + ab ) 1 + a (1 + ab )( a + b )

a
.

a
1
1
(1 + ab ) 1 + b (1 + b )2

b (1 + ab )( a + b )
a
1 + b)
(
+
+
ab
1
1
(
)

Cng li theo v hai bt ng thc trn ta c iu phi chng minh.


1
1
1
1
1
+
Vy ta c: P=
+

+
2
2
2

(1 + b ) (1 + c ) 1 + ab (1 + c )2
(1 + a )

c2 + c + 1
1
1
.
= +
=
2
1 (1 + c )2
c
+
1
(
)
1+
c
Ta chng minh
c2 + c + 1

( c + 1)

3
2
4c 2 + 4c + 4 3 c 2 + 2c + 1 ( c 1) 0 (lun ng).
4

3
t ti a= b= c= 1 .
4
V d 2. Cho x,y,z l cc s thc dng tha mn xyz = 1 .

Vy gi tr nh nht ca P bng

Tm gi tr nh nht ca biu thc P =

(1 + x )

(1 + y )

4
3 (1 + z )

Li gii
S dng bt ng thc:

(1 + x )

(1 + y )

1
1
z
.
=
=
1
1 + xy 1 +
1+ z
z
609

www.TaiLieuLuyenThi.com
Khm ph t duy K thut gii bt T Bi ton Max Min ng Thnh Nam

Khi P

z
4
3z 3 + 6 z 2 + 3z + 4
.
+
=
3
1 + z 3 (1 + z )3
3 ( z + 1)

2
D on ng thc xy ra khi x =y =z =1 P = .
3
Ta chng minh
3z 3 + 6 z 2 + 3z + 4
3 ( z + 1)

2
2
z 3 3 z + 2 0 ( z + 2 )( z 1) 0 (lun ng).
3

Vy gi tr nh nht ca P bng

2
t ti x= y= z= 1 .
3

V d 3. Cho x,y,z l cc s thc dng thay i tha mn xyz = 1 .


Tm gi tr nh nht ca biu thc P =

(1 + x )

(1 + y )

(1 + z )3

Li gii
S dng bt ng thc AM-GM ta c:
1

(1 + x )

(1 + y )

(1 + z )

+
+
+

(1 + x )

1
3
+
8 2 (1 + x )2

1
3
.
+
8 2 (1 + y )2

1
3
+
8 2 (1 + z )2

(1 + y )
1

(1 + z )

Cng theo v ba t ng thc trn ta c:


3 3 1
1
1 3 1
1 3 z2 + z + 1 9

2P +
+
+

+
=
.

8 2 (1 + x )2 (1 + y )2 (1 + z )2 2 1 + xy (1 + z )2 2 ( z + 1)2 8

3
Vy gi tr nh nht ca P bng t ti x= y= z= 1 .
8

b
c
a
ta c bi ton trong thi K thi chn HSG
=
,y =
,z
a
b
c
quc gia nm 2006 nh sau:
Bi ton. (VMO 2006) Cho a,b,c l cc s thc dng.

Nhn xt. Thay=


x

a b c 3
Chng minh rng
+
+
.
a+b b+c c+a 8
610

www.TaiLieuLuyenThi.com
Cty TNHH MTV DVVH Khang Vit

V d 4. Cho x, y, z l cc s thc dng thay i tha mn x y z v xyz = 1 .


Tm gi tr nh nht ca biu thc P =

( x + 1)

( y + 1)

( z + 1)2

Li gii
1
1
1

+
+
P
2
2
2
2
2
2

+
+
+
+
+
x
1
z
1
x
1
y
1
z
1
( ) ( )
) ( ) ( )
(
S dng bt ng thc ph ta c
1

Do x z

z2 + z + 1
2
2
2
2
.
+
=
+
=
2.
1 + xy (1 + z )2 1 + 1 (1 + z )2
(1 + z )2
z

Ta chng minh 2.

z2 + z + 1

(1 + z )

Vy GTNN ca P bng

3
2
( z 1) 0 (lun ng).
2

3
khi x= y= z= 1 .
2

V d 5. Cho cc s thc dng a,b,c,d thay i tha mn a 2 + b 2 + c 2 + d 2 =


1.
Chng minh rng (1 a )(1 b )(1 c )(1 d ) abcd .
Li gii
Ta phi chng minh
t
=
x

1 a
1 b
1 c
1 d
ta c 0 < x, y, z , t < 1 .
=
,y =
,z =
,t
a
b
c
d

Theo gi thit ta c:

Ta c:

Suy ra:

1 a 1 b 1 c 1 d
1.
.
.
.
a
b
c
d

(1 + x )

(1 + x )

(1 + y )

(1 + y )

(1 + z )

(1 + z )

(1 + t )

(1 + t )2

=
1.

1
1
2 + xy + zt
.
+
=
1 + xy 1 + zt 1 + xy + zt + xyzt

2 + xy + zt
1 xyzt 1 ta c iu phi chng minh.
1 + xy + zt + xyzt

ng thc xy ra khi v ch khi a= b= c= d=

1
.
2

611

www.TaiLieuLuyenThi.com
Khm ph t duy K thut gii bt T Bi ton Max Min ng Thnh Nam

BI TP RN LUYN
Bi 1. Cho a,b,c,d l cc s thc dng . Chng minh
2

a b c d

+
+
+
1.
a+b b+c c+d d +a
HD: Vit li bt ng thc di dng:
1
b
1 +
a

1
c
1 +
b

1
d
1 +
c

1
a

1 +
d

1.

b
c
d
a
ta c xyzt = 1 v a v bt ng thc quen thuc
=
,y =
,z =
,t
a
b
c
d

t=
x
1

(1 + x )

(1 + y )

(1 + z )

(1 + t )2

1 vi x,y,z,t l cc s thc dng tha

mn xyzt = 1 .
Bi 2. Cho a,b,c l cc s thc dng. Tm gi tr nh nht ca biu thc
2

a b c
P=
+
+
.
a+b b+c c+a
HD: Vit li P di dng:
P=

1
b
1 +
a

vi=
x

1
c
1 +
b

1
a
1 +
c

(1 + x )

(1 + y )

(1 + z )2

b
c
a
nn ta c xyz = 1 .
=
,y =
,z
a
b
c

S dng bt ng thc ph ta c:
Suy ra: P

(1 + x )

(1 + y )

1
.
1 + xy

z2 + z + 1 3
1
1
1
1
+
=
+
=
.
1 + xy (1 + z )2 1 + 1 (1 + z )2
( z + 1)2 4
z

3
t ti a= b= c .
4
Nhn xt. T kt qu trn ta chng minh c vi mi a,b,c dng ta lun c

Vy gi tr nh nht ca P bng

a b c 3

+
+
.
a+b b+c c+a 8
612

www.TaiLieuLuyenThi.com
Cty TNHH MTV DVVH Khang Vit

a
b
c
theo chng minh trn ta c:
=
,B =
,C
a+b
b+c
c+a

Tht vy,=
t A

3
.
4
S dng bt ng thc AM-GM ta c:
A2 + B 2 + C 2

1 3
1 3
1 3
A3 + A3 + A2 , B3 + B3 + B 2 , C 3 + C 3 + C 2 .
8 2
8 2
8 2
Cng li theo v ba t ng thc trn ta c:

3 3
9
3
2 A3 + B3 + C 3 + A2 + B 2 + C 2 A3 + B3 + C 3 .
8 2
8
8
Bi ton c chng minh. ng thc xy ra khi a= b= c .

Bi 3. Cho a,b,c l cc s thc dng thay i tha mn a b c v abc = 1 .


Tm gi tr nh nht ca biu thc P =

(1 + a )

(1 + b )

(1 + c )2

Bi 4. Cho a,b,c l cc s thc dng tha mn a b c .


Tm gi tr nh nht ca biu thc
1

P=

( a + 1)

( b + 1)

( c + 1)

ab + bc + ca
.
4

Bi 5. Cho x, y, z l cc s thc dng thay i tha mn x y z v xyz = 1 .


Tm gi tr nh nht ca biu thc P =
HD: Do x z

( x + 1)

( x + 1)

( y + 1)

( z + 1)2

( z + 1)2

1
1
1
.
P 2
+
+
( x + 1)2 ( y + 1)2 ( z + 1)2

Vy ta c:
1
1
1
2
2
2
2
z2 + z + 1

+
.
2.
P 2
+
+
=
+
=
2
(1 + x )2 (1 + y )2 (1 + z )2 1 + xy (1 + z )2 1 + 1 (1 + z )2
1
z
+
(
)

Ta chng minh 2.

z2 + z + 1

(1 + z )

3
2
( z 1) 0 (lun ng).
2
613

www.TaiLieuLuyenThi.com
Khm ph t duy K thut gii bt T Bi ton Max Min ng Thnh Nam

3
khi x= y= z= 1 .
2
Bi 6. Cho a,b,c,d l cc s thc dng thay i tha mn abcd = 1 .
Tm gi tr nh nht ca biu thc

Vy GTNN ca P bng

P=

(1 + a )

(1 + b )

(1 + c )

(1 + d )2

HD: S dng bt ng thc ph ta c:


1

(1 + a )

(1 + b )

(1 + c )

(1 + d )

1
1
ab + cd + 2
=
+
= 1.
1 + ab 1 + cd 1 + ab + cd + abcd

Vy gi tr nh nht ca P bng 1 t ti a= b= c= d= 1 .
1

Bi ton 11. Vi mi s thc dng a,b ta c

1
b

( a + b )2

Du bng xy ra khi v ch khi a = b .


Chng minh.
S dng bt ng thc AM-GM cho hai s dng ta c:
1
a

1
b

2
2
8

= 2.
2
ab a + b
(a + b)

ng thc xy ra khi v ch khi a = b .


y ch l mt bt ng thc rt ra t bt ng thc AM GM cho hai s
dng ta cng xt mt s bi ton p dng.
CC V D
V d 1. Cho x, y, z l cc s thc khng m tha mn iu kin x 2 + y 2 + z 2 3 y .
Tm gi tr nh nht ca biu thc P =

( x + 1)

( y + 2)

( z + 3)2

Li gii
Ta c:
1

( x + 1)

614

( y + 2)

( x + 1)

1
y
+ 1
2

2
8
.

2
y
y

( x + 1) + 1 x + 1 + + 1
2
2

www.TaiLieuLuyenThi.com
Cty TNHH MTV DVVH Khang Vit

8
y

x + 1 + + 1
2

( z + 3)

16
64
.

2
y

1
1
3
x
z
+
+
+
+
) x + 1 + + 1 + z + 3

(
2

Mt khc: 2 x + 4 y + 2 z x 2 + 1 + y 2 + 4 + z 2 + 1 = x 2 + y 2 + z 2 + 6 3 y + 6
2x + y + 2z 6 x +

y
64
+ z 3 P
=
1 .
2
( 3 + 2 + 3)2

Vy GTNN ca P bng 1 khi x= z= 1, y= 2 .


V d 2. Cho a,b,c l cc s thc khng m v khng ng thi bng 0 thay i
tha mn iu kin a 2 + b 2 + c 2 6b .
1

Tm gi tr nh nht ca biu thc P=

(a + b + c)

( b + 11)

( c + 6 )2

Li gii
S dng bt ng thc ph
1

(a + b + c)

( c + 6)

1
x

1
y

( x + y )2

( a + b + 2c + 6 ) 2

ta c:

1
1
64

.
Suy ra P 8
+
2
2
( a + b + 2c + 6 )
( a + 2b + 2c + 17 )2
+
b
11
(
)

Mt khc theo bt ng thc AM-GM ta c:

) (

) (

2a + 10b + 4c a 2 + 1 + b 2 + 25 + c 2 + 4 6b + 30 a + 2b + 2c 15 .

Do P

64

(15 + 17 )

1
.
=
16

Vy gi tr nh nht ca P bng

1
t ti =
a 1,=
b 5,=
c 4.
16

BI TP RN LUYN
Bi 1. Cho a,b,c l cc s thc i mt phn bit.
1
1
1
.
Tm gi tr nh nht ca biu thc P = a 2 + b 2 + c 2
+
+
( a b )2 ( b c )2 ( c a )2

615

www.TaiLieuLuyenThi.com
Khm ph t duy K thut gii bt T Bi ton Max Min ng Thnh Nam

HD: Khng mt tnh tng qut gi s a > b > c khi ta c:


1

( a b )( b c ) ( a c )2
.
2
a

c
(
)
a 2 + b2 + c2 a 2 + c2
(a b)

(b c )

Suy ra

2
a c)
(

( a c )2

9
=
.
2
a
c

(
) 2
1

ng thc xy ra khi v ch khi b = 0, a = c .


Vy gi tr nh nht ca P bng

9
t ti a =
c, b =
0 hoc cc hon v.
2

Bi 2. Cho a,b,c l cc s thc i mt phn bit thuc on [ 0;2] .


Tm gi tr nh nht ca biu thc
P=

(a b)

(b c )

( c a )2

HD: Khng mt tnh tng qut gi s a > b > c ta c:


1

(a b)

Do P

(b c )

(a c)

2
8
.

b
b

c
(
)(
) ( a c )2

9
, a, c [ 0;2] .
4

Vy gi tr nh nht ca P bng

616

9
t ti =
a 2,=
b 1,=
c 0 hoc cc hon v.
4

www.TaiLieuLuyenThi.com
Cty TNHH MTV DVVH Khang Vit

CH 9: BI TON CHN LC

BT NG THC V CC TR BA BIN
Trong ch ny ti tng hp li mt s bi ton bt ng thc v cc tr ba
bin hay trong cc thi th, thi chn Hc sinh gii cp tnh v thnh ph cc
nm cng mt s bi ton xut trn Din n hc tp trc tuyn Mathlinks.vn.
V sau mt s bi ton c b sung thm mt s bi ton tng t dnh cho bn c
rn luyn.
Bi 1. Cho x,y,z l cc s thc dng tha mn iu kin x(1 yz) = y + z.

(z + z

Tm gi tr ln nht ca
biu thc P
=

xy

( x + y ) ( z 2 + 1)

1 + z2

2z

1 + z2

Li gii
Theo gi thit ta c z (1 + xy ) = x y x > y .
S dng bt ng thc C S ta c

(z + z

xy

(z

Khi P

= z 2 1 + 2 xy + xy = z 2 (1 + xy ) + 2 z 2 xy = z ( x y ) + 2 z 2 xy
2

2
+ z 4 ( x y ) + 4 xy =
( x + y ) z z2 + 1

( x + y ) z z2 + 1 +
2z
2z
z
.
= +
( x + y ) ( z 2 + 1) (1 + z 2 ) z 2 + 1 z 2 + 1 (1 + z 2 ) z 2 + 1
z

Xt hm s
=
g ( z)

g '( z ) =

z +1
2

3 1 z2

(z

+1

1+ z

2z
2

z2 + 1

vi z dng ta c

; g '( z ) = 0 z = 1 .

z +1
2

Ta c g(z) i du t dng sang m khi i qua 1 v vy g ( z ) g (1) =


2.
Vi z = 1, x = 2 + 1, y = 2 1 th P bng

2.

Vy gi tr ln nht ca P bng 2 .
Bi 2. Cho a, b, c l cc s khng m tho mn iu kin ab + bc + ca =
4.
Tm gi tr nh nht ca biu thc
P=

Ta chng minh

a 2 + 9bc + b 2 + 9ac + c 2 + 9ba .


Li gii

a 2 + 9bc + b 2 + 9ac + c 2 + 9ba 5 ab + bc + ca .


617

www.TaiLieuLuyenThi.com
Khm ph t duy K thut gii bt T Bi ton Max Min ng Thnh Nam

Khng mt tnh tng qut gi s c = min {a, b, c} .


S dng bt ng thc Mincopski ta c

( a + b ) 2 + 9c (

a 2 + 9bc + b 2 + 9ac

a+ b

V c 2 + 9ab 3 ab .
Vy ta ch cn chng minh

( a + b )2 + 9c (

a+ b

( a + b )2 + 9c (

( a + b ) + 9c
2

+ 3 ab 5 ab + bc + ca

a+ b
a+ b

5 ab + bc + ca 3 ab

9ab + 25 ( ab + bc + ca ) 30 ab ( ab + bc + ca )

( a + b ) + 18c ab + 30 ab ( ab + bc + ca ) 34ab + 16c ( a + b )


2

( a + b ) + 18c ab 4ab 6c ( a + b )
2

c ( a + b )

+30 ab ( ab + bc + ca ) ab +
0
3


c (a + b)

ab ( ab + bc + ca ) ab +

3
c (a + b)

ab ( ab + bc + ca ) ab +
=
c (a + b)
3

ab ( ab + bc + ca ) + ab +
3

Ta c

c ( a + b )( 3ab bc ca )
9 ab ( ab + bc + ca ) + 9ab + 3c ( a + b )

0.

Vy ta chng minh ( a + b ) + 18c ab 4ab 6c ( a + b ) 0 .


2

Bt ng thc c dng thun nht nn chun ho a + b =


1.
a+b 1
.
t=
x
ab ,0 c x =
2
2
Ta cn chng minh f ( x) =4 x 2 + 18cx 6c + 1 0 .
V tri l tam thc bc hai vi h s x2 m nn ch cn ch ra rng

1
min f (c); f 0 .
2

2
1
Tht vy ta c f =3c 0; f (c) =5c 2 + ( 3c 1) > 0 .
2
Bt ng thc c chng minh.

618

www.TaiLieuLuyenThi.com
Cty TNHH MTV DVVH Khang Vit

Vy ta c a 2 + 9bc + b 2 + 9ac + c 2 + 9ba 10 .


Du bng t ti a= b= 2, c= 0 hoc cc hon v.
Vy gi tr nh nht ca P bng 10.
Bi 3. Cho a,b,c l di ba cnh ca mt tam gic v tha mn iu kin
a 2 + b2 + c2 =
3 . Tm gi tr nh nht ca biu thc
a+b
b+c
c+a
.
P=
+
+
a+bc
b+ca
c+ a b
Li gii
Ta chng minh gi tr nh nht ca P bng 6.
Tht vy ch cn chng minh rng
a+b

b+c

c+a

64

a 2 + b2 + c2
.
3

a+bc
b+ca
c+ a b
Bt ng thc c dng thun nht nn chun ho a + b + c =
3.

Ta cn chng minh rng

a 2 + b2 + c2
3 a
3b
3c
.
+
+
64
3
3 2a
3 2b
3 2c

3 x
x2 + 3
3

, x 0; .
2
3 2x
2
Tht vy bt ng thc cho tng ng vi:

Ta chng minh

4 (3 x ) x2 + 3
2

( 3 2 x ) ( x 1)2 ( 2 x3 + x 2 + 12 x + 9 ) 0 .

Bt ng thc c chng minh.


p dng bt ng thc kt hp bt ng thc AM GM ta c
3 a
3b
3c
a 2 + b 2 + c 2 + 3.3

+
+
2
3 2a
3 2b
3 2c

2 4 33 a 2 + b 2 + c 2 = 6 4

a 2 + b2 + c2
3

Ta c pcm.
Vy gi tr nh nht ca P bng 6 t ti a= b= c= 1 .
Bi 4. Cho x,y,z l cc s thc dng tho mn iu kin
x2 + y 2 + z 2 + 1 2 x + 3 y .

Tm gi tr nh nht ca biu thc


P=

x3 + x 2 + 36 y 3 + y 2 + 36 2 z 3 + z 2 + 9
.
+
+
2x + 2
4y + 4
2z + 1

619

www.TaiLieuLuyenThi.com
Khm ph t duy K thut gii bt T Bi ton Max Min ng Thnh Nam

Li gii
2

x
y
18
9
9
.
+
+ z2 +
+
+
2
4
x + 1 y + 1 2z + 1

Ta c P =

Da vo biu thc ca P trn ta d on du bng t ti


x + 1 = y + 1 = 2z + 1 x = y = 2z .
Thay vo iu kin ta c: x= y= 2, z= 1 .
Da vo ta c cc nh gi sau y:
S dng bt ng thc C S ta c
x2 y 2
2 x2 + y 2 + 4 z 2 1
2
=
+
+ z2
( 2x + y + 2z ) .
2
4
4
16
18
9
9
9
9
9
9
+
+
=
+
+
+
x + 1 y + 1 2z + 1 x + 1 x + 1 y + 1 2z + 1

Do P

9.42
144
=
2 ( x + 1) + y + 1 + 2 z + 1 2 x + y + 2 z + 4

144
1
2
+ ( 2x + y + 2z ) .
2 x + y + 2 z + 4 16

Ch gi thit bi ton ta c 2 x + 3 y x 2 + y 2 + z 2 + 1
=

(x

) (

) (

+ 4 + y2 + 4 + z2 + 1 8

4x + 4 y + 2z 8
2x + y + 2z 8

1 2 144
t t = 2 x + y + 2 z , t ( 0;8] ta c P =
t +
t+4
16

( t 8)2 ( t + 20 ) + 16 16 .
16 ( t + 4 )

ng thc xy ra khi v ch khi x= y= 2, z= 1 .


Vy gi tr nh nht ca P bng 16.
Bi 5. Cho a,b,c l cc s thc dng tho mn iu kin a 2 + b 2 + c 2 =
14 .
Tm gi tr ln nht ca biu thc
=
P

4(a + c)
a + 3c + 28
2

4a
a + bc + 7
2

(a + b)

3
.
a (b + c )

Li gii
Ch n gi thit bi ton v tch a(b+c) ta c nh gi nh sau:
14 + 2bc = a 2 + ( b + c ) 2a ( b + c ) a ( b + c ) 7 + bc .
2

620

www.TaiLieuLuyenThi.com
Cty TNHH MTV DVVH Khang Vit

Suy ra

4a
a + bc + 7
2

4a

4
.
=
a + a (b + c ) a + b + c
2

a+b+c
S dng bt ng thc AM GM ta c: a ( b + c )

3
3
12
.

=
2
a (b + c )
( a + b + c )2
a+b+c

Cng theo v hai bt ng thc trn ta c:


4a
3
12
4

+
2
2
a + bc + 7 a ( b + c )
(a + b + c) a + b + c

1
1 1 1

=
12
+
a+b+c 6 3 3
ng thc xy ra khi v ch khi a = b + c; a + b + c = 6 .
S dng bt ng thc C S ta c:

3
2
2
1 2
2
2
2
1 + a + 3c ( a + c ) a + 3c ( a + c ) .
3
4

Suy ra

4(a + c)
a + 3c + 28
2

( a + b )2

4(a + c)

3
( a + c )2 + 28
4
2
1
( a + b 5) .
25
5

1
2
( a + c 4) + .
25
5

Ch hai bt ng thc trn ta s dng bt ng thc dng tip tuyn.


Cng theo v hai bt ng thc trn v s dng bt ng thc C S ta c:
4(a + c)
5
3a + 2b + c 9

2
2
2
25
a + 3c + 28 ( a + b )

(a

)(

+ b 2 + c 2 32 + 22 + 12 9
25

1
=
5

1 1 8
T suy ra Pmax = + = t ti=
a 3,=
b 2,=
c 1.
3 5 15
Nhn xt. Ta c th nh gi phn thc u tin bng cch khc nh sau:
a 2 + 3c 2 + 28 = 2a 2 + b 2 + 4c 2 + 14 2a 2 + 4bc + 14 .

Mt khc: 2a 2 + 2bc + 2 ( bc + 7 ) 2a 2 + 2bc + 2a ( b + c ) = 2 ( a + b )( a + c ) .


621

www.TaiLieuLuyenThi.com
Khm ph t duy K thut gii bt T Bi ton Max Min ng Thnh Nam

Suy ra

4(a + c)

2
.
a+b

a + 3c + 28
2
4
5
12
Do P
+

a + b a + b + c ( a + b )2 ( a + b + c )2
2

8 1 a +b5 1 a +b+c6
8



15 5 a + b 3 a + b + c 15
Ta c kt qu tng t trn.
Bi tp tng t
1) Cho a,b,c l cc s thc dng tho mn iu kin a + b + c =
3.
Tm gi tr ln nht ca biu thc
=

( 2a + b + c )2 + ( 2b + c + a )2 + 8
P=
2
2
2a 2 + ( b + c )
2b 2 + ( c + a )

(a + b 3 c + 3) .
2

9c

2) Cho a,b,c l cc s thc dng tho mn iu kin a 2 + b 2 + c 2 =


26 .
Tm gi tr ln nht ca biu thc
=
P

4(a + c)
2a + 4bc + 26
2

4a
a + bc + 13
2

4
7
.

a ( b + c ) ( a + b )2

Bi 6. Cho a, b, c l cc s thuc on [0, 3].


Tm gi tr ln nht ca biu thc P =

( a b )( b c )( c a )( a + b + c ) .

Li gii
Gi s a = max {a, b, c} .
Nu a b c A 0 .
Nu a c b .
Khi
P (a, b, c) P (a,0, c) = ( a b )( b c )( c a )( a + b + c ) ac ( a c )( a + c )
=

( a c ) ( a b )( c b )( a + b + c ) ac ( a + c )

= ( a c ) b b 2 ac c 2 a 2 0

Do P (a, b, c) P (a,0, c) = ac ( a c )( a + c ) = a3c ac3 .


Xt hm s f (=
a ) a3c ac3 trn on [0;3] ta c

f=
'(a ) c 3a 2 c 2 0; 3 a c 0 .

Do f(a) ng bin trn on [0;3] suy ra


622

www.TaiLieuLuyenThi.com
Cty TNHH MTV DVVH Khang Vit

f (a ) f (3) =g (c) =27c 3c3 .

Xt hm s g (=
c) 27c 3c3 trn on [0;3] ta c
g '(c) = 27 9c 2 ; g '(c) = 0 c = 3 .

Ta c g(c) i du t dng sang m khi i qua c = 3 nn g(c) t cc i ti


c= 3.

V vy g (c) g ( 3) =
18 3 .
a 3,=
b 0,=
c
Suy ra Pmax = 18 3 . Du bng t t=

Bi tp tng t
Cho a, b, c l cc s thuc on [0, 2].
Tm gi tr ln nht ca biu thc P =

3.

( a b )( b c )( c a )( a + b + c ) .

Bi 7. Cho x,y,z l cc s thc dng tha mn iu kin x + 3 y 9 z v x > y > z .


2

x
z
y
Tm gi tr nh nht ca biu thc P =
+ 3
.
+ 3
xz
yz
x y
Li gii

Ta c ( a 3) 0 a 2 6a 9 . ng thc xy ra khi v ch khi a = 3 .


2

( b 1)2 0 b2 2b 1 . ng thc xy ra khi v ch khi b = 1 .


p dng vo bi ton ta c:
2

x
z
x
y
z
y
Ta c:
9;
1;
1.
6.
2.
2.

y
z
y
z
x
z
x
z
x
y
x
y

x
y
z
Suy ra: P 6
+
+
15 = 6Q 15 .
yz xz x y

Vi Q =

x
y
z
x2
z2
y
.
+
+
=
+
+
y z x z x y x( y z) z ( x y) x z

( x + z)
x2
z2
.
+

x( y z) z ( x y) x( y z) + z ( x y)
2

S dng bt ng thc C-S ta c:


Suy ra: Q

( x + z )2 + y
( x + z )2
y
+
=
x( y z) + z ( x y) x z y( x z) x z

2
2
x+z
y
y
y x+z

=
+
=
+
.
1


y x z x z x z y

623

www.TaiLieuLuyenThi.com
Khm ph t duy K thut gii bt T Bi ton Max Min ng Thnh Nam
2

x+z

x+z
x+z
Mt khc
2 0
4.
4.
y
y

y
x+z
x+z
y
4x + 4z 3y
1 + 4.
=
4 4.
3. =
5.

xz
y
xz
xz
xz

V x + 3 y 9 z .

Do Q

Ti=
x 3,=
y 2,=
z 1 th P = 15 .
Vy gi tr nh nht ca P bng 15 t ti=
x 3,=
y 2,=
z 1.
Cch 2: Theo gi thit tn ti cc s dng a,b sao cho

x =+
( a 1) z, y =+
( b 1) z, ( a > b ) .

Ta c: x + 3 y 9 z ( x z ) + 3 ( y z ) 5 z a + 3b 5 .
2

a +1
b +1
1
Khi P =
+ 3
+ 3
.
b
a
a b
S dng bt ng thc C-S ta c
2
2
a + 1 2
1
b +1
1
a +1 b +1
3
3
+
+
+
+

3
.
a
a b
a
a b
b
b

( 9 + 3 + 3)

1
1
a +1 b +1
a b b +1 b +1
+
+
=
+
+
+
b
a
a b a b
b
b
a
1 a b b +1 b +1 b +1 b +1
2
2
+
+
=
+
+
.
5 3b
5 3b
a b b
b
b
b

Xt Q =

b +1 b +1
2
Mt khc
+
+
= 5+
b
5 3b
b

) (13b + 20

b 1

b ( 5 3b )

b +5

) 5, b 0; 5 .

Suy ra Q 5 P 15 .
ng thc xy ra khi v ch khi b =1 a =2 2 x =3 y =6 z .
Nhn xt. y l mt bi ton hay v kh biu thc ca P l dng ng bc nn
chc hn nhiu hc sinh s ngh n php t nh trn.
Bi 8. Vi mi a,b,c tm s thc M nh nht tho mn bt ng thc

ab a 2 b 2 + bc b 2 c 2 + ca c 2 a 2 M a 2 + b 2 + c 2

Ch ab a b
2

) + bc (b

Li gii

) + ca ( c

a2 =

( a b )( b c )( c a )( a + b + c ) .

Vy ta cn tm s thc M nh nht sao cho

( a b )( b c )( c a )( a + b + c ) M ( a 2 + b2 + c 2 )
624

vi mi a,b,c.

www.TaiLieuLuyenThi.com
Cty TNHH MTV DVVH Khang Vit

Bt ng thc thun nht nn chun ho a 2 + b 2 + c 2 =


1.
Ta cn tm M nh nht sao cho
M ( a b )( b c )( c a )( a + b + c ) =
M max ( a b )( b c )( c a )( a + b + c ) .

K hiu P =

( a b )( b c )( c a )( a + b + c )

Khng mt tnh tng qut gi s a b c khi


2

1
ac
3
(a b)(b c)
P (a c) ( a + b + c ) .
4
2

V (a c) 2 =
( a b ) + ( b c )

=( a c ) + ( b c ) + 2(a b)(b c)
2

( a c ) + (b c ) +
2

1
( a c )2
2

3
(a c) 2 (a b) 2 + (b c) 2 + (c a ) 2
2

1 2
2
2
2
2 3
(
)
(
)
(
)
.( a + b + c )
a
b
b
c
c
a

4 3

ng thc xy ra khi v ch khi a b = b c a + c = 2b .


Khi s dng bt ng thc AM GM ta c
P

(a b) 2 + (b c) 2 + (c a ) 2
2
a + b + c ) + 3.
(

2
3

P
2
4

4
2
9 2
+ c2
. 34 a 2 + b 2=
32
32

a 2 + b 2 + c 2 =
1

ng thc xy ra khi v ch khi a + c =


2b

(a b) 2 + (b c) 2 + (c a ) 2
( a + b + c )2 =

Vy gi tr cn tm ca M l

9 2
.
32

Bi 9. Cho a,b,c l cc s thc dng tha mn iu kin a + 6 ( b + c ) 15 .


Tm gi tr nh nht ca biu thc P= 3 ( a + b + c ) 5abc .

625

www.TaiLieuLuyenThi.com
Khm ph t duy K thut gii bt T Bi ton Max Min ng Thnh Nam

Li gii
Nhn xt. iu kin i xng vi b v c nn ta nhm li biu thc ca P nh sau
P = a ( 3 5bc ) + 3 ( b + c ) .

+ Nu 3 5bc > 0 P > 0 .


3
5
+ Nu 3 5bc 0 bc khi theo gi thit ta c a 15 6 ( b + c ) ; b + c < .
5
2
Suy ra
P 15 6 ( b + c ) ( 3 5bc ) + 3 ( b + c ) =
15 ( b + c ) + 30bc ( b + c ) + 45 75bc
=
15 ( b + c ) + 15bc ( 2b + 2c 5 ) + 45 15 ( b + c ) +
=

15
( b + c )2 ( 2b + 2c 5) + 45
4

15
75
( b + c )3 ( b + c )2 15 ( b + c ) + 45
2
4
15 3 75 2
3 3
t t = b + c 2 bc 2
t t 15t + 45 .
> khi P f (t ) =
5 2
4
4

15 3 75 2
3
t t 15t + 45 vi t > ta c
4
4
2
15 2
15
f '(t ) =
3t 5t 2 = ( t 2 )( 3t + 1) ; f '(t ) = 0 t = 2 .
2
2
Ta c f(t) i du t m sang dng khi i qua t = 2 nn f(t) t cc tiu ti t = 2 .
Do P f (t ) f (2) =
0 . ng thc xy ra khi v ch khi b= c= 1, a= 3 .

Xt hm s f (t ) =

Vy gi tr nh nht ca P bng 0 t ti a= 3, b= c= 1 .
Bi 10. Cho x,y,z l cc s thc khng m tha mn iu kin

5 x 2 + y 2 + z 2 = 6 ( xy + yz + zx ) .

Tm gi tr ln nht ca biu thc =


P

2( x + y + z ) y2 + z2 .

Li gii
Nhn xt. Biu thc ca P i xng vi 2 bin y v z nn ta ngh n vic nh gi
biu thc P theo y + z v ng thc iu kin c dng ng cp nn lm ta suy
ngh n vic nh gi x theo y + z . tm gi tr ln nht ca P ta tm cn trn
ca x theo y + z .
Ta c
5
1
5 x 2 + ( y + z ) 2 5 x 2 + 5( y 2 + z 2 )= 6( xy + yz + zx) 6 x( y + z ) + 6. ( y + z ) 2 .
2
4
Do 5 x 2 6 x( y + z ) + ( y + z ) 2 0,
626

www.TaiLieuLuyenThi.com
Cty TNHH MTV DVVH Khang Vit

Suy ra x + y + z 2( y + z ) .
1
1
1
Khi P 2( x + y + z ) ( y + z )2 4( y + z ) ( y + z )2 = 2 y + z ( y + z )2 .
2
2
2

y+z =
t , khi t 0 v P 2t

t4
.
2

(1)

1
Xt hm s f (t=
) 2t t 4 vi t 0.
2

Ta c f '(t ) = 2 2t 3 ; f '(t ) = 0 t = 1.
Suy ra bng bin thin:
t

f '(t )

1
+

0
3
2

f (t )

3
vi mi t 0.
(2)
Da vo bng bin thin ta c f (t ) f (1) =
2
x= y + z
x = 1
3

T (1) v (2) ta c P , du ng thc xy ra khi=

y
z

1
2
y= z=
y + z =

1
2

Vy gi tr ln nht ca P l

3
1
.
, t ti x= 1, y= z=
2
2

Bi tp tng t
Cho x,y,z l cc s thc khng m tha mn iu kin

8 x2 + y 2 + z 2 = 3( x + y + z ) .

Tm gi tr ln nht ca biu thc =


P

2( x + y + z ) y2 + z2 .

Bi 11. Cho x,y,z l cc s thc dng thay i tha mn iu kin


x2 + y 2 + z 2 =
1.

Tm gi tr ln nht ca biu thc P =

xy

1 + z2
Li gii

yz
1 + x2

x3 y 3 + y 3 z 3
24 x3 z 3

Ta c:

627

www.TaiLieuLuyenThi.com
Khm ph t duy K thut gii bt T Bi ton Max Min ng Thnh Nam

x3 y 3 + y 3 z 3
xy
yz
1 y3 y3
=
+

+ .

1 + z 2 1 + x2
24 x3 z 3
x 2 + y 2 + 2 z 2 y 2 + z 2 + 2 x 2 24 x3 z 3
S dng bt ng thc AM-GM ta c:
xy

yz

xy
x2 + y 2 + 2 z 2

yz
y 2 + z 2 + 2 x2

( x + y )2

4 x2 + y 2 + 2 z 2

) 4( y

( y + z )2
2

+ z 2 + 2 x2

S dng bt ng thc C-S ta c:

( x + y )2
x2 + y 2 + 2 z 2

( y + z )2
y 2 + z 2 + 2 x2

x2
x2 + z 2

y2
y2 + z2

z2
x2 + z 2

y2
x2 + y 2

1 y2 1 y2

+
1
1 + + 1 +
4 x 2 4 z 2
x2 + y 2 y 2 + z 2
3 1 y2 y2
=
+ +
2 4 x 2 z 2
1 3 1 y 2 y 2 1 y3 y3
Suy ra: P + 2 + 2 3 + 3
4 2 4 x
z 24 x
z
=
1+

y2

y2

3 1 3 y 2 3 y 2 2 y3 2 y3
=+ 2 + 2 3 3
8 48 x
z
x
z
3 1 3 y 2 2 y3 1 y 2 2 y3
=
+
3 + 3 2 3
8 48 x 2
x 48 z
z

Xt hm s f (=
t ) 3t 2 2t 3 trn ( 0;+ ) , ta c:
f '(t ) =6t 6t 2 =6t (1 t ) suy ra f(t) i du t dng sang m khi i qua
t = 1 do f(t) t cc i ti t = 1 trn ( 0;+ ) .

Suy ra f (t ) f (1) = 1, t > 0 .


3 1
Tc P +
8 48

y
f +
x

5
y 3 1
f + (1 + 1) = .
12
z 8 48

5
t ti x= y= z=
12

.
3
Bi 12. Cho a,b,c l cc s thc khng ng thi bng 0 tho mn iu kin
a+b+c =
0.
Vy gi tr ln nht ca P bng

Tm gi tr ln nht ca biu thc P =

628

13a 2b 2 c 2 2abc 2

(a

+b +c
2

2 3

www.TaiLieuLuyenThi.com
Cty TNHH MTV DVVH Khang Vit

Li gii
Theo gi thit a,b,c l ba nghim ca phng trnh: x3 + qx r =
0 vi
p = a + b + c = 0, q = ab + bc + ca, r = abc .

iu kin phng trnh c ba nghim l hm s y = x3 + qx r c


yCD . yCT 0 .
q 0
q

Ta c y ' =
3 x + q; y ' =
0 x =

q .
3 x =
3

Khi
2

q q

2 4q 3
2q q 2q
q
27
y

r
0 q3 r 2 .
. y
= r +

= r +

3 3
3 3 3
3
27
4

Ta ch cn tm gi tr ln nht ca P dng vy xt 13r 2 2r 2 > 0 .


Khi
13r 2 2r 2 13r 2 2r 2
Ch a 2 + b 2 + c 2 =
.
p 2 2q =
2q P =
=
(2q )3
8q3
2

13r 2 2r 2 13r 2 2r 2 13
1
1
1 1 1 1
1
P=

= + .
3
2
27
54 27 r 27 r
4 27 r 2
4
8q
8. r 2
4

p 0=
=
p 0

ng thc xy ra khi v ch khi r =


2
q =
3 .

27
r = 2
q3 = r 2
4

Do a,b,c l ba nghim ca phng trnh

x 2 3 x + 2 = 0 ( x 1)

x =1
.
x = 2

( x + 2) = 0

Vy gi tr ln nht ca P bng t ti a = b = 1, c = 2 hoc cc hon v.


Bi 13. Cho a,b,c l cc s thc dng tha mn iu kin
a 2 + b 2 + ( a + b ) c + 4c 2 =
4.

Tm gi tr ln nht ca biu thc


P=

a (b + c )
a+c

b(a + c)
b+c

8 7c 2 c
.
c (a + b)
629

www.TaiLieuLuyenThi.com
Khm ph t duy K thut gii bt T Bi ton Max Min ng Thnh Nam

Li gii
Ta c: a 2 + b 2 + c ( a + b ) + 4c 2 =4 4 4c 2 + c ( a + b ) +

1
( a + b )2 .
2

t 2 + 2tc + 8c 2 8 0 ( t + c ) 8 7c 2 t c + 8 7c 2 .
2

a (b + c )

b(a + c)

1
.
a+c
b+c
c
t a + c= x, b + c= y khi theo gi thit ta c:

Do P

( x c )2 + ( y c )2 + ( x + y 2c ) c + 4c 2 =4 x 2 + y 2 c ( x + y ) =4 4c 2 .
Mt khc:
a (b + c )
a+c

b(a + c)
b+c

y2 ( x c)
x

x2 ( y c )
y

x2 y 2
= x2 + y 2 c +
.
y
x

S dng bt ng thc AM-GM ta c:


x2 y 2
1
1
+
x + y P x 2 + y 2 c ( x + y ) =4c 2 + 4 .
y
x
c
c

1
Xt hm s f (c) =
4c 2 + 4 vi c > 0 ta c:
c
1
1
1
f '(c) =8c + 2 ; f '(c) =0 8c = 2 c = .
2
c
c

Ta c f(c) i du t dng sang m khi i qua c =


c=

1
nn f(c) t cc tiu ti
2

1
1
hay P f (c) f =
1.
2
2

Vy gi tr ln nht ca P bng 1 t ti a= b= 1, c=

1
.
2

Bi tp tng t

1) Cho a,b,c l cc s thc dng tho mn iu kin a > b; ( a + c )( b + c ) =


1.
Tm gi tr nh nht ca biu thc P =

(a b)

(b + c )

12

( c + a )2

2) Cho a,b,c l cc s thc dng tho mn iu kin a 2 + b 2 + ( a + b ) c + 4c 2 =


4.
Tm gi tr ln nht ca biu thc P =

630

a (b + c )
a+c

b(a + c)
b+c

1
.
c

www.TaiLieuLuyenThi.com
Cty TNHH MTV DVVH Khang Vit

Bi 14. Cho a,b,c l cc s thc dng. Tm gi tr nh nht ca biu thc


=
P

( a + b + c )( ab + bc + ca ) +
abc

4bc

( b + c )2

Li gii
S dng bt ng thc AM GM ta c:

( a + b + c )( ab + bc + ca ) =
b + c a ( b + c ) b2 + c2
3+
+
+
abc

a
bc
2 ( b + c ) b2 + c2
3+
+
bc
bc

bc

2 (b + c ) (b + c )
b+c
=
+
=+
1+
1

bc
bc
bc

b+c
4bc
Do P
.
+1+
bc
( b + c )2
2

t t
=

( b + c )2 ,
bc

( t 4 ) ta c

4
P f (t ) =+
1
t+ .
t

4
Xt hm s f (t ) =+
1 t + vi t 4 ta c
t

4 t t 8
=
0, t 4 do f(t) l hm ng bin trn [ 4;+ ) suy ra
2 t t2
2t 2
P f (t ) f (4) =
4 . ng thc xy ra khi v ch khi a= b= c .
f '(=
t)

Cch 2: Ta c th nh gi biu thc trong cn nh sau:

( a + b + c )( ab + bc + ca ) = ( a + b + c ) a ( b + c ) + bc

= a 2 ( b + c ) + bc ( b + c ) + a ( b + c ) + abc
2

2a ( b + c ) bc + abc + a ( b + c )

Suy ra

( a + b + c )( ab + bc + ca )

2a ( b + c ) bc + abc + a ( b + c )

abc
Ta c kt qu tng t trn.

abc

b+c
=
+1.
bc

1 2
Bi 15. Cho x,y,z,t l cc s thc thuc on ; .
2 3
Tm gi tr ln nht v gi tr nh nht ca biu thc
2

z+t
x+z
=
P 9
.
+ 16
x+t
x+ y
631

www.TaiLieuLuyenThi.com
Khm ph t duy K thut gii bt T Bi ton Max Min ng Thnh Nam

Li gii
S dng bt ng thc AM-GM ta c:
2
2
z+t
z+t
( x + z )( z + t )
x+z
x+z
9
16
2
9
24.
+

.16
( x + t )( x + y )
x+t
x+t
x+ y
x+ y
2

1 1

x + t +
( 2 x + 1)( 2t + 1)
2 2
24.
=
18.
2 2
4( x + t )
( x + t ) +
3 3

1 4 xt + 1
1 1
= 18 +
18 + = 18
2 2
2 4 ( x + t )

Do 2 ( x + t ) 4 xt 1 = (1 2 x )( 2t 1) 0 .
Vy gi tr nh nht ca P bng 18 t ti x = y =

2
1
.
, z= t=
3
2

Tm gi tr ln nht ca P
2 1 2
2
x+
+
x+
x+z
7
3
2
3
3 l hm nghch bin vi x).
Ta c:

= (do
1
x+t x+t
x+t
+ t 3 ( 2t + 1)
2
2
2
+t
z+t
2
49
2
Tng t ta c:
3
=
t + . Suy ra P f (=
+
+
t)
16
t

.
x+ y 1 + 1
3
3
( 2t + 1)2
2 2
Xt hm s f (=
t)

49

( 2t + 1)

2
+ 16 t + lin tc trn on
3

1 2
2 ; 3 ta c:

196
196
77
2
1 2
f '(t ) = 32 t +
32 +
=
>0.
3
3
6
3 ( 2t + 1)
2 3 1
2. + 1
2
1 2
Do f(t) l hm ng bin trn ; .
2 3
2 337
Do P f (t ) f =.
3
3

1
2
.
, z= t=
2
3
337
1
2
Vy gi tr ln nht ca P bng
t ti x = y =
.
, z= t=
2
3
3
632

ng thc xy ra khi v ch khi x = y =

www.TaiLieuLuyenThi.com
Cty TNHH MTV DVVH Khang Vit

Bi 16. Cho x,y,z l cc s thc tha mn iu kin x 2 + y 2 + z 2 =


3.
Tm gi tr ln nht ca biu thc P=

3x 2 + 7 y + 3x 2 + 7 z + 5 y + 5 z .

Li gii
S dng bt ng thc C-S ta c:

P 2 3 6 x 2 + 7 y + 7 z + 5 y + 5 z= 18 x 2 + 2 ( y + z )

18 x 2 + 2 2 y 2 + z 2 = 18 x 2 + 2 2 3 x 2

Xt hm s f ( x) =18 x 2 + 2 2 3 x 2 lin tc trn 3; 3 ta c:

2x

f '( x) =
36 x
2 3 x2

x = 0
x = 0

0
x =
1 .
; f '( x) =
2

=
2
3
x
2

x = 1

Ta c f (0)= 2 6, f ( 3)= f ( 3)= 3, f (1)= f (1)= 5 .


Suy ra P f ( x)

max

x 3; 3

f ( x) =
5.

ng thc xy ra khi v ch khi x =


1, y =
z=
1.
Vy gi tr ln nht ca P bng 5 t ti x =
1, y =
z=
1 hoc x= y= z= 1 .

Bi 17. Cho x,y,z l cc s thc dng tha mn iu kin y +=


z x y2 + z2 .
Tm gi tr nh nht ca biu thc
1
1
1
4
.
P=
+
+
+
2
2
2
( x + 1) ( y + 1) ( z + 1) ( x + 1)( y + 1)( z + 1)
Li gii
1
2
2
Theo gi thit ta c: y +=
z x y2 + z2 x ( y + z ) y + z .
2
x
S dng bt ng thc AM-GM ta c:

( y + 1)2 ( z + 1)2 ( y + 1)( z + 1) ( y + z + 2 )2


1
4

(1 + x )(1 + y )(1 + z ) (1 + x )( y + z + 2 )2

2 x2
=
2
( x + 1)2
2

2
+

x2
.
=
2
3
x + 1)
(
2

(1 + x ) + 2
x

633

www.TaiLieuLuyenThi.com
Khm ph t duy K thut gii bt T Bi ton Max Min ng Thnh Nam

Suy ra P

4 x2

(1 + x )2 (1 + x )2 (1 + x )3

Xt hm s f ( x) =
f '( x) =

2 x2

2 ( 5 x 1)

(1 + x )

2 x3 + 6 x 2 + x + 1

(1 + x )3

2 x3 + 6 x 2 + x + 1
.
= 3
(1 + x )

vi x > 0 ta c:

1
; f '( x) = 0 x = .
5

Ta c f(x) i du t m sang dng khi i qua x =


x=

1
nn f(x) t cc tiu ti
5

1
1 91
hay P f ( x) f =.
5
5 108

1
, y= z= 5 .
5
1
91
t ti x= , y= z= 5 .
Vy gi tr nh nht ca P bng
5
108
Bi 18. Cho a,b,c l cc s thc khng m tha mn iu kin a > b > c v
3ab + 5bc + 7ca 9 . Tm gi tr nh nht ca biu thc
32
1
1
.
P=
+
+
4
4
( a b ) ( b c ) ( c a )4

ng thc xy ra khi v ch khi x=

Li gii
Theo gi thit ta c a > b > c 0 9 7ca + 5bc + 3ab 3ab ab 3 .
Khi :
1
1
a 2b 2 32

+
+
( a b )4 a 4 b4 9 ( a b )4 a 4 b4

2
2 2
2
2
1
32a b
32
a
b 1
a b

=
=
+
+
+
+

2
2
2
2

9 a 2 2ab + b 2
b
a 9 a b
b a

+ 2

b a

32

1 32
a b
2

.
t t =+ , ( t > 2 ) do a > b khi P=
f (t )
+
t

2
9 ( t 2 )2
b a

Xt hm s
=
f (t )

634

1 32
2

vi t > 2 ta c:
+
t

2
9 ( t 2 )2

www.TaiLieuLuyenThi.com
Cty TNHH MTV DVVH Khang Vit

1
64

; f '(t ) =
f '(t ) =
+
2
t
0
9 ( t 2 )3

2t ( t 2 ) = 64 t = 4
3

Ta c f(t) i du t m sang dng khi i qua t = 4 nn ti t = 4 f(t) t cc


10
.
tiu hay P f (t ) f (4) =
9
ng thc xy ra khi v ch khi:

=
a
6+3 3
c = 0

b = 2 + 3 2 3 3 .
ab = 3
a b

+ =
c = 0
4

b a

Vy gi tr nh nht ca P bng

10
t ti
9

a = 6 + 3 3 , b = 2 + 3 2 3 3 , c =0 .

Bi 19. Cho a,b,c l cc s thc khng m tho mn c = min {a, b, c}


v a 2 + b 2 + c 2 =
3 . Tm gi tr ln nht ca biu thc
6
.
P = 2ab + 3bc + 3ca +
a+b+c
Li gii
Trc ht ta c: 2ab + 3bc + 3ca ( a + b + c ) 1
2

Suy ra P ( a + b + c ) +
2

t t = a + b + c,

3 < t 3 khi P f (t ) = t 2 +

Xt hm s f (t ) = t 2 +
6

6
1 .
a+b+c

6
1 vi t
t

6
1.
t

3;3 ta c:

> 0, t 3;3 P f (t ) f (3)= 10 .


t2
Vy gi tr ln nht ca P bng 10 t ti a= b= c= 1 .
f '(t )= 2t

Nhn xt. Bt ng thc 2ab + 3bc + 3ca ( a + b + c ) 1 c c nh d on


2

du bng t ti a= b= c= 1 v tng a v kho st hm s vi t = a + b + c .

635

www.TaiLieuLuyenThi.com
Khm ph t duy K thut gii bt T Bi ton Max Min ng Thnh Nam

Chng minh.

( a + b + c )2 1 2ab 3bc 3ca =

2 bc ca

= 2 c ( a + b ) 2 c 2 a 2 + b2 = 2 c 2 3 c2

= 2 2c 2 3 c 2 2

2c 2 + 3 c 2 1 c 2
=
0
2
2

Bi tp tng t
Cho a,b,c l cc s thc khng m c c = min {a, b, c} v a 2 + b 2 + c 2 =
3.
Tm gi tr ln nht=
ca biu thc P

2ab + 3bc + 3ca


6
.

2
( a + b + 2c )2 17

Bi 20. Cho a,b,c l cc s thc dng tha mn iu kin abc ( a + b + c ) =


4.
Tm gi tr nh nht ca
biu thc P
=

( a + b )( a + c )

8bc

bc b + c 2 + 8
2

Li gii
Nhn xt. Nhn vo biu thc P cho ta thy P i xng vi b v c nn ta tm cch
nh gi xoay quanh hai bin ny. V d thy p dng AM-GM cho a + b, a + c .
S dng bt ng thc AM-GM ta c:
a + b + a + c 2a + b + c
.
=
( a + b )( a + c )
2
2
Vy tm cch nh gi theo 2a + b + c .
8bc
8
8
Ta c:
= = 2
2
bc b 2 + c 2 + 8 b 2 + c 2 + 8 b + c + 2a ( a + b + c )
bc
8
16

=
2
1
( b + c ) 2 + 2a ( b + c ) + 2a 2 ( 2a + b + c )
2
2
16
Suy ra P
.

2a + b + c ( 2a + b + c )2

2 16
t t = 2a + b + c, ( t > 0 ) khi P f (t ) = 2 .
t t
16
2 16
+ ( b + c ) + 4bc 16 t 4 .
bc
bc
2 16
2 32
Xt hm s f (t )=
vi t 4 ta c f '(t ) = 2 + 3 ; f '(t ) =0 t =16 .
t t2
t
t

Ta c: t 2 = 4a 2 + 4a ( b + c ) + ( b + c ) =
2

636

www.TaiLieuLuyenThi.com
Cty TNHH MTV DVVH Khang Vit

1
Lp bng bin thin suy ra P f (t ) f (4) =
.
2
ng thc xy ra khi a =
2 2, b =
c=
2.
Bi 21. Cho a,b,c l cc s thc dng tha mn iu kin a b c .
a+b

Tm gi tr nh nht=
ca biu thc P

ab 2 + bc 2 + ca 2
Li gii

b a 2 + ac + c 2
b+c

).

Do a b c > 0 nn ta c:

( b a )( b c ) 0 b2 + ac ab + bc ab2 + ca 2 a 2b + abc

ab 2 + bc 2 + ca 2 a 2b + abc + bc 2 = b a 2 + ac + c 2

Mt khc: a + b 2b, b + c 2b suy ra:


P

=2

2b

b a 2 + ac + c 2

a 2 + ac + c 2

b a 2 + ac + c 2

2b

1 a 2 + ac + c 2
2
2
b

ng thc xy ra khi v ch khi a = b = c,

1
4
a =b =c = .
2
3

a + ac + c
4
Vy gi tr nh nht ca P bng 2 t ti a= b= c=
.
3
Bi 22. Cho a,b,c l cc s thc dng tha mn iu kin a + b + c =
1.
2

a2
b2
4c 2
.
+
+
1 + b 1 + a 2 + 2a 2 + 2b 2
Li gii

Tm gi tr nh nht ca biu thc P =

Ta c c = 1 ( a + b ) 1 2 a 2 + b

4c

2 + 2 a 2 + b2

(
2(a

)
+b )

4 1 2 a 2 + b 2


2+

a 2 + b2
a2
b2
a4
b4
+ =
+

1 + b 1 + a a 2 + a 2b b 2 + ab 2 a 2 + b 2 + ab ( a + b )

a 2 + b2

(a
1
+ (a
2

+ b2

+ b2

)
) 2(a
2

+ b2

2 a 2 + b2
=
2 + 2 a 2 + b2

637

www.TaiLieuLuyenThi.com
Khm ph t duy K thut gii bt T Bi ton Max Min ng Thnh Nam

Suy ra P

t t =

2 a +b
2

2 + 2 a 2 + b2

(
2(a

)
+b )

4 1 2 a 2 + b 2

+
2+

2 a 2 + b 2 , ( t > 0 ) ta c P=
f (t )

t 2 + 4 (1 t )
5t 2 8t + 4
=
t+2
t+2
2

5t 2 8t + 4
vi t > 0 ta c
t+2

Xt hm s f (t ) =

5 t 2 + 4t 4
f '(t ) =
; f '(t ) =0 t =2 + 2 2 ( t > 0 ) .
( t + 2 )2

Ta c f(t) i du t m sang dng khi i qua t =2 + 2 2 nn f(t) t cc

tiu ti t =2 + 2 2 hay P f (t ) f 2 + 2 =
2 20 2 28 .

2
2
2 a + b =2 + 2 2
a =b =1 + 2

ng thc xy =
ra khi v ch khi a b

a + b + c =
c= 3 2 2
1

Vy gi tr nh nht P bng 20 2 28 t ti a =b =1 + 2, c =3 2 2 .
Bi 23. Cho x,y,z l cc s thc khng m. Tm gi tr ln nht ca biu thc
1
5
.
=
P

x + 2 y + 3 z + 1 ( x + 2 y )( 3z + 1)
Li gii
S dng bt ng thc AM-GM ta c:
x + 2 y + 3z + 1

( x + 2 y )( 3z + 1)

ng thc xy ra khi v ch khi x + 2 y =3 z + 1


P

1
20

x + 2 y + 3 z + 1 ( x + 2 y + 3 z + 1)2
2

1
1
1
1
=
20
+

x + 2 y + 3 z + 1 40 80 80

Vi x + 2 y = 20; z =

638

19
1
1
th P bng
. Vy GTLN ca P bng
.
3
80
80

www.TaiLieuLuyenThi.com
Cty TNHH MTV DVVH Khang Vit

1
Bi 24. Cho a,b,c l cc s thc dng thuc on ;1 .
2
a
b
c
Tm gi tr ln nht ca biu
thc P
.
=
.
+
b+c a+c a+b
Li gii

( c + a )( c + b ) ( c +
ab

t
=
t

ab
ab

)=
2

c
2c

+ 1
+ 1

a
+
b

ab

1

c
+
2
a + b 2c

+ 1

a+b

c 1
, t 1.
a+b 4

Ta c P f (t ) =
f (t ) =
t+
Xt hm s f (t ) = t +

f '(t ) = 1

( 2t + 1)3

( 2t + 1)2

( 2t + 1)

1
lin tc trn on ;1 ta c
4

( 2t + 1)3 4 0, t 1 ;1 .
>
4
( 2t + 1)3

10
1
a = b = ; c =1
9
2
Bi 25. Cho a,b,c l cc s thc dng tha mn iu kin a + b + c =
1.

Do Pmax = f (1) =

Tm gi tr nh nht ca biu thc=


P a3b3c3 ( a bc )( b ca )( c ab ) .
Li gii
Ta chng minh 8a b c ( a bc )( b ca )( c ab ) .
2 2 2

a bc > 0
Khng mt tnh tng qut gi s a b c khi do a, b, c ( 0;1)
b ca > 0

Nu c ab < 0 bt ng thc lun ng.


Nu c ab 0 khi ta chng minh 2ab
Tht vy 2ab

( a bc )( b ca )

( a bc )( b ca ) 4a 2b2 ( a bc )( b ca )

4a 2b 2 ab a 2 c b 2 c + abc 2 4a 2b 2 + c a 2 + b 2 ab c 2 + 1 0

4a 2b 2 + c ( a b ) + ab 2c c 2 1 0 4a 2b 2 + c ( a b ) ab ( c 1) 0
2

639

www.TaiLieuLuyenThi.com
Khm ph t duy K thut gii bt T Bi ton Max Min ng Thnh Nam
2
2
2
ab 4ab ( a + b ) + c ( a b ) 0 ( c ab )( a b ) 0 (lun ng).

Tng t ta c 2bc ( b ca )( c ab ) ;2ca ( c ab )( a bc ) .


Nhn theo v 3 bt ng thc trn ta c ngay iu phi chng minh.
1
ng thc xy ra khi v ch khi a= b= c=
.
3
Khi P a3b3c3 8a 2b 2 c 2 .
3

1
a+b+c
t
ta c P f (t ) =
t 3 8t 2 .
t abc, 0 < t
=
=

3
27

1
Xt hm s f (t=
) t 3 8t 2 vi t 0; ta c
27
1
f '(t=
) 3t 2 16=
t t ( 3t 16 ) < 0, t 0; .
27
1
Do f(t) l hm nghch bin trn 0; .
27
215
1
.
Suy ra P f (t ) f =

19683
27

1
.
3

ng thc xy ra khi v ch khi a= b= c=

215
1
t ti a= b= c=
.
19683
3
Bi 26. Cho a,b,c l cc s thc dng. Tm gi tr ln nht ca biu thc

Vy gi tr nh nht ca P bng

=
P

25 5

( a + 1)(b + 1)( c + 1)
2

36
.
a+b+c

Li gii

)(

)(

S dng bt ng thc: a 2 + 1 b 2 + 1 c 2 + 1

5
( a + b + c + 1)2 .
16

Chng minh xem chng 2.


100
36
.
Khi P

a + b + c +1 a + b + c
100 36
t t = a + b + c khi P f (t ) =
.

t +1 t
100
36
3
Ta c f '(t ) =
+ 2 ; f '(t ) =0 6 ( t + 1) =10t t = .
2
2
( t + 1) t
640

www.TaiLieuLuyenThi.com
Cty TNHH MTV DVVH Khang Vit

3
3
nn f(t) t cc i ti t = .
2
2
1
3
Suy ra P f (t ) f =
.
16 . ng thc xy ra khi v ch khi a= b= c=
2
2

Ta c f(t) i du t dng sang m khi i qua t =

Vy gi tr ln nht ca P bng 16 t ti a= b= c=

1
.
2

Bi 27. Cho x,y,z l di 3 cnh mt tam gic.


Tm gi tr nh nht ca biu thc
P = 1+

24 ( y + z x )
x

24 ( z + x y )

+ 1+

+ 1+

24 ( x + y z )
z

Li gii
t a = x + y z , b = y + z x, c = x + z y khi :
P = 1+

48a
48b
48c
.
+ 1+
+ 1+
b+c
c+a
a+b

Khng mt tnh tng qut gi s c = min {a, b, c} .


S dng bt ng thc Mincopski ta c:
2

1+

48a
48a
48b
48b
+ 1+
22 +
+
=
b+c
c+a
b
+
c
c
+
a

Mt khc:

a
b
4 + 48
+
.
b
+
c
c
+
a

a
b
a+b
.
+
2
b+c
c+a
a + b + 2c
2

a
b 2
3
2
Tht vy ta c:
+
a ( b + c ) + b ( c + a ) ( a + b ) .
c+a
b+c

a (b + c ) + b (c + a )
2

3
2
a + b)
c (a + b)
(

a 2 ( b + c ) + b2 ( c + a )

(a + b)

2
a b ) ( a + b 2c )
(
=

c (a + b)

(a + b)
( a + b )3

a 2 ( b + c ) + b 2 ( c + a ) ( a + b )3 c ( a + b )2
3

Do

Suy ra

4(a + b)
.
=
a + b + 2c

a
b
a+b
.
+
2
b+c
c+a
a + b + 2c

Vy P 2 1 +

48 ( a + b )
a + b + 2c

+ 1+

48c
.
a+b
641

www.TaiLieuLuyenThi.com
Khm ph t duy K thut gii bt T Bi ton Max Min ng Thnh Nam

t t + 1=

1+

Suy ra P 2

48 ( a + b )
48c
, (0 < t 4) 1 +
=
a+b
a + b + 2c

t 2 + 2t + 1176
t 2 + 2t + 24

Ta chng minh 2

t 2 + 2t + 24

+ t + 1.

t 2 + 2t + 1176

t 2 + 2t + 1176

t 2 + 2t + 24

+ t + 1 15 2

t 2 + 2t + 1176
t 2 + 2t + 24

14 t

4 t 2 + 2t + 1176 (14 t ) t 2 + 2t + 24 t (18 t )( t 4 ) 0 (lun ng).


2

ng thc xy ra khi v ch khi t = 4 2c = a + b a = b = c x = y = z .


Vy gi tr nh nht ca P bng 15 t ti x= y= z .
1 3
Bi 28. Cho a,b,c l cc s thc thuc on ; v tho mn iu kin
2 2
a+b+c =
3 . Tm gi tr nh nht ca biu thc

1 1 1
P= 7 + + 6 a 4 + b 4 + c 4 .
a b c
Li gii

1 3
V a, b, c ; a + b c = 3 2c 0 . Tng t b + c a 0, c + a b 0 .
2 2
Do

) (

2 a 2b 2 + b 2 c 2 + a 2 a 2 a 4 + b 4 + c 4 = ( a + b + c )( a + b c )( b + c a )( c + a b ) 0 .

Suy ra 2 a 2b 2 + b 2 c 2 + c 2 a 2 + a 4 + b 4 + c 4 2 a 4 + b 4 + c 4 .
a 4 + b4 + c4

1 2
a + b2 + c2
2

1 1 1
Suy ra P 7 + + 3 a 2 + b 2 + c 2
a b c

3
t a = y + z , b = z + x, c = x + y x, y, z 0 v x + y + z = .
2
Ta c
1
1 1 1
1
1
1
2
1
1
+ +=
+
+
= .( x + y + z )
+
+

a b c y+z z+x x+ y 3
y+z z+x x+ y
=

642

y
z
y
z
2 x
2 x
+
+
+ 3 2 +
+
+

3 y + z z + x x+ y
3 y + z z + x x+ y

www.TaiLieuLuyenThi.com
Cty TNHH MTV DVVH Khang Vit

x
x2
x2
=

y + z x ( y + z ) xy + yz + zx
y
y2
y2
= =
z + x y ( z + x ) xy + yz + zx
z
z2
z2
= =
x + y z ( x + y ) xy + yz + zx

Suy ra

x
y
z
x2 + y 2 + z 2
9
+
+

=
2
y + z z + x x + y xy + yz + zx 4 ( xy + yz + zx )

V a 2 + b 2 + c 2 = 4 x 2 + y 2 + z 2 + xy + yz + zx

2
2
9

= 4 ( x + y + z ) ( xy + yz + zx ) = 4 xy yz zx

2
9
9

Suy ra P 7 2 +
2 12 xy yz zx
3 4 ( xy + yz + zx )
4

14
63
9

+
12 xy yz zx
3 6 ( xy + yz + zx )
4

x + y + z)
(
3
khi
t t = xy + yz + zx, 0 < t
=

3
4

P f (t ) =

14 63
9
+ 12 t .
3 6t
4
2

14 63
9
3
Xt hm s f (t ) = + 12 t vi t 0; ta c
3 6t
4
4
f '(t ) =

3 16t 3 36t 2 + 7
2t

) ; f '(t ) = 0 16t

36t 2 + 7 = 0 t =

Ta c f(t) i du t m sang dng khi i qua t =


t=

3
1
(do 0 < t ).
2
4

1
nn f(t) t cc tiu ti
2

1
133
1
hay P f (t ) f =
.

12
2
2

643

www.TaiLieuLuyenThi.com
Khm ph t duy K thut gii bt T Bi ton Max Min ng Thnh Nam

x + y + z =
2

1
ng thc xy ra khi v ch khi xy + yz + zx = ( x; y; z ) = 0; ;1 hoc
2
2

xyz = 0

1 3
cc hon v suy ra ( a; b; c ) = ;1; hoc cc hon v.
2 2

Vy gi tr nh nht ca P bng

1
3
133
t ti =
hoc cc hon v.
a
,=
b 1,=
c
2
2
12

Bi 29. Cho a,b,c l cc s thc thuc on [ 0;2] .


Tm gi tr ln nht ca biu thc
P= ab(a 2 b 2 ) + bc(b 2 c 2 ) + ca (c 2 a 2 ) .

Li gii
Khng mt tnh tng qut, gi s a = max{a, b, c}
Ta c :
P= ab(a 2 b 2 ) + c(b3 a3 ) + c3 (a b)= ab(a 2 b 2 ) + c(b a )(a 2 + ab + b 2 c 2 )

V a b 0, a c 0 nn c(b a )(b 2 + ab + a 2 c 2 ) 0
Suy ra: P ab(a 2 b 2 ) 2b(4 b 2 ) (do b [ 0;2] )
Mt khc : F = 2b(4 b 2 ) 0 .
Ta c F 2= 4b 2 (2 b 2 )(4 b 2 )= 2. 2b 2 .(4 b 2 ).(4 b 2 )

p dng bt ng thc AM-GM ta c:


3

2b 2 + 4 b 2 + 4 b 2
512
32 3
F 2.
=
0 F
2.

3
27
9

Du bng xy ra khi=
a 2,=
b

2 3
=
, c 0.
3

Vy gi tr ln nht ca biu thc P =


=
a 2,=
b

644

2 3
=
, c 0.
3

32 3
, t c chng hn khi
9

www.TaiLieuLuyenThi.com
Cty TNHH MTV DVVH Khang Vit

Bi 30. Cho a,b,c l cc s thc dng thuc on [1;2] v tha mn iu kin


2
( a + b + c )=

P=

4 ( ab + bc + ca 1) . Tm gi tr nh nht ca biu thc


a+bc
b+ca
c+ ab
2
.
+
+
+2 2
2
c
a
b
a + b + c2 2
Li gii

t x = a + b c, y = b + c a, z = a + c b, ( x, y, z > 0 )
Theo gi thit ta c: xy + yz + zx =
4.
Khi =
P

Ta c:
=

x
+

y+z

z
+
x + y

y
+
z+x

x + y

y
+
z+x

x
y
z
+
+
+ 2

y+z z+x x+ y

Ta c:
=

x
+
2
y+z

4
x2 + y 2 + z 2

xy

yz

( y + z )( x + z )

( y + x )( z + x )

.
( z + y )( x + y )
zx

x
x
y
z
1
y
z
+
+
= ( xy + yz + zx )
+
+

y+z z+x x+ y 4
y+z z+x x+ y

x 2 + y 2 + z 2 xyz x
y
z x2 + y 2 + z 2
.
+
+
+

4
4 y+z z+x x+ y
4

xy

=
( y + z )( x + z )

Tng t:

xy

( xy + zx )( xy + yz )
yz

( y + x )( z + x )

2 xy
xy
.

2 xy + yz + zx xy + yz + zx

yz
,
xy + yz + zx

zx

( z + y )( x + y )

zx
.
xy + yz + zx

Cng theo v ba bt ng thc trn ta c:


xy

( y + z )( x + z )
Do

x
+
y+z

Suy ra: P

t t =

yz

( y + x )( z + x )
y
+
z+x

( z + y )( x + y )

1.

x2 + y 2 + z 2
+2 .
4

x+ y

x2 + y 2 + z 2
+4+
2

zx

4
x + y2 + z2
2

x 2 + y 2 + z 2 , t xy + yz + zx = 2 khi P f (t )=

t2
4
+4+ .
2
t
645

www.TaiLieuLuyenThi.com
Khm ph t duy K thut gii bt T Bi ton Max Min ng Thnh Nam

t2
4
+ 4 + vi t 2 ta c:
2
t

Xt hm s f (t )=
f '(t ) =

2 t +8
2

4
t

; f '(t ) = 0 t 3 = 4 2 t 2 + 8 t = 2 2 .

Ta c f(t) i du t m sang dng khi i qua t = 2 2 nn f(t) t cc tiu ti


t = 2 2 hay P f (t ) f (2 2) =
3 2.

Vy gi tr nh nht ca P bng 3 2 t ti =
a 1,=
b 2,=
c 1 hoc cc hon v.
Bi 31. Cho cc s thc dng a, b, c tha mn iu kin abc = 1 .
1
.
Tm gi tr nh nht ca biu thc P = a 2b + b 2 c + c 2 a +
6 3
a + b3 + c 3
Li gii
a
b
c
t x =
a 2b =
a.ab = ; y =
b2c =
b.bc = ; z =
c2a =
c.ca = xyz =
1.
c
a
b
1
V abc = 1 nn ta bin i c P thnh P = x + y + z +
2
6
xy + yz 2 + zx 2
Ta c x 2 y + y 2 z + z 2 x 3 3 x 2 y. y 2 z.z 2 x =
3.
S dng bt ng thc AM GM ta c
1
6

xy 2 + yz 2 + zx 2

=
6

( xy

3.6 x 2 y + y 2 z + z 2 x

+ yz 2 + zx 2

)( x
3

( xy

+ yz + zx
2

y + y 2 z + z 2 x .3xyz

)( x

y + y 2 z + z 2 x .3xyz

S dng bt ng thc AM-GM ta c:

( xy

+ yz + zx
2

)(

x 2 y + y 2 x + y 2 z + z 2 y + z 2 x + x 2 z + 3xyz
x y + y z + z x .3xyz

( x + y + z )( xy + yz + zx) ( x + y + z )
=

3
27 2

Suy ra P x + y + z +

3.3 3
( x + y + z )3/2

t t =x + y + z ,(t 3) P f (t ) =t +
646

3.3 3
t 3/2

www.TaiLieuLuyenThi.com
Cty TNHH MTV DVVH Khang Vit

1
3.3 3
Xt hm s f (t ) =
t + 3/2 ; t 3 ta thu c: P 3 + 6 .
3
t
ng thc xy ra khi v ch khi a= b= c= 1 .
1
Vy gi tr nh nht ca P bng 3 + 6 t ti a= b= c= 1 .
3

Bi 32. Cho a,b,c l cc s thc khng m tho mn iu kin a 2 + b 2 + c 2 =


2.
Tm gi tr ln nht ca biu thc
a
b
a + b 2ab
.
P= 2
+ 2

a + b + c b + c + a ( a + b + c )2
Li gii

S dng bt ng thc C S ta c a 2 + b + c (1 + b + c ) ( a + b + c )

a
a2 + b + c

Tng t ta c
T suy ra

a (1 + b + c )

( a + b + c )2
b

.
b (1 + c + a )

( a + b + c )2
a ( b + c ) + b ( c + a ) + 2ab
P
1.
( a + b + c )2
b +c+a
2

Bi v 2 + 2bc = a 2 + ( b + c ) 2a ( b + c ) a ( b + c ) 1 + bc
2

2 + 2ca = b 2 + ( c + a ) 2b ( c + a ) b ( c + a ) 1 + ca
2

a ( b + c ) + b ( c + a ) + 2ab 2 + c ( a + b ) + 2ab
= a 2 + b 2 + c 2 + c ( a + b ) + 2ab ( a + b + c )

Vy gi tr ln nht ca P bng 1 t ti a= b= 1, c= 0 .
Bi tp tng t
Cho a,b,c l cc s thc dng c tng bng 3. Chng minh
1
1
1
+ 2
+ 2
1.
2
a +b+c b +c+a c +a+b
Bi 33. Cho a,b,c l cc s thc dng tho mn iu kin

( a + 5b + 3c )

1 1
1 39
+
+ =.
a
4
b
3
c 4

Tm gi tr nh nht ca biu thc P =

2 ( a + 3c )( a + 3c 4b ) 7c 2 + 4c a 2 + 7c 2
2ac

647

www.TaiLieuLuyenThi.com
Khm ph t duy K thut gii bt T Bi ton Max Min ng Thnh Nam

Li gii
Theo gi thit ta c
39
1
1
1
1 1
1 1
1 1
=( a + 4b + 3c ) +
+ + b +
+ 9 + b +
+
4
a 4b 3c
a 4b 3c
a 4b 3c .

3ac 2b ( a + 3c )

1
1 1
Ch . AM GM ta c ( a + 4b + 3c ) +
+ 9.
a 4b 3c
Khi

2 ( a + 3c ) 8b ( a + 3c ) 7c 2 + 4c a 2 + 7c 2
2

P=

2ac

2 ( a + 3c ) 12ac 7c 2 + 4c a 2 + 7c 2
2

2ac

2a 2 + 11c 2 + 4c a 2 + 7c 2
=
2ac

2 x 2 + 11 + 4 x 2 + 7
a
=
,x
c
2x

Xt hm s f ( x) =

( 2x
f '( x) =

11

2 x 2 + 11 + 4 x 2 + 7
vi x dng ta c
2x

x 2 + 7 28

2 x2 x2 + 7

; f '( x) = 0 x = 3 .

15
.
Ta c f(x) i du t m sang dng khi i qua x=3 nn f ( x) f (3) =
2
Du bng xy ra khi v ch khi =
a 4=
b 3c .
15
.
Vy gi tr nh nht ca P bng
2
Bi tp tng t
1) Cho a,b,c l cc s thc dng tho mn iu kin

( a + 3b + c )

1 1 1 21
+
+ =.
a 2b c 2

Tm gi tr nh nht ca biu thc P =

a 2 + ( a + c )( a + c 2b ) + 2a 2 + 2c 2

S: Pmin
= 5;=
a 2=
b c.
2) Cho a,b,c l cc s thc dng tho mn iu kin

( 3a + 2b + c )

1 2 3
+ + =
30 .
a b c

648

ac

www.TaiLieuLuyenThi.com
Cty TNHH MTV DVVH Khang Vit

Tm gi tr nh nht ca biu thc P =

b + 2c 7 72a 2 + c 2
.
a

S: Pmax =
55;2a =
3b =
6c .
Bi 34. Cho x, y, z l cc s thc khng m tha mn iu kin
x2 + y 2 + 2 x + 2 y + 1 + z =
3.

Tm gi tr ln nht ca biu thc P = x 4 + y 4 + z 4 .


Li gii
Trc ht ta chng minh:

x2 + y 2 + 2 x + 2 y + 1 1 + x2 + y 2 .

Tht vy bt ng thc tng ng vi:


x2 + y 2 + 2 x + 2 y + 1 1 + x2 + y 2 + 2 x2 + y 2 .
x + y x 2 + y 2 xy 0 , bt ng thc lun ng.

Du bng xy ra khi v ch khi xy = 0 .


Suy ra 3 z 1 + x 2 + y 2 x 2 + y 2 2 z x 2 + y 2 ( 2 z ) v 0 z 2 .
2

P=
x 4 + y 4 + z 4 =x 2 + y 2

+ z 4 2 x2 y 2 x2 + y 2

+ z4 (2 z ) + z4 .
4

Xt hm s f ( z ) =( 2 z ) + z 4 , ta c f '( z ) = 4 z 3 ( 2 z )
4

) = 0 z =1.

Ta c=
f (0) 16,
=
f (1) 2,=
f (2) 16 .
Suy ra P f ( z ) 16 .
xy = 0

x = 0
z = 2

Du bng xy ra khi v ch khi


0 hoc
y =
=
z
0

z = 2

2
2
x
y
2
x
2
y
1
z
3
+
+
+
+
+
=

cc hon v.

Vy P ln nht bng 16 khi v ch khi ( x, y, z ) = ( 0,0, 2 ) v cc hon v.


Bi 35. Cho a,b,c l cc s thc tha mn iu kin abc = 1 .
Tm gi tr nh nht ca biu thc
5 3
1
1
.
. 2
+ 2
2
3
(a a + 1)(b b + 1) c c + 1

=
P

Li gii
Ta chng minh vi mi s thc a v b ta lun c

)(

) (

3 1 a + a 2 1 b + b 2 2 1 ab + a 2b 2 .
649

www.TaiLieuLuyenThi.com
Khm ph t duy K thut gii bt T Bi ton Max Min ng Thnh Nam

Chng minh. Vit li bt ng thc di dng:

(a

3a + 3 b 2 3a 2 5a + 3 b + 3a 2 3a + 1 0 .

V tri l tam thc bc hai ca b c a 2 3a + 3 > 0, a v;

b =
3a 2 5a + 3

)(

) (

4 a 2 3a + 3 3a 2 3a + 1 =
a 2 3a + 1 0, a .

Do v tri lun khng m v bi ton c chng minh.

3
a =
2

2
3
a 3a + 1 =

0
b =

ng thc xy ra khi v ch khi


3a 2 5a + 3

b
=

a = 3 +
2 a 2 3a + 3

b = 3 +
2

) 23 ( a b

)(

Khi a 2 a + 1 b 2 b + 1
Suy ra P

1
c2 c + 1

Xt hm s f (c) =
f '(c) =

+ 2k .

c2 c + 1

c +1

2 1 1
c2 c + 1
1 2.
.
2 +=
3c c
3c 2

; f '(c) = 0 3kc 2 + ( 6k 2 ) c + 1= 0 .

7+3 5
2
0

650

+
+

5 3
2

5 1

vi c > 0 ta c

7+3 5
3 5
hoc c =
.
c=
2
2
Bng bin thin:
c
3 5
0
2
f(c)
0
+

f(c)

.
5

5 3
3kc 2 + 1
vi k =
.
=
2
2
6
c c +1 c c +1
2.
3c 2

3kc 2 + ( 6k 2 ) c + 1

(c

ab +=
1

3kc 2 + 1

2 2

1 5
3

www.TaiLieuLuyenThi.com
Cty TNHH MTV DVVH Khang Vit

7 + 3 5 1 5
Da vo bng bin thin suy ra P f (c) f
= .
2
3

ng thc xy ra khi v ch khi a= b=

3 5
7+3 5
.
, c=
2
2

3 5
7+3 5
1 5
t ti a= b=
.
, c=
2
2
3
Bi 36. Cho x,y,z l cc s thc dng tha mn iu kin

Vy gi tr nh nht ca P bng

2.
( x y )2 + ( y z )2 + ( z x )2 =

Tm gi tr nh nht ca biu thc


1 1 1 2 ( x + y + z 1)
.
P = ( x + y + z ) + +
3 xyz
x y z
Li gii

Khng mt tnh tng qut gi s x = max { x, y, z} . t y + z = 2t x t .


Theo gi thit ta c x 2 + y 2 + z 2 xy yz zx =
1.
( x + y + z ) =1 + 3 ( xy + yz + zx ) (1) .
2

Khi P = ( x + y + z ) .

xy + yz + zx 2 ( x + y + z 1)

3 xyz
xyz

( x + y + z ) ( ( x + y + z )2 1) 2 ( x + y + z 1)

3 xyz

( x + y + z )3 3 ( x + y + z ) + 2

3 xyz

V (1) x 2 + ( y + z ) x ( y + z ) 3 yz =
1.
2

3 yz = x 2 + 4t 2 2tx 1 .

Suy ra P

3
x + 2t ) 3 ( x + 2t ) + 2
(=

x x + 4t 2tx 1
2

x3 + 6tx 2 + 12t 2 3 x + 8t 3 6t + 2

x x 2tx + 4t 2 1
2

Mt khc ta c:
2 = ( x y) + ( x z) + ( y z) ( x y) + ( x z)
2

1
( 2 x y z )2 .
2

( 2x y z ) 4 2x y z 2 x t + 1 .
2

Ta chng minh P 9 .
651

www.TaiLieuLuyenThi.com
Khm ph t duy K thut gii bt T Bi ton Max Min ng Thnh Nam

Tht vy bt ng thc tng ng vi

x3 + 6tx 2 + 12t 2 3 x + 8t 3 6t + 2

x x 2 2tx + 4t 2 1

x3 + 6tx 2 + 12t 2 3 x + 8t 3 6t + 2 9 x x 2 2tx + 4t 2 1

8 x3 24 x 2t + 24 xt 2 8t 3 6 ( x t ) 2 0 .

4 ( x t ) 3 ( x t ) 1 0 ( x t 1) 2 ( x t ) + 1 0 (lun ng v x t + 1 ).
2

y = z

x = 2
y+z

.
ng thc xy ra khi v ch khi x = t + 1 =
+1

2
y= z= 1

( x y )2 + ( y z )2 + ( z x )2 =
2

Vy gi tr nh nht ca P bng 9 t ti ( x; y; z ) = ( 2;1;1) hoc cc hon v.


Cch 2: Ta chng minh P 9 .
Vit li bt ng thc di dng:
1 1 1 2 ( x + y + z 1)
+ +
9
3 xyz
x y z

( x + y + z )

3 ( x + y + z )( xy + yz + z ) + 2 27 xyz 2 ( x + y + z ) 0

3 ( x + y + z )( xy + yz + zx ) 9 xyz + 2 2 ( x + y + z ) 0
3 x 2 ( y + z ) + y 2 ( z + x ) + z 2 ( x + y ) 6 xyz + 2 2 ( x + y + z ) 0

3 x 2 ( y + z ) + y 2 ( z + x ) + z 2 ( x + y ) 6 xyz +

2
2
2
+2 ( x + y + z ) ( x y ) + ( y z ) + ( z x ) 0

( 2 z x y )( x y ) + ( 2 x z y )( z y ) + ( 2 y x z )( x z ) + 2 0
2

0
a + b + c =
t a = x y; b = y z; c = z x 2
.
2
2
2
a + b + c =

Ta cn chng minh ( c b ) a 2 + ( b a ) c 2 + ( a c ) b 2 + 2 0 .
M = ( a b )( b c )( a c ) 2 .

Ta c M 2 =
( a b) (b c ) (c a ) .
2

Trong 3 s a,b,c lun c hai s cng du khng mt tnh tng qut gi s hai s
l a v b, khi ab 0 .
652

www.TaiLieuLuyenThi.com
Cty TNHH MTV DVVH Khang Vit

V iu kin tr thnh: a 2 + b 2 + ( a + b ) =
2
2

a 2 + ab + b 2 =
1.
Khi
M2 =
( a b ) ( 2a + b ) ( 2b + a ) =(1 3ab )( 2 + 3ab )
2

2
3

2 + 3ab 2 + 3ab
1 3ab + 2 + 3ab
=
4.(1 3ab ) .
.
4
4
=
2
2
3

Suy ra M 2 . ng thc xy ra khi v ch khi

=
y 0
a 0
x=
ab =
0

=
z 0.
b 0
y=
( x; y; z ) =
(1;1;2 ) ; ( 2;1;1)

Vy gi tr nh nht ca P bng 9 t ti ( x; y; z ) = ( 2;1;1) hoc cc hon v.

653

www.TaiLieuLuyenThi.com
Khm ph t duy K thut gii bt T Bi ton Max Min ng Thnh Nam

Chng 4:
S PHNG PHP CHNG MINH BT NG THC KHC
CH 1: K THUT LNG GIC HA
Trong ch ny ti cp n mt s php lng gic ho c bn v mt s
bt ng thc trong tam gic v ng dng. Mt ch l cc bi ton trnh by trong
ch c gii bng phng php lng gic ho c th c gii bng phng
php i s thng thng.
A. NI DUNG PHNG PHP
1. Cc php lng gic ho c bn

Nu x R th t x = Rcos , [ 0; ] ; hoc x = Rsin , , .
2 2

Nu x R th t x =

R
cos

[ 0, c ) ,3 .
2

x= a + R cos
2
Nu ( x a ) + ( y b=
, ( = 2 ) .
)2 R 2 , ( > 0) th t
y= b + R sin
2
2
x= + aR cos
x y
2
+
Nu
, ( = 2 ) .

=R , a, b > 0 th t
a b
y= + bR sin

Nu trong bi ton xut hin ( ax ) + b 2 , ( a, b > 0 ) =


t x
2

b

tg , , .
a
2 2

V d 1. Cho bn s thc x, y, u, v tho mn iu kin x 2 + y 2 = u 2 + v 2 = 1 .


Chng minh rng 2 u ( x y ) + v ( x + y ) 2 .
Li gii
t x cos
=
=
a, y sin
=
a; u cos
=
b, v sin b .
Ta c u ( x y ) + v ( x + y ) = cosb ( cosa sina ) + sinb ( cosa + sina )

= cos ( a b ) + sin ( b a )
=

2 cos b a 2; 2
4

Bt ng thc c chng minh.

654

www.TaiLieuLuyenThi.com
Cty TNHH MTV DVVH Khang Vit

V d 2. Chng minh rng vi mi s thc a,b thuc on [-1;1] ta c

(1 b )(1 a ) ) 2 .

a 1 b 2 + b 1 a 2 + 3 ab

Li gii
a = cos
Do a 1, b 1 nn ta t:
b = cos

Khi : a 1 b 2 + b 1 a 2 + 3 ab

( , [0, ])

(1 b )(1 a ) )
2

= cos .sin + cos .sin + 3 ( cos .cos sin .sin )

= sin( + ) + 3.cos( + =
) 2 cos( + )
6
a 1 b 2 + b 1 a 2 + 3 ab

(1 b )(1 a ) 2
2

Bt ng thc c chng minh.


Bi tp tng t

1) Chng minh rng vi mi s thc x,y thuc on [ 2; 2] ta c

x 4 y 2 + y 4 x2 4 .
2) Cho a , b 1 . Chng minh rng : a b 1 + b a 1 ab .
V d 3. Cho a,b l hai s thc tho mn iu kin ab 0 .
Chng minh rng 2 2 2

a 2 (a 4b) 2
a 2 + 4b 2
Li gii

2 2 2.


t a 2btg , , khi
=
2 2
a 2 (a 4b) 2 tg 2 (tg 2) 2
=
= 4(tg 1).cos 2
2
2
2
a + 4b
1 + tg
= 2sin 2 2(1 + cos 2=
) 2(sin 2 cos 2 ) 2

= 2 2 sin(2 ) 2 2 2 2, 2 2 2
2
Bt ng thc c chng minh.

655

www.TaiLieuLuyenThi.com
Khm ph t duy K thut gii bt T Bi ton Max Min ng Thnh Nam

Bi tp tng t
Cho x,y l cc s thc chng minh rng

1 ( x y )(1 + xy ) 1

.
2 (1 + x 2 )(1 + y 2 ) 2

V d 4. Cho a,b,x,y,z l cc s thc dng c a + b =


1.
Chng minh rng ( x + y )

( y + z)

64abxy 2 z ( ax + y + bz ) .
2

Li gii
Cc ng thc v bt ng thc trong tam gic
sin A sin B sin C 4 cos

A
B
C
cos cos
2
2
2

sin 2 A sin 2 B sin 2 C 2 1 cos A cos B cos C


sin 2 A sin 2 B sin 2C 4sin A sin B sinC
tan A tan B tan C tan A tan B tan C

A
B
B
C
C
A
tan tan tan tan tan
2
2
2
2
2
2
1 cot A cot B cot B cot C cot C cot A
1 tan

1 cos 2 A cos 2 B cos 2 B 2 cos A cos B cos C

Mt s php lng gic ho a v chng minh bt ng thc trong tam gic


+ Nu a,b,c l cc s thc dng tha mn a + b + c =
abc khi tn ti ba gc
mt tam gic sao=
cho a tan
=
A, b tan
=
B, c tan C .
+ Nu a,b,c l cc s thc dng tha mn ab + bc + ca =
1 khi tn ti ba gc

A
2

B
2

mt tam gic sao=


cho a tan
=
, b tan
=
, c tan

C
.
2

+ Nu a,b,c l cc s thc dng tha mn a 2 + b 2 + c 2 + 2abc =


1 khi tn ti
ba gc nhn ca mt tam gic sao=
cho a cos
=
A, b cos
=
B, c cos C .
ng thc quen thuc:

(a

+ 1)( b 2 + 1)( c 2 + 1)=

( ab + bc + ca 1) + ( a + b + c abc )
2

Bi ton 1. Chng minh rng vi mi tam gic ABC ta c

cos A + cos B + cos C

3
.
2

Chng minh.

A+ B
A B
A+ B
cos
2 cos 2
+1.
2
2
2
A+ B
A B
Do A,B,C l cc gc ca tam gic nn 1 > cos
> 0;0 cos
1.
2
2

=
VT 2 cos

656

www.TaiLieuLuyenThi.com
Cty TNHH MTV DVVH Khang Vit

A+ B
A+ B
3
A+ B 1 3

Do , VT 2 cos
+ 2 cos
+ 1=
2 cos
.
2
2
2
2
2 2

Du bng xy ra khi v ch khi tam gic u.


Tng qut vi x,y,z l cc s thc dng v A,B,C l cc gc ca mt tam gic ta
c

cos A cos B cosC x 2 + y 2 + z 2


.
+
+

x
y
z
2 xyz

Chng minh.
Bt ng thc tng ng vi:

2 yz cos A + 2 zx cos B + 2 xy cos C x 2 + y 2 + z 2

x 2 + y 2 + z 2 + 2 yz cos ( B + C ) 2 zx cos B 2 xy cos C 0


x 2 + y 2 + z 2 + 2 yz cos B cos C 2 yz sin B sin C 2 zx cos B 2 xy cos C 0
( x z cos B y cos C ) + ( z sin B y sinC ) 0 .
2

Bt ng thc c chng minh.

sin A
x

ng thc xy ra khi v ch khi =

sin B sin C
.
=
y
z

V d 1. Cho a,b,c l di ba cnh mt tam gic v x,y,z l cc s thc tho mn

a
cy + bz =

b.
az + cx =
bx + ay =
c

Chng minh rng x + y + z

3
.
2
Li gii

D tnh c

=
x

b2 + c2 a 2
c2 + a 2 b2
a 2 + b2 c2
.
=
;y =
;z
2bc
2ca
2ab

3
.
2
Bt ng thc lun ng ng thc xy ra khi v ch khi

Bi ton a v chng minh cos A + cos B + cos C

x= y= z=

1
; a= b= c .
2

657

www.TaiLieuLuyenThi.com
Khm ph t duy K thut gii bt T Bi ton Max Min ng Thnh Nam

Cch 2: T gi thit ta c:

c
b
y+ x=
1
a
a
a
c
z+ x=
1
b
b
b
a
x+ y=
1
c
c

b c
a c
a b
Cng theo v ba phng trnh trn ta c: x( + ) + y ( + ) + z ( + ) =
3.
c b
c a
b a
S dng AM-GM ta c ngay pcm.
V d 2. Cho a, b, c l cc s thc khc 0 v c > b .

Tm gi tr ln nht ca biu thc P =

bc + a 2 b a 2 + c 2 + c a 2 + b 2
a 2 + b2 a 2 + c2

Li gii
P
=
=

bc + a

(a 2 + c 2 )(a 2 + b 2 )
bc + a 2
(a 2 + c 2 )(a 2 + b 2 )

+
+

b
a 2 + b2

a2 + c2

b(c b)

(c b)

a 2 + b2

c (c b )

(c b)

a2 + c2

V 3 s a,b,c khc O nn trong h trc ta Oxy chn 3 im


A(0; a ); B (b;0); C (c;0), b < c l ba nh mt tam gic.

Ta c AB =
(b; a ); AC =
(c; a ); BC =
(c b;0) .
Xt tam gic ABC c

AB, AC )
cos A cos(
=
=

BA; BC )
cos B cos(
=
=

CA, CB)
cos C cos(
=
=

bc + a 2
(a 2 + b 2 )(a 2 + c 2 )
b ( c b )
| c b | a 2 + b2
c (c b )

| c b | a2 + c2
3
P cos A + cos B + cos C .
=
2

658

;
;

www.TaiLieuLuyenThi.com
Cty TNHH MTV DVVH Khang Vit

Du bng xy ra khi v ch khi tam gic ABC u.


3
khi c > 0; b =
c; a =
3c .
2
V d 3. Cho a,b,c l cc s thc dng tha mn iu kin a + b + c =
abc .

Vy gi tr ln nht ca P bng

Chng minh rng

1
2 a +1
2

1
3 b +1
2

1
4 c +1
2

29
.
48

Li gii
Theo gi thit tn ti 3 gc nhn ca mt tam gic tha mn
=
a tan
=
A, b tan
=
B, c tan C .
Khi :
1
1
cos A
= =
2
2 a 2 + 1 2 tan 2 A + 1
1
1
cos B
= =
3
3 b 2 + 1 3 tan 2 B + 1
1
1
cos C
= =
2
2
4
4 c + 1 4 tan C + 1

cos A cos B cos C 29


+
+

2
3
4
48
Bt ng thc ny lun ng bi v

Ta cn chng minh:

cos A cos B cos C 22 + 32 + 42 29


+
+

=
2
3
4
2.2.3.4
48
sin A sin B sin C
ng thc xy ra khi v ch khi: = =
2
3
4

tan A

tan B

tanC

2 tan 2 A + 1 3 tan 2 B + 1 4 tan 2 C + 1


a
b
c

=
=
2
2
2 a + 1 3 b + 1 4 c2 + 1

V d 4. Cho a,b,c l cc s thc dng tha mn iu kin

1
1 1
+
+ =
6
3a 2b c

3
a
b
c
.

+
+
2
a + 4bc
b + 9ca
c + 36ab
Li gii
Phn tch biu thc v phi trc

Chng minh rng

659

www.TaiLieuLuyenThi.com
Khm ph t duy K thut gii bt T Bi ton Max Min ng Thnh Nam

1
1
1
+
+
4bc
9ca
36ab
1+
1+
1+
a
b
c
1
1
1
=
+
+
2
2
2
bc
ca
ab
1+ 2
1+ 3
1+ 6

a
b
c

VP =

t =
x 2

bc
ca
ab
,=
y 3
,=
z 6
xy= 6c; yz= 18a; zx
= 12b .
a
b
c

Theo gi thit ta c:
1
1 1
1
1
1
+
+ =6
+
+
=1
3a 2b c
18a 12b 6c
1
1
1

+ +
=1 x + y + z = xyz
yz zx xy

Cho ta thy du hiu ca lng gic ho vy tn ti ba gc mt tam gic sao cho


=
x tan
=
A, y tan
=
B, z tan C .
Ta cn chng minh
1
1 + tan 2 A

1
1 + tan 2 B

1
1 + tan 2 C

3
2

3
2
Bt ng thc cui ng v ta c iu phi chng minh.
cos A + cos B + cos C

ng thc xy ra khi v ch khi 3

bc
= 3
a

bc = 4 a
a = 1 / 6

ca
1

= 3 ca = b b = 1 / 4 .
b
3

c = 1 / 2

ab
= 3
ab = 12 c

Bi tp tng t
Cho a,b,c l cc s thc dng tho mn iu kin
Chng minh rng

660

1 1
1
+
+ =
6.
a 2b 3c

a
b
c
1
.
.
.

a + 36bc b + 9ca c + 4ab 27

www.TaiLieuLuyenThi.com
Cty TNHH MTV DVVH Khang Vit

Bi ton 2. Chng minh rng vi mi tam gic ABC ta c


cos A + cos B + sin C

3 3
.
2

Chng minh.
A+ B
A B
cos
+ sin C
2
2
A+ B
C
C
C 2sin 1 + cos
2cos
+ sin=
2
2
2

cos A +=
cos B + sin C 2cos

C
C
C
C

2 1 cos 2 1 + cos =
2 1 cos 1 + cos
=
2
2
2
2

Ch s dng bt ng thc AM GM ta c


C
C
3 3 1 cos + 3 1 + cos
4
C
C
2
2 3

3 1 cos 1 + cos
=

2
2
4
2

C
C
3 3

2 1 cos 1 + cos
2
2
2

Bt ng thc c chng minh.


2
.
6
3
V d 1. Cho a,b,c l cc s thc dng c tng bng 1. Chng minh rng

ng thc xy ra khi v ch khi A= B=

, C=

a
b
abc
3 3
.
+
+
1+
a + bc b + ca c + ab
4
Li gii
Gi P l biu thc v tri ca bt ng thc.

ab
c .
+
+
Ta c P =
bc
ca
ab
1+
1+
1+
a
b
c
Ch theo gi thit ta c
1

a=
+b+c

ab ca
ab bc
bc ca
.
+
.
+ =
.
1.
c
b
c
a
a
b

661

www.TaiLieuLuyenThi.com
Khm ph t duy K thut gii bt T Bi ton Max Min ng Thnh Nam

Suy ra tn ti 3 gc mt tam gic sao cho


bc
A ca
B ab
C
= tan
=
,
tan
=
,
tan .
a
2
b
2
c
2

C
2

1
+
+
A
C
2
2 B
1 + tan
1 + tan
1 + tan 2
2
2
2
A
B
1
= cos 2 + cos 2 + sin C
2
2 2
1
3 2
= 1 + ( cos A + cos B + sin C ) 1 +
2
4
Bt ng thc c chng minh. ng thc xy ra khi v ch khi

Khi P =

tan

a=
b=
2 3 3, c =
74 3.

Bng cch chng minh tng t cc bt ng thc trn ta chng minh c


Chng minh rng vi mi tam gic ABC ta c
3 3
;
2
1
A
B
C
cos A.cos B.cos C sin .sin .sin
8
2
2
2
3
cos 2 A + cos 2 B + cos 2 C ;
4

sin A + sin B + sin C

3
sin A + sin B cos C ;
2
5
3 cos A sin B + 3 sin C ;
2
3
sin 2 A + sin 2 B + cos 2C ;
2
5
2
Bi ton 3. Chng minh rng vi mi tam gic ABC ta c
cos 2 A + 3 ( cos 2 B + cos 2C )

cos 2 A + cos 2 B + cos 2 C + 2cos A.cos B.cos C =


1.

Chng minh.
ng thc cho tng ng vi:

662

www.TaiLieuLuyenThi.com
Cty TNHH MTV DVVH Khang Vit

cos 2 B + cos 2 C + 2cos A.cos B.cos C =


sin 2 A
( cos B + cos C ) 2cos B.cos C + 2cos A.cos B.cos C =
sin 2 A
2

B+C
B C
A
A
A
cos 2
4cos B cos C sin 2 =
4sin 2 cos 2
2
2
2
2
2
B

C
A
cos 2
cos B cos C =
cos 2
2
2
1 + cos( B C )
1 + cos A

cos B.cos C =
2
2
cos( B C ) 2cos B.cos C = cos A
cos( B C ) + cos( B + C ) 2cos B.cos C =
0
4cos 2

ng thc cui ng v ta c pcm.


Nh vy vi cc s thc dng x,y,z tho mn iu kin
x 2 + y 2 + z 2 + 2 xyz =
1.

Khi tn ti ba gc nhn trong mt tam gic sao cho


=
x cos
=
A; y cos
=
B; z cos C .
Tht vy ta lun c

cos 2 A + cos 2 B + cos 2 C + 2cos A.cos B.cos C =


1
A+ B +C
A+ B C
B+C A
C + A B
cos
=
.cos
.cos
.cos
0
2
2
2
2

Vi A, B, C 0; th A + B + C =
.
2
V d 1. Cho a,b,c l cc s thc dng tho mn iu kin

a +1 b +1 c +1
3
Chng minh rng ab + bc + ca .
2
Li gii
Gi thit bi ton tng ng vi:
2

2.
=

a 2 b 2 + b 2 c 2 + c 2 a 2 + 2a 2 b 2 c 2 =
1 ab, bc, ca ( 0;1) .

Do tn ti ba gc nhn trong mt tam gic tho mn:


=
ab cos
=
A, bc cos
=
B, ca cos C .
Bt ng thc tr thnh: cos A + cos B + cos C

3
.
2

Bt ng thc cui lun ng v ta c pcm.


663

www.TaiLieuLuyenThi.com
Khm ph t duy K thut gii bt T Bi ton Max Min ng Thnh Nam

1
.
2

ng thc xy ra khi v ch khi a= b= c=

V d 2. Cho a,b,c l cc s thc dng tho mn iu kin a + b + c + 1 =


4abc .
4bc 1
4ca 1 1
.
+
+
bc
ca
ab
Li gii
1
1
1
1
Gi thit bi ton tng ng vi
+
+
+ 2.
=
1.
4bc 4ca 4ab
8abc
Do tn ti ba gc nhn trong mt tam gic tho mn:

Tm gi tr ln nht ca biu thc P =

1
1
1
= 2cos
=
A;
2cos
=
B;
2cos C .
bc
ca
ab
Khi

P 4cos 2 A
=

1
2

cos A

1 + 4cos 2 B

cos B

1 + 4cos 2 C

= 2sin 2 A + 2sin 2 B 4sin C + 4


2

= 4sin( A + B) cos( A B) 4sin 2 C + 4


4sin C 4sin 2 C + 4 = ( 2sin C 1) + 5 5
2

3
C = 6
a= b=
ng thc xy ra khi v ch khi

3 .
A= B= 5 =
c 2 3 + 3

12

3
, c= 2 3 + 3 .
3
V d 3. Cho a,b,c l cc s thc dng tho mn iu kin abc = a + b + c + 2 .

Vy gi tr ln nht ca P bng 5 t ti a= b=

Chng minh rng 2

ab + bc + ca 4 + abc .

Li gii
Gi thit bi ton tng ng vi:
1
1
1
1
+ +
+ 2.
=
1.
bc ca ab
abc
Do tn ti ba gc nhn mt tam gic sao cho
1
1
1
= cos
=
A,
cos
=
B,
cos C .
bc
ca
ab
1
1
1
1
Ta cn chng minh 2
+
+
4+
cos A.cos B.cos C
cos A cos B cos C
664

www.TaiLieuLuyenThi.com
Cty TNHH MTV DVVH Khang Vit

2 ( cos A.cos B + cos B.cos C + cos C.cos A ) 4cos A.cos B.cos C + 1 .

chng minh bt ng thc cui ta chng minh nh sau


1
2cos
sin 2 A.cot A.sin 2 B.cot B ( sin 2 A.cot B + sin 2 B.cot A )
A.cos B
=
2
1
2cos B.cos C ( sin 2 B.cot C + sin 2C.cot B )
2
1
2cos C.cos A ( sin 2C.cot A + sin 2 A.cot C )
2
Cng li th v v khai trin v phi ta c ngay iu phi chng minh.
ng thc xy ra khi v ch khi A = B = C a = b = c = 2 .
Bi tp tng t
Cho a,b,c l cc s thc dng tho mn iu kin ab + bc + ca + abc =
4.
Tm gi tr nh nht ca biu thc P = ab + bc + ca + ab + bc + ca .
V d 4. Cho a,b,c,x,y,z l cc s thc dng tho mn iu kin
ax 2 + by 2 + cz 2 + xyz =
4abc .

Chng minh rng a + b + c x + y + z .


Li gii
x2
y2
z2
xyz
+
+
+ 2.
=
1.
4bc 4ca 4ab
8abc
Do tn ti ba gc mt tam gic sao cho:

Gi thit cho tng ng vi:

x
y
z
= cos
=
A;
cos
=
B;
cos C .
2 bc
2 ca
2 ab

Ta cn chng minh a + b + c 2 bc cos A + 2 ca cos B + 2 ab cos C .


Bt ng thc cui lun ng v ta c pcm.
B. BI TP CHN LC
Bi 1. Cho x,y,z l cc s thc khng m tha mn iu kin xz + yz + 1 =xy .
Tm gi tr ln nht ca biu thc P =

2x
x2 + 1

2y
y2 + 1

z2 1
z2 + 1

Li gii
TH1: Nu xy = 1 z = 0 P =

2 xy ( x + y ) + 2 x + 2 y
x y + x + y +1
2 2

1 =

4x + 4 y

( x + y )2

1 1.

xy 1
TH2: Nu xy > 1 z =
.
x+ y
665

www.TaiLieuLuyenThi.com
Khm ph t duy K thut gii bt T Bi ton Max Min ng Thnh Nam

xy 1

t x =
tan a, y =
tan b, z = =
cot ( a + b ) vi a, b 0; , a + b > .
2
x+ y
2
Khi : P =sin 2a + sin 2b + cos(2a + 2 b) =1 2sin 2 (a + b) + 2sin(a + b)cos(a b)
2

1
3 3

1 2sin 2 (a + b) + 2sin(a + b) =
2 sin(a + b) + .
2
2 2

a = b

1
5

a =b =
sin(a + b) =
2
12
ng thc xy ra khi v ch khi


a, b 0; , a + b >
2
2

5
5
x =y =tan
=
cot
=3
2 + 3, z =
12
6
3
Vy gi tr ln nht ca P bng t ti x =
y=
2 + 3, z =
3
2
Bi 2. Cho a,b,c l cc s thc dng tha mn iu kin ab + bc + ca =
1

Tm gi tr ln nht ca biu thc P =

a
1+ a

b
1+ b

3c
1 + c2

Li gii
Theo gi thit tn ti ba gc ca mt tam gic tha mn
A
B
C
=
a tan
=
, b tan
=
, c tan .
2
2
2
Khi :
A
tan
a
A
A 1
b
1
c
C
2
=
=
sin=
cos
sinA v
=
=
sin B;
sin .
2
2
A
2
2 2
2
2
1+ a
1+ b
1 + c2
1 + tan 2
2
T suy ra
1
C
A+ B
A B
C
P=
cos
+ 3sin
( sin A + sin B ) + 3sin = sin
2
2
2
2
2

= cos

666

C
A B
C
C
C
cos
+ 3sin cos + 3sin
2
2
2
2
2

(1 + 9 ) cos 2

C
2C
+ sin
=

2
2

10

www.TaiLieuLuyenThi.com
Cty TNHH MTV DVVH Khang Vit

A B

cos 2 = 1

ng thc xy ra khi v ch khi


C
sin

C
2
cos =
2
3

a = b = 10 3, c = 3 .

Vy gi tr ln nht ca P bng 10 t ti a =
b = 10 3, c =
3.
Cch 2: Ta x l bng i s nh sau
S dng gi thit ab + bc + ca =
1 ta c:
a

b
a
b
=
+
+
2
2
1+ a
1+ b
ab + bc + ca + a
ab + bc + ca + b 2
a (b + c ) + b (c + a )
a
b
=
+
=
( a + b )( a + c ) ( b + a )( b + c ) ( a + b )( b + c )( c + a )
2

2ab + c ( a + b )

( a + b )( b + c )( c + a )
c + ab
=

( c + a )( c + b )

Do P

1
1 + c2

3c
1 + c2

ab ( a + b ) + c ( a + b )

( a + b )( b + c )( c + a )
1

( c + a )( c + b )

(1 + 9 )

1+ c

1
1 + c2

c2
10 .
=
1 + c 2

ng thc xy ra khi v ch khi a =


b = 10 3, c =
3.
Bi 3. Cho x,y,z l cc s thc dng tha mn iu kin xyz + x + y =
z v z 1 .
Tm gi tr ln nht ca biu thc
1
1
z
.
P = 4
+
+
2
2

1 + y 1 + z2
1+ x

Li gii
Theo gi thit ta c: xy +

y x
+ =
1 nn tn ti 3 gc ca mt tam gic sao cho:
z z

A
B 1
C
=
x tan
=
, y tan
=
,
tan .
2
2 z
2

Khi :

667

www.TaiLieuLuyenThi.com
Khm ph t duy K thut gii bt T Bi ton Max Min ng Thnh Nam

tan

1
1
2 = 4 cos 2 A + cos 2 B + 1 sinC
+
+
P = 4

2
2 2

1 + tan 2 A 1 + tan 2 B 1 + tan 2 C


2
2
2

1
A+ B
A B 1
4 + 2 ( cos A + cos B ) + sinC =
4 + 4cos
cos
=
+ sin C
2
2
2
2
C
A B 1
C 1
= 4 + 4sin cos
+ sin C 4 + 4sin + sin C
2
2
2
2 2
C
C
9+4 2
.
1 450 C 900 P
2
2
2

Mt khc z 1 tan

ng thc xy ra khi v ch khi z= 1, x= y=

2 1.

Bi 4. Cho x, y, z l cc s thc dng thay i tha mn iu kin xyz + x + z =


y.
Tm gi tr ln nht ca biu thc
P=

2
x +1
2

2
y +1
2

4z
z +1
2

(z

3z
2

+1

z2 + 1

Li gii

t x = tan A, y = tan B, z = tan C , 0 < A, B, C < .


2

Theo gi thit ta c:
yz
tan B tan C
x=
tan A =
= tan ( B C ) A = B C + k .
1 + yz
1 + tan B tan C
Do

2
Khi :

< A B + C < k =
0 A=
B C A B =
C .

1
1
4 tan C
3tan C

P = 2

+
2
2
1 + tan A 1 + tan B
1 + tan 2 C
1 + tan 2 C 1 + tan 2 C

= 2 cos 2 A cos 2 B 4sin C + 3sin C cos 2 C


= cos 2 A cos 2 B 4sin C + 3sinCcos 2 C
=
2sin ( A + B ) sin ( A B ) 4sin C + 3sinCcos 2 C
= 2sin C sin ( A + B ) 4sin C + 3sinCcos 2 C

C sin C 3cos 2 C =
2 sin C 1 3sin 2 C
2sin C 4sin C + 3sinCcos 2=

Nu sin C >
668

3
3
s dng bt ng thc AM GM ta c:
P < 0 xt vi sin C
3
3

www.TaiLieuLuyenThi.com
Cty TNHH MTV DVVH Khang Vit

P = sin C 1 3sin C
2

)(

6sin 2 C 1 3sin 2 C 1 3sin 2 C

6
3

6sin 2 C + 1 3sin 2 C + 1 3sin 2 C

3
2

=
9
6
ng thc xy ra khi v ch khi

1
=
tan C
3

2
=
, tan A
4

2
=
, tan B
2

Vy gi tr ln nht ca P bng

2
t ti
=
x
9

=
sin C

2
=
2x =
2, y =
,z
2
2
=
,z
2

=
2, y

2
.
4

2
.
4

Cch 2: Theo gi thit ta c:


yz
2
2
4z
3z
=
x
=
P
2

+
2
1 + yz
y +1
yz
z2 + 1
z2 + 1 z2 + 1

+1
1 + yz

=
P

(y

2 (1 + yz )
2

)(

+1 z +1
2

(
)(

2
y +1
2

4z
z +1
2

)
)

(z

3z
2

+1

z2 + 1

2 z 2 y + y 2 1 z
3z

4z +
=
2
2
2
2
y +1 z +1
z +1
z + 1 z2 + 1

) (
)( )

S dng bt ng thc C S ta c

2
2
2
2
2 z 2 y + y 2 1 z 2 z 4 y + y 1 1 + z
2z

=
2
2
2
2
y +1 z +1
y +1 z +1
1 + z2

(
)(

)
)

2z

Suy ra P
t t
=

1 + z2
z

z +1
2

4z
z2 + 1

(z

3z
2

+1

z
=
3
+
2
z2 + 1
z +1

z
z2 + 1

, ( t ( 0;1) ) khi P f (t ) =
3t 3 + t .
t( 0;1)

Ta c f '(t )= 9t 2 + 1; f '(t )= 0
t=

1
.
3

669

www.TaiLieuLuyenThi.com
Khm ph t duy K thut gii bt T Bi ton Max Min ng Thnh Nam

Ta c f(t) i du dng sang m khi i qua t =

1
1
nn f(t) t cc i ti t =
3
3

1 2
trn khong ( 0;1) hay f (t ) f =
.
3 9

2
2
t ti
=
x
=
,y =
2, z
9
2
Bi 5. Cho x,y,z l cc s thc dng tha mn iu kin

Vy gi tr ln nht ca P bng

2
.
4

4 x 2 + 4 y 2 + 4 z 2 + 16 xyz =
1.

Tm gi tr nh nht ca biu thc P =

x + y + z + 4 xyz
.
1 + 4 ( xy + yz + zx )

Li gii
Theo gi thit tn ti 3 gc nhn ca mt tam gic sao cho:
=
2 x cos
=
A, 2 y cos
=
B, 2 z cos C .
3

A
B
B
C
C
A

tan tan + tan tan + tan tan

A
B
C
1
2
2
2
2
2
2 =
Khi tan 2 tan 2 tan 2
.

2
2
2
3
27

cos 2

A
B
C
A
B
C
cos 2 cos 2 27sin 2 sin 2 sin 2
2
2
2
2
2
2

(1 + cos A )(1 + cos B )(1 + cos C ) 27 (1 cos A )(1 cos B )(1 cos C )
(1 + 2 x )(1 + 2 y )(1 + 2 z ) 27 (1 2 x )(1 2 y )(1 2 z )
28 ( x + y + z + 4 xyz ) 13 (1 + 4 xy + 4 yz + 4 zx )
=
P

x + y + z + 4 xyz
13

1 + 4 xy + 4 yx + 4 zx 28

Bt ng thc c chng minh. ng thc xy ra khi v ch khi x= y= z=


Bi 6. Cho a,b,c l cc s thc dng c tng bng 1. Chng minh
bc
abc
abc 2 3 5
.
3

+
a
+
bc
c
+
ab
b
+
ca
4

Li gii
ab ca
ab bc
bc ca
.
+
.
+ =
.
1.
c
b
c
a
a
b
Suy ra tn ti 3 gc mt tam gic sao cho

Ch theo gi thit ta c a =
+b+c

670

1
.
4

www.TaiLieuLuyenThi.com
Cty TNHH MTV DVVH Khang Vit

bc
A ca
B ab
C
= tan
=
,
tan
=
,
tan .
a
2
b
2
c
2
Khi

C
B

2 A
tan
tan
tan 2

2
2
3

+
A
C
2
2
1 + tan
1 + tan 2 B
1 + tan
2
2
2

A 1

1
3 sin 2 sin C + sin B
2 2

P
=

=
=

1
2

3 cos A sin B + 3 sin C +

Bt ng thc cui ng do

3
5
3
+
2
4 2

3 cos A sin B + 3 sin C

5
.
2

A= B=

a= b= 2 3 3
6
ng thc xy ra khi v ch khi
.

C = 2
c= 7 4 3

3
Bi 7. Cho a,b,c l cc s thc dng tha mn iu kin a 2 + b 2 + c 2 + 2abc =
1.

Chng minh rng a 2 + b 2 + c 2 4 a 2b 2 + b 2 c 2 + c 2 a 2 .


Li gii
Theo gi thit tn ti ba gc nhn ca mt tam gic tha mn:
=
a cos
=
A, b cos
=
B, c cos C .
Ta cn chng minh:

cos 2 A + cos 2 B + cos 2 C 4 cos 2 A cos 2 B + cos 2 B cos 2 C + cos 2 C cos 2 A

cot 2 A

cyc 1 + cot

4
cyc

cot 2 A cot 2 B

(1 + cot A)(1 + cot B )


2

cot 2 A
cot 2 A cot 2 B
4
2
cyc ( cotA + cotB )( cot A + cot C )
cyc ( cot A + cotB ) ( cot C + cot A )( cotC + cot B )

cot 2 A cot 2 B
(lun ng).
cyc cot A + cot B

( cot B + cot C ) cot 2 A 4


cyc

cot 2 A cot 2 B
cot A cot B ( cot A + cot B=
)
cyc cot A + cot B
cyc

Do 4

( cot B + cot C ) cot 2 A .


cyc

671

www.TaiLieuLuyenThi.com
Khm ph t duy K thut gii bt T Bi ton Max Min ng Thnh Nam

1
.
2

ng thc xy ra khi v ch khi a= b= c=


Cch 2: Ta a v chng minh

(a

)(

) (
+ c ) 2(a b

+ b 2 + c 2 + 2abc a 2 + b 2 + c 2 4 a 2b 2 + b 2 c 2 + c 2 a 2

a 4 + b 4 + c 4 + 2abc a 2 + b 2

2 2

+ b2c2 + c2 a 2

Ta c:

(
)
) abc ( a + b + c ) .

1
3
abc , a + b + c 2 a 2 + b 2 + c 2 a b c = 2 4abc a b c 0 .
8
2

Do 2abc a 2 + b 2 + c 2

Ta a v chng minh bt ng thc:

a 4 + b 4 + c 4 + abc ( a + b + c ) 2 a 2b 2 + b 2 c 2 + c 2 a 2

a 2 ( a b )( a c ) + b 2 ( b c )( b a ) + c 2 ( c a )( c b ) 0

Khng mt tnh tng qut gi s a b c khi c 2 ( c a )( c b ) 0 v


a 2 ( a b )( a c ) + b 2 ( b c )( b a ) = ( a b ) a 2 ( a c ) b 2 ( b c )

2
2
( a b ) a ( b c ) b ( b c )

=( a b )( b c ) a 2 b 2 0

Bt ng thc c chng minh. ng thc xy ra khi v ch khi a= b= c


Bi tp tng t
Cho a,b,c l cc s thc khng m tha mn a 2 + b 2 + c 2 + abc =
4.

)(

)(

Chng minh rng a 2 + b 2 abc b 2 + c 2 abc c 2 + a 2 abc a 2b 2 c 2 .


Bi 8. Cho x,y,z l cc s thc dng tho mn iu kin 6 x + 3 y + 2 z =
xyz .
x yz

Tm gi tr ln nht ca biu thc P =


x

+ 14

(y

Li gii
Gi thit bi ton tng ng vi

672

)(

+4 z +9

2
6
3
+
+ =
1.
xy yz zx

www.TaiLieuLuyenThi.com
Cty TNHH MTV DVVH Khang Vit

1
2
3
t a = , b = , c = ab + bc + ca =1. Khi tn ti ba gc mt tam gic
x
y
z

A
B
C
sao=
cho a tan
=
, b tan
=
, c tan .
2
2
2

Khi P
=

1
1
A
B
C 4 3
.
.=
.
cos
cos cos
2
2
2
2
2
9
1+ a
1+ b
1+ c
2

ng thc xy ra khi v ch khi x= y= 2 2, z= 3 2 .


C. BI TP RN LUYN
Bi 1. Cho a,b l hai s thc bt k chng minh rng

( a b )(1 ab )

(1 + a )(1 + b )
2

1
.
2

Bi 2. Cho x,y l hai s thc tho mn iu kin 36 x 2 + 16 y 2 =


5.
Tm gi tr ln nht v gi tr nh nht ca biu thc: Z =y 2 x + 5 .
5
5
sin a th t gi thit ta c y =
cos a .
6
4
Bi 3. Chng minh rng vi mi s thc a,b,c ta c

Bi 2. t x =

a b

a 2 + 1. b 2 + 1

bc
b 2 + 1. c 2 + 1

ca
c 2 + 1. a 2 + 1

Bi 4. Cho a, b, c l cc s thc dng tho mn iu kin abc + c + 2b =


2a .
1

Chng minh rng

b2

c2

3
.
2

1+ a
1+ b
4+c
Bi 5. Cho a, b, c l cc s thc dng tha mn iu kin a.b + bc + ca =
1.
2

3 3
.
2
1+ a
1+ b
1+ c
Bi 6. Cho a,b,c l cc s thc thuc khong (0;1). Chng minh rng

Chng minh rng

abc +

(1 a )(1 b )(1 c ) < 1 .

Bi 7. Cho a,b,c l cc s thc dng tho mn iu kin a + b + c =


1.
a
b
c
9
+
+
.
a + bc b + ca c + ab 4
Bi 8. Cho a,b,c l cc s thc dng tha mn iu kin a + b + c =
abc .

Chng minh rng

673

www.TaiLieuLuyenThi.com
Khm ph t duy K thut gii bt T Bi ton Max Min ng Thnh Nam

Tm gi tr ln nht ca biu thc P =

2
a2 + 1

b2 + 1

1
c2 + 1

Bi 9. Cho x,y,z l cc s thc dng tha mn iu kin xyz + x y + z =


0.
Tm gi tr ln nht ca biu thc P =

2
x +1
2

2
y +1
2

3
z +1
2

Bi 10. Cho a,b,c l cc s thc dng tha mn iu kin


ab + bc + ca
=

( a + 1)(b + 1)( c + 1) + 1 .
2

Chng minh rng a + b + c 3 .


Bi 11. Cho a,b,c l cc s thc dng tho mn iu kin a + b + c =
abc .
a
b
c
.
+
+
a + bc b + ca c + ab
Bi 12. Cho a,b,c l cc s thc dng tho mn iu kin a + b + c =
abc .

Tm gi tr nh nht ca biu thc P =

a
c
b
Tm gi tr nh nht ca biu thc P = + 3
+
.
a + bc
b + ca c + ab

Bi 13. Cho a,b,c l cc s thc dng tho mn iu kin

1 1
1
+
+ =
6.
a 2b 3c

a
b
c
1
.
.
.

a + 36bc b + 9ca c + 4ab 27


Bi 14. Cho a,b,c l cc s thc dng tho mn iu kin
2009ac + ab + bc =
2009 .

Chng minh rng

2
2b 2
3
.
Tm gi tr ln nht ca biu thc P =

+ 2
2
2
2
a + 1 b + 2009
c +1

Bi 15. Cho a,b,c l cc s thc khng m tha mn a 2 + b 2 + c 2 + abc =


4.

)(

)(

Chng minh rng a 2 + b 2 abc b 2 + c 2 abc c 2 + a 2 abc a 2b 2 c 2 .


Bi 16. Cho a,b,c l cc s thc dng tho mn iu kin ab + bc + ca + abc =
4.
Chng minh rng

ab + bc + ca 3 .

Bi 17. Chox,y,z l cc s thc dng tho mn iu kin xy + yz + zx =


xyz .
Chng minh rng

2
1 + x2

1
1 + y2

1
1 + z2

9
.
4

Bi 18. Cho a,b,c l cc s thc dng tho mn iu kin a + b + c =


6.
Tm gi tr ln nht ca biu thc
674

www.TaiLieuLuyenThi.com
Cty TNHH MTV DVVH Khang Vit

P=
(3 a )(3 b)(3 c)(

1
2 2

b c

2 2

c a

1
2 2

a b

).

Bi 19. Cho a,b,c l cc s thc dng tho mn iu kin a 2 + b 2 + c 2 + abc =


4.
Chng minh rng a 4 a 2 + b 4 b 2 + c 4 c 2 3 3 .
Bi 20. Cho a,b,c l cc s thc thuc khong (0;2) tho mn iu kin
ab + bc + ca + abc =
4.
4 a 2 + 4 b2 + 4 c2 3 3 .

Chng minh rng

Bi 21. Cho x,y,z l cc s thc dng tho mn iu kin xy =


1+ z ( x + y) .
Tm gi tr ln nht ca
biu thc P
=

2 xy ( xy + 1)

(1 + x )(1 + y )
2

z +1
2

D. HNG DN GII P S
Bi 1. t
=
a tanx
=
;b tany ta c
VT=

( tan x tan y )(1 tan x.tan y )=

(1 + tan x )(1 + tan y )


2

sin( x y ) cos( x y )=

1
1
sin 2( x y ) .
2
2

Bt ng thc c chng minh.


5
5
5
5
sin a
cos a + 5
sin a
cos a = Z 5 (*)
6
2
6
2
5 5 49 23
37
2
Phng trnh (*) c nghim khi v ch khi ( Z 5 ) + =
Z .
36 4 36
6
6
Bi 3. =
t a tanx
=
; b tany
=
; c tan z .

Nn Z =

Bt ng thc cho tng ng vi:


tan x tan y
1 + tan 2 x . 1 + tan 2 y

tan y tan z
1 + tan 2 y . 1 + tan 2 z

tan x tan z
1 + tan 2 x . 1 + tan 2 z

sin( x y ) + sin( y z ) sin( x z )

Bt ng cui ng bi v
sin( x y ) + sin( y z ) sin( x y ) + sin( y z ) sin( x z ) .

Bt ng thc c chng minh.


Bi tp tng t
Chng minh rng vi mi s thc a,b,c ta c
a b
a 2 + 2015. b 2 + 2015

bc
b 2 + 2015. c 2 + 2015

ca
c 2 + 2015. a 2 + 2015

675

www.TaiLieuLuyenThi.com
Khm ph t duy K thut gii bt T Bi ton Max Min ng Thnh Nam

Bi 4. Theo gi thit tn ti ba gc mt tam gic sao cho:


a = tanA ;

1
2
= tan B ; = tan C .
c
b

Bt ng thc cho tng ng vi: cos A + cos B + cos C

3
.
2

Bt ng thc cui lun ng v ta c pcm.


A
B
C
Bi 5.=
t a tan
=
; b tan . T gi thit suy ra c = tan ; vi A, B, C l 3 gc
2
2
2
ca tam gic nhn ABC
Bt ng thc tng ng vi:
1
3 3
(lun ng).
(tan A + tan B + tan C )
2
2
2
Bi 6.=
t a sin
=
x; b

2
sin
=
y; c sin 2 z .

Ta cn chng minh sin x.sin y.sin z + cos x.cos y.cos z < 1 .


Bt ng thc ng bi v
sin x.sin y.sin z + cos x.cos y.cos z sin x.sin y + cos x.cos =
y cos( x y ) 1 .
Du bng khng xy ra v ta c pcm.
Bi 7. Ta cn chng minh

1
1
1
9
+
+

bc
ca
ab 4
1+
1+
1+
a
b
c

ab bc
bc ca
ca ab
.
+
.
+
.
=
1.
c
a
a
b
b
c
Do tn ti ba gc mt tam gic sao cho

Theo gi thit ta c

bc
A ca
B ab
C
= tan
=
;
tan
=
;
tan .
a
2
b
2
c
2

Bt ng thc tr thnh:

1
1
9
+
+
.
A
B
C
4
1 + tan 2
1 + tan 2
1 + tan 2
2
2
2
A
B
C 9
cos 2 + cos 2 + cos 2
2
2
2 4
3 + cos A + cos B + cos C 9

2
4
3
cos A + cos B + cos C
2

Bt ng cui ng v ta c pcm. ng thc xy ra khi v ch khi a= b= c=


676

1
.
3

www.TaiLieuLuyenThi.com
Cty TNHH MTV DVVH Khang Vit

Bi 8. Theo gi thit tn ti 3 gc nhn ca mt tam gic sao cho:


=
a tan
=
A, b tan
=
B, c tan C .
Khi :
P = 2cosA + cos B + cos C = 2cosA + 2cos

B+C
B C
.
cos
2
2
2

2 4sin 2

C
C
C 1 7 7
+ 2sin =4 sin +
2
2
2 4
4 4

Bi 9. T gi thit ta c: xz +

x z
+ =
1.
y y

A 1
B
C
Do tn ti 3 gc ca mt tam gic sao=
cho x tan
=
,
tan
=
, z tan .
2 y
2
2

Khi ta c:
2
2
3
A
B
C
P=

+
= 2cos 2 2sin 2 + 3cos 2
A
B
C
2
2
2
1 + tan 2
1 + cot 2
1 + tan 2
2
2
2
C
A+ B
A B
C
cos
= cos A + cos B + 3 3sin 2 = 2cos
+ 3 3sin 2
2
2
2
2
2

2sin

C
C
C 1 10 10
3sin 2 + 3 =3 sin +
2
2
2 3
3
3

C 1
ng thc xy ra khi v ch khi sin= =
,A B
2 3
1
1
.
=
x
,=
y
2,=
z
2
2 2
1
1
.
=
,y =
2, z
2
2 2
Cch 2: T iu kin cho php ta gim bin biu thc P v hai bin x v y.

Vy gi tr ln nht ca P bng

Khi ta c:
P=

2 y 2 x2

xyz + x y + z = 0 z =

yx
0 y x.
1 + xy

(1 + x )(1 + y )
2

10
t ti
=
x
3

3
2

yx

+1
1 + xy

2 y 2 x2

(1 + x )(1 + y
2

(
)
) (1 + x )(1 + y )
+

3 x 2 y 2 + 2 xy + 1
2

677

www.TaiLieuLuyenThi.com
Khm ph t duy K thut gii bt T Bi ton Max Min ng Thnh Nam

2 y 2 x2

) (
)(1 + y )

3 x 2 y 2 + x 2 + y 2 + 1 3 x 2 2 xy + y 2

(1 + x )(1 + y )
(1 + x
2( y x )
3( x y )
( y x )( 5 x y ) + 3

=
+3
(1 + x )(1 + y ) (1 + x )(1 + y ) (1 + x )(1 + y )
2

S dng bt ng thc AM-GM v C-S ta c:


3( y x ) + 5x y
2
2
2
3 ( y x )( 5 x y )
= ( x + y) 1+ x 1+ y .
2

)(

1
10
Suy ra P + 3 = .
3
3
1

3 y 3 x = 5 x y
1

x =
2 z= .
ng thc xy ra khi v ch khi x 1

2 2
1 = y
y = 2

Bi tp tng t
1) Cho a,b,c l cc s thc dng tha mn iu kin b = a + c + abc .
Tm gi tr ln nht ca biu thc P =

.
a +1 b +1 c +1
2) Cho a,b,c l cc s thc dng tha mn iu kin 2014ac + bc + ca =
2014 .
2

2
2b 2
3
.
Tm gi tr ln nht ca biu thc P =

+ 2
2
2
a + 1 b + 2014 c + 1
3) Cho x,y,z l cc s thc dng tha mn iu kin xy + yz + zx =
1.

Tm gi tr ln nht ca biu thc P =

x
x +1
2

y
y +1
2

2z

z +1
4) Cho x,y,z l cc s thc dng tha mn iu kin xz yz xy =
1.

Tm gi tr nh nht ca biu thc P =

2 x2
1 + x2

2 y2
1 + y2

3z 2
1 + z2

Bi 10. S dng ng thc:

( a + 1)(b + 1)( c + 1)=


2

( ab + bc + ca 1)2 + ( a + b + c abc )2 .

Suy ra a + b + c =
abc .
Do tn ti 3 gc ca mt tam gic tha mn
=
a tan
=
A, b tan
=
B, c tan C .
Bi ton a v chng minh tan A + tan B + tan C 3 3 .
Bt ng thc cui lun ng v ta c pcm.
678

www.TaiLieuLuyenThi.com
Cty TNHH MTV DVVH Khang Vit

Bi 11. iu kin bi ton tng ng vi:


ab bc
bc ca
ca ab
ab bc ca
.
.
+
.
+
.
=
.
.
c
a
a
b
b
c
c
a
b
Do tn ti ba gc trong mt tam gic sao cho
ab
bc
ca
= tan
=
A,
tan
=
B,
tan C .
c
a
b

Khi P =

1 + tan A 1 + tan B 1 + tan C


ng thc xy ra khi v ch khi a= b= c= 3 .
Bi 12. iu kin bi ton tng ng vi:
2

= cos 2 A + cos 2 B + cos 2 C

3
.
4

ab bc
bc ca
ca ab
ab bc ca
.
.
+
.
+
.
=
.
.
c
a
a
b
b
c
c
a
b
Do tn ti ba gc trong mt tam gic sao cho
bc
ca
ab
= tan
=
A,
tan
=
B,
tan C .
a
b
c

1
1
1

+
Khi P = 2 + 3

2
2
1 + tan A
1 + tan B 1 + tan C

=
cos 2 A + 3 cos 2 B + cos 2 C

cos 2 A + 3 ( cos 2 B + cos 2C ) + 1 + 2 3


2

4 3 3
4

5
Ch cos 2 A + 3 ( cos 2 B + cos 2C ) .
2
3+ 2 3
ng thc xy ra khi v ch khi a =
.
7 + 4 3, b =
c=
3
Bi 13. Gi thit cho tng ng vi:

6bc + 3ac + 2ab= 36abc


6

6bc + 3ac + 2ab 36abc


=
abc
abc

bc
ac
ab
bc
ac
ab
6
.3
.2
+3
+2
=
a
b
c
a
b
c

A
B
C
A
B
C
+ cot + cot =
cot .cot .cot .
2
2
2
2
2
2
Suy ra tn ti ba gc trong mt tam gic sao cho:

Trong tam gic ta c: cot

bc
A
ab
C
ca
B
= cot .
=
2
cot
=
;3
cot ; 6
a
2
c
2
b
2
679

www.TaiLieuLuyenThi.com
Khm ph t duy K thut gii bt T Bi ton Max Min ng Thnh Nam

V vy VT =

1
1
.
.
.
A
B
2
2
2C
1 + cot
1 + cot
1 + cot
2
2
2
2

= sin 2

A 2B
C 1
A B
A+ B
C
sin
sin
=
cos
cos
sin
2
2
2 4
2
2
2
2

1
A B
C
C 1
C
C
= cos
sin sin 1 sin sin
4
2
2
2 4
2
2
3

C
C
C
1 sin + 1 sin + 2sin

1
1
C
C
C 1
2
2
2
=
=
1 sin 1 sin 2sin
8
2
2
2 8
3
27

Bt ng thc c chng minh.

b
b
+
.c =
1.
2009 2009
Suy ra tn ti ba gc mt tam gic sao cho:
A b
B
C
=
a tan
=
;
tan ; c = tan .
2 2009
2
2
Khi
2
2
3
A
B
C
P=

+
= 2cos 2 2sin 2 + 3cos 2
1
A
C
2
2
2
1 + tan 2
1+
1 + tan 2
2
2
2 B
tan
2

Bi 14. T gi thit ta c: ac + a.

1
C
A B
C 1
A B 10
) =3 + cos 2
cos
=cos A + cos B + 3(1 sin
3 sin

2
3
2
2
2
3
3

Bi tp tng t
1) Cho a,b,c l cc s thc dng tho mn iu kin abc + a + b =
c.
Tm gi tr ln nht ca biu thc P =

c2

.
a 2 + 1 b2 + 1 c2 + 1
2) Cho a,b,c l cc s thc dng tho mn iu kin abc + a + b =
c.
Tm gi tr ln nht ca
biu thc P
=

2 (1 + ab )

(1 + a )(1 + b )
2

c
c +1
2

3) Cho a,b,c l cc s thc dng tho mn iu kin ab =1 + bc + ca .


Tm gi tr ln nht ca biu thc P =
680

a2

b2

a +1 b +1 c +1
2

www.TaiLieuLuyenThi.com
Cty TNHH MTV DVVH Khang Vit

4) Cho a,b,c l cc s thc dng tho mn iu kin ab =1 + bc + ca .


3

Tm gi tr nh nht ca biu thc P =

3c

.
a +1 b +1 c +1
5) Cho a,b,c l cc s thc dng tho mn iu kin ab =1 + bc + ca .
Tm gi tr nh nht ca biu thc P =

a +1 b +1 c +1
2

yz
zx
Bi 15. t a 2=
=
, b 2=
,c 2
( x + y )( x + z )
( y + z )( y + z )

4 y 2 z + yz 2 + x 2 z + xz 2 2 xyz

Khi a + b abc
=
2

Tng t suy ra:

(a

+ b abc
2

( x + y )( y + z )( z + x )

4 yz 2 + xz 2

( x + y )( y + z )( z + x )

.
xy

( z + x )( z + y )

4 yz 2 + xz 2

( x + y )( y + z )( z + x )

a 2b 2 c 2

Bi 16. Bt ng tng ng vi: cos A + cos B + cos C

3
.
2

ab
bc
ca
.
=
;cos B =
;cos C
2
2
2
Bi 17. Theo gi thit tn ti cc gc nhn trong mt tam gic sao cho
=
x tan
=
A; y tan
=
B; z tan C .

Vi cos A
=

Khi gi P l biu thc v tri ta c


2
1
1
P=
+
+
1 + tan 2 A
1 + tan 2 B
1 + tan 2 C
= 2cos A + cos B + cos C

=
2 1 2sin 2

2 1 2sin 2

A
A
B C
+ 2sin cos
2
2
2
A
A
+ 2sin
2
2
2

9
A 1
9

= 4 sin
4
2 4
4

Bt ng thc c chng minh.


Bi 18. Trc ht,ta bin i P v dng:
8P =

(a + b c)(b + c a )(c + a b)( a 2 + b 2 + c 2 )


a 2b 2 c 2
681

www.TaiLieuLuyenThi.com
Khm ph t duy K thut gii bt T Bi ton Max Min ng Thnh Nam

By gi ta s chng minh rng:


(a + b + c)(a + b c)(b + c a )(c + a b)(a 2 + b 2 + c 2 ) 9a 2b 2 c 2 (1)

Tht vy,nu (a + b c)(b + c a)(c + a b) 0 bt ng thc trn ng


Nu (a + b c)(b + c a)(c + a b) > 0 ta thy rng 3 s a + b c; b + c a;
c + a b khng th c ng thi 2 s m.
V th, ta c th quy bi ton v chng minh bt ng thc ng vi 3 cnh 1
tam gic.
Ta xt tam gic ABC vi bn knh ngoi tip=
R; a BC
=
; b CA
=
; c AB .
Ta c : 9R 2 a 2 + b 2 + c 2 hay

9a 2 b 2 c 2
16 S 2

(a 2 + b 2 + c 2 ) .

hay 9a 2b 2 c 2 (a + b + c)(a + b c)(b + c a )(c + a b)(a 2 + b 2 + c 2 )


Vy ta c (1) c chng minh.
3
9 3
Suy ra 8 P = hay P .
16
6 2
Bi 19. Theo gi thit tn ti ba gc nhn trong mt tam gic sao cho
=
a 2cos
=
A; b 2cos
=
B; c 2cos C .
Bt ng thc tr thnh:
cos sA 4 cos 2 A + cos B 4 cos 2 B + cos C 4 cos 2 C 3 3
cos A sin A + cos B sin B + cos C sin C

3 3
2

cos A cos B cos C


.
Khng mt tnh tng qut gi s A B C
sin A sin B sin C

Theo bt ng thc Chebyshev ta c


1
( cos A + cos B + cos C )( sin A + sin B + sin C )
3
1 3 3 3 3 3
. .
=
3 2 2
4
Bt ng thc cui ng v ta c pcm.
Bi 20. S dng bt ng thc C S ta c
cos A sin A + cos B sin B + cos C sin C

4 a 2 + 4 b 2 + 4 c 2 3 12 a 2 b 2 c 2 .

Ta ch cn chng minh a 2 + b 2 + c 2 3 .
Tht vy theo gi thit tn ti ba gc nhn mt tam gic sao cho
=
cos A
682

bc
=
,cos B
2

ca
=
,cos C
2

ab
.
2

www.TaiLieuLuyenThi.com
Cty TNHH MTV DVVH Khang Vit

Suy ra a
=

2cos B cos C
2cos C cos A
2cos A cos B
.
=
,b =
,c
cos A
cos B
cos C
2

2cos B cos C 2cos C cos A 2cos A cos B


Vy ta chng minh
+
+
3.
cos A
cos B
cos C

Bt ng thc ng do
2

3
cos B cos C cos C cos A cos A cos B
2
2
2

+
+
cos A + cos B + cos C
4
cos A cos B cos C
Ta c pcm.

Bi 21. Theo gi thit ta c

1 1
1
+ z + =
1.
xy
x y

1
A 1
B
C
Do tn ti ba gc mt tam gic sao=
cho
tan
=
,
tan
=
, z tan .
x
2 y
2
2

Khi s dng bt ng thc C S v AM GM ta c

1
2 1 +
xy
2
z
z

=
+ 2

+ 2
P
1
1 z +1

1
1 z +1

1
+
1
+
1 + 2 1 + 2

x
y

x2
y2

tan

C
2

+
+ 2 =
+
+
.
1
1
z + 1 1 + tan 2 A 1 + tan 2 B 1 + tan 2 C
+
1
2
2
2
x2
y2
A
B 1
= cos 2 + cos 2 + sin C
2
2 2
3
1+
1
4+3 3
2
= 1 + ( cos A + cos B + sin C ) 1 +
=
.
2
2
4
1+

Vy gi tr ln nht ca P bng
t ti A = B =

,C =

4+3 3
,
4

2
x = y = 2 + 3; z = 3 .
3

683

www.TaiLieuLuyenThi.com
Khm ph t duy K thut gii bt T Bi ton Max Min ng Thnh Nam

CH 2: K THUT S DNG BT NG THC SCHUR


Ni dung ch ny cp n bt ng thc Schur v mt s bi ton p
dng dng c bn nht. chi tit hn cc bn c th tm c mt s bi vit trong
[1].[2].
A. NI DUNG PHNG PHP
Vi mi a,b,c,k l cc s thc khng m ta c
a k ( a b )( a c ) + b k ( b c )( b a ) + c k ( c a )( c b ) 0

Vi k = 1 ta c bt ng thc Schur bc 3
a3 + b3 + c3 + 3abc ab ( a + b ) + bc ( b + c ) + ca ( c + a )

Mt s dng tng ng:

( x + y + z )3 + 9 xyz 4 ( x + y + z )( xy + yz + zx ) ;

( x + y + z ) ( x 2 + y 2 + z 2 ) + 9 xyz 2 ( x + y + z )( xy + yz + zx ) .
Vi k = 2 ta c bt ng thc Schur bc 4:

a 4 + b 4 + c 4 + abc ( a + b + c ) ab a 2 + b 2 + bc b 2 + c 2 + ca c 2 + a 2 .

Trong qu trnh s dng bt ng thc thng a v chng minh cc bt ng


thc rng buc gia p,q,r c gi l phng php bt ng thc Schur kt hp
i bin p, q, r.
Cc hng ng thc cn lu
Ta t p = x + y + z; q = xy + yz + zx; r = xyz , khi
ab ( a + b ) + bc ( b + c ) + ca ( c + a ) = pq 3r ;

( a + b )( b + c )( c + a ) =

pq r ;

ab a 2 + b 2 + bc b 2 + c 2 + ca c 2 + a 2 = p 2 q 2q 2 pr ;

( a + b )( a + c ) + ( b + c )( b + a ) + ( c + a )( c + b ) =

p 2 + q;

a 2 + b 2 + c 2 = p 2 2 q;
a3 + b3 + c3 = p3 3 pq + 3r ;
a 4 + b 4 + c 4 = p 4 4 p 2 q + 2q 2 + 4 pr.

Nu t L = p 2 q 2 + 18 pqr 27 r 2 4q3 4 p3 r , khi

684

www.TaiLieuLuyenThi.com
Cty TNHH MTV DVVH Khang Vit

pq + 3r L
a 2b + b 2 c + c 2 a =
2
L
( a b )( b c )( c a ) =

Ngoi ra nh gi r vi p,q ta thng s dng hai bt ng thc:

p 4 p q2

r max 0;
9

)(

p2 q 4 p q2

r max 0;

6p

B. BI TP MU
Bi 1. Cho a,b,c l cc s thc dng c tng bng 3. Chng minh

12
15 .
ab + bc + ca
Li gii
S dng bt ng thc Schur bc 3 ta c
abc +

( a + b + c )3 + 9abc 4 ( ab + bc + ca )( a + b + c ) 3abc 4 ( ab + bc + ca ) 9
Ta cn chng minh 4 ( ab + bc + ca ) 9 +

36
15
ab + bc + ca

4 ( ab + bc + ca ) 24 ( ab + bc + ca ) + 36 0 4 ( ab + bc + ca 3) 0
2

Bt ng thc lun ng. ng thc xy ra khi v ch khi a= b= c= 1 .


Bi 2. Cho a,b,c l cc s thc dng c tch bng 1. Chng minh
2
1
3
.
+
a + b + c 3 ab + bc + ca
Li gii
S dng bt ng thc Schur bc 3 ta c

( ab + bc + ca )3 + 9a 2b2c 2 4 ( ab + bc + ca ) abc ( a + b + c )
( ab + bc + ca )3 + 9
a+b+c
4 ( ab + bc + ca )
8 ( ab + bc + ca )
1
3
Vy ta ch cn chng minh
+
3
( ab + bc + ca ) + 9 3 ab + bc + ca

t t = ab + bc + ca, t 3 3 a 2b 2 c 2 = 3 bt ng thc tr thnh


685

www.TaiLieuLuyenThi.com
Khm ph t duy K thut gii bt T Bi ton Max Min ng Thnh Nam

8t

) (

1 3
+ 24t 2 + t t 3 + 9 9 t 3 + 9
t +9 3 t
3

t 4 9t 3 + 24t 2 + 9t 81 0 ( t 3) t 3 6t 2 + 6t + 27 0

Lun ng vi t 3 . Bt ng thc c chng minh.


ng thc xy ra khi v ch khi a= b= c= 1 .
Bi 3. Cho a,b,c l cc s thc dng tho mn iu kin a 2 + b 2 + c 2 =
3.
Chng minh rng 12 + 9abc 7 ( ab + bc + ca ) .
t t = a + b + c,

3t 3

Li gii

( a + b + c )2 a 2 b 2 c 2

=
ab + bc + ca

=
2
S dng bt ng thc Schur bc 3 ta c

t2 3
.
2

( a + b + c )3 + 9abc 4 ( a + b + c )( ab + bc + ca ) 9abc 2t ( t 2 3) t 3 .

Vy ta ch cn chng minh

7 t2 3

) ( t 3)

( 2t + 5) 0 .
2
Bt ng thc c chng minh. ng thc xy ra khi v ch khi a= b= c= 1 .
Bi 4. Cho a,b,c l cc s thc dng tho mn iu kin ab + bc + ca =
3.
12 + 2t t 3 t
2

Chng minh rng a3 + b3 + c3 + 7 abc 10 .


Li gii
Ta c a3 + b3 + c3 = ( a + b + c ) 3 ( a + b )( b + c )( c + a )
3

( a + b + c )3 3 ( a + b + c )( ab + bc + ca ) + 3abc

t t = a + b + c, t 3 ( ab + bc + ca ) = 3 bt ng thc tr thnh
10abc + t 3 9t 10 0 .
Theo bt ng thc Schur bc 3 ta c

( a + b + c )3 + 9abc 4 ( a + b + c )( ab + bc + ca ) 9abc 12t t 3 .


+ Nu t 2 3 10abc + t 3 9t 10 > 12t 9t 10 = 3t 10 > 0 .
+ Nu 3 t 2 3 abc

686

12t t 3
.
9

www.TaiLieuLuyenThi.com
Cty TNHH MTV DVVH Khang Vit

Ta cn chng minh 10.

12t t 3 3
+ t 9t 10 0
9

( t 3) t 2 3t + 30 0

) )

( t 3) 3 ( 4 t ) + 16 t 2 + 2 0

Lun ng. Bt ng thc c chng minh. ng thc xy ra khi v ch khi


a= b= c= 1 .
Bi 5. Cho a,b,c l cc s thc dng. Chng minh

( a + b + c )( ab + bc + ca ) ( a3 + b3 + c3 ) ( a 2 + b2 + c 2 )

Li gii
Chun ho p = a + b + c = 1 . t q = ab + bc + ca; r = abc .
Ta c a 2 + b 2 + c 2 =1 2q; a3 + b3 + c3 =1 3q + 3r .
Khi ta cn chng minh: q (1 3q + 3r ) (1 2q ) .
3

S dng q 2=

( ab + bc + ca )2 3abc ( a + b + c )=

3r .

Do q (1 3q + 3r ) =
q 3q 2 + 3qr q 3q 2 + q3 .
Vy ch cn chng minh q 3q 2 + q3 (1 2q ) (1 q )( 3q 1) 0 .
3

Bt ng thc lun ng v ta c pcm. ng thc xy ra khi v ch khi a= b= c .


Bi 5. Cho x,y,z l cc s thc khng m tha mn

( x + y )( y + z )( z + x ) > 0 v

xy + yz + zx =
1 . Tm gi tr nh nht ca biu thc

P=

x ( x + y )( x + z ) + y ( y + z )( y + x ) + z ( z + x )( z + y )
x+ y+z

Li gii

Ta c:

x ( x + y )( x + =
z)

x x 2 + x ( y + z ) + yz
=

Suy ra

x ( x + y )( x + z )
=
x+ y+z

x2 +

x 2 ( x + y + z ) + xyz .

xyz
.
x+ y+z

Tng t ta c:
y ( y + z )( y + x )
x+ y+z

z ( z + x )( z + y )
xyz
xyz
.
=
y2 +
;
=
z2 +
x+ y+z
x+ y+z
x+ y+z

687

www.TaiLieuLuyenThi.com
Khm ph t duy K thut gii bt T Bi ton Max Min ng Thnh Nam

Suy ra P =
x2 +

xyz
xyz
xyz
.
+ y2 +
+ z2 +
x+ y+z
x+ y+z
x+ y+z

S dng bt ng thc Mincopski ta c:

xyz
P ( x + y + z ) + 3
x+ y+z
Theo bt ng thc schur ta c:
2

( x + y + z )2 +

9 xyz
.
x+ y+z

( x + y + z )3 + 9 xyz 4 ( x + y + z )( xy + yz + zx )
(x + y + z) +
2

9 xyz
4 ( xy + yz + zx ) =
4
x+ y+z

Do P 2 . ng thc xy ra khi v ch khi x= y= z=

1
.
3

Bi 6. Cho x,y,z l cc s thc khng m tha mn x + y + z =


1 v xy + yz + zx > 0 ;
k l mt s thc dng.
Tm gi tr nh nht ca biu thc P =

k x 2 + y 2 + z 2 + 9 xyz
xy + yz + zx

Li gii
Ta c:
k ( x + y + z ) 2k ( xy + yz + zx ) + 9 xyz
2

P=

xy + yz + zx

9 xyz
k
+
2k .
xy + yz + zx xy + yz + zx

Theo bt ng thc Schur ta c:

( x + y + z )3 + 9 xyz 4 ( x + y + z )( xy + yz + zx )
1 + 9 xyz 4 ( xy + yz + zx )

Suy ra P

TH1: Nu

9 xyz
1
4
zy + yz + zx
xy + yz + zx

k
1
k 1
+4
=
2k
+ 4 2k .
xy + yz + zx
xy + yz + zx
xy + yz + zx

2
x + y + z)
(
k > 1 k 1 > 0; xy + yz + zx
=

ng thc xy ra khi v ch khi x= y= z=

1
.
3

TH2: Nu 0 < k 1 khi s dng: x 2 + y 2 + z 2 +

688

1
P 3 ( k 1) + 4 2k = k + 1 .
3

9 xyz
2 ( xy + yz + zx ) .
x+ y+z

www.TaiLieuLuyenThi.com
Cty TNHH MTV DVVH Khang Vit

Ta c:

x + y 2 + z 2 + 9 xyz 2 ( xy + yz + zx ) k x 2 + y 2 + z 2 + 9kxyz 2k ( xy + yz + zx ) .
2

Mt khc: k 1 k x 2 + y 2 + z 2 + 9 xyz k x 2 + y 2 + z 2 + 9kxyz .


Do P
x= y=

k x 2 + y 2 + z 2 + 9kxyz
xy + yz + zx

2k . ng thc xy ra khi v ch khi

1
, z= 0 hoc cc hon v.
2

Bi 7. Cho a,b,c l cc s thc khng m tho mn iu kin a 2 + b 2 + c 2 =


3.
Chng minh rng ( 2 ab )( 2 bc )( 2 ca ) 1 .
Li gii
Bt ng thc cn chng minh tng ng vi

( 2 ab )( 2 bc )( 2 ca ) 1 0
7 4 ( ab + bc + ca ) + 2abc ( a + b + c ) a 2b 2 c 2 0
2
2
( a + b + c ) ( a + b + c abc ) 4 ( ab + bc + ca ) + 7 0
t t = a + b + c,

3 t 3 ab + bc + ca =

( a + b + c )2 a 2 b 2 c 2
2

t2 3
.
2

S dng bt ng thc Schur bc 3 ta c

( a + b + c )3 + 9abc 4 ( a + b + c )( ab + bc + ca )

9abc 2t t 2 3 t 3 = t 3 6t abc

t 3 6t
9

a 2 + b2 + c2
Ch abc
1
=

a + b + c abc 3 3 abc abc 3abc abc = 2abc 0


a + b + c abc t

t 3 6t 15t t 3
=
9
9
2

15t t 3
Vy ta chng minh t
2 t2 3 + 7 0
9

81.13 81t 2 t 2 t 2 15

)(

9 t 2 9 + 9 t 2 12 t 2 0

689

www.TaiLieuLuyenThi.com
Khm ph t duy K thut gii bt T Bi ton Max Min ng Thnh Nam

Lun ng do

3 t 3 . Bt ng thc c chng minh ng thc xy ra khi

v ch khi a= b= c= 1 .
Bi 8. Cho a,b,c l cc s thc dng tho mn iu kin abc = 1 .
Chng minh rng

b + c 3 c + a 3 a + b 5( a + b + c) + 9
.
+
+

2a
2b
2c
8

Li gii
t a =x3 , b =y 3 , c =z 3 xyz =1 .
Bt ng thc cho tr thnh:
yz 3

3
3
3
y3 + z3
z 3 + x3
x3 + y 3 5 x + y + z + 9
3
3
.
+ zx
+ xy

2
2
2
8

S dng bt ng thc AM GM ta c
y3 + z3
=
yz 3
2

y+z
. yz. yz. yz. y 2 yz + z 2
2

3
y + z 3 yz + y 2 yz + z 2
y+z
.
=

2
4
2

Tng t ta c zx 3

3
3
z 3 + x3 z + x
3 x + y x+ y .

;
xy

2
2
2
2

Cng theo v ba bt ng thc trn ta c


y3 + z3
z 3 + x3
x3 + y 3 ( x + y ) + ( y + z ) + ( z + x )
.
+ zx 3
+ xy 3

2
2
2
8
3

yz 3

Vy ta ch cn chng minh

( x + y )3 + ( y + z )3 + ( z + x )3 5 ( x3 + y3 + z 3 ) + 9
x3 + y 3 + z 3 + 3 xy ( x + y ) + yz ( y + z ) + zx ( z + x )
x3 + y 3 + z 3 + 3 xyz xy ( x + y ) + yz ( y + z ) + zx ( z + x )

Bt ng thc cui l bt ng thc Schur bc 3 nn ta c pcm.


ng thc xy ra khi v ch khi a= b= c= 1 .

690

www.TaiLieuLuyenThi.com
Cty TNHH MTV DVVH Khang Vit

C. BI TP RN LUYN
Bi 1. Cho a,b,c l cc s thc dng c tng bng 1. Chng minh
5
2 a3 + b3 + c3 + 3 a 2 + b 2 + c 2 + 12abc .
3
Bi 2. Tm hng s dng k ln nht tho mn bt ng thc vi mi a,b,c l cc
s khng m c tng bng 3.

) (

a3 + b3 + c3 + kabc 3 + k .
Bi 3. Chng minh vi a,b,c l cc s thc dng c tch bng 1, ta c
a+b b+c c+a
1 1 1

+
+
+ 6 2 a + b + c + + + .
c
a
b
a b c

Bi 4. Cho a,b,c l cc s thc dng. Chng minh


a 2 + bc
3

abc b + c
2

+3

b 2 + ca

abc c + a
2

+3

c 2 + ab

abc a + b
2

9
.
a+b+c

Bi 5. Cho a,b,c l cc s thc dng c tng bng 3. Chng minh


1
1
1
3
+
+
.
9 ab 9 bc 9 ca 8
Bi 6. Cho a,b,c l cc s thc dng tho mn iu kin a 2 + b 2 + c 2 =
3.
Chng minh rng 5 ( a + b + c ) +

3
18 .
abc

Bi 7. Cho a,b,c l cc s thc khng m tho mn iu kin a 2 + b 2 + c 2 =


3.
Chng minh rng 21 + 18abc 13 ( ab + bc + ca ) .
Bi 8. Cho a,b,c l cc s thc khng m tho mn iu kin a 2 + b 2 + c 2 =
3.
Chng minh rng

1
1
1
+
+
1.
5 2ab 5 2bc 5 2ca

D. HNG DN GII P S
Bi 1. Bt ng thc tng ng vi

5
(a + b + c)
) (
)
3
10
( a + b + c ) + 2abc 2ab ( a + b ) + 2bc ( b + c ) + 2ca ( c + a )
3
2 a3 + b3 + c3 + 3 a 2 + b 2 + c 2 ( a + b + c ) + 12abc
3

Bt ng thc trn l tng ca hai bt ng thc

691

www.TaiLieuLuyenThi.com
Khm ph t duy K thut gii bt T Bi ton Max Min ng Thnh Nam

(
2(a

)
+ c ) + 6abc 2ab ( a + b ) + 2bc ( b + c ) + 2ca ( c + a )

4 3
a + b3 + c3 4abc
3
3

+ b3

Bt ng thc c chng minh. ng thc xy ra khi v ch khi a= b= c=

1
.
3

3
27
15
,c = 0 k + 3
k .
2
4
4
15
Ta chng minh k = l gi tr cn tm.
4
Tht vy bt ng thc tr thnh
15
27
a3 + b3 + c3 + abc
4
4

Bi 2. Cho a = b =

(
3( a

4 a 3 + b3 + c3 + 15abc ( a + b + c )
3

+ b3 + c3 + 3abc 3 ab ( a + b ) + bc ( b + c ) + ca ( c + a )

y chnh l bt ng thc Schur bc 3.


x
y
z
.
=
,b =
,c
y
z
x

Bi 3. Theo gi thit tn ti cc s thc dng x,y,z sao cho=


a
Bt ng thc tr thnh
x3 + y 3 + z 3 + 3 xyz xy ( x + y ) + yz ( y + z ) + zx ( z + x ) .

y chnh l bt ng thc Schur bc 3.


Bi 4. S dng bt ng thc AM GM ta c

abc b 2 + c 2
a + bc
2

a 2 + bc

abc b + c
2

b+c+

a b2 + c2
a + bc
3
2

ab ( a + b ) + bc ( b + c ) + ca ( c + a )
=
3 a 2 + bc

3 a 2 + bc

ab ( a + b ) + bc ( b + c ) + ca ( c + a )

Tng t cho 2 cn thc cn li

3 a 2 + b 2 + c 2 + ab + bc + ca

V ta ch cn chng minh

ab ( a + b ) + bc ( b + c ) + ca ( c + a )

a3 + b3 + c3 + 3abc ab ( a + b ) + bc ( b + c ) + ca ( c + a ) .

Bt ng thc cui chnh l bt ng thc Schur bc 3.


692

9
a+b+c

www.TaiLieuLuyenThi.com
Cty TNHH MTV DVVH Khang Vit

Bi 5. Quy ng v rt gn vi iu kin a + b + c =
3 bt ng thc cho tng
ng vi a 2b 2 c 2 19abc + 33 ( ab + bc + ca ) 81 0 .
t p = a + b + c = 3; q = ab + bc + ca; r = abc .
Ta cn chng minh r 2 19r + 33q 81 0 .
Theo bt ng thc Schur bc ba ta c

p 4 p q2

r max 0;
9

12 q
) =
.
max 0;
2

+ Nu q 2 3 33q + r 2 > 33q > 100 19r + 81 bt ng thc ng.


+ Nu q 2 3 r

12 q 2
q 3( 4 r ) .
3

Vy ta ch cn chng minh r 2 19r + 33 3 ( 4 r ) 81 0


33 3 ( 4 r ) 81 + 19r r 2

332.3 ( 4 r ) 81 + 19r r 2

(1 r ) r 3 37 r 2 + 162r + 6507 0
3

a+b+c
1.
Bt ng thc cui ng v r
=
3

Bt ng thc c chng minh. ng thc xy ra khi v ch khi a= b= c= 1 .

Bi 7. t t = a + b + c,

3 t 3 ab + bc + ca =

t2 3
.
2

Theo bt ng thc Schur bc 3 ta c

( a + b + c )3 + 9abc 4 ( a + b + c )( ab + bc + ca ) 9abc 2t ( t 2 3) t 3 =

+ Nu t 6 ab + bc + ca
+ Nu

t 3 6t .

3
13.3
21 + 18abc 13 ( ab + bc + ca ) 21
>0.
2
2

6 t 3 21 + 18abc 13 ( ab + bc + ca )

21 + 2 t 6t
3

13 t 2 3
2

2
4t 3 13t 2 24t + 81 ( t 3) 4t t + 27
=
=
0, t 3
2
2
Bt ng thc c chng minh. ng thc xy ra khi v ch khi a= b= c= 1 .

693

www.TaiLieuLuyenThi.com
Khm ph t duy K thut gii bt T Bi ton Max Min ng Thnh Nam

CH 3: K THUT DN BIN
A. NI DUNG PHNG PHP
Mt bt ng thc i xng 3 bin a,b,c ta k hiu l P ( a, b, c ) ta phi chng
minh P ( a, b, c ) 0 .
Dn bin chnh l mt cch tch bt ng thc cn chng minh thnh hai bt
ng thc n gin hn tc l nh gi P (a, b, c) qua mt biu thc trung gian
b2 + c2 b2 + c2
b + c b + c
chng hn P a, bc , bc , P a,
,
,
P
a
,
,

2
2
2
2

,... sau

nh gi biu thc trung gian.


Nh vy tm tt theo 2 bc nh sau :
Bc 1 : Chng minh P ( a, b, c ) PTG v cn sp th t cc bin bt ng thc ng.
Bc 2 : Chng minh PTG 0 .
Thng thng thc hin bc 2 trc kim tra tnh ng ca phng php
dn bin, sau thc hin bc 1, ch sp th t cc bin chng hn
a = min {a, b, c} hoc a = max {a, b, c} .
Di y ti trnh by mt s phng php dn bin c th ph thuc vo iu
kin bi ton.
1. Nu iu kin bi ton cho tng 3 s a + b + c =
k
b+c b+c
Ta tm cch nh gi P ( a, b, c ) P a,
,
0
2
2

b+c b+c
k a k a
,
,
P
=
a,
P a,
0
2
2
2
2

Khi ta cn sp th t a = min {a, b, c} hoc a = max {a, b, c} .


V d 1. Cho a, b, c l cc s thc dng tho mn iu kin a + b + c = 1.
1 1 1
+ + + 48(ab + bc + ca ) 25 .
a b c
Li gii
1 1 1
Ta cn chng minh: P (a, b, c) = + + + 48(ab + bc + ca ) 25 0 (1) .
a b c

Chng minh rng

Khng mt tnh tng qut, c th gi s a = max {a, b, c} .


chng minh (1), ta s ln lt chng minh
b+c b+c
i) P (a, b, c) P (a,
,
) vi a, b, c > 0, a + b + c = 1, a = max {a, b, c} .
2
2
694

www.TaiLieuLuyenThi.com
Cty TNHH MTV DVVH Khang Vit

ii) P ( a, t , t ) 0 vi a + 2t =
1.
chng minh i), ta xt
P ( a , b, c ) P ( a ,

4
b+c b+c 1 1
b+c
+ 48 bc
,
)= +

2
2
b c b+c
2

1
(b c) 2
=
12
bc(b + c)

(b + c ) 2 .
1
2
V a max {a, b, c} a , b + c bc ( b + c )
=
3
3
4
27
1
27
Suy ra
12
12 > 0 .
bc(b + c)
2
3

Vy i) c chng minh.
chng minh ii), ta c
1 2
+ + 48(2at + t 2 ) 25
a t
1
2
=
+ + 48(2(1 2t )t + t 2 ) 25
1 2t t

P ( a, t , t ) =

2(4t 1) 2 (3t 1) 2
0
(1 2t )t

1
1
hoc t = tng ng vi
3
4
1
1
1
hoc a= b= c=
.
a=
, b= c=
2
4
3
Bt ng thc c chng minh.
Nhn xt.
1) Khi thc hnh phng php dn bin, nn bt u t bt ng thc ii) trc vi
cc l do sau:
+ Tm c cc im nghi vn xy ra du bng. Bit c im xy ra du bng,
chng ta c th tm c cc cch tip cn thch hp.
+ Nu khng chng minh c ii) th vic dn bin l v ch. V vy phi lm
bc ny trc.
b+c b+c
2) Bt ng thc P (a, b, c) P (a,
,
) ni chung khng ng vi mi a, b, c.
2
2
S dng tnh i xng ca bt ng thc, ta c th sp xp th t a, b, c bt
ng thc ny ng.

Du bng xy ra khi v ch khi t =

695

www.TaiLieuLuyenThi.com
Khm ph t duy K thut gii bt T Bi ton Max Min ng Thnh Nam

3) Vic chn gi tr dn bin n ph thuc vo biu thc ca P v iu kin


rng buc. Trong trng hp bi ton trn, do c iu kin a + b + c = 1 nn ta
bt buc phi dn bin n cc bin m iu kin ny khng thay i.
4) Kho lo nh gi iu kin ca bin ta hon ton dn c bin v chng minh
bt ng thc trong trng hp hai bin bng nhau(xem v d 2).
V d 2. Cho a,b,c l cc s thc dng. Chng minh rng
a 2 + b 2 + c 2 + abc + 5 3 ( a + b + c ) .

Li gii
Ta cn chng minh P (a, b, c) = a 2 + b 2 + c 2 + abc + 5 3 ( a + b + c ) 0 .
Gi s c l s nh nht trong ba s a, b, c. Xt hiu

P (a, b, c) P ( ab , ab , c) = a b

) (
2

a+ b

a + b a + b (a b) (2 c)
,
,c =
P ( a , b, c ) P
2
4
2

+ Nu c 1 a + b 2 P (a, b, c) P ( ab , ab , c) .
a+b a+b
+ Nu c < 1 P(a, b, c) P
,
,c .
2
2

Do vy ta ch cn chng minh bt ng thc trong trng hp hai s bng nhau.

P ( ab , ab , c)= t 2 ( c + 2 ) 6t + c 2 3c + 5, t=

ab .

Ta c 't =9 ( c + 2 ) c 2 3c + 5 = ( c 1) ( c + 1) 0 .
2

iu chng t bt ng thc c chng minh. ng thc xy ra khi v ch


khi a= b= c= 1 .
Nhn xt. Ngoi ra ta c th chng minh n gin bng Nguyn l Dirichlet.
Trong ba s (a 1),(b 1),(c 1) lun c hai s cng du gi s l

( a 1)( b 1) 0 ab a + b 1 abc ac + bc c .
Vy ta ch cn chng minh a 2 + b 2 + c 2 + 5 + ac + bc c 3 ( a + b + c ) .
Ta c VT

1
( a + b )2 2ab + ( c 3)( a + b ) + c 2 4c + 5 0 .
2

Ch a +b =
( c 1) 0 ta c pcm.
( c 3) 2 c 2 4c + 5 =
2

2. Nu iu kin bi ton cho tch ba s bng 1

Ta tm cch nh gi P ( a, b, c ) P a, bc , bc 0
696

www.TaiLieuLuyenThi.com
Cty TNHH MTV DVVH Khang Vit

P a,=
bc , bc
P 2 , x, x 0 vi x = bc .
x

Khi ta cn sp th t a = min {a, b, c} hoc a = max {a, b, c} .


b
Ch . Mt s trng hp dn bin v f (ta, b / t , c) vi t ;1 .
a
V d 1. Cho a,b,c l cc s thc dng c tch bng 1.

1 1 1
6
+ + +
5.
a b c a+b+c
Li gii
1 1 1
6
Ta cn chng minh: P (a, b, c) = + + +
5 0 (1) .
a b c a+b+c

Chng minh rng

Khng mt tnh tng qut =


gi s a max {a, b, c} a bc .
chng minh bt ng thc (1) ta thc hin chng minh 2 bt ng thc sau:

i) P ( a, b, c ) P a, bc , bc vi abc
= 1,=
a max {a, b, c} .
1

ii) P 2 , x, x 0 vi x = bc .
x

Chng minh bt ng thc i)


Ta c:

P (a, b, c) P (a, bc , bc ) =
=

b c

bc

(
(

6 2 bc b c
1 1
2
+
+
b c
bc ( a + b + c ) a + 2 bc

Do a bc , b + c 2 bc

1 1 1
6
2
6
1

+ + +
+
+

a b c a+b+c a
bc a + 2 bc

b c

(a + b + c)(a + 2

bc

( a + b + c ) a + 2 bc 6bc

bc ( a + b + c ) a + 2 bc

b c

( a + b + c ) a + 2 bc 6bc 3 bc a + 2 bc 6bc = 3a bc > 0


1

Do P (a, b, c) P (a, bc , bc ) =
f 2 , x, x vi x = bc .
x

697

www.TaiLieuLuyenThi.com
Khm ph t duy K thut gii bt T Bi ton Max Min ng Thnh Nam

Chng minh bt ng thc ii)


Ta ch cn chng minh x 2 +

2
x

2x +

1
x2

5 0 .

( x 1) 2 x 4 + 4 x3 4 x 2 x + 2 0 (lun ng).
2

Bt ng thc c chng minh.


ng thc xy ra khi v ch khi x =1 a =b =c =1 .
3. Nu iu kin bi ton cho tng bnh phng ca 3 s a 2 + b 2 + c 2 =
k

b2 + c2 b2 + c2
0
Ta tm cch nh gi P ( a, b, c ) P a,
,

2
2

b2 + c2 b2 + c2
k a2 k a2
P a,
,
,
P =
a,

2
2
2
2

Khi ta cn sp th t a = min {a, b, c} hoc a = max {a, b, c} .


V d 1. Cho a,b,c l cc s thc khng m tha mn a 2 + b 2 + c 2 =
3.
Chng minh rng a + b + c a 2b 2 + b 2 c 2 + c 2 a 2 .
Li gii
Khng mt tnh tng qut gi s
a b c a 1, b 2 + c 2 2 b + c b 2 + c 2 =2 .

Ta cn chng minh:

f ( a , b, c ) = a + b + c a 2 b 2 b 2 c 2 c 2 a 2 = a + b + c a 2 b 2 + c 2 b 2 c 2 0 .

Xt

b2 + c2 b2 + c2
f ( a , b, c ) f a ,
,

2
2

b2 + c2
b2 + c2
2
2
=b+c2
b c +
2

(b

c2
4

( b + c )2 2 ( b 2 + c 2 )

b + c + 2 b2 + c2

( b + c )2
1
=
( b c )
4
b + c + 2 b2 + c2

698

www.TaiLieuLuyenThi.com
Cty TNHH MTV DVVH Khang Vit

( b + c )2
4

( b + c )3 + ( b + c )2

b + c + 2 b2 + c2

2 b2 + c2 4
=
4 b + c + 2 b 2 + c 2

2 2 + 2 2.2 4

4 b + c + 2 b 2 + c 2

>0

Vy ta ch cn chng minh

b2 + c2 b2 + c2
,
f a,

2
2

3 a2 3 a2
f a,
,

2
2

) 14 (3 a )

2 2

a + 2 3 a2 a2 3 a2 +

3
3
2
( a 1) ( a + 1)
4
3 a + 2 3 a2

0 (lun ng).

3
3
3
3 3 2
2
( a + 1) =
a + 2a 0, a [ 0;1] .
( a + 1)2
4
4
4 4
3 a + 2 3 a2

Bt ng thc c chng minh. ng thc xy ra khi v ch khi a= b= c= 1 .


4. Dn bin bng hm s
Di y ti trnh by k thut dn bin bng hm s dng n gin nht khi
iu kin bi ton cho tng cc s khng i (a + b + c =
k ) du bng t ti
mt bin bng 0 hoc 2 s bng nhau.
Khi sp th t li bin s gi s a b c lc du bng t ti
2

b+c
c =0 bc =0 hoc b =c bc =
.
2

t= b + c
t2
nn ta t
.
,0 s
4
s = bc
a bt ng thc cn chng minh v dng hm s f(s) v ch ra rng f(s)
t2
nghch bin hoc ng bin trn 0; .
4
699

www.TaiLieuLuyenThi.com
Khm ph t duy K thut gii bt T Bi ton Max Min ng Thnh Nam

+ Nu du bng t ti mt s bng 0 ta cn ch ra rng f(s) nghch bin tc


f ( s ) f (0) , lc a v chng minh bt ng thc vi 2 bin a v t vi
a+t =
k . Vy bi ton a v chng minh bt ng thc mt bin s.

t2
tc f(s) l hm ng bin.
4
V d 1. Cho a,b,c l cc s thc khng m c tng bng 1. Chng minh

+ Nu du bng t ti 2 s bng nhau khi b = c s =

(a

)(

)(

+ b2 b2 + c2 c2 + a 2

Li gii
Khng mt tnh tng qut gi s a b c .
Gi P l biu thc v tri ta c

1
.
32

2
2
P = ( b + c ) 2bc . a 4 + a 2 ( b + c ) 2bc + b 2 c 2 .

t2
t b + c = t , bc = s, s 0; ta c
4

P = f ( s) = t 2 2s a 4 + a 2 t 2 2s + s 2 .

t2
Xt hm s f ( s ) = t 2 2 s a 4 + a 2 t 2 2 s + s 2 vi s 0; ta c

)(

f '( s ) =
2 a 4 + a 2 t 2 2 s + s 2 + 2 t 2 2 s s a 2 .

Ch s

t2 1 2
1
= a 2 s a 2 0 f '( s ) 0 .
4 4 3
9

t2
Do f(s) l hm nghch bin trn on 0; .
4

Suy ra P = f ( s ) f (0) = t 2 a 4 + a 2t 2 = a 2t 2 a 2 + t 2

1
2
2
= t 2 (1 t ) t 2 + (1 t )

32
2
2
2
Ch kho st hm mt bin g (t )= t 2 (1 t ) t 2 + (1 t ) trn on 0; ta

3
c kt qu trn.
Bt ng thc c chng minh.
1
ng thc xy ra khi v ch khi a= b=
, c= 0 hoc cc hon v.
2

700

www.TaiLieuLuyenThi.com
Cty TNHH MTV DVVH Khang Vit
2

c
c

Cch 2: Gi
=
s c min {a, b, c} b + c b + ; a 2 + c 2 a + v
2
2

c
c

a 2 + b2 a + + b + .
2
2

c
c
t x = a + , y = b + x + y = 1; P x 2 y 2 x 2 + y 2 .
2
2
Khi s dng bt ng thc AM GM cho 2 s dng ta c
Px y

2 2

xy
1 x+ y
x +
y
.2 xy x 2 + y 2
=

2
2 2
2

2 xy + x 2 + y 2
1
.
=

2
32

Bi tp tng t
Cho a,b,c l cc s thc khng m c tng bng 1. Chng minh

(a

)(

)(

1
.
256
V d 2. Cho a,b,c l cc s thc dng c tng bng 3. Chng minh
3

+ b3 b3 + c 3 c 3 + a 3

bc ca ab 9
21
+ +
+ abc .
a
b
c 4
4
Li gii

Gi P l biu thc v tri v gi s a = max {a, b, c} ta c

2
b2 + c2 9
bc
s a t 2s 9
P =+ a
+ abc =+
+ as .
bc 4
a
a
s
4

2
s a t 2s 9
Xt hm s f ( s ) =
+
+ as trn na khong
a
s
4

t2
0; ta c
4

t2
2
2
2
9a 2 + 4 4a 2 t
s
a
a
t
9
+
4

4
1 at
9
f '( s ) = 2 + a =
16
a s
4
4as 2
4as 2

(9a + 4) t 64a
=
t.
2

64as 2

t.

2 9a 2 + 4 64a 2
64as 2

8 46a 2
=
t.
<0
64as 2

t2
Vy f(s) l hm nghch bin trn 0; suy ra
4

2
2
t2 t2
9a ( 3 a )
9at 2 ( 3 a )
.
+ 2a +
=
+ 2a +
P = f (s) f =
4 4a
16
4a
16

701

www.TaiLieuLuyenThi.com
Khm ph t duy K thut gii bt T Bi ton Max Min ng Thnh Nam

Ch .

2
9a ( 3 a )
( 3 a ) 2 + 2a =
+

9 ( a 1) ( a 2 )

4a
16
Bt ng thc c chng minh.

16a

21 21
.
4
4

1 1
ng thc xy ra khi v ch khi ( a; b; c ) = (1;1;1) ; 2; ; hoc cc hon v.
2 2

V d 3. Cho a,b,c l cc s thc dng tho mn iu kin a 2 + b 2 + c 2 =


3.
Chng minh rng a + b + c +

7 1 1 1 81
.
+ +
20 a b c 20

Li gii
Khng mt tnh tng qut gi s a = max {a, b, c} .
t
.
2
Gi P l biu thc v tri ta c

t t =
b2 + c2 ; s =
bc, s

7 1
b 2 + c 2 + 2bc
+

20 a
bc

7 1
t + 2s
=a + t + 2 s + +

20 a
s
P =a + b 2 + c 2 + 2bc +

Xt hm s f ( s ) =a + t + 2 s +

7 1
t + 2s
+
ta c
20 a
s

7 (t + s )
20 s 2 7 ( t + s )
1
f '( s ) =

=2
<0
t + 2 s 20 s 2 t + 2 s
20 s t + 2 s
t
20 s 2 7 ( t + 2 s ) 20 s 2 7.4=
s 4 s ( 5s 7 ) < 0, s 1
2
V vy f(s) l hm nghch bin do

7 1 2 2
t
2t + +
P=
f ( s ) f =+
a

20 a
t
2
7 1
2 2 81
= 3 t + 2t +
+

20 3 t
t 20
V d 4. Cho a,b,c l cc s thc khng m c tng bng 3. Chng minh

(a

)(

Li gii
Khng mt tnh tng qut gi s a b c .
702

+ b3 + c3 a3b3 + b3c3 + c3 a3 36 ( ab + bc + ca ) .

www.TaiLieuLuyenThi.com
Cty TNHH MTV DVVH Khang Vit

Vit li bt ng thc cho di dng

3
3
P = a3 + ( b + c ) 3bc ( b + c ) . a3 ( b + c ) 3bc ( b + c ) + b3c3 36a ( b + c ) 36bc 0

t2
t b + c = t , bc = s, s 0; ta c
4

P=

(a

)(

+ t 3 3ts a3t 3 3a3ts + s 3 36at 36 s .

Xt hm s f ( s ) =

t2
a3 + t 3 3ts a3t 3 3a3ts + s3 36at 36s vi s 0; ta c
4

)(

) (

)(

f '( s ) =
3t a3t 3 3a3ts + s 3 + a3 + t 3 3ts 3s 2 3a 3t 36 0 .

V s 2 a3t 0 vi t 2, a 1, s 1 .
t2
Do f(s) l hm nghch bin trn 0; suy ra
4

3
f( s ) f(0) = a3 + t 3 a3t 3 36at = ( a + t ) 3at ( a + t ) a3t 3 36at

= 27 9t ( 3 t ) t 3 ( 3 t ) 36t ( 3 t )
3

Vy ta ch cn chng minh 27 9t ( 3 t ) t 2 ( 3 t ) 36 0, t [ 0;2] .


2

9
t x = t ( 3 t ) , ta chng minh
4

( 27 9 x ) x 2 36 x3 3x 2 + 4 0 ( x 2 )2 ( x + 1) 0 (lun ng).
Bt ng thc c chng minh.
ng thc xy ra khi v ch khi =
a 2,=
b 1,=
c 0 hoc cc hon v.
V d 5. Cho x,y,z l cc s thc khng m. Chng minh
x4 ( y + z ) + y 4 ( z + x ) + z 4 ( x + y )

1
( x + y + z )5 .
12

Li gii
Bt ng thc c dng thun nht bc 5 ta chun ho x + y + z =
1.
Ta phi chng minh
x4 ( y + z ) + y 4 ( z + x ) + z 4 ( x + y )

1
.
12

Gi P l biu thc v tri v=


gi s x max { x, y, z} x

1
ta c
3
703

www.TaiLieuLuyenThi.com
Khm ph t duy K thut gii bt T Bi ton Max Min ng Thnh Nam

) (

P= x 4 ( y + z ) + yz y 3 + z 3 + x y 4 + z 4

3
4
2
= x 4 ( y + z ) + yz ( y + z ) 3 yz ( y + z ) + x ( y + z ) + 2 y 2 z 2 4 ( y + z ) yz

= tx 4 + xt 4 + ( 2 x 3t ) s 2 + t 2 ( t 4 x ) s

t2
t= y + z
Vi
, s 0; .
4
s = yz
t2
Xt hm s f ( s ) = tx 4 + xt 4 + ( 2 x 3t ) s 2 + t 2 ( t 4 x ) s trn on 0; ta c
4

f '( s ) =
2 ( 2 x 3t ) s + t 2 ( t 4 x ) 2 ( 2 x 3t ) .

t2 2
t3
+ t (t 4x ) =
3 xt 2 0 .
4
2

t2
Do f(s) l hm nghch bin trn on 0; .
4

V vy P = f ( s ) f (0) = tx 4 + xt 4 = xt x3 + t 3 = x (1 x ) x3 + (1 x )

) 121 .

Bt ng thc c chng minh.


3+ 6 3 6 3 6 3+ 6
;
;
ng thc xy ra khi v ch khi ( x; y; z ) = 0;
; 0;

6
6
6
6

v cc hon v ca mi b s tng ng.


V d 6. Cho x,y,z l cc s thc khng m. Chng minh

(8 x

)(

)(

+ yz 8 y 2 + zx 8 z 2 + xy ( x + y + z ) .
6

Li gii
Bt ng thc c dng thun nht bc 6 ta hun ho x + y + z =
1.

)(

)(

Ta phi chng minh 8 x 2 + yz 8 y 2 + zx 8 z 2 + xy 1 .


1
Khng mt tnh tng qut=
gi s x max { x, y, z} x .
3
Gi P l biu thc v tri ca bt ng thc ta c
P=

704

(8 x
= (8 x
= (8 x

2
2
2

)
(
)
+ yz ){64 y z + 8 x ( y + z ) 3 yz ( y + z ) + x

+ s )( 64 s + 8 xt 24 xts + x s )

+ yz 64 y 2 z 2 + 8 x y 3 + z 3 + x 2 yz

2 2

yz

www.TaiLieuLuyenThi.com
Cty TNHH MTV DVVH Khang Vit

t2
t= y + z
Vi
, s 0; .
4
s = yz
t2
Xt hm s f ( s ) = 8 x 2 + s 64 s 2 + 8 xt 3 24 xts + x 2 s trn on 0; .
4

)(

)(

f '( s )= 64 s 2 + 8 xt 3 24 xts + x 2 s + 8 x 2 + s 128s + x 2 24 xt

f ''( s )= 128s 24 xt + x 2 + 128s + x 2 24 xt + 128 8 x 2 + s

171

f '''( s ) = 384 f ''( s ) f ''(0) = 1026 x 2 48 xt = 48 x


xt > 0
8

t2
t2
Do f(s) l hm li trn on 0; do f(s) t max ti 0 hoc
.
4
4
3

x + 1 x 2
Ta c f (0) = 64 x t = 64 x (1 x ) 64
= 1 ; v
2

3 3

t2
f =
4

2 t 2 4
x 2t 2
3
+
+
+
8
x
4
t
2
xt

4
4

x 2 (1 x )
(1 x )2
4 (1 x )4 + 2 x (1 x )3 +
1
= 8x2 +

4
4

Vy ta c max P bng 1. Bt ng thc c chng minh.


1 1 1 1 1
Du bng t ti ( x; y; z ) = ; ; ; 0; ; hoc cc hon v ca mi b s
3 3 3 2 2
tng ng.
Bi tp tng t
Cho x,y,z l cc s thc khng m tho mn iu kin xy + yz + zx > 0 .

Chng minh rng

1
8 x 2 + yz

1
8 y 2 + zx

1
8 z 2 + xy

3
.
x+ y+z

Ch . Bi ton ny l h qu ca bt ng thc trn.


Nh vy qua vic dn bin bng hm s n gin qua tng v tch nh trn
cc bi ton c x l kh n gin. Bn c th so snh li gii cc bi ton trn
vi cc cch dn bin cng nh phng khc thy c hiu qu u vit ca
phng php ny.

705

www.TaiLieuLuyenThi.com
Khm ph t duy K thut gii bt T Bi ton Max Min ng Thnh Nam

B. BI TP CHN LC

)(

)(

Bi 1. Cho tam gic ABC chng minh rng 1 + cos 2 A 1 + cos 2 B 1 + cos 2 C
Li gii
Khng mt tnh tng qut=
gi s C min { A, B, C} 0 < C

)(

A+ B

Ta chng minh 1 + cos A 1 + cos B 1 + cos 2


.
2

Tht vy bt ng thc cho tng ng vi


sin 2

A B
6cos C cos ( A B ) 1 0 .
2

1
Bt ng thc ng bi v 6cos C cos ( A B ) 1 6. 2 = 1 > 0 .
2
T suy ra

)(

)(

A+ B

2
1 + cos A 1 + cos B 1 + cos C 1 + cos 2
1 + cos C .
2

Vy ta ch cn chng minh
2

125

2 A+ B
2
1 + cos
1 + cos C
2
64

125
C

1 + sin 2 1 + cos 2 C
2
64

125
1 cos C
2
1 +
1 + cos C
2
64

( 2cos C 1) 4cos 2 C 20cos C + 19 0


2

2
( 2cos C 1) 4cos 2 C 1 + 20 (1 cos C ) 0

Bt ng thc cui lun ng do 4cos 2 C 1 0;1 cos C > 0 .


Bt ng thc c chng minh.
ng thc xy ra khi v ch khi A= B= C= 600 .
Bi 2. Cho x,y,z l cc s thc khng m tho mn iu kin x + y + z =
1.

Chng minh rng xy + yz + zx 12 x3 + y 3 + z 3

706

)( x

y + y 2 z 2 + z 2 x2 .

2 2

125
.
64

www.TaiLieuLuyenThi.com
Cty TNHH MTV DVVH Khang Vit

Li gii
1
Khng mt tnh tng qut=
gi s x max { x, y, z} x .
3
Ta cn chng minh

xy + yz + zx 12 x3 + y 3 + z 3

)( x

y + y 2 z 2 + z 2 x2 0 .

2 2

Gi P l biu thc v tri ta c


3
2
P = x ( y + z ) + yz 12 x3 + ( y + z ) 3 yz ( y + z ) . x 2 ( y + z ) 2 x 2 yz + y 2 z 2

)(

= xt + s 12 x3 + t 3 3ts x 2t 2 2 x 2 s + s 2

t2
t= y + z
Vi
, s 0; .
4
s = yz

)(

Xt hm s f ( s ) = xt + s 12 x3 + t 3 3ts x 2t 2 2 x 2 s + s 2 lin tc trn on


t2
0; ta c
4

)(

f '( s ) =1 + 36t x 2t 2 2 x 2 s + s 2 + 24 x3 + t 3 3ts x 2 s 0 do x 2 s 0 .


t2
Do f(s) l hm ng bin trn on 0; .
4
V vy

3
2
f ( s ) f (0) = xt 12 x3 + t 3 x 2t 2 = x (1 x ) 12 x3 + (1 x ) x 2 (1 x )

=x (1 x ) 1 12 x (1 x ) (1 3x (1 x ) )
2
1
= x (1 x ) 6 x (1 x ) 1 0, x ;1
3
Bt ng thc c chng minh.

x (1 x ) =
0

1 x= 1, y= z= 0
6 x (1 x ) =

3+ 3
3 3

0
yz
=
,y
,z 0.
ng thc xy ra khi v ch khi

=
=
=
x
6
6
x + y + z =
1

3+ 3
3 3

, y 0,=
=
=
z
x 1
x
6
6
3

707

www.TaiLieuLuyenThi.com
Khm ph t duy K thut gii bt T Bi ton Max Min ng Thnh Nam

Bi tp tng t
Cho x,y,z l cc s thc khng m. Chng minh
9
x3 + y 3 + z 3 ( xy + yz + zx ) .
( x + y + z )5
4 6 9
Bi 3. Cho x,y,z l cc s thc dng c tng bng 1. Chng minh

x
y
z
9
+
+
.
1 yz 1 zx 1 xy 8

Li gii
Bt ng thc v tri l hin nhin theo C S tht vy

( x + y + z)
x
y
z
1
+
+

=
1.
1 yz 1 zx 1 xy x (1 yz ) + y (1 zx ) + z (1 xy ) 1 3xyz
2

Ta chng minh bt ng v phi.


Gi P l biu thc v tri v gi s x = max { x, y, z} ta c

y + z x ( y + z ) 2 yz
x
=
P
+
1 yz
x 2 yz x ( y + z ) + 1

t x t 2 2s
x
=
+ 2
1 s
x s xt + 1

t2
t= y + z
vi
, s 0; .
4
s = yz

t x t 2 2s
x
Xt hm s f=
lin tc trn
( s)
+ 2
1 s
x s xt + 1

t2
0; ta c
4

(
(

x x 2t 3 xt + 2
x
x3t 3 x 2t + 2 x
x
=2 +
>0
f '( s ) =2 +
2
2
(1 s )
(1 s )
x 2 s xt + 1
x 2 s xt + 1

do x 2t 3 xt + 2 > 2 3 xt 2

3
3 5
( x + t )2 = 2 = > 0 .
4
4 4

t2
Vy f(s) l hm ng bin trn 0; .
4

708

www.TaiLieuLuyenThi.com
Cty TNHH MTV DVVH Khang Vit

xt 2
2

t2
x
Do f ( s ) f = 2 + 2 2
4
x t
1 t
xt + 1
4
4
4x

4 (1 x )

= 2.

4 (1 x ) 2 x (1 x )

x 2 (1 x ) 4 x (1 x ) + 4
2

3 x3 5 x 2 + 3 x + 3
x 4 3 x3 + x 2 x 6

Ta chng minh 2.

3 x3 5 x 2 + 3 x + 3

x 3x + x x 6
Bt ng thc lun ng.
4

9
2
( 3 x 1) x 2 + 3 x 6 0 .
8

1
.
3

Bt ng thc c chng minh. ng thc xy ra khi v ch khi x= y= z=


Bi tp tng t
Cho x,y,z l cc s thc dng tho mn iu kin x 2 + y 2 + z 2 =
1.
9
1
1
1
27
.

+
+

4 1 + xy 1 + yz 1 + zx 4 ( x + y + z )2

Chng minh rng

Bi 4. Cho a,b,c l cc s thc dng tha mn abc = 1 . Chng minh rng

( a + b )( b + c )( c + a ) 4 ( a + b + c 1) .
Li gii
Khng mt tnh tng qut ta c th gi s a b c
a 1 v ta cn chng minh

( a + b )( a + c )( b + c ) 4 ( a + b + c 1) 0
f (a, b, c) = a 2 + a ( b + c ) + bc ( b + c ) 4 ( a + b + c 1) 0 .

Xt
f (a, b, c) f (a, bc , bc )

) (

2
= a 2 b + c 2 bc + a ( b + c ) 4bc + bc b + c 2 bc 4 b + c 2 bc

b c

a 2 + a b + c + 2 bc + bc 4 0

a 2 + a b + c + 2 bc + bc 4 = a 2 + 2 a + a ( b + c ) + bc 4
a 2 + 2 a + 2a bc + bc 4 = a 2 + 4 a + bc 4 1 + bc > 0
709

www.TaiLieuLuyenThi.com
Khm ph t duy K thut gii bt T Bi ton Max Min ng Thnh Nam

Do a 1 .
1

V vy ta ch cn chng minh f (a, bc , bc ) 0 f 2 , x, x 0 vi x = bc


x

1
1
1

2 x 2 + x 2 + x 4 2 x + 2 1 0
x
x
x

)
+ 1) 0

x3 + 1 2 x + 2 x 4 x3 x 6 4 x 4 + 4 x3 2 x + 1 0
( x 1)

(x

+ 2 x3 x 2

2
2

( x 1) x 2 1 + 2 x3 + x 2 0 (lun ng).

ng thc xy ra khi v ch khi x =1 a =b =c =1 .


Cch 2: Ta c:

( a + b )( b + c )( c + a ) = ( a + b + c )( ab + bc + ca ) abc = ( a + b + c )( ab + bc + ca ) 1 .
Mt khc:

( ab + bc + ca )2 3abc ( a + b + c )= 3 ( a + b + c ) ab + bc + ca

3( a + b + c ) .

Vy bi ton c chng minh nu bt ng thc sau ng:

( a + b + c ) 3 ( a + b + c ) 1 4 ( a + b + c 1) .
Tht vy t t =

a + b + c , t 3 . Xt hm s f (t ) =

Ta c f=
'(t ) 3 3t 2 8t > 0, t 3 nn

3t 3 4t 2 + 3 vi t 3 .

f (t ) ng bin trn 3; + hay

f (t ) f ( 3) =
0.

Do ( a + b + c ) 3 ( a + b + c ) 1 4 ( a + b + c 1) .
Bt ng thc c chng minh.
Bi 5. Cho a,b,c l cc s thc dng c tch bng 1. Chng minh
3
1 1 1
a 2 + b2 + c2 + 6 a + b + c + + + .
2
a b c

Li gii
3
1 1 1
t P ( a, b, c ) = a 2 + b 2 + c 2 + 6 a + b + c + + + .
2
a b c

Khng mt tnh tng qut gi s a= min {a, b, c} x=

710

bc 1

www.TaiLieuLuyenThi.com
Cty TNHH MTV DVVH Khang Vit

Ta chng minh P ( a, b, c ) P a, bc , bc

Tht vy

P ( a, b, c ) P a, bc , bc =

1
2

1
2
1

3
1 1
2
b + c 2 bc + +

b c
2
bc
2
2
3
b c 2 b + c 3
bc

2
3
b c 8bc 3
bc

( b c )2

) (

b c

( 8 3 3) 0

Mt khc
6
5
4
3
2
1
x 6 x + 12 x 6 x 3 x + 2
P a,=
bc , bc P=
,
x
,
x
2

2 x4
x

2
( x 1)2 x 2 2 x 1 + x 2 + 1

0
4
2x
Bt ng thc c chng minh. ng thc xy ra khi v ch khi a= b= c= 1 .
Bi 6. Cho a,b,c l cc s thc khng m c tng bng 3. Chng minh

)(

)(

1 a 2 a + 1 b2 b + 1 c2 c + 1 7 .

Li gii
Khng mt tnh tng qut=
gi s a max {a, b, c} a 1, b + c 2 .
t2
t= b + c
t
, s 0; ta c
4
s = bc

P=
=
=

(a

(a
(a

2
2

a + 1 b 2 c 2 bc ( b + c ) + b 2 + c 2 + bc ( b + c ) + 1

)
a + 1) s

2
a + 1 b 2 c 2 + ( b + c ) ( b + c ) bc (1 + b + c ) + 1

+ t 2 t s ( t + 1) + 1

(a

Xt hm s f ( s=
)
f '( s=
)

(a

a + 1 s 2 + t 2 t s ( t + 1) + 1 ta c

a + 1 ( 2 s t 1) < 0

v 2s t 1

t2
t 2 2t 2 t ( t 2 ) 2
t 1 =
<0.
2
2
2
711

www.TaiLieuLuyenThi.com
Khm ph t duy K thut gii bt T Bi ton Max Min ng Thnh Nam

t2
t2
Do f(s) l hm nghch bin trn on 0; . V vy f P f (0) .
4
4

Ta c

(a

f (0)=

)(

) (a

a +1 t2 t +1 =

2
a + 1 ( 3 a ) ( 3 a ) + 1 7 .

3
7
V bt ng thc tng ng vi: a ( a 3) a + 0 .
2
4

ng vi mi a [ 0;3] .
t2
Tng t: f =
4

t4

t2
a + 1 + t 2 t ( t + 1) + 1
4
16

t4

t2
2
= ( 3 t ) ( 3 t ) + 1 + t 2 t ( t + 1) + 1 1

16
4

bi v bt ng thc tng ng vi

(a

( t 2 )2 ( t 4 5t 3 + 15t 2 24t + 24 ) 0 (lun ng).

Bi 7. Cho a,b,c l cc s thc khng m tha mn iu kin a + b + c =


3.

)(

)(

Chng minh rng a 2 + a + 1 b 2 + b + 1 c 2 + c + 1 27 .


Li gii
Khng mt tnh tng qut gi s a = min {a, b, c} a 1, b + c = 3 a 2 .
2

b + c 2 b + c
Ta chng minh b + b + 1 c + c + 1
+ 1 .
+
2
2

Tht vy bt ng thc tng ng vi:

)(

1
( b c )2 b2 + c 2 + 6bc + 4b + 4c 4 0 .
16

Bt ng thc lun ng v b 2 + c 2 + 6bc + 4b + 4c 4 4 ( b + c ) 4 > 0 .


2

b + c 2 b + c
Vy ta ch cn chng minh a + a + 1
+ 1 27 .
+
2
2

3 a 2 3 a
a + a + 1
+ 1 27 .
+
2
2

712

www.TaiLieuLuyenThi.com
Cty TNHH MTV DVVH Khang Vit

)(

a 2 + a + 1 a 2 8a + 19

432 .

( a 1) a 4 13a3 + 60a 2 85a 71 0 .


2

Lun ng v

a 4 13a3 + 60a 2 85a 71 = a 4 13a3 85a 11 + 60 a 2 1 < 0, a [ 0;1] .

ng thc xy ra khi v ch khi a= b= c= 1 .


Nhn xt. Li gii trn xut pht t cch dn bin

(a

P (a, b, c)=

)(

)(

+ a + 1 b2 + b + 1 c2 + c + 1 .

b+c b+c
P ( a , b, c ) P a ,
,
(ng) ta ch cn gi s a = min {a, b, c} .
2
2

Bi 8. Cho a,b,c l cc s thc tha mn iu kin a + b + c =


6 v a, b, c 1 .

)(

)(

Tm gi tr ln nht ca biu thc P =a 2 + 2 b 2 + 2 c 2 + 2 .


Li gii
Khng mt tnh tng qut gi s
c = min {a, b, c} c

a+b+c
= 2; a + b = 6 c 4 .
3
2

a + b 2

Ta chng minh a + 2 b + 2
+ 2 .
2

Tht vy bt ng thc tng ng vi :

)(

16 a 2b 2 + 2a 2 + 2b 2 + 4 ( a + b ) + 16 ( a + b ) + 64
4

( a + b ) 16a 2b 2 16 ( a b )
4

2
2
2
( a + b ) 4ab . ( a + b ) + 4ab 16 ( a b )

2
2
( a b ) ( a + b ) + 4ab 16 0

Bt ng thc cui lun ng do ( a + b ) 16 .


2

a + b 2

2
Do P c + 2 .
+ 2 = c + 2
2

6 c 2

.
+ 2
2

6 c 2

Xt hm s f (c) =
c + 2 .
+ 2 trn on [ 0;2] ta c
2

713

www.TaiLieuLuyenThi.com
Khm ph t duy K thut gii bt T Bi ton Max Min ng Thnh Nam

3 5
c 20c 4 + 156c3 552c 2 + 800c 352
8
3
= ( c 2 ) c 2 12c + 44 c 2 6c + 4 0, c [ 0;2]
8

f '(c) =

)(

Do f(c) l hm ng bin trn on [ 0;2] .


Suy ra P f (c) f (2) =
216 . ng thc xy ra khi v ch khi a= b= c= 2 .
Vy gi tr ln nht ca P bng 216 t ti a= b= c= 2 .
Nhn xt. Ch ng thc

( 2 + a )( 2 + b )
2

2
a + b 2
a b)
(
a 2 + 6ab + b 2 16 .
2 +

=
2
16

)(

)(

Vi nhng bi ton c tch i xng k + a 2 k + b 2 k + c 2 vi k > 0 ta nh

gi gia k + a

)( k + b )
2

a + b 2
vi k +
.
2
2

a + b 2
( a b )2 a 2 + b2 + 6ab 8k .
Ta c : k + a k + b k +
=

16
2
Bi 9. Cho a,b,c l cc s thc khng m tha mn iu kin a + b + c =
1.

)(

Tm gi tr ln nht ca biu thc P= ab a 2 + b 2 + bc b 2 + c 2 + ca c 2 + a 2 .


Li gii
t2
Khng mt tnh tng qut gi s a b c t b + c = t , bc = s, s 0; .
4

Khi a = 1 t

b+c t
2
= t v
2
2
3

P= a3 ( b + c ) + a b3 + c3 + bc b 2 + c 2

3
2
= a3 ( b + c ) + a ( b + c ) 3bc ( b + c ) + bc ( b + c ) 2bc

) (

=a3t + a t 3 3st + s t 2 2 s

) (

Xt hm s f ( s ) =a3t + a t 3 3st + s t 2 2 s ta c
f '( s ) = t 2 4 s 3at = t ( t 3a ) 4 s 0 .

714

www.TaiLieuLuyenThi.com
Cty TNHH MTV DVVH Khang Vit

t2
Do f(s) l hm nghch bin trn on 0; .
4

Suy ra f ( s ) f (0) = a3t + at 3 = (1 t ) t + (1 t ) t 3 .


3

3
2
Xt hm s g (t ) = (1 t ) t + (1 t ) t 2 vi t 0; ta c
3

f '(t ) = (1 2t ) ; f '(t ) = 0 t =
3

1
.
2

Ta c f(t) i du t dng sang m khi i qua t =


t=

1
nn f(t) t cc i ti
2

1
1 1
hay P f (t ) f =
.
2
2 8

1
, c= 0 hoc cc hon v.
2
Nhn xt. Vi a,b,c khng m v a + b + c =
k vi chng minh tng t ta c c

ng thc xy ra khi v ch khi a= b=

k4
.
8
Bi 10. Cho a,b,c l cc s thc khng m tha mn iu kin a + b + c =
1.
Tm gi tr ln nht v nh nht ca biu thc
ab a 2 + b 2 + bc b 2 + c 2 + ca c 2 + a 2

P=

b + c +1 c + a +1
Li gii
Tm gi tr ln nht ca P
Vi a,b,c khng m ta lun c
a

P=
=

b,

c
a + b3 + 1
3

c.
b + c +1
c + a +1
a + b3 + 1
Cng theo v ba bt ng thc trn ta c P a + b + c =
1.
ng thc xy ra khi v ch khi mt s bng 1 v hai s bng 0.
Tm gi tr nh nht ca P
S dng bt ng thc C-S ta c
3

a,

( a + b + c )2

) (

) (

b3 + c 3 + 1 c 3 + a 3 + 1 a 3 + b3 + 1 a b3 + c 3 + 1 + b c 3 + a 3 + 1 + c a 3 + b3 + 1

) (

) (

a b + c + b c + a 3 + c a 3 + b3 + 1
3

715

www.TaiLieuLuyenThi.com
Khm ph t duy K thut gii bt T Bi ton Max Min ng Thnh Nam

Vy tm gi tr nh nht ca P ta tm gi tr ln nht ca biu thc

) (

) (

M = a b3 + c 3 + b c 3 + a 3 + c a 3 + b3 .
t2
Khng mt tnh tng qut gi s a b c t b + c = t , bc = s, s 0;
4

Khi a = 1 t

b+c t
2
= t v
2
2
3

M= a3 ( b + c ) + a b3 + c3 + bc b 2 + c 2

3
2
= a3 ( b + c ) + a ( b + c ) 3bc ( b + c ) + bc ( b + c ) 2bc

) (

=a3t + a t 3 3st + s t 2 2 s

) (

Xt hm s f ( s ) =a3t + a t 3 3st + s t 2 2 s ta c
f '( s ) = t 2 4 s 3at = t ( t 3a ) 4 s 0 .

t2
Do f(s) l hm nghch bin trn on 0; .
4

Suy ra f ( s ) f (0) = a3t + at 3 = (1 t ) t + (1 t ) t 3 .


3

3
2
Xt hm s g (t ) = (1 t ) t + (1 t ) t 2 vi t 0; ta c
3

f '(t ) = (1 2t ) ; f '(t ) = 0 t =
3

1
.
2

Ta c f(t) i du t dng sang m khi i qua t =


t=

1
nn f(t) t cc i ti
2

1
1 1
hay M f (t ) f =
.
2
2 8

1
, c= 0 hoc cc hon v.
2
1
1
Tm c GTLN ca M bng t ti a= b=
, c= 0 hoc cc hon v.
8
2
1
8
Suy ra P
.
=
1 9
1+
8
8
1
Vy gi tr nh nht ca P bng t ti a= b=
, c= 0 hoc cc hon v.
9
2

ng thc xy ra khi v ch khi a= b=

716

www.TaiLieuLuyenThi.com
Cty TNHH MTV DVVH Khang Vit

Nhn xt. Vi a,b,c khng m v a + b + c =


k vi chng minh tng t ta c c

ab a 2 + b 2 + bc b 2 + c 2 + ca c 2 + a 2

k4
.
8

Bi tp tng t
Cho a,b,c l cc s thc khng m c tng bng 1. Chng minh
a
1+ b + c
2

b
1+ c + a
2

c
1+ a + b
2

4
.
5

Bi 11. Cho a,b,c l cc s thc khng m tha mn iu kin ab + bc + ca =


1.
1
1
1
5
+
+
.
a+b b+c c+a 2
Li gii

Chng minh rng

Khng mt tnh tng qut gi s a = max {a, b, c} .


t2
t= b + c
1 s
t
, s 0; ta c at + s =1 a =
t
s = bc
4

Gi P l biu thc v tri ta c


1
1
1
1
2a + b + c
+
+
=
+ 2
P=
b + c a + b a + c b + c a + ab + bc + ca
1 s
2
+t
2.
1 2a + t 1
1 t 2s + t + 2
t
=
+
=
+
=
+
2
t a 2 + 1 t 1 s 2
t
t 2 + ( s 1)
1
+

2
1 t 2 s + t + 2
trn on
Xt hm s f ( s )= +
2
t
t 2 + ( s 1)

f '( s ) =

2t ( s 1) t s
2t a s

=
2 2
2
t 2 + ( s 1)
t 2 + ( s 1)
2

) (

t2
0; ta c
4

t2

2t 3 s
4
0

2 2
t 2 + ( s 1)

) (
2

t2
Do f(s) l hm ng bin trn on 0;
4

2
1 t t +2
V vy f ( s ) f (0) =
+ 2
t
t +1

)
717

www.TaiLieuLuyenThi.com
Khm ph t duy K thut gii bt T Bi ton Max Min ng Thnh Nam

2
1 t t +2 5
2
+ 2
( t 1) 2t 2 t + 2 0 (lun ng).
t
2
t +1
Bt ng thc c chng minh. ng thc xy ra khi v ch khi
a= b= 1, c= 0 hoc cc hon v.

Ta cn chng minh

Cch 2: Khng mt tnh tng qut=


gi s c max {a, b, c} c

a+b
.
2

1 ab
vo bt ng thc cn chng minh ta c
a+b
1
1
1
5
1
1
1
5
t P (a, b, c=
)
+
+
=
+
+
.
1
1
ab
ab

2
a+b b+c c+a 2 a+b a+
b+
a+b
a+b

Thay c =

Xt hiu P (a, b, c) P a + b,
,0
a
+
b

1

1
1
1
1
=
+
+
+a+b+

a+b
a + b a + 1 ab b + 1 ab a + b + 1
a+b
a+b
a+b

1
a+b a+b

=
+

+
+
1
a
b
(
)
1 + a 2 1 + b2
1 + ( a + b )2

Ta chng minh

1
1+ a

a+b
1 + a2

1
1+ b

1+

+
1
+
a
b
(
)
1 + b2
1 + ( a + b )2
a+b

1
1 + (a + b)

ab ab ( a + b ) + 2ab 2 0

ab ( a + b ) 2 (1 ab ) (lun ng).
2

V 2 (1 ab =
) 2c ( a + b ) ( a + b ) ab ( a + b )
2

Vy ta cn chng minh
P ( a + b,

1
1
1
5
,0) 0
+a+b+

1
a+b
a+b 2
a+b+
a+b

718

a+b
1 + ( a + b)

1 + ( a + b)
a+b

5
.
2

www.TaiLieuLuyenThi.com
Cty TNHH MTV DVVH Khang Vit

2(a + b)
= 2.
a+b
a+b
1 5
Ta cn chng minh x + 2 x 2 5 x + 2 0 ( 2 x 1)( x 2 ) 0 (lun ng).
x 2
ng thc xy ra khi v ch khi

t x=

1 + (a + b)

1
a + b =
a= c= 1, b= 0

.
=
+ bc + ca 1

ab
b= c= 1, a= 0

2
0
ab ab ( a + b ) + 2ab 2 =

Bi ton c chng minh ng thc xy ra khi v ch khi a= b= 1, c= 0 hoc


cc hon v.
Cch 3: t p = a + b + c, q = ab + bc + ca, r = abc .
Bt ng thc c vit li di dng:
1 + p2 5
( p 2 )( 2 p 1) + 5r 0 .
pr 2

+ Nu p 2 bt ng thc hin nhin ng.


3 p 2 theo bt ng thc Schur bc 3, ta c

+ Nu
r

p 4q p 2
9

)=
4p p
.
3

4 p p3
0
9
5 p ( p + 2 ) 9 ( 2 p 1)
(2 p)
0
9

Ta ch cn chng minh ( p 2 )( 2 p 1) + 5.

( 2 p ) 5 p2 8 p + 9 0

Bt ng thc lun ng ta c iu phi chng minh.


Bi 12. Cho a,b,c l cc s thc khng m tho mn iu kin ab + bc + ca =
1.
Chng minh rng

1
1
1
1
+
+

2.
a+b b+c c+a a+b+c
Li gii

Khng mt tnh tng qut gi s a = min {a, b, c} .


+ Nu b + c > 2

1
1
> , khi theo bi ton trn ta c
a+b+c
2
719

www.TaiLieuLuyenThi.com
Khm ph t duy K thut gii bt T Bi ton Max Min ng Thnh Nam

1
1
1
5
1
1
1
1
+
+

+
+

> 2.
a+b b+c c+a 2
a+b b+c c+a a+b+c
Bt ng thc c chng minh.
t2
t= b + c
Ta xt vi b + c 2, khi t
, s 0; ,0 < t 2
s = bc
4

1 s
.
t
Gi biu thc v tri l P ta c
1
1
1
1
1
2a + b + c
1
P=
+
+

=
+

2
b+c a+b a+c a+b+c b+c
a
+
b+c
a +1
1 s
2.
+t
1 2a + t
1
1
1
t
=
+ 2

=
+

2
1 s
t a +1 a + t t 1 s
+t

+1
t
t

Ta c at + s =1 a =

1
t 2 + 2 2s
t
=
+ t.

2
t
s + t 2 + 1
t 2 + ( s 1)
1
t 2 + 2 2s
t
Xt hm s f ( s ) =
trn on
+ t.

2
2
t
s + t 2 + 1
t + ( s 1)
2
2t ( s 1) t 2 s
t

f '( s ) =
=
2
2
2
2
2
s + t + 1
t + ( s 1)

) (

) (t

2t 3 a 2 s
2

+ ( s 1)

t2
0; ta c
4

) ( s + t + 1)

2 2

Bi v a 2 s 0 .
t2
Do f(s) l hm nghch bin trn on 0; . V vy f ( s )
4

t2
Ta ch cn chng minh f 2 .
4

t2
+2
t
t
1
2
Ta c: f 2 0 + t.
2
20
2
4
t
2
3
t


t
+1
t 2 + 1
4
4

( 2 t ) 6t 4 11t 3 + 10t 2 12t + 8 0 t 2 (lun ng).


720

t2
f .
4

0.

www.TaiLieuLuyenThi.com
Cty TNHH MTV DVVH Khang Vit

Bt ng thc c chng minh. ng thc xy ra khi v ch khi a= b= 1, c= 0


hoc cc hon v.
Cch 2: t p = a + b + c, q = ab + bc + ca, r = abc .
Bt ng thc c vit li di dng:
1 + p2 1
2 p 2 ( p 2 ) + r (1 + 2 p ) 0 .
pr p

+ Nu p 2 bt ng thc hin nhin ng.


+ Nu

3 p 2 theo bt ng thc Schur bc 3 ta c r

p 4q p 2
9

)=
4p p
.
3

Ta ch cn chng minh
4 p p3
(1 + 2 p ) 0 p ( 2 p )( p 1)2 0 .
9
Bt ng thc lun ng ta c iu phi chng minh.
Bi 13. Cho a,b,c l cc s thc khng tho mn iu kin ab + bc + ca 3 .
p2 ( 2 p ) +

Chng minh rg vi mi s thc k

(a

)(

1
ta c
3

)(

+ k b 2 + k c 2 + k ( k + 1) .
3

Li gii
Ch ng thc

(a

)(

)(

) ( k ( a + b + c ) abc )

+ k b 2 + k c 2 + k=

+ k ( ab + bc + ca k ) .
2

1
Trc tin ta chng minh k = bt ng thc ng, tht vy
3
2

1
1
1 1
2 1 2 1 2 1 1
a + b + c + ab + bc + ca 3 = + 1 .
3
3
3
3
3
3
3 3

Vi k ' k

(a

)(

1
ta c
3

)(

+ k ' b2 + k ' c 2 + k ' = a 2 + k + ( k ' k ) b2 + k + ( k ' k ) c 2 + k + ( k ' k )

)(

)(

3
3
3 a 2 + k b2 + k c 2 + k + k ' k (1 + k + k ' k ) = ( k '+ 1)

Bi 14. Cho x,y,z l cc s thc khng m tho mn iu kin xy + yz + zx =


1.
Chng minh rng

1
+
x+ y

1
+
y+z

1
1
.
2+
z+x
2
721

www.TaiLieuLuyenThi.com
Khm ph t duy K thut gii bt T Bi ton Max Min ng Thnh Nam

Li gii
t2
t= y + z
Khng mt tnh tng qut gi s x = max { x, y, z} t
, s 0; .
4
s = yz

1 s
.
t
Gi P l biu thc v tri ta c

Ta c xt + s =1 x =

x+ y + x+z
2 x + y + z + 2 x2 + 1
1
1
P=
+
=
+
y+z
y+z
x 2 + xy + xz + yz
x2 + 1
1 s
2.
+t
1
2
1
2t
t 2 + 2 2s
t
.
.
t
=
+
+
=
+
+
2
2
2
2
2
2
t
t
1
t
s
+

(
)
1 s
1
t
s
+

1
s

(
)

+1

+1
t
t

Xt hm
s f ( s ) t.
=

f '( s ) =
2t.

t 2 + 2 2s
t 2 + (1 s )

(1 s )2 t 2 s
t 2 + (1 s )

2 2

2t
t 2 + (1 s )

2t (1 s )

t2
trn on 0; ta c
4

2t 3 x 2 s

=
3
2
2
t2 + 1 s 2
( ) t + (1 s )

2t (1 s )
t2 + 1 s 2
( )

t2
Do f(s) l hm ng bin trn on 0; .
4
2

t
V vy f ( s ) f (0) =t. 2
+
= t + 2
.
t +1
t + 1
t2 +1
t2 + 2

2t

1
t
1
.
+ t+ 2
2+
t
2
t +1
Bi 15. Cho a,b,c l cc s thc khng m tho mn iu kin ab + bc + ca > 0.
1
1
1
4
.
Chng minh
+
+

4a 2 + bc
4b 2 + ca
4c 2 + ab a + b + c

Do P

Li gii
Gi s a = max {a, b, c} , chng minh
1
4a 2 + bc
722

1
4b 2 + ca

a+b
.
,t =
2
4t 2 + tc
2

www.TaiLieuLuyenThi.com
Cty TNHH MTV DVVH Khang Vit

Tht vy theo bt ng thc AM GM ta c


1
4a + bc
2

4b + ca
2

) (

( 4a

2
2

)(

)(

+ bc 4b 2 + ca

2
1
V 4t 2 + tc 4a 2 + bc 4b 2 + ca =
( a b )2 c 2 + a 2 + b2 + 6ab 3c ( a + b ) 0 .
4

Theo AM GM ta c

1
4c + ab
2

1
4c + t 2
2

Vy ta ch cn chng minh
2
1
4
4
+

=
2
2
2
2
+
+
+c
a
b
c
t
4t + tc
4c + t

2
1
1
1
4
2

2
2
2
t 2t + c t
4t + tc t 4c + t
t
4c
2

2t + c
4t 2 + tc 2t + 4t 2 + tc
4c 2 + t 2 t + 4c 2 + t 2

2
1
4c

0
t ( 2t + c )
4t 2 + tc 2t + 4t 2 + tc
t 4c 2 + t 2 t + 4c 2 + t 2

Bt ng thc trn l tng ca hai bt ng thc sau


1
1

0 (1)
2
2
3t ( 2t + c )
4t + tc 2t + 4t + tc

5
4c

0 (2)
3t ( 2t + c ) t 4c 2 + t 2 t + 4c 2 + t 2

Bt ng thc c chng minh.


ng thc xy ra khi v ch khi=
a b=
, c 0 hoc cc hon v.
Bi 16. (Vit Nam TST 2006) Cho x,y,z l cc s thc thuc on [1;2]. Chng
1 1 1
x
y
z
minh ( x + y + z ) + + 6
+
+
.
x y z
y+z z+x x+ y
Li gii
Ta cn chng minh
1 1 1 x
y
z
f ( x, y , z ) = ( x + y + z ) + + 6
+
+
0.
x y z y+z z+x x+ y
723

www.TaiLieuLuyenThi.com
Khm ph t duy K thut gii bt T Bi ton Max Min ng Thnh Nam

Khng mt tnh tng qut gi s x = max { x, y, z} .


Xt hiu:
2
6( x + y + z )( y z ) 2
y + z y + z ( x + y + z )( y z )
f ( x, y , z ) f x,
,
=

2
2
yz ( y + z )
( x + y )( x + z )(2 x + y + z )

( x + y + z )( y z ) 2 [( x + y )( z + x)(2 x + y + z ) 6 yz ( y + z )]
( x + y )( y + z )( z + x)(2 x + y + z )

v x l s ln nht trong ba s nn ( x + y )( z + x)(2 x + y + z ) > 6 yz ( y + z ) .


y+z
1
2
V vy ta ch cn chng minh f ( x, t , t ) 0 .

nn f ( x, y, z ) f ( x, t , t ) vi
=
t

Tht vy, bt ng thc tng ng vi:


2t
(t x) 2 (2t x)
1 2 x
( x + 2t ) + 6 +

0, .

tx(t + x)
x t 2t t + x

Bt ng thc cui ng do 2t 2 x .
ng thc xy ra khi v ch khi x= y= z; x= 2, y= z= 1 .
Bi ton c chng minh.
C. BI TP RN LUYN
Bi 1. Cho a,b,c l di 3 cnh mt tam gic tho mn iu kin a 2 + b 2 + c 2 =
3.
Chng minh rng a + b + c 2 + abc .
Bi 2. Cho a,b,c l cc s thc khng m tho mn iu kin a 2 + b 2 + c 2 =
3.
Chng minh rng ( 2 ab )( 2 bc )( 2 ca ) 1 .
Bi 3. Cho a,b,c l cc s thc dng c tch bng 1. Chng minh
1 1 1
13
25
.
+ + +

a b c a + b + c +1 4
Bi 4. Cho a,b,c l cc s thc dng tha mn a, b, c 1 v a + b + c =
6.

)(

)(

Tm gi tr ln nht ca biu thc P =


1 + a 2 1 + b2 1 + c2 .
3
Bi 5. Cho a,b,c l cc s thc khng m tha mn a + b + c = .
2

)(

)(

Tm gi tr nh nht ca biu thc P =


1 + a 2 1 + b2 1 + c2 .
Bi 6. Cho a,b,c l cc s thc khng m tha mn a + b + c =
1.

)(

)(

Tm gi tr ln nht ca biu thc P =


a 3 + b3 b3 + c 3 c 3 + a 3 .
724

www.TaiLieuLuyenThi.com
Cty TNHH MTV DVVH Khang Vit

Bi 7. Cho x,y,z l cc s thc dng c tch bng 1. Chng minh


x 2 + y 2 + z 2 + x + y + z 2 ( xy + yz + zx ) .

Bi 8. Cho a,b,c l cc s thc dng c tch bng 1. Chng minh


1
1 1 1
a 2 + b2 + c2 a + b + c + + + .
2
a b c
Bi 9. Cho x,y,z l cc s thc dng c tng bng 1. Chng minh

x
1
y
z 21
54 x3 + y 3 + z 3 + + .
xyz
yz zx xy 2
Bi 10. Cho a,b,c l cc s thc khng m c tng bng 1. Chng minh

(a

)(

)(

1
.
256
Bi 11. Cho a,b,c l cc s thc dng c tch bng 1. Chng minh
3

+ b3 b3 + c 3 c 3 + a 3

3.
a a +1 b b +1 c c +1
Bi 12. Cho a,b,c l cc s thc khng m c tng bng 1. Chng minh
2

4
.
5
1+ b + c
1+ c + a
1+ a + b
Bi 13. Cho x,y,z l cc s thc khng m tho mn iu kin x + y + z =
3.
2

)(

)(

Chng minh rng 3 x 2 + 5 3 y 2 + 5 3 y 2 + 5 512 .


Bi 14. Cho x,y,z l cc s thc khng m. Chng minh
9
x3 + y 3 + z 3 ( xy + yz + zx ) .
( x + y + z )5
4 6 9

Bi 15. Cho x,y,z l cc s thc dng tho mn iu kin x 2 + y 2 + z 2 =


1.
Chng minh rng

9
1
1
1
27
.

+
+

4 1 + xy 1 + yz 1 + zx 4 ( x + y + z )2

Bi 16. Cho a,b,c l cc s thc khng m tho mn iu kin ab + bc + ca > 0 .


Chng minh rng

1
a 2 + bc

1
b 2 + ca

1
c 2 + ab

6
.
a+b+c

D. HNG DN GII P S
Bi 1. Bt ng thc cn chng minh tng ng vi
P ( a, b, c ) = a + b + c abc 2 0 .

Khng mt tnh tng qut ga s a = max {a, b, c} ta c


725

www.TaiLieuLuyenThi.com
Khm ph t duy K thut gii bt T Bi ton Max Min ng Thnh Nam

3= a 2 + b 2 + c 2 a 2 +

1
3
( b + c )2 > a 2 1 a < 2 .
2
2

b2 + c2 b2 + c2
Ta chng minh P ( a, b, c ) P a,
,

2
2

Tht vy

b2 + c2 b2 + c2
P ( a , b, c ) P a ,
,

2
2

a (b c )

( b c )2

b + c + 2 b2 + c2

( b c )2 a

b + c + 2 b2 + c2

2
b c)
(
2

2
a + 2 3 a2

( b c )2 a 2 + a

2
Mt khc

b2 + c2

= b + c 2 b2 + c2 + a
bc
2

2 3 a 2 2 0, a 1; 2

b2 + c2 b2 + c2
,
P a,

2
2

a 3 a2
2
=+
2
2 3a
a

3 a2
= 1

Ch 1

3 a 2

a
+ a 2

3 a2
0
2

3 a2
2
+ a 2 0 a2 3 a2 2 ( 2 a )
2

( a 1) 8 a3 a 2 0

Lun ng. Bt ng thc c chng minh ng thc xy ra khi v ch khi


a= b= c= 1 .
726

www.TaiLieuLuyenThi.com
Cty TNHH MTV DVVH Khang Vit

Cch 2: Gi
s a max {a, b, c} a 1 v v a,b,c l di ba cnh mt tam gic
=
nn a b + c 2 .
Khi
P = a + b + c 2 abc = a + b 2 + c 2 + 2bc 2 abc = a + t + 2 s 2 as .
t
Vi t =
b2 + c2 , s =
bc, s 0; .
2
t
Xt hm s f ( s ) = a + t + 2 s 2 as vi s 0; ta c
2

1
1

<0.
f '( s ) = a; f ''( s ) =
t + 2s
( t + 2s )3

V vy f '( s ) f '(0)=

1
t

a=

3 a2

a < 0, a [1;2] .

t
V vy f(s) nghch bin trn on 0; suy ra
2

at
t
f ( s ) f = a + 2t 2
2
2

) 2

a 3 a2

= a + 2 3 a

3 a2
= 1

3 a 2

a
+ a 2 0

Bi 2. Khng mt tnh tng qut =


gi s a min {a, b, c} a 1 ta c
P ( a, b, c ) =( 2 ab )( 2 ac ) =4 2a ( b + c ) + a 2bc .

b2 + c2 b2 + c2
Ta chng minh P ( a, b, c ) P a,
,

2
2

Tht vy bt ng thc tng ng vi

2a ( b + c ) + a 2bc 2a 2 b 2 + c 2 + a 2 .

b2 + c2
2

a
2
2 2 b 2 + c 2 ( b + c ) ( b c )

727

www.TaiLieuLuyenThi.com
Khm ph t duy K thut gii bt T Bi ton Max Min ng Thnh Nam

4 (b c )

2 b2 + c2 + b + c

a (b c )

4 a ( b + c ) + a 2 b2 + c2

Ta c

4 a ( b + c ) a 2 b 2 + c 2 4 2a 2 b 2 + c 2

=
4 2a 2 3 a

)
)(
(

)
)

3 a2
2 a

2.

4 1 a2 2 a2
=
0
2 + a 2 3 a2

Vy ta ch cn chng minh

b2 + c2
b2 + c2

2
2
a

2
2

Ta c 2 a 2 + 1 ( a + 1) =

1 a2 + 1

( a 1)2 0 a 2 + 1

( a + 1)2
2

Vy ta ch cn chng minh

3 a2

1
2
a
+

a
( )

3 a2
4 ( a + 1) 2 a

3 a2
a 2 ( a + 1)

a a 4 + 2a 3 2a 2 6a + 5
0
0
2

3
a

2 + ( a + 1)
2

a ( a 1) a 2 + 4a + 5 0
2

Lun ng. Bt ng thc c chng minh ng thc xy ra khi v ch khi


a= b= c= 1 .
Bi 3. Bt ng thc cn chng minh c dng
P ( a , b, c ) =

1 1 1
13
25
+ + +

0.
a b c a + b + c +1 4

Khng mt tnh tng qut gi s a= max {a, b, c} a 1, x=


Ta chng minh

P ( a, b, c ) P a, bc , bc .

728

bc 1 .

www.TaiLieuLuyenThi.com
Cty TNHH MTV DVVH Khang Vit

Tht vy

P ( a, b, c ) P a, bc , bc =
=

1 1
2
13
13
+
+

b c
bc a + b + c + 1 a + 2 bc + 1

b c

bc

13

b c

( a + b + c + 1) ( a + 2

bc + 1

1
13
b
c
=
bc ( a + b + c + 1) a + 2 bc + 1

13
b c 1
0
( 3 + 1)( 3 + 1)

2
13
25
1

Mt khc P a, bc , bc = P 2 , x, x = x 2 + +

x 2x + 1 + 1
4
x

2
x

( x 1)2 (8 x 4 + 20 x3 18 x 2 9 x + 8)
0, x ( 0;1] .
4 x ( x + 1) ( 2 x 2 x + 1)

T suy ra iu phi chng minh. ng thc xy ra khi v ch khi a= b= c= 1 .


Bi 4.

)(

Ta c 1 + a 2 1 + b 2

a + b 2
1
2
1 +
( a b ) a 2 + 6ab + b 2 8 0 vi
=

16
2

mi a, b 1 .

Do 1 + a

)(1 + b )
2

a + b 2
1 +
.
2
2

a + b 2
6 c 2
2
2
Suy ra P 1 +
1 + c = 1 +
1 + c 125 .
2
2
ng thc xy ra khi v ch khi a= b= c= 2 .
Vy gi tr ln nht ca P bng 125 t ti a= b= c= 2 .
Bi 5. Khng mt tnh tng qut gi s

1
3

c 2 , a + b = 2 c 1

=
c max {a, b, c}
2
1
ab ( a + b ) =

4
4
729

www.TaiLieuLuyenThi.com
Khm ph t duy K thut gii bt T Bi ton Max Min ng Thnh Nam

Khi 1 + a

)(1 + b )
2

a + b 2
1 +
.
2
2

a + b 2
( a b )2 a 2 + 6ab + b2 8 0 .
Tht vy 1 + a 1 + b 1 +
=

16
2
ng thc xy ra khi v ch khi a = b .

)(

Khi P 1 + c

a + b 2
1 + c2
1 +
=
2

ng thc xy ra khi v ch khi a= b= c=

)(

)(

Cch 2 : Ta c 1 + a 2 1 + b 2 1 + c 2

3 c 2 125
.
1 +
64
4 2

1
.
2

= a 2 + b 2 + c 2 + a 2b 2 + b 2 c 2 + c 2 a 2 + a 2b 2 c 2 + 1
=

( a + b + c )2 2 ( ab + bc + ca ) + ( ab + bc + ca )2 2abc ( a + b + c ) + a 2b2c 2 + 1

13
2
2 ( ab + bc + ca ) + ( ab + bc + ca ) + a 2b 2 c 2 3abc
4

( ab + bc + ca 1)2 + abc

3 1 11 125
+ =
2 4 8
64
3

1
3
2
a+b+c 1
c)
; abc
=
.
Do ab + bc + ca ( a + b + =

3
4
3

ng thc xy ra khi v ch khi a= b= c=

1
.
2

t2
Bi 6. Khng mt tnh tng qut gi s a b c t t =
b + c, s =
bc, s 0; ta
4
c a = 1 t .

Khi P = a 6 + a3 b3 + c3 + b3c3 b3 + c3
.

3
3
= a 6 + a3 ( b + c ) 3a3bc ( b + c ) + b3c3 . ( b + c ) 3bc ( b + c )

)(

)
Xt hm s f ( s ) =( a + a t 3a ts + s )( t 3ts ) ta c
f '( s ) = ( 3s 3a t )( t 3ts ) 3t ( a + a t 3a ts + s ) .
= a 6 + a3t 3 3a3ts + s 3 t 3 3ts
6

730

3 3

3 3

www.TaiLieuLuyenThi.com
Cty TNHH MTV DVVH Khang Vit

S dng bt ng thc AM-GM ta c a 6 + a3t 3 3a3ts + s 3 0 .


Theo iu kin suy ra

t4
3s 2 3a3t t 3 3ts = 3t s 2 a3t t 2 3s 3t s 2 t 2 3s 0 .

)(

) (

)(

3
3
Do f '( s ) 0 P= f ( s ) f (0)= t 3 a 6 + a3t 3 = t 3 (1 t ) t 3 + (1 t ) .

3
3
2
Xt hm s g (t )= t 3 (1 t ) t 3 + (1 t ) vi t 0; ta c

t = 0
2
g '(t ) =
3t ( t 1) ( 2t 1) ; f '(t ) =
0 1 (do t 0; ).
t =
3
2
2

1
nn g(t) t cc i ti
2
1
1
1
1
t = hay P g (t ) g =. ng thc xy ra khi v ch khi a= b=
,
2
2
2 256
c = 0 hoc cc hon v.

Ta c g(t) i du t dng sang m khi i qua t =

1
1
t ti a= b=
, c= 0 hoc cc hon v.
256
2
Nhn xt. Vi a,b,c khng m tha mn a + b + c =
k . Chng minh tng t ta c

Vy gi tr ln nht ca P bng

(a

)(

)(

+ b3 b3 + c 3 c 3 + a 3

k9
.
256

Bi 7. Ta cn chng minh
P ( x, y, z ) = x 2 + y 2 + z 2 + x + y + z 2 ( xy + yz + zx ) 0 .

Khng mt tnh tng qut =


gi s x min { x, y, z} x 1, yz 1 .

) (

Xt hiu P ( x, y, z ) P x, yz , yz=

y z

) ( y + z 2x + 1 + 2
2

yz 0 .

Ta ch cn chng minh P x, =
yz , yz
P 2 , t , t 0 vi t = yz .
t

Tht vy bt ng thc tng ng vi

( t 1) 2t 3 + 4t 2 + 2t + 1
4
+ + 2t 0
0.
t
t4 t2
t4
Bt ng thc c chng minh. ng thc xy ra khi v ch khi x= y= z= 1 .
2

731

www.TaiLieuLuyenThi.com
Khm ph t duy K thut gii bt T Bi ton Max Min ng Thnh Nam

Bi 8. Bt ng thc cn chng minh tng ng vi


1
1 1 1
P ( a , b, c ) = a 2 + b 2 + c 2 a + b + c + + + 0 .
2
a b c

Khng mt tnh tng qut gi s =


a min {a, b, c} =
t

bc 1 .

Ta chng minh P (a, b, c) P (a, t , t ) .

1
1
Tht vy P (a, b, c) P(a, t , t ) = ( b c ) 2 ( b + c ) 2 1 + .
2 bc

( b c ) 2 . 4 (1 + 1) 0
2

1
(t 1) 2 (3t 4 + 4t 3 + 5t 2 + 4t + 2)
Mt khc =
P (a, t , t ) P=
( 2 , t, t)
0.
t
2t 4
Bt ng thc c chng minh. ng thc xy ra khi v ch khi a= b= c= 1 .
Bi 9. Ta cn chng minh

P ( x, y , z ) =

x
1
y
z 21
54 x3 + y 3 + z 3 + + 0 .
xyz
yz zx xy 2

Ch
=
P ( x, y , z )

( x + y + z )2 x 2 y 2 z 2 54
xyz

(x

+ y3 + z3

21
4

1 1 1
21
= 2 + + 27 x3 + y 3 + z 3
x y z
2

Khng mt tnh tng qut gi s x = max { x, y, z} ta chng minh


y+z y+z
P ( x, y , z ) P x,
,
.
2
2

( y z )2 ( 4 81yz ( y + z )2 )
0
2 yz ( y + z )

y+z y+z
Xt hiu P ( x, y, z ) P x,
,=

2
2

y+z y+z

y+z+
+

y+z y+z
2
2
2
Bi v =
yz ( y + z ) 4 y.z.
.
4
2
2
4

732

( y + z )4 1 2 4 =
4


4 3

4
81

www.TaiLieuLuyenThi.com
Cty TNHH MTV DVVH Khang Vit

Ta cn chng minh
y+z y+z
1 x 1 x
P(=
x,
,
) P ( x,
,
)0
2
2
2
2
1

(1 x )3 21 0
4
2 +
27 x3 +

x 1 x
4 2

( 3x 2 )2 ( 9 x3 + 12 x 2 6 x + 1)

0
2 x (1 x )

Bt ng thc cui lun ng. Bt ng thc c chng minh.


2
1
hoc cc hon v.
ng thc xy ra khi v ch khi x=
, y= z=
3
6
Bi 12. Gi P l biu thc v tri v s dng bt ng thc C S ta c
P
=

( a + b + c )2

) (

) (

a 1 + b2 + c2 + b 1 + c2 + a 2 + c 1 + a 2 + b2

1 + a ( b + c ) + a b 2 + c 2 + bc ( b + c )
2

Ta tm gi tr ln nht ca biu thc vi gi thit a = max {a, b, c} .

Q = a 2 ( b + c ) + a b 2 + c 2 + bc ( b + c ) = a 2t + a t 2 2 s + ts .

t2
Xt hm s f ( s ) = a 2t + a t 2 2 s + ts trn on 0; ta c
4

1
a+t
f '( s ) =t 2a 0 f ( s ) f (0) = a 2t + at 2 = at
= .
4
2

Do P

1
5
.
1+ Q 4

ng thc xy ra khi v ch khi a= b=

1
, c= 0 hoc cc hon v.
2

Bi 13. =
Gi s x max { x, y, z} y + z 2, x 1 ta c

x + y 2

3
+
=
3 y + 5 3 z + 5 3
5

( x y )2 40 3 x 2 + y 2 + 6 xy 0 .

2
16
2

)(

733

www.TaiLieuLuyenThi.com
Khm ph t duy K thut gii bt T Bi ton Max Min ng Thnh Nam
2

x + y 2

Vy ta chng minh 3 x + 5 3
5
+
512

3 x 2

3 x + 5 3
+ 5 512 0
2

( x 1) 3 x 4 30 x3 + 144 x 2 306 x + 317 0


2

Bt ng thc lun ng.


Bi ton c chng minh ng thc xy ra khi v ch khi x= y= z= 1 .

)(

)(

Bi 16. HD: a v chng minh 64 a 2 + bc b 2 + ca c 2 + ab ( a + b + c ) .

734

You might also like